Skip to main content

Full text of "Elementary manual on applied mechanics .."

See other formats


OF  THE 


'  •     ,     •»• 

.  •    •    . 


UNIVERSITY  OF  CALIFORNIA. 


.    • 

»•••• 
Class 


ELEMENTARY  MANUAL 

ON 

APPLIED   MECHANICS 

NINTH  EDITION 
THOROUGHLY  REVISED 


GRIFFIN'S  SCIENTIFIC  TEXT-BOOKS. 

By  PROFESSOR  JAMIESON,  M.Inst.C.E.,  M.Inst.E.E.,  F.R.S.E.,  Formerly 
Professor  of  Engineering  in  the  Glasgow  and  West  of  Scotland  Technical  College; 
Consulting  Engineer  and  Electrician,  16  Kosslyn  Terrace,  Kelvinside,  Glasgow. 


INTRODUCTORY   MANUALS. 

Crown  8vo.      With  Illustrations  and  Examination  Papers. 

STEAM  AND  THE  STEAM-ENGINE  (Elementary 

Manual  of).     For  First- Year  Students,  forming  an  Introduction  to  the 
Author's  larger  Work.     Twelfth  Edition,  Revised.     3s.  6d. 
"  Should  be  in  the  hands  of  EVEBY  engineering  apprentice." — Practical  Engineer. 

MAGNETISM  AND  ELECTRICITY  (Practical  Elementary 

Manual  of).      For  First-Year  Students.     With  Stud.  Inst.  C.E.  and 
B.  of  E.   Exam.  Questions.      Eighth  Edition,  Revised  and  Enlarged. 
3s.  6d. 
"A  THOROUGHLY  TRUSTWORTHY  Text-Book.     PfiACTicAi,  and  clear." — Nature.. 

APPLIED    MECHANICS  (Elementary  Manual  of). 

For  First-Year  Students.     With  B.  of  E.,  C.  and  G.,  and  Stud.  Inst. 

C.E.  Questions.     Ninth  Edition,  Revised.     3s.  6d. 

"The  work  has  VERY  HIGH  QUALITIES,  which  may  be  condensed  into  the  one  word 
CIEAB." — Science  and  Art. 


ADVANCED   TEXT-BOOKS. 

In  Large  Crown  8vo.     Fully  Illustrated. 

STEAM  AND  STEAM-ENGINES,  INCLUDING  TURBINES 

AND  BOILERS.    For  the  Use  of  Engineers  and  for  Students  preparing 
for  Examinations.    With  800  pp.,  over  350  Illustrations,  11  Plates,  many 
B.  of  E.,  C.  and  G.,  Questions  and  Answers,  and  all  Inst.  C.E.  Exams. 
on  Theory  of  Heat  Engines.     Sixteenth  Edition,  Revised.     10s.  6d. 
"  THE  BEST  BOOK  yet  published  for  the  use  of  Students." — Engineer. 

APPLIED  MECHANICS  &  MECHANICAL  ENGINEERING. 

Including  All  the  Inst.  C.E.  Exam*,  in  Section  A :  (1)  Applied 
Mechanics  :  (2)  Strength  and  Elasticity  of  Materials  ;  (3a)  Theory  of 
Structures  ;  Section  B  :  (ii)  Hydraulics  ;  Theory  of  Machines.  Also 
B.  of  E.  ;  C.  and  G.  Questions. 

VOL.      I.  APPLIED  MECHANICS,  pp.  i.-xviii. +382.     Price  6s. 
VOL.    II.  STRENGTH  OF  MATERIALS,  pp.  i.-xviii.  +  264.  Price  5s. 
VOL.  III.  THEORY  OF  STRUCTURES,  pp.  i.-xvi.+232.     Price  5s. 
VOL.  IV.  HYDRAULICS,  pp.  i.-xvi.+274,    Price  5s. 
VOL.    V.  THEORY  OF  MACHINES,  pp.  i.-xx.+502.    Price  7s.  6d. 
***  Each  of  the  above  volumes  is  complete  in  itself,  and  is  sold  separately. 

A  POCKET  BOOK  OF  ELECTRICAL  RULES  AND  TABLES. 

For  Electricians  and  Engineers.      By  JOHN  MUNRO,  C.E.,  and  Prof. 
JAMIESON.    Nineteenth  Edition.     Pocket  Size.     Leather,  8s.  6d. 
"  WONDEBFULLT  PERFECT.  .  .  .  Worthy  of  the  highest  commendation."— Electrician . 


Complete  Catalogue  sent  post  free  on  application. 


LONDON:    CHARLES   GRIFFIN  &  Co.,  LTD.,  EXETER  ST.,  STRAND. 


OF   THE 

UNIVERSITY 

OF 


ELEMENTARY    MANUAL 


ON 


APPLIED    MECHANICS 

SPECIALLY  ARRANGED  TO  SUIT  THOSE  PREPARING  FOR  THE 
INSTITUTION  OF  CIVIL  ENGINEERS;  ROYAL  INSTITUTE  OF 
BRITISH  ARCHITECTS  ;  CITY  AND  GUILDS  OF  LONDON  INSTI- 
TUTE ;  BRITISH  AND  COLONIAL  BOARDS  OF  EDUCATION  AND 
ALL  KINDS  OF  FIRST  YE1R  ENGINEERING  STUDENTS. 


BY 

ANDEEW  JAMIESON,  M.  INST.  C.E. 

FORMERLY  PROFESSOR  OF  ENGINEERING,  THE  GLASGOW  TECHNICAL  COLLEGE 

FELLOW  OF  THE  ROYAL  SOCIETY,  EDINBURGH;   MEMBER  OF  THE  INSTITUTION  OF 

ELECTRICAL  ENGINEERS;    AUTHOR  OF  "TEXT-BOOKS  ON  STEAM  AND  STEAM 

ENGINES,"    "ADVANCED  APPLIED  MECHANICS,"  "MAGNETISM  AND 

ELECTRICITY,"  "ELECTRICAL  ENGINEERING  RULES  AND  TABLES." 


NINTH  EDITION,  THOROUGHLY  REVISED 


diagrams,  arithmetical  Samples,  examination 
anb 


Of   THE 

UNIVERSITY 

OF 


|L,ONDON 

CHARLES  GRIFFIN'  AND  COMPANY,  LIMITED, 

EXETER    STREET    STRAND. 

1910 
[All  rights  reserved.] 


PREFACE  TO  NINTH  EDITION 


THIS  Elementary  Manual  on  Applied  Mechanics  has  been  care- 
fully revised,  whilst  several  important  alterations  and  additions 
have  been  made  to  it. 

I  have  added  a  drawing  and  description  to  the  end  of  Lecture 
VIII  of  Butters  Brothers  and  Co.'s  new  1909  "Jib  Crane 
Arrester." 

A  new  Frontis  Plate,  as  well  as  a  new  plate  for  Lecture  XXI, 
showing  the  latest  1909  improvements  in  "  Alfred  Herbert's 
Electrically  Driven  Turret  Lathe,"  together  with  a  description 
thereof,  have  been  duly  inserted  at  the  proper  places. 

Owing  to  a  very  clear  and  useful  hint  which  was  given  in  The 
Electrician,  by  the  reviewer  of  the  previous  Edition,  I  have  trans- 
ferred the  former  Lectures  numbered  XXIV  and  XXV,  upon 
"  Bending  and  Shearing  Stresses  in  Beams  and  Metal  Structures," 
<fec.,  to  my  new  and  more  advanced  book,  Volume  III,  which  deals 
entirely  with  the  "Theory  of  Structures."  This  change  has 
enabled  me  not  only  to  reduce  the  total  number  of  Lectures  in 
the  book  from  twenty-eight  to  twenty- six,  but  to  re-arrange  and 
considerably  extend  the  last  Lecture.  To  it,  I  have  added  an 
account  of  "  Sir  Joseph  Whit  worth's  Early  Realisations  of 
Mechanical  Accuracy  " ;  as  well  as  a  detailed  description  of  one  of 
his  "  Millionth  Measuring  Machines." 

Further,  I  have  described  by  aid  of  six  very  clear  photo-process 
diagrams,  a  "  New  Set  of  English  Standard  Guages."  I  have 
specially  to  thank  Mr.  H.  M.  Budgett,  of  the  Crown  Works, 
Chelmsford,  England,  not  only  for  supplying  these  good  figures, 
but  also  for  permitting  me  to  copy  the  Certificate  given  to  him 
last  autumn  by  The  National  Physical  Laboratory  of  their  very 
careful  tests  of  these  British-made  gauges. 

Special  attention  has  been  paid  to  the  Appendices  A,  B,  0,  and 
D,  where  Teachers  and  Stage  I  Students  will  find,  not  only  the 
latest  General  Instructions  of  the  Board  of  Education  (B.  of  E.) 
and  the  City  and  Guilds  of  London  (C.  &  G.);  but  also  the  concise 
abstract  from  the  Rules  and  Syllabus  for  the  examination  of 
Students  of  The  Institution  of  Civil  Engineers  (Stud.  I.C.B.). 
Further,  all  the  questions  given  by  these  three  well-known  and 
respected  Examining  Bodies,  up  to  and  including  the  year  1908, 
have  been  collected  and  arranged  under  the  numbers  of  the  various 


204114 


VI  PREFACE 

Lectures  to  which  they  naturally  belong.  Finally,  the  entire 
examination  papers  for  1909  and  1910  have  been  given  as  issued, 
with  the  view  that  students  should  test  their  knowledge  of  this 
book  by  trying  these  papers  under  exact  Examination  conditions. 
At  all  events,  I  find  that  this  is  one  of  the  best  methods  of  pre- 
paring my  C.E.  and  other  Engineering  Correspondence  Students  for 
these  and  several  other  Examinations  and  Appointments. 

I  have  again  to  thank  Mr.  John  Ramsay,  Assoc.  M.  Tnst.  C.E., 
for  his  assistance  with  this  book. 

ANDREW  JAMIESON. 

Consulting  Engineer  and  Electrician. 
1C  ROSSLYN  TERRACE, 

KELVINSIDE,  GLASGOW. 
August  1910. 


PEEFACE  TO  EIGHTH  EDITION 


I  HAVE  taken  advantage  of  the  working  out  of  many  Questions 
at  the  end  of  the  several  Lectures,  and  of  various  Examination 
Papers  in  the  Appendices,  by  my  "  Science  Correspondence 
Students,''  to  obtain  a  number  of  checked  Answers.  These 
Answers  have  been  inserted  at  their  Questions  up  to  Lecture 
XXIY ;  but  thereafter,  they  have  been  printed  just  before  the 
Index,  at  the  end  of  Appendix  C. 

The  1907  Examination  Papers  for  Stage  I  of  the  Board  of 
Education  (B.  of  E.),  South  Kensington  ;  the  City  and  Guilds 
(C.and  G.)  of  London  Institute, Ordinary  Grade,  Part  I.  Sections  A 
and  B  on  Mechanical  Engineering;  and  the  Entrance  or  Students' 
Examinations  of  The  Institution  of  Civil  Engineers  (Stud.  I.C.E.) 
in  Elementary  Mechanics  for  1907  and  Feb.  1908  will  be  found 
in  AppeLdix  B,  along  with  those  of  previous  }ears,  the  Rules, 
and  an  Abstract  of  the  Syllabus  for  these  Examinations. 


PKEFACE  Vll 

These  Answers  and  new  up-to  date  Examination  Questions, 
which  have  been  taken  from  the  best  sources,  should  prove  of 
great  interest  and  assistance  to  Teachers  and  Students  of  Applied 
Mechanics. 

At  the  end  of  Lecture  I  a  new  page  has  been  added.  It  deals 
with  the  terms  and  definitions  of  words  used  in  measuring  Pres- 
sures, as  well  as  a  discussion  of  the  recent  endeavour  by  some 
authors  to  introduce  the  words  Thi*ust  and  Resultant  Thrust.  I 
see  no  necessity  for  these  terms  and  shall  feel  obliged  if  practical 
Engineering  Reviewers,  Teachers  and  Students  will  give  me 
their  opinion  after  carefully  studying  my  remarks. 

At  the  end  of  Lecture  XVIII  a  part  of  a  new  page  has  been  in- 
serted, wherein  are  given  concise  distinctions  between  solids,  liquids 
and  gases,  with  definitions  of  perfect  viscous  and  elastic  fluids. 

A  more  detailed  and  clearer  explanation  of  the  application  and 
uses  of  Limit  Gauges  as  well  as  of  the  construction  and  uses  of 
Berridge's  Tangentometer  have  been  given  in  Lecture  XXVIII 
Students  should  study  and  apply  this  instrument  to  find  the 
sines,  cosines,  tangents,  &c.,  of  different  angles  and  then  check 
their  results  by  the  Tables  near  the  end  of  Appendix  C,  on 
"  Functions  of  Angles,"  before  they  finish  Lecture  IV. 

Owing  to  the  necessity  for  young  engineers  becoming  early 
acquainted  with  Electrical  Engineering  terms  and  units  of 
measurement,  I  have  added  to  appendix  C  what  is  believed  to  be 
the  latest  and  most  correct  set  of  definitions  for  the  Nomenclature 
with  their  Abbreviations  or  Symbols  of  the  C.G.S.  and  of  the 
authorised  Practical  Electrical  Units. 

Finally,  the  whole  book  has  been  most  carefully  revised  and 
brought  up  to  date. 

I  have  much  pleasure  in  thanking  Forrest  Sutherland,  B.Sc., 
Science  and  Technical  Educationist,  Natal,  South  Africa,  for  his 
useful  suggestions,  and  John  Ramsay,  Assoc.  M.  Inst.  C.E.,  for 
the  interest  which  he  has  taken  and  the  help  he  has  given  with 
this  new  Edition.  The  Author  hopes  that  this  eighth  edition 
will  be  found  to  be  an  advance  upon  previous  issues.  He  also 
desires  reviewers,  teachers  and  students  to  kindly  let  him  know 
in  what  way  they  think  that  he  can  still  further  improve  this 
"  First  Year  Manual  on  Appli  ed  Mechanics." 

f         ANDREW  JAM  IE  SON, 

Consulting  Engineer  and  Electrician. 
16  KCSSLYB^TEKBACE, 

KELVINSIDE,  GLASGOW. 


INSTRUCTIONS  TO  BE  FOLLOWED  IN  THE  WRITING 
OF  HOME  EXERCISES. 

1.  Put  the  date  of  handing  in  each  exercise  at  the  right-hand  top  corner. 

2.  Leave  a  margin  an  inch  wide  on  th«  left-hand  side  of  each  page ;  and 
in  the  margin  place  the  number  of  the  question,  and  nothing  more. 

3.  Leave  *  space  of  *t  least  three  lines  between  your  answers  for  remarki 
or  corrections. 

4.  Be  sure  yon  understand  exactly  what  the  question  requires  you  to  answer, 
then  give  all  it  requires,  but  no  more.     If  unable  to  answer  any  question,  write 
down  its  number  and  the  reason  why. 

5.  Make  your  answers  concise,  clear,  and  exact ;    and  accompany  them, 
whenever  practicable,  by  an  illustrative  sketch. 

6.  Make  all  sketches  large,  open,  and  in  the  centre  of  the  page,  and  do  not 
crowd  any  writing  about  them. 

NOTE. — The  character  of  the  sketches  will  be  considered  in  awarding  the 
marks  to  the  several  questions.  Neat  sketches  and  an  "  Index  to 
Parts,"  with  the  first  letter  of  name  of  Part,  will  always  receive  more 
marks  than  a  bare  written  description. 

7.  Every  sketch  must  be  accompanied  by  an  "  Index  to  Parts  "  written 
immediately  beneath  it,  and  must  accompany  the  answer  it  is  designed  ie 
illustrate. 

NOTE.— The  initial  letter  or  letters  of  the  name  of  the  Part  must  be  used, 
and  not  A,  IB,  C,  or  i,  2,  3,  &c, 

8.  Unless  otherwise  specially  requested  by  the  question,  every  sketch  must 
be  accompanied  by  a  concise,  written  description. 

9.  Every  answer  which  receives  less  than  half  of  the  full  marks  awarded  to 
It,  must  be  re-written  correctly  for  next  evening,  before  the  usual  class  workf 
and  headed  "Re-written." 

REMARKS. — Students  are  strongly  reromme?  r  e«l  to  wr  te  out  each 
answer  in  scroll  first,  and  then  to  compare  it  with  the  question.  After 
committing  it  to  their  book,  they  should  then  read  it  over  a  second  time, 
to  correct  any  errors  they  may  discover.  Reasonable  and  easily  intelli- 
gible contractions  are  permitted.  Students  are  invited  to  ask  questions 
and  explanations  regarding  anything  they  do  not  understand.  Except  in 
special  cases,  arrears  of  Home  work  will  not  receive  marks. 

N.B. — Students  who  from  any  cause  have  been  absent  from  a  lecture,  must 
tend  a  post-card  or  note  of  explanation  to  the  teacher.  If  they  miss  any  exercise 
or  exercises,  they  must  state  the  reason  (in  red  ink,  or  underlined)  in  their 
exercise  books  when  handing  them  in  next  night.  If  these  rules  are  not  com- 
,  then  marks  will  be  deducted. 


CONTENTS. 


FAOl 

LECTURE  I. 

Definition  of  Applied  Mechanics — Force — Matter — Unit  of  Force 
—The  Elements  of  a  Force — Graphic  Representation  of 
Forces — Forces  in  Equilibrium — Action  and  Reaction — 
Resultant  and  Components — Resultant  of  Forces  acting 
in  a  Straight  Line — Terms  and  Definitions  used  in  Measur- 
ing or  Calculating  Pressures  and  their  Effects — Definitions 
of  Scalar,  Vector  and  Rotor  Quantities — Engineering  Cal- 
culations .  1-5 

LECTURE   II. 

Work— Unit  of  Work— Examples  I.  II.  III.  IV.— Work  done 
against  a  Variable  Resistance — Example  V. — Diagrams  of 
Work— With  Uniform  Resistance — With  a  Uniformly  In- 
creasing Resistance — With  a  Uniformly  Decreasing  Resis- 
tance— With  a  Combination  of  Uniform  and  Variable 
Loads — Example  VI. — Power  or  Activity — Units  of  Power 
— The  Horse-power  Unit — To  find  the  Horse-power  of  any 
working  Agent— Example  VII. — Uses  of  Squared  Paper — 
Clark's  Adjustable  Curve — Example  VIII. — Questions  .  6-ao 

LECTURE   III. 

The  Moment  of  a  Force— Principle  of  Momenta  applied  to  the 
Lever — Experiments  I.  II.  III. — Pressure  on  and  Reaction 
from  the  Fulcrum  —  Equilibrant  and  Resultant  of  two 
Parallel  Forces — Couples — Centre  of  Parallel  Forces  or 
Position  of  Equilibrant  and  Resultant — Centre  of  Gravity 
j:  —Examples  of  Centre  of  Gravity — The  Lever  when  its 

Weight  is  taken  into  Account — Examples  I.  II. — Position 
of  the  Fulcrum — Example  III. — Questions         .        •        .      21-34 

LECTURE   IV. 

Practical  Applications  of  the  Lever — The  Steelyard,  or  Roman 
Balance— Graduation  of  the  Steelyard— The  Lever  Safety 
Valve — Example  I.— Lever  Machine  for  Testing  Tensile 


CONTEXTS, 

PAGES 

Strength  of  Materials — Straight  Levers  acted  on  by  In- 
dined  Forces — Bent  Levers — The  Bell  Crank  Lever — Bent 
Lever  Balance — Duplex  Bent  Lever,  or  Lumberer's  Tongs — 
Turkns,  or  Pincers — Examples  II.  and  III. — Toggle  Joints 
—  Questions  .........  35-51 


LECTURE  V. 

The  Principle  of  Work — Work  put  in,  Work  lost,  Useful  Work — 
Efficiency  of  a  Machine — Principle  of  Work  applied  to  the 
Lever — Experiments  I.  II. — Wheel  and  Axle — The  Prin- 
ciple of  Moments  applied  to  the  Wheel  and  Axle— The 
Principle  of  Work  applied  to  the  Wheel  and  Axle— Experi- 
ment III. — The  Winch  Barrel — Example  L— Ship's  Cap^an 
The  Fusee — Questions  ..«..,.  52-62 

LECTURE   VI. 

Pulleys— Snatch  Block— Block  and  Tackle— Theoretical  Advan- 
tage—Velocity Ratio— The  Principle  of  Work  applied  to 
the  Block  and  Tackle — Actual  or  Working  Advantage — 
Work  pnt  in  —  Work  got  out  —  Efficiency  —  Percentage 
Efficiency— Example  I.— Questions  .  ,  .  .  63-71 

LECTURE  VII. 

The  Wheel  and  Compound  Axle,  or  Chinese  Windlass — The  Prin- 
ciple of  Moments  applied  to  the  Wheel  and  Compound 
Axle — The  Principle  of  Work  applied  to  the  Wheel  and 
Compound  Axle — Examples  I.  II. — Weston's  Differential 
Pulley  Block — The  Principle  of  Work  applied  to  Weston's 
Differential  Pulley  Block— Experiment  I.— Cause  of  the 
Load  not  overhauling  the  Chain— Questions  .  .  .72-  79 


LECTURE  VIII. 

Graphic    Demonstration    of    Three    Forces    in    Equilibrium — 
Parallelogram  of  Forces — Triangle  of  Forces— Three  Equal 
Forces  in  Equilibrium — Two  Forces  acting  at  Right  Angles 
— Resolution  of  a  Force  into  Two  Components  at  Right 
Angles — Resultant  of  Two  Forces  acting  at  any  Angle  on 
a  Point — Resultant  of  any  number  of    Forces  acting  at  a 
Point — Example  I. — Stresses  in  Jib  Cranes — Jib  Arrester 
— Example  II.,  III. — Stresses  on  a  Simple  Roof — Example 
IV.— Questions 80-92 


CONTENTS.  XI 

LECTURE  IX. 

PA6B» 

Inclined  Planes -The  Inclined  Plane  without  Friction — When  the 
Force  acts  Parallel  to  the  Plane— Example  I. — When  the 
Force  acts  Parallel  to  the  Base— Example  II.— When  the 
Force  acts  at  any  Angle  to  the  Inclined  Plane — Example  III. 
— The  Principle  of  Work  applied  to  the  Inclined  Plane- 
Example  IV. — Questions 93-IOO 

LECTURE  X. 

Friction — Heat  is  Developed  when  Force  overcomes  Friction — 
Laws  of  Friction — Apparatus  for  Demonstrating  First  and 
Second  Laws  of  Friction— Experiment  I. — Example  I. — 
Angle  of  Repose  or  Angle  of  Friction — Experiment  II. — 
Diagram  of  Angles  of  Repose — Limiting  Angle  of  Resistance 
— Experiment  III. — Apparatus  for  Demonstration  of  the 
Third  Law  of  Friction — Experiment  IV. — Lubrication — 
Anti-Friction  Wheel  Ball  —  Bearings  —  Work  done  on 
Inclines,  including  Friction — Example  II. — Questions  101-1x5 

LECTURE  XI. 

Difference  of  Tension  in  the  Leading  and  Following  Parts  of  a 
Driving  Belt,— Brake  Horse-Power  transmitted  by  Belts — 
Examples  I.  II. — Velocity  Ratios  in  Lelo  Gearing— Examples 
III.  IV.— Open  and  Crossed  Belts — Fast  and  Loose  Pulleys 
— Belt  Gearing  Reversing  Motions— Stepped  Speed  Cones 
with  Starting  and  Stopping  Gear — Driving  and  Following 
Pulleys  in  Different  Planes— Shape  of  Pulley  Face — 
Questions ...  116-129 

LECTURE  XII. 

Velocity  Ratio  of  Two  Friction  Circular  Discs — Pitch  Surfaces 
and  Pitch  Circ'es— Pitch  of  Teeth  in  Wheel  Gearing— Rack 
and  Pinion  Velocity  Ratio  in  Wheel  Gearing— Example  I. 
— Principle  of  Work  applied  to  Wheel  Gearing— Examples 
II.  III.— Questions 130-139 

LECTURE  XIII. 

Sirgle-purchase  Winch  or  Crab— Example  I. — Double-purchase 
Winch  or  Crab— Example  II. — Wheel  Gearing  in  Jib-Cranes 
—Questions 140-147 


aii  CONTENTS. 

LECTURE  XIV. 

FAQK8 

Screws — The  Spiral,  Helix,  or  Ideal  Line  of  a  Screw  Thread — The 
Screw  viewed  as  an  Inclined  Plane — Characteristics  and 
Conditions  to  be  fulfilled  by  Screw  Threads— Different 
Forms  of  Screw  Threads— Whitworth's  V-Threads— Whit- 
worth's  Tables  of  Standard  V-Threads,  Nuts  and  Bolt 
Heads  —  Seller's  V-Thread  —  The  Square  Thread  —  The 
Rounded  Thread — The  Buttress  Thread — Right  and  Left- 
hand  Screws — The  Screw  Coupling  for  Railway  Carriages — 
Single,  Double  and  Treble  Threaded  Screws— Backlash  in 
Wheel  and  Screw  Gearings— Questions  ....  148-159 

LECTURE  XV. 

Efficiency,  &c.,  of  a  Combined  Lever,  Screw  and  Pulley  Gear — 
Example  I. — Bottle  Screw- Jack — Example  II. — Traversing 
Screw-Jack — Screw  Press  for  Bales — Screw  Bench  Vice — 
Example  III. — Endless  Screw  and  Worm-Wheel—Com- 
bined Pulley,  Worm,  Worm-Wheel  and  Winch-Drum— 
Worm- Wheel  Lifting  Gear — Example  IV. — Questions  .  160-173 


LECTURE  XVI. 

General  Idea  of  the  Mechanism  in  a  Screw-cutting  Lathe— Motions 
of  the  Saddle  and  Slide  Rest— Velocity  Ratio  of  the  Change 
Wheels — Rules  for  Calculating  the  Required  Number  of 
Teeth  in  Change  Wheels — Examples  I.  II. — Movable  Head- 
stock  for  a  Common  Lathe — Descriptions  of  a  Screw-cutting 
Lathe  and  of  an  Electrically  Driven  Hexagon  Turret  Lathe, 
with  Frontis-plates  and  complete  sets  of  Detail  Drawings 
— Questions 174-206 


LECTURE  XVII. 

Hydraulics — Definition  of  a  Liquid— Axioms  relating  to  a  Liquid 
at  Rest — Transmission  of  Pressure  by  Liquids— Pascal's 
Law — "Head"  or  Pressure  of  a  Liquid  at  Different  Depths 
— Total  Pressure  on  a  Horizontal  Plane  immersed  in  a 
Liquid  —  Lord  Kelvin's  Wire-testing  Machine  —  Total 
Pressure  on  any  Surface  immersed  in  a  Liquid — Examples 
I.  II. — Questions *.  207-213 


CONTENTS.  Xlll 

< 

LECTURE  XVIII. 

tAom 

tJsef  ul  Data  regarding  Fresh  and  Salt  Water— Example*  I.  II.  III. 
IV. — Centre  of  Pressure — Immersion  of  Solids — Law  of 
Archimedes — Floating  Bodies — Example  V. — Atmospheric 
Pressure — The  Mercurial  Barometer — Example  VI. — Low 
Pressure  and  Vacuum  Water  Gauges — Example  VII. — The 
Siphon — Distinction  between  Solids,  Liquids  and  Gases — 
Definitions  of  perfect,  viscous,  and  elastic  Fluids — Cohesion 
— Questions  .  .  » 214-226 

LECTURE   XIX. 

Hydraulic  Machines — The  Common  Suction  Pump— Example  I. — 
The  Plunger,  or  Single-acting  Force  Pump — Example  II. — 
Force  Pump  with  Air  Vessel — Continuous-delivery  Single- 
acting  Force  Pump  without  Air  Vessel — Combined  Plunger 
and  Bucket  Pump — Double-acting  Force  Pump — Example 
III. — Centrifugal  Pumps — Example  IV.— Questions  .  .  227-240 

LECTURE  XX. 

Bramah's  Hydraulic  Press — Bramah's  Leather  Collar  Packing- 
Examples  I.  II.  —  Large  Hydraulic  Press  for  Flanging 
Boiler  Plates — The  Hydraulic  Jack— Weem's  Compound 
Screw  and  Hydraulic  Jack — Example  III. — The  Hydraulic 
Bear  or  Portable  Punching  Machine— The  Hydraulic  Accu- 
mulator— Example  IV. — Questions 241-258 

LECTURE   XXI. 

Motion  and  Velocity — Uniform,  Variable,  Linear,  and  Angular 
Velocity — Unit  of  Velocity— Acceleration— Unit  of  Ac- 
celeration—  Acceleration  due  to  Gravity — Graphic  Re- 
presentation of  Velocities— Composition  and  Resolution 

of  Velocities Newton's  Laws  of  Motion — Formulae  for 

Falling  Bodies  —  Formulas  for  Linear  Velocity  —  with 
Uniform  Acceleration — Atwood's  Machine  with  Experi- 
ments —  Results  and  Formulae  —  Galileo's  and  Eater's 
Pendulum  Experiments — The  Path  of  a  Projected  Bodj 
— Centrifugal  Force  due  to  Motion  in  a  Circle — Cen- 
trifugal Force  Machine  —  Experiments  I.  II.  III.  — 
Example  I. — Balancing  High-speed  Machinery — Cen- 
trifugal Stress  in  the  Arms  of  a  Fly-wheel— Example  IL 
—Energy  —  Potential  Energy— Kinetic  Energy  —  Accu 
mulated  Work — Accumulated  Work  in  a  Rotating  Body— 


XIV  CONTENTS. 

The  Fly- wheel— Radius  of  Gyration — Example  III.— The 
Fly  Press— Example  IV. — The  Energy  Stored  in  a  Rotating 
Fly-wheel— Motion  on  Bicycle  and  Railway  Curves- 
Momentum — Examples  VI.  to  IX. — Questions  .  •  .  259-296 

LECTURE  XXII. 

Some  Properties  of  Materials  employed  by  Mechanics — Essen- 
tial Properties — Extension— Impenetrability — Contingent 
Properties  —  Divisibility — Porosity — Density — Cohesion — 
Compressibility  and  Dilatabilifcy  —  Rigidity  —  Tenacity- 
Malleability  —  Ductility  —  Elasticity  —  Fusibility  —  Load, 
Stress,  and  Strain — Total  Stress,  and  Intensity  of  Stress- 
Tensile  Stress  and  Stress — Example  I. — Compressive  Stress 
and  Strain — Example  II. — Limiting  Stress  or  Ultimate 
Strength — Safe  Loads  and  Elasticity — Limit  of  Elasticity 
— Hooke's  Law — Factors  of  Safety — Modulus  of  Elasticity 
—Ratio  of  Stress  to  Strain— Examples  III.-V.— Resilience 
or  Work  Done  in  Extending  or  Compressing  a  Bar  within 
the  Elastic  Limit— Examples  VI. -IX.  — Single  Riveted 
Lap  Joints— Example  X. — Questions.  •  •  .  297-315 


LECTURE  XXIII. 

•Stresses  in  Chains — Shearing  Stress  and  Strain— Example  I. — 
Torque  or  Twisting  Moment — Torsion  of  wires — Table 
giving  the  strength,  moduli  of  Elasticity  and  Rigidity  of 
various  materials — Strength  of  Solid  Round  Shafts — Ex- 
ample II. — Table  giving  the  Horse-Power  which  steel 
shafting  will  transmit  at  various  speeds  —  Strength  of 
Hollow  Round  Shafts— Relation  between  the  Twisting 
Moment  and  Horse-Power  transmitted  by  Shafting,  as  well 
as  the  Diameter  necessary  to  transmit  a  given  Horse- 
Power—Examples  III.  IV.— Questions  •  .  .  316-328 

LECTURE  XXIV. 

Hooke's  Coupling  or  Universal  Joint — Double  Hooke's  Joint — 
Sun  and  Planet  Wheels — Cams — Heart  Wheel  or  Heart- 
shaped  Cam — Cam  for  Intermittent  Motion — Quick  Return 
Cam — Example — Pawl  and  Ratchet  Wheel — Reversible 
Pawl— Masked  Ratchet— Silent  Feed— Watt's  Parallel 
Motion — Parallel  Motion — Questions  ....  329-342 


CONTENTS.  XV 

LECTURE  XXV. 

Reversing  Motions — Planing  Machine — Reversing  by  Friction 
Cones  and  Bevel  Wheels — Whitworth's  Reversing  Gear — 
Quick  Return  Reversing  Motion — Whitworth's  Quick 
Return  Motion — Whitworth's  Slotting  Machine — Common 
Quick  Return — Horizontal  Shaping  Machine — Quick  Re- 
turn with  Elliptic  Wheels — Vertical  Slotting  Machine — 
Questions  .  .  .  343~354 

LECTURE  XXVI. 

Measuring  Tools  and  Gauges — Limit  Gauges — Micrometer  Screw 
Gauge — Sir  Joseph  Whitworth's  Early  Realisation  of 
Mechanical  Accuracy — Improved  Equivalents  Micrometer 
Gauge— A  New  Set  of  English  Gauges— Whitworth's 
Millionth  Measuring  Machine — Whitworth's  Standard 
Workshop  Measuring  Machine — Questions  .  .  .  355-366 


APPENDICES,  Rules  and  Examination  Papers  of  The  Board  of 
Education,  City  and  Guilds  of  London,  and  Institution  of 
Civil  Engineers  for  Admission  of  Students  —  C.G.S.  Sys- 
tem of  Units — Practical  Electrical  Units — Examination 
Tables  of  Useful  Constants,  Logarithms,  Antilogarithms, 
and  Functions  of  Angles 367-419 

INDEX 421-430 


ELEMENTARY  MANUAL 

•N 

APPLIED  MECHANICS. 


LECTURE  I. 

CONTENTS.  —  Definition  of  Applied  Mechanics  —  Force  —  Matter  —  Unit  of 
Force  —  The  Elements  of  a  Force  —  Graphic  Representation  of  Forces 
_  Forces  in  Equilibrium  —  Action  and  Reaction  —  Resultant  and  Com- 
ponents —  Resultant  of  Forces  acting  in  a  Straight  Line  —  Terms  and 
Definitions  used  in  Measuring  or  Calculating  Pressures  and  their 
Effects  —  Definitions  of  Scalar,  Vector  and  Rotor  Quantities  —  Engi- 
neering Calculations. 

Applied  Mechanics  is  that  branch  of  applied  science  which  not 
only  explains  the  principles  upon  which  machines  are  designed, 
made  and  act,  but  also  describes  their  construction  and  applica- 
tions, as  well  as  how  to  calculate  and  test  their  strength  and 
efficiency. 

Before  a  student  can  successfully  master  any  science,  he  must 
thoroughly  understand  the  units  of  measurement  that  have  been 
adopted  in  calculating  results,  and  he  should  also  have  a  clear 
conception  of  the  exact  meaning  of  the  various  terms  employed. 
Consequently,  we  shall  commence  the  study  of  Elementary  Applied 
Mechanics  with  definitions  and  with  units  of  force,  work  and 
power.* 

Force  is  any  cause  which  produces,  or  tends  to  produce,  motion  or 
change  of  motion  in  the  matter  upon  which  it  acts. 

Matter  is  anything  which  can  be  perceived  by  one  or  more  of  the 
senses,  and  which  can  be  acted  on  by  force. 

Matter  exists  under  three  conditions  :  (i)  Solids,  (2)  Liquids, 
(3)  Gases.  For  example,  pieces  of  wood  and  of  iron  are  solids  ; 
water  and  mercury  are  liquids  ;  whilst  air  and  oxygen  are  gases. 

*  For  the  units  of  length,  surface  and  cubic  measure,  and  for  the 
mensuration  of  areas  and  solids,  the  student  is  referred  to  Lectures  I.  II. 
and  III.  of  Author's  "Elementary  Manual  on  Steam  and  the  Steam 
Engine,"  issued  by  the  publishers  of  this  book.  *. 

A 


2  LECTURE   I. 

Bodies  are  therefore  limited  portions  of  matter.  When  the 
resistance  to  motion  of  a  body  is  equal  to  or  greater  than  the 
force  applied,  so  that  no  motion  takes  place,  the  body  is  said  to 
be  subjected  to  pressure. 

Solids  do  not  yield  readily  to  pressure,  for  they  tend  to  retain 
their  original  shape  and  size,  whereas  liquids  and  gases  yield  to 
a  very  slight  pressure,  and  consequently  possess  no  definite  shape. 
A  gas  differs  from  a  liquid  since  it  possesses  the  property  of 
indefinite  expansion.  A  liquid  has  therefore  a  definite  size,  but 
not  a  definite  shape,  whilst  a  gas  has  neither  definite  shape  nor 
definite  size. 

Unit  of  Force. — The  British  unit  of  force  is  the  pound  avoir- 
dupois, or  GRAVITATION  UNIT  or  ENGINEER'S  UNIT.  The  magni- 
tude of  a  force  is  therefore  reckoned  by  the  number  of  pounds  of 
matter  which  the  force  would  support  against  gravity.  For 
example,  a  force  of  i  Ib.  means  that  force  which  would  just  lift 
the  weight  of  a  fixed  mass  at  a  fixed  place  if  acted  on  by  gravity 
&lone.  But  the  force  of  gravity  varies  at  different  parts  of  the 
earth's  surface,  being  slightly  greater  at  the  Poles  than  at  the 
Equator.  Consequently,  our  engineer's  unit  of  force  is  only  an 
absolute  or  invariable  one  at  a  standard  place,  such  as  at  Green- 
wich sea  level. 

The  Elements  of  a  Force. — When  a  force  acts  upon  a  body, 
then,  in  order  to  fully  determine  its  effect  we  must  know  the 
thre«  following  elements: — (i)  The  point  or  place  of  application 
of  the  force.  (2)  The  direction  in  which  the  force  acts.  (3)  The 
magnitude  of  the  force. 

(i)  Place  of  Application. — In  the  case  of  the  force  of  gravity 
acting  on  a  body,  the  place  of  application  may  be  considered  to  be 
the  whole  mass  of  the  body,  or  we  may  estimate  the  whole  weight 
of  the  body  as  concentrated  at  one  point,  termed  the  centre  of 

*  Where  great  accuracy  of  measurement  is  required  an  absolute  or 
invariable  unit  offeree  must  be  selected.  An  absolute  unit  of  force  may  be 
defined  as  that  force  which,  acting  for  unit  time  on  unit  mass,  will  produce 
unit  change  of  velocity.  If  the  units  of  time,  mass,  and  velocity  be  the 
second,  pound,  and  foot  per  second  respectively,  then  we  may  define  the 
absolute  unit  of  force  (called  the  poundal)  as  that  force  which,  acting  for  one 
second  on  a  mass  of  one  pound  would  produce  a  change  in  velocity  of  one  foot 
per  second.  It  has  been  determined  experimentally  that  if  a  body  be  let 
fall  freely  in  vacuo,  near  the  earth's  surface,  the  attractive  force  of  the 
earth  will  produce  a  change  of  velocity  every  second  of  g  ( =  32^2  nearly) 
feet  per  second.  Clearly,  then,  the  gravitation  unit  is  g  times  the  abso- 
lute unit.  Hence  the  following  relation  between  the  gravitation  and 
absolute  units  of  force: — A  force  of  one  pound  =  g poundals,  or  a  force 
of  one  poundal  =  ifg  pound. 

In  this  book  the  gravitation  or  the  engineer's  unit  of  force  and  work 
will  be  used. 


ELEMENTS    OF    A    FORCE.      (  3 

gravity  of  the  body.  When  an  extended  surface  is  subjected  to 
pressure,  as  in  the  case  of  a  tank  containing  a  liquid,  or  the  piston 
of  an  engine  subjected  to  the  pressure  of  a  gas,  the  whole  area 
under  pressure  may  be  considered  as  the  place  of  application. 
When  a  body  is  pulled  by  means  of  a  rope,  or  pushed  by  means 
of  a  rod,  or  supported  on  a  small  area,  then  we  consider  the  force 
as  acting  at  a  point. 

(2)  Direction.  —  The  direction  of  a  force  is  the  line  or  path  in 
which  it  tends  to  move  the  body  on  which  it  acts. 

(3)  Magnitude.  —  The  magnitude  of  the  force  is    the  pound* 
pull  or  pressure  which  the  force  exerts  upon  the  body  on  whicii 
it  acts. 

Graphic  Representation  of  Forces.  —  When  a  force  acts  on 
a  body  at  a  point,  its  three  elements  may  be  conveniently  repre- 
sented as  follows  :  — 


, 
j  '  1\>  '  20      P 

SCALE  DIAGRAM  OF  A  FORCE. 

Where  0  represents  the  point  of  application,  the  straight  line 
OP  (with  the  arrow-head),  shows  the  direction  in  which  the  force 
acts,  and  the  length  of  the  divided  line  OP  indicates  to  scale  the 
magnitude  of  the  force.*  (See  end  of  this  lecture  for  further 
definitions.) 

Forces  in  Equilibrium.  —  (i)  When  any  number  of  forces 
acting  upon  a  body  neutralize  each  other's  effects  (i.e.,  leave  the 
body  in  the  same  condition  as  to  rest  or  motion  as  before  th« 
application  of  the  forces),  these  forces  are  said  to  be  in  equi- 
librium. 

,(2)  Forces  which  are  in  equilibrium  may  be  applied  to  or 
removed  from  a  rigid  body  without  altering  its  condition  as  to 
rest  or  motion. 

(3)  Two  equal  and  opposite  forces  destroy  each  other's  effects  ; 
and,  conversely,  no  two  forces  can  destroy  each  other's  effects  un- 
less they  are  equal  and  opposite. 

(4)  A  force  will  have  the  same  effect  at  whatever  point  in  its 
own  direction  it  may  be  supposed  to  act  ;  and,  conversely,  if  a 
force  have  the  same  effect  whether  it  act  at  one  or  other  of  two 
given  points,  then  the  straight  line  joining  these  points  (with  the 
suitably  directed  arrowhead)  will  be  the  direction  of  the  force. 

Action  and  Reaction.  —  (i)  Whenever  a  fixed  rigid  body  is 

*  In  the  case  of  the  above  figure  the  force  is  represented  as  equal  to 
22  Ibs.  Students  will  find  it  convenient  to  plot  down  the  repiesentation 
of  forces  in  their  exercise  books  to  a  scale  of  ^  of  an  inch  to  a  pound,  01 
hundredweight,  or  ton,  according  to  the  values  of  the  forces. 


4  LECTURE   I. 

acted  on  by  a  force,  then  naturally  there  is  at  once  set  up  in  that 
body  a  secondary  force,  or  a  force  of  reaction,  equal  and  opposite 
in  direction  to  the  primary  force. 

(2)  Hence  action  and  reaction  are  equal  and  opposite,  and 
neutralize  each  other's  effects. 

For  Example. — Suppose  a  weight  is  placed  on  a  rigid  horizontal 
table.  In  the  table  there  is  set  up  an  opposing  force  or  upward 
reaction  which  exactly  counterbalances  the  downward  force  of  the 
weight.  If  this  were  not  the  case,  then  motion  would  take  place, 
and  either  the  table  would  give  way,  or  the  weight  would  sink 
through  the  table ! 

Resultant  and  Components. — (i)  If  any  number  of  forces 
acting  upon  a  body  be  replaced  by  a  single  force  which  shall  have 
the  same  effect,  then  this  force  is  termed  the  resultant  of  these 
forces,  and  the  forces  are  called  the  components  of  their  resultant. 

(2)  The  operation  of  finding  the  resultant  of  any  number  of 
forces  is  called  the  composition  of  forces;  and  finding  the  com- 
ponents is  termed  the  resolution  of  forces. 

Resultant  of  Forces  acting  in  a  Straight  Line. — (i)  The 
resultant  of  any  number  of  forces  acting  in  the  one  direction 
along  one  straight  line  is  equal  to  their  sum,  and  acts  in  that 
direction. 

For  Example. — Let  P1P,P3P4  be  any  four  forces  acting  in  one 
direction  along  one  straight  line,  then  their  resultant — 

R    =    P!    +    P,    +    P3    +    P4 

(2)  If  the  forces  do  not  all  act  in  one  direction,  then  the  re- 
sultant is  equal  to  the  difference  between  the  resultant  of  those 
acting  in  one  direction  and  the  resultant  of  those  which  act  in 
the  opposite  direction,  and  has  the  direction  of  the  greater  of  the 
two  resultants. 

For  Example. — Let  Pfff4  be  any  four  forces  acting  along 
one  straight  line  to  the  right  hand  or  in  a  positive  direction ;  and 
QiQaQ«  ^e  any  three  forces  acting  along  the  same  straight  line, 
but  in  an  opposite  or  left-hand  or  negative  direction,  and  let 

Qi  +  Q,  +  Q3  be  less  than  pi  +  p»  +  pi  +  P4 

Then  the  resultant, 

R  =   (P,  +  P,   +  P,   *   P4)  -  (Q,   +  Q,   +   Q8) 
and  acts  in  the  same  direction  as  PtP2P3P4,  and  along  the  same 
straight  line. 

If  equilibrium  existed  between  these  two  sets  of  oppositely 
directed  forces,  then  their  algebraical  sum  would  be  zero,  or  the 
resultant  would  vanish  ;  t.e., 

(P,  +  P,   +  P,   +   P4)   -   (Q,  +  Q,  +  Q,)  -  K  -  O 


TERMS,    ETC.,    USED   IN   MEASURING   PRESSURES.  C 

A  familiar  illustration  of  the  above  reasoning  is  the  game  of 
"  the  tug  of  war,"  when,  say,  a  batch  of  sailors  are  pitted  against 
a  corresponding  number  of  soldiers,  each  batch  pulling  their 
utmost  at  the  opposite  ends  of  a  rope,  and  in  opposite  directions, 
with  the  view  of  obtaining  a  resultant. 

Terms  aid  Definitions  used  in  Measuring  or  in  Calcu- 
lating the  Values  of  Pressures  and  their  JJffects. — The  only 
terms  (except  in  special  cases)  which  are  herein  used  for  the  above  pur- 
poses, are :— ^ 

(1)  Pressure;  (2)  Total  Pressure ;  (3)  Intensity  of  Pressure ;  (4)  Resultant 
Pressure;  (5)  Centre  of  Pressure. 

DEFINITIONS.— (i)  Pressure  is  a  general  term  for  the  value  or  amount 
of  force  acting  between  bodies.  In  certain  cases  the  words  push  or  pull 
may  be  used. 

Note. — The  general  term  Pressure  has  hitherto  been  loosely  used  t* 
mean  either  the  Total  Pressure  (P)  or  to  indicate  the  Intensity  of  Pressure 
(p).  Hence  it  is  desirable  to  clearly  state  in  all  problems  and  writings 
which  of  these  is  meant  when  the  term  pressure  is  alone  used. 

(2)  Total  Pressure  (P)  means  the  whole  force  acting  between  bodies.     It 
is   usual   to  measure  or  estimate  total  pressure  in  Ibs.,  thus,  P=pA.  Ibs. 
But,  when  the  values  are  great,  it  may  be  given  or  found  in  cwts.,  or  in 
tons. 

(3)  Pressure  or,  more  exactly,  Intensity  of  Pressure  (p)  is  the  pressure  per 
unit  area  ;  (e.g.)  Ibs.  per  square  inch  or  Ibs.  per  square  fooc.     Ex.  Let  p  = 
10  Ibs.  per  sq.  in.  ;  A=ioo  sq.  ins.     Then  P  =pA>  =  lox  100=1000  Ibs. 

(4)  Resultant  Pressure  (PR)  is  the  mean  pressure  per  unit  area  (p)  multi- 
plied by  the  total  area  (A)  under  consi  Oration.     Here,  PR=^A. 

(5)  Centre  of  Pressure  (Pc)   is  the  point  at  which  a  single  force  will 
balance  the  total  pressure.     Or,  it  may  be  defined  as  the  point  at  which  the 
resultant  pressure  acts. 

Differences  in  Nomenclature. — Students  may  find  in  "Examina- 
tion Questions,"  or  in  papers  read  before  various  Institutions,  and  in 
books  by  different  authors,  other  terms  than  those  just  defined.  For 
example,  the  word  Thrust  has  been  recently  introduced  to  mean  the  total 
pressure  (P);  and  the  simple  word  pressure  to  indicate  the  Intensity  of 
Pressure  (p),  or  Pressure  in  Ibs.  per  square  inch  (p),  as  defined  above. 

Moreover,  to  show  the  necessity  for  a  more  definite  distinction  between 
total  and  intensity  of  pressure,  if  you  ask  an  Engineer  or  Manufacturer 
the  pressure  of  steam  in  his  boiler,  he  will  say  (e.g.)  100  Ibs.  ;  when  it  is 
actually  100  Ibs.  per  square  inch  by  the  steam  gauge,  i.e.,  where^;  =  ioolbs. 

Resultant  Thrust  is  sometimes  used  for  the  aforementioned  Resultant 
Pressure  or  Pressure  Resultant. 

The  Author  believes  that  these  new  terms  only  confuse 
students.  It  has  hitherto  been  the  custom  amongst  Mechanical 
Engineers  and  Naval  Architects  to  chiefly  confine  the  words  Thrust 
and  Resultant  Thrust  to  problems  and  cases  dealing  with  the 
screw  propellers  and  the  thrust  blocks  in  connection  with  the 
screw  shafts  of  steamships. 


DEFINITIONS — SCALAR,   VECTOR,  AND  ROTOR. 


NOTE  FOB  PAGE  3. — The  word  Vector  was  used  in  defining  the  conditions 
of  equilibrium  in  frames,  consequently  it  may  be  as  well  to  define  the 
following  terms  here  : — 

Scalar. — A  quantity  which  has  no  relation  to  definite  direction  in  space, 
or  which  is  considered  apart  from  such  direction,  is  called  a  "Scalar  "  or 
"  Scalar-Quantity." 

Vector. — A  geometrical  quantity  which  is  related  to  a  definite  direction 
in  space  is  called  a  "Vector"  or  "Vector-Quantity." 

Vector-Quantity. — This  requires  for  its  complete  determination  (i)  the 
magnitude,  (2)  the  direction,  and  (3)  the  sense  to  be  given. 

A  vector-quantity  may  be  geometrically  represented  by  a  line,  if — 

(1)  The  length  of  the  line  represents  to  scale  the  magnitude  of  the  quantity. 

(2)  The  line  be  placed  in  the  proper  direction. 

(3)  The  proper  sense  or  way  be  given  to  the  line. 

The  sense  is  usually  indicated  by  an  arrowhead  on  the  line. 
The  line  itself  with  its  direction  and  sense  is  called  a  Vector-}- 
Suppose  that  a  force  of  known  magnitude  acts  along  a  line  from  P  to 
->-Q  ;  then,  the  Vector  is  written  down  as  PQ7  with  a  bar- line  over  the  two 
letters  P  and  Q. 

Any  quantity,  whether  scalar  or  vector  (considered  as  occupying  a  definite 
position  in  space),  is  t-aid  to  be  localised.  Thus  the  mass  of  a  body  in  a 
given  position  is  a  localised  scalar,  and  a  force  acting  on  a  body  at  a 
definite  point  is  a  localised  vector. 

Vector  /Sum. — The  sum  of  a  number  of  vectors  is  often  called  the 
Kesultant  Vector,  and  in  relation  to  this  resultant  the  other  Vectors  are 
called  Components. 

To  add  a  number  of  vectors,  place  the  first  anywhere,  the  beginning  of 
the  second  to  the  end  of  the  first,  and  so  on,  then  the  vector  from  the 
beginning  of  the  first  to  the  end  of  the  last  is  the  SUM  OF  THE  GIVEN 
VECTORS  (Henrici  and  Turner}. 

Rotor.  — A  localised  vector  is  called  a  Rotor  (Clifford}. 

EXAMPLE. — If  A,  B,  C,  D  denote  four  vectors 
acting  at  the  point  0  to  find  the  sum  or  result- 
ant. 

From  any  convenient  point  a,  draw  ab  parallel  in 
direction  and  containing  as  many  units  of  length  as 
there  are  units  in  the  vector  A.  From  the  end  b, 
draw  be  parallel  to  B,  and  equal  in  magnitude  to 
it.  Similarly,  cd  parallel  and  equal  to  C,  and  de 
equal  and  parallel  to  D.  The  resultant  in  direction 
and  magnitude  of  the  four  vectors  is  the  dotted 
line  joining  the  initial  point  a  to  the  final  point  e. 
»  ,-  ->a  If  a  vector  E,  equal  in  magnitude  but  in  the 

/.''  \c  contrary  direction,  acts  at  O,  then  the  five  vectors 

S  \     A,  B,  C,  D,  E  will  have  a  sum  equal  to  zero,  or  the 

five  vectors  are  in  equilibrium. 

The  polygon  abcdea,  when  applied  to  forces  is 
called  the  polygon  of  forces.  Hence,  if  any  number 
of  vectors  act  at  a  point  and  can  be  represented 
by  the  sides  of  a  closed  polygon  taken  in  order,  the 


SUM  OP  VECTORS 


ACTING  AT  A  POINT,  vectors  are  in  equilibrium. 


NOTES  ON   ENGINEERING  CALCULATIONS.  5& 

We  shall  return  to  the  graphic  representation  of  forces,  &c., 
when  we  come  to  deal  with  the  parallelogram  and  triangle  of 
forces  and  their  application  to  ascertaining  the  stresses  on  simple 
structures.* 

Note  Regarding   Engineering  Calculations.— Engineering 

students  should  clearly  understand,  that  there  is  no  necessity  or  advantage 
to  be  gained  in  working  out  their  arithmetical  results  to  a  greater  nicety, 
than  the  tool.-',  rules,  gauges,  and  instruments  placed  at  their  disposal 
will  enable  them  to  measure  with  accuracy. 

For  example,  a  skilled  mechanic  who  is  furnished  with  a  steel  footrule 
and  callipers  may  express  his  ideas  of  length  to  the  Vt»  °r  at  best  to  the 
rH  ('OI)  of  an  inch,  which  is  equivalent  to  i  part  in  1200.  It  would  be 
ridiculous,  therefore,  to  ask  him  to  calculate  such  lengths  to  the  third 
decimal  place. 

A  carpenter  who  uses  the  well-known  3  ft.  rule  may  be  perfectly 
satisfied  if  he  measures  to  the  ^  of  an  inch,  i.e.,  to  I  part  in  500. 

A  mason  will  usually  be  satisfied  if  he  can  measure  to  within  £  of  an  inch  1 

The  captain  of  a  sailing  vessel  could  not  be  expected  to  spot  the  position 
of  his  ship  at  sea,  to  within  a  couple  of  nautical  miles ;  so  it  is  no  use  to 
ask  him  to  place  his  boat  on  a  particular  meridian  1 

Note  on  Questions  in  Proportion.— When  dealing  with  all  such 
questions,  it  is  best  for  the  student  to  ask  himself — 

(1)  What  is  required?    Then  to  put  the  corresponding  given  value  in 

the  3rd  term,  with  x,  y,  or  2  in  the  4th  for  the  value  to  be  found. 

(2)  Will  the  answer  be  greater  or  less  than  the  value  in  the  3rd  term  T 

Then  to  put  the  greater  or  the  less  given  value  (according  to  this 
answer)  into  the  2nd  term,  and  the  remaining  known  quantity 
in  the  ist  term. 

The  answer  for  x,  y,  or  z  is  then  equal  to  the  product  of  the  2nd  and  yd 
terms,  divided  by  the  ist. 
EXAMPLES  : — 

ist   :    2nd   :  :    $rd      :    4th  (Term*). 
(a)  If,  10    :    100    ::    1000    :    x. 

Then, 
(6)  If,  loo 

This  method  is  more  convenient  to  the  elementary  student  than  dealing 
directly  with  fractional  ratios. 

*  We  have  intentionally  made  this  Lecture  a  short  one,  and  have  not 
appended  any  question*,  "because  at  the  first  meeting  of  a  session  the 
Lecturer  has  to  give  a  series  of  general  instructions  to  his  students,  and 
the  class  is  seldom  so  completely  formed  as  to  make  it  worth  while  setting 
any  home  work  until  the  second  meeting. 


LEOTTJBE  II. 

CONTENTS.— Work— Unit  of  Work— Examples  I.  II.  III.  IV.— Work  done 
against  a  Variable  Kcsistance— Example  V.— Diagrams  of  Work— 
With  Uniform  Resistance — With  a  Uniformly  Increasing  Resistance— 
With  a  Uniformly  Decreasing  Resistance— With  a  Combination  of 
Uniform  and  Variable  Loads — Example  VI. — Power  or  Activity — 
Units  of  Power— The  Horse-power  Unit— To  find  the  Horse-power  of 
any  working  Agent — Example  VII. — Uses  of  Squared  Paper — Clark's 
Adjustable  Curve — Example  VIII.— Questions. 

Work. — If  a  force  acts  upon  a  body  and  causes  that  body  to 
move  through  a  distance,  then  the  force  is  said  to  have  done  work. 
It  does  not  matter  how  long  the  operation  takes,  whether  a  second, 
a  minute,  an  hour,  or  a  day,  or  even  a  year,  the  same  amount  of 
work  is  done  by  the  force  acting  through  the  distance.  Time, 
therefore,  does  not  come  into  the  question  of  estimating  work 
done,  but  we  must  have  a  force  overcoming  a  resistance  through 
a  definite  distance.  If  the  force  applied  be  inadequate  to  over- 
come the  resistance  of  the  body  to  motion,  then  no  work  is  done. 
The  amount  of  work  done  therefore  depends  solely  upon  the  pro- 
duct of  the  force  applied  (or  the  resistance  overcome)  and  the 
distance  through  which  it  acts  in  its  own  direction. 

Or,  Work   =   Force    x    Distance. 

Unit  of  Work.* — The  unit  of  work,  is  the  work  done  in  over- 
coming unit  force  through  unit  distance.  £Tow,  since  the  British 
unit  of  force  is  th^  pound,  and  unit  distance  the  foot,  the  British 
unit  of  work  is  called  the  foot-pound,  and  is  therefore  the  work 
done  when  a  resistance  of  i  Ib.  is  overcome  through  a  distance  of 
i  foot. 

EXAMPLE  I. — If  a  weight  of  i  Ib.  be  elevated  a  vertical  dis- 
tance of  i  ft.  against  the  force  of  gravity,  then  i  foot-pound  of 

*  In  the  case  of  heavy  work  the  unit  foot-ton  is  sometimes  used  in  this 
country.  A  foot-ton  simply  means  one  ton  raised  one  foot  high  against 
gravity,  or  a  force  of  one  ton  exerted  through  a  distance  of  one  foot,  or  a 
resistance  of  one  ton  overcome  for  a  distance  of  one  foot.  In  Electrical 
Engineering  the  unit  of  work  is  the  work  done  in  overcoming  a  resistance 
of  one  dyne  through  a  distance  of  one  centimetre.  It  is  called  the  Erg. 
Since  the  weight  of  i  gramme  is  =  981  dynes,  the  work  done  in  raising 
i  gramme  through  a  vertical  height  of  i  centimetre  against  the  force  of 
gravity  is  981  ergs  or  (g)  ergs.  One  foot-pound  —  i  '356  x  io7  ergs. 


UNIT    OF   WORK. 


work  has  been  performed.  If  10  pounds  be  elevated  vertically 
through  a  distance  of  10  ft.,  then  result  is  (10  x  io)=  100  ft.-lbs. 
of  work. 


Fig.  for  Example  III. 


UNIT  OF  WORK. 

W  =  i  lb.  weight. 
R  =  i  Ib.  reaction. 


ILLUSTRATING  WORK  DONE. 

W  =  Weight  in  Ibs. 
P  =  Pull  in  Ibs. 
L  =  Length  in  feet. 


EXAMPLE  II. — If  a  body  offers  a  constant  resistance  to  motion 
in  any  direction  of  P  Ibs.,  and  if  it  be  forced  along  a  distance 
of  L  ft.,  in  that  direction,  then  tbo  work  done  is  P  x  L  ft.-lbs. 
Or,     Work  done  =        Force        x        Distance 

i.e.      Foot  pounds       =        P  Ibs.       x        L  feet. 

Suppose  a  cart  with  its  load  weighs  W  Ibs.  and  offers  a  con- 
stant resistance  of  P  Ibs.  to  traction  along  a  road,  and  that  it  is 
pulled  through  a  distance  of  L  feet ;  then, 

The  work  done  =  P  x  L  (ft.-lbs.) 

EXAMPLE  III. — In  drawing  a  loaded  cart  along  a  level  road,  a 
horse  has  to  exert  a  constant  pull  of  100  Ibs. ;  how  much  work 
will  be  done  in  10  minutes  supposing  the  horse  to  walk  at  the 
rate  of  6000  yards  an  hour  ? 


Distance  in  feet  through 
which  the  resistance  of 
100  Ibs.  is  overcome  in  10 
minutes. 


_6ooo  (yds.)  x  10  (m.) 
60  (m.) 

=  3000  ft. 


3  (ft-) 


Work  done  in  10  minutes  =  P  x  L. 


=  100  x  3000. 

=  300,000  ft.-lbs. 


EXAMPLE  IV. — A  traction  engine  is  employed  to  draw  a  loaded 
waggon  along  a  level  road  where  the  resistance  to  be  overcome  is 


8 


LECTURE    II. 


100  Ibs.  per  ton.     How  many  foot-pounds  of  work  are  expended 
in  drawing  10  tons  over  100  yards  ? 


TRACTION  ENGINE  AND  LOAD. 


1.  Tractive  force  =  100  Ibs.  per  ton. 

2.  Total  pull,  P,  =  100  (Ibs.)  x  10  (tons) 

3.  Distance,  L,     -  100  (yds.)  x  3  (ft.) 

4.  Work  done      =  P  x  L. 

„  „  =  IOO    X     IO    X     100     X    3. 

„          „  300,000  ft.-lbs. 


Work  Done  against  a  Variable  Kesi  stance.— If  the  resist- 
ance varies  whilst  the  force  overcom- 
ing it  acts  through  a  known  distance, 
then  the  work  clone  will  be  measured 
by  the  product  of  the  average  resist- 
ance and  the  distance.  If  the  resist- 
ance varies  uniformly,  its  average  can 
be  found  by  adding  its  values  at  the 
commencement  and  end  of  the  motion, 
and  dividing  by  two. 

EXAMPLE  V. — Explain  the  method  of 
estimating  the  work  done  by  a  force, 
awid  define  the  unit  of  work.  The  sur- 
face of  the  water  in  a  well  is  at  a  depth 
of  20  feet,  and  when  500  gallons  have 
been  pumped  out,  the  surface  is  lowered 
to  26  feet.  Find  the  number  of  units 
of  work  done  in  the  operation,  the 
weight  of  a  gallon  of  water  being  10  Ibs. 
(S.  and  A.  Exam.  1887.) 
For  an  answer  to  the  first  part  of  this  question  refer  to  the 
previous  part  of  this  lecture. 


WORK  VARYING 
UNIFORMLY. 


i.  Weight  of  water  raised 


=  weight  of  500  gallons. 
=  500  x  10  Ibs. 


Or, 


P,    =5000  Ibs. 


DIAGRAMS    OF    WORK. 


[Distance   through   which    the 

2.  Mean  height  water  is  lifted  =  -j      centre  of  gravity,  G  (of  raised 

[     water),  has  been  elevated. 


Or, 

3.  Work  done 


20  +  26 
2 

21  ft. 


ft. 


5000  x  23. 
115,000  ft.-lbs. 


" 


,« 10 ft. M 


Plbs. 
I 


\ 
Bibs. 


DIAGRAM  OF  UNIFORM  WORK. 


Diagrams  of  Work. — (i)  Against  a  Uniform  Resistance. — If 
the  resistance  overcome  is  uniform,  then  the  work  done  may  be 
graphically  represented  by  the  area 
of  a  rectangle. 

To  find  the  work  done  in  over- 
coming a  uniform  resistance  of 
5  Ibs.  through  a  distance  of  10  ft. : 
Plot  down  a  vertical  line  to  any 
convenient  scale  to  represent  P  (or 
5  Ibs.)  and  a  horizontal  line  to  the 
same  scale  to  represent  L  (or  10  ft.)  Then  complete  the  rectangle. 

The  area  P  x  L  or  5  x  10  =  50  ft.-lbs.  of  work. 

In  the  accompanying  figure  a  scale  of  -^  inch  has  been  used 
to  represent  both  i  Ib.  and  i  ft.,  consequently  each  of  the  SHIM  11 
squares  represents  to  scale  one  foot-pound  of  work. 

(2)  With  a  Uniformly  Increasing  Resistance, — If  the  resist- 
ance uniformly  increases  —  for  ex- 
ample, in  the  raising  of  a  length  of 
rope  or  chain  vertically  by  one  end 
from  the  ground,  then  the  work 
done  may  be  graphically  represented 
by  the  ai  ea  of  a  right-angled  triangle, 
where  P  represents  the  total  weight 
of  chain  in  Ibs.,  and  L  its  total 
length  in  feet. 


Plbs. 


DIAGRAM  OF  WORK  FOR  AN 
INCREASING  RESISTANCE. 


The  Total  Work  done  = 


PxL 


ft.-lbs. 


Here  the  work  done  per  foot  of  length  of  chain  lifted,  uniformly 
increases  from  a  minimum  to  a  maximum,  until  the  whole  rope 
or  chain  is  off  the  ground.  When  any  known  length,  I,  has  been 
lifted,  then  the  area  enclosed  by  the  triangle  whose  horizontal 
side  is  I,  and  vertical  side  p  represents  the  work  done. 


IO 


LECTURE 


(3)  With  a  Uniformly  Decreasing  Resistance. — If  the  resistance 
uniformly  decreases,  as  in  the  case  of 
winding  a  rope  or  chain  upon  the 
barrel  of  a  winch  or  crane,  then  the 
work  done  will  also  be  represented 
graphically  by  the  area  of  a  right- 
angled  triangle,  where  P  represents 
the  total  weight  of  rope  or  chain  in 
pounds  being  lifted  at  the  start,  and 
L  its  length  in  feet. 


DIAGRAM  OF  WORK  FOR  A 
DECREASING  KESISTANCE. 


Px  L 

.•.  The  Total  Work  done  =  —     -  ft.-lbs. 


Here  the  work  done  per  foot  of  length  of  chain  lifted,  gradually 
diminishes  from  a  maximum  at  the  start  to  a  minimum,  when  the 
last  foot  is  being  lifted. 

As  in  Case  (2),  you  can  at  any  time  know  the  work  done  or  still 
to  be  done  from  the  scale  diagram,  if  you  know  the  length  of  chain 
lifted  or  to  be  lifted. 

For  example,  if  I  feet  have  still  to  come  on  to  the  barrel,  then 
the  vertical  ordinate  p  on  the  scale  diagram  will  represent  the 


pull  being  exerted  at  the  time,  and  consequently  i represents 

the  work  still  to  be  done. 

Or,  generally,  with  any  gradually 
increasing  or  decreasing  resistance 
the  work  done  is  equal  to  the  mean 
of  the  average  resistance  in  Ibs. 
x  the  distance  through  which  it 
acts  in  feet. 

(4)  With  a  Combination  of  Uni- 
form and  Variable  Loads. — "When 
one  part  of  a  load  is  uniform  and 
another  part  variable,  as  in  the  case 
of  lifting  a  weight  with  a  chain, 
by  winding  the  chain  on  the  barrel 
of  a  winch  or  crane,  the  diagram 
of  work  for  the  uniform  load  is 
naturally  a  rectangle,  and  for  the 
chain  a  triangle  if  the  chain  is 
completely  wound  on  to  the  barrel, 
or  a  trapezoid  if  there  is  still  some 
portion  of  it  to  be  lifted.* 

*  See  p.  5  of  the  Author's  "  Elementary  Manual  on  Steam  and  the  Steam 
Engine"  for  how  to  find  the  Area  of  a  Trapezoid. 


^ 

•1 

X 

N. 

n 

\1 
xx 

(1 

It 

m 

N. 

L 

—    —  -> 

Q 


K 


M 


DIAGRAM  OF  WORK  FOR  A 
COMBINATION  OF  UNI- 
FORM AND  VARIABLE 
LOADS. 


DIAGRAMS    OF   WORK.  II 

Let  P     .     —the  uniform  pull  required  to  lift  the  load  *r  over- 

come the  uniform  resistance. 
L     .     =  the  distance  the  weight  is  lifted. 
Pv  Pi    =  t^e  weights  of  chain  hanging  at  the  commence- 

ment and  at  the  finish  of  the  lift. 
Work  done  in  lifting  the  uniform  load  =  P  x  L 

Work  done  in  lifting  the  variable  load  =*-±  —  ^?  x  L 


.-.  Whole  work  =  P  x  L+Li  x  L= 

2  2 

„         „      =  Area  of  the  figure,  DEFC. 

The  diagram  DEFC  represents  the  work  done  and  also 
the  variation  of  the  resistance  during  the  lift.  The  rectangle 
ABCD  represents  the  work  done  in  overcoming  the  uniform  load, 
and  the  trapezoid  ABFE  the  work  done  in  overcoming  the 
variable  load.  The  resistance  at  any  instant  of  the  lift  will  be 
represented  by  the  vertical  line  drawn  from  the  horizontal  base 
DC  to  the  inclined  line  EF  through  the  point  on  DC  or  AB 
which  represents  the  position  of  the  load  at  that  instant.  The 
part  of  this  vertical  fine  intercepted  between  AB  and  EF  will 
represent  the  resistance  offered  by  the  variable  part  of  the  load 
at  the  instant  considered.  Thus,  at  the  commencement  of  the 
lift  the  total  resistance  is  P  +pl  and  represented  by  DE,  at  tho 
end  of  the  lift  the  total  resistance  is  P  +  p,  and  represented  by 
CF.  At  £,  J,  and  j  of  the  lift  the  total  resistances  are  repre- 
sented by  the  vertical  lines  GH,  KL,  and  MN  respectively,  whil  i 
the  resistances  due  to  the  variable  part  of  the  load  at  these  poh-ts 
are  represented  by  the  lengths  grH,  £L,  and  rriN  respectively 
If  the  final  resistance  due  bo  the  variable  part  of  the  load  was 
zero  (as  would  be  the  case  if  the  whole  of  the  chain  were  wound 
on  to  the  barrel)  then  the  diagram  of  work  for  this  part  of  the 
load  would  be  the  triangle  AEB. 

EXAMPLE  VI.  —  Explain  fully  the  mode  of  measuring  the  work 
done  by  a  force.  What  unit  is  adopted  ?  A  weight  of  2  cwts. 
is  drawn  from  a  mine,  30  fathoms  deep,  by  a  chain  weighing  i  lb. 
per  linear  foot  ;  find  the  number  of  units  of  work  done.  (S.  and 
A.  Exam.  1893.)  Also  find  the  resistance  offered  when  the 
weight  has  been  raised  through  J,  ^,  and  j  of  the  whole  depth 
respectively. 

ANSWER.  —  (i)  The  work  done  by  a  force  is  measured  by  the 

product  of  the  force  into  the  distance  throagh  whieh  that  force 

moves  in  its  own  direction.     If  P  be  the  force  in  pounds,  and  L  the 

distance  in  feet  through  which  it  moves  in  its  own  direction,  then 

Work  done-  Px  L  f  t,  Ibi, 


12  LECTURE    II. 

(2)  The  unit  of  work  adopted  in  this  country  is  the  work  done 
when  a  force  of  one  pound  is  moved  through  a  distance  of  i  foot, 
and  is  called  the  foot-pound  (ft.-lb.). 

(3)  Referring   to  the  previous  figure,  make  AB  to  represent 
30  x  6  =  1 80  ft.,  the  depth  of  the  mine,  AD  to  represent  2x112 
=  224   Ibs.   the   weight    of   material   raised,    AE    to   represent 
i  x  1 80=  1 80  Ibs.  the  weight  of  chain  at  beginning  of  lift.     Then 
assuming  the  whole  of  the  chain  to  be  wound  up,  complete  the 
rectangle  ABCD,  and  join  E  and  B.      The  area  of  the  figure 
DEBO  then  represents  the  work  done. 

.  • .  Work  done  =  area  DEBC  -  \  (DE  +  CB)  x  AB.     But  DE  = 
DA  +  AE  =  224  +  180  =  404  Ibs. 
CB  =  DA  =224  Ibs.,  AB  =  1 80  ft. 
.".Work  done  =  \  (404  +  224)  x  180  ft.-lbs.  =  56,520  ft.-lbs. 

(4)  The  resistance  at  \  lift,  or  when  the  weight  has  been  raised 
45  ft.,  is  Gh  =  Gg  +  gh  =  224  +  $x  1 80  =  359  Ibs. 

At  ^  lift  the  resistance  is  K£  =K&  +  kl  =  224  + Jx  180  = 
314  Ibs. 

At  f  lift  the  resistance  is  M?i  =  Mm4-  mn=  224  +  |  x  180  = 
269  Ibs. 

Power  or  Activity  is  the  rate  of  doing  work* — In  estimating 
or  testing  the  power  of  any  agent  the  time  in  which  the  work  is 
done  must  be  noted  and  taken  into  account.  Consequently,  we 
speak  of  the  activity  or  power  of  a  man,  of  a  horse,  or  of  an 
engine,  as  capable  of  doing  so  many  foot-pounds  of  work  per 
minute. 

Units  of  Power,  f — The  unit  of  power  adopted  in  this  country 
is  called  the  horse-power.  It  is  the  rate  of  doing  work  at  33,000 
ft.-lbs.  per  min.,  or  550  ft.-lbs.  per  sec.,  or  1,980,000  ft.-lbs  per 
hour. 

The  Horse-power  Unit  was  introduced  by  James  Watt,  the 
great  improver  of  the  steam  engine,  for  the  purpose  of  reckoning 
the  power  developed  by  his  engines.  He  had  ascertained  by 
experiment  that  an  average  cart-horse  could  develop  22,000  foot- 
pounds of  work  per  minute,  and  being  anxious  to  give  good  value 
to  the  purchasers  of  his  engines,  he  added  50  per  cent,  to  this 
amount,  thus  obtaining  (22,000+  11,000)  the  33,000  foot-pounds 
per  minute  unit,  by  which  the  power  of  steam  and  other  engines 
has  ever  since  been  estimated. 

*  The  vrordpoweris  very  frequently  misapplied  by  writers  and  students, 
for  they  often  call  the  mere  pull,  pressure,  or  force  exercised  on  or  by  an 
agent  the  power.  Students  should  strenuously  avoid  this  misuse  of  the 
word  power,  and  never  employ  it  in  any  other  sense  than  as  expressing  a 
rate  of  doing  work,  or  activity. 

t  In  Electrical  Engineering  the  Unit  of  Power  is  called  the  Watt,  and  it 
equals  lo7  ergs  per  second,  or  746  Watts  —  i  horse-power. 


TO  FIND  THE  HORSE-POWEK  OF  ANT  WOKKTNO  AGENT.      1  3 

To  find  the  Horse-power  of  any  Working  Agent.—  Divide 
the  number  of  foot-pounds  of  work  which  it  does  in  one  minute  by 
33»000- 

Let  P  =  Pull  exerted  or  resistance  overcome  in  pounds. 
L  =  Length  or  distance  through  which  P  acts. 
M  =  Minutes  the  agent  is  at  work. 
H.P.  =  Ilorse-power. 
Then, 

H  P  -     FxL      •  p  =  II.  P.  x  33000x11  .  L  _  H.  P.  x  33000  x  M 
33000  x  M  '  L  P 

EXAMPLE  VII  —  In  what  way  is  the  rate  of  doing  work  measured 
hi  horse-power  ? 

If  40  cubic  feet  of  water  be  raised  per  minute  through  330  feet, 
what  horse-power  of  engine  will  be  required,  supposing  that  there 
is  no  loss  of  friction  or  other  resistances  ?  Note.  —  I  cubic  foot  of 
water  weighs  62  J  Ibs.  (S.  and  A,  Exam.  1892). 

ANSWER.  —  The  rate  of  doing  work,  as  measured  in  horse-power, 
is  equivalent  to  33,000  foot-pounds  of  work  done  per  minute. 

ist.  i  cubic  foot  of  water  weighs  62  J  Ibs. 

•  •.40  cubic  feet  of  water  weigh  40x62^  =  2500  Ibs. 

2nd.  Work  done  per  minute  =  ^         c  2500  (Ibs.)  x  330  ft. 

M  I 

3rd.  .-.  H.P.=       PxL      =  2500  x  330  =  825000  =  25> 

33000  x  M      33000  x  i       33000 

Note.  —  Students  will  find  it  a  great  advantage,  as  well  as  a  sav- 
ing of  time  not  to  multiply  figures  together  until  the  last  stage  of 
the  answer  has  been  reached,  and  then  to  cancel  all  common 
factors  in  numerator  and  denominator.     For  example,  in  the 
answer  to  the  above  question  we  might  proceed  thus  — 
i  st.  40  cubic  feet  of  water  =  40  x  62  \  Ibs. 
2nd.  Work  done  per  minute  =  40  x  62!  x  330  ft.  -Ibs. 

4  i 

40x62^x^0 
3rd.  .•.  Horse-power  of  engine  =  -  — 


10 


10          10 

The  process  consists  in  this  —  the  factor,  330,  can  be  cancelled 
from  numerator  and  denominator,  leaving  100  as  the  denominator. 
The  factor,  10,  can  then  be  divided  out  of  40  in  the  numerator 
and  from  the  100  in  the  denominator,  thus  leaving  4  x  62  J  as  the 
numerator  and  10  as  the  denominator.  The  remainder  of  the 
work  is  evident. 


LECTURE    H. 


Uses  of  Squared  Paper. — Squared  paper  is  made  by  drawing 
a  number  of  equally  spaced  horizontal  lines  and  crossing  these  by 
vertical  ones  at  the  same  distance  apart.  The  paper  is  conse- 
quently covered  with  a  large  number  of  little  squares,  the  sides  of 
which  are  usually  one-tenth  of  an  inch  in  length.  In  order  to 
facilitate  the  measurement  of  distances,  every  tenth  line,  and  some- 
times every  fifth,  is  heavier  or  of  a  different  colour  to  the  others. 

By  the  aid  of  squared  paper  we  can  graphically  represent  how 
two  varying  quantities  depend  upon  each  other.  For  example, 
take  the  case  of  a  chain  being  gradually  lifted  from  the  ground,  as 
already  considered  in  connection  with  diagrams  of  work  and 
shown  by  the  first  of  the  following  figures.  The  length  and 
weight  of  chain  lifted  will  alter  as  the  upper  end  is  raised  ;  but 
the  suspended  weight  will  always  depend  upon  the  length  which 
hangs  freely  from  the  spring  balance.  In  fact,  any  change  in  the 
length  lifted  will  produce  a  corresponding  change  in  the  load 
registered  by  the  balance.  If  we  note  the  pull  indicated  by  the 
balance  for  different  lengths  of  hanging  chain,  we  shall  be  able  to 
obtain  a  line  or  curve  which  will  show  to  the  eye  how  these  two 
quantities  depend  upon  one  another.  Suppose  we  obtain  the 
following  results : — 


Length  of  "^ 
chain  lifted  V 
in  feet.  J 

I 

2 

3 

3'5 

4 

S 

6 

7 

8 

9 

10 

Weight  on 

chain  lifted  V 

0-8 

i  '4 

2'0 

2'5 

27 

3'S 

4  '3 

.TO 

5  '6 

6'2 

7-0 

in  Ibs.  J 

Corrected  ^ 
values  in  > 
Ibs.  J 

07 

I  '4 

,, 

- 

2'8 

3'5 

4*2 

4  '9 

,« 

6-3 

7-0 

We  shall  represent  the  lengths  of  the  hanging  chain  by  horizon- 
tal distances  which  are  termed  abscissa,  and  the  corresponding 
weights  by  vertical  distances  called  ordinates.  We  choose  such  a 
scale  for  these  quantities  as  will  enable  us  to  get  them  all  upon 
the  squared  paper  ;  at  the  same  time  we  keep  the  scales  as  large 
as  possible.  In  this  case  we  have  chosen  five  divisions  horizontally 
to  represent  one  foot,  and  five  divisions  vertically  for  one  pound. 
It  is,  however,  not  necessary  to  adopt  equal  scales  for  abscissae 
and  ordinates,  but  we  should  select  the  most  convenient  scale  for 
each  according  to  circumstances. 


USES   OF    SQUARED    PAPER.  I  5 

To  find  the  point  corresponding  to  the  fourth  column  in  the 
table,  take  B  at  3-5  on  the  horizontal  scale,  and  C  at  2-5  on  the 
vertical  one.  Draw  B  A  vertically  and  C  A  horizontally.  Then, 
the  intersection  of  these  two  lines  is  the  point  required.  In 
practice,  these  lines  are  not  actually  drawn,  but  the  point  is 
found  by  the  eye  with  the  assistance  of  the  lines  on  the  paper,  and 
a  x  or  ©  is  placed  to  mark  its  position.  When  all  the  points  have 
been  thus  plotted  from  the  table,  we  draw  a  mean  line  or  curve 
between  them.  In  this  case,  it  is  a  straight  line  passing  through 


Abscissae     or     Length 


Femt 


HANGING  CHAIN.  SQUARED  PAPER. 

RELATION  BETWEEN  LENGTH  AND  WEIGHT  OP  CHAIN. 


the  origin  0.  It  will  be  seen  that  this  line  does  not  pass  through 
all  the  points,  but  that  some  of  them  are  on  one  side  and  some  on 
the  other.  This  may  be  due  to  errors  of  observation  or  to 
irregularities  in  the  chain.  If  we  know  that  the  chain  is  uniform, 
the  points  ought  to  lie  along  the  straight  line  we  have  drawn, 
and  we  can  correct  for  errors  of  observation.  Thus,  the  point  A 
should  have  been  at  D,  and  the  correct  weight  for  that  length  of 
chain  is  represented  by  BD,  which  is  2*45  Ibs.  We  can  correct 
the  other  values  in  the  same  way,  and  so  obtain  the  numbers 
shown  in  the  third  line  of  the  table. 

When  the  points  lie  approximately  in  a  straight  line,  the 
nearest  mean  straight  line  is  best  found  with  the  help  of  a  fine 
thread  which  is  stretched  and  moved  among  the  points  until  it 


1 6  LECTURE    II. 

lies  most  evenly  amongst  them.  The  positions  of  its  ends  are 
then  marked  and  a  line  drawn  with  a  straight-edge  through  these 
marks.  When  the  points  do  not  lie  near  a  straight  line  a  smooth 
curve  may  be  drawn  through  them,  either  freehand,  or  by  aid  of 
French  curves,  a  thin  strip  of  wood  or  steel,  or  Clark's  paiant 


It — ~ 


CLARK'S  PATENT  ADJUSTABLE  CURVE, 

adjustable  curve.  This  consists  of  a  flexible  strip  of  celluloid 
with  a  brass  loop  A,  for  the  thumb  of  the  left  hand,  and  another 
B,  for  the  second  or  third  finger,  as  shown  by  the  accompanying 
figures.  Now,  if  these  two  loops  are  drawn  together  the  celluloid 
will  be  formed  into  some  curve,  the  shape  of  which  can  be  ad- 
justed by  moving  the  sliding  rod  C  with  the  right  hand  and 
fixing  this  curve  by  means  of  a  cord  (not  shown  in  the  figure) 
joining  B  with  a  V-shaped  groove  in  A.  It  may  also  be  used  as 


USES   OF  SQUARED   PAPK*. 


a  set  curve  for  the  purpose  of  transferring  a  curve  from  one 
drawing  to  another. 

When  we  represent  distances  along  one  scale  of  the  squared 
paper  and  forces  along  the  other,  then  the  areas  such  as  O  D  B 
indicate  to  scale  the  work  done  in  raising  the  length  of  chain  O  B. 
This  is  true,  whether  the  line  0  D  is  straight  or  curved.  We 
can,  however,  represent  any  two  quantities  which  depend  on  one 
another  by  a  curve  on  squared  paper. 

EXAMPLE  VIIT.— A  body  is  being  acted  upon  by  a  variable  lifting  force. 
When  the  body  is  lifted  x  feet  the  force  F  Ib.  is  observed. 


X 

0 

15 

25 

50 

70 

100 

125 

150 

180 

210 

F 

530 

525 

516 

490 

425 

300 

210 

160 

110 

90 

Plot  on  squared  paper  and  find  the  average  value  of  .Ffrom  x  =  o  to 
*=2io.  What  is  the  work  done  by  F  when  the  body  has  been  lifted 
210  feet?  (B.  of  E.  1904.) 

ANSWER— 


r~ 

i 

500 

•SK 

f: 

si^ij 

i:::::i 

. 

i- 

F      : 

-                4- 

400 

-^ 

|    :: 

rJI 

T 

rTrrHrr 

—  5p 

•*- 

t: 

:l::::: 

^  *vj 

300 

.  t  :  ...: 

Tt-f1 

i 

w 

|i 

^p-:::::::: 

?00 

ft 

:  jK§:i;::: 

"«* 

:|::::: 

-  ---[- 

L 

-  ^*  s 

.     , 

100 

k 

1 

::-:(- 

t 

—  H  —  -^ 

F:|S::| 

a 

""]  ' 

=  B 

0 

1 

• 

7  5^y?c 

es  { 

|§^S 

S::::i 

:  C 

20 40       60       80       100      120      140      160      180  -  200  210 

DIAGRAM  OP  WORK  DONE  BY  A  VARIABLE  FORCE. 

In  order  that  we  may  obtain  the  average  value  of  F,  we  must  nse  squared 
paper  since  the  lifting  force  is  variable.  Mark  off  the  distances  x  in  feet 
(to  gcale)  along  the  abscissa  0  C,  and  at  each  of  the  values  of  «,  plot 
ordinates  corresponding  to  the  values  of  F  in  Ibs.  Join  all  the  points  by 
a  curve  as  shown  in  the  above  diagram.  The  area  of  this  diagram  below 
the  curved  line  A  B,  represents  the  total  work  done  by  the  variable  force  Fibs., 
in  passing  through  a  distance  of  210  feet. 

We  can  obtain  the  area  of  the  diagram  O  A  B  C  either  by  aid  of  aplanimeter 
or  by  inding  the  average  value  of  F  and  multiplying  this  by  the  distance 


1 LECTURE   H. 

through  which  Fhas  moved,  viz.,  210  feet.  But  the  ordinary  process  used 
by  engineers  for  finding  the  area  of  a  diagram  such  as  0  A  B  C  is  to  divide 
9-0  into  10  equal  parts  and  then  to  measure  the  value  of  F  at  the  centre 
of  each  part.  The  sum  of  these  values  divided  by  10  gives  us  the  average 
value  of  F.  If  this  number  be  now  multiplied  by  the  value  of  0  C  the 
final  result  will  be  the  area  of  the  diagram  in  foot-pounds. 

Thus,  by  the  latter  method  we  find  the  average  value  of  F=305  Ibs. 

Hence  the  total  work  =  average  value  of  Fx  distance  it  acts. 

Or,  the  total  work  done  =  3o$  x  2io-64,050  ft.-lbs. 


LECTURE  H.— QUESTIONS.  Ip 

LECTURE  II. — QUESTIONS. 

1.  Define  tbe  unit  of  work.     What  name  is  given  to  this  unit?    In 
drawing  a  load  a  horse  exerts  a  constant  pull  of  120  Ibs. ;  how  much  work 
will  be  done  in  15  minutes,  supposing  the  horse  to  walk  at  the  rate  of 
3  miles  an  hour  1    Ans.  475,200  ft.-lbs. 

2.  How  is  the  work  done  by  a  force  measured  ?    The  resistance  to 
traction  on  a  level  road  is  150  Ibs.  per  ton  of  weight  moved  ;  how  many 
foot-pounds  of  work  are  expended  in  drawing  6  tons  through  a  distance 
of  150  yards  1     Ant.  405,000  ft.-lbs. 

3.  Distinguish  between  force  and  work  done  by  a  force.     How  is  each 
respectively  measured  !    A  traction  engine  draws  a  load  of  20  tons  along 
a  level  road,  the  tractive  force  on  the  load  being  150  Ibs.  per  ton.     Find 
the  work  done  upon  the  load  in  drawing  it  through  a  distance  of  500  yards. 
Ans.  4,500,000  ft.-lbs. 

4.  Find  the  number  of  units  of  mechanical  work  expended  in  raising 
136  cubic  feet  of  water  to  a  h/.ight  of  20  yards.     The  weight  of  a  cubio 
foot  of  water  is  62^  Ibs.     Ans,  510,000  ft.-lbs. 

5.  A  weight  of  4  tons  is  raised  from  a  depth  of  222  yards  in  a  period  of 
45  seconds  ;  calculate  the  amount  of  work  done.     Ant.  5,967,360  ft. -Ibs. 

6.  A  hole  is  punched  through  a  plate  of  wrought-iron  one-half  inch  in 
thickness,  and  the  pressure  actuating  the  punch  is  estimated  at  36  tons. 
Assuming  that  tbe  resistance  to  the  punch  is  uniform,  find  the  number  of 
foot-pounds  of  work  done.    Ans.  3360  ft. -Ibs. 

7.  How  is  work  done  by  a  force  measured  ?    Give  some  examples.     Set 
out  a  diagram  of  the  work  done  in  drawing  a  body  weighing  10  Ibs.  up  a 
smooth  incline  4  feet  high,  marking  dimensions. 

8.  A  train  of  12  coal  waggons  weighing  133  tons  is  lifted  by  hydraulic 
power  (two  waggons  being  raised  at  a  time)  through  20  feet  in  12  minutes. 
Estimate  the  work  done  in  foot-tons.     Taking  the  average  of  work  done, 
how  many  foot-pounds  are    done    per    minute?     Ant.    2660    ft. -tons  ; 
496,533*3  ft.-lbs.  per  minute. 

9.  The  plunger  of  a  force-pump  Is  8f  inches  in  diameter,  the  length  of 
the  stroke  is  2  feet  6  inches,  and  the  pressure  of  tbe  water  is  50  Ibs.  per 
square  inch  ;  find  the  number  of  units  of  work  done  in  one  stroke,  »nd 
plot  out  a  diagram  of  work  to  scale.     Ans.  7517  ft.-lbs. 

10.  A  chain  30  feet  long,  and  weighing  100  pounds  per  yard,  lies  coiled 
on  the  ground.     Find  by  calculation  and  by  a  scale  diagram  of  work  how 
many  units  of  work  would  be  expended  in  just  raising  it  by  the  top  end 
from  the  ground.     Ans.  15,000  ft.-lbs. 

11.  A  chain,  weighing  30  Ibs.  to  the  fathom,  is  employed  to  lift  i  ton  to 
a  height  of  30  ft.  by  winding  the  chain  on  a  barrel.     Find  by  calculation 
and  by  a  scale  diagram  of  work,  how  many  units  of  work  will  be  expended 
— (a)  when  the  outer  end  of  the  chain  is  brought  home  to  the  barrel ; 
(b)  when  18  feet  of  it  are  still  hanging  free  with  the  weight  at  the  end  of 

it.     Ans.  (a)  69,450  ft.-lbs.  ;  (b)  28,320  ft.-lbs. 

12.  Define  the  following  mechanical  terms  : — Force,  work,  unit  of  work, 
power,  activity,  and  horse-power.     A  horse  drawing  a  cart  at  the  rate  of 
2  miles  per  hour  exerts  a  traction  of  156  Ibs. ;  find  the  number  of  units  of 
work  done  in  one  minute.     Ans.  27,456  ft. -Ibs. 

13.  In  what  way  is  the  rate  of  doing  work  measured  in  horse-power  ? 
If  loo  cubic  feet  of  water  be  raised  per  minute  through  330  feet,  what 
horse-power  of  engine  will  be  required,  supposing  that  there  is  no  loss  by 
friction  or  other  resistances  1    A<M.  62'$  h.p. 

14.  If  a  horse,  walking  at  the  rate  of  2$  miles  per  hour,  draws  104  Ibs. 
out  of  a  well  by  means  of  a  cord  going  over  a  wheel,  how  many  unite  of 
work  would  he  perform  in  one  minute  t    Ant.  22,880  fc-lbi. 


LECTURE  II. — QUESTIONS. 


15.  What  unit  do  you  employ  in  measuring  force,  and  what  unit  in 
measuring  the  work  done  by  a  force  ?    A  horse  exerting  a  pull  of  40  Ibs. 
per  ton  draws  a  load  of  15  cwt.  along  a  level  road  ;  how  far  will  the  horse 
travel  in  10  minutes  if  he  does  work  on  the  load  at  the  rate  of  £  horse- 
power ?    Ans.  5500  ft. 

16.  Distinguish  between  the  expressions  "foot-pound"  and  "horse- 
power "  by  giving  a  clear  definition  of  each.     A  bucket  when   filled  with 
water  weighs  180  Ibs.,  and  is  raised  at  a  uniform  rate  from  a  depth  of 
150  feet  in  eight  minutes.     Find  the  work  done  in  one  minute. 

An*-  3375  ft.-lbs. 

17.  What   work   in   foot-pounds  is  done  in  raising  the  materials  for 
building  a  brick  wall  50'  high,  12'  long,  and  2'  3"  in  thickness,  if  one  cubic 
foot  of  brickwork  weighs  112  Ibs.  ?    Ans.  3,780,000  ft. -Ibs. 

1 8.  A  man  of  150  Ibs.  climbs  a  hill  regularly  1200'  vertically  per  hour  ; 
at  another  time  he  climbs  a  staircase  at  2^'  per  second.    Find  in  each  case 
the  horse-power  expended  by  the  man.     Ans.  '69  h.-p.  ;  '68  h.p. 

19.  An  express  train  going   at   40    miles   per  hour  weighs  150  tons ; 
the  average  pull  on  it  is  12  Ibs.  per  ton,  what  is  the  horse-power  exerted? 
This  power  is  only  40  per  cent,  of  the  total  indicated  power  of  the  engine  ; 
find  the  indicated  power.     Ans.  192  h.-p.;  I.  H.  P.  =  480. 

20.  Water  at  750  Ibs.  per  square  inch  pressure  acts  on  a  piston  one 
square  foot  in  area,  through  a  stroke  of  I  foot ;  what  is  the  work  that 
such  water  does  per  cubic  foot  ?  and  per  gallon  ?     If  an  hydraulic  com- 
pany charges  18  pence  for  a  thousand  gallons  of  such  water,  how  much 
work  is  given  for  each  penny  ?    Ans.   108,000  ft.-lbs.  ;    17,280  ft.-lbs. ; 
960,000  ft.-lbs. 

21.  Explain   how   squared  paper   is   used,  and  mention  a  few  of  the 
purposes  to  which  it  is  applied. 

22.  Plot  out  a  curve  from  the  following  data  showing  the  pressure  on  a 
piston  at  various  distances  from  the  commencement  of  the  stroke : — 


Distance  in  feet. 

o 

•I 

'2 

'3 

'4 

•5 

•6 

7 

•8 

'9 

IX) 

Pressure    in    \ 
Ibs.  per 
square  inch.    J 

20 

21 

21 

20 

19 

18-5 

18 

I3-5 

9 

4'5 

o 

23.  A  chain  weighing  10  Ibs.  per  foot  of  its  length  is  240  feet  long,  and 
hangs  vertically  ;  what  work  is  done  in  winding  the  chain  upon  a  drum  ? 
Am.  288,000  ft.- Ibs. 


C     -21      ) 


LECTURE  III. 

CONTENTS. — The  Moment  of  a  Force — Principle  of  Moments  applied  to  the 
Lever — Experiments  I.  II.  III. —Pressure  on  and  Reaction  from  the 
Fulcrum — Equilibrant  and  Resultant  of  two  Parallel  Forces — Oouples 
— Centre  of  Parallel  Forces  or  Position  of  Equilibrant  and  Resultant — • 
Centre  of  Gravity — Examples  of  Centre  of  Gravity — The  Lever  when 
its  weight  is  taken  into  Account — Examples  I.  II. — Position  of  the 
Fulcrum— Example  III. — Questions. 

The  Moment  of  a  Force,  with  r expect  to  a  point,  is  equal  to  the 
force  Multiplied  %  the  perpendicular  distance  from  the  point  to  its 
line  of  action. 

For  example,  suppose  a  body  to  be  resting  on  the  point  O,  and 
a  force,  P,  to  be  applied  to  the  body  in  the  direction  PA.  Then, 
if  the  perpendicular  distance  from  0 
to  the  line  of  action  of  the  force  be 
UA,  the  moment  of  the  force  P,  tending 
fo  turn  the  body  about  the  point  O,  is 
P  x  AO.  If  the  force  be  reckoned 
in  pounds,  and  the  perpendicular  dis- 
tance in  feet,  the  product  will  be  in 
pounds-feet.  The  student  must  there- 
fore  avoid  confusing  the  answer  with 
ft.-lbs.  of  work. 

Principle  of  Moments. — If  any  number  of  forces  act  in  one 
plane  on  a  rigid  body,  and  if  these  forces  are  in  equilibrium  ;  then 
the  principle  of  moments  asserts  that  the  sum  of  the  moments  of 
those  forces  which  tend  to  turn  the  body  in  one  direction  about  a 
point,  is  equal  to  the  sum  of  the  moments  of  the  forces  which  t#nd  to 
turn  the  body  in  the  opposite  direction  about  the  same  point. 

Or,  to  state  the  principle  more  concisely,  the  opposing  moments 
about  the  point  are  equal  when  equilibrium  exists. 

If  the  moments  of  those  forces  which  tend  to  turn  the  body  to 
the  right  hand  (i.e.,  in  the  direction  of  the  motion  of  the  hands 
of  a  clock)  be  called  positive  (  + ),  and  the  moments  of  the  remain- 
ing set  of  forces  which  tend  to  turn  the  body  to  the  left  hand  («".«., 
in  the  opposite  direction  to  the  movement  of  the  hands  of  a  clock) 
be  called  negative  (  —  ),  then  the  algebraical  sum  of  the  moments  of 
the  forces  which  act  in  one  plane,  and  which  are  in  equilibrium 
about  a  point,  is  zero. 


22 


LECTURE    III. 


Principle  of  Moments  applied  to  the  Lever.— A  lever  is 
simply  a  rigid  rod,  bar,  or  beam,  capable  of  turning  about  a  fixed 

point  called  the  fulcrum  (F).  Act- 
ing on  the  lever  in  one  direction  is  a 
force  or  set  of  forces  which  we  shall 
term  the  pull  or  pressure  (P),  and 
in  the  other  direction  there  is  the 
resistance  or  set  of  resistances  to  be 
overcome,  which  we  shall  term  the 
weight  (W).  The  pressure,  P,  and 
the  weight,  W,  produce  a  reaction 
at  the  fulcrum,  which  is  called  the 
equilibrant  (E). 

The  parts   of  the  lever  between 
the  fulcrum  and  the  pressure  and 
between  the  fulcrum  and  the  weight 
are  called  the  arms  of  the  lever. 
w  The  accompanying  three  figures 

LEVERS  IN  EQUILIBRIUM.         show  three  ways  in  which  F,  P  and 

W  may  be  arranged  with  a  straight 

lever.*     In  each  case,  the  opposing  moments  about  the  julcrum  are 
equal,  when  the  lever  is  in  equilibrium. 

Or,          .         .     P    x    AF     =     W    x    BF 

satisfies  the  conditions  for  equilibrium  in  the  case  of  a  lever. 

EXPERIMENT  I. — To  prove  the  foregoing  statements,  take  a 
rigid  homogeneous  bar,  AB,  of  uniform  section.  Let  the  bar  be 
of  yellow  pine,  i  inch  deep,  ^  inch  broad,  and  32  inches  long. 
Attach  to  the  ends,  A  and  B,  light  flexible  cords  with  small  hooks 
at  their  lower  ends,  and  attach  to  the  middle  of  the  bar  at  F 
another  light  flexible  cord,  and  pass  this  cord  over  a  pulley  having 
a  minimum  of  friction  at  its  bearings.  Fix  such  a  weight  to  the 
free  end  of  this  middle  cord  as  will  just  counterpoise  the  bar  and 
cords.  Test  the  accuracy  of  this  preliminary  adjustment  by 

*  The  levers  represented  by  the  above  three  figures  are  assumed  to  be 
without  weight.  A  force,  P,  acts  through  a  perfectly  flexible,  weightless 
cord  at  A,  and  another  force,  W,  acts  also  through  an  exactly  similar 
cord  at  B,  with  the  fulcrum  at  F  in  each  case.  In  the  second  and  third 
case  the  cord  attached  at  A  passes  over  a  frictionless  pulley  in  order  to 
give  the  necessary  direction  to  the  force  P.  These  three  relative  positions 
of  P,  W  and  F  used  to  be  termed  the  first,  second  and  third  order  of  levers; 
but  there  is  no  necessity  for  any  such  distinction,  since  all  the  student  has 
to  remember  is  this,  that  when  equilibrium  exists  the  opposing  moments 

about  the  fulcrum  are  equal,  i.e.,  (P  x  AF  =  W  x  BF),  or,  _  =  _,  or  — =  — 

W     AF         P      BF 
The  ratio  W  to  P  is  termed  the  theoretical  advantage  of  the  lever. 


PRINCIPLE    OF    MOMENTS    APPLIED    TO    THE    LEVER. 


observing  whether  the  bar  hangs  horizontal,  and,  if  pulled  down 
or  up  a  little,  whether  the  weight  balances  the  bar  and  cords, 
Now  affix  equal  weights,  P,  of,  say,  4  oz.,  to  the  cords  hanging 


P=4  oz. 


EXPERIMENT  I.  ON  PARALLEL  FORCES. 

from  the  ends  A  and  B,  and  add  an  equilibrating  weight,  E,  of 
8  oz.  to  the  end  of  the  central  cord.  You  will  find  that  the  bar 
will  come  to  rest  in  a  horizontal  position,  thus  proving  that  — 


i.e., 
Or, 
i.e., 


P 

(oz.) 


P        x    BF 

4  (oz.)    x    1  6" 

BF  :  AF 

BF          16         4 

i 

AF         76 

i 

AF    = 

1  6"    = 

P:P   :: 
P 
P 

Also,  that  the  equilibrant, 

E     =     P     +     P 
8  oz.  =  4  oz.  +  4  oz. 

If  P  and  P  are  now  removed  from  the  ends  A  and  B,  and  a 
single  weight,  R,  of  8  oz.  be  hung  from  F  (as  represented  by  the 
vertical  dotted  line  and  arrow),  the  result  as  far  as  the  balancing 
of  the  system  is  concerned  will  be  unaffected. 

Consequently,        R  E  P      +      P 

i.e.,  8  oz.    =    8  oz.    —    4  oz.    +    4  oz. 

Or,  the  resultant  of  two  equal  parallel  forces  acting  in  the  same 
direction  is  equal  to  the  sum  of  the  two  forces,  and  acts  midway 
between  them  and  parallel  to  them  —  i.e.,  at  the  same  point  as  the 
equilibrant,  and  in  the  same  line  therewith,  but  in  the  opposite 
direction. 


LECTURE    IIT. 


EXPERIMENT  II. — Take  another  rigid  homogeneous  bar,  AB,  of 
the  same  uniform  section  as  the  previous  bar,  but  let  its  length 
be  24  inches.  Attach  cords  with  hooks  to  the  ends  A  and  B,  and 
t/o  a  point  F,  say  8  inches  from  A  and  16  inches  from  B.  Pass 
this  latter  cord  over  the  guide-pulley,  and  fix  it  there  until  you 


8'-' 


=24  oz. 


Q=  ti  oz. 


=16  oz. 

EXPERIMENT  II.  ON  PARALLEL  FORCES. 

have  just  added  sufficient  weight  to  the  end  At)  balance  the 
longer  end  BF;  then  unfix  the  end  of  the  middle  cord,  and 
attach  such  a  weight  to  it  as  will  counterpoise  the  whole  system. 
Now  attach  to  the  cord  at  A  a  weight  P  =  1 6  oz.  ;  to  the  other 
end,  B,  a  weight  Q ;  and  a  weight  at  E,  so  as  to  again  balance 
the  whole  system.  It  will  be  found  that  Q  equals  8  oz.  and 
E  equals  24  oz.,  thus  proving  that — 


Or, 


i.e., 


P         x    AF  =        Q        x 

1  6  (oz.)    x     8''  =8  (oz.)    x 

P  :Q  ::    BF  :  AF 

P         _BF  16         2 

Q   =     "AF  8~         I 


BF 

i6v 


Or,  the  point  F  is  twice  the  distance  from  the  end  B  that  it  is 
from  the  end  A,  and  P  has  twice  the  value  of  Q. 
Also,  that  the  equilibrant, 

E       =       P        +       Q 

For,  24  oz.    =    1 6  oz.    +    8  oz. 

If  P  and  Q  be  now  removed  from  the  cords  at  A  and  B,  and  a 
single  weight,  R,  of  24  oz.,  be  hung  from  F  (as  represented  by 
the  vertical  dotted  line  and  arrow),  the  result,  as  far  as  the  balanc- 
ing of  the  system  is  concerned,  will  be  unaffected. 


PRESSURE    ON,  AND    REACTION    FROM,  THE   FULCRUM.        25 

Consequently,        R      -       E      =      P       +     Q 
For,  24  oz.  =  24  oz.  =   1 6  oz.  +  8  oz. 

Or,  the  resultant  of  any  tvto  parallel  forces  acting  in  the  same 
direction  is  equal  to  the  sum  of  the  two  forces,  and  acts  parallel  to 
them  and  at  a  point  between  them,  so  that  the  ratio  of  the  forces  is 
inversely  proportional  to  their  distances  from  the  point ;  or  so  that — 
P  :  Q  ::  BF  :  AF 

Pressure  on,  and  Reaction  from,  the  Fulcrum. — You  may 

also  conclude  from  these  two  experiments,  if  the  lever  had 
been  balanced  on  a  knife-edge  or  journals,  that  the  pressure  on 
the  fulcrum  due  to  the  forces  P  and  Q  would  have  been  equal  to 
and  act  in  the  direction  of  the  resultant  R,  and  that  the  reaction 
from  the  fulcrum  would  have  been  equal  to  and  act  in  the 
direction  of  the  equilibrant  E. 

EXPERIMENT  III. — Supposing  that  in  the  last  experiment,  after 
adjusting  the  lever  by  placing  a  counterpoise  weight  at  A,  in  order 
to  bring  the  beam  to  a  horizontal  position,  and  after  balancing 
the  weight  of  the  beam  and  cords  by  an  equivalent  weight  at 
position  E,  you  added  a  weight  Q,  of  8  oz.,  to  the  cord  at  B,  and 
a  weight  E,  of  24  oz.,  to  the  cord  attached  at  F,  the  beam  would 
turn,  and  would  only  be  brought  to  a  horizontal  position  by 
attaching  a  weight  at  A  of  16  oz.  Hence  you  observe  that  P 
acts  at  A  as  the  equilibrant  both  in  direction  and  magnitude  to 
the  two  unequal  parallel  wdike  forces  Q  and  E.  Consequently 
a  force  equal  and  opposite  in  direction  to  P  would  be  the  resultant 
of  the  two  forces  Q^and  E ;  and  it  would  replace  their  combined 
effect  on  the  balanced  beam. 

Further,  P  =  E  -  Q; 

for,  1 6  oz.  =  24  oz.  -  8  oz. 

And  the  moments  about  the  position  where  the  equilibrant  acts 
are  equal, 

for  Q  x  BA  =  E  x  FA 

i.e.,  8  (oz.)  x  24"  =  24  (oz.)  x  8" 

Equilrbrant  and  Resultant  of  Two  Parallel  Forces.— 

From  the  above  experiments  you  conclude  that  the  equilibrant 
and  the  resultant  of  any  two  like  parallel  forces  are  equal  to 
their  sum,  and  any  two  unlike  parallel  forces  are  equal  to  their 
difference.  ,  . 

Couples.— When  the  two  parallel  forces  are  equal  and  act  in 
opposite  directions  upon  a  body,  they  are  termed  a  couple.  Ihe 
perpendicular  distance  between  the  two  forces  is  termed  iM 
arm  of  the  couple,"  and  the  "  moment  of  the  couple"  is  the  product  of 


LECTURE  III. 


-me  of  the  forces  and  the  arm.     A  couple  simply  tends  to  cause 
rotation  of  the  body  upon  which  it  acts,  for  it  has  no  resultant, 


snce 


=  P  -  P  =  0. 


One  couple  can,  however,  be  equilibrated  or  balanced  by  another 
oouple  of  an  equal  moment,  actingxm  the.same  plane,  and  tending  to 


A  COUPLE. 


TWO   BALANCING  COUPLES. 


turn  the  body  in  the  opposite  direction.  In  the  accompanying 
figure  the  couple  P,AB,P  will  be  balanced  by  the  couple  Q,CD,Q 
if  their  moments  are  equal ;  i.e.,  if 

P    x  AB  =  Q  x  CD.* 

We  shall  frequently  have  to  refer  to  practical  examples  of 
couples,  such  as  in  a  ship's  capstan,  the  screw  press  used  in  copy- 
ing manuscript,  pressing  bales  of  goods,  and  the  fly  press  for 
punching  holes  in  thin  plates,  or  for  stamping  or  embossing 
metals,  &c. 

Centre  of  Parallel  Forces,  or  Position  of  Equilibrant 
and  Resultant. — From  Experiment  III.  and  the  accompanying 
figure  to  Experiment  II.,  you  conclude  that  the  position  where 
the  equilibrant  and  resultant  act  is  such,  with  respect  to  the 
positions  where  the  forces  act,  that  the  moments  of  the  forces 
about  that  position  are  equal  and  opposite  in  effect  upon  the 
lever. 


For,  Q  x  BA  =  E  (orK) 


FA;  or,          = 

E         BA 


*  Let  P  = -8  Ibs.,  and  Q  =  10  Ibs.  :  AB  =  10  ft.  and  CD  =  8  ft. 
Then,          P  x  AB  =  Q    x  CD 
Or,  8  x   no  =:  10  x  8 

80  =  So 


CENTRE    OF    PARALLEL    FORCES. 


27 


8 

8 

i 

X 

8fl     • 
j 

or, 

= 

24 

24 

3* 

X 

FB; 

or, 

P 

E~~ 

FB 
AB 

X 

1  6"; 

or, 

16  _ 

16 

o 

24 

24 

*> 

j 

i.e.,   8  (oz.)  x   24"  =    24  (oz..) 
And  P    x  AB  =  E  (or  R) 

i.e.,  1 6  oz.    x    24"  =    24   (oz.) 

The  fulcrum  F,  where  the  equilibrant  and  resultant  act,  is 
termed  the  centre  of  the  two  parallel  forces,  and  it  is  J  of  the 
length  of  the  lever  from  one  end,  and  f  from  the  other  end. 

Reasoning  generally  trom  this  particular  case,  if  you  have 
any  two  unequal  unlike  parallel  forces,  P  and  Q,  acting  on  a 
body  in  the  directions  AP  and  BQ  respectively,  and  of  which  Q 


CENTRE  OP  PARALLEL  UNLIKE  FORCES. 

is  the  greater  force,  then  if  the  line  AB  be  drawn  perpendicular 
to  the  directions  of  these  forces,  and  prolonged,  a  single  force  E3 
parallel  to  P  and  Q  and  equal  to  Q  —  P,  will  balance  these  two 
forces  at  a  point  C,  so  thai  the  moments  about  C  are  equal  and 
opposite  ;  or, 

P  x  AC  =  Q  x  BC 

Further,  a  force  R,  equal  and  opposite  to  E,  acting  at  C,  will 
represent  the  resultant  of  P  and  Q.  This  point,  C,  is  termed  the 
centre  of  the  parallel  forces. 

The  position  of  the  point  C,  which  is  determined  by  the  above 
equation,  is  not  affected  by  the  directions  of  the  forces  so  long  as 
they  act  at  the  same  points  A  and  B,  and  have  the  same  mag- 
nitudes. 

You  may  imagine  any  number  of  parallel  forces  acting  in  one 
plane  being  replaced  by  a  single  force.  For  in  the  above  case 
you  have  formed  a  resultant,  R,  for  the  two  forces  P  and  Q  • 
consequently  you  could  find  a  resultant,  RI}  for  R  and  any  other 
parallel  force — say  S ;  and  so  on  for  any  number. 


28  LECTURE    HI. 

The  fined  resultant  of  the  whole  of  the  forces  would  act  at  a  point 
which  would  be  the  centre  of  the  system  of  the  whole  of  the  parallel 
forces  acting  on  the  body. 

Centre  of  Gravity. — Since  gravity  attracts  towards  the  earth 
each  particle  of  matter  of  which  a  body  is  composed,  the  weight 
of  a  body  may  be  considered  as  the  sum  of  a  system  of  parallel 
forces.  The  centre  of  these  parallel  forces  is  called  the  centre  of 
gravity  of  the  body,  and  is  the  point  where  the  resultant  of  the 
weights  of  all  the  particles  composing  the  body  acts. 

The  following  statements  in  small  type,  which  are  generally 
proved  as  propositions  and  corollaries  in  books  on  Elementary 
Theoretical  Mechanics,  should  be  remembered  by  the  student : — 

1.  If  a  body  is  symmetrical,  the  centre  of  gravity  (or  e.g.  of  the  body)  coin- 
cides with  the  centre  of  the  mass. 

2.  If  a  body  be  uniform,  the  c.g.  coincides  with  the  centre  of  volume. 

3.  In  a  very  thin  plate  of  uniform  density  the  c.g.  coincides  with  the 
centre  of  surface. 

4.  If  the  c.g.  of  a  body  be  determined  for  any  one  position  of  the  body, 
the  same  point  is  the  c.g.  for  every  other  position. 

5.  If  a  body  be  supported  on  its  centre  of  gravity,  the  body  will  balance 
in  any  position.     Or,  a  body  will  balance  about  its  c.g.  in  all  positions. 

6.  If  a  body  balance  in  all  positions  about  a  straight  line  through  it,  the 
c.g.  lies  in  that  line. 

7.  If  the  c.g.  be  vertically  above  or  below  the  point  of  support,  the  body 
will  rest  in  that  position.     Hence,  if  you  balance  or  support  a  body  from 
two  different  points,  the  c<g.  lies  in  the  intersection  of  the  two  vertical 
lines  from  the  two  points  respectively.     Or,  if  you  balance  a  body  on  an 
edge,  the  &jg.  is  In  the  vertical  plane  passing  through  that  edge.     Balance 
it  again  on  a  different  edge,   thus   finding  another  plane  which  passes 
through  the  c.g.      Then  the  c.g.  lies  in  the  straight  line  constituting  the 
intersection  of  the  two  planes.    Balance  the  body  for  a  third  time  in 
another  position,  then  the  point  where  this  third  vertical  plane  intersects 
with  the  straight  line  will  be  the  c.g.  of  the  body. 

8.  The  c.g.  of  regular  geometrical  bodies  may  easily  be  found  by  mere 
inspection  when  they  are  of  uniform  density. 

For  Example.— The  c.g.  of  a  line  is  at  the  middle  of  the  line  ;  of  a  circle 
at  its  centre  ;  of  a  sphere  at  its  centre ;  of  the  surface  of  a  uniform  cylinder 
and  of  a  solid  cylinder  at  the  centre  of  the  axis  ;  of  a  parallelogram  at  t>he 
intersection  of  its  diagonals  ;  of  a  triangle  at  the  intersection  of  straight 
lines  drawn  from  two  of  the  angles  to  the  middle  points  of  the  opposite 
sides — i.e.,  at  a  distance  from  one  of  the  angles  along  one  of  these  lines 
equal  to  |  of  the  line  ;  of  the  perimeter  of  a  triangle  (i.e.,  of  three  uniform 
rods  forming  a  triangle)  at  the  intersection  of  the  two  straight  lines  which 
bisect  two  of  the  angles  of  the  triangle  formed  by  joining  the  centres  of 
the  three  uniform  rods ;  of  a  polygon  at  the  point  of  application  of  the 
resultant  of  the  parallel  forces  represented  by  the  areas  of  the  respective 
triangles  into  which  the  polygon  may  be  formed,  and  where  each  of  these 
forces  is  considered  to  act  at  the  c.g.  of  its  own  triangle  ;  of  a  pyramid  at 
f  of  the  line  from  the  vertex  to  the  c.g.  of  the  base  ;  of  a  cone  at  §  of  the 
axis  from  the  vertex  ;  of  the  curved  surface  of  a  cone  at  f  of  the  axis  from 
the  vertex ;  of  a.  prism  at  the  middle  of  the  line  connecting  the  c.$r.'s  of  its 
ends. 


THE    LEVER.  2Q 

The   Lever  when  its  Weight  is  taken  into  Account. — 

In  this  case  we  have  to  add  the  moment  due  to  the  weight  of  the 
lever,  to  the  moment  of  P  or  of  W  according  as  it  acts  along 
with  the  one  force  or  with  the  other ;  i.e.,  according  as  the  e.g.  of 
the  lever  is  on  the  same  side  of  the  fulcrum  as  P  or  W.  When 
the  lever  is  of  uniform  section  and  density  throughout,  then  the 
e.g.  of  the  lever  is  at  its  middle  point,  and  consequently  the 
whole  weight  of  the  lever  may  be  considered  as  concentrated  and 
acting  at  that  point. 


A 

t 


WEIGHT  OF  LEVER  CONSIDERED. 

Let  AB  be  a  uniform  lever,  of  weight  w,  acting  at  its  e.g.  or 
middle  point  C,  let  a  weight,  IF,  be  attached  to  the  end  B,  then 
the  force  P,  which  will  have  to  be  applied  to  the  other  end  A,  in 
order  to  balance  the  whole  about  the  fulcrum  F,  will  be  found 
by  taking  moments  about  F. 

Thus,  P  x  AF  4-  w  x  CF  =  W  x  BF 

Or*  p_WxBF-wxCF 

EXAMPLE  I. — A  uniform  lever,  5  ft.  long,  of  30  Ibs.  weight,  is 
placed  on  a  fulcrum  10  in.  from  one  end,  and  has  a  weight  of 
100  Ibs.  attached  to  the  short  end.  What  force  must  be  applied, 
and  in  what  direction,  in  order  to  produce  equilibrium  ?  Also, 
what  is  the  pressure  on  the  fulcrum,  and  in  what  direction  does 
the  reaction  from  the  fulcrum  act  ? 

1 .  Referring  to  'the  above  figure,  we  find  from  the  question  that 

AB  =  5  ft.  =  60  in.;  BF=  10  in.  .*.    AF  =  50  in.  and  CF  =  20  in. 
-  W  =  100  Ibs.  and  w  =  30  Ibs. 

2.  By  the  principle  of  moments — 

The  Opposing  Moments  about  the  Fulcrum  are  equal. 

Consequently,     P  x  AF  +  w  x  CF  =  W  x  BF 

p_WxBF-tpxGF 
AF 

Substituting  the  numerical  values— 

P  =  100x10-30x20  _  8  lbg 

5° 

*  If  the  e.g.  of  the  lever  was  on  the  opposite  side  of  the  fulcrum,  i.e., 
on  the  side  of  W,  then  P  x  AF=*  W  x  BF  +  to  x  CF. 


3O  LECTURE   III. 

3.  P  acts  vertically  downwards,  since  the  moment  due  to  the 
weight  of  the  lever  is  not  sufficient  to  equalise  the  moment  due  to 
the  weight  W  about  the  point  F. 

4.  The  pressure  on  the  fulcrum  is  evidently  equal  to  the  sum  of 
all  the  forces,  since  all  the  forces  act  in  one  direction,  or  vertically 
downwards.     It  is  therefore  equal  to 

W    +   w   +    P   ^    soo    4-    30    +    8   =    138  Ibs. 

5.  The  reaction  from  the  fulcrum  is  equal  and  opposite  in  direc- 
tion to  this  resultant.     It  therefore  acts  vertically  upwards,  and 
is  the  equilibrant  of  the  whole  of  the  forces,  for  a  vertical  force  of 
138  Ibs.  applied  to  the  lever  at  F  would  counterpoise  or  just  lift 
the  whole  bar  with  the  attached  weights  P  and  W. 

EXAMPLE  II.  —  Suppose  everything  the  same  as  in  the  previous 
example  but  the  weight  of  the  lever,  which  you  may  consider  as 
now  equal  to  60  Ibs.  ;  what  force  P  would  be  required,  and  in 
what  direction  would  it  have  to  act,  in  order  to  produce  equili- 
brium ?  Also,  what  would  be  the  resultant  or  downward  pressure 
at  F. 

1.  You  observe  at  once  that  the  moment  of  the  weight  of  the 
lever  is  greater  than  the  moment  of  W  about  the  fulcrum. 

For,  w    x  OF    >   W    x    BF 

Since,  60  x     20    >    100  x    10 

Consequently  by  the  principle  of  moments  P  must  act  against 
w,  or  vertically  upwards,  so  as  to  assist  W,  in  order  that  the  oppos- 
ing moments  about  the  fulcrum  may  be  equal. 

2.  The  formula  therefore  becomes 

!0xCF-PxAF 
Or,  «>xCF 


.   u?xCF-WxBF    -D 

" 


Substituting  the  numerical  values,  we  have 

6ox20-iGoxio_p_1  lbs 

5o 

3.  The  resultant  pivssure  at  F  is  equal  to  the  algebraical  sum  of 
the  forces,  or 

W   +   w   -   P   =    100    +    60-4    =    156  lbs. 

And  acts  vertically  downwards.     The  equilibrant  would  there- 
fore be  156  lbs.  acting  on  the  lever  at  Fand  vertically  upwards. 

Position  of  the  Fulcrum.  —  In  answering  questions  which  give 
the  magnitude  of  the  forces  with  which  they  act,  and  require  only 
an  answer  for  the  position  of  the  fulcrum,  the  student  has 


THE   LEVER.  31 

simply  to  employ  the  general  formula  for  the  principle  of  mo- 
ments, and  then  to  substitute  the  known  numerical  values  in 
order  to  get  the  unknown.  Or,  he  may  reason  out  the  formula 
into  the  following  shape,  and  then  interpolate  the  numerical 
values.  Referring  to  the  last  figure,  suppose  that  the  distance 
AF  is  required  : 

Then,  neglecting  the  weight  of  the  lever,  we  have  by  the  principle 
of  moments  — 


Or,       PxAF  +  Wx  AF  =  AF(P  +  W)    =WxBA 

WxBA 
•  •  AF  =   P  +  W 

Now,  taking  the  weight  of  the  lever  into  account  ,  we  have  by  thd 
principle  of  moments  : 

PxAF  +  wxCF  =  Wx  BF. 
Or, 


r»  BA 

PxAF  +  wxAF  +  Wx  AF  =  WxBA  +  M?x— 


W 

EXAMPLE  III.  —  Where  should  the  fulcrum  be  placed  under  a 
uniform  lever  in  order  to  produce  equilibrium,  if  the  lever  is  5  ft. 
long,  weighs  30  Ibs.,  and  has  weights  of  100  and  8  Ibs.  respectively 
hung  at  its  ends. 

From  the  above  general  equation  for  equilibrium  —  viz.  : 


8  +  3Q-I-  TOO 
Which  proves  the  data  given  in  Example  I.  to  be  correct, 


32  LECTURE   III. — QUESTIONS. 


LECTURE  III. — QUESTIONS. 

1.  Define  what  is  meant  by  "  the  moment  of  a  force,*1  and  give  an  ex 
ample  with  a  sketch. 

2.  State  "  the  principle  of  moments,"  and  apply  it  to  the  case  of  a 
simple  straight  lever. 

3.  A  weight  of  10  Ibs.  on  the  end  of  a  lever  100  inches  from  the  centre 
of  motion  is  found  to  balance  a  weight  of  100  Ibs.  at  a  distance  of  10  inches. 
Explain  the  natural  law  which  governs  matter  and  motion,  upon  which  the 
above  mechanical  fact  depends.     (Answer  this  by  giving  the  definition  of  the 
principle  of  moments.) 

4.  Describe  an  experiment  to  prove  the  equality  of  the  moments  when 
the  pull  is  between  the  weight  and  the  fulcrum  and  acts  in  the  opposite 
direction  to  the  weight. 

5.  In  the  case  of  a  straight  lever,  how  would  you  ascertain  the  pressure 
on  and  the  reaction  from  the  fulcrum  ? 

6.  Three  forces,  of  12,  :o  and  2  Ibs.,  act  along  parallel  lines  on  a  rigid 
body ;  show  by  a  sketch  how  they  may  be  adjusted  so  as  to  be  in  equili- 
brium 1    Ans.  The  force  of  12  Ibs.  must  act  as  the  equilibrant  to  the  forces 
2  and  10  Ibs. — i.e.,  in  a  line  with  their  resultant,  but  in  the  opposite  direc- 
tion. 

7.  Two  parallel  forces  of  10  and  12  Ibs.  act  in  opposite  directions  on  a 
rigid  body,  and  at  2  fe^t  apart.     Where  is  the  centre  of  the  two  forces, 
and  what  is  their  resultant  1    Ans.  10  feet  from  the  force  of  12  Ibs.,  2  Ibs. 

8.  Define  the  "  centre  of  gravity  "  of  a  body,  and  show  how  you  would 
find  it  experimentally  in  the  case  of  any  irregular  body.     Give  an  example. 

9.  State  the  rule  which  applies  when  two  unequal  forces  balance  on 
opposite  sides  of  the  fulcrum  of  a  straight  lever,  the  weight  of  the  lever 
being  neglected.     A  uniform  straight  lever,  4  feet  long,  weighs  10  Ibs.,  the 
fulcrum  is  at  one  end ;  find  what  upward  force  acting  at  the  other  end 
will  keep  the  lever  horizontal  when  a  weight  of  10  Ibs.  is  hung  at  a  dis- 
tance of  i  foot  from  the  fulcrum.     Find  also  the  pressure  on  the  fulcrum 
and  the  direction  in  which  it  acts.     Ans.  7'5  Ibs.  ;  12-5  Ibs.  downwards. 

10.  A  uniform  bar,  4  feet  long  and  weighing  4  Ibs.,  can  turn  about  a 
fulcrum  at  one  end,  and  a  weight  of  10  Ibs.  is  hung  upon  the  bar  at  a  dis- 
tance of  i  foot  from  the  fulcrum.     Find  the  upward  force  at  the  free  end 
which  will  keep  the  bar  horizontal.     Ans.  4-5  Ibs. 

11.  A  uniform  bar  of  metal  10  inches  long  weighs  4  Ibs.,  and  a  weight 
of  6  Ibs.  is  hung  from  one  end.     Find  the  fulcrum  or  point  upon  which  the 
bar  will  balance.     Ans.  2  inches  from  the  6  Ibs.  weight. 

12.  Two  parallel  forces  whose  magnitudes  are  8  and  12  Ibs.  respectively, 
act  in  the  same  direction  on  a  rigid  body  at  points  10  inches  apart.     Find 
the  magnitude  and  line  of  action  of  the  resultant  of  the  two  forces. 

Ans.  20  Ibs.  at  a  point  6  inches  from  the  force  of  8  Ibs. 

13.  A  uniform  lever  is  5  feet  long,  and  weighs  10  Ibs.,  the  fulcrum  being 
at  one  end.     A  weight  of  30  Ibs.  is  hung  at  a  distance  of  4  feet  from  the 
fulcrum  ;  what  upward  force  acting  at  the  middle  point  of  the  lever  will 
keep  it  in  a  horizontal  position  1    Ans.  58  Ibs. 

14.  Define  "  moment  of  a  force."     How  is  it  measured  ?     A  bar  of  metal 
of  uniform  section  weighs  5  Ibs.,  and  a  weight  of  10  Ibs.  hangs  from  one 
end.     It  is  found  that  the  bar  balances  on  a  knife  edge  at  9  inches  from 
the  end  at  which  the  weight  hangs ;  what  is  the  length  of  the  bar  I 

4  ft.  6  in. 


LECTDBE  HI. — QUESTIONS.  JJ 

15.  State  the  principle  of  the  lever,  and  prove  it  when  F  and  W  act  on 
opposite  aides  of  the  fulcrum.     A  weight  of  5  Ibs.  is  hang  at  one  end  of 
a  uniform  bar,  which  is  balanced  over  a  knife  edge  at  a  point  14  inches 
from  the  end  at  which  the  weight  hangs.     The  bar  weighs  30  Ibs. ;  find  its 
length.     Ant.  32$  inches. 

16.  State  the  principle  of  the  lever.    A  uniform  straight  bar,  14  inches 
long,  weighs  4  Ibs. ;  it  is  used  as  a  lever,  and  an  8  Ib.  weight  is  suspended 
at  one  end.     Find  the  position  of  the  fulcrum  when  there  is  equilibrium. 
Ant.  2&  inches  from  the  8  Ib.  weight. 


34  NOTES    AND    QUESTIONS. 


(     35     ) 


LECTURE   IV. 

CONTENTS. — Practical  Applications  of  the  Lever — The  Steelyard,  or  Roman 
Balance— Graduation  of  the  Steelyard — The  Lever  Safety  Valve — 
Example  I. — Lever  Machine  for  Testing  Tensile  Strength  of  Materials 
— Straight  Levers  acted  on  by  Inclined  Forces — Bent  Levers— The 
Bell  Crank  Lever — Bent  Lever  Balance — Duplex  Bent  Lever,  or  Lum- 
berer's Tongs  — Turkus,  or  Pincers — Examples  II.  and  111. — Toggle 
Joints— Questions. 

IN    this  Lecture  we  shall   give  a  number  of  examples   of   the 
application  of  the  lever. 


STEELYARD,  OB  ROMAN  BALANCE. 
INDEX  TO  PARTS. 


F  represents  Fulcrum. 
GA          „          Graduated  arm. 
SW          „          Sliding  weight. 

P          „          Pull  due  to  SW 
SP          „          Scale  pan. 


W  represents  Weight  in  SP. 
AF          „          Distance  of  P 

from  F. 

BF          „          Distance  of  W 
fromF. 


36  LECTURE   IV« 

The  Steelyard,  or  Roman  Balance,  is  a  straight  lever  with 
unequal  arms,  having  a  movable  or  sliding  weight  on  the  longer 
arm.  It  is  very  much  used  by  butchers  for  weighing  the  carcasses 
of  cattle  and  sheep,  and  in  such  cases  it  generally  has  two  fulcra 
and  two  scales  of  division  corresponding  to  them,  the  one  set 
being,  say,  for  hundredweights  and  the  other  for  pounds. 

Graduation  of  the  Steelyard.  —  The  practical  method  of  gradu- 
ating the  steelyard  is  to  put  unit  weight  (say  i  Ib.)  into  the  scale 
pan,  SP  (or  attach  it  to  the  hook  on  the  shorter  arm  if  there 
should  be  no  such  pan),  and  mark  the  position  where  the  sliding 
weight,  SW,  has  to  be  placed  in  order  to  cause  equilibrum.  Mark 
this  position  i  on  the  scale.  Then  put  in  two  units  (say  2  Ibs.) 
into  SP,  and  adjust  SW  as  before,  marking  its  new  position  as 
2  on  the  scale  ;  and  so  on  until  SW  is  at  the  end  of  the  longer 
arm. 

In  tnis  form  of  steelyard,  if  the  differences  of  the  weights  W, 
corresponding  to  successive  distances,  i  to  2,  to  3,  <fcc.,  be  the  same, 
the  graduations  will  be  equal  to  each  other.  This  may  be  proved 
in  the  following  manner  :  —  First  of  all,  it  is  clear  that  the  instru- 
ment can  be  so  constructed  that  the  centre  of  gravity  of  the  beam 
and  scale  pan  may  occupy  one  or  other  of  three  different  positions. 
The  centre  of  gravity  may  coincide  with  F,  or  it  may  be  on  the 
longer  arm,  or  it  may  be  on  the  shorter  arm. 

Suppose  the  centre  of  gravity  to  coincide  with  F,  the  fulcrum. 

Let  the  scale  pan  be  loaded  to  the  extent  of  W  units,  and 
suppose  that  the  sliding  weight  of  P  units  has  to  be  placed  at  A 
in  order  to  keep  the  beam  horizontal. 

B          F    .    .    ,  A  At  t        r    .    .   A« 


8   9  1    11  12 


A  1    2   3    4    5    tf  7 
W  P 

STEELYARD  WITH  THE  CENTRE  OP  GRAVITY  COINCIDING 
WITH  THE  FULCRUM. 

Then,  PxFA  =  WxFB          .         .         (i) 

Increase  W  by  one  unit,  and  to  restore  equilibrium,  let  P  be 
placed  at  Ar     Then,  for  equilibrium  we  must  have 

PxFA1==(W+i)xFB         .         (2) 

Subtracting  corresponding  members  of  equations  (i)  and  (2) 
we  get  P(FAt  -  FA)  =  ( W  +  i  -  W)  x  FB, 

Or,  P  x  AAj  =  FB, 

FB 
=  ^p-         •        •         •         (3) 


THE  LEVER.  J7 

Increase  W  by  n  units,  and  let  P  occupy  the  position  An. 
Then,  for  equilibrium,  we  must  have 

PxFAra  =  (W  +  n)xFB       .         (4) 

As  before,  subtract  the  corresponding  members  of  (i)  and  (4), 
when  we  get  P  x  A  A*  =  n  x  FB, 

FB 


Or,  AAn  =  n  x  A At  by  equation  (3) 

Thus  we  see,  that  the  graduations  are  all  equal  for  equal  incre- 
ments of  W. 

The  student  will  readily  observe  that  the  zero  of  the  scale  is  at 
F,  and  by  putting  W=  i  in  equation  (i)  we  can  fix  the  position 
of  the  first  number  on  the  scale 

i.e.t  P  x  Fi  =  i  x  FB, 

Or,  Fi=™. 

Nexti  suppose  the  centre  of  gravity  to  lie  in  the  longer  arm  at  G. 

B     |    .F  G  .       A  Ai   t  _  |    ,    l    l  A* 

|      /   ^>3\  4    5    6    7    ]s   9  10  11  12  13  14  15 
I  j 

W  P 

STEELYARD  WITH  THE  CENTRE  OF  GRAVITY  IN  THE' 
LONGER  ARM  AT  G. 

Let  w  =  weight  of  beam  and  scale  pan,  and  suppose  P  at  A 
and  to  at  G  to  balance  W  units  at  B.  Then,  for  equilibrium, 
we  have 

P  x  FA +  10  x  FG  =  W  x  FB  .         .        (5) 

As  before,  increase  W  by  one  unit,  and  let  P  be  shifted  to  A, 
in  order  to  restore  equilibrium,  then  we  must  have 

Subtracting  (5)  from  (6)  we  get 

PxAA1  =  FB, 

_FB 
Or,  AAt  —  p      .         .         ,         /  \ 

Now  increase  W  by  n  units,  and  let  P  occupy  the  position  A*, 
then 

P  x  FA,.  +  w  x  FG  =  (W  +  n)  x  FB       .         (8) 
Subtracting  (5)  from  (8)  we  get 

PxAAM  =  nxFB, 

FB 
Or,  AA,.  =  n  x  -p-, 

That  is,  AA,,  =  n  x  AA,  by  equation  (7) 


38  LECTURE  IV. 

Thus  we  again  see  that  the  graduations  are   equal  for  equal 
increments  of  the  weight  W. 

To  find  the  zero  of  the  scale  in  this  case  : 
In  equation  (5)  put  W  =  0,  then 
PxFO 


That  is,  the  zero  is  in  the  shorter  arm  at  a  distance  from  F, 
represented  by  -Jp  —  units  of  distance,  the  units  in  this  case 

being  the  same  as  those  measuring  FG. 

By  making  W=  i  in  equation  (5)  the  position  of  the  first  figure 
on  the  scale  can  be  fixed,  and  then  the  whole  beam  graduated, 
since  all  the  divisions  are  of  the  same  size. 

One  important  point  to  be  observed  in  this  arrangement  is, 
that  when  the  centre  of  gravity  lies  in  the  longer  arm,  there  is  a 
limit  to  the  smallness  of  the  weight  which  can  be  weighed  in  the 
scale  pan,  since  the  sliding  weight  moves  along  the  longer  arm 
only. 

Let  P  coincide  with  F,  then  the  weight  W  which  must  be 
placed  in  the  scale  pan  in  order  to  just  balance  the  weight  of  the 
beam  and  scale  pan  at  G  is 


. 

Any  weight  less  than  this  cannot  be  weighed.  This  is  not  an 
objection  to  the  instrument  where  the  weights  to  be  measured 
are  great,  as  in  the  case  of  the  butcher's  steelyard  used  for 
weighing  heavy  carcasses. 

When  the  centre  of  gravity  lies  in  the  shorter  arm  the  graduations 
will  still  be  equal.  The  reasoning  is  the  same  as  in  the  last  two 
cases.  The  student  can  also  prove  that  the  zero  of  the  scale  is 
on  the  longer  arm  at  the  point  0  given  by  the  equation 


In  this  case  all  weights,  however  small,  can  be  weighed. 

The  Lever  Safety  Valve.*  —  The  lever  safety  valve  is  a  simple 

*  For  a  more  detailed  description  of  safety  valves  and  their  action 
refer  to  Lecture  XXVII.  of  the  author's  Elementary  Manual  on  •'  Steam 
and  the  Steam  Engine." 


THE    LEVER    SAFETY    VALVE. 


39 


contrivance  fixed  on  the  top  of  a  boiler  for  the  purpose  of  auto- 
matically preventing  the  steam  exceeding  an  agreed-upon  working 
pressure. 

Referring  to  the  next  figure,  YC  is  a  cast-iron  valve  chest, 
containing  a  tightly-fitted  gun-metal  valve  seat,  VS,  on  which 
rests  a  steam-tight  gun-metal  valve,  V.  On  the  centre  of  the 
upper  side  of  this  valve  rests  a  conical  steel  pin  attached  to  a 
straight  lever  by  an  eye  and  bolt.  One  end  of  this  lever  is  free 
to  turn  en  a  fulcrum  fixed  to  the  upper  flange  of  the  valve  chest, 
and  a  lock-fast  cast-iron  weight  is  placed  near  the  other  end,  so 


LOCKFAST  LEVEB  SAFETY  VALVE. 
INDEX  TO  PARTS. 


VC  represents  Valve  chest. 


VS 


Valve  seat. 


V  represents  Valve. 

P  «i         Locking  pin. 


that  the  downward  moment  of  the  weight  about  the  fulcrum 
balances  the  upward  moment  of  the  steam  pressure  on  the  valve 
about  the  same  fulcrum. 

Let  L     =  length  of  lever  in  inches  from  fulcrum  to  the  e.g.  of 

the  weight,  W. 
F     =  Distance  in  inches  from  fulcrum  to  centre  line  of 

valve,  V. 

^     =          „  „  „  to  e.g.  of  the  lever. 

W     =  "Weight  in  Ibs.  of  the  cast-iron  counterpoise  block. 
w/   =        „  „  lever. 

w*   =        „  „  valve. 

P     =:  Pressure  of  steam  in  Ibs.  per  square  inch* 


4<3  LECTURE    IV, 

d  =  Diameter  of  valve  in  inches. 
A  ='Area  of  valve  in  square  inches  =  -d2. 
P  x  A  =  Total  pressure  in  Ibs.  on  the  valve. 

Then,  by  taking  moments  about  the  fulcrum,  we  find  the  pressure 
of  steam  per  square  inch  which  will  balance  the  several  forces. 

for  the  upward  moment  =  the  sum  of  the  downward  moments. 

(P  x  A-  W,)F       =  (W  x  L)  +  (W,  x  G). 
Or,  (PxA)xF    =  (WxL)  +  (W,xG)  +  (W.xF) 


AxE 

If  we  neglect  the  weight  of  the  lever  and  the  valve  — 
Then,  (PxA)xF  =  WxL 

»-?s 

EXAMPLE  I.  —  A  valve,  3  inches  in  diameter,  is  held  down  by  a 
lever  and  weight,  the  length  of  the  lever  being  10  inches,  and  the 
valve  spindle  being  3  inches  from  the  fulcrum.  You  are  to  dis- 
regard the  weight  of  the  lever  and  to  find  the  pressure  per  square 
inch  which  will  lift  the  valve  when  the  weight  hung  at  the  end 
of  the  lever  is  25  Ibs. 


-10 


W  =25  Ibs. 

Referring  to  the  previous  figure  as  well  as  to  the  accompanying 
one,  we  see  from  the  question  that 

<*  =  3"  •'•  A  =  ^2  =  .7854x3x3  =  7.07  sq.  ins. ; 

BF=  3',  AF=  10"  and  W  =  25  Ibs. 
Taking  moments  about  F,  we  get — 

(PxA)xBF  =  Wx  AF 

P  x  7.07  x      3  =  25  x  10 

P  =  H'8  Ibs.  per  square  inch. 

Testing  Machine. — The  following  figures  illustrate  a  machine 
which  is  used  for  testing  the  tensile  strength  of  iron,  steel  and 
such  like  materials.  It  consists  of  a  combination  of  levers.  After 


LEVER  TESTING    MACHINE. 


>HW 


LEVEB  MACHINE  FOR  TESTING  TENSILE  STRENGTH  OF  MATERIALS. 


A, 


L, 


7 

MW 


f 

W 


n 

R       BWa 


A2 


DIAGRAM  OF  THE  LEVERS. 


INDEX  TO  PARTS. 


LjL,  represent  Levers. 
F,F2         „          Fulcra. 

P        „          Pull,   or  dead 
weights. 


A 


i 


Where  P  acts  on  L,. 
Specimen  under  te.->t. 


BpA,,  represents  Where  R  acts  on 

L,,L2. 

B.,         „         Where  W  acts  on  S. 
M\V          „          Movable  weight. 
BW  Balance  weight. 

HW         „          Hand-wheel    and 
screw  for  elevat- 
ing Flf  &c. 


2  LECTURE    IV. 

mastering  the  general  arrangement  of  the  machine  by  comparing 
the  index  to  parts  with  the  side  elevation,  the  student  should 
refer  to  the  accompanying  skeleton  diagram  (where  the  same  index 
letters  have  been  used),  from  which  he  will  readily  understand  how 
the  stresses  are  transmitted  and  magnified. 

Looking  at  the  second  of  the  above  figures,  or  skeleton  diagram 
of  the  levers,  it  will  be  seen  that  when  equilibrium  exists  between 
the  stress  W  on  the  specimen  S,  and  the  pull  P,  applied  at  Av 


P  x  A^\  =  E  x  B^j,  and  R  x  A2F2  =  W  x  B3F2 
PxA,F,     Wx 

*i*\          ~  A 

P  x  A1F1  x  A2F8 
-L 


PxA,F,     WxB,F2 

~ 


.  .      _ 
Consequently,  W= 

Straight  Levers  Acted  on  b3  Inclined  Forces.  —  In  the 
previous  Examples  and  in  Leeeure  III.  we  have  considered  the 
forces  P  and  W  as  acting  at  right  angles  to  the  straight  levers. 
In  such  cases  the  forces  had  the  greatest  advantage,  or  their 
turning  moments  were  a  maximum.  But  the  principle  of  moments 
is  equally  applicable  to  inclined  forces  acting  on  straight  levers  and 
to  bent  levers. 

Xs 
/     \ 


W 

STRAIGHT  LEVEES  WITH  INCLINED  FORCES. 

For,  let  AjBj  be  a  straight  lever  acted  on  by  inclined  forces, 
P  and  W.  "Draw  from  the  fulcrum,  F,  lines  at  right  angles  to 
the  produced  directions  of  the  forces  as  shown  by  the  dotted  lines 
in  the  above  figure. 

Then,  the  effective  arms  for  the  forces  P  and  W  are  respectively 
A8F  and  B,F  ;  and  equilibrium  takes  place  when  their  momenta 
about  F  are  equal  ; 

i.e.,  when  P  x  A,F  =  W  x  B  F 

Or,  P:  W::B,F:  A,F. 

Bent  Levers.  —  The  BeU  Crank  Lever.  —  The  same  principle 
and  action  hold  good  in  the  case  of  bent  levers.  Take  an  ordi- 


BENT    LEVERS. 


43 


nary  right-angle  bell  crank  lever,  as  shown  by  the  accompanying 
figure.  Here  the  effective  arms  are  equal  to  the  actual  arms  of 
the  lever,  because  the  forces  have  been  shown  as  acting  at  right 
angles  to  their  respective  arms,  or  with  maximum  turning 
moments. 

Therefore,  P  x   AF  =  W  x  BF. 

But,  if  the  lever  be  turned  round  through  any  angle  by,  say,  an 
extra  pull  at  P,  then,  in  order  to  ascertain  the  virtual  moments 
we  should  have  to  draw  lines  at  right  angles  from  F  on  the 
directions  of  P  and  W  in  order  to  calculate  their  effective 
arms. 


S.P. 


w 


BELL  CRANK  LEVER 


BENT  LEVER  BALANCE. 


Bent  Lever  Balance. — Examine  an  ordinary  bent  lever 
balance,  such  as  is  frequently  used  for  weighing  letters  and  light 
parcels,  where  the  force  P  is  a  constant  quantity,  and  the  variable 
force  W  is  represented  by  the  article  to  be  weighed.  As  showu 
by  the  accompanying  figure,  the  effective  arms  change  with  each 
weight  to  be  ascertained,  and  consequently  the  scale  S  of  this 
balance  has  to  be  graduated  by  trial,  or  by  introducing  standard 
pounds,  such  as  SP,  or  other  units,  and  marking  the  values  on 
the  scale  opposite  the  position  where  the  end  of  the  pointer  on 
P  comes  to  rest.  Or,  the  graduation  might  be  done  by  plotting 
the  various  positions  of  the  arms  and  values  of  the  forces  to  scale, 
In  the  illustration  we  have  evidently  got  equilibrium  when 

P  x  AF  =  W  x  BF. 

Duplex  Bent  Lever,  or  Lumberer's  Tongs. — The  accom- 
panying illustration  shows  a  very  useful  and  simple  application  of 
the  bent  lever,  which  is  used  at  the  end  of  a  winch  or  crane  chain, 


44 


LECTURE   IV. 


for  affixing  to  and  holding  fast  stones,  logs  of  wood,  blocks  of  ice, 
or  other  heavy  articles  when  they  have  to  be  lifted. 

P,  P  indicate  the  directions  of  the  pulling  forces  on  the  short 
chains  between  the  ends  of  the  shorter  arms  and  the  common  link 
which  is  attached  to  the  crane  chain.  F  is  the  common  fulcrum, 
and  W,  W  show  the  directions  of  the  forces  with  which  the  article 
is  gripped.  The  student  will  be  able  to  draw  a  diagram  of  the 
forces  and  calculate  their  effective  moments  for  himself  for  any 
particular  case. 

The  Turkus,  or  Pincers.— The  ordinary  carpenter's  turkus, 
or  pincers,  which  is  frequently  used  for  extracting  nails  from  wood, 
is  another  familiar  illustration  of  the  duplex  bent  lever.  As 
shown  by  the  accompanying  figure,  the  forces  P^Pj,  represent  the 
forces  with  which  the  pincers  is  gripped  by  the  hand  after  the 
jaws  have  been  closed  on  the  neck  of  the  nail,  and  the  force  B 


TONGS  OE  DUPLEX  BENT  LEVEE. 


TUKKUS,  OE  PlNCEES. 


the  pressure  which  has  to  be  exerted  by  the  arm  and  body  in 
order  to  extract  the  nail  from  the  wood — i.e.,  to  overcome  the 
frictional  resistance,  W,  between  the  wood  and  the  nail.  As 
shown  by  the  separate  diagram  of  forces  in  dotted  lines,  straight 
lines  have  been  drawn,  not  from  the  joint  of  the  pincers,  but 
from  a  position  representing  the  fulcrum  F  (or  point  where  the 
nose  of  the  pincers  rests  on  the  wood),  perpendicular  to  the  direc- 
tions of  the  forces  P  and  W,  in  order  to  obtain  the  lengths  AF 
and  BF  of  the  effective  arms  of  the  bent  lever. 

Here  again,    P   x  AF  =  W   x  BF. 

EXAMPLE  II. — The  handle  of  a  claw-hammer  is  15  inches 
long,  and  the  claw  is  3  inches  long.  What  resistance  of  a  nail 
would  be  overcome  by  the  application  of  a  pressure  of  50  Ibs4  at 
the  end  of  the  handle  ? 


BENT    LEVERS.  45 

You  are  required  to  show,  by  a  diagram,  the  manner  in  which 
you  arrive  at  your  result.  (S.  and  A.  Exam.  1892.) 

ANSWER. — Here  we  have  a  simple  case  of  a  bent  lever,  with 
fulcrum  at  F,  and  effective  arms,  AF,  BF,  1 5  and  3  inches  long 


w 


P-50  LBS. 


EXAMPLE  OP  A  BENT  LEVER. 

respectively.     Let  W  represent  the  resistance  in  Ibs.  offered  by 
the  nail  at  B.     Then,  by  taking  moments  about  F,  we  get 

W  x  BF  =   P  x  AF 

Or,  W   x    3      =  50  x    15 

...  W    =  5°   x   I5=  250 Ibs. 
3 

EXAMPLE  III. — State  the  mechanical  law  known  as  the  Principle 
of  the  Lever.  In  a  pair  of  pincers  the  jaws  meet  at  i  J  inches  from 
the  pin  forming  the  joint.  The  handles  are  grasped  with  a  force 
of  50  Ibs.  on  each  handle  at  a  distance  of  8  inches  from  the  pin. 
Find  the  compressive  force  on  an  object  held  between  the  jaws, 
and  also  the  pressure  upon  the  pin.  (S.  and  A.  Exam.  1888.) 

Let  P  denote  the  force  of  50  Ibs.  with  which  the  handles  are 
grasped  at  a  distance  of  8  inches  from  F,  the  pin.  Let  W  denote 


P7=50/6s. 


R=316-6lb*.  |P,-50/to. 

PINCERS  OR  NIPPERS. 

the  compressive  force  on  the  object  O,  and  R  the  resultant  reaction 
or  pressure  on  the  pin  or  fulcrum  F.  Although  there  are  two 
levers  here,  each  having  a  common  fulcrum,  F,  it  is  best  to  con- 

NoU.—  It  is  a  mistake  to  speak  of  the  "  Principle  of  the  Lever  "  ;  what 
is  evidently  meant  is  the  Principle  of  Moments  as  applied  to  the  lever. 


4<  LECTURE    IV. 

sider  the  action  of  one  lever  only.  Suppose  the  lower  handle,  H,, 
to  be  fixed,  and  consider  the  action  of  the  upper  handle,  Ht.  It 
then  becomes  a  simple  question  on  the  lever. 

(1)  To  find  W,  take  moments  round  F,  then 

W  x   ij"  =  50  x   8" 
.•.  W  =  266-6  Ibs. 

(2)  To  find  R,  the  pressure  on  the  pin  F,  take  moments  round  O 
then 

R  x   ij"  =  50  x  (ij  +  8")  -  50  x   9J" 

,'.  R  =  316  8  Ibs. 

Or,  since  R  must  be  the  resultant  of  3?  and  W,  we  get 
R  =  P  +  W  =  $o  +  266-6  =  316-6  Ibs. 

The  M  Toggle,"  or  "  Knuckle  Joint,"  consists  of  a  well- 
known  combination  of  levers.  It  is  characterised  by  its  capability 
of  exerting  an  enormous  force  through  a  short  distance  by  means 
of  very  compact  and  simple  elements.  This  device  has  been 
applied  in  many  well-known  cases,  such  as  in  the  stone-crushing 
machine,  certain  brakes,  printing  and  several  forms  of  packing 
presses,  and  in  the  familiar  frame  by  which  carriage  hoods  are 
held  in  position. 

The  accompanying  diagrams  illustrate  the  principle  of  the 
"  Toggle,"  and  the  plan  of  its  application  to  a  cane  mill. 

In  Fig.  i,  A  A'  are  two  links  jointed  at  B,  G,  and  D.  The 
point  G  is  fixed,  whilst  the  point  7?  is  free  to  move  vertically  along 
the  line  E  C.  The  centre  D  is  connected  with  the  point  F  by 
means  of  a  third  link,  formed  partly  of  a  spring  S.  The  point  F 
is  supposed  to  be  fixed  in  relation  to  its  distance  from  the  vertical 
line  E  C,  but  is  free  to  move  in  a  line  parallel  to  E  C.  Under 
these  conditions,  a  force  acting  in  the  direction  of  the  arrow  along 
the  line  G  H  will  resist  a  much  greater  force  acting  along  the 
line  E  (7,  and  tending  to  move  the  point  B  upwards.  The  force 
along  G  H  acts  with  a  leverage  equal  to  the  distance  from  J  to 
C,  whilst  the  force  along  B  D  has  only  a  leverage  equal  to  the 
short  length  K  C.  The  nearer  the  centre  D  approaches  the 
vertical  line,  the  greater  will  be  the  leverage  of  the  force  acting 
on  the  centre  D  in  the  direction  G  H,  and  the  less  will  be  the 
leverage  of  the  force  acting  in  the  direction  E  G  through  the 
link  A'.  In  practice,  a  few  railway  buffer  springs,  corresponding 
to  S  in  Fig.  i,  combined  with  simple  links  in  a  compact  arrange- 
ment, occupying  little  space,  are  sufficient  to  resist  forces  amount- 
ing to  several  hundreds  of  tons.  The  line  E  G  corresponds  to 
Wie  centre  line  of  one  of  the  top  cover  bolts  of  a  cane  mill,  G 


"THE    TOGGLE,"   OR  "KNUCKLE    JOIflT." 


47 


corresponding  to  the  nut  at  the  top  of  the  bolt,  and  therefore 
one  of  the  fixed  points  in  the  system ;  whilst  B  may  be  taken  as 
representing  a  point  in  the  top  cover  itself,  which  is  supposed  to 
be  free  to  move  up  and  down. 

Fig.    2    illustrates  the  operation   of  the  system,   and    directs 
attention  to  a  feature  of  great  practical  interest  and  value.     The 

FIG.  i.  FIG.  2. 


Iti— 


y    4     ^MOCGr 


E 


THE  PEINCIPLB  OP  THE  TOGGLE  JOINT. 

parts  in  this  figure  correspond  with  those  in  Fig.  i,  but  it  is 
supposed  that  a  force  acting  in  the  direction  E  C  has  raised  the 
point  B  from  its  original  position,  as  in  Fig.  i,  to  the  position  B'. 
The  point  D  has  consequently  been  moved  further  away  from 
the  vertical  line  EC  to  the  position  D'.  In  this  position,  the 
leverage  of  the  force  acting  along  G  H  is  reduced,  whilst  the 
leverage  of  the  force  acting  through  the  link  A'  is  increased, 
as  compared  with  the  previous  conditions  in  Fig.  i  ;  but  it  will 
be  seen  that  Z>.  in  moving  to  the  position  />',  has  compressed  the 
spring  S,  and  therefore  increased  its  resistance. 
Hence,  by  the  principle  of  moments  : 

(Force  along  B  D)  x  K  C  =  (  Force  along  G  F)  x  J  0. 
/.  Force  along  B  D  =  (Force  along  G  F) 


NOTE.—  See  the  Tangentometer  at  end  of  Lecture  XXVIII. 


LECTURE    IV. 


T>    J 

But  the  vertical  force  E  B  =  (Force  aloog  B  D)  x  ^-^. 

J  C1       R  T 
„  „  .        „       „     =  (Force  along  G  F)  x  ^-^  x  g-j-j. 

But  if  the  links  B  D  and  D  C  are  equal,  then  B  J  =  J  C. 

J  Q  2 

Hence,  force  along  E  B  =  (force  along  G  F)  x  7^—^ — TT^. 


If  the  movement  along  B  C  is  very  small  compared  with  the 
length  of  the  links  B  D  and  D  C,  then  J  C  may  be  considered 
as  a  constant  length.  Hence,  when  the  spring  is  so  adjusted  that 

its  resistance  to  compression 
is  proportional  to  K  C,  the 
pressure  E  B  upon  the  rollers 
will  be  nearly  constant.  This 
arrangement  of  the  "  toggle 
joint,"  therefore,  automati- 
cally permits  of  light  or  heavy 
feeding  of  the  canes  in  a  sugar 
mill  without  bringing  undue 
stresses  upon  the  various 
parts,  and  thus  diminishes 
the  chance  of  breakdowns. 

The  following  illustration 
shows  the  actual  construction 
of  the  "  Toggles  "  as  applied 
to  the  top  roller  of  an  ordinary 
3-roller  cane-crushing  mill. 
The  left-hand  half  of  the 
illustration  shows  the  posi- 
tions of  the  parts  when  the 
mill  is  empty.  In  this  con- 
dition the  top  roller  rests  in  its 
bearings  and  gix'es  the  mini- 
mum openings  between  the 
rollers.  The  right-hand  half 
of  the  illustration  shows  the 
condition  of  the  system  when 

PATENT  TOGGLE  APPARATUS.          the  mill  is  taking   a   heavy 

feed.     It  will    be    seen  flint 
as  the  roller  rises  it  lifts  the  top  caps,  which  are  under  the  contiol 


THE    TOGGLE,"   OR    "KNUCKLE    JOINT.'' 


49 


of  the  "  Toggles,"  until  the  upward  pressure  is  balanced  by  the 
resistance  of  the  "  Toggles." 


v. 


TOGGLE  JOINT  AS  APPLIED  TO  A  SUGAR-CANE  MILL. 

The  above  is  an  illustration  of  a  mill  made  by  the  Messrs. 
Mirrlees,  Watson  &  Co.,  Ltd.,  of  Glasgow,  with  rollers  32  inches 
in  diameter  by  60  inches  long.  The  arrangement  of  their  patent 
"  Toggles "  in  this  case  is  of  a  special  kind.  The  vertical  bolts 
of  the  "  Toggles  "  are  formed  by  continuations  of  the  top  cover 
bolts,  but  the  mill  covers  or  caps  are  fixed  by  nuts  screwed  directly 
down  upon  them,  and  do  not  lift  with  the  roller.  The  communi- 
cation between  the  "  Toggles  "  and  the  top  roller  is  made  by  means 
of  plungers  formed  on  the  under  side  of  the  bottom  plates  of  the 
"  Toggles,"  and  working  through  the  top  caps.  The  only  advantage 
of  this  arrangement  is,  that  it-  permits  the  top  cap  to  be  used  as 
a  brace  to  bind  the  upper  jaws  of  the  mill  cheek  together,  and 
thus  adds  in  some  measure  to  the  strength  of  the  cheek. 


5O  LECTUEB    IV. — QUESTIONS. 


LECTUBE  IV.— QUESTIONS. 

1.  Sketch  and  describe  the  steelyard,  or  Roman  balance,  and  explain 
fully  how  the  graduations  on  the  scale  are  equal  for  equal  differences  in 
the  weights  applied  to  the  shorter  arm. 

2.  Sketch   and   describe    a    lockfast   lever   safety  valve.      A  valve,  3 
inches  in  diameter,  is  held  down  by  a  lever  and  weight,  the  length  of  the 
lever  being  30  inches,   and  the  valve  spindle   being   4  inches  from  tho 
fulcrum.     You  are  to  disregard  the  weight  of  the  lever  and  to  find  the 
pressure  per  square  inch  which  will  lift  the  valve  when  the  weight  hung 
at  the  end  of  the  lever  is  56  Ibs.    Ana.  59*4  Ibs. 

3.  The  diameter  of  a  safety  valve  is  3",  its  weight  3^  Ibs.  ;  length  of 
lever  is  30",  and  its  weight  16  Ibs.  ;  the  distance  from  fulcrum  to  centre 
of  valve  is  3",  and  to  e.g.  of  lever  12".      Find  where  a  weight  of  50  Ibs. 
must  be  placed  on  the  lever  in  order  that  steam  may  just  blow  off  at  70  Ibs. 
per  square  inch  by  gauge.     Ans.  25-65  inches  from  the  fulcrum. 

4.  The  safety  valve  of  a  boiler  is  required  to  blow  off  steam  at  100  Ibs. 
per  square  inch  by  gauge.     The  dead  weight  is  100  Ibs.,  weight  of  lever 
10  Ibs.,  and  of  valve  5  Ibs.  ;  diameter  of  valve  3^",  distance  from  centre  of 
valve  to  fulcrum  4",  from  e.g.  of  lever  to  fulcrum  15".     Where  should  you 
place  the  weight  on  the  lever?    Ans.  36-9  ins.  from  fulcrum. 

5.  Sketch  and  describe  a  lever  machine  for  testing  the  tensile  strength 
of  materials.     If  the  advantage,  or  ratio  of  resistance  R  to  pull  P  in  the  first 
lever,  is  56  to  I,  and  of  the  second  lever  40  to  I,  what  stress  will  be  produced 
on  the  test  specimen  when  P=  100  Ibs.  1    Ans.  100  tons. 

6.  A  force  of  100  Ibs.  acts  at  one  end  of  a  straight  lever,  but  at  an  angle 
of  60°  to  it.     What  force  acting  at  the  other  end  of  the  lever,  at  an  angle  of 
45°  to  it,  will  keep  the  lever  in  equilibrium  if  the  fulcrum  be  placed  half 
the  distance  from  the  first  force  that  it  is  from  the  second  ?    Draw  a  dia- 
gram of  the  forces  and  their  effective  arms.     Ans.  61.25  lbs- 

7.  Sketch  a  bell  crank  lever,  to  convey  a  small  movement  from  one 
line  to  another,  cutting  each  other  at  60°  ;  the  distances  moved  through 
to  be  as  i  to  2. 

8.  The  handle  of  a  claw-hammer  is  12  inches  long,  and  the  claw  is  a 
inches  long.     What  resistance  of  a  nail  would  be  overcome  by  the  appli- 
cation of  a  pressure  of  40  lbs.  at  the  end  of  the  handle  ?  Show,  by  a  diagram, 
the  manner  in  which  you  arrive  at  your  result.     Ans.  240  lbs. 

9.  In  a  pair  of  pincers  the  jaws  meet  at  i^  inches  from  the  pin  forming 
the  joint.     The  handles  are  grasped  with  a  force  of  30  lbs.  on  each  handle 
at  a  distance  of  7^  inches  from  the  pin.     Find  the  compressive  force  on 
an  object  held  between  the  jaws,  and  also  the  pressure  upon  the  pin. 
Sketch  the  apparatus  and  show  the  direction  and  values  of  all  the  forces. 
Ans.  1 80  lbs  ;  210  lbs. 

10.  There  is  a  contrivance  for  obtaining  great  pressure  through  a  small 
distance,  commonly  termed  the  toggle  or  toggle  joint.     Will  you  explain 
it,  and  show  wherein  its  peculiar  action  and  efficiency  consist  ? 

11.  Explain  the  mechanical  advantage  of  the  combination  known  as  a 
toggle  joint.      Show  its  application  in  printing  machinery,  or  in   stone 
crushing  machines,    or   in   any   other  instances   with  which   you    are 
acquainted. 


NOTES    AND    QUESTIONS 


12.  Sketch  any  one  form  of  toggle  joint  with  which  you  are  acquainted, 
and  poinc  out  its  object.  (C.  &  G.,  1903,  O.,  Sec.  A.) 

13.  The  figure  shows  a  bent  lever  AOB  with  a  frictionless  fulcrum  0. 
AOis  12",  BO  is  24".     The  force  Q,  of  1000  Ib.  acts  at  A,  what  force  P 
acting  at  B  will  produce  a  balance  ?    Work  the  question  graphically  or  in 
any    other    way.     Neglect  the  weight  of  the   lever.     (B.    of  E.    1904.) 
Ant.     Force  P  =  550  Ibs. 


NOTE— Before  answering  this  question  and  any  future  questions  in- 
volvirxr  angles,  or  the  ratios  of  the  sides  of  a  right  angled  triangle, 
students  should  refer  to  the  construction,  action  and  uses  of  the  Tan- 
gentometer  at  the  end  of  Lecture  XXVIII. 


LECTURE  V. 

CONTENTS. — The  Principle  of  Work — Work  put  in,  Work  lost,  Useful 
Work — Efficiency  of  a  Machine— Principle  of  Work  applied  to  the 
Lever — Experiments  I.  II.  —  Wheel  and  Axle  —  The  Principle  of 
Moments  applied  to  the  Wheel  and  Axle —The  Principle  of  Work 
applied  to  the  Wheel  and  Axle — Experiment  III.  —The  Winch  Barrel 
— Example  I. — Ship's  Capstan — The  Fusee — Questions. 

The  Principle  of  Work.* — The  principle  of  work  is  applicable 
to  all  machines,  and  may  be  stated  as  follows : — 

The  work  put  into  a  machine  is  equal  to  the  work  absorbed  by 
the  machine  plus  the  work  given  out  by  the  machine. 

Or,          WORK  PUT  IN  =  LOST  WORK  +  USEFUL  WORK. 

This  is  an  aociom.  But,  nevertheless,  many  deluded  would-be 
inventors  have  spent  much  time  and  money  in  devising  "perpetual 
motion  "  appliances,  or  machines  which  should  turn  out  as  much 
work  as,  or  even  more  than,  was  put  into  them  ! 

1.  When  a  machine  is  employed  to  perform  mechanical  work, 
a  certain  force  must  be  applied  to  one  part  of  it  in  order  to  move 
the  machine  and  to  perform  work  at  another  part. 

The  product  of  this  applied  force  and  the  distance  through 
which  it  acts  constitute  tlie  whole  work  put  into  the  machine. 

2.  Some  of  this  work  must  be  expended  in  merely  keeping  the 
different  parts  in  motion,  against  natural  resistances  due  to  fric- 
tion at  the  fulcra  or  journals,  and  friction  between  moving  parts 
and  the  air  or  water  in  the  case  of  an  hydraulic  apparatus.     The 
work  so  absorbed  is  termed  lost  work. 

The  mean  value  of  the  frictional  resistances,  multiplied  by  the 
mean  distance  through  which  they  are  overcome,  constitute  the 
work  lost  in  the  mechanism.  One  great  object  to  be  kept  in  view, 
in  designing  most  machines,  is  to  minimise  this  lost  work  by 
minimising  the  internal  resistances  to  motion  in  the  machine 

*  The  Principle  of  Work  is  usually  stated  as  follows  in  books  on  Mecha- 
nics, but  I  find  that  engineering  students  much  prefer  the  above  definition. 
"  If  a  system  of  bodies  be  at  rest  under  the  action  of  any  forces,  and  be 
moved  a  very  little,  no  work  will  be  done."  "  Conversely  :  If  no  work  is 
done  during  this  small  movement,  the  forces  are  in  equilibrium." — Prof. 
Goodeve's  "  Manual  of  Mechanics,"  p.  73. 


PRINCIPLE   OF   WORK    APPLIED    TO  THE    LEVER.  S3 

itself ;  but  you  must  remember  that  these  can  never  be  entirely  dis- 
posed of,  as  has  only  too  often  been  conjectured  by  "  perpetual 
motion  "  faddists. 

3.  The  remainder  goes  to  do  the  useful  work  for  which  the 
machine  was  designed,  and  therefore — 

4.  The  efficiency  of  a  machine  = the  work  Sot  out' 

the  work  put  in. 

To  impress  these  facts  on  the  mind  of  the  student  we  present 
them  in  the  following  condensed  form  : — 

1.  Work  put  in  =  force  applied  x  the  distance  it  acts. 

2.  Work  lost      = force  absorbed  in  overcoming  internal  resistances 

x  the  distance  it  acts. 

3.  Useful  work  =  force  given  out  x  the  distance  it  acts. 

4.  Efficiency  [    =  ratio  of  work  got  out  to  ivork  put  in. 

5.  Work  put  in  =  lost  work  +  useful  work. 

Principle  of  Work  applied  to  the  Lever. — In  applying  the 
above  "  principle  of  work  "  to  the  lever,  we  will  take  the  liberty 
of  neglecting  the  lost 
work.  We  shall  therefore 
assume  that  the  friction 
at  the  fulcrum  is  so  small 
that  it  may  be  neglected 
for  the  purpose  we  have 
in  view.  (^£ 

EXPERIMENT    I.  —  Let      F  fr A| 

AjF   be  a   straight   lever  £3/| 

without    weight,     having  yy 

its  fulcrum  at  Fa  force,        PmNCIPLE  OP  WoRK  APPLIED  T0  A 
W ,  acting  vertically  down-  T  __ 

j       p  AI  •  T~»  iMByJoLK. 

wards  from   the  point  B1? 

and  a  force,  P,  acting  vertically  upwards  at  the  end  Alt  keeping 
W  in  equilibrium.  Now  imagine  the  lever  elevated  to  the  posi- 
tion A,F. 

The  work  put  in  at  A:  =  P   x  the  vertical  distance  from  At  to  A^. 

The  work  got  out  at  Bx  =  W  x  the  vertical  distance  from  Bx  to  B,. 

Therefore,  since  we  neglect  all  frictional  resistances — 

The  work  put  in   =   the  work  got  out 
Or,  P    x    A>A,   =   W    x    B^, 

ie  P          B'B> 

W         AA 

But  by  Euclid  the  triangles  A4FA,  and  BjFBa  are  similar  in  every 
respect. 


54 


LECTURE   V. 


Therefore, 

Hence, 
Or, 


W   ' 
Af  = 


BF 


But  this  is  the  equation  we  proved  in  Lecture  III.  with  respect 
to  the  lever  as  complying  with  the  "  principle  of  moments." 
Hence  the  "principle  of  work  "  and  the  "  principle  of  moments  "  are 
in  agreement. 

In  the  accompanying  figure  the  force  P  has  been  shown  as 
elevated  through  12",  and  the  force  W  as  elevated  through  6", 

Therefore,  P    x    12"   =   W    x    6" 

4  -  T,  '  7 

P  being  half  the  magnitude  of  W,  it  has  to  be  elevated  through 
double  the  distance  in  order  that  the  same  amount  of  work  may 
be  done  in  the  same  time. 

EXPERIMENT  II. — Consider  the  case  of  a  simple  lever,  where  a 
weight,  W,  at  B  is  balanced  by  another  weight,  P,  at  A,  around 


PRINCIPLE  OP  WORK  APPLIED  TO  JL  LEVEB. 

a  fulcrum  at  F,  without  friction.     Let  the  lever  be  turned  through 
90°,  or  a  quarter  of  a  revolution — i.e.,  from  a  horizontal  position, 
AB,  to  a  vertical  position,  A'B'. 
Then  by  the  definition  of  work — 

The  work  put  in  at   A  =  P  x  A'F,  and 
The  work  got  out  at  B  =  W  x  B'F. 


THE   WHEEL   AND    AXLE. 


55 


It  does  not  matter  in  the  slightest  degree  how  circuitous  the 
paths  P  and  "W  take  in  passing  from  their  original  to  their  new 
positions  in  this  case,  since  all  we  require  to  know  is  the  vertical 
distances  through  which  P  is  depressed  and  "W  elevated. 

Consequently,  by  the  "  Principle  of  Work" 

P    x    A'F     =     W 


But,  A'F   =   AF,  and  B'F     =   BF, 

.-.    Substituting  AF  for  A'F,  and  BF  for  B'F, 
We  get,  P    x    AF      =     W    x    BF 

But  this  is  the  equation  for  the  "principle  of  moments"  which 
we  have  again  deduced  from  the  "  principle  of  tuork  "  by  another 
and  simpler  form  of  reasoning.  We  find  that  this  latter  method 
appeals  more  directly  to  the  minds  of  young  engineering  students 
than  the  proofs  usually  found  in  books  on  Mechanics. 

The  Wheel  and  Axle.  —  The  wheel  and  axle  has  been  used  for 
centuries  for  drawing  water  by  a  bucket  from  a  well.  It  is  used 
by  the  navvy  for  lifting  the 
material  which  he  excavates 
from  the  earth,  by  the  mason 
for  raising  stones,  bricks  and 
mortar,  and  by  many  other 
tradesmen  for  a  variety  of 
purposes  ;  as  well  as  by  the 
quartermaster  as  a  steering- 
gear,  and  the  able  seaman  as 
a  capstan.  The  accompanying 
illustration  shows  the  form  it 
takes  when  used  for  elevating 
goods  in  a  store  or  mill.*  It 
is  simply  a  practical  arrange- 
ment for  continuing  the  action 
of  the  lever  as  long  as  re- 

quired.    So  long  as  a  sufficient  WHEEL  AND  AXLE. 

pull   is    applied   to  the  rope, 

which  fits  into  the  grooved  wheel,  to  overcome  the  resistance  of 
the  load  attached  to  the  chain  hook,  the  weight  will  be  raised. 
The  wheel  and  axle  is  therefore  a  form  of  lever  by  which  a  weight 
may  be  raised  through  any  desired  height. 

The  Principle  of  Moments  applied  to  the  Wheel  and 
Axle.  —  In  the  diagram  let  the  larger  circle  represent  the  circum- 
ference of  a  wheel  of  radius,  R,  to  the  periphery  of  which  a  force, 

*  The  above  figure  represents  a  wheel  and  axle  as  supplied  by  Messrs. 
P.  &  W.  MacLellan,  of  Glasgow. 


LECTURE   V. 


P,  is  applied.  Let  the  smaller  circle  represent  the  circumference 
of  the  axle  or  barrel  of  radius,  r,  to  the  periphery  of  which  is 
applied  a  resistance  W.  Let  the  forces  P 
and  W  act  in  the  same  direction  and  verti- 
cally downwards.  Join  the  points  where  the 
lines  of  action  of  the  forces  are  tangents  to 
the  wheel  and  axle  by  a  straight  line,  AB. 
Then,  AB  passes  through  the  common  centre 
of  the  circles — i.e.,  through  their  common 
centre  of  motion  or  fulcrum  F,  and  AF  is 
the  effective  arm  for  the  force  P,  whilst  BF 
is  the  effective  arm  for  the  force  W.  In 
fact,  AFB  is  a  straight  lever  in  equilibrium, 
with  the  fulcrum  at  F. 
Therefore,  taking  moments  about  F,  we  have — 

P      x      AF     =     W      x      BF 
Or,  P      x        R,       =      W      x       r. 

The  Principle  of  Work  applied  to  the  Wheel  and  Axle. 

EXPERIMENT  III. — Take  a  model  of  the  wheel  and  axle  as  illus- 
trated by  the  accompanying  figure.  Let,  forces,  P  and  W,  act  in 
equilibrium,  as  in  the  previous  case,  at  radii  R  and  r  respectively. 


w 
WHEEL  AND  AXLE. 


W 


MODEL  TO  TEST  THE  PEINCIPLE  OF  WOEK  APPLIED  TO  THE 
WHEEL  AND  AXLE. 

Now  mark  carefully  with  a  piece  of  coloured  chalk  or  ink  the  exact 
positions  where  the  tape  supporting  P  is  a  tangent  to  the  wheel, 


THE    WINCH    BARREL. 


and  where  the  cord  supporting  W  is  a  tangent  to  the  barrel.  Pull 
P  until  the  wheel  and  barrel  have  just  made  one  complete  revolu- 
tion. Then,  neglecting  any  force  required  to  overcome  friction  at 
the  bearings  of  the  spindle — 

The  work  put  in  by  P      =       P     x    27rB, 
The  work  got  out  in  raising  W      =      W    x    2irr 

But  the  work  put  in      =      the  work  got  out 
•.    P    x     27rR       =       W     x     2rrr 

Cancelling  2n-  from  each  side  of  the  equation  — 
We  have  P    x    R     =     W    x  r. 

But  this  is  the  same  equation  as  we  obtained  above  by  applying 
the  "principle  of  moments."  Therefore,  we  see  that  the  "prin- 
ciple of  moments"  and  the  "principle  of  work"  harmonise. 

The  Winch  Barrel. — The  wheel  may  be  replaced  by  a  handle 
H,  and  the  mere  axle  by  a  barrel  or  drum  D,  of  any  desired  size. 


W 

SIDE  VIEW.  END  VIEW. 

WINCH  BARREL  AND  HANDLE. 

EXAMPLE  I. — A  man  exerts  a  constant  force  of  30  Ibs.  on  a 
winch  handle  of  15"  radius  ;  what  weight  will  he  be  able  to  lift 
attached  to  a  rope  hanging  from  a  barrel  of  5"  radius  ? 

By  the  principles  of  moments  and  of  work ;  and  interpolating 
the  numerical  values — 

Px  R-Wxr 
30  x  i5=Wx  5 

...    w  =  3^115  =  BO  Ib.. 

Ship's  Capstan. — A  partly  sectional,  partly  outside  view  of 
this  useful  machine  is  illustrated  by  the  following  figure : 

A  capstan  is  generally  fixed  upon  the  forecastle  of  a  ship,  or 
near  to  the  side  of  a  quay  or  dock,  for  the  purposes  of  warping  and 


LECTURE   V. 


7°°°° 

!BF      - 

SHIP'S 

CAPSTAN. 

INDEX 

TO  PAETS. 

OH  represents  Capstan  head. 
SS          ,,          Spokes  or  arms. 

E          „          Eadius  of  S. 

B          „          Barrel. 

r  Radius  of  B. 


PR  represents  Pall  and  Ratchet. 

F          „          Frame. 
CP         „          Capstan  pillar. 
FS         „          Footstep  of  CP. 


berthing  the  vessels.  The  above  illustration  shows  a  capstan  as 
built  into  a  forecastle,  where  the  round  turned  footstep,  FS,  of 
the  vertical  cast-iron  capstan  pillar,  CP,  bears  in  a  cast-iron  or 
cast-steel  shoe  fitted  upon  the  steel  or  wrought  beams  of  the  main 
deck.  The  frame  F,  which  supports  the  casing  for  the  pall  and 
ratchet  gear,  may  be  the  beams  of  the  upper  or  forecastle  deck. 
A  strong  rope  made  fast  on  shore  is  passed  several  times  round 
the  capstan  barrel  B,  and  the  slack  end  of  the  rope  is  coiled  on 
deck.  The  addition  of  the  rope  to  the  barrel  increases  the  effective 
arm  or  radius  r,  at  which  the  resistance  of  the  ship  acts  by  half 
the  diameter  of  the  rope.  Eight  or  any  desired  less  number  of 
wooden  spokes,  S,  S,  having  their  inner  ends  squared  and  tapered, 
are  fixed  into  hollow  square  holes  in  the  cast-iron  capstan 
head  CH.  Then,  just  as  many  sailors  as  may  be  required  to 


SHIP'S    CAPSTAN.  59 

overcome  the  resistance  of  the  ship  apply  themselves  to  the  outer 
rounded  ends  of  the  spokes,  and  push  away  as  hard  as  they  can. 

It  will  be  observed  that,  calling,  p,  the  force  applied  by  each 
sailor  at  radius  R ;  then,  when  we  have  two  saiJors  acting  on 
diametrically  opposite  spokes  /?,  2R,  p  forms  a  couple  tending  to 
cause  rotation  of  the  capstan  in  one  direction.  Consequently 
from  the  property  of  couples  (as  we  showed  in  Lecture  III.)  this 
couple  can  only  be  balanced  by  another  couple  acting  in  the 
opposite  direction  and  having  an  equal  moment.  Such  another 
couple  exists,  when  the  resistance  of  the  ship,  W,  acting  with  an 
arm,  r  (equal  to  the  distance  from  centre  of  capstan  to  centre  of 
rope),  balances  the  corresponding  reaction  at  the  centre  of  the 
capstan  barrel.  Hence,  when  the  force  applied  by  the  two  sailors 
is  balanced  by  the  resistance  to  motion  of  the  ship,  we  have  the 
one  couple  just  balancing  the  other  one. 

Or  Couple  p,  2R,  p  balancing  couple  W,  r,  W 

i.e.,  p  x  2B  =  W  x  r 

In  the  same  way,  with  two,  three,  or  four  pairs  of  sailors,  each 
pair  being  supposed  to  act  on  diametrically  opposite  spokes,  we 
have  two,  three,  or  four  couples  acting  in  one  direction,  balanced 
by  one  couple,  viz.,  the  resistance  of  the  ship  into  the  distance 
from  the  centre  of  the  capstan  barrel  and  the  reaction  from  that 
centre.* 

In  the  case  of  four  sailors  just  being  able  to  move  the  ship,  two 
couples,  p,  2R,  p+p,  2R,  p,  balance  one  couple,  W,  r,  W; 

*'.«.,  p  x  2R.+J9  x  2R  =  W  xr 

The  Fusee. — As  an  illustration  of  the  lever  action  and  of 
work  put  into  and  got  out  of  a  machine,  we  cannot  do  better 
than  finish  this  lecture  by  a  description  of  the  construction 
and  action  of  the  simple  yet  most  ingenious  contrivance  termed 
the  fusee.  In  good  watches  and  clocks,  where  the  elastic  force 
of  a  coiled  spring  is  used  to  drive  the  works,  the  fusee  is  used 
for  the  purpose  of  compensating  the  gradually  diminishing  pull 
of  the  uncoiling  spring.  The  driving  of  the  works  at  a  constant 
rate  is  the  object  for  which  a  watch  or  clock  is  designed.  This 
naturally  entails  a  constant  resistance  to  be  overcome,  but  since 
one  of  the  most  compact  and  convenient  forms  of  mechanism  into 

*  The  student  should  draw  a  plan  of  the  capstan  barrel,  and  show  radial 
lines  to  indicate  one,  two,  or  more  pairs  of  diametrically  opposite  spokes 
with  forces,  p,  acting  at  their  ends,  all  tending  to  turn  the  barrel  in  one 
direction.  He  will  then  see  that  a  couple  formed  by  resistance  to  the  stress 
on  the  rope,  and  an  equal  reaction  from  the  centre  of  motion,  will  be  required 
to  act  in  the  other  direction  in  order  that  equilibrium  may  take  place. 


6o 


LECTURE   V. 


which  mechanical  force  can  be  stored  is  that  of  a  coiled  spring, 
and  since  the  very  nature  of  the  spring  is  such  that  its  force 
decreases  as  it  uncoils,  we  must  employ  some  compensating 
device  between  this  variable  driving  force  and  the  constant 
resistance.  The  fusee  does  this  in  a  most  accurate  and  complete 


THE  FUSEE  FOE  A  CLOCK  OR  WATCH. 


INDEX  TO  PARTS. 

F  represents  Fusee. 
B          „          Barrel. 
BW  Ratchet  wheel. 


TW  represents  Toothed  wheel. 
WS          ,,          Winding  square. 


manner.  Looking  at  the  accompanying  figures  and  index  to  parts, 
we  see  that  the  barrel  B,  which  contains  the  watch  or  clock 
spring,  is  of  uniform  diameter,  and  that  between  the  outside  of 
this  barrel  and  the  fusee,  or  spirally  grooved  cone,  there  passes  a 
cord  or  chain.  When  the  winding  key  is  applied  to  the  winding 
square  WS,  and  turned  in  the  proper  direction,  a  tension  is 
applied  to  the  cord,  and  it  is  wound  upon  the  spiral  cone,  thus 
coiling  up  the  spring  inside  the  barrel  B ;  for  the  outer  end  of 
this  spring  is  fixed  to  the  periphery  of  the  barrel,  and  the  inner 
end  to  its  spindle  or  axle,  is  in  direct  gear  with  the  works  of  the 
clock.  When  the  spring  is  fully  wound  up  it  has  the  greatest 
force,  but  it  acts  with  the  least  advantage,  since  then  the  cord  is 
on  the  smallest  groove  of  the  cone  pulley.  When  the  spring  is 
almost  uncoiled  it  acts  with  the  greatest  advantage,  for  then  the 
eord  is  on  the  largest  groove  of  the  cone.  Consequently  the  radii 


THE   FUSEE.  6 1 

of  the  grooves  of  this  cone  are  made  to  increase  in  proportion  as 
the  force  applied  to  the  cord  decreases  in  order  that  there  shall 
be  a  constant  turning  effort  on  the  works  of  the  clock. 

The  work  put  in  when  winding  up  the  coiled  spring,  is  given  up 
by  it  in  overcoming  the  frictional  resistances  of  the  different  parts 
of  the  mechanism. 

Or  the  work  put  in  =  lost  work,  for  the  whole  of  the  work  put 
in  is  devoted  to  simply  keeping  the  parts  of  the  machine  in 
motion,  thus  leaving  nothing  for  other  work,  unless  the  clock  is 
used  to  strike  a  bell  or  do  some  other  kind  of  work. 


62  LECTURE    Y. — QUESTIONS. 


LECTURE  V. — QUESTIONS. 

1.  State  the  "  Principle  of  Work,"  and  explain  the  manner  in  which  it 
is  applied  in  determining  the  relation  of  a  P  to  W  in  the  lever.     A  lever, 
centred  at  one  end,  is  15  feet  long,  and  a  weight  of  W  Ibs.  hangs  from  the 
opposite  end.     The  weight  W  is  supported  by  an  upward  pressure  of 
28,270  Ibs.  at  13  feet  from  the  fulcrum.     Find  W.     Ans.  24,5006  Ibs. 

2.  Define  work  put  in,  lost  in,  and  got  out  of  a  machine,  and  prove  that 
the  work  put  in  =  lost  work  plus  the  useful  work.     How  are  the  "advan- 
tage "  and  the  efficiency  of  a  machine  reckoned  ? 

3.  Sketch  and  describe  the  wheel  and  axle.     Apply  both  the  "principle 
of  moments"  and  the  "principle  of  work"  to  find  the  relation  between 
the  force  applied  and  the  weight  raised  by  aid  of  this  machine.     A  wheel 
and  axle  is  required  so  that  the  force  applied  at  the  circumference  of  the 
wheel  in  moving  through  a  distance  of  10  feet    shall  raise  a  weight  of 
4  cwts.  through  a  height  of  2  feet.     If  the  diameter  of  the  axle  is  10  inches, 
find  the  force  applied  in  Ibs.,  and  the  radius  of  the  wheel  in  feet.     Ans. 
89-6  Ibs.  ;  2  feet  I  inch. 

4.  The  crank  or  handle  which  turns  a  windlass  is  14  inches  in  length  j 
what  must  be  the  diameter  of  the  axle  when  a  man  exerting  a  force  of 
60  Ibs.  upon  the  handle  raises  a  tub  of  coals  weighing  2  cwt.  1    Ans. 
7^  inches. 

5.  In  a  windlass  the  barrel  is  8  inches  diameter,  the  rope  is  I J  inches 
diameter,  and  the  crank  handle  15^  inches  long.     What  force  must  b« 
applied  at  the  handle  to  raise  2  cwt.  1    Also,  what  weight  would  be  raised 
by  a  constant  force  of  30  Ibs.  applied  at  the  handle  1    Ans.  66-8  Ibs. ; 
100-5  Ibs. 

6.  A  capstan  is  worked  by  four  men  ;  each  man  exerts  a  constant  force 
of  30  Ibs.  at  a  distance  of  4  feet  from  the  axis.     A  rope  of  f -inch  diameter 
is  wound  round  the  drum,  of  5|  inches  radius.     Find  the  pull  on  the  rope 
which  balances  the  pressure  on  handles.     Make  a  diagram  showing  the 
action  of  the  forces,  and  find  the  pressure  on  the  central  shaft  of  the  cap- 
stan.    Ans.  921  6  Ibs.  ;  921-6  Ibs. 

7.  Describe,  with  a  sketch,  the  spring-barrel  and  fusee  of  a  clock  or  of  a 
watch.     Explain  its  action  by  reference  to  the  principle  of  moments. 

8.  A  ship's  capstan   has   a  ratchet-wheel  with  two  detents   or  pawls, 
arranged  so  that  when  one  is  engaged  with  a  tooth  of  the  wheel  the  point 
of  the  other  is  midway  between  two  teeth.     Sketch  the  arrangement,  and 
say  why  in  this  case  two  detents  or  pawls  are  better  than  one. 

9.  Find  the  average  horse-power  exerted  by  a  winding  engine  to  lift 
3  tons  from  a  pit  f  mile  deep  at  a  uniform  speed  in  two  minutes,  supposing 
that  30  per  cent,  of  the  total  work  done  is  lost  in  friction. 

(0.  &  G.,  1905,  O.,  Sec.  A.)     Ans.  576  H.P. 


LECTURE  VI. 

CONTENTS.—  Pulleys—  Snatcn  Block—  Block  and  Tackle  —Theoretical  Ad- 
vantage —  Velocity  Ratio  —  The  Principle  of  Work  applied  to  the  Block 
and  Tackle—  Actual  or  Working  Advantage  —  Work  put  in—  Work 
got  oat  —  Efficiency  —  Percentage  Efficiency  —  Example  I.  —  Questions. 

Pulleys.  —  Suppose  you  had  to  elevate  a  sack  of  flour  from  the 
ground  to  an  upper  storey  of  a  mill  or  store,  you  might  place  it 
upon  your  back  and  carry  it  up  the  stairs.  In  doing  so,  you 
would  expend  so  many  foot-pounds  of  work.  Let  the  sack  of 
flour  be  100  Ibs.,  your  own  weight  150  Ibs.,  and  the  height  to 
which  it  is  raised  be  30  feet.  Then  the 

Work  done  in  elevating  the  flour  =  100  Ibs.  x  30'  =  3000  ft.-  Ibs. 
„  „  yourself  =150  „    x  30'  =  4500      „ 

Total  work  done  a»  250  „    x  30'  =  7500      „ 

And  your  efficiency  as  a  machine  would  be  found  thus  — 

Mechanical  efficiency  =°sefulwork         work  got  out  =  3000  ft-lba. 
total  work  work  put  in      7500  ft.-lbs. 

Or,  your  percentage  efficiency  would  be  found  from  the  propor- 
tion — 

7500    :    3000   ::    100    :    x 


7500 

In  other  words,  60  per  cent,  of  the  total  work  done  is  lost  work, 
and  only  40  per  cent,  is  useful  work. 

If  instead  of  carrying  the  sack  upstairs,  you  found  ready  to 
hand  a  long  rope  (with  its  two  ends  close  to  the  ground)  that  had 
been  passed  over  a  smooth  iron  hook  fixed  to  the  outside  wall 
above  an  outside  landing  for  the  particular  storey  of  the  building, 
and,  if  you  attached  one  end  of  this  rope  to  the  sack  and  found 
that  by  pulling  with  all  your  strength  (or  say  with  a  force  of 
150  Ibs.,  i.e.,  equal  to  your  weight)  on  the  other  end,  you  could 
just  lift  the  sack.  Then,  if  by  this  means  you  elevated  the  sack 
to  the  landing,  you  would  have  expended  less  work  than  by  the 
iormer  method  ;  for, 


64 


LECTURE   VI. 


Work  done  in  elevating  flour      =  loolbs.  x  30'=  3000  ft.-lbs. 
„  against  friction,  &c.  =    50  ,,    x  30'  =  1500      „ 

Total  work  done 


=150  „    x  30  =  4500 


.-.  Mechanical  efficiency  =  Hi6*"1 


or,  ut^  3000^.5 

work  put  in      4500 


total  work 
And  the  percentage  efficiency  is  therefore  66-6. 

For,  4500     :     3000    ::    100 
3000  x  100 
45°° 


66-6   % 


Hence  33.3  per  cent.,  or  ^  of  the  total 
work  put  in  by  you  in  pulling  at  one 
side  of  the  rope,  is  spent  in  overcoming 
the  friction  between  the  rope  and  the 
hook  and  bending  the  rope  over  the  hook, 
whilst  only  6  6'  6  per  cent.,  or  |,  remain 
for  elevating  the  sack  of  flour. 

If,  instead  of  the  iron  hook  you  had 
found  a  double-flanged  deep  V-grooved 
pulley  with  a  rope  over  it,  as  in  the  ac- 
companying illustration,  and  that  this 
pulley  revolved  so  easily  on  its  bearings 
that  you  had  only  to  pull  with  a  constant 

force  of  no  Ibs.  in  order  to  lift  the  sack  of  flour  from  the  ground 

up  to  the  3o-feet  level,  then  — 

Work  done  in  elevating  flour      =  100  Ibs.  x  30'  =  3000  ft.-lbs. 
„  against  friction,  &c.  =    10    „    x  30'=    300      „ 


PULLEY  AND  WEIGHTS. 


Total  work  done 


no 


30  =3300 


.*.  Mechanical  efficiency  =  lisefnl  work  .  Or  ^J^  out=3ooo  =  .^ 
total  work   '      '  work  put  in      3300 

And  the  percentage  efficiency  is  90-9 
For  3300     :     3000    ::    100     :     x 

x   =    300Q  *  IPO    sf    9Q.9   % 


Hence  only  9-1  per  cent,  of  the  total  work  put  in  is  lost  work 
In  overcoming  friction  at  the  pulley  bearing  and  in  bending  the 
rope  over  the  pulley. 

You  see,  therefore,  what  a  useful  machine  a  pulley  is,  not  only 
for  enabling  you  to  change  the  direction  of  a  force,  but  aho  for 
the  saving  of  labour. 


SNATCH   BLOCK,  ,  65 

A  pldley  is  simply  a  wfael  and  axle  wherein  their  radii  are  one 
and  the  same,  or  a  lever  with  equal  arms.  Hence  the  principles  of 
moments  and  of  work  may  be  applied  to  it  in  the  same  way  ag 
we  applied  them  to  the  lever  and  to  the  wheel  and  axle. 

Snatch  Block. — If  you  should  require  to  put  the  bend  of  a 
rope  on  a  pulley,  and  at  the  same  time  prevent  the  possibility  of 
the  rope  coming  out  of  the  groove,  without  having  to  reeve  the 
end  of  the  rope  between  its  cheeks,  you  would  use  what  is  called  a 
snatch  block.     One  form  of  snatch  block  is  illus- 
trated by  the  accompanying  figure,  where  on  the 
side  of  one  cheek  there  is   a  sneck  or  snatch, 
which  is  turned  to  one  side,  to  enable  the  bend  of 
the  rope  to  be  placed  around  the  U  groove  of  the 
pulley.     The  snatch  then  falls  down  and  closes 
upon  the  central  pin.      Another    form    has    a 
hinged  snatch  which  can  be  lifted  up  at  right 
angles  to  the  face  of  the  cheek,  and  after  the 
rope  has  been  put  on  the  pulley  the  snatch  is 
closed  down  and  locked  by  a  pin  attached  to  a 
short  chain  fixed  to  the  side  of  the  cheek,  just 
like  an  ordinary  front  hinge  for  closing  a  chest. 
The  single  movable  pulley,  which  is  used  for  sup- 
porting the  load  to  be  lifted  by  a  Chinese  wind- 
lass or  by   a  jib  crane,   is    sometimes   called   a  *u'. J"" 
snatch  block  (see  the  illustration  of  the  wheel  and    ' 
compound  axle  in  next  Lecture,  and  of  jib  cranes  in  Lectures 
VIII.  and  XIII.).     In  the  latter  case  the  chain   passes   from 
the  barrel  of  the  crane  over  the  pulley  at  the  point  of  the  jib,  then 
vertically  down,  underneath  the  snatch-block  pulley,  and  vertically 
upwards  to  a  point  on  the  under  side  of  the  jib  where  it  is  fixed 
by  an  eye-shackle  with  a  bolt  and  nut.     If  the  load,  including  the 
weight  of  the  snatch-block,  be  W,  then,  neglecting  friction,  the 

W 

pull  P  on  the  chain  will  be  — ;  f  or  W  is  supported  by  two  ver- 
tical or  parallel  parts  of  the  chain,  each  part  carrying  half  the 
load,  or  W  =  2?.  If  the  load  be  elevated  any  distance  L,  then 
the  chain  will  have  to  be  pulled  in  on  the  barrel  a  distance  of  2L, 
for  by  the  principle  of  work 

The  pull  x  its  distance  =  the  load  x  its  distance. 
Or,  P  x   2L  =  W   x  L. 

The  theoretical  advantage  is  therefore  2  to  i,  or  a  certain  force 
would  lift  double  the  weight,  neglecting  friction. 

Block  and  Tackle. — Passing  over  the  various  arrangements 
of  pulleys  for  lifting  weights  which  are  treated  of  in  theoretical 


66 


LECTURE   VI. 


mechanics,  we  come  to  this  well-known  and  Useful  contrivance. 
As  will  be  seen  from  the  accompanying  sketch,  it  consists  of  a 
number  of  pulleys  (or  sheaves  as  they  are  technically  termed)  free 

to  run  round  on  a  turned  central 
iron  or  steel  spindle,  and  inserted 
in  a  block,  having  their  iron  divi- 
sions between  each  pulley,  and  strong 
iron  cheeks  fixed  to  a  swivel  joint- 
terminating  in  an  iron  hook  hung 
from  an  eye  bolt.  Three  sheaves 
are  shown  in  this  block,  but  the 
number  may  range  from  one  up- 
wards, according  to  the  size  and 
work  to  be  done.  There  is  a  simi- 
larly constructed  block  with  two 
sheaves,  to  which  the  weight  to 
be  raised,  or  the  body  to  be  pulled, 
is  attached,  and  this  is  called  the 
movable  block,  whereas  the  upper 
or  home  one  is  termed  the  fixed 
block.  Around  the  pulleys  of  both 
blocks  there  is  reeved  a  rope  with 
the  inner  end  made  fast  to  an  eye 
on  the  movable  block,  whilst  the 
free  end  hangs  from  one  of  the 
outside  sheaves  ;  but  this  arrange- 
ment is  frequently  reversed,  for  the 
inner  end  of  the  rope  may  be  at- 
tached to  an  eye  on  the  fixed  block, 
and  the  free  end  may  spring  from 
the  other  one  (see  the  figure  in  con- 
nection with  Example  I.  of  this 
Lecture).  The  free  end  of  the  rope 
is  then  ready  to  be  pulled  by  the 
hands  or  by  aid  of  a  winch. 

Now,     neglecting     friction,    and 

supposing  the  rope  to  be  perfectly  flexible,  a  force,  P,  applied  to 
the  free  end  of  the  rope  would  be  transmitted  throughout  it  to 
the  other  end  at  the  movable  block.  Hence  the  effect  of  this 
force  in  overcoming  a  resistance,  W,  is  multiplied  by  the  number, 
n>  of  parts  of  tfie  rope  which  spring  from  the  movable  block. 


BLOCK  AND  TACKLE. 


Or, 


W 


And  (i)  The  theoretical  advantage 


W 
P 


I  UNIVERSITY    I 

X^LIFOR!*^^ 


THB    BLOCK    AND   TACKLE/  67 

(2)  The  velocity  ratio,  or  ratio  of  the  distance  through  which 
P  acts,  to  that  through  which  W  is  overcome  in  the  same  time. 

Or,  Velocity  ratio  =  P'8  ^tapce  =  ^ 

W  s  distance       i 

In  the  figure  there  are  shown  three  pulleys  in  the  upper  block 
and  two  in  the  lower,  with  five  parts  of  rope  springing  from  the 
latter ;  therefore  in  this  case  n  =  5. 

Here  W  =  nP  =  5P;  or,  P  *  ^  -  ^ 

n          5 

since  P  must  pass  through  five  times  the  distance  that  W  does 
in  the  same  time. 

The  velocity  ratio  =  ^  d^stapce    =  »  =  * 
W's  distance        i       1 

So  that  the  theoretical  advantage  and  the  velocity  ratio  have  the 
same  algebraical  expression  and  numerical  value.  -  (See  note,  p.  68.) 
The  Principle  of  Work  applied  to  the  Block  and  Tackle. 
— Using  the  very  kind  of  block  and  tackle  represented  by  the 
previous  figure,  attach  a  light  Salter's  spring  balance  by  its  hook 
to  the  rope  where  the  hand  is  shown.  Fix  such  a  weight  to  the 
lower  block  that  the  weight  of  rope  between  the  blocks,  the 
movable  block,  and  the  load  are  60  Ibs.  Call  this  W.  Now 
pull  the  ring  of  the  spring  balance  until  the  load  rises  slowly  and 
uniformly,  and  note  the  reading  on  the  balance  ;  let  it  be  18  Ibs., 
and  let  the  weight  of  this  balance  and  the  hanging  free  end  of 
the  rope,  which  is  assisting  the  arm,  be  2  Ibs.  Call  this  total 
pull  of  20  Ibs.  P ;  then  : 

(3)    27U  actual  or  Dicing  advance =wei8ht  raised=W=6olbs.=8 

pull  applied      P     20  Ibs.     1 

Lift  W  up  through  one  foot  exactly,  and  measure  the  length  of 
rope  which  you  have  pulled  out  from  the  upper  block,  and  you 
will  find  that  it  is  five  feet ;  hemce,* 

(4)  The  work  put  in  =  P  x  n=  20  Ibs.  x  5  ft.  =  100  ft.-lbs. 

(5)  The  work  got  out  =  W  x  i  =  60  Ibs.  x  i  ft.  =    60  ft.-lbs. 

Work  got  out          60  ft.-lbs. 

(6)  The  efficiency         =-— — -J     — —    _ =8-3 

Work  put  in        I00  ft.-lbs. 

(7)  The  percentage  efficiency  =  '6   x  100  =  60  % 

In  the  same  way  the  efficiency  of  uny  other  block  and  tackle 
may  be  found,  and  the  student  should  carry  out  a  series  of 

*  The  above  results  were  obtained  by  the  Author  from  a  block  and 
tackle  of  the  same  kind  as  that  shown  by  the  previous  figure,  at  a  demon- 
stration in  his  Junior  Applied  Mechanics  class. 


68 


LECTUEE   VI. 


experiments  in  a  laboratory  or  workshop  so  as  to  impress  the 
various  measurements  and  the  results  on  his  memory.  He  will 
find  that  if  the  efficiency  is  over  50  per  cent,  a  comparatively 
small  load  will  run  down  and  overhaul  the  free  end  of  the  rope, 
unless  it  has  some  restraining  force  applied  to  it,  or  be  fixed  to 
some  rigid  body.  It  is  for  this  reason  that 
sailors,  who  work  very  much  with  ordinary 
block  and  tackle,  always  "  belay  "  tne  free 
end  of  the  rope  when  they  have  adjusted 
their  sails  or  have  heaved  up  a  body  to 
the  required  height. 

EXAMPLE  I. — A  tackle,  consisting  of  an 
ordinary  double  and  treble  block,  is  em- 
ployed for  lifting  a  weight  of  600  Ibs. 
attached  to  the  double  block.  What 
force  is  required,  -  neglecting  friction  ? 
If  the  tackle  is  reversed,  so  that  the 
weight  is  attached  to  the  treble  block, 
the  free  end  of  the  rope  being  pulled 
upwards,  what  force  would  now  be  re- 
quired to  lift  the  weight?  (S.  and  A. 
Exam.  1892.) 

BLOCK  AND  TACKLE  ANSWER.— First    Case.— By  an  inspec- 

2ND  CASE,  EXAMPLE  I.      ,.         -  ,.  „          .    J..  .   T 

tion  or  the  previous  figure  in  this  Lecture, 

it'  will  be  apparent  that  the  weight  W  is  supported  by  Jive  parts 
of  the  rope,  or  n  =  5. 


*,- -- 

n 


W 
T 


—  =  120  Ibs. 
5 


/Second  Case.  —  Here  the  system  is  inverted,  so  that  the  block 
with  the  three  pulleys  is  lowermost,  as  shown  by  the  accompany- 
ing figure.  In  this  case  it  is  evident  that  there  are  six  parts  of 
the  rope  supporting  W,  or  n  =  6. 


n 


Note. — If  a  machine  be  supposed  to  work  without  friction,  then  the  ratio 
of  the  resd&tanoe  overcome  to  the  effort  applied  is  termed  the  theoretical 
or  hypothetical  mechanical  advantage.  If,  however,  friction  be  taken  into 
account  and  an  effort  P  be  able  to  overcome  a  resistance  W,  then  the  ratio 
W 
—  is  termed  the  mechanical  advantage. 


LBCTURB   VI. — QUESTIONS.   v  69 

LBCTUBB  VI.— QUESTIONS. 

1.  Suppose   that  your  weight  is  10  stone  10  Ibs.,  and  that  you  lift  a 
weight  of  \  cwt.  on  your  shoulderrand  walk  upstairs  with  it  to  a  height  of 
20  ft.  ;  what  work  have  you  expended,  and  what  will  be  your  efficiency  as 
a  machine  1    Am.  4120  ft-lbs. ;  27  per  cent. 

2.  Suppose  that  you  had  a  rope  passed  round  a  beam  of  wood,  and  that 
you  attached  4  cwt.  to  one  end  and  pulled  with  a  force  of  84  Ibs.  on  the 
other  end  and  then  elevated  it  10  ft. :  (a)  what  work  have  you  put  in  ? 
(b)  what  is  the  percentage  efficiency  of   the  arrangement  ?  (c)  what  is  the 
percentage  of  lost  work  ?    ATM.  (a)  840  ft. -Ibs.  ;  (b)  66-6  ;  33-3. 

3.  Suppose  that  a  weight  of  4  cwt.  is  attached  to  one  end  of  a  rope  passed 
round  a  pulley,  and  that  you  lift  it  10  ft.  by  pulling  on  the  other  end  of 
the  rope  with  a  force  of  70  Ibs. :  what  percentage  of  the  work  done  is  lost 
in  overcoming  the  friction  at  the  pulley  ?    Ans.  20  per  cent. 

4.  What  will  be  the  difference,  and  why,  in  the  tension  on  the  chain  of 
a  crane  when  a  snatch-block  is  used,  and  when  the  weight  is  lifted  directly 
Sketch  a  snatch-block,  and  describe  its  construction  and  action. 

5.  In  a  rope  and  pulley  lifting  block  with  three  sheaves  in  the  upper 
block,  and  two  sheaves  in  the  lower  block,  find  the  theoretical  advantage 
gained.    Give  the  reason  for  your  answer,  and  sketch  the  arrangement, 
showing  where  the  rope  is  to  be  attached.     Arts.  W  :  P  : :  5  :  I. 

6.  Sketch  an  arrangement  of  5  equal  pulley  sheaves  for  lifting  a  weight 
of  i  ton.     What  force  is  exerted  on  the  rope  in  your  arrangement  ?     Ex- 
plain the  mo^e  of  arriving  at  this  numerical  result  by  the  principle  of 
work.     An».  With  3  pulleys    in    upper    block    and    2    in  lower  block, 
P  =  448  Ibs. 

7.  A  tackle  is  formed  of  two  blocks,  each  weighing  15  Ibs.,  the  lower 
one  being  a  single  movable  pulley,  and  the  upper  or  fixed  block  having  two 
sheaves  ;  the  parts  of  the  cord  are  vertical,  and  the  standing  end  is  fixed 
to  the  movable  block  ;  what  pull  on  the  cord  will  support  200  Ibs.  hung 
from  the  movable  block,  and  what  will  then  be  the  pressure  on  the  point 
of  support  of  the  upper  block  ?    Give  a  sketch.     Ant.  71*6  Ibs. ;  301-6  Ibs. 

8.  A  weighf  of  400  Ibs.  is  being  raised  by  a  pair  of  pulley  blocks,  each 
having  two  sheaves.    The  standing  part  of  the  rope  is  fixed  to  the  upper 
block,  and  the  parts  of  the  rope,  whose  weight  may  be  disregarded,  are 
considered  to  be  vertical.     Each  block  weighs  10  Ibs.;  what  is  the  pres- 
sure at  the  point  from  which  the  upper  block  hangs  1    An*.  522-5. 

9.  A  tackle,  consisting  of  an  ordinary  double  and  treble  block,  is  em- 
ployed for  lifting  a  weight  of  1000  Ibs.  attached  to  the  double  block.  What 
force  is  required,  neglecting  friction  ?     If  the  tackl*  is  reversed,  so  that 
the  weight  is  attached  to  the  treble  block,  the  free  «nd  of  the  rope  being 
pulled  upwards,  what  force  would  now  be  required  to  lift  the  weight  ? 
Sketch  the  two  arrangements.     Ans.  200  Ibs.;  166-6  Ibs. 

10.  Apply  the  "  principle  of  work  "  to  find  the  relation  between  the  force 
applied  and  the  weight  raised  by  an   ordinary  set  of  block  and  tackle. 
State  what  is  meant  by  the  following  terms  :— {i)  velocity-ratio  ;  (2)  theo 
retical  mechanical  advantage ;  (3)  actual  or  working  advantage ;  (4)  work 
put  in ;    (5)  work  got  out ;   (6)  efficiency  of  an  apparatus  or  machine  ; 
(7)  percentage  efficiency. 

11.  With  an  ordinary  block  and  tackle  haying  3  pulleys  in  upper  block 
and  2  in  lower  block — i.e.,  5  ropes  attached  to  lower  block — it  is  found  that 
a  pull  of  50  Ibs.  is  required  to  raise  a  weight  of  165  Ibs.     Find— (i)  Theo 
retical  advantage  and  velocity  ratio  =  5  :  i  ;  (2)  Actual  advantage  =3- 3  :  I ; 
(3)  Efficiency  of  apparatus =-66 ;  (4)  Percentage  efficiency  of  apparatus =66. 


70  LECTURE   VI. — QUESTIONS. 

12.  If  the  upper  block  of  a  set  of  pulleys  and  tackle  has  four  equal 
sheaves,  and  the  lower  block  three  equal  sheaves,  and  if  a  weight  of  one 
ton  is  hung  on  the  lower  block,  one  end  of  the  rope  being  fixed  to  the 
ground  and  the  other  end  free,  what  pull  upon  the  free  end  will  raise  the 
weight,  and  what  distance  will  the  weight  rise  for  every  yard  of  increase 
of  length  in  the  free  end?     If  the  rope  be  fastened  to  the  lower  block 
instead  of  to  the  ground,  what  pull  will  raise  the  weight  ? 

Ans.  373  Ibs.  ;  6  inches  ;  and  320  Ibs. 

13.  A  machine  is  concealed  from   sight  except   that  there  are  two 
vertical  ropes  ;  when  one  of  these  is  pulled  down  the  other  rises.     How 
would  you  find  the  efficiency  of  this  lifting  machine?     What  do  you 
mean  by  velocity  ratio,  and  by  mechanical  advantage  ?  (S.  E.  B.  1901.) 

14.  In  a  lifting  machine  an   effort  of  26*6  Ibs.  just  raised  a  load  of 
2260  Ibs.  ;  what  is  the  mechanical  advantage  1    If  the  efficiency  is  0755, 
what  is  the  velocity  ratio ?     Ans.  85  ;  113.  (B.  of  E.,  1902.) 

15.  Distinguish  between  force,  work  and  rate  of  work.     Find  the  pull  on 
the  draw  bar  exerted  by  a  locomotive  which    develops  600  horse-power 
when  travelling  at  60  miles  an  hour — the   mechanicaf  efficiency  of  the 
locomotive  being  taken  as  60  per  cent.     Ans.  2250  Ibs.     (.0.  &  G.,  1903,  0.) 

16.  Define  the  terms  mechanical  advantage,  velocity  ratio,  and  efficiency,  as 
applied  to  lifting  tackle. 

In  a  lifting  machine  an  effort  of  26  Ibs.  just  raises  a  load  of  2200  Ibs.. 
and  the  efficiency  is  075.     Find  the  values  of  the  mechanical  advantage 
and  velocity  ratio.     If,  with  the  same  machine,  a  load  of  12  Ibs.  lifts  a 
load  of  600  Ibs.,  what  is  the  new  efficiency?     (C.  &  G.,  1905,  0.,  Sec.  A. 
Ans    Mech.  Adv.  =  84-6 ;  Vel.  Ratio  =  112-5  : 1  ;  Efficiency  =•  '44. 


KOTES    AND    QUESTIONS. 


LECTURE  VII. 


CONTENTS.—  The  Wheel  and  Compound  Axle,  or  Chinese  Windlass—  The" 
Principle  of  Moments  applied  to  the  Wheel  and  Compound  Axle  —  The 
Principle  of  Work  applied  to  the  Wheel  and  Compound  Axle  —  Ex- 
amples I.  II.  —  Weston's  Differential  Pulley  Block  —  The  Principle  of 
Work  applied  to  Weston's  Differential  Pulley  Block—  Experiment  I.  — 
Cause  of  the  Load  not  overhauling  the  Chain—  Questions. 

The  Wheel  and  Compound  Axle,  or   Chinese   Windlass. 

—  This  ingenious  contrivance  was  first  devised  by  the  Chinese 
for  the  purpose  of  lifting  weights.  The  theoretical  mechanical 
advantage  is  very  great,  but  it  possesses  the  disadvantage  of  re- 
quiring a  long  length  of  rope  to  lift  the  weight  a  small  height. 

Its  construction  and  action  will  be  easily  understood  from  the 
accompanying  side  and  end  views,  which  are  taken  from  a  model 


SIDE  VIEW.  END  VIEW 

(Without  End  Bearing). 

THE  WHEEL  AND  COMPOUND  AXLE. 


THE   WHEEL   AND    COMPOUND    AXLfc.  73 

made  in  the  author's  engineering  workshop  for  the  purpose  of 
demonstrating  its  action  and  efficiency  to  his  students. 

The  Principle  of  Moments  applied  to  the  Wheel  and 
Compound  Axle.  —  Taking  moments  about  the  axle,  we  have, 
when  there  is  equilibrium  between  P  and  W, 


The  Principle  of  Work  applied  to  the  Wheel  and  Com- 
pound Axle.  —  Neglecting  friction,  and  supposing  the  rope  to  be 
perfectly  flexible,  cause  the  wheel  to  make  one  complete  revolu- 
tion in  the  direction  shown  by  the  arrow  near  its  circumference 
on  the  end  view. 

Then,  by  the  principle  of  work, 

The  work  put  in  =  the  work  got  out. 
Or,         P  x  its  distance      =  W  x  its  distance  ;  * 
i.e.,        P  x  circumference  =  W  x  J  of  the  difference  of  the  cir- 
of  wheel  cumferences  of  the  larger  and 

smaller  axles.* 

Or,  P  x  27rR  =  W  x  £(27^  -  2irrJ 

(Dividing  both  sides  of  the  equation  by  2r)  — 


Which  is  the  same  result  as  the  one  above  ;  consequently  the 
principle  of  moments  and  the  principle  of  work  agree. 

EXAMPLE  I.  —  In  a  compound  wheel  and  axle,  where  the  weight 
hangs  on  a  single  movable  pulley,  the  diameters  of  the  two  por- 
tions of  the  axle  are  3  and  2  inches  respectively,  and  the  lever 
handle  which  rotates  the  axle  is  12  inches  in  length.  If  a  force 

\v 

*  If  —  iff    raised  the  circumference  of  the  larger  circle  on  one  side, 

W 

then  —  is  lowered  at  the  same  time  on  the  other  side,  the  circumference 

of  the  smaller  axle  ;  consequently  W  will  be  elevated  a  distance  equal  to 
fto//the  difference  of  the  circumferences  of  two  axles,  or=£  (2Trrl-2irr^. 


74  LECTURE  vn. 

of  10  Ibs.  be  applied  to  the  end  of  the  lever  handle,  what  weight 
can  be  raised  ? 

ANSWER.  —  Here  P=  10  Ibs.  ;  R=  12"  ;  rv=  1.5"  and  r,=  i". 
By  the  principles  of  moments  and  of  work  — 


.«.  \V=  10  x  12  x  4  =  480  Ibs. 

EXAMPLE  II.—  In  a  compound  wheel  and  axle,  let  the  diameter 
of  the  large  axle  be  6  inches,  and  that  of  the  smaller  axle  4  inches, 
and  the  length  of  the  handle  20  inches  ;  find  the  ratio  of  the 
velocity  of  the  handle  to  that  of  the  weight  raised. 

ANSWER.—  Here  R  =  20"  ;  r,  =  3"  ;  r,  =  2". 
By  the  principles  of  moments  and  of  work— 


...  p-ifri-v; 

W          R 

P     1(3-2) 

W~        20 

.Stttf  6y  the  principle  of  work — 

P  x  its  distance  =  W  x  its  distance 
i  x  P's  distance  =  40  x  Ws  distance 
.•.  The  velocity  ratio, 

Q  P's   distance  _  40 

Ws  distance      1 

Weston's  Differential  Pulley  Block.— This  practical  appli- 
cation of  the  Chinese  windlass  is  simply  a  compound  axle  without 
the  wheel.  Or,  where  R  =  rr 

Hence,  P  x  R  =  —  (R  _  r) 

2 

where  R  is  the  radius  of  the  larger  axle-  or  pulley,  and  r  the 
radius  of  the  smaller  one.  A  fter  describing  Weston's  differentia] 
pulley  block,  we  will  deduce  this  formula  from  the  "  principle  of 
work  "  by  the  same  kind  of  reasoning  as  we  adopted  in  the  case 
of  the  wheel  and  compound  axle.  We  leave  the  student,  however 
to  apply  the  "principle  of  moments,"  whereby  he  should  get  the 
same  results, 


WESTON'S  DIFFERENTIAL  PULLEY  BLOCK. 


75 


As  will  be  gathered  from  an  inspection  of  the  accompanying  out- 
side view  and  the  small  diagram  showing  the  directions  of  the  forces 
and  their  arms,  it  wrill  be  seen  that  the 
apparatus  consists  of  three  parts — (i) 
an  upper  block  ;  (2)  an  endless  chain ; 
(3)  a  movable  lower  block  or  snatch- 
block.  The  upper  block  has  a  hook 
with  swivel  joint,  from  which  the  iron 
frame  is  suspended.  In  the  centre  of 
this  frame  is  a  turned  steel  axle  on 
which  rotates  a 
couple  of  pul- 
leys cast  in  one 
piece,  and  there- 
fore rigidly  con- 
nected together. 
The  one  pulley 
is  slightly  larger 
than  the  other, 
and  both  pulleys 
have  V-grooved 
peripheries  with 
side  ridges  or 
teeth  cast  on 
the  inner  sides 
of  the  grooves, 
so  as  to  fit  the 

pitch  of  the  links 

*c     ,1          ,    .         SKELETON  FIGURE  OF 
the     chain,   WESTON>S  DIFFERENTIAL 
which        passes  PULLEY  BLOCK. 

over  them    and 

thereby  prevent  it  slipping  over  the 
surface  of  the  pulleys.  The  lower  or 
movable  pulley  is  simply  an  ordinary 
smooth  V-grooved  pulley  with  swivel 
and  hook  like  that  already  described 
under  the  heading  "Snatch  Block." 
The  endless  chain  is  an  ordinary  open- 
linked  chain  of  uniform  pitch  and  size  of  link.  It  passes  from 
the  position  where  the  hand  or  pull,  P,  is  applied,  over  the  larger 
pulley  of  the  upper  block,  underneath  the  lower  pulley,  over  the 
smaller  of  the  upper  block  pulleys,  and  back  to  the  starting-point. 
(See  also  the  small  figure.)  When  a  pull,  P,  is  applied  at  this  part 
of  the  chain  (if  there  were  no  friction),  it  would  be  transmitted  with 
undimimshed  value  throughout  its  whole  length  where  the  tension 


WESTON'S  DIFFERENTIAL 

PULLEY  BLOCK. 
(BY  HOLT  &  WILLETT.) 


76  LECTURE   VII. 

can  act  ;  but,  as  we  shall  see  afterwards,  a  large  proportion  of  this 
force  is  absorbed  in  overcoming  friction.  The  stress  due  to  the 
load  W  is  divided  equally  between  the  two  vertical  parte  of  the 
chain  connected  to  the  lower  block,  and  if  W  is  moved  through  any 

distance,  the  stress  —  must  act  through  double  that  distance. 

The  Principle  of  Work  applied  to  Weston's  Differential 
Pulley  Block  and  Tackle.  —  Theoretically  (i.e.,  leaving  friction 
out  of  account,  the  weight  of  the  hanging  part  of  the  chain  and 
tlie  weight  of  the  lower  block),  we  have  by  the  principle  of  wwk, 
in  one  revolution  of  the  upper  pulleys  — 

P  x  its  distance  =  W  x  its  distance. 

P  x  circumference  of\_W  (difference  of  the  circumferences  of 
the  larger  pulley  J  ~  2  1    the  larger  and  smaller  pulleys, 

Px27rR      =—  (27rR-27rr) 
(Dividing  each  side  of  the  equation  by  2r) 
PxR 


2lv 

(i)  The  Theoretical  Mechanical  Advantage  or  ratio  of  W  to  P 
is  found  directly  from  the  above  equation  by  simple  transposition. 
W       2R 


(2)  The  Velocity  Ratio  (or  ratio  of  the  distance  passed  through 
by  P  to  the  distance  passed  through  by  W  in  the  same  time)  is 
also  found  in  the  same  way. 

•    P'S     distal  27rR  2R* 

"  Ws  distance         i(2n-R  -  2irr)     R  —  r 

Or,  the   velocity   ratio   has  the   same    numerical   value  as  the 
theoretical  advantage. 

EXPERIMENT  I.  —  With  a  Weston's  differential  pulley  block, 
having  in  the  upper  block  one  pulley  with  an  effective  radius  of 
4"  (i.e.,  from  the  centre  of  the  pulley  to  the  centre  of  the  chain 
which  passes  round  it),  and  a  smaller  pulley  with  an  effective 
radius  of  3^",  you  can  just  lift  a  total  load  of  100  Ibs.  (including 
the  dead  weight,  the  lower  block,  and  the  hanging  parts  of  the 
chain)  by  a  pull  of  20  Ibs.  on  the  chain. 

*  Dividing  numerator  and  denominator  by  r  does  not  alter  the  fraction 


WESTON'S  PULLEY  BLOCK  AND  TACKLE,     77 

In  this  case  the  theoretical  advantage  and  the  velocity  ratio  are 
«ach  equal  to  — 

2R          2  x4*        8     16 


Or,  the  pull  on  the  forward  side  of  the  chain  must  act  through 
1  6  ft.  for  every  foot  the  load  is  raised. 

(3)  The  Actual  or  Working  Advantage  of  the  machine  is,  how- 
ever, only  as  — 

W     IPO  Ibs.     6 
P  =  20  Ibs.  =f 

(4)  The  Work  put  in  in  lifting  W  i  ft.  is 

Px  16  =  20  Ibs.  x  i6'  =  320  ft.-lbs. 

(5)  The  Work  got  out  is   =  W  x  i  =  100  Ibs.  x  i'  =  100  ft.-lbs. 

//:\  m     w  -  Work  got  out     100  ft.  Ibs. 

(6)  The  Efficiency  \*        =^  —  r-5  ir=~  =  ---  *  —  n—  =  -3125. 

Work  put  in      320  ft.  Ibs.      •>1*->' 

(7)  The  Percentage  Efficiency  is 

=  .3125x100  =  31.25%. 

This  is  a  very  low  efficiency  for  a  machine,  but  it  accounts  for 
one  of  the  useful  properties  of  the  Weston's  differential  pulley 
block  —  viz.,  that  you  can  lift  a  weight  by  it,  then  let  go  your 
hold  of  the  chain,  and  the  weight  will  remain  hanging  in  the 
exact  position  you  left  it,  without  overhauling  the  chain  in  the 
slightest  degree.  It  is  therefore  an  extremely  useful  appliance 
in  engineering  workshops  where,  for  example,  a  slide  valve  and 
its  valve  casing  port  face  have  to  be  scraped  so  as  to  fit  each  other. 
After  rubbing  the  valve  on  the  port  face,  you  can  lift  the  valve 
by  aid  of  a  Weston's  block,  and  leave  it  hanging,  without  any 
fear  of  its  overhauling  the  chain  which  supports  it,  until  you  have 
scraped  off  the  high  or  hard  parts  from  the  port  face,  when  you 
can  lower  it  for  another  rub.  Or,  in  the  case  of  having  to  adjust 
the  centres  of  a  heavy  job  to  be  turned  in  a  lathe,  you  can  lif  t  the 
job  from  the  lathe  by  a  Weston's  block,  and  leave  it  hanging 
quite  free  at  the  most  convenient  height  to  be  acted  upon,  until 
you  are  ready  to  lower  it  again  into  position.  Of  course,  with 
such  apparatus,  although  the  theoretical  advantage  is  great,  the 
actual  or  working  advantage  is  small  ;  yet  this  property  of  not 
overhauling  is  of  such  importance  that  appliances  possessing  it  are 
constantly  being  used  in  every  engineering  workshop. 

Cause  of  the  Load  not  Overhauling  the  Chain.  —  In  the 
first  place,  the  chain  cannot  slip  round  the  pulleys  of  the  upper 
block,  because  the  links  of  the  chain  fit  into  the  notches  or 


78  LECTURE  m 

between  the  outstanding  teeth  or  ridges  cast  in  their  grooves. 
In  the  second  place,  the  friction  between  the  pulleys  of  the  upper 
block  and  their  axle  is  so  great,  that  more  than  50  %  of  the  "  work 
put  in  "  is  expended  in  overcoming  it. 

To  prove  this,  take  the  case  of  the  above  experiment.  When 
the  pull  P  and  the  load  W  are  both  in  action,  the  downward 
pressure  (due  to  these  two  forces  alone)  between  the  pulleys  and 
their  axle  is  120  Ibs.  (TOO  Ibs.  for  W  4- 20  Ibs.  for  P).  Now, 
remove  the  20  Ibs.  pull,  and  you  only  relieve  the  pressure  causing 
friction  by  £;  for  TOO  Ibs.  (or  £  of  120  Ibs.)  is  still  there.  But 
friction  is  practically  proportional  to  the  pressure  in  such  a  case, 
and  therefore,  although  the  pull  required  to  lift  the  load  be 
removed,  £  of  the  total  friction  will  remain  at  the  upper  block, 
and  the  friction  at  the  lower  block  is  unaltered.  In  lifting  the 
load  of  TOO  Ibs.  i  ft.,  there  was  put  into  the  machine  320  ft.-lbs., 
or  220  ft.-lbs.  was  lost  work,  required  solely  to  overcome  friction. 
Consequently,  to  lower  the  load  of  100  Ibs.  i  ft.  there  would  have 
to  be  expended  at  least  £  of  220  ft.-lbs.,  or  not  less  than  183  ft.-lbs. 
But  the  load  can  of  itself  only  give  out  100  ft.-lbs.  in  descending 
i  ft. ;  therefore  it  must  be  assisted  by  at  least  83  ft.-lbs. 
(183  ft.-lbs.  —  100  ft.-lbs.)  put  into  the  chain  on  the  slack  side,  or 
where  it  comes  down  from  the  smaller  pulley. 

This  principle  of  the  weight  not  running  down  (or  overhauling, 
as  it  is  technically  termed)  is  common  to  all  machines  wherein 
more  than  50  %  of  the  force  applied  is  spent  in  merely  overcoming 
frictional  resistance.* 

*  The  student  should  be  most  earnestly  warned  against  such  expres- 
sions as  the  following,  which  are  only  too  common  in  books  dealing  with 
Applied  Mechanics : — "  By  increasing  the  number  of  sheaves  in  a  pair  of 
pulley  blocks,  the  power  may  be  increased."  Now,  power,  or  the  rate  of 
doing  work,  can  never  be  increased  by  any  continuously  acting  mechanical 
device,  so  long  as  the  work  given  out  depends  directly  on  the  work  put  in. 
It  is  simply  the  force  which  can  be  augmented,  whilst  the  distance 
through  which  it  acts  is  diminished.  Of  course,  in  the  case  of  a  pile- 
driver  where  the  weight  is  lifted  slowly  and  let  go  suddenly,  so  that  the 
rate  of  giving  out  work  is  greater  than  the  rate  of  putting  it  in,  it  is  true 
that  the  power  is  increased.  But  this  is  not  a  continuous  acting  mechani- 
cal device  in  the  sense  referred  to  above.  The  fundamental  principle  to 
be  observed  is,  that  no  more  work  can  be  got  out  than  has  been  put  in. 
The  term  "  mechanical  powers  "  should  also  be  avoided,  and  the  expres- 
sions "  simple  machines  "  or  "  mechanical  elements  "  used  instead. 


LECTURE   VII. — QUESTIONS^  79 


LECTUBB  VII.— QUESTIONS. 

1.  Sketch  and  describe  the  wheel  and  compound  axle,  or  Chinese  wind- 
lass.   Apply  the  "  principle  of  moments  "  and  the  "  principle  of  work  "  to 
find  the  formula  for  the  relation  between  the  force  applied  and  the  weight 
raised  by  this  machine. 

2.  In  a  compound  wheel  and  axle  the  diameter  of  the  two  parts  of  the 
axle  are  5  and  6  inches  respectively.     The  weight  raised,  viz.,  Wy  hangs 
from  a  single  movable  pulley  in  the  usual  manner,  and  is  supported  by  a 
pressure,  P,  applied  perpendicularly  to  a  lever  handle  15  inches  in  length. 
Find  the  ratio  of  P  to  W.    Sketch  and  describe  the  compound  wheel  and 
axle,  and  state  its  inconveniences.    Ans.  i  :  60. 

3.  A  force  of  20  Ibs.  draws  up  W  Ibs.  by  means  of  a  wheel  and  compound 
axle.    The  diameter  of  the  wheel  is  5  feet,  and  the  diameters  of  the  parts 
of  the  compound  axle  are  9  and  1 1   inches  respectively ;  find  W.    Ans. 
1 200  Ibs. 

4.  In  a  compound  wheel  and  axle,  let  the  diameter  of  the  larger  axle  be 
8  inches,  and  the  radios  of  the  smaller  one  2  inches,  while  the  force  applied 
to  the  handle  passes  through  47*12  inches  in  one  revolution.    Find  the 
ratio  of  the  velocity  of  the  handle  to  that  of  the  weight  being  raised. 
Ans.  7-5:1. 

5.  Explain  the  mechanical  principle  upon  which  Weston's  pulley  block 
is  constructed,  and  give  a  skeleton  diagram  showing  the  direction  of  all 
the  forces  at  work.     If  the  weight  which  is  being  raised  is  left  hanging, 
and  the  pull  removed,  why  does  the  weight  not  descend  T 

6.  Sketch  and  describe  Weston's  differential  pulley  block.    If  the  diame- 
ters of  the  pulleys  are  4  and  4}  inches,  what  weight  can  be  raised  by  a 
force  of  20  Ibs.  ?    If  the  weight  to  be  raised  is  half  a  ton,  what  force  must 
be  applied  to  the  leading  side  of  the  chain  ?    (Neglect  friction.)    Ans. 
360  Ibs.  ;  62-2  Ibs. 

7.  Determine  the  relation  between  P  and  W  in  Weston's  differential 
pulley  block — (i)  by  the  "principle  of  moments"  ;  (2)  by  the  "principle 
of  work. " 

8.  If  a  weight  is  raised  by  a  Weston's  differential  pulley  block  at  the 
rate  of  5  ft.  per  minute,  and  the  diameters  of  the  pulley  of  the  compound 
sheaves  are  7  and  8  inches  respectively,  at  what  rate  must  the  chain  be 
hauled  ?    Work  out  answer  in  full  from  the  principle  of  work.    Ans.  80  ft. 
per  minute. 

9.  By  experiment  with  a  Weston's  differential  pulley  block  it  was  found 
that  a  pull  of  15  Ibs.  on  the  leading  side  of  the  chain  was  required  to  lift 
a  weight  of  60  Ibs.  (including  the  weight  of  the  lower  pulley  and  hook). 
The  dimensions  of  the  apparatus  were — radius  of  larger  pulley,  2  inches  ; 
radios  of  smaller  pulley,  175  inches.     Find — (i)  the  theoretical  advan- 
tage ;  (2)  the  actual  or  working  advantage  ;  (3)  the  efficiency  or  modulus  ; 
(4)  the  percentage  efficiency  of  the  apparatus.     Why  does  the  weight 
remain  suspended  when  there  is  no  pull  on  the  chain  T    Ans.  (i)  16  :  i ; 
(2)  4  :  i  ;  (3)  -25 ;  (4)  25. 

10.  In  a  Weston  policy  block,  the  diameters  of  the  two  pulleys  are  8  ms. 
and  7£  ins.  respectively,  and  it  is  found  that  a  pull  of  25  Ib.  is  sufficient 
to  raise  a  weight  of  240  Ib.     Find  the  efficiency  of  the  tackle.     Ans.  30%. 

(C.  &  G.,  1903,  O.,  Sec.  A-) 

11.  Describe  how  you  would  proceed  to  determine  experimentally  (i)  the 
velocity  ratio,  (2)  the  mechanical  efficiency  of  a  Differential  Pulley  Block. 

(B.  of  E.,  1904.) 
I 


(     8o     ) 


LECTURE 

CONTENTS. — Graphic  Demonstration  of  Three  Forces  in  Equilibrium- 
Parallelogram  of  Forces — Triangle  of  Forces — Three  Equal  Forces  in 
Equilibrium — Two  Forces  acting  at  Right  Angles — Resolution  of  a 
force  into  Two  Components  at  Right  Angles — Resultant  of  Two  Forces 
acting  at  any  Angle  on  a  Point — Resultant  of  any  number  of  Forces 
acting  at  a  Point — Example  I. — Stresses  in  Jib  Cranes — Examples 
II.  III. — Stresses  on  a  Simple  Roof— Example  IV. — Questions. 

IN  Lecture  I.  we  explained  and  illustrated  how  a  force  may  be 
represented  by  a  straight  line  both  in  direction  and  magnitude, 
and  we  defined  the  terms  components,  equilibrant,  resultant,  re- 
solution, and  composition  of  forces.  We  will  now  discuss  briefly 
the  case  of  three  forces  in  equilibrium  when  acting  towards  or 
from  a  point,  as  well  as  the  parallelogram  and  the  triangle  of 
forces  with  examples,  before  taking  up  the  inclined  plane  and 
friction. 

Graphic  Demonstration  Of  Three  Forces  in  Equilibrium. 
— EXPERIMENT  I. — Take  a  black  board  which  (for  convenience  of 
handling  and  demonstration  before  a  class)  may  be  of  the  form 
shown  by  the  accompanying  figure.  Select  two  movable  clamps, 
each  fitted  with  a  small  V-grooved  pulley  about  2  inches  in  dia- 
meter, with  a  minimum  of  friction  at  their  bearings,  and  fix  them 
to  the  outside  of  the  board  as  indicated.  Pass  a  very  fine  flexible 
cord  over  the  pulleys,  and  attach  to  the  ends  of  this  cord  S  hooks. 
Hang  from  these  hooks  weights  of  say  24  oz.  and  32  oz.,  and  from 
the  cord  (anywhere  between  the  pulleys)  another  cord  with  an 
S  hook  and  a  weight  of,  say,  40  oz.  After  a  few  up-and-down 
oscillations  these  three  weights  will  come  tp  rest  in  the  definite 
position  shown  by  the  figure,  and  if  you  disturb  them  from  this 
position  they  will  invariably  return  to  it  again.  Consequently, 
you  conclude  that  the  three  forces  acting  from  their  common 
point  of  attachment  are  in  equilibrium,  and  that  the  force  40  oz« 
is  the  equilibrant  of  the  two  forces  24  oz.  and  32  oz. 

*  This  Lecture  may  require  two  meetings  of  a  class  when  the  students 
have  had  no  previous  training  in  Theoretical  Mechanics.  In  any  case,  it 
will  be  well  to  spend  at  least  one  with  a  revisal  hour  before  the  written 
examination,  which  should  now  take  place  upon  the  work  gone  over  since 
the  beginning  of  the  session,  prior  to  the  Christmas  holidays. 


FORCES    IN    EQUILIBRIUM. 


81 


With  a  piece  of  finely  pointed  white  chalk,  draw  lines  (from  the 
point  where  the  three  forces  act)  on  the  black  board  parallel  to  the 
cords,  and  plot  off  from  this  point  to  any  convenient  scale  (say  by 
aid  of  a  two-foot  rule)  distances  along  them  to  represent  their 
respective  magnitudes.  Extend  from  the  same  point  in  an  upward 
vertical  direction  another  line,  and  mark  it  off  to  represent  40  oz. 
This  line  evidently  corresponds,  in  point  of  application,  direction, 
and  magnitude,  to  the  resultant  of  the  components  (24  oz.  and 
•$2  oz.),  for  it  is  equal  and  opposite  in  direction  to  their  equili- 


GRAPHIC  REPRESENTATION  OF  FORCES  IN  EQUILIBRIUM. 

brant.     From  the  extremity  of  this  resultant  draw  lines  joining 
tlie  outer  ends  of  the  components  (24  oz.  and  32  oz.).     Then  you 
a  parallelogram  whoso  adjacent  sides  from  the   point   of 


have 


application,  represent,  both  in  direction  and  magnitude,  the  com- 
ponent forces,  and  whose  diagonal  represents,  also  both  in  direc- 
tion and  magnitude,  their  resultant. 

If  any  other  pair  of  convenient  weights  be  selected  and  applied 
in  the  same  way,  you  can  find  an  equilibrant  and  resultant  for 
them.  From  these  experiments  you  conclude  that  a  general  prin- 
ciple, termed  the  "  parallelogram  of  forces,"  is  true  without  having 
recourse  fa  any  special  mathematical  reasoning. 


LECTUKE   VIII. 


Parallelogram  of  Forces. — If  two  forces,  acting  simultane- 
ously towards  or  from  a  point,  be  represented  in  direction  and 
magnitude  by  the  adjacent  sides  of  a  parallelogram,  then  the  re- 
sultant of  these  forces  will  be  represented  in  direction  and  magnitude 
by  the  diagonal  of  the  parallelogram  which  passes  through  their  point 
of  intersection. 

For  example,  let  any  two  forces,  P  and  Q,  act  from  the  point 
0  at  any  convenient  angle,  say  60°,  then,  if  OA  and  OB  be 

plotted  to  scale  to  represent  these 
forces  in  direction  and  magnitude, 
the  diagonal  OD  of  the  parallelo- 
gram OADB  will  represent  in  direc- 
tion and  to  the  same  scale  their 
resultant  R.  But  the  resultant  E, 
is  equal  and  opposite  in  direction 
to  a  force  E,  which  would  exactly 
balance  the  effect  of  P  and  Q,  or  to 
a  force  represented  in  direction  and 
in  magnitude  by  the  line  DO.  Further,  since  the  side  AD  is  equal 
and  parallel  to  the  side  OB,  it  may  be  taken  to  represent  Q  in 
direction  and  magnitude.  Hence  we  have  the  three  sides  of  the 
triangle  OAD  taken  in  the  order  OA,  AD,  DO,  representing  in 
direction  and  magnitude  three  forces,  P,  Q,  E,  in  equilibrium, 
acting  from  the  point  O.  Hence  we  have  a  general  proposition 
termed  the  "  triangle  of  forces,"  or  a  deduction  from  the  "paral- 
lelogram of  forces." 


Component 

PARALLELOGRAM  OF  FORCES. 


TRIANGLE  OF  FORCES. 


Triangle  of  Forces. — If  three  forces  acting  towards  or  from  a 
point  are  in  equilibrium,  and  a  triangle  be  drawn  with  its  sides 


TRIANGLE    OF   FORCES.  83 

respectively  parallel  to  those  forces  taken  in  due  order,  then  the 
forces  will  be  represented  to  scale  by  the  sides  of  the  triangle. 

CONVERSELY  :  —  If  three  forces  acting  towards  or  from  a  point 
are  represented  in  direction  and  to  scale  by  the  sides  of  a  triangle 
taken  in  due  order,  these  three  forces  are  in  equilibrium. 

For  example,  let  the  three  forces  P,  Q  and  E  act  from  the 
point  0,  and  be  in  equilibrium.  Draw  a  triangle  with  its  sides, 
P,  Q,  E,  respectively  parallel  to  these  forces  ;  then  the  sides  of 
this  triangle,  taken  in  that  order,  represent  to  the  same  scale  these 
forces.  Or,  if  the  triangle,  whose  sides  are  respectively  P,  Q  and 
E,  represent  in  direction  and  to  scale  the  three  forces  P,  Q  and 
E,  as  they  act  from  a  point  0,  these  forces  are  in  equilibrium. 
We  have  shown  by  a  dotted  line  the  resultant  R,  and  its  direc- 
tion as  opposed  to  E,  by  the  same  side  of  the  triangle. 

It  is  quite  evident  that  if  the  forces  P,  Q  and  E  acted 
towards  the  point  0,  instead  of  from  it,  the  triangle  P,  Q,  E 
would  still  represent  these  forces  in  magnitude,  but  the  direction 
of  all  the  arrows  would  have  to  be  pointed  the  opposite  way. 

SPECIAL  CASES.  —  Three  Equal  Forces  in  Equilibrium.  —  It  can 
easily  be  proved  by  the  apparatus  used  for  Experiment  I.,  or  by  construe 
tion,  that  if  you  have  three  equal  forces  in  equilibrium  they  must  act  at 
120°  from  each  other,  and  that  the  triangle  representing  their  directions 
and  magnitudes  will  be  an  equilateral  triangle,  or  a  triangle  whose  angles 
are  each  equal  to  60°. 

Two   Forces   acting  at   Hight   Angles.  —  In  this  case  it  can  be 

E  roved  by  the  same  apparatus,  or  by  Euclid,  Book  I.  Prop.  47,  that  any  two 
Drees  P  and  Q,  acting  at  right  angles  to  each  other,  have  a  resultant  K,  or 
arc  balanced  by  a  third  force  E,  of  such  magnitude  that  — 


Consequently,  if  you  hare  any  two  forces  in  the  proportion  of  3  to  4 
acting  at  right  angles  to  each  other,  their  resultant  will  proportionate!/ 
be  5. 

For,  suppose  P  =  3a,  Q=4<z,  where  a  is  any  number  of  units  of  force. 
Then,  R 


.•.  K= 
.*.  P: 

Or,  P:Q:R  =  3:4:5. 

Conversely,  if  any  two  forces  in  the  proportion  of  3  to  4  units  are 
balanced  by  a  third  force  proportionately  of  5  units,  the  forces  3  and  4 
must  be  acting  at  right  angles  to  each  other. 


84 


LECTURE    VIII. 


Resolution  of  a  Force  into  Two  Components  at  Right  Angles 
to  each  other.* — Let  R  be  the  force  to  be  resolved,  P  and  Q  the  com- 
ponents, and  let  K  make  an  angle,  0,  with  the  force  Q. 


Then  . 

And  . 

Also    . 


.    R .  Cos  9  =  Q  ;  f  or  Cos  0  =  § 

R 

.    E  .  Sin  0  =  P  ;  for  Sin  d  =  ? 

B-— =i=Tane 


Resultant  of  Two  Forces  acting  at  any  Angle  on  a  Point. — 

The  proof  of  this  general  case  must  be  left  to  the   Author's  Advanced 
Treatise  on  Applied  Mechanics,  but  the  formula  may  be  given  here,  viz.  : 

R2  =  P2  +  Q2  +  2P  x  Q  Cos  a 

where  P  and  Q  are  any  two  forces,   R  their  resultant,  and  a  the  angle 
between  the  directions  of  the  forces  P  and  Q. 

If  P=Q,  then— 

R2  =  P2  +  P2  +  2P2  Cos  a  =  2P2  +  2P2  Cos  a 

or,  R2  =  2P2  (i  +  Cos  a) 


/.  R  =   2P  Cos  2. 

2 


=  4P2  Cos  - 
(Since  Cos  a  =  2  Cos2--!) 


Resultant  of  any  Number  of  Forces  Acting  at  a  Point.— Let 
3j,  P,2,  Pa,  &c.,  be  any  number  of  forces  acting  at  a  point  ;  then,  by  the 
parallelogram  of  forces  find  a  resultant,  R,, 
for  P,  and  Pa  ;  and  a  resultant,  Ey  for  R,  and 
Ps ;  and  so  on.  The  last  resultant  will  be  the 
resultant  of  all  the  forces. 

Example  I. — Forces  3,  5  and  7  units  act 
from  a  central  point  O  at  equal  angles.  Find 
the  resultant. 

AKSWEB. — Let  OA,  OB  and  OC  represent 
the  forces  in  direction  and  magnitude.  Then 
you  can  follow  out  the  above  rule  and  find  a 
resultant  for,  say,  3  and  5— call  this  R,  ;  and 
finally  find  a  resultant  for  R,  and  7.  But  it  is 
obvious  that  you  may  subtract  3  units  from 
each  OA  them  without  affecting  the  result, 
since  the  forces  are  acting  at  equal  angles 
from  each  other.  This  will  destroy  one  of 
them,  and  leave  OBj  to  represent  2  units, 
and  OC,  to  represent  4  units.  Then,  by  the 
parallelogram  of  forces  you  find  the  resultant 
R  =  3'5  units. 

*  The  reverse  of  this  may  be  applied  to  the  composition  of  two  or  more 
forces  acting  at  a  point  in  one  plane,  but  we  will  leave  the  demonstration 
of  such  problems,  as  well  as  that  of  the  polygon  of  forces,  to  our  Advanced 
Book  on  Applied  Mechanics, 


STRESSES   IN   JIB    CRAVES. 


Stresses  in  Jib  Cranes. — As  a  practical  example  of  the  appli- 
cation of  the  "  triangle  of  forces,"  take  the  case  of  an  ordinary 
hand-worked  jib  crane.  The  load  is  suspended  from  the  hook 
H  of  the  snatch-block  SB ;  or,  in  the  case  of  a  crane  for  lifting 
light  loads  quickly,  to  a  simple  hook  with  a  swivel  attached  directly 


Note. — See  under  page  heading  "  Notes 
and  Questions "  at  the  end  of  this 
Lecture,  a  Fig.  and  description  of  Butters 
Brothers  and  Co.'s  PATENT  NEW  JIB 
ARRESTER. 


HAND-WORKED  JIB  CRANE. 
INDEX  TO  PARTS. 


J  represents  Jib. 
JP         „          Jib  pulley. 
SB         „          Snatch-block. 

H  Hook. 


CP  represents  Central  post. 

F          „          Framing. 
WG          „          Wheel  gear. 

C         „         Chain. 
TR         „          Tie-rods. 

to  the  end  shackle  of  the  chain  C,  as  it  comes  down  from  the  jib 
pulley  JP,  instead  of  the  chain  passing  round  a  snatch-block 
pulley,  and  up  to  an  eyebolt  near  the  point  of  the  jib. 


86 


LECTURE    VIII. 


1 i)  The  load  produces  a  tension  on  the  chain  C. 

(2)  A    thrust   along    the  jib   J,    from   the  jib  pulley  to  the 
eye-bolt   connecting   the   shoe  of  the  jib   to   the  bottom  of  the 
framing  F. 

(3)  A  tension  in  the  tie-rods  from  the  top  of  the  framing  to 
their  connection  with  the  top  of  the  jib. 

(4)  This  tension  on  the  tie-rods  produces  a  horizontal  pull, 
tending  to  bend  and  break  the  crane-post  CP,  where  it  leaves  the 
upper  foundation  plate-bearing  and  joins  the  framing.     Cranes 
for  heavy  lifts  require  a  back  balance  weight  to  counteract  this 
force.     (See  the  third  figure  in  Lecture  XIII.) 

(5)  It  also  causes  an  upward  pull,  tending  to  unship  or  lift  the 
crane-poet  from  its  bearings  in  the  upper  and  lower  foundation 
plates. 

The  directions  and  values  of  these  stresses  will  be  better  under- 
stood by  the  student  after  considering  a  particular  example. 

EXAMPLE  II. — In  a  hand-worked  jib  crane  of  the  form  shown 
by  the  above  figure,  the  length  of  the  jib  is  30  feet,  the  lengths  of 
the  tie  rods  are  25  feet  each,  and  the  vertical  distance  between 
the  attachments  of  the  tie-rods  and  of  the  jib  to  the  framing,  is 
12  feet.  Find  the  stresses  produced  on  these  parts  of  the  crane 


FRAME  ANP  STRESS  PIAGRAV  FOR  A  JIB  GRANS. 


v  STRESSES  IN   JIB    CRANES.  87 

by  a  load  of  10  tons  hung  from  the  hook,  neglecting  all  other 
stresses  produced  by  the  weight  of  the  several  parts  of  the 
crane. 

ANSWER. — First,  draw  a  "frame  diagram,"  or  figure  to  scale, 
representing  the  directions  and  the  lengths  of  centre  lines  of  the 
various  parts  under  stress.  As  shown  by  the  frame  diagram  of 
the  accompanying  upper  figure,  AB  represents  the  1 2  feet  vertical 
distance  between  the  foot  of  the  jib  and  inner  ends  of  the  tie-rods 
marked  post,  BC  represents  the  30  feet  jib,  AC  the  25  feet  tie- 
rods,  and  CW  the  lo-ton  load — all  to  the  same  scale. 

Now,  it  is  evident  from  an  inspection  of  this  figure  that  the  load 
W  causes — 

(1)  A  vertical  downward  tension  on  the  chain  from  C  to  W. 

(2)  A  thrust  or  compression  along  the  jib  from  C  to  B,  which 
produces  an  equal  and  opposite  reaction  from  the  framing  at  B 
along  the  jib  from  B  to  C. 

(3)  A  tension  on  the  tie-rods  from  A  to  C. 

(4)  This  tension  on  the  tie-rods  may  be  resolved  into  a  hori- 
zontal pull  or  force  from  A  towards  the  direction  of  W,  tending 
to  bend  or  break  the  poet  about  B. 

(5)  Also,  a  vertical  upward  pull  or  force  in  the  post  from  B 
toward^  A. 

The  student  should  mark  the  directions  of  these  various  forces 
by  arrowheads  on  his  frame  diagrams. 

Second,  draw  a  "  stress  diagram,"  viz.,  ab,  vertical  and  to  a  con- 
venient scale,  to  represent  the  downward  force  of  the  jo-ton  load 
on  the  point  C  ;  be,  parallel  to  the  reaction  along  the  jib  from  B  to 
C ;  and  ca,  parallel  to  the  tension  in  the  tie-rods  from  G  to  A. 

Then,  by  "  the  triangle  of  forces,"  since  we  have  three  forces 
acting  from  the  point  C  (viz.,  load,  reaction  along  jib,  and  tension 
in  tie-rods)  in  equilibrium,  and  since  we  have  drawn  a  triangle  with 
its  sides  respectively  parallel  to  these  forces  taken  in  due  order  ; 
the  forces  will  be  represented  to  one  scale  by  the  sides  of  this 
triangle.  Consequently,  ab  represents  the  load  to  scale  ;  be  the 
reaction  along  the  jib;  and  ca  the  tension  in  the  tie-rods.  Now, 
this  tension  on  the  tie-rods  may  be  resolved  into  vertical  and 
horizontal  components  by  the  method  already  described  in  this 
Lecture  ;  therefore,  a  vertical  line,  ad,  represents  the  vertical 
component  or  upward  pull  on  the  post,  and  dc  the  horizontal  pull 
on  the  same,  both  in  direction  and  to  the  same  scale  as  ab  repre- 
sents the  load.  By  applying  the  scale  to  which  ab  has  been 
drawn  to  represent  10  tons  (viz.,  yV'to  i  ton),  be  shows  25  tons; 
ca,  20-7  tons  (which  would  be  10-35  tons  on  eac*1  tie-rod  if  they 
were  parallel  to  each  other,  but  more  if  inclined  from  the  jib-head 
to  the  outside  of  the  framing) ;  cd,  20  tons;  and  adt  4  tons. 


88 


LECTURE   VIII. 


Force 
Diagram 


VC 


TENSION  IN  THE  TIE-ROD 
AND  THRUST  IN  THE  JIB 
OF  A  CRANE. 


EXAMPLE  III. — In  a  common  crane  the  jib  is  15  feet  long,  and 
the  tie-rod  1 2  feet.  The  tie-rod  is  attached  to  the  crane  post  at  a 
point  5  feet  above  the  foot  of  the  jib.  If  a  weight  of  6  tons  be 
hung  from  the  point  of  the  jib,  find  the  tension  in  the  tie-rod  and 
the  thrust  in  the  jib. 

ANSWER. — First  draw  the  frame  diagram  as  explained  in  Ex- 
ample II.,  marking  the  lengths  of  the  parts  by  dotted  lines  and 

arrow-heads.  (The  student  in  his 
diagrams  should  also  mark  the  direc- 
tions of  the  stresses.) 

Second,  on  the  line  of  action  of 
the  weight  W  draw  ca  to  scale  to 
represent  the  direction  and  magni- 
tude of  the  weight,  6  tons.  Then 
draw  cb  parallel  to  the  jib,  and  ab 
parallel  to  the  tie-rod.  The  triangle, 
cab,  represents  by  its  sides  to  one 
scale  the  magnitudes  of  the  forces 
— viz.,  14.4  tons  tension  in  the  tie- 
rods  and  1 8  tons  thrust  or  reaction 
in  the  jib.* 

Stresses  on  a  Simple  Roof. — 

EXAMPLE  IV. — The  weight  on  each  principal  of  a  simple 
triangular  roof  is  i  ton.  Find  the  stresses  on  the  points  of 
support  and  in  the  S€sveral  members  of  the  principal. 

ANSWER. — First,  draw  a  frame  diagram  of  the  principal,  where 
AB  and  AC  represent  the  direction  and  length  of  the  rafters, 
and  BO  represents  the  tie-beam. 

Then,  the  whole  weight  may  be  supposed  to  act  in  a  vertically 
downward  direction,  AW,  from  the  junction  of  the  rafters  through 
the  middle  of  the  tie-beam.  This  weight  naturally  produces  a 

W 

pressure  at  B  and  at  0  of  — .     It  also  produces  at  these  points 

reactions  Kx  and  K2,  each  equal  to  — ,  since  the  whole  is  sym- 
metrically balanced  about  the  central  vertical  line  AW.  Further, 
there  is  a  stress  of  compression  along  the  rafters  in  the  directions 
AB  and  AC,  and  consequently  an  equal  and  opposite  reaction 

*  We  have  purposely  used  the  letters  ABC  and  abc  differently  placed 
from  the  previous  figures  in  Example  II.,  and  have  drawn  the  stress  dia- 
gram in  a  different  position,  in  order  to  teach  the  student  that  he  must  not 
depend  upon  his  memory  with  regard  to  letters,  but  upon  a  clear  under- 
standing of  the  "triangle  of  forces."  Students  should  draw  their  frame 
and  stress  diagrams  to  as  large  a  scale  as  their  exercise  books  will  admit. 


STRESSES    IN    JIB    CRANES. 


89 


along  those  members  from  B  to  A,  and  from  C  to  A.  Also,  there 
is  an  equal  tension  on  the  tie-beam  from  its  centre  towards  B  and 
towards  C. 

Second,  draw  the  stress  diagram  for  the  three  forces  that  are  in 
equilibrium  at  the  bearing  C  (viz.,  the  vertical  downward  pressure 

W 

— ,  the  horizontal  tension  along  the  tie-beam  and  the  reaction  along 

the  rafter  from  C  to  A)  by  plotting  DC  as  a  vertical  line  to  scale  to 

W 

represent  — ,  or  10  cwt.,  and  drawing  DE  parallel  to  AC,  and 

producing  the  tension  on  the  tie-beam  until  it  meets  this  line  DE. 
Therefore,  the  other  sides  of  the  triangle  DCE  represent  in 


R 


cwt. 


FEAME  AND  STEESS  DIAGBAMS  OP  A  SIMPLE  ROOF. 

direction  and  to  the  same  scale  as  DC  represents  10  cwt. ;  the  ten- 
sion on  the  tie-beam  by  CE,  equal  to  loj  cwt. ;  and  the  reaction 
along  the  rafter  by  ED,  equal  to  1 5  cwt. 

In  a  precisely  similar  manner  the  stresses  at  the  bearing  B  may 
be  found  by  the  "  triangle  of  forces." 

We  will  leave  the  more  complicated  questions  in  graphic  statics 
to  our  book  on  the  Advanced  Stage  of  Applied  Mechanics,  since 
we  believe  the  elementary  or  tirst  year's  student  will  find  that 
what  has  been  included  in  this  Lecture  is  sufficient  to  enable  him 
to  understand  what  will  be  brought  before  him  in  the  future 
Lectures  of  this  book,  as  well  as  prepare  him  for  answering  the 
various  problems  which  are  likely  to  be  asked  of  him. 


90  LECTURE  VIII. — QUESTIONS. 

LECTURE  vm. — QUESTIONS. 

1.  State  the  principle  of  the  parallelogram  of  forces,  and  explain  how  you 
would  prove  the  truth  thereof  by  experiment.     A  vertical  force  of  50  Ibs. 
is  balanced  by  two  forces  of  30  Ibs.  and  40  Ibs.     Find  their  directions  and 
the  angle  between  them. 

2.  Represent  the  point  of  application,  the  direction  and  the  magnitude 
(to  a  scale  of  ^  inch  to  a  pound)  of  the  following  forces  : — 10  Ibs.  acting 
northwards,  15  Ibs.  acting  eastwards,  20  Ibs.  acting  southwards,  and  25  Ibs. 
acting  westwards,  all  from  one  point.     Find  their  resultant,  and  its  direc- 
tion.    Ans.  14*14  Ibs.  acting  south-west. 

3.  State  the  principle  of  the  triangle  of  forces.     Three  forces,  P,  Q  and 
R,  act  from  or  towards  a  point,  and  are  in  equilibrium.     Show  graphically 
how  you  would  represent  their  magnitude  and  direction  by  the  three  sides 
of  a  triangle  taken  in  order.     Explain  the  converse  of  this  question. 

4.  Two  ends  of  a  piece  of  cord  are  fastened  to  two  nails  in  a  wall  8  ft. 
apart  in  a  horizontal  line.     The  cord  is  10  feet  in  length,  and  has  a  knot 
4  ft.  from  one  end,  from  which  point  a  weight  of  25  Ibs.  is  suspended. 
Find  by  construction  the  stresses  on  the  nails,  and  indicate  their  direction 
by  arrows.     A ns.  22'$  Ibs. ;  17-5  Ibs. 

5.  Show  how  to  resolve  a  force  into  two  components  at  right  angles  to 
each  other.     A  force  of  100  Ibs.  acts  at  (ist)  30°,  (2nd)  45°,  (3rd)  60°,  (4th) 
75°  to  the  horizontal.     Find  by  construction  the  vertical  and  horizontal 
components    for    each    case,    and    prove    your    results    by    calculation. 
Ans.    Vertical   components  1—50    Ibs. ;    707  Ibs.  ;     86'6   Ibs. ;     96*6   Ibs. 
Horizontal  components  : — 86 '6  Ibs.  ;  70*7  Ibs.  ;  50  Ibs.  ;  25-8  Ibs. 

6.  Sketch  an  ordinary  hand-worked  jib-crane.     Explain  its  action  by  an 
index  to  parts,  and  show  how  the  various  stresses  due  to  a  load  on  the 
chain  act,  by  aid  of  a  frame  and  a  stress  diagram.     Nine  tons  is  hung 
from  the  end  of  the  jib  of  a  crane,  which  is  inclined  to  the  horizontal  at 
an  angle  of  60°.     If  the  compression  on  the  jib  is  16  tons,  find  by  frame 
and  stress  diagrams  the  tension  on  the  tie-rod.     Ans.  9-4  tons. 

7.  In  a  crane,  show  the  method  of  estimating  the  tension  of  the  tie-rod 
and  thrust  on  the  jib  when  a  given  weight  is  hung  from  the  end  of  the  jib 
If  the  load  =  6  tons,  and  the  tension  of  the  tie-rod  (which  makes  an  angle 
of  60°  with  the  vertical)  =  18  tons,  find  by  a  diagram  drawn  to  scale  the 
thrust  on  the  jib.     Ans.  21  '6  tons. 

8.  In  a  common  crane  the  jib  is  30  ft.  long  and  the  tie-rod  24  ft.     The 
tie-rod  is  attached  to  the  crane-post  at  a  point  10  ft.  above  the  foot  of  the 
jib.     If  a  weight  of  10  tons  be  hung  from  the  point  of  the  jib,  find  by  con- 
structing a  frame  and  a  stress  diagram — (a)  the  tension  on  the  tie-rod  ; 
(b)  the  thrust  on  the  jib;  (c)  the  horizontal  pull  on  the  post ;  (d)  the  up- 
ward pull  on  the  same.     Ans.  (a)  24  tons ;  (b)  30  tons  ;  (c)  2i-2  ;  (d)  ii'2 
tons. 

9.  A  symmetrical  pair  of   steps,  hinged  together  at  the  top  and  con- 
nected together  by  a  string  at  the  bottom,  stands  on  a  smooth  horizontal 
plane.     If  the  length  of  each  side  be  3  feet  3  inches,  and  the  string  be 
3  feet  in  length,  find  the  tension  o£  the  string  when  a  person  of  140  Ibs.  in 
weight  stands  on  the  steps  at  a  height  of  2  feet  from  the  ground  ?     How 
is  the  tension   of  the  string  affected  as  the  person  ascends  the  steps  ? 
Ans.  24-8  Ibs.    When  the  person  is  at  the  very  foot  of  the  steps  the 


LECTURE    VIII. — QUESTIONS.  Ql 

tension  =o;  when  he  is  at  the  top,  the  tension  is  a  maximum  of  nearly 
36-25  Ibs. 

10.  A  rectangular  trap-door  measuring  4  feet  square  and   weighing 
75  Ibs.  is  hinged   with    one   edge    horizontal,  and   is  supported   in  the 
horizontal  position  by  a  chain  which  is  connected  with  the  middle  point 
of  the  outer  edge  of  the  trap-door,  and  with  a  point  vertically  over  the 
middle   point   of   the  edge  in  which  the   hinges  are  fixed,  but   7   feet 
above  it.     Sketch  the  arrangement,  and  determine  the  tension  upcn  the 
chain  and  the  reaction  on  the  hinges.     Ana.  T  =  K  =  43  Ibs. 

(I.C.E.  Feb.  1902.) 

11.  A  load,  W,  of  2000  Ibs.  is  hung  from  a  pin,  P.,  at  which  pieces  AP 
and  BP,  meet  like  the  tie  and  jib  of  a  crane.     The  angles  WPB  and  WPA 
are  30°  and  60°  respectively.     Show  by  a  sketch  how  to  find  the  forces  in 
AP  and  BP.     Distinguish  as  to  a  piece  being  a  strut  or  a  tie.     Ant,  Strut, 
3464  Ibs,  ;  tie,  2000  Ibs. 

12.  Two  pieces  in  a  hinged  structure  meet  at  a   pin,  and  a  load  is 
applied  at  the  pin.     Show  how  we  find  the  pushing  or  pulling  forces  in  the 
pieces.    Describe  an  apparatus  which  enables  your  method  to  be  illus- 
trated. 

13.  The   weight  of   a  chain  hanging  from  two  points  of  support  is 
500  Ibs.     Its  inclinations  to  the  horizontal  at  the  points  of  support  are  30* 
and  50°  respectively  ;  what  are  the  tensions  at  the  points  of  support  f 
Ans.  326-4  Ibs.  ;  440  Ibs. 

14.  The  weight  of  a  chain  hanging  from  two  points  of  support  is 
220  Ibs. ;  its  inclinations  to  the  horizontal  at  the  points  of  support  are  25° 
and  42°  respectively ;  what  are  the  tensions  in  the  chain  at  the  points  of 
support?     Ana.  ^=177 '6  Ibs.  ;  T^=2i6'6  Ibs. 

15.  A  chain  weighing  800  Ib.    is  hung  from  its  two  ends,  which  are 
inclined  to  the  horizontal  at  40°  and  60°  respectively.    What  are  the 
forces  in  the  chain  at  the  points  of  suspension  ?    Ans.  400  and  600  Ibs. 

(B.  of  E.,  1903.) 

16.  In  a  lo-ton  crane  the  jib  is  35  ft.  long,  the  tie  30  ft.  long,  and  the 
crane  post  15  ft.  high.     Neglecting  the  effect  of  the  tension  in  the  chain, 
obtain  the  longitudinal  forces  in  the  tie  and  jib     Ans.  Stress  in  tie  = 
20  tons  ;  stress  in  jib  =  23£  tons.  (C.  &  G.,  1903,  O.,   Sec.  B.) 

17.  A  machine  5  tons  in  weight  is  supported  by  two  chains  ;  one  of  these 
goes  up  to  an  eyebolt  in  a  wall  and  is  inclined  20°  to  the  horizontal  ;  the 
other  goes  up  to  a  roof  principal  and  is  inclined  73°  to  the  horizontal ;  find 
the  pulling  forces  in  the  chains.     You  may  use  a  graphical  or  any  other 
method  of  calculation  you  please.    Ans.   1*47  and  47  tons. 

(B.  of  E.  1904.) 

1 8.  A  cord  is  wrapped  six  times  round  four  parallel  rods,  which  pass 
through  the  four  corners  of  a  rectangle,  and  are  perpendicular  to  the  plane 
of  the  rectangle  ;  the  long  sides  of  the  rectangle  are  2^  times  the  length 
of  the  short  sides.    The  cord,  while  being  wrapped  round  the  rods,  is  kept 
taut  with  a  uniform  tension  of  7^  pounds.     Find  graphically,  or  in  any 
other  way,  the  force    exerted  by  the  cord  on  any  one  of  the   rods. 
Ans.  Force  on  any  one  rod  =  64  Ibs.  (B.  of  E.,  1905.) 

19.  A  roof  trnss  consists  of  two  rafters,  equally  inclined,  connected  by  a 
horizontal  tie  at  a  distance  of  a  feet  below  the  apex.     If  the  total  span  be 
I  feet  and  the  load  carried  at  the  apex  be  w  tons,  show  that  the  tension  in 
the  tie  rod  is  wL^a  tons.  (C.  &  GM  1904,  0.,  Sec.  B.) 


92  NOTES   AND   QUESTIONS. 

Many  fatal  accidents  have  taken  place  due  to  the  falling  of 
the  jibs  of  hand  and  steam  worked  cranes.  The  following 
simple  device  has  been  recently  applied  by  a  Glasgow  firm  with 
the  object  of  avoiding  such  accidents. 


1909  PATENT  JIB  ARRESTER,  BY  BUTTERS  BROTHERS  &  Co. 
GLASGOW. 

In  the  event  of  any  of  the  jib  gear  or  of  the  jib  rope  of  the 
crane  breaking,  the  spring  A  contracts,  thus  relieving  the  rope 
B,  which  holds  the  clutch  C  in  position.  The  two  springs  at  the 
point  of  the  jib  marked  D  then  contract,  and  pull  the  clutch  C 
inwards.  This  catches  the  lift  rope  E,  on  the  pulley  at  the  point 
of  the  jib  F,  thus  holding  up  the  jib  and  preventing  it  from 
falling,  as  well  as  keeping  the  load  "W  in  position. 


(     93     ) 


LECTURE  IX. 

CONTENTS.— Inclined  Planes— The  Inclined  Plane  without  Friction— 
When  the  Force  acts  Parallel  to  the  Plane— Example  I.— When  the 
Force  acts  Parallel  to  the  Base — Example  II. — When  the  Force  acts 
at  any  Angle  to  the  Inclined  Plane — Example  III. — The  Principle  of 
Work  applied  to  the  Inclined  Plane — Example  IV. — Questions. 

Inclined  Planes, — An  inclined  plane  is  a  plane  surface  inclined 
to  the  horizontal,  whereby  a  certain  force  may  be  used  to  raise  a 
greater  weight  to  a  desired  height  than  could  be  done  by  applying 
it  directly  to  elevate  the  weight  vertically.  Inclined  planes  are 
also  used  for  easing  down  weights  with  less  retarding  force  than 
would  be  necessary  to  lower  them  vertically.  In  another  form, 
called  the  wedge,  inclined  planes  are  employed  for  splitting  bodies, 
or  different  parts  of  the  same  body,  asunder,  as  in  the  case  of  the 
steel  wedge  used  by  the  woodman  to  split  up  logs  for  firewood  and 
other  purposes.  Wedges  are  also  used  for  forcing  bodies  together, 
and  for  fixing  them  tightly  in  a  desired  position  ;  or  for  elevating 
them  through  a  small  distance,  as  in  the  case  of  the  levelling  of 
the  heavy  cast-iron  sole-plate  of  an  engine.  And  further,  as  we 
shall  have  occasion  to  prove,  the  well-known  screw,  in  whatever 
form  it  may  be  ap^jted,  is  simply  an  inclined  plane  of  a  particular 
shape. 

The  Inclined  Plane  without  Friction. — In  the  first  place, 
we  will  consider  the  inclined  plane  with  a  body  placed  thereon  and 
kept  in  position  by  a  force  applied  to  the  body,  when  all  friction 
between  the  plane  and  the  body  is  supposed  to  be  absent  or  negligible 
— i.e.,  both  the  plane  and  the  body  are  assumed  to  be  perfectly 
smooth.  There  are  three  cases  of  this  statical  problem. 

(i)  When  the  force  supporting  the  body  acts  parallel  to  the 
inclined  plane. 

(2)  When  the  force  acts  horizontally. 
3)  When  the  force  acts  at  any  angle  to  the  plane. 
Case  I. — Let  the  force  P  act  parallel  to  the  plane,  and  let  the 
accompanying  figure  represent  a  vertical  section  through  the  plane 
and  the  e.g.  of  the  body.  Let  a  be  the  e.g.  of  the  body;  W  its  weight, 
acting  vertically  downwards  along  the  line  aW ;  P  the  necessary 
pull  (to  keep  the  body  in  position)  applied  along  the  line  «P, 


94 


LECTURE    IX. 


parallel  to  the  plane  AB ;  and  R  the  reaction  from  the  plane  (due 
to  the  weight  of  the  body  resting  thereon),  acting  along  the  line 
aR,  at  right  angles  to  the  plane. 

Also,  let  the  length  of  the  plane  AB  be  indicated  by  I ;  its 
height,  BC,  by  h  ;  its  base,  AC,  by  b  ;  and  the  angle  of  the  plane 
to  the  horizontal  by  a. 

Now,  by  the  "  triangle  of  forces,"  since  we  have  three  forces, 
W,  P  and  R,  acting  at  a,  the  eg.  of  the  body,  and  since  these 
forces  are  in  equilibrium,  if  we  construct  a  triangle  whose  sides 


INCLINED  PLANE,  CASE  1. 
WHEN  P  ACTS  PARALLEL  TO  PLANE  AB. 

are  parallel  to  these  forces,  they  will  represent  them  in  direction 
and  in  magnitude. 

Therefore,  plot  off  along  the  line  aW  a  distance  «6,  to  repre- 
sent the  weight  of  the  body  W,  to  any  convenient  scale.  From,  b, 
draw  a  line  be  parallel  to  P,  and  from,  a,  extend  the  direction  of 
R  to,  c,  by  the  line  ac. 

Then,  W  :  P  :  R  :  :  ab  :  be  :  ca 

But  by  Euclid  the  triangle  abc  is  similar  to  the  triangle  ABC. 
.-.     ab:  be  :  ca  : :  AB  :  BC  :CA 
AB:BC:CA::     I    :    h  :   b 
R  ::     I    :   h   :   b 


W  :    P 


And, 
Consequently, 

Or, 


Or, 

Precisely  the  same  results  will  be  arrived  at  if  (as  shown  by  the 
right-hand  side  of  the  figure)  we  considered  the  vertical  side  BC 


P 

W 

h             R     b           ,   P         h 

=  7'       w=7;  andn  =;  6 

P 

-   Sin  a  ;    5   =   Cos  a 

p 

;    and  —  =  Tan  a 

W 

W 

R 

INCLINED    PLANES.         -  95 

of  the  triangle  ABC  as  representing  W,  and  then  have  drawn  a 
line  from  C  on  the  direction  of  AB,  parallel  to  R.  It  will  form 
a  useful  exercise  for  the  student  if  in  every  case  he  will  plot  down 
both  methods,  and  mark  along  the  sides  of  the  triangle  of  forces 
the  respective  forces  which  they  respectively  represent. 

EXAMPLE  I. — A  weight  of  100  Ibs.  is  supported  on  a  smooth 
inclined  plane  by  a  force  P,  acting  parallel  to  the  plane.  If  the 
incline  be  i  in  10,  find  P,  and  give  the  reasoning  by  which  you 
establish  the  result. 

ANSWER. — Draw  a  figure  exactly  the  same  as  tnat  accom- 
panying Case  1.  and  mark  W=  100  Ibs.,  1=  10,  and  h  =  i.  Then 
by  the  "  triangle  of  forces  "  : 

P_       BC  *  * 

W  =  AB  I       =  To 

P   =  W  -L  -  .12?   =  10  Ibs. 

10  10 

Case  2. — Let  the  force  P  act  parallel  to  the  base,  with  the  same 


INCLINED  PLANE,  CASE  2. 
WHEN  P  ACTS  PARALLEL  TO  BASE  AC. 

signification  for  each  of  the  forces  and  parts  of  the  inclined  plane, 
and  the  same  assumptions.    Then  plot  off  ere,  along  aW,  to  represent 
W  ;  draw  cb  parallel  to  P,  and  extend  the  direction  of  K  back- 
wards  along  06,  until  it  meets  cb  at  the  point  6 
Then  W  :  P  :  R  : :  ac  :  cb  :  ba. 

But  by  Euclid  the  triangle  acb  is  similar  to  the  triangle  ACB. 

.-.    ac    :  cb  :   ba  ::AC:CB  :  BA 
And,  AC  :CB:  BA::    b    :   h  :    I 

G 


96 

LECTURE   IX. 

Consequently, 

W    :  P  !   R 

I 

Or,                    | 

_  *. 

R 

W 

=      -  ;  and 

P 

R  ~ 

Or, 

P 

<Sxr 

=  Tan 

a'>  w 

=  Seoo;  and-£  =  i 

Precisely  the  same  results  will  be  arrived  at  if  (as  shown  by 
the  right-hand  side  of  the  figure)  we  considered  the  vertical  side 
BC  of  the  triangle  ABC  as  representing  W,  and  then  draw  a  line 
from  C  parallel  to  R,  and  a  line  BD,  parallel  to  P,  to  meet  the 
line  CD. 

EXAMPLE  II. — A  force  of  100  Ibs.  is  supported  on  a  smooth 
inclined  plane  by  a  force  P  acting  parallel  to  the  base.  If  the 
incline  be  i  in  10,  find  P. 

ANSWER. — Draw  a  figure  exactly  the  same  as  that  accom- 
panying Case  2,  and  mark  W  =  100  Ibs.,  1=  10,  and  h  =  i. 

Then,  by  the  "  triangle  of  forces  "  : 

JP  __  CB       h  i  i  ri 

W 


A<J 


100 


^99     9-95 
=  10-05  Ibs. 


9'95  9*95 

Case  3. — Let  the  force  P  act  at  any  angle  6  to  the  inclined  plane 
AB.  With  the  same  signification  for  each  of  the  forces  and  parts 
of  the  inclined  plane,  and  the  same  assumptions,  plot  off  from  a, 


INCLINED  PLANE,  CASE  a.' 

"WHEN  P  ACTS  AT  ANY  ANGLE  TO  PLANE 


INCLINED  PLANKS.  97 

along  the  line  aW,  a  distance  am,  to  any  convenient  scale  to 
represent  the  weight  of  the  body  W.  From  this  point,  m,  draw 
a  line  mn  parallel  to  P,  and  .extend  the  direction  of  R  backwards 
to  meet  this  line.  This  small  triangle,  amn,  will  be  a  "  triangle 
of  forces,"  for  W,  P  and  R,  which  are  in  equilibrium  about  the 
e.g.  of  the  body  at  a. 

But  in  this  case  the  student  will  probably  realise  the  proof  of 
the  problem  more  easily  if  he  considers  BC  as  representing  to 
scale  the  weight  W,  and  then  draws  CD  parallel  to  R,  and  DB 
parallel  to  P, 

When  W  :  R  :  P  :  :  BC  :  CD  :  DB, 

or  the  triangle  BCD  is  the  "  triangle  of  forces,"  representing  the 
forces  W,  R  and  P  in  direction  and  magnitude  by  the  sides  BC, 
CD  and  DB  respectively. 

P     DB     R=CD.       dP_55 
Or'  W  =  BC'W     BC'  R~CD 

If  we  resolve  the  force  P  (which  acts  at  the  angle  6  to  the  in- 
clined plane)  parallel  to  the  plane,  then  we  can  treat  the  com- 
ponents of  P  exactly  in  the  same  way  as  we  did  the  simple  force 
P  in  Case  1. 

If  we  resolve  P  into  the  direction  of  R,  then  this  component 
acts  with  R,  and  is  evidently  balanced  by  the  resolved  part  of  W 
in  the  same  direction — i.e.,  along  the  line,  an. 

EXAMPLE  III. — A  weight  of  100  Ibs.  is  supported  on  a  smooth 
inclined  plane  by  a  force  P,  acting  at  60"  in  an  upward  direction 
from  the  inclined  plane.  If  the  incline  be  i  in  10,  find  P. 

ANSWER. — Draw  a  figure  exactly  the  same  as  that  accompanying 
Case  3,  and  mark  W=  100  Ibs.,  1=  10,  A=  i,  and  6  =  60*. 

Then  by  the  "  triangle  of  forces,"  BC  represents  W,  and  DB 
represents  P  to  scale.  Measuring  their  respective  lengths  we  get 

P  =  W??  =100  H  =20  Ibs. 

BC  ioo 

Principle  of  "Work  applied  to  tLe   Inclined  Plane. — 

Referring  to  the  figure  for  Case  1,  let  the  body,  whilst  under 

the  action  of  the  three  forces  W,  P  and  R,  be  moved  the  whole 

length  of  the  incline.     Therefore  P  acts  from  A  to  B,  and  at  the 

same  time  W  acts  through  a  vertical  height  CB.     Consequently, 

neglecting  friction  as  before,  we  have  by  the  "  principle  of  work  " — 

The  work  put  in  =  The  work  got  out 

P  x  its  distance  =  W  x  its  distance 

PxAB  =  WxCB 


98  LECTURE   IX. 

But  this  is  precisely  the  same  result  as  we  got  by  applying  the 
principle  of  the  "triangle  of  forces."  Hence,  the  "principle  of 
work"  agrees  with  the  "triangle  of  forces"  in  respect  to  the 
inclined  plane. 

Cases  2  and  3  may  be  treated  by  the  student  in  exactly  the 
same  way,  and  the  correct  results  will  be  the  same  as  those  found 
by  the  "  triangle  of  forces." 

-  EXAMPLE  IV.  —  An  inclined  plane  is  used  for  withdrawing 
barrels  from  a  celJar  by  securing  two  ropes  to  the  top  of  the 
incline  at  B,  then  passing  them  down  the  incline,  half  round 
the  barrel,  and  up  to  the  horizontal 
platform  at  the  top  of  the  incline,  where 
two  men  pull  on  the  ropes  in  a  direction 
parallel  to  the  plane.  If  the  weight,  W, 
of  the  barrel  is  200  Ibs.,  the  length,  Z,  of 
the  incline  20  ft.,  and  the  height  10  ft., 
find,  by  the  principle  of  work,  the  least 
force  which  must  be  exerted  by  the  two 
RAISING  BARRELS  BY  men?  and  the  work  expended,  neglecting 


--------  5 


?NCLUINED  PLA^E.  AN°    f™*™**  in  drawing  the  barrel  from  the 

cellar. 

Let  the  accompanying  figure  represent  a  vertical  cross  section 
through  the  middle  of  the  barrel  and  the  inclined  plane.     Then  a 
statical  force,  P,  applied  at  the  e.g.  of  the  barrel,  would  just  balance 
its  weight,  W,  and  the  reaction  from  the  plane  (not  shown). 
By  the  principle  of  work,  neglecting  friction  — 
The  work  put  in  =  The  work  got  out. 
P  x  its  distance  =  W  x  its  distance. 

?xl  =  Wxh. 

P=W  7  =  200^°'  -i  oo  Ibs. 

I  20 

But  by  passing  the  rope  round  the  barrel,  as  explained  in  the 
question,  this  force  P  is  halved  on  the  ropes  (see  Lecture  VI.  on 
the  pulley  and  snatch-block).  Therefore  the  least  force  which 
the  two  men  must  exert  in  order  just  to  move  the  barrel  wiH  be  — 

£.122.60  bs. 

2  2 

But  this  force  acts  through  a  distance  2^  =  40'  ft.  ;  therefore  the. 
work  expended  will  be  — 

p 

—  x  2  2  =  50  Ibs.  x  40'  =  2000  ft.-lbs. 

Or,  work  got  out  =?  W  *  /?  =F  299  Ibs.  x  10'  =  20OO  ft.-lbs. 


INCLINED   PLANES.   _;  ;  99 

In  this  question  we  have  a  combination  of  the  pnlley  and  the 
inclined  plane.  The  inner  ends  of  the  two  ropes  being  fixed  at 
the  top  of  the  inclined  plane,  the  force  with  which  the  men  act 
on  the  free  ends  is  communicated  throughout  the  ropes,  so  that 
the  stress  in  the  ropes  on  each  side  of  the  barrel  balances  the 
force  P,  that  would  be  required  to  move  the  barrel  up  the  incline 
if  applied  at  its  centre  of  gravity. 

Or,  the  theoretical  advantage  due  to  the  pulley  part  of    the 

.          .  P       2P       2 

system  is,      .         .         .     p  =  ^r  =  - 


Then  for  the  inclined  plane  part  we  have  by  the  "  triangle  of 
forces,"  or  by  the  "  principle  of  work,"  a  theoretical  advantage  of  — 


Therefore,  the  total  theoretical  advantage  is  the  product  of  the 
two  separate  advantages,  viz.  — 

2P       W_2        2       4 

P  XP"  =  7XI=I 

Consequently,  a  force  of  i  Ib.  applied  at  the  free  end  of  the  rope 
would  balance  a  weight  of  4  Ibs.  on  the  incline.  Or,  as  in  the 
question,  and,  neglecting  friction,  a  barrel  weighing  200  Ibs.  requires 
a  pull  of  50  Ibs.  to  move  it  up  the  inclined  plane. 

We  have  simply  split  up  the  total  advantage  in  this  way  to 
show  the  student  that  the  combined  advantages  of  the  several 
parts  of  a  compound  machine  must  equal  the  advantage  of  the 
whole.  We  might  have  said  at  once,  as  we  have  done  before  in 
other  cases  — 

The  Theoretical  Advantage  =  p-  =  -  =  y 

NOTE.  —  I  have  this  day  (Sept.  9,  1892)  witnessed  the  interesting  opera- 
tion of  lowering  four  very  large  25-ton  steam  boilers  of  the  marine  type, 
down  an  incline  of  about  100  feet  in  length  by  the  method  described  in  the 
foregoing  question.  One  man,  by  aid  of  an  ordinary  block  and  tackle, 
supplied  the  requisite  restraining  force  on  the  free  end  of  the  rope. 


100  LECTURE  IX. — QUESTIONS. 

LEOTUBE  IX. — QUESTIONS. 

1.  Prove  by  the  triangle  of  forces  (drawn  to  scale)  the  relation  between 
the  weight  W  of  a  body  resting  on  a  smooth  inclined  plane,  the  reaction, 
R,  from  the  plane,  and  the  force,  P,  necessary  to  just  balance  the  weight — 
(i)  when  the  force,  P,  acts  parallel  to  the  plane  ;  (2)  when  it  acts  parallel 
to  the  base ;  (3)  when  it  acts  at  an  angle,  6,  to  the  plane. 

2.  A  ball,  weighing  100  Ibs.,  rests  on  an  inclined  plane,  being  held  in 
position  by  a  string  which  is  fastened  to  a  bracket  so  as  to  be  parallel  to 
the  plane.    The  height  of  the  plane  being  £  of  the  length,  find  the  tension 
of  the  string  and  the  pressure  perpendicular  to  the  plane.    Establish  your 
results  by  reasoning  on  known  principles,  such  as  the  principle  of  work  or 
that  of  the  parallelogram  of  forces.     Am.  P  =  33'3  Ibs.,  and  B=94*3  Ibs. 

3.  Prove  the  relation  between  W,  P,  and  R,  acting  on  a  body  resting  on 
a  smooth  inclined  plane  by  the  "principle  of  work"  for  cases  1,  2  and  3 
in  this  Lecture.     An  incline  is  i  ft.  in  vertical  height  for  15  in  length.     A 
weight  of  100  Ibs.  rests  on  the  plane  and  is  held  up  by  friction  ;  make  a 
diagram  for  estimating  the  pressure  on  the  plane,  and  find  its  amount. 
Ans.  99'7  Ibs. 

4.  Friction  being  neglected,  find  the  force,  acting  parallel  to  the  plane, 
which  will  support  i  ton  on  an  incline  of  i  ft.  vertical  and  10  ft.  along  the 
incline.     Prove  the  formula  which  you  employ.     If  the  incline  were  i  ft. 
vertical  and  280  ft.  along  the  incline,  find  the  force  in  pounds  which  would 
support  i  ton.  Ans.  224  Ibs.,  and  8  Ibs. 

5.  A  smooth  incline  plane  has  a  vertical  side  of  i  ft.,  and  a  length  of 
10  ft.  ;  what  work  is  done  in  pulling  10  Ibs.  up  8  ft.  of  the  incline  1    Am. 
8  ft. -Ibs. 

6.  When  a  body  is  raised  through  a  given  height,  how  is  the  work  done 
estimated  ?    A  body  weighing  8  cwt.  is  drawn  along  100  ft.  up  an  incline, 
which  rises  2  ft.  in  height  for  every  5  ft.  along  the  incline  ;  the  resistance 
of  friction  being  neglected,  find  the  work  done.     Ans.  35,840  f t.-lbs. 

7.  A  smooth  incline  is  8  ft.  long,  and  the  total  vertical  rise  from  the 
bottom  to  the  top  thereof  is  2  ft.     What  amount  of  work  is  performed  in 
drawing  a  weight  of  100  Ibs.  up  4  ft.  of  the  incline,  and  what  is  the  least 
force  which  will  do  this  work  ?    Ans.  100  ft.-lbs. ;  25  Ibs. 

8.  Friction  is  neglected,  and  it  is  found  that  a  force  acting  horizontally 
will  move  10  Ibs.  up  5  ft.  of  an  incline  rising  i  in  4.    Find  the  work  done, 
and  find  also  the  force  parallel  to  the  plane  which  will  just  support  the 
weight  of  10  Ibs.     Ans.  12*5  ft.-lbs.  ;  2*5  Ibs. 

9.  A  car  laden  with  20  passengers  is  drawn  up  an  incline,  one  end  of 
which  is  1 60  ft.  above  the  other  ;  the  car,  when  empty,  weighs  3  tons,  and 
the  average  weight  of  each  passenger  is  140  Ibs.      Find  the  number  of 
fU-lbs.  of  work  done  in  ascending  the  incline,  neglecting  friction. 
Ans.  1,523,200  ft.-lbs.,  or  680  ft. -tons. 

10.  It  will  be  observed  that  draymen  sometimes  lower  heavy  casks  into 
cellars  by  means  of  an  inclined  plane  and  a  rope.     One  end  of  the  rope  is 
secured  to  the  upper  end  of  the  inclined  plane,  and  is  then  passed  under 
and  over  the  cask,  the  men  holding  back  by  means  of  the  loose  end.    Now, 
supposing  the  incline  to  be  at  an  angle  of  45  degrees,  explain  the  mechanical 
principles  that  are  here  applied,  and  find  the  advantage.    Ans.  21/2:1. 

1 1.  A  barrel  weighing  5  cwt.  is  lowered  into  a  cellar  down  a  smooth  slide 
inclined  at  an  angle  of  45  degrees  with  the  vertical.    It  is  lowered  by  means 
of  two  ropes  passing  under  the  barrel,  one  end  of  each  rope  being  fixed, 
while  two  men  pay  out  the  other  ends  of  the  ropes.    What  pull  in  Ibs. 
must  each  man  exert  in  order  that  the  barrel  may  be  supported  at  anj 
point  ?    -<*  Ant.  99  Ibs.  nearly. 


LECTURE  X. 

CONTENTS. — Friction — Heat  is  Developed  when  Force  overcomes  Friction 
— Laws  of  Friction — Apparatus  for  Demonstrating  First  and  Second 
Laws  of  Friction — Experiment  I. — Example  I. — Angle  of  Repose  or 
Angle  of  Friction— Experiment  II.— Diagram  of  Angles  of  Repose — 
Limiting  Angle  of  Resistance — Experiment  III. — Apparatus  for 
Demonstration  of  the  Third  Law  of  Friction — Experiment  IV. — 
Lubrication — Anti-Friction  Wheels — Ball  Bearings — Work  done  on 
Inclines,  including  Friction — Example  II. — Questions. 

Friction. — Whenever  a  body  is  caused  to  slide  over  another 
body,  an  opposing  resistance  is  at  once  experienced.  This  natural 
resistance  is  termed  friction.*  The  true  cause  of  friction  is  the 
roughness  of  the  surfaces  in  contact.  The  smoother  the  sliding 
surfaces  are  made  the  less  will  be  the  friction.  Friction  cannot, 
however,  be  entirely  eliminated  by  any  known  means,  for  even 
the  most  microscopical  protuberances  on  the  smoothest  of  sur- 
faces seem  to  fit  into  corresponding  hollows  on  other  equally 
smooth  places,  so  that  some  force  is  required  to  make  the  one 
body  slide  over  the  other. 

Friction  has  its  advantages  as  well  as  its  disadvantages.  For 
example,  if  it  were  not  for  friction  we  could  not  walk,  neither 
could  a  locomotive  start  from  a  railway  station,  nor  could  it  be 
brought  to  rest  in  the  usual  speedy  manner.  Friction  is  also 
essential  to  the  utility  of  nails,  screws,  wedges,  driving  belts,  <fec. 
On  the  other  hand,  power  is  often  expended  in  overcoming 
friction  with  the  result  of  much  wear  and  tear  in  machinery. 
For  example,  in  the  case  of  working  the  slide  valves  of  locomotive 
engines  as  much  as  twenty  horse-power  is  required  in  moving 
these  essential  parts  when  running  at  full  speed,  f 

It  is  the  duty  of  the  engineer  to  reduce  friction  to  a  minimum 
in  the  case  of  the  bearings  of  engines,  shafting,  and  machines 
generally,  in  order  that  a  minimum  of  work  may  be  expended  in 
moving  them.  He  has,  however,  also  to  devise  means  of  pro- 
ducing a  maximum  of  friction  in  the  case  of  certain  pulleys,  grips, 
clutches,  brakes,  and  such  like  appliances,  where  motion  has  to 
be  transmitted  by  aid  of  friction,  or  bodies  in  motion  (such  as  a 

*  French  writers  call  friction  a  passive  resistance,  because  it  Is  only 
apparent  when  one  body  tends  to  move  or  pass  over  another. 

f  See  the  Author's  "Elementary  Manual  on  Steam  and  the  Steam 
Engine,"  page  182,  for  an  arithmetical  example. 


102  LECTUKE    X. 

moving  train)  have  to  be  brought  to  rest  quickly  when  nearing  a 
station. 

Heat  is  Developed  when  Force  overcomes  Friction. — 
When  a  body  is  kept  moving  by  a  force,  part  (or  in  certain 
cases  it  may  be  the  whole)  of  the  mechanical  force  is  expended  in 
overcoming  fractional  resistance.  This  lost  work  is  directly  trans- 
formed into  heat  in  the  act  of  overcoming  the  frictional  resistance 
through  a  distance.  For  example  : — A  person  slips  down  a  vertical 
rope  by  holding  it  between  his  hands  and  his  legs.  The  force  of 
gravity  impels  him  downwards,  overcoming  the  frictional  resist- 
ance between  his  hands  and  limbs  and  the  rope,  with  the  conse- 
quence that  they  become  severely  heated,  especially  if  he  happens 
to  slip  down  quickly.  A  boy  takes  a  run,  and  then  slides  along  a 
level  piece  of  ice.  The  foot-pounds  of  work  stored  up  in  him  just 
before  he  begins  to  slide  are  expended  partly  in  overcoming  the 
frictional  resistance  between  the  soles  of  his  boots  and  the  ice, 
and  partly  in  the  frictional  resistance  between  his  clothes  and  the 
air.  As  a  consequence,  he  will  find  that  by  the  time  he  gets  to 
the  end  of  the  slide  his  soles  are  considerably  warmed.  If  the  ice 
were  perfectly  level,  infinitely  long,  and  if  there  were  absolutely 
no  friction  between  it  and  his  boots,  and  if  there  were  no  fric- 
tional resistance  between  him  and  the  air,  then  he  would  slide  on 
for  ever  1  If  we  could  diminish  the  frictional  resistance  between 
the  skin  of  a  ship  and  the  water,  and  between  the  exposed  parts 
of  the  ship  and  the  air,  to  nothing,  then  all  that  would  be  required 
to  transport  her  across  the  Atlantic  would  be  a  strong  force 
applied  at  the  start  until  she  attained  the  desired  speed,  when  she 
would  proceed  forward,  and  arrive  at  her  destination  with  undi- 
minished  velocity  !  In  reality,  however,  we  find  it  necessary  to 
employ  steam  engines  of  10,000  horse-power  continuously  in  order 
to  propel  an  Atlantic  "  greyhound "  of  5000  tons  at  twenty 
knots  in  the  calmest  of  weather.  About  one-half  of  this  power 
is  absorbed  in  overcoming  the  frictional  resistance  of  the  ship 
through  the  water  and  air,  and  the  other  half  in  the  frictional 
and  other  losses  due  to  the  working  of  the  propelling  machinery. 
Examples  of  the  conversion  of  mechanical  work  into  heat  are 
so  familiar  to  you  all,  being  in  fact  brought  prominently  before 
your  notice  every  day  of  your  existence,  that  we  need  not  further 
enlarge  upon  this  question  except  to  remind  the  student  of 
Dr.  Joule's  discovery  of  the  rate  of  exchange  between  heat  and 
work.  He  found  by  experiment  that  if  work  is  transformed  into 
heat,  every  772  ft.-lbs.  of  work  will  produce  i  heat  unit,  or  that 
quantity  of  heat  which  would  raise  i  Ib.  of  water  i  °  Fahr.* 

*  For  further  examples  and  an  explanation  of  Dr.  Joule's  experiments 
?ee  the  Author's  Treatise  on  Steam  and  the  Steam  Engine. 


FRICTION. 


103 


Laws  of  Friction. — From  1831  to  1 8 34  General  Morin  carried 
out  an  extensive  series  of  experiments  at  Metz  on  friction  for 
plane  surfaces,  with  different  areas,  pressures  and  velocities,  from 
which  he  arrived  at  certain  conclusions.  These  conclusions  were 
for  a  long  time  regarded  as  constituting  the  fundamental  laws  of 
friction.  They  have  been  since  proved  to  beonly  true  within  the 
limits  of  his  experiments,  for,  they  do  not  hold  good  for  great 
pressures  and  high  velocities,  neither  are  they  true  for  fluid, 
rolling,  or  axle  friction.  For  the  latest  and  most  reliable  experi- 
ments we  must  refer  to  the  Proc.  of  the  Inst.  of  Mechanical 
Engineers,  1883,  lS85>  1888,  and  1891. 

ist  Law.  Friction  ia  directly  proportional  to  the  pressure  between 
two  surfaces,  if  they  remain  in  the  same  condition. 

2nd  Law.  Friction  is  independent  of  the  areas  in  contact. 

3rd  Law.  Friction  is  independent  of  velocity. 

Apparatus  for  Demonstrating  First  and  Second  Laws  of 
Friction. — Nevertheless,  it  will  be  both  interesting  and  instruc- 
tive to  students  to  have  these  three  laws  demonstrated  by  the 
following  simple  apparatus  : — 


G.P 


APPARATUS  FOR  DEMONSTRATING  THE  FIRST  AND  SECOND  LAWS 
OP  FRICTION. 


INDEX  TO  PARTS. 

IP  represents  Inclined  plane.  I      B  represents  Box  for  planes. 

GP         „          Guide  pulley.  PP          „          Pinching  pin. 

Q         „          Quadrant.  |    LL         „          Legs. 


IO4  LECTURE   X. 

The  inclined  plane  IP  is  fitted  with  a  joint  at  its  left-hand 
end,  and  after  slackening  the  pinching  pin  PP,  it  may  be  raised 
to  any  desired  angle  or  fixed  in  a  level  position  by  tightening  the 
pin.  The  desired  position  is  found  by  reading  off  the  angle  oppo- 
site the  plumb-ball  line  on  the  graduated  degree  scale  of  the 
quadrant  Q.  In  the  box  B  may  be  kept  planes  of  glass,  brass, 
iron,  steel,  &c.,  as  well  as  the  different  kinds  of  wood  to  be  experi- 
mented upon.  These  planes  are  fixed  on  IP,  in  a  central  position, 
by  means  of  a  catch,  and  the  bodies  to  be  laid  upon  them  should 
be  fitted  with  a  small  hook  opposite  their  e.g.,  to  which  a  fine 
flexible  silk  cord  can  be  attached  and  passed  over  the  guide- 
pulley  GP,  which  should  turn  very  freely  on  its  bearings.*  The 
pull  P  is  best  effected  by  attaching  to  this  cord  a  small  tin  pail 
into  which  shot  may  be  dropped  one  by  one  until  the  body  moves 
freely  on  the  plane.  The  pail  and  shot  may  then  be  unhooked 
and  weighed  in  a  balance. 

Demonstrations  of  the  First  Law  of  Friction. — EXPERI- 
MENT  I. — Fix  the  inclined  plane  IP  in  a  truly  horizontal  posi- 
tion.    Take  from  the  box  B, 
say,  a  long  strip  of  planed 
yellow  pine  and  a  small  block 
of  the  same  kind  of  wood, 
and  let   its  weight   be   W. 
Adjust  the  strip  along  the 
middle  line  of  IP  by  means 
PEOOF  OF  FIRST  LAW  OF  FRICTION.        of  the  sneck  or  catch,  and 

place  the  block  therein.  At- 
tach the  silk  thread  to  the  hook  on  the  forward  side  of  the  block, 
and  pass  the  same  over  the  practically  f  rictionless  pulley.  Hang  a 
little  tin  pail  from  the  free  end  of  the  silk  thread,  and  drop  small 
shot  one  by  one  into  the  pail  until  the  block  moves  freely  over  the 
yellow  pine  strip  when  aided  by  a  little  tapping  on  the  table. 
Unhook  the  pail  containing  the  shot,  and  weigh  it  as  carefully  as 
you  weighed  the  block  of  yellow  pine.  Let  it  equal  P  units. 

Then  P  is  the  force  which  just  overcomes  the  directly  opposing 
passive  resistance,  called  friction,  between  the  surface  of  the  yellow 

*  The  guide-pulley  bracket  should  be  fitted  with  a  stiff  joint  and  with  a 
telescope  arm,  so  that  the  pulley  may  be  raised  or  lowered  in  order  to 
bring  the  direction  of  the  pull  P  on  the  cord  parallel  to  the  plane,  or 
parallel  to  its  base,  or  adjusted  to  any  desired  angle  with  respect  to  the 
plane,  in  order  to  demonstrate  Cases  i,  2  and  3  of  the  inclined  plane  in 
Lecture  IX.  By  having,  say,  a  £"  slot  along  the  middle  of  the  plane,  and 
by  lowering  the  pulley,  Case  2,  wherein  the  pull  on  the  body  is  parallel  to 
the  base,  may  be  readily  demonstrated;  and  by  pulling  out  the  telescope 
arm  of  the  bracket,  and  turning  up  the  bracket,  Case  3,  wherein  the  pull 
makes  an  angle,  0,  with  the  plane,  may  be  verified.- 


FRICTION.  105 

p 

pine  block  and  strip ;  and  the  ratio  ^  is  termed  the  co-efficient  of 

friction.  Now  put  another  block  of  weight  W  on  the  top  of  the  one 
just  tested,  so  as  to  double  the  pressure  on  the  sliding  surface,  and 
put  in  shot  until  the  block  moves  when  aided  by  a  little  vibration, 
so  as  to  overcome  the  greater  resistance  to  starting  the  body  in 
motion  than  to  keep  it  moving.*  You  find  on  weighing  the  pail 
and  shot  that  it  is  now  2?.  Consequently  the  co-efficient  of 

2p  P 

friction  has  not  altered,  for  — =^-  is  the  same  fraction  as  ^. 

EXAMPLE  I. — Suppose  you  take  a  very  small  block  of  woocf 
(say  i/io"  thick,  2"  long  and  i"  broad;  in  fact,  so  light  that  its 
weight  is  negligible),  and  place  a  i-lb.  weight  on  the  top  of  it ;  you 
will  find  that  575  oz.  are  required  to  cause  motion  of  this  piece 
of  wood  over  the  surface  of  the  yellow  pine  strip.  You  therefore 
conclude  that  the  co  -efficient  of  friction  is 

P       c-7C  oz. 
ft=  Wi6oz,   =  *353'  °r  friction=  '353  W. 

Now,  place  a  2-lb.  weight  on  the  upper  piece  of  wood,  and  you 
find  that  it  requires  more  shot  in  the  pail  to  move  it.  Weigh  the 
pail  and  the  shot  again  just  after  you  have  obtained  free  move- 
ment of  the  one  bodf  over  thi  other,  and  you  will  fi*4  that  it 
amounts  to  11.5  oz. 

P      11.5 

Consequently,  ^  =  — -  —  -353  as  before. 

If,  however,  you  put  a  lo-lb.  weight  on  the  upper  piece  of 
wood,  you  will  obtain  a  different  result,  thus  proving  the  first  law 
and  the  variation  therefrom;  because  in  this  latter  case  the 
pressure  is  so  great,  compared  with  the  first  and  second  experi- 
ments, that  the  grains  of  the  upper  piece  of  wood  enter  those  of 
the  lower,  and  bring  into  play  another  condition  of  affairs — viz., 
the  gripping  action  of  the  one  set  of  grains  on  the  other  set.  If 
you  had  taken  a  large  plank  of  yellow  pine,  weighing,  say,  100  Ibs., 
and  had  placed  it  on  another  similar  plank,  the  co-efficient  of 
friction  would  have  a  certain  value.  If  you  had  even  put  a  loo-lb. 
weight  on  the  upper  plank,  the  co-efficient  of  friction  might  not 
have  varied  perceptibly.  But  if  you  placed  a  weight  of  1000  Ibs. 

*  Statical  friction,  or  the  friction  of  repose,  is  that  resistance  which 
opposes  the  commencing  of  the  motion.  If  a  body  be  allowed  to  rest  on 
another  for  some  time,  it  requires  more  force  to  move  it  than  if  it  had  only 
been  stationary  for  a  few  seconds. 


106  LBCTtJKB  X. 

on  the  upper  plank,  the  co-efficient  of  friction  would  be  con- 
siderably altered.  Hence  you  observe  that  this  first  law  only 
holds  good  between  narrow  limits.* 

Angle  of  Repose,  or  Angle  of  Friction.  —  EXPERIMENT  II.  — 
Another  way  of  proving  the  first  law  of  friction  is  to  disconnect 
the  silk  thread  and  the  shot-pail  from  the  upper  body,  and  tilt  up 
the  inclined  plane  to  such  an  angle,  a,  with  the  horizontal  that, 
(with  the  aid  of  a  little  tapping)  the  weighted  block  of  yellow 
pine  just  slides  slowly  down  the  incline.  Here  we  have  simply 
the  force  of  gravity  acting  on  the  body  and  overcoming  friction. 
At  the  moment  the  body  just  begins  to  slide  we  have  the  weight,  W, 
of  the  body  acting  vertically  downwards,  R  the  reaction  from  the 
plane  at  right  angles  to  the  surface,  and  F,  the  passive  resistance 
of  friction,  acting  parallel  to  the  plane  in  the  direction  of  aP 
in  the  first  figure  in  this  Lecture.  Now,  these  three  forces  act 
from  the  e.g.  of  the  body,  and  they  are  in  equilibrium.  R  is  equal 
to  the  resolved  part  of  W  at  right  angles  to  the  plane  (or  R  = 
W  Cos  a),  and  it  represents  the  pressure  between  the  surfaces. 
F  is  the  resolved  part  of  W,  parallel  to  the  plane  (or  F  =  W  Sin  a). 

TT 

and  ^  is  the  co-efficient  of  friction. 

XV 

F  _  W  Sin  a  _  m  _  h  _  height  of  plane 

"  R      W  Cos  a  b     base  of  plane 

The  angle  a,  to  which  the  plane  must  be  inclined  before  the 
free  body  will  slip  over  the  fixed  one,  has  been  termed  the  "  anyle 
of  repose"  or  "  angle  of  friction" 

Therefore,  ike  tangent  of  the  angle  of  repose  is  equal  to  the  co- 
efficient of  friction. 
p 

But  —  was  proved  by  the  previous  experiment  to  be  also  equal 

to  the  co-efficient  of  friction, 

P         F  h 

*  '  W   -   R   -  1 

Or,  P    *=     /iW  and     F 


*  Sir  Robert  Stawell  Ball,  when  Professor  of  Mechanism  at  the  Royal 
College  of  Science,  Ireland,  tried  a  careful  experiment  in  the  above  way 
with  a  smooth  horizontal  surface  of  pine  72"  x  1  1",  and  a  slide,  also  of  pine, 
9"x9"  grain  crosswise.  He  loaded  and  started  the  slide,  and  applied  a 
force  sufficient  to  maintain  it  in  uniform  motion,  and  he  found  that  on 
increasing  the  load  from  14  to  112  Ibs.,  by  increments  of  14  Ibs.,  the  co- 
efficient of  friction  diminished  from  -336  to  -262.  From  these  experiments 
he  constructed  the  empirical  formula  for  this  case  that  F  =  '9  +  -266  R, 
where  F  is  the  factional  resistance  and  R  the  reaction  from  the  surface  or 
net  load- 


FRICTION. 


107 


where  fi  is  the  Greek  letter  universally  adopted  to  represent  co- 
efficients of  friction. 

The  accompanying  figure  is  a  diagram  of  the  "  angles  of  repose  ' 
for  various  common  materials,  together  with  the  numerical 
values  of  or  their  co-efficients  of  friction. 


Jbrlcate£ 
ro      rtO~ 

A.NGLES  OP  REPOSE. 

Limiting   Angle   of  Resistance,  or    Sliding   Angle. — A 

third  way  of  proving  the  first  law  of  friction  is  to  place  the  bodies 
so  that  the  sliding  surface  is  perfectly  level.  Then  begin  by 
pressing  the  upper  body  through  the  intervention  of  a  compres- 
sion spring-balance  fitted  with  a  sharp  point,  so  that  it  will  not 
slip  off,  and  with  a  clinometer  to  indicate  the  angle  through  which 
it  is  tilted  away  from  the  perpendicular.  Now  gradually  incline 
your  pressure  to  the  perpendicular,  until  you  arrive  at  such  an 
angle  as  will  just  cause  the  upper  body  to  slide  over  the  under 
one.  This  angle  is  termed  the  "sliding  angle"  or  "  limiting  angle 
of  resistance"  because  it  is  the  limit,  or  maximum  angle  which 
the  reaction  from  the  surface  can  make  with  the  perpendicular  to 
the  surfaces,  for  the  reaction  must  act  in  the  directly  opposite 
direction  to  the  pressing  force.*  Again,  apply  the  spring- balance, 
but  with  double  the  registered  pressure,  and  you  can  just  incline 
this  force  to  the  same  angle  as  before.  If,  however,  you  pre&f. 
with  ten  times  the  former  force,  you  would  probably  be  able  to 
act  at  a  greater  angle  than  before.  It  will  be  seen  from  this 
experiment  that 

The  Limiting  Angle  of  Res-istance  =  The  Angle  of  Repose. 

*  Here  the  weight  of  the  tipper  body  is  supposed  to  be  negligible  in 
Comparison  with  the  inclined  pressure  upon  jtt 


io8 


LECTURE   X. 


PKOOF  OF  SECOND  LAW  OF  FRICTION. 


Demonstration  of  the  Second  Law  of  Friction. — Ex* 
PERIMENT  III. — Take  a  block  of  planed  yellow  pine,  and  cut  it  into 
two  equal  pieces  at  right  angles  to  the  planed  surface.  Place  one 
piece  on  the  horizontal  strip  of  yellow  pine  (used  in  previously 
demonstrating  the  first  law),  with  the  planed  side  next  to  it, 

and  put  the  other  piece  on 
the  top  of  it,  as  shown  by 
the  second  figure  in  this 
lecture.  Now  ascertain  the 
horizontal  pull,  P,  required 
to  overcome  friction.  Then 
attach  the  top  piece  to  the 
bottom  one,  as  shown  by  the 
accompanying  figure,  so  that 

the  area  of  the  surface  in  contact  is  doubled,  and  you  will  find 
that  the  same  horizontal  force,  P,  will  cause  it  to  move.  If 
you  take  a  long  planed  block  and  cut  it  into  ten  equal  pieces, 
each  of  the  same  size  as  one  of  the  above  pieces,  and  try  the 
experiment  in  a  similar  manner,  you  will  be  able  to  increase 
the  area  of  contact  tenfold,  and  you  will  then  find  that  the 

p 
ratio  ^  is  not  exactly  the  same  with  the  surface  of  one  block 

in  contact  with  the  strip,  as  when  the  surface  of  the  whole  ten 
came  into  action  at  once.  The  result  of  increasing  the  area  in 
contact  may  also  be  tried  by  placing  the  blocks  on  the  inclined 
plane,  and  observing  the  angle  to  which  the  plane  is  tilted  when 
they  begin  to  slide  down  the  plane. 


APPARATUS  FOR  DEMONSTRATING  THE  FIRST  AND 
THIRD  LAWS  OF  FRICTION. 


INDEX  TO  PARTS. 


L  represents  Lever. 


F 
SR 
SP 
TP 

q 


Fulcrum. 
Small  roller. 
Scale-pan. 
Test-piece. 
Cylinder. 


H  represents  Handle. 


R 

S 

P 

GS 


Rack. 

Spiral  spring. 

Pointer. 

Graduated  scaled 

Base  of  apparatus. 


IKICnON — LUBRICATION.  109 

Demonstration  of  the  Third  Law  of  Friction. — The  pre- 
ceding figure  represents  the  apparatus  belonging  to  the  Applied 
Mechanics  Department  of  the  Royal  College  of  Science,  South 
Kensington  (as  described  by  Prof.  Goodeve  in  his  "  Manual  of 
Applied  Mechanics  "),  for  demonstrating  the  first  and  third  laws 
of  friction. 

If  the  weights,  W,  be  removed  from  the  scale-pan  SP,  then 
there  will  be  but  a  slight  pressure  between  the  lower  surface  of 
the  test-piece  TP,  and  the  roller  cylinder  C.  Consequently,  on 
turning  the  handle  H  in  the  direction  of  the  arrow,  there  will  be 
a  slight  pull  on  the  cord,  causing  the  pointer  P  to  move  a  degree 
or  two  over  the  graduated  scale  GS.  The  pointer  should  there- 
fore be  set  back  to  zero. 

EXPERIMENT  IY. — Put  a  weight,  W,  of  say  5  Ibs.,  into  the 
scale-pan,  and  turn  the  cylinder  slowly  by  the  handle  as  before. 
The  pointer  deflects  so  many  degrees.  Increase  the  weight  W 
to  10  Ibs.,  and  the  pointer  instantly  indicates  twice  the  amount 
of  friction ;  put  in  15  Ibs.,  and  it  shows  treble  the  friction  ;  thus 
demonstrating  the  first  law.  Then  turn  the  handle  faster  and 
faster,  and  the  pointer  remains  fairly  stationary,  thus  proving 
within  certain  limits  that  friction  is  independent  of  the  velocity.* 

Lubrication. — Lubricants,  such  as  tallow,  grease,  soft  soap, 
and  many  kinds  of  oils,  are  used  to  reduce  friction.  Both  skill 
and  knowledge  are  required  to  decide  upon  the  best  kind  of  lubri- 
cant and  the  proper  amount  for  different  cases.  Lubrication  and 
lubricants  should  receive  greater  attention  from  the  engineer,  for 
the  satisfactory  working  and  length  of  life  of  most  machines  de- 
pend so  largely  upon  effective  lubrication.  Where  very  heavy 
pressures  and  high  speeds  are  experienced  as  in  some  cases  of 
electrical  machinery,  it  pays  to  use  the  very  best  kind  of  oil,  and 
to  distribute  it  to  all  the  bearings  from  one  common  centre  under 
pressure  by  means  of  a  force-pump.  It  thereby  flows  in  a  con- 
tinuous stream  through  the  bearings  to  a  filtering  tank,  from 
which  it  is  again  and  again  pumped  on  its  soothing  mission  for 
months  on  end,  without  change  or  great  loss  in  quantity.  This  is 
a  very  different  state  of  matters  from  the  "  travelling  oil-can " 
system,  where  the  amount  applied  may  vary,  and  the  times  of  appli- 
cation may  be  erratic,  according  to  the  opinion  of  the  attendant. 

Anti-Friction  Wheels. — In  the  case  of  delicate  machinery, 
such  as  in  Atwood's  machine  for  ascertaining  by  experiment  the 
acceleration  of  gravity,  and  in  Lord  Kelvin's  mouse-mill  for 
driving  the  paper  rollers  of  his  Syphon  Recorder,  when  receiving 

*  Sea  Molesworth'i  Pocketbook  of  Engineering  Formulae,  and  the  Trans- 
actions of  the  Institution  of  Mechanical  Engineers,  for  results  of  friction 
experiment!  with  shafts  ran  at  different  speeds. 


I  10 


LECTURE   X. 


ANTI-FRICTION 
WHEELS. 


telegraphic  signals  from  long  submarine  cables,  anti-friction 
wheels  are  used  for  the  purpose  of  reducing  the  friction  to  a 
minimum.  The  accompanying  figures  illustrate  one  pair  of 
anti-friction  wheels.  The  spindle  S,  which  carries  the  driving- 
wheel,  instead  of  resting  on  two  ordinary 
bearings,  is  supported  by  two  wheels  at 
each  end,  so  that  a  rolling  contact  is  pro- 
duced between  it  and  the  wheels.  This 
form  of  contact  implies  far  less  friction  to 
begin  with,  than  a  sliding  or  scraping  con- 
tact. Besides,  the  small  amount  of  force 
required  to  overcome  the  friction  between 
the  spindle  and  the  rims  of  these  wheels, 
has  a  great  advantage  or  leverage  given 
to  it,  in  as  far  as,  it  acts  with  an  arm 
equal  to  the  radius  of  the  wheels  FW\  and 
FW2.  This  enables  it  to  turn  them  with 
great  ease  at  a  slow  rate  in  the  very  small 
bearings  Bj  and  B2. 
In  merely  overcoming  friction  at  a  bearing,  there  is  a  con- 
siderable advantage  in  using  large  pulleys  ;  for,  the  force  necessary 
at  the  periphery  of  the  pulley  to  overcome  the  friction  at  the  bear- 
ing, is  inversely  proportional  to  the  radius  or  diameter  of  the 
pulley.  (See  Lecture  XI.  fig.  i). 

Ball  Bearings. — Another  example  of  the  effect  of  rolling 
contact  reducing  friction  is  found  in  the  use  of  ball  bearings, 
which  are  now  so  common  in  all  kinds  of  cycles  and  in  high-class 
foot-driven  lathes.* 

When  it  is  necessary  to  move  heavy  beams,  guns,  &c.,  a  common 
practice  is  to  place  them  on  rollers  or  on  two  channel  iron 
girders  o  with  round  cannon-shot  between  them,  when  a  com- 
paratively small  force,  properly  applied,  will  have  the  desired 
effect. 

We  will  have  to  return  to  this  subject  in  the  Advanced  Course 
when  dealing  with  the  friction  between  shafts  and  their  bearings, 
and  the  various  means  that  have  been  adopted  for  minimising 
the  same.  In  the  meantime,  we  will  complete  this  Lecture  with 
an  example  of  work  done  on  an  incline  when  friction  is  included. 
Work  done  on  Inclines,  including  Friction. — The  method 
of  calculating  the  work  expended  in  moving  a  body  along  a  smooth 
inclined  plane  was  fully  dealt  with  in  Lecture  IX. ;  consequently, 
the  student  is  prepared,  after  what  has  been  said  about  friction  in 


*  Refer  to  Lecture  XVI.,  p.  183. 


WORK    DONE    ON    INCLINES    INCLUDING    FRICTION.        I  I  I 

this  Lecture,  to  consider  the  case  of  pulling  a  body  up  or  down 
a  plane  when  the  co-efficient  of  friction  between  the  body  and  the 
plane  is  known. 

The  total  work  expended  is  evidently  divisible  into  two  distinct 
portions — 

1 i )  The  work  done  with  or  against  tJie  action  of  gravity,  accord- 
ing  as  the  body  is  moved  down  or  up  the  inclined  plane  =  W  x  h 
(where  h  is  the  height  of  the  plane). 

(2)  The  work  done  against  friction  =  F  x  I  (where  I  is  the  length 
of  plane  passed  over). 

The  work  to  be  done  against  friction  is  the  same  whether  the 
body  is  urged  up  or  down  tho  incline;  for  it  is  equal  to  the 
r^-effiomnt  of  friction  x  the  reaction  of  the  plane  x  the  distance 
through  which  it  is  moved. 
Or,  FxZ  =  /ixRxJ 

But  by  Lecture  IX.       B  x  1  =  W  x  b;  .•.  F  x  Z  =  /z  x  W  x  b 
Or,  the  work  done  against  friction  in  moving  a  body  along  the 
inclined  distance  I,  is  equal  to  the  work  done  in  moving  the  same 
body  along  a  horizontal  distance  b,  equal  to  the  base  of  the  incline. 

If  the  work  to  be  done  in  overcoming  friction,  is  equal  to  the 
work  capable  of  being  done  on  the  body  by  gravity,  the  body  will 
be  in  equilibrium,  and  the  inclination  of  the  plane  is  equal  to  the 
angle  of  repose. 

If  the  work  to  be  done  in  overcoming  friction  is  less  than  the 
work  which  gravity  can  do  on  the  body,  the  body  will  slide  down 
the  incline,  or,  in  technical  language,  the  machine  will  overhaul. 

EXAMPLE  II. — What  is  the  co-efficient  of  friction,  and  how  is  it 
ascertained  ?  There  is  an  inclined  plane  of  i  foot  vertical  to 
10  feet  horizontal;  what  work  is  done  in  moving  700  Ibs.  5  feet 
along  the  plane,  the  co-efficient  of  friction  being  -08  ?  (S.  and  A. 
Exam.  1892.) 


w 
FIGURE  FOB  EXAMPLE  II. 

ANSWER. — The  co-efficient  of  friction  for  two  bodies  in  contact 
is  the  passive  resistance  (opposing  the  motion  of  the  one  over  the 
ofher)  divided  by  the  reaction  or  normal  vressure  between  the  surfaces 
in  contact — 

Friction      F 

i.e. ,  Co-emcient  or  fnction  =  -^ —. —  =  ^  =  u. 

Reaction     R 

For  methods  of  ascertaining  co-efficients  of  friction,  see  the  text  in  this 
Lecture,  -a 


112  LECTURE   X. 

Total  work  done  =  work  done  against  gravity  +  work  done  against  frwtion. 

Referring  to  the  accompanying  figure,  we  see  that — 

Si)  Work  done  against  gravity   =   W  x  DE 
2)  Work  done  against  friction   =    F  x  AD 


Total  work  done   =   W  x  DE  +  F  x  AD 


We  have  therefore  only  to  substitute  the  numerical  values  corre- 
sponding to  these  letters  in  order  to  arrive  at  the  result.  From  the  ques- 
tion W  =  7oo  Ibs.  From  the  figure  we  see  that  DE  is  parallel  to  BC  ;  con- 
sequently by  Euclid  the  A",  ADE,  and  ABC  are  similar  in  every  respect  ; 
and  therefore 

DE  :  BC  :  :  AD  :  AB  ;  or,  DE=B°*AP. 

Ai> 

But,  also  by  Euclid,       AB=  ^  AC2  +  BC2=  V  io2+i2=  10-05  ft.  (nearly) 
Consequently,  DB-?55.       =JJL     =  -497  ft. 


And,  F     *     AtB 

From  the  question  we  are  told  that  |i  =  -08,  and  we  learn  from  Lecture  IX. 
that 

B  :  W  :  :  AC  :  AB  ;  or,  R=  WAC  =  7io=696.s  lbs. 


.'.  F=/iR=-o8x  696-5  =  55-72  Ibs. 

Hence  Total  Work   =  W    x    DE    +    FxAD 

»         »         »        =  7oo  *  -497'  +  55-72x5' 

„         „         „        =  347-9  ft.-lbs.  +  278-6  ft.-lbs. 

„         „         „        =  626-5  ft.  -Ibs. 

NOTE.  —  For  the  work  done  against  friction  quite  a  simple  way  would  have  been  to  hard 
taken  the  formula  deduced  on  the  previous  page— 

Vix.  :  FxZ=MXWx&=/mXWxAE=  o8X7cx>X4-97  =  278'6  ft.-lbs. 

*••    IB-BO   •••  A8-«*p-22*=-w 

APPROXIMATE  ANSWER.  —  Since  the  inclination  of  the  plane  is  so 
very  small  in  this  case,  we  might  have  assumed  that 

R  =  W;  AB  =  AC,  andDE  =  JBC 
Then, 

(1)  Work  done  against  gravity  =  W  x  DE  =  700  x  %         =  350  ft.-lbs. 

(2)  Work  done  against  friction  =  F  x  AD=-o8  x  700  x  5  =  280  ft.-lbs. 

0%  TQtal  work  =  W  x  DE  +  F  x  AD  =  350  +  280=  630  ft.-lbs. 


LECTUEE    X. — QUESTIONS.  -  113 


LECTURE  X.— QUESTIONS. 

1.  What  is  friction,  and  how  does  it  act  ?    What  is  developed  ^  hen  force 
overcomes  friction  ?    How  do  you  measure  the  result  ? 

2.  Explain  by  sketches  and  concise  description  how  the  laws  of  friction 
may  be  tested  experimentally.     What  is  meant  by  the  "  co-efficient  of  fric- 
tion," "angle  of  repose,"   "angle  of  friction,"  and  "  sliding  angle "  or 
11  limiting  angle  of  resistance  "  ? 

3.  How  is  the  co-efficient  of  friction  between  two  surfaces  ascertained 
approximately  by  experiment?      When  two  rough  surfaces  are  pressed 
together,  how  much  may  the  line  of  pressure  be  inclined  to  the  common 
perpendicular  to  the  surfaces  in  contact  before  motion  ensues  ? 

4-  What  is  the  co-efficient  of  friction  when  the  angle  of  repose  is — 
(a)  5°  42' ;  (6)  1 1°  18' ;  (c)  16°  4* ;  (d)  21°  48' ;  (e)  26°  36' :  (/)  30°  ;  (g)  45°  T 
Draw  the  angles  to  scale.  Ans.  (a)  -i  ;  (b)  -2;  (c)«3;  (d)  -4;  (e)  -5 ; 

(/)'5774:  (9)  i. 

5.  An  inclined  plane  is  100  feet  long  and  20  feet  high.     A  body  weighing 
100  Ibs.  is  pulled  up  from  the  bottom  to  the  top,  and  then  down  again.     If 
the  co-efficient  of  friction  between  the  body  and  the  plane  is  -5,  what 
work  was  expended  in  each  case  ?    What  would   require   to  be  the  co- 
efficient of  friction  in  order  that  the  body  might  just  slide  down  of  its 

own  accord?    Ans.  6,900  ft.-lbs.  ;  2,900  ft.-lbs.  ;  /i=  _=  ^£.=-204. 

b        12 

6.  What  is  the  co-efficient  of  friction,  and  how  is  it  ascertained  ?    There 
is  an  inclined  plane  of  i  foot  vertical  to  5  feet  horizontal ;  what  work  is 
done  in  moving  100  Ibs.  through  100  feet  along  the  plane,  the  co- efficient 
of  friction  being  •!  1  Ans.  2940  ft.-lbs. 

7.  An  incline  is  80  feet  long,  with  a  rise  of  20  feet.     A  body  weighing 
100  Ibs.  is  drawn  40  feet  along  the  incline  ;  what  work  is  expended  if  the 
co-efficient  of  friction  is  -6  ?     Ans.  3,323  ft. -Ibs. 

8.  A  weight  of  5  cwts.  resting  on  a  Horizontal  plane  requires  a  horizontal 
force  of  loo  Ibs.  to  move  it  against  friction.     What  is  the  co-efficient  of 
friction  ?    Ans.  -18. 

9.  A  plank  of  oak  lies  on  a  floor  with  a  rope  attached  to  it.     When  the 
rope  is  pulled  horizontally  with  a  force  of  70  Ibs.  it  just  moves,  but  when 
pulled  at  an  angle  of  30°  to  the  floor  a  force  of  60  Ibs.  moves  it.     What  is 
the  weight  of  the  plank  and  the  co-efficient  of  friction  between  it  and  the 
floor  ?     Ans.  116-6  Ibs. ;  -6. 

10.  Suppose  a  locomotive  weighs    30  tons,  and  that  the  share  of  this 
weight  borne  by  the  driving  wheel  is  10  tons.     Then,  if  the  co-efficient  of 
friction  between  the  wheels  and  the  rails  be  -2,  what  load  will  the  engine 
draw  on  the  level  if  the  required  co-efficient  of  traction  be  10  Ibs.  per  ton 
of  train  load  ?    What  load  will  this  engine  draw  at  the  same  rate  up  an 
incline  of  i  in  20  ?    Ans.  448  tons  (including  engine) ;  36-  72  tons  (in- 
cluding engine). 

11.  State  the  laws  of  friction,  and  explain  the  contrivance  known  as 
friction  wheels.     What   is   the    advantage  of  ball  bearings  for  bicycles? 
Sketch  in  section  such  a  bearing. 

12.  What  are  lubricants,  and  for  what  purposes  are  they  used  in  machin- 
ery ?    What  kind  of  lubricant  would  you  use  for  the  moving  parts  of  a 
very  high-speed  engine  and  direct-driven  dynamo,  and  how  would  you 
apply  it  so  as  to  be  able  to  use  it  over  and  over  again  ? 


114  LECTURE   X. — QUESTIONS. 

13.  What  is  friction  ?    What  is  meant  by  limiting  friction,  by  sliding 
friction,  and  by  the  co-efficient  of  friction  ?    A  weight  of  5  cwts.  resting 
on  a  horizontal  plane,  requires  a  horizontal  force  of  108  Ibs.  to  move  it 
against  friction.     What  in  that  case  is  the  value  of  the  co-efficient  of 
friction  1    Ans.  "192. 

14.  How  would  you  experimentally  determine  the  nature  of  the  friction 
between  clean,  smooth  surfaces,  say  of  oak,  and  what  sort  of  law  would 
you  expect  to  find  ? 

15.  Describe  any  experiment  which  you  have  made  or  seen  for  finding 
the  laws  of  solid  friction.     What  are  the  laws  so  found  ?     Are  they  quite 
true  ?     How  do  they  differ  from  the  laws  of  fluid  friction  ? 

16.  Sketch  and  describe  an  apparatus  for  determining  the  co-efficient 
of  sliding  friction  between  two  planed  surfaces  of  oak. 

If  you  have  made  this  or  a  similar  experiment  describe  the  behaviour 
of  the  sliding  piece,  and  any  troubles  you  may  have  had.  State  how  you 
would  conduct  the  experiment  so  as  to  establish  the  principal  facts 
concerning  such  friction.  (B.  of  E.,  1902.) 

17.  The  tractive  resistance  of  a  train  weighing  335  tons  is  1 1  Ibs.  per  ton. 
If  the  effective  horse-pov  er  of  the  lo  'emotive  is  600,  estimate  the  uniform 
speed  obtainable  when  ascending  an  incline  of  I  in  200.     Ans.  2662  ft. 
per  min.  or  30  miles  per  hour.  (C.  &  G.,  1904,0.,  Sec.  A.) 


NOTES    AND    QUESTIONS. 


LECTURE  XL 


CONTENTS.  —  Difference  of  Tension  in  the  Leading  and  Following  Parts  of 
a  Driving  Belt  —  Brake  Horse-Power  transmitted  by  Belts  —  Examples 
I.  II.  —  Velocity  Ratios  in  Belt  Gearing—  Examples  III.  IV.—  Open  and 
Crossed  Belts—  Fast  and  Loose  Pulleys—  Belt  Gearing  Reversing 
Motions—  Stepped  Speed  Cones  with  Starting  and  Stopping  Gear  — 
Driving  and  Following  Pulleys  in  Different  Planes—  Shape  of  Pulley 
Face—  Questions. 

WE  shall  devote  this  Lecture  to  the  transmission  of  power  by 
belting  and  to  belt-gearing. 

Difference   of  Tension  in  the  Leading   and  Following 
Parts  of  a  Driving-Beit.  —  In  Lecture  VI.,  when  discussing  the 
case  of  the  simple  pulley,  we  assumed  that  the  belt  or  rope  passing 
over  the  pulley  was  perfectly  flexible,  and  that  there  was  no  fric- 
tion at  the  axle  of  the  pulley.     Conse- 
quently, we  found  that  equal  weights 
would  balance  each  other,  or  that  the 
tension   of  the  two  sides  of  the  belt 
were  equal.     A  little  consideration  of 
the  subject  will  show  that  when  one 
pulley  is  driven  from  another  one  by 
an  endless  belt  or  rope,  the  tension  on 
the  driving  side  must  be  greater  than 
that  on  the  following  side. 

i.  Take  the  case  of  an  ordinary 
vertical  pulley  with  its  axle  or  shaft 
resting  in  two  bearings  (one  on  each 
side  of  the  pulley),  with  a  belt  or  rope 
passed  over  it,  and  with  weights  at- 
tached to  the  free  ends  of  the  same. 
Here  we  must  have  a  certain  amount 
of  friction  between  the  axle  and  its 
bearings,  which  can  only  be  overcome 
by  a  force  applied  to  the  circumference 
of  the  pulley. 

T    ,  T?  _  (  Force  required  to  overcome  friction  at  the  circum- 

.Lieu       .     J?  !  —  1     fsrence  of  the  axle  or  shaft. 


DIFFERENCE   OF  TENSION 
DUE  TO  FRICTION. 


DIFFERENCE  OF   TENSION.  117 

Let  .     .     rl=    Radius  of  the  axle. 

•p  _  /Force  required  to  overcome  the  friction  of  the  axle 
t      \     when  acting  at  the  circumference  of  the  pulley. 

„    .     .     r2  =    Radius  of  the  pulley  to  centre  of  belt. 
Then,     Ftxr1=    F2xr2. 

[Weight  attached  to  the  left-hand  side  of  the  belt, 
Let  .      .  W  =  j     and  which  therefore  produces  a  tension  on  the 
I     slack  side  =  T,. 

(Least  weight  on  the  right-hand  side  of  the  belt 
that  will  produce  motion,  and  which  therefore 
produces  a  tension  on  the  driving  side=T«j. 

Then  taking  moments  about  the  centre  of  the  axle,  we  have — 

VVl  x  r2  +  F2  x  r2  =  W2  x  ra 
Or,  .         .        T.xrf  +  Ffxr,  =  Td  x  ra 

Dividing  both  sides  of  the  equation  by  r,  we  get 

T.+F,=Td 

.-.    F2  =  Td-T, 

Or,  expressed  in  words,  the  force  F2,  acting  at  the  circumference 
of  the  pulley  (which  is  required  to  overcome  the  friction  of  the 
axle)  is  equal  to  the  tension  Td  on  the  driving  or  forward  side  of 
the  pulley,  minus  the  tension  T,  on  the  slack  or  following  side. 

In  order  that  the  periphery  of  the  pulley  may  move  at  the  same 
rate  as  the  under  face  of  the  belt,  we  must  have  sufficient  tension 
on  each  part  of  the  latter,  and  the  co-efficient  of  friction  between 
them  must  not  have  less  than  a  certain  value.  Too  great  adhe- 
sion between  them  would  result  in  a  loss  of  work,  for  in  that  case 
an  extra  force  would  have  to  be  applied  solely  for  the  purpose  of 
pulling  the  belt  from  the  pulley. 

2.  Take  the  case  of  one  vertical  pulley  of  diameter  D,  driving 
another  vertical  pulley  of  diameter  d  by  means  of  an  endless  belt, 
rope,  or  chain  in  the  direction  of 
the  arrows  shown  on  the  accom- 
panying figure.  Whenever  the 
pulley  T>  is  moved,  the  tension  on 
the  driving  side  Td  tends  to 
stretch  the  belt  on  that  side,  and 
this  tension  increases  until  the 

pulley  d  begins  to  move  ;  whereas  BELT  DRIVIKG. 

the  tension  on  the  following  or 
slack  side,  T,,  is  gradually  diminished  until  the  difference  of  the 
tensions  (Td  -  Ta)  produces  a  uniform  velocity  of  the  belt.  Of 
course  the  tension  on  the  slack  side  must  be  sufficient  to  pro- 


ITS 


LECTURE    XI. 


vent  the  slipping  of  the  belt  on  either  of  the  pulleys  if  the 
periphery  of  the  driven  pulley  is  to  keep  pace  with  the  peri' 
phery  of  the  driving  one.  In  order  that  there  may  be  a  mini- 
mum chance  of  the  belt  slipping,  its  slack  side  should  always 
run  from  the  top  side  of  the  driving  pulley.  By  so  arranging  the 
drive,  the  sag  of  the  belt  on  the  slack  side  will  cause  it  to  encom- 
pass a  greater  length  of  the  circumferences  of  both  pulleys.  The 
motion  of  the  belt  will  be  easier,  and  the  wear  and  tear  of  the 
bearings  will  be  less,  because  there  will  be  less  total  stress 
(Td  +  T,)  tending  to  draw  the  pulleys  together  for  the  transmis- 
sion of  a  certain  horse-power,  than  if  the  slack  side  left  the  under 
side  of  the  driving-pulley.  Referring  to  the  previous  figure,  if  the 
slack  side  leaves  the  top  side  of  the  pulley  D,  it  grips  the  same 
from  position  4,  round  the  back  of  the  pulley  to  5,  and  the  pulley 
d  from  6  round  to  3 ;  whereas,  if  D  were  rotated  in  the  opposite 
direction,  we  should  have  the  slack  side  entering  on  it  at  i,  and 
only  gripping  it  as  far  as  position  8  ;  entering  on  d  at  7,  and  only 
gripping  it  to  position  2,  thus  having  far  less  grip  on  the  pulleys 
and  thereby  encouraging  the  natural  tendency  of  the  belt  to  slip 
on  the  pulleys.* 

Brake  Horse-power  transmitted  by  Belts. 

Let.         .     V  =  Velocity  of  belt  in  feet  per  minute. 
„    .         .     P  =  (Td  —  T4)  the  net  pull  causing  motion  in  Ibs. 

Then,   B.H.P.=    YP Y(T^~T») 

33>°°°       33>°°°- 

Let .         .     D  «  Diameter  of  driving  pulley  in  feet  =  2r. 
Then        .  vD  *=  Circumference  of  driving  pulley  in  feet  =  2wr. 
Let  .      n  =  Number  of  revolutions  of  pulley  per  minute. 

Then        .     Y  =  nDn  =  2irrn  «=  velocity  of  belt  (with  no  slip). 

And,  the  B.H.P.  =  7rPnP  =  27rmP 

33>°°<>     33>°°° 

EXAMPLE  I. — A  pulley  6'  in  diameter  is  driven  at  100  revolu- 
tions per  minute  and  transmits  motion  to  another  pulley  by  means 
of  a  belt  without  slip.  If  the  tension  on  the  driving  side  of  the 
belt  is  1 20  Ibs.  and  on  the  slack  side  20  Ibs.,  what  is  the  brake 
horse-power  being  transmitted  ? 

*  The  previous  figure  should  have  been  drawn  with  the  full  and  dotted 
.Ines  at  T,,  reversed,  but  the  student  will  easily  follow  the  explanation. 


BRAKE   HORSE-  POWER   TRANSMITTED  BY  BELTS..         1  1  9 
ANSWER.—  Here  r  =  3';  n  =  100;  P  =  (Td  -  T.)  =  (i  20  -  20)  =  loolbs. 

•    B  H  P  =  27rmP  =  2  x  y  x  3  x  IPO  x  IPO  _  5.71 
33,000  33>°°° 

EXAMPLE  II.  —  What  must  be  the  number  of  revolutions  per 
minute  of  a  driving  pulley  6'  in  diameter,  in  order  that  it  may 
transmit  5-71  B.H.P.  by  a  belt  to  another  pulley,  if  the  net  pull 
on  the  belt  Ls  100  Ibs.  ? 

ANSWER.  —  Here  we  have  the  same  data  to  go  upon  as  in 
Example  I.,  except  that  we  are  given  the  B.H.P.  instead  of  the 
revolutions  per  minute.  Then,  transposing  every  quantity 
except  n  (the  revolutions  per  minute)  to  one  side  of  the  above 
equation,  we  have 

n  =  (B.H.P.)  x  33,000  =    5-71x33,000  =  100  r 
27irP  2  x  Hf  x  3  x  100 

In  precisely  the  same  way,  if  you  were  given  the  power  to  be  trans- 
mitted, the  revolutions  per  minute,  the  difference  of  tension  on  the  two 
sides  of  the  belt,  and  you  were  asked  for  the  diameter  of  the  pulley,  the 
formula  would  appear  thus  — 

n  _(B.H.  P.)  x  33,000 

7T«P 

If  it  was  the  difference  of  tension  in  the  belt  that  was  asked  for,  then  — 


You  would  (after  arranging  the  formula  in  this  way,  so  as  to  keep  the 
unknown  quantity  on  one  side  of  the  equation)  simply  have  to  substitute 
the  numerical  values  corresponding  to  the  different  symbols,  and  then 
cancel  out  the  figures  in  numerator  and  denominator,  in  order  to  reduce 
the  long  multiplication  and  division  to  a  minimum,  and  thereby  arrive  at 
the  result  as  quickly  as  possible. 

Velocity  Ratios  in  Belt  Gearing.  —  Let  two  or  more  pulleys 
be  connected  by  belting  in  the  manner  shown  by  the  accompany- 
ing figure.  Then,  if  there  is  no  slipping  of  the  belts,  the  circum- 


VELOCITY  RATIOS  IN  BELT  GEARING. 

f  erential  speeds  of  the  pulleys  will  be  the  same  as  the  velocity  of 
the  belts  passing  round  them. 


120  LECTURE   XI. 


Let  Dj,  D,       •*  Diameters  of  the  drivers* 
„    F1?  F,       =  Diameters  of  the  followers. 
„    NDi,  N"Dj   =  Number  of  revolutions  per  minute  of  the  drivers. 
„    Np  ,  Np     =  Number  of  revolutions  per  minute  of  followers. 

Then,  taking  the  first  pair  of  pulleys,  Dj  and  F,  we  have  — 
Circumferential  speed  of  driven  =  Circumferential  speed  of  follower  i. 


(Divide  both  sides  by  *•) 


Or,  The  product  of  the  diameter  of  -\ 
the  driver  and  its  number  of 
revolutions  per  minute. 


i  The  product  of  the  dia- 
meter of  the  follower 
and  its  number  of  re- 


(     volutions  per  minute. 

D      "N" 
Or  ......        ^»-jg     .        .        .    (1) 

i.e.,  The  ratio  of  the  diameters  of  the  pulleys  is  in  the  inverse 
ratio  of  their  speeds  or  revolutions  per  minute. 

Treating  the  motion  of  the  second  set  of  pulleys  in  exactly  the 
same  way,  we  have  — 
Circumferential  speed,  of  Df  =  circumferential  speed,  of  F, 


(Divide  both  sides  by  TT) 

D,NDj  =  F,N,, 

<*  .....         f  =  ?'          '         •         •         •     (2) 

J  a      -^  »3 

(But  the  revolutions  of  F,  and  of  D2  are  the  same)  .'.  Nri  =  ND 
Or,    .        .        .        D,  N,     -   F,N,       .-.  NF>  -  ?i/?' 

X« 

_F,NF 

Consequently     .         .     ^    -^ 

•    -rr1 

i 

/  (Dividing  both  numerator)    ND  =   _D!_  =    Ft  x  F, 
I  and  denominator  by  NFi)J    ]^pi         D2       =    DX  x  D, 

^a 

Or,  we  might  have  arrived  at  the  same  result  by  multiplying  equation* 
(1)  and  (2)  together.    Thus— 


VELOCITY  RATIOS   IN  BELT   GEARING.  121 

Or,   Speed  of  first  driver  m   Product  of  diameters  of  followers 

Speed  of  last  follower  Product  of  diameters  of  drivers 

Or,       .        NDi  x  Dj  x  D,    =    NFj  x  Ft  x  F, 
t.«.,  Speed  of  first  driver  x  dia-}  _  (Speed  of  last  follower  x  diame- 
.    meters  o/JAe  drwers        J  ~  (  ters  of  the  followers. 

In  the  same  way  we  may  treat  any  number  of  drivers  and 
followers  by  this  general  formula — viz., 
Speed  or  number  of  revolutions} 


per  minute  of  the  first  driver  [ 
x  the  successive  diameters  oft 
the  drivers  ) 


Speed  of  the  last  follower  x  the 
successive  diameters  of  the 
followers. 


Precisely  the  same  rule  holds  good  for  discs  driven  by  contact 
friction  and  for  wheel  gearing,  as  you  will  find  from  the  next 
lecture ;  but  in  friction  gearing  and  wheel  gearing  the  driver  and 
the  follower  move  in  different  directions,  whereas  in  belt  gearing 
they  move  in  the  same  or  in  the  opposite  direction,  according  as 
the  driving  belts  are  "  open  "  or  "  crossed." 

EXAMPLE  III. — Referring  to  the  previous  figure,  suppose  that  a 
driving  pulley,  Dp  is  connected  by  a  belt  to  a  follower,  Fp 
whilst  it  moves  at  100  revolutions  per  minute.  If  the  diameter 
of  the  driver  is  6'  and  of  the  follower  3',  what  will  be  the  number 
of  revolutions  per  minute  of  the  follower  1 

By  the  previous  formula  for  two  pulleys, 
Dt  x  NDi  =  Ft  x  N?i 

XT          D,  x  ND        6'  x  100 
.•.   -NPi  „  -t_— — *  = »  200r.p.m. 

*!  3 

EXAMPLE  IT. — Referring  to  the  previous  figure,  suppose  that 
a  driving  pulley  Da  (4'  diameter),  is  geared  to  a  follower,  Ft 
(2'  in  diameter),  and  that  a  second  driver  D,  (4/  diameter),  fixed 
to  the  same  shaft  as  Fp  is  geared  to  a  second  follower  F,  (i'  dia- 
meter). If  D  makes  60  revolutions  per  minute,  what  is  the  speed 
ofF,? 

By  the  previous  formula  for  four  pulleys, 
NDi  x  Dt  x    D,    =  Np>  x  Ft   x  Ff 


**    =480 


r.p.m. 


Open  and  Crossed  Belts. — By  referring  to  the  next  figure, 
the  student  will  observe  that  the  left-hand  end  view  shows  what 


122  LECTURE   XI. 

is  termed  an  open  belt,  OB,  and  that  the  right  hand  end  view 
shows  a  crossed  belt,  013.  In  the  case  of  open  belts,  the  driver 
and  the  follower  rotate  in  the  same  direction  (as  may  be  seen  from 
the  second  and  third  figures  in  this  Lecture) ;  whereas,  with 
crossed  belt  driving,  the  follower  revolves  in  the  opposite  direction 
to  that  of  the  driver,  just  as  it  does  when  direct  friction  or  wheel- 
gearing  is  used. 

Fast  and  Loose  Pulleys. — As  will  be  seen  from  the  two  front 
views  in  the  next  set  of  illustrations,  the  open  and  the  crossed 
belts  are  shown  passing  from  the  broad  driving-pulleys  DP,  to 
the  broad  loose  pulleys  LP.  Loose  pulleys  are  generally  bushed 
with  gun-metal,  and  then  bored  out  so  as  to  fit  their  shafts  easily. 
This  permits  them  to  rotate  without  turning  the  shaft  upon  which 
they  bear.  The  pulleys,  FP,  are  keyed  hard  on  to  the  shafts,  so 
that  when  the  belt  is  forced  over  upon  them  by  means  of  the 
shifting  forks,  SF,  the  machines  connected  with  the  same  are  set 
agoing.  This  simple  combination  of  fast  and  loose  pulleys  there- 
fore enables  a  machine  to  be  stopped  or  started  at  pleasure,  without 
interfering  with  the  motion  of  the  driving  pulley  and  the  belt. 
In  ordinary  cases  where  there  is  only  one  driving  belt  required, 
the  loose  pulley  is  of  the  same  breadth  as  the  fixed  pulley.* 

Belt- Gearing  Reversing  Motions. — In  many  kinds  of 
machine  tools  it  is  desirable  to  be  able  to  drive  the  tool  first  in 
one  direction  and  then  in  the  opposite  direction,  as  well  as  to 
start  or  stop  it.  This  is  frequently  effected  by  a  combination  of 
open  and  crossed  belts  with  fast  and  loose  pulleys,  as  illustrated 
by  the  accompanying  figure. 

From  what  has  just  been  said  about  open  and  crossed  belts,  as 
well  as  fast  and  loose  pulleys,  the  student  will  have  no  difficulty 
in  understanding  this  arrangement  of  reversing  gear.  If  applied 
to  a  machine  for  planing  metals,  the  shaft  which  is  keyed  to  the 
fixed  pulley  FP  would  be  connected  either  through  wheel  gearing 
and  a  rack,  or  through  a  central  screw,  to  the  travelling  table  of 
machine  upon  which  the  job  to  be  acted  upon  is  secured.  When- 
ever the  table  had  been  moved  backwards  to  the  end  of  the 
required  stroke  by  the  crossed  belt,  the  shifting  fork  SF  would 
be  pushed  forward  by  an  outstanding  arm  or  kicker  attached  to 

*  See  the  set  of  figures  after  the  next,  where  El  is  the  driving  belt 
engaging  the  fixed  pulley,  FP  ;  and  where  LP  is  the  loose  pulley,  to  which 
the  belt  may  be  shifted  by  means  of  the  shifting-fork,  SF,  whenever  it  is 
desirable  to  stop  the  speed  cones,  SCj,  SC^  and  the  machine  to  which  they 
are  connected.  In  the  first  front  view  of  the  next  set  of  figures,  the  driving 
pulley,  DP,  and  both  of  the  loose  pulleys,  LP,  are  drawn  too  narrow.  They 
should  have  been  represented  half  as  wide  again,  in  order  to  prevent  the 
belts  slipping  over  the  outside  edge,  when  the  other  belt  is  shifted  on  to 
the  fixed  pulley  situated  between  them. 


BELT-GEARING   REVERSING   MOTIONS. 


123 


the  side  of  the  table  at  such  a  position  as  would  cause  the  crossed 
belt  CB  to  be  shifted  from  the  central  fixed  pulley  to  its  loose  one, 
and  at  the  same  time  bring  over  the  open  belt  from  its  loose  pulley 
to  the  central  fixed  one.  Whenever  the  planing  tool  had  finished 
its  cut  on  the  metal,  the  shifting  fork  would  be  pulled  backward 
by  another  similar  outstanding  arm  or  kicker  (also  attached  to 
the  travelling  table  of  the  planing  machine,  at  a  position  just 
beyond  the  end  of  the  required  stroke  for  the  particular  job  under 
operation),  thereby  shifting  the  open  belt  OB  from  FP,  to  its  loose 
pulley,  LP,  and  at  the  same  time  pulling  over  the  crossed  belt,  CB, 


L^FP 

FRONT  VIEW, 
Forward  and  Return  at  same  Speed. 


FRONT  VIEW. 
Quick  Return. 


BELT  GEARING  REVERSING  MOTIONS. 


INDEX  TO  PARTS. 


DP  represents  Driving  pulleys. 
FP          „          Fixed  pulleys". 
LP          „          Loose  pulleys. 


OB  represents  Open  belts. 
CB          „  Crossed  belts. 

SF          „  Shifting  forks. 


from  its  loose  pulley  to  the  central  fixed  pulley,  thus  causing  the 
table  to  make  the  return  stroke. 

'.The  left-hand  front  view,  with  its  accompanying  end  views,  shov 
the  necessary  arrangements  when  the  forward  and  backward 
velocities  of  the  table  are  equal.  The  right-hand  front  view  illus- 
trates the  case  wherein  the  backward  or  non-planing  motion  is 
intentionally  made  quicker  than  the  forward  or  cutting  stroke,  so 
as  to  save  time,  by  having  the  back  motion  fixed  pulley,  FP,  and  its 
corresponding  loose  one,  LP,  made  smaller  than  the  forward  set. 
The  end  views  for  this  latter  case  would  be  similar  to  the  former 
onp.  with  the  exception  that  the  crossed  belt  would  engage  P  smaller 


124 


LECTUKE  XI. 


pulley  of  the  same  size  as  shown  by  the  front  view.  This  latter 
arrangement  can  evidently  be  employed  to  obtain  a  fast  or  a  slow 
motion  in  the  same  direction,  by  simply  having  both  belts  open  or 
both  crossed. 

Stepped  Speed  Cones  with  Starting  and  Stopping  Gear. 

In  many  machines,  such  as  lathes,  planers  and  other   machine 

tools,  it  is  very  desirable  not  only  to  be  able  to  start  and  stop 
them,  but  also  to  alter  their  speed  so  as  to  suit  different  classes  of 


END  VIEW.  SIDE  VIEW. 

STEPPED  SPEED  CONES  WITH  STARTING  AND  STOPPING  GEAB. 

INDEX  TO  PARTS. 


HBt,HB2  represent  Hangingbrackets 
for  supporting 
shaft,  &c. 

SCj,SC2          „         Speed  cones. 
FP          „         Fast  pulley. 
LP          „        Loose  pulley. 


B,,B2  represent  Belts. 

H          „         Handle. 
SB          „         Sliding  bar. 
SF          „         Shifting  fork. 
W         „        Weight  to  fix  SB  in 
positions  ^ ^.. 


work,  without  affecting  the  motion  of  the  prime  motor  or  that  of 
the  shop  driving-shaft.  These  objects  are  generally  attained  by  a 
combination  of  fast  and  loose  pulleys  with  what  are  termed 
"  stepped  speed  cones."  The  accompanying  side  and  end  views 
illustrate  the  arrangement  as  usually  carried  out  in  engineering 
works.  When  the  starting-handle,  H,  is  turned  to  the  right 
hand,  it  pulls  over  the  sliding-bar,  SB,  with  its  shifting-fork,  SF, 
which  moves  the  belt,  Bp  from  the  loose  pulley,  LP,  to  the  fixed 
one,  FP ;  thus  setting  the  speed  cones,  SGp  SC2,  and  thereby  the 


DRIVING   AND   FOLLOWING   PULLETS.  12$ 

machine  in  motion.  When  the  handle  is  turned  to  the  left,  it 
pushes  the  sliding-bar  and  shifting-fork  also  in  that  direction, 
thus  moving  the  belt  from  the  fixed  to  the  loose  pulley,  which 
allow  the  cones  and  machine  to  come  to  rest.  In  each  case  the 
weight,  W,  causes  a  notch  in  the  sliding-bar  to  engage  with  its 
left-hand  supporting  bracket,  thereby  preventing  the  shifting  fork 
from  pushing  the  driving  belt  too  far,  or  off  either  pulley,  and  at 
the  same  time  ensuring  that  it  remains  in  the  desired  position. 
Both  supporting  brackets  for  the  sliding-bar,  SB,  are  merely 
right-angle  extensions  from  the  hanging  brackets,  HBj,  HB,, 
which  carry  the  upper  shaft  with  its  cone  and  pulleys. 

The  upper  and  lower  speed  cones,  SCj,  SO,,  are  generally  made 
of  the  same  size  and  shape,  but  they  are  always  keyed  to  their 
respective  shafts  in  opposite  directions.  Consequently,  if  it 
should  be  desirable  to  run  the  machine  fast  for  light  work,  the 
belt,  Br  is  shifted  on  to  the  largest  pulley  of  the  upper  cone  and 
the  smallest  one  of  the  lower  cone.  If  the  machine  is  required 
to  move  slowly  for  heavy  cuts,  then  the  belt  is  placed  on  the 
smallest  upper  pulley  and  the  largest  lower  one.  Any  desired 
intermediate  speed  between  these  extremes  is  obtained  by  adjusting 
the  belt  on  one  or  other  of  the  remaining  sets  of  pulleys  of  the 
upper  and  lower  cones. 

The  student  can  easily  prove  to  himself  (by  drawing  down  the 
arrangement  to  scale)  that  such  stepped  speed  cones,  if  connected 
by  a  crossed  belt  on  one  pair  of  its  pulleys,  will  produce  the  same 
tension  in  the  belt  with  any  other  pair.*  With  open  belt-driving 
the  tightness  of  the  belt  will  not  be  the  same  when  on  one  pair  of 
the  pulleys  as  when  on  another ;  but  the  difference  is  so  small 
that  it  can  generally  be  disregarded  in  practice  without  having 
recourse  to  tightening  or  slackening  the  same. 

Driving  and  Following  Pulleys  in  Different  Planes. — It 
is  often  necessary  to  drive  a  follower  placed  in  a  different  plane 
from  the  driver.  The  accompanying  set  of  illustrations  show 
very  clearly  how  this  is  effected.  The  important  precaution  to  be 
observed  is,  that  the  leading  or  on-going  part  of  the  belt  must 
enter  upon  the  follower  in  a  fair  or  direct  line  with  its  plane  o/ 
rotation.  If  this  rule  be  attended  to,  then  power  may  be  trans- 
mitted between  two  non-parallel  shafts,  as  shown  by  the  first 
tigure,  even  if  their  centi-e  lines  are  in  planes  at  right  angles  to 
each  other — i.e.,  when  the  belt  is  working  with  quarter-twist. 
When  two  shafts  are  in  planes  at  right  angles  to  each  other,  and 

*  The  algebraical  proof  of  this  will  be  considered  in  our  "Advanced 
Book  on  Applied  Mechanics."  The  student  should  refer  to  the  general 
view  and  to  the  detail  drawings  of  the  stepped  speed  cones  in  the  foot- 
driven  screw-eutting  lathe  illustrated  in  Lecture  XVI. 


126 


LECTURE  XI. 


Tullis's  Thick-sided  Leather 
Chain  Belt,  Working  Quarter- 
twist,  and  Transmitting  l^ower 
between  two  shafts  which  are 
not  parallel.  No  Guide  Pul- 
leys are  required  for  this  drive. 


Flat  Belt  Working  Quarter-twist  and 
Transmitting  Power  between  two  right- 
angled  shafts,  with  leading  Guide  Pulley 
(GP)  to  remove  the  twist  from  the  Belt 
before  it  enters  upon  the  Follower,  and  to 
give  the  belt  more  grip  on  the  pulleys. 


Flat  Belt  Transmitting  Power 
over  Guide  Pulleys  between  two 
non -parallel  shafts  in  the  same 
plane. 


Flat  Belt  Transmitting  Power  between 
two  parallel  shafts  not  in  the  same  plane 
by  aid  of  guide  pulleys  (GP). 


SHAPE   OF   PULLEY  FACE.  I2/ 

it  is  found  desirable  to  remove  the  twist  from  the  belt  before  it 
enters  upon  the  follower,  then  a  guide-pulley,  GP,  must  be  used 
&£  shown  by  the  second  figure.  When  the  shafts  are  parallel,  but 
not  in  the  same  plane,  then  the  power  must  be  transmitted  by  aid 
of  two  guide-pulleys,  as  seen  from  an  inspection  of  the  third 
figure.  Or,  should  the  shafts  not  be  parallel,  but  in  the  same 
plane,  two  guide-pulleys  are  necessary,  as  in  the  fourth  figure. 
Guide-pulleys,  if  supported  by  spindles  running  in  adjustable 
bearings  or  brackets,  may  be  made  serviceable  as  tightening- 
pulleys  for  the  purpose  of  taking  up  the  slack  of  the  belt,  and 
thus  giving  the  necessary  grip  for  transmitting  more  power  with 
a  steadier  drive  than  can  be  obtained  without  them. 

Shape  of  Pulley  Face. — The  student  will  have  observed  that 
the  faces  or  rims  of  the  fast  and  loose  pulleys,  as  well  as  those  of 
the  stepped  cones  in  the  previous  set  of  figures,  are  slightly 
curved.  This  convex  curvature,  or  double  coning,  is  purposely 
done  in  order  to  ensure  that  the  belt  may  ride  easily  and  fairly 
in  the  centre  line  of  the  pulley  face  without  inclining  to  either 
side.  A  flat  band,  if  placed  on  the  smaller  end  of  a  revolving 
straight  conical  pulley,  will  naturally  tend  to  rise  to  the  larger 
end  of  the  cone.  Consequently,  if  each  half  of  the  face  of  a 
pulley  is  coned  (or,  which  amounts  to  the  same  thing,  if  the  rim 
of  the  pulley  be  curved  so  as  to  have  its  largest  diameter  in  the 
middle  of  its  face),  each  half  of  the  breadth  of  the  belt  will  have 
an  equal  tendency  towards  the  middle  of  the  pulley's  rim.  When 
very  fast  driving  and  sudden  severe  stresses  are  brought  to  bear 
upon  a  machine,  as  in  the  case  of  circular  saws,  morticing 
machines,  and  emery-wheel  grinders,  it  is  found  necessary  to  fit 
the  pulleys  with  side  flanges,  in  addition  to  curving  their  rims,  in 
order  to  prevent  the  belts  from  sliding  off  the  pulley's  face  to  one 
"ide  or  to  the  other. 

N.B.— The  student  may  be  referred  to  the  Author's  Text-book  on 
"Applied  Mechanics  and  Mechanical  Engineering,"  Chapters  XVII.  and 
XVIII.,  for  further  information  on  belt,  rope,  and  chain  gearing. 


128  LECTUKE   XL QUESTIONS 


LECTURE  XI. — QUESTIONS. 

1.  In  machinery,  where  one  pulley  drives  another  by  means  of  an  end- 
Jess  belt,  there  is  a  difference  of  tension  in  the  two  parts  of  the  belt.  Why 
is  this  ?    The  pulley  on  an  engine  shaft  is  5  feet  in  diameter,  and  it  makes 
100  revolutions  per  minute.     The  motion  is  transmitted  from  this  pulley 
to  the  main  shaft  by  a  belt  running  on  a  pulley,  and  the  difference  in  tension 
between  the  tight  and  slack  sides  of  the  belt  is  1 15  Ibs.    What  is  the  work 
done  per  minute  in  overcoming  the  resistance  to  motion  of  the  main  shaft  ? 

Ant.  180,642  ft.-lbs. 

2.  Deduce  from  the  "principle  of  work"    a   formula  for   the    brake 
horse-power  transmitted  by  a  belt.     The  pull  on  the  driving  side  of  a  belt 
is  200  Ibs.  and  on  the  following  side  100  Ibs.,  whilst  the  belt  has  a  velocity 
of  990  ft.  per  minute.     Find  the  number  of  units  of  work  performed  in  two 
minutes  and  the  B.H.P.  transmitted.     Ans.  198,000  ft.-lbs.,  3  B.H.P. 

3.  State  and  prove  the  rule  for  estimating  the  relative  speeds  of  two 
pulleys  connected  by  a  belt.     Also,  the  velocity  ratio  between  the  first 
driver  and  the  last  follower  in  belt  gearing,  where  there  are  two  or  more 
drivers  and  a  corresponding  number  of  followers.    [A  main  shaft  carrying 
a  pulley  of  12  inches  diameter  and  running  at  60  revolutions  per  minute, 
communicates  motion  by  a  belt  to  a  pulley  of  12  inches  diameter,  fixed  to 
a  countershaft.     A  second  pulley  on  the  countershaft,  of  8|  inches  dia. 
meter,  carries  on  the  motion  to  a  revolving  spindle  which  is  keyed  to  a 
pulley  of  4^  inches   diameter.      Sketch  the  arrangement  and  find  the 
number  of  revolutions  per  minute  made  by  this  last  pulley.     Ana.  123-5 

4.  Two  pulleys  are  connected  by  a  driving  belt,  and  the  sum  of  their 
diameters  is  30  inches ;  one  pulley  makes  2  revolutions  while  the  other 
makes  3  revolutions  ;  find  their  respective  diametere,     Ans.  18",  12". 

5.  An  engine  works  normally  at  106  revolutions  per  minute.     At  that 
speed  it  was  found  that  it  drove  by  belting  a  dynamo  at  420  revolutions 
per  minute,  but  to  show  off  the  electric  lights  at  their  normal  candle 
power  the  dynamo  had  to  be  run  at  460  revolutions  per  minute.     At  what 
speed  was  the  engine  being  driven  1    Ans.  116  revolutions  per  minute. 

6.  A  pulley  of  3  feet  radius  rotates  at  100  revolutions  per  minute  and 
transmits  motion  to  another  pulley  of  36  inches  diameter.     If  there  is 
10  per  cent,  slip  on  the  belt  what  will  be  the  speed  of  the  follower  ? 
What  will  be  the  net  driving  pull  on  the  belt  if  5  B.H.P.  is  transmitted  by 
it  ?    Ans.  180  revolutions  per  minute ;  97-2  Ibs. 

7.  Sketch  an  arrangement  of  pulleys  and  bands  for  obtaining  a  reversing 
motion  from  a  shaft  driven  at  a  constant  rate  in  one  direction,  and  describe 
the  action  of  the  combination. 

8.  Sketch  a  combination  of  fast  and  loose  pulleys  as  used  for  setting  in 
motion,  or  stopping  machinery.     Explain  the  construction  adopted  for  re- 
taining a  flat  belt  upon  a  pulley,  pointing  out  where  the  fork  is  to  be 
applied,  and  why. 

9.  Sketch  and  describe  a  good  form  of  slow  forward  and  quick  return 
for  a  shaping  machine. 

10.  Sketch  and  describe  an  arrangement  for  driving  the  table  of  a  plan- 
ing  machine  by  means  of  a  screw,  so  that  the  table  may  travel  50  per  cent, 
faster  in  the  return  than  in  the  forward  or  cutting  stroke. 


LECTURE  XI.  —  QUESTIONS^  I2Q 

11.  What  is  the  object  of  using  guide-pulleys  in  machinery  ?    Mention 
Instances  of  their  use,  and  show  how  the  directions  of  their  axes  are 
ascertained. 

12.  Describe,  with  a  sketch,  the  mode  of  reversing  the  motion  of  the 
table  in  a  planing  machine,  when  a  screw  is  employed  to  drive  the  table. 

13.  A  rope  transmits  20  horse-power  to  a  rope  pulley  of  8  feet  diameter  ; 
draw  a  section  of  the  rope  in  its  groove.     If  the  pulley  makes  100  revolu- 
tions per  minute,  what  is  the  speed  of  the  rope  in  feet  per  minute  ?  What 
is  the  difference  of  the  tensile  forces  in  the  rope  on  the  two  sides  of  the 
pulley  ?    As  it  is  the  difference  between  the  tensile  forces  in  a  belt  or  rope 
that  is  important  for  power,  why  is  it  necessary  to  have  any  pull  on  the 
slack  side  ?    ^n*.  2513  ft.  per  min.,  and  262-5  lbs- 

14.  What  are  cone  or  speed  pulleys  ?    Describe  the  use  of  such  pulleys 
in  any  machine  with  which  you  are  acquainted.     The  spindle  of  a  lathe 
can,  by  moving  the  belt  on  its  cone  pulleys,  be  driven  at  four  hundred 
revolutions  per  minute  when  at  its  greatest  and  at  100  revolutions  per 
minute  when   running  at  its  lowest  speed.     If   the   revolutions  of  the 
driving  shaft  are  kept  constant  throughout,  and  the  largest  diameter  of 
the  speed  cones  is  20",  what  must  be  the  diameter  of  the  smallest  steps 
on  the  pulleys  ;  the  speed  pulleys  on  the  two  shafts  being  of  the  same 
size  ?    Sketch  the  pulleys  in  position.     An*.  10  inches. 

15.  Upon  what  does  the  limiting  difference  of  tensions  in  the  tight  and 
slack    sides    of  a  moving    belt  depend?      If  the  working   stress   in  a 
belt  of  sectional  area  a  square  inches  be  /pounds  per  square  inch,  and  the 
ratio  of  the  tensions  in  the  tight  and  slack  sides  be  m,  find  the  horse-power 
that  can  be  safely  transmitted  when  the  speed  of  the  belt  is  v  feet  per 
second.  (0.  &  G.,  1903,  0.,  Sec.  A.) 


550 

1  6.  A  belt  transmits  60  H.P.  to  a  pulley  16  inches  in  diameter  running 
at  263  revolutions  per  minute.  What  is  the  difference  of  the  tensions  on 
the  tight  and  slack  sides  1  Ans.  Td  -  T,  =  1796  lbs.  (B.  of  E.,  1904-) 

17.  State  the  condition  that  has  to  be  satisfied  in  order  that  the  same 
belt  may  work  on  two  or  more  pairs  of  pulleys  keyed  to  parallel  shafts 
(i)  when  the  belt  is  crossed,  (2)  when  open.     Explain  also  why  a  belli 
always  climbs  to  the  section  of  the  pulley  where  the  diameter  is  greatest. 

(C.  &  G.,  1905,  O.,  Sec.  A.) 

18.  Explain  any  means  with  which  you  are  acquainted  for  determining 
the  brake  horse-power  of  an  engine  provided  with  a  fly-wheel. 

A  rope  is  wrapped  once  round  a  fly-wheel.  One  end  of  the  rope  carries 
a  weight  of  500  lbs.,  and  the  other  end  is  led  upwards,  and  is  attached  to 
a  spring  balance.  When  the  revolutions  are  105  per  minute,  the  pull  in 
the  spring  balance  varied  between  10  and  20  lbs.  If  the  diameter  of  the 
wheel  be  8  ft.,  and  of  the  rope  I  in.,  find  the  average  brake  horse-power. 
Ant.  39-6  B.H.P.  (C.  &  G.,  1005,  0.,  Se^.  A.) 


130 


LECTURE  XII 

CONTENTS.  —  Velocity  Ratio  of  Two  Friction  Circular  Discs  —  Pitch  Surfaces 
and  Pitch  Circles—  Pitch  of  Teeth  in  Wheel  Gearing—  Rack  and  Pinion 
Velocity  Ration  in  Wheel  Gearing  —  Example  I.  —  Principle  of  Work 
applied  to  Wheel  Gearing  —  Examples  II.  III.  —  Questions. 

Velocity  Ratio  of  Two  Circular  Friction   Discs.  —  If   two 

truly  centred  circular  discs  or  cylindrical  rollers,  having  their 
shafts  parallel  to  each  other  and  free  to  turn  in  fixed  bearings, 
be  brought  into  firm  contact  ;  then,  if  one  of  them  be  driven 
round,  and  if  there  be  no  slipping,  the  other  one  will  rotate  in 
the  opposite  direction  with  the  same  circumferential  speed  or 
surface  velocity  (see  the  next  figure). 

Consequently,  their  velocity  ratio  will  be  in  the  inverse  ratio  to 
their  diameters. 

This  may  be  proved  in  exactly  the  same  way  as  we  found  the 
velocity  ratio  of  two  pulleys  driven  by  a  belt  in  Lecture  XI. 

Let  Dx  =  Diameter  of  the  driving  disc. 
„    Fx  =  Diameter  of  the  following  disc. 
„  ND   =  Number  of  revolutions  per  minute  of  Dr 
„  N,    =  Number  of  revolutions  per  minute  of  Fr 

Then,  The  peripheral  velocity  of  Dl  =  Peripheral  velocity  of  Ft 
i.e  .....         irD^-irFjlSr^ 

Or,     ....          D1ND1  =  F1NF1 
i.e.   The  Driver's  diameter  x  its  speed  =  Follower's  diam?  x  its  speed. 


Speed  of  the  Driver      =  Diameter  of  Follower 
Speed  of  the  Follower          Diameter  of  Driver 

This  velocity  ratio  may  also  be  proved  in  the  following  way  :  — 

Let  the   two   circles   centred   at  A  and   B  represent  a  cross 

flection  of  the  two  friction  discs  in  contact  at  C  ;  and  let  them 

move  by  rolling  contact  through  the  angles  6  and  <j>  respectively 

in  the  same  time. 

Since    the   magnitude  of   an   angle   in   circular    measure   is 


PITCH    SURFACES   AND   PITCH   CIRCLES.  13! 

always  =  the  length  of  the  arc  subtended  by  the  angle  at  the  centre  of 
\he  circle  -r  the  radius  of  the  circle. 


VELOCITY  RATIO  OF  Two  CIRCULAR  Discs. 


Then, 

\  a 

But,  the  arc  DC  =  the  arc  EC  since  there  is  no  slipping. 
Consequently, 

DC 

The  angular  velocity  of  circle  A  _  6         r,     _  r3 
The  angular  velocity  of  circle  B      <£         EC       rl 

Or,* 

The  angular  velocity  or  speed  of  driver,  A  _  Radius  of  follower  B 
The  angular  velocity  or  speed  of  follower  B     Radius  of  driver  A 

Pitch  Surfaces  and  Pitch  Circles.  —  In  the  case  of  the  two 
discs  or  rollers  just  considered,  their  cylindrical  surfaces  are  termed 
the  pitch  surfaces;  and  the  two  circles  in  the  previous  figure 
(which  is  simply  a  representation  of  their  cross  section,  or  section 
in  the  plane  of  their  rotation)  are  called  the  pitch  circles. 

*  The  angular  velocity  of  a  rotating  disc  is  the  angle  described  by  its 
radius  in  unit  time. 

The  relation  between  angular  velocity  and  linear  velocity  may  be  shown 
thus:  —  Let  w=the  angular  velocity;  whilst  v  =  the  linear  velocity  of  a 
point  at  radius  r  from  the  centre  of  motion  when  the  disc  makes  *  revo- 
lutions in  unit  time  ; 

Then  wxr=v  ;  or,  w=-;  but  v=2vrnt 


132  LECTURE   XII. 

When  the  resistance  to  motion  of  the  follower  is  great,  the 
discs  have  to  be  provided  with  teeth  in  order  to  prevent  slipping. 

Consequently,  the  pitch  surfaces  and  the  pitch  circles  of  such 
toothed  rollers,  toothed  wheels,  or  spur  wheel  and  pinion,  are  the 
surfaces  and  the  circles  of  their  rolling  contact.* 

Pitch  of  Teeth  in  Wheel  Gearing. — The  linear  or  the 
circular  distance  from  the  centre  of  one  tooth  to  the  centre  of  the 
next  one,  or  the  distance  from  one  edge  of  a  tooth  to  the  corre- 
sponding edge  of  its  neighbouring  one,  as  measured  on  the  pitch 
circle,  is  termed  the  pitch  of  the  teeth  of  a  wheel. 

Let  D  =  Diameter  of  a  wheel  or  pinion  at  its  pitch  circle. 
p   =  Pitch  of  the  teeth  in  the  wheel  or  pinion. 
n   =  Number  of  teeth  in  the  wheel  or  pinion. 
ThenvrD  =  p   x  n 

For  the  circumference  of  the  pitch  circle  must  bo  equal  to  the 
pitch  between  any  two  neighbouring  teeth  x  the  number  of  teeth 
in  the  wheel  or  pinion  ;  since  the  pitch  between  each  pair  of  teeth 
must  be  the  same  all  round  the  pitch  circle,  otherwise  the  wheel 
would  not  gear  properly  with  any  other  wheel  or  pinion  of  the 
same  pitch. 

Back  and  Pinion. — If  a  straight  bar  of  iron  be  furnished  with 
teeth  on  one  side  it  is  called  a  rack.  It  may  therefore  be  con- 
sidered as  a  wheel  of  infinite  radius.  When  a  rack  has  a  pinion 
of  the  same  pitch  geared  with  it,  the  two  form  the  useful  combi- 
nation termed  the  rack  and  pinion.  It  is  employed  for  moving 
to  and  fro  the  tables  of  planing  machines  and  large  saw  benches, 
as  well  as  for  elevating  and  lowering  sluices  in  dams,  &c. 


Pinion 

END  VIEW.  SIDE  VIEW. 

BACK  AND  PINION  APPLIED  TO  A  SAW-MILL  TABLE. 

The  accompanying  illustrations  show  the  second  of  these  appli- 
cations, where  two  parallel  racks  are  fitted  to  the  under  side  of 

*  When  a  large  toothed  wheel  gears  with  a  small  one,  the  larger  is 
termed  a  &pur- wheel  and  the  smaller  a  pinion.  It  is  not  possible  in  the 
space  allotted  to  this  elementary  manual  to  enter  into  the  best  forms  of 
the  teeth  of  different  kinds  of  wheel  gearing.  This  subject  is  taken  up  in 
our  "Advanced  Text  Book,"  Vol.  I.,  Part  II. 


VELOCITY   RATIO   IN  WHEEL   GEARING.  133 

two  movable  tables  or  platforms.  Upon  the  upper  side  of  one 
of  the  tables  is  laid  a  log  of  wood  adjusted  in  the  desrred  position 
by  wedges.  The  tables  are  each  carried  and  guided  by  four 
rollers  turning  on  fixed  spindles.  To  the  projecting  end  of  the 
pinion  shaft  there  is  fitted  a  lever  handle,  so  that  by  merely 
turning  this  handle  in  one  direction,  the  racks,  tables,  and  log  of 
wood  are  pushed  forward  upon  the  projecting  circular  saw  which 
revolves  between  the  platforms,  and  if  turned  in  the  opposite 
direction  they  are  drawn  backwards.  The  pinions  with  their  shaft 
and  handle,  have  no  linear  motion,  for  the  shaft  is  simply  free 
to  rotate  in  fixed  bearings. 

The  rack  and  pinion  with  their  handle  constitute  a  modification 
of  the  wheel  and  axle,  or  lever  and  winch  barrel,  where  the  re- 
sistance offered  by  the  rack  and  its  load  is  overcome  by  a  force 
applied  to  the  handle.  Every  revolution  of  the  handle  turns  the 
pinion,  and  consequently  moves  the  rack  through  a  linear  dis- 
tance equal  to  the  circumference  of  the  pinion's  pitch  circle. 
The  principles  of  moments  and  of  work  can  therefore  be  applied 
to  this  machine  in  exactly  the  same  way  as  we  applied  them  to 
the  wheel  and  axle  and  the  winch. 

If  P  =  Pull  acting  on  the  handles, 
R  =  Radius  of  handle, 
r  =  Radius  of  pinion's  pitch  circle, 
W  =  Weight  or  resistance  overcome  ; 

Then        .         .        P  x  2n-R  =  W  x   2wr 
P  x       R  =  W  x       r 

Theoretical  advantage    .       .  * 


. 

P         r       W  s  velocity 

Velocity  Ratio  in  Wheel  Gearing.  —  From  what  has  been 
said  about  belt  gearing,  pitch  surfaces,  pitch  circles,  and  pitch  of 
teeth,  it  must  be  at  once  apparent  to  the  student  that  the  same 
rule  which  was  worked  out  in  Lecture  XL,  in  connection  with 
belt  gearing,  will  equally  apply  to  the  case  of  wheel  gearing, 
where  there  are  an  equal  number  of  drivers  and  followers.  In 
the  accompanying  figure,  where  there  are  three  drivers  and  three 
followers, 

Let  DI}  D,,  D,  =  Diameters  of  the  drivers. 
„    Fp  Ff,  F,  =  Diameters  of  the  followers. 
„    ND  ,  NPt   =  Number  of  revolutions  in  the  same  time  of  th» 
first  driver  and  the  last  follower. 

Then,  following  the  same  reasoning  as  was  expounded  in  Lecture 
XL  for  the  velocity  ratio  of  belt  gearing,  we  have 


134 


LECTURE    XII. 


The  speed  of  the  first  driver  x  ' 
the  successive  diameters  of 
the  drivers 


x  T>2  x  D3 


Or,  . 


:The  speed  of  the  last  follower 
x  the  successive  diameters 
of  the  followers. 

NF3  x  FL  x  F2  x  F3 

Fs/    T^     v    1R^ 
i      X     -C  „     X     J? - 


x  D2   X   D3 


The  speed  of  first  driver     _  Product  of  the  diameters  of  the  followers. 
tThe  speed  of  last  follower       Product  of  the  diameters  of  the  drivers. 
In  the  above  equation  we  may  substitute  the  radii,  or  the  cir- 

H 


SIDE  VIEW.  PLAN. 

WHEEL  GEAKING  IN  A  TEIPLE  PUKCHASE  WINCH 

cumferences,  or  the  number  of  teeth  in  the  drivers  and  in  the 
followers  respectively,  for  their  diameters ;  consequently, 

Let  rD  ,   rD3,  rD   =  Radii  of  the  respective  drivers. 
»   CD[>  Cv  CD^  =  Circumferences  ,, 

„   Ti1,  nD  ,  n3  =  Number  of  teeth  in 

3  =  Radii  of  the  respective  roilowers. 


/,  CF2,  CF3  =  Circumferences 
S  w_, ,  np*  =  Number  of  teeth  in 


Then, 
Or, 


N 


x  r 


J), 


,    XT* 
1          Dl 

L  x  CD  x  CD  x  CD  =  NF  x  Cr  x  CF  x 

123  312 


PRINCIPLE   OF   WORK  APPLIED  TO    WIIEEL-GEAKING.     135 


Or,        ND  x  nv  x  WD   x  n^  =  N^  x  n  >         a 

EXAMPLE  I.—  Three  drivers  of  10,  20,  and  30  teeth  each,  gear 
respectively  with  three  followers  of  40,  80,  and  120  teeth  each. 
Ascertain  the  velocity  ratio  between  the  first  driver  and  the  tost 
follower. 

By  the  above  formula  — 

ND  x  TIDI  x  nD^  x  n»3   =  NF;jX  n¥^  x  np^nr^ 
N        nFl  x  np  x  nF 


Substituting  the  coire-^ 

spending      numerical  I  ND  _  40  x  80  x  120     4x4x4     64 
values  for  the  letters,  j  ^  ~~  i0  x  20  x  30    =         i         "1 
we  get  } 

Principle  of  Work  applied  to  Wheel-gearing.—  Beferring 
to  the  previous  figure,  it  is  perfectly  evident  from  the  former 
applications  to  other  machines  of  the  "  principle  of  work,"  that, 
neglecting  friction,  the  force  applied  (to  the  handles  of  the 
machine)  x  the  distance  through  which  it  acts,  will  be  equal  to  the 
weight  raised  x  the  distance  through  which  it  is  elevated. 

Let  P  =  Push  applied  to  the  handles  in  Ibs. 

„  R  =  Radius  or  leverage  at  which  P  acts. 

w  W  =  Weight  raised  by  the  rope  on  the  barrel  B. 

„  r  =  Radius  or  leverage  with  which  W  acts. 

„  Dj,  D,,  Ds  =  Diameters  of  the  driving  wheels. 
„  F,,  F8,  F,=  Diameters  of  the  following  wheels. 
„  ND  =  Number  of  revolutions  of  the  first  driver,  "Dv  or  of 

the  handles,  H. 
„  NP  =  Number  of  revolutions  in  the  same  time  of  the  last 

follower,  F3,  or  of  the  barrel,  B. 
Then,  by  the  principle  of  work  and  neglecting  friction— 

P  x  its  distance  *  =  W  x  its  distance. 
t.«.,     .  P  x  2?rR  x  ND  =  W  x  2?rr  x  Np 
(Divide  both  sides  of  the  equation  by  2ir) 
.%  PxRxND  =WxrxNP 
~  PxR    NF  P8    NPsxr 

=  or= 


*  It  is  evident  that  in  order  to  'obtain  the  distance  through  which  P 
acts,  we  must  multiply  the  circumference  of  the  circle  described  by  the 
handles  by  the  number  of  revolutions  they  make  ;  and  in  the  same  way 
the  circumference  of  the  fearrel  must  be  multiplied  by  the  revolutions 
which  it  makes  in  the  same  time,  in  order  to  get  W's  distance. 


Of    THE 

UNIVERSITY 


136  LECTURE    XII. 

But  by  the  previous  equation  for  velocity  ratios, 
NF      D,  x  D2  x  P, 
ND'-^XF.XF, 
P  x  R     D!  x  D2  x  P8 
"W^Tr=F1xFaxF, 

Or,  P  x  R  x  F!  x  F2  x  F3  =  W  x  r  x  Dx  x  D2  x  D, 
Hence  the  general  rule  for  work  done  in  wheel-gearing  "Pxits 
leverage  x  the  diameters  (or  radii,  or  circumferences,  or  number  of 
teeth)  of  all  the  followers  =  W  x  its  leverage  x  the  diameters  (or  radii, 
or  circumferences,  or  number  of  teeth)  of  all  the  drivers. 

EXAMPLE  II. — It  four  men  exert  a  constant  force  of  15  Ibs. 
each  on  the  handles  of  a  compound  crab  or  winch  (such  as  that 
illustrated  by  the  previous  figure),  and  if  the  leverage  of  the 
handles  is  15",  whilst  the  weight  to  be  raised  acts  on  the  barrel 
or  drum  at  a  leverage  of  5'',  what  load  will  they  lift  if  the 
respective  diameters  of  the  drivers  are  12",  20",  and  20";  and 
of  the  followers,  36'',  80''  and  100",  neglecting  friction? 

ANSWER. — In  this  case,  P  =  4x15  =  60  Ibs.;  R=i5";  r=5"; 
1^=12";  D8=2o";  D3=2o";  ^  =  36";  F,  =  So",  and  F3=  100". 

By  the  above  formula  and  by  substituting  the  corresponding 
numerical  values  \ve  have — 

P  x  R  x  Ft  x  F8  x  F3  =  W  x  r  x  DL  x  D,  x  D, 
60  x  15''  x  36"  x  80"  x  ioo"=  W  x  5"  x  12"  x  20"  x  20" 

3345 
60  x  4ft  xg0x  $0x|00 

$  x  U  x  £0  x  ^0 
Or,       ....      W  =  6o  x  3  x  3  x  4X  5  =  10,800  Ibs. 

EXAMPLE  III. — If  40  %  of  the  force  applied  to  the  handles  be 
absorbed  in  overcoming  internal  friction  in  the  above  example  of 
a  winch,  what  weight  can  then  be  raised  by  the  four  men,  each 
acting,  as  before,  with  a  constant  force  of  15  Ibs.  ? 

ANSWER. — If  40  %  of  the  applied  force  be  lost  in  overcoming 
friction,  then  only  60  %  is  left  for  effective  work,  or  the  efficiency 
or  modulus  of  the  machine  is  said  to  be  0-6.* 

Consequently,        100  :  60  :  :  10,800  Ibs.  :  x  Ibs. 

60  x  10,800 
.«.  x=  — -? =  6480  Ibs. 

100 

*  The  term  modulus  of  a  machine  is  only  another  expression  for  the  more 
ppropriate  phrase,  efficiency  of  a  machine. 


LECTURE    XII. QUESTIONS.  137 

LECTURE  XII.— QUESTIONS. 

f .  When  two  circular  discs  with  fixed  centres  are  in  firm  contact  and 
roll  uniformly  together,  state  and  prove  the  rule  for  estimating  their 
relative  speeds  of  rotation. 

2.  Define  the  pitch  circle  of  a  toothed  wheel.    When  two  pitch  circles, 
A  and  B,  of  diameters  2  and  3  respectively,  roll  together,  prove  that  the 
angular  velocity  of  A  is  to  that  of  B  as  3  to  2.     Three  spur  wheels,  A,  B,  C, 
with  parallel  axes,  are  in  gear.     A  has  8  teeth,  B  has  32  teeth,  and  C  has 
42  teeth.     How  many  turns  will  A  make  upon  its  axis  while  C  goes  round 
8  times  ?     Why  is  B  termed  an  idle  wheel  J     Ant.  42  turns.     (Sec  Aote  to 
Question  10  re'ldle  Wheel.) 

3.  What  is  the  pitch  of  a  tooth  in  a  spur  wheel  ?    Two  parallel  shafts, 
whose  axes  are  to  be  as  nearly  as  possible  2  feet  6  inches  apart,  are  to  be 
connected  by  a  pair  of  spur  wheels,  so  that  while  the  driver  runs  at  100 
revolutions  per  minute,  the  follower  is  required  to  run  at  only  25  revolu- 
tions per  minute.      Sketch  the  arrangement,  and  mark  on  each  wheel  its 
diameter  and  the  number  of  teeth,  supposing  the  pitch  of  a  tooth  to  be 
ii  inch. 

Ans.  The  follower  is  48  inches  diameter  with  120  teeth. 
The  driver  is  12          „  „  30      „ 

4.  Define  the  "pitch  surface"  and  the  "pitch  circle"  of  a  toothed 
wheel.     Two  parallel  axes  are  at  a  distance  of  10  inches,  and  they  are  to 
rotate  with  velocities  as  the  numbers  2  and  3  respectively.     What  should 
be  the  diameters  of  the  pitch  circles  of  a  pair  of  wheels  which  would  give 
this  motion.     Find  pitch  of  teeth  on  the  smaller  wheel  if  the  larger  has 
24  teeth  1    Ans.   12  ins.  and  8  ins.  ;  1-57  inch. 

5.  Sketch  and  describe  the  "rack  and  pinion  "  and  give  instances  from 
personal  observation  of  its  application.     A  pinion  of  3-2"  diameter  has 
teeth  of  i"  pitch,  and  gears  with  a  straight  rack  applied  to  a  sluice  gate. 
If  the  weight  of  the  sluice  and  rack  be  100  Ibs.  and  the  lever  handle 
descnbes  a  circle  of  40*2"  in  each  turn,  what  force  must  be  applied  to  the 
handle  to  lift  the  gate  ?    How  many  feet  will  the  sluice  be  lifted  by  six 
turns  of  the  handle  ?    Ans.  25  Ibs. ;  5  ft. 

6.  Sketch  the  arrangement  known  as  the  rack  and  pinion.     Apply  the 
"  principle  of  moments  "  and  the  "  principle  of  work"  to  find  the  relation 
between  the  force  applied  and  the  weight  raised  by  aid  of  this  machine. 
A  pinion  has  sixteen  teeth  of  |-inch  pitch  in  gear  with  a  rack.     If  the 
pinion  makes  3^  turns,  through  what  distance  has  the  rack  been  moved  ? 
If  the  pinion  is  turned  by  a  "handle  14  inches  long,  and  with  a  force  of 
35  Ibs.  applied  to  the  handle,  find  the  force  with  which  the  rack  is  urged 
lorward.     A  ns.  49  inches  ;  223  Ibs. 

7.  Deduce  the  formula  for  the  velocity  ratio  in  wheel  gearing  where  there 
are  three  drivers  and  three  followers,  and  state  the  rule  derived  therefrom 
in  general  terms.     Three  drivers  of  20,  30,  and  40  teeth  respectively  gear 
with  three  followers  of  40,  60,  and  80  teeth.     If  the  first  driver  makes  160 
revolutions,  how  many  revolutions  will  the  last  follower  make  1     Ans.  20. 

8.  In  the  previous  question,  if  the  handles  attached  to  the  first  driver 
have  each  a  radius  of  15%  and  the  drum  connected  to  the  last  follower  be 
15"  diameter,  what  force  must  be  applied  to  the  handles  in  order  that 
they  may  lift  1120  Ibs.   supposing  that  the  efficiency  of  the  machine  is 
70  per  cent.  1    Ans.  100  Ibs. 

9.  The  hour  and  minute  hands  of  a  clock  are  on  the  same  arbor  or  axis, 
and  the  hour  hand  takes  its  motion  from  the  minute  hand.    Devise  some 
train  of  wheels  for  connecting  the  two  hands. 


138  LECTURE   XII. — QUESTIONS. 

10.  How  would  you  determine  the  "pitch  circles,"  and  the  proper 
"  pitch  of  the  teeth "  for  a  pair  of  spur-wheels  ?  What  would  be  the 
diameter  of  the  pitch  circle  of  a  spur-wheel  having  80  teeth  of  f-inch  pitch  1 
Ant.  19  inches. 

Three  spur-wheels  A,  B,  C  are  on  parallel  axes,  and  are  in  gear.  A  has 
so  teeth,  B  has  35  teeth,  and  C  has  55  teeth.  How  many  revolutions  upon 
its  axis  will  be  made  by  A  for  every  4  revolutions  of  C  ?  Why  is  B  called 
an  idle  wheel  and  what  is  its  use  ?  Ans.  22  revs. 

Note  re  "Idle  or  Intermediate  Wheel" — When  a  wheel  is  carried  on  a 
separate  axle  and  is  interposed  between  two  other  wheels  (or  is  introduced 
into  a  train  of  wheels),  merely  for  the  purpose  of  changing  the  relative 
directions  of  rotation  of  the  first  and  last  wheel,  then  such  intermediate 
wheel  is  called  an  idle  wheel,  because  it  does  not  affect  the  numerical 
value  of  the  train,  but  only  its  sign.  For  examples,  see  Vol.  I.  of  my 
"  Text  Book  of  Applied  Mechanics  and  Mechanical  Engineering,"  Lectun* 


HOTES  AND    QUESTIONS*  139 


(      140    ) 


LECTUKE   XIII. 

CONTENTS.  —  Single-purchase  Winch  or  Crab— Example  I. — Double-pur- 
chase Winch  or  Crab — Example  II. — Wheel  Gearing  in  Jib-Cranes— 
Questions. 

IN  this  Lecture  we  will  apply  the  principles  and  formulae  dis- 
cussed in  the  previous  one  to  a  few  practical  applications  of 
gearing  in  machines  for  lifting  weights. 

Single- pur  chase  Winch  or  Crab. — The  comparatively  small 
working  advantage  of  the  simple  hand-driven  wheel  and  axle  or 


SINGLE-PURCHASE  WINCH  OR  CRAB. 

By  Messrs.  Loudon  Bros.,  Glasgow.   . 

handle  and  winch  barrel  (illustrated  in  Lecture  V.)  renders  it  unfit 
for  lifting  greater  weights  than  one  or  two  hundredweight.  Con- 
sequently, whenever  heavier  loads  have  to  he  raised  by  manual 


SINGLE-PURCHASE    WINCH  OR  CRAB.  141 

labour,  one  of  the  most  useful  machines  that  can  be  employed  is 
the  single-purchase  crab.  As  will  be  seen  from  the  accompanying 
perspective  view,  this  machine  consists  of  a  pair  of  lever  handles 
fitted  to  the  squared  ends  of  a  round  shaft  carrying  a  pinion. 
This  pinion  gears  with  a  spur-wheel  keyed  to  a  lower  shaft,  upon 
which  is  also  fixed  a  drum  or  barrel.  To  a  hook  or  eye  on  the 
inside  neck  of  the  left-hand  flange  of  this  barrel  the  rope  or  chain 
(to  be  connected  to  the  load)  is  attached.  Therefore,  the  turning 
of  the  handles  causes  the  barrel  to  rotate  and  wind  the  rope  upon 
it,  thereby  elevating  the  load.  Both  shafts  turn  in  bearings 
bored  in  the  cast-iron  end  standards  or  A  frames.  These  frames 
are  bound  tightly  together  and  kept  at  a  fixed  distance  apart  by 
three  wrought-iron  collared  stays,  secured  on  the  outside  by  screw 
nuts.  To  the  outside  right-hand  end  of  the  barrel  shaft  there  is 
keyed  a  friction  pulley  acted  on  by  a  steel  brake-strap,  for  the 
purpose  of  enabling  the  labourers  to  lower  a  load  gently  or 
quickly  without  enduring  the  stress  and  danger  of  pulling  back 
on  the  handles.  In  fact,  after  applying  the  brake-strap  by  its 
outstanding  handle,  they  can  lift  the  claw  pawl  which  is  hinged  on 
the  top  stay  (and  which  keeps  the  pinion  in  gear  with  the  spur- 
wheel  when  in  the  position  shown  on  the  figure)  and  by  pulling 
the  upper  shaft  to  the  right,  disengage  the  pinion  from  its  wheel. 
Then,  by  adjusting  the  pawl  into  the  other  groove  of  this  shaft, 
they  are  free  to  lower  the  load  by  the  brake  without  having  the 
handles  flying  round.  Between  the  right  hand  flange  of  the 
barrel  and  its  neighbouring  A  frame  there  is  a  ratchet-wheel  (not 
seen  on  the  figure).  This  ratchet-wheel  is  generally  cast  along 
with  the  barrel.  Its  pawl,  which  is  hinged  to  the  inner  side  of 
the  standard,  can  therefore  be  dropped  down  so  as  to  engage 
with  a  tooth  of  the  stop-wheel,  whenever  it  is  necessary  to  cease 
heaving  up  a  heavy  weight;  thereby  preventing  the  machine 
overhauling,  and  giving  the  labourers  freedom  to  leave  the  handles 
and  attend  to  other  duties. 

EXAMPLE  I. — In  a  single-purchase  crab  the  lever  handles  are  each 
1 6"  long,  the  diameter  of  the  barrel  is  8" ;  the  pinion  or  driver 
has  12  teeth,  and  the  wheel  or  follower  60  teeth.  If  two  mei 
apply  a  constant  force  of  20  Ibs.  each  to  the  handles,  and  are  just 
able  to  raise  a  weight  of  600  Ibs.  to  a  height  of  20  feet  in  two 
minutes,  find — (i)  the  theoretical  advantage;  (2)  the  working 
advantage ;  (3)  the  work  put  in  for  every  foot  the  weight  is 
Hf ted ;  (4)  the  work  got  out  for  every  foot  the  weight  is  lifted ; 
(5)  the  efficiency;  (6)  the  percentage  efficiency  of  the  machine; 
(7)  the  H.P.  developed  by  the  two  men. 

ANSWEB. — Referring  to  the  notation  in  last  Lecture,  we  have 
P  =  2  x  20  ibs.  —  40  ltw. ;  R=i6";  r  =  4";  nD=  12  teeth;  n,  =  6o 


142 


LECTUKE 


teeth  ;  WT  =  the  theoretical  weight  that  would  be  raised  if  there 
were  no  friction  j  WA  =  600  Ibs.  (the  actual  weight  raised)  ;  h  =  20 
feet. 

W 

(  i  )  Theoretical  advantage  =  -pr 

By  the  principle  of  work  (neglecting  friction.) 
P  x  by  its  distance*  =  WT  x  its  distance.* 

P  X  27rR  X  ny  =  WT  X  27JT  X  71D 

P  x    K,    x  n¥  =  WT  x    r 


(Substituting  the  above  numerical  values  we  get) 


i 

ft  x 

_,  WT     800     20 

Consequently,      .         .     -^~  =  --  =  -T— 
j-        40        * 

WA     600  Ibs.     15 

(2)  Working  advantage       =^p~=  40  lbs.  =Y 

(3)  Work  put  in  for  every  foot  WA  is  raised.     From  equation 
(i)  we  see  that  for  every  foot  WA  is  raised  P  must  have  gone 

20 
through  20  feet,  since  the  velocity  ratio  is  — 

.'.  P  x  20  =  40  Ibs.  x  20  =  800  ft.-lbs. 

(4)  Work  got  out  for  every  foot  WA  is  raised 

=  WAx  i'  =  6oo  Ibs.  x  i'  =  600  ft.-lbs. 

.  _  .  Work  got  out     600  ft.-lbs. 

(<;)  The  efficiency  *=  ^TF  —  r2    .  •     =o  -  FT-TI  —  =  •!$ 

Work  put  in      800  ft.-lbs.       /;> 

(6)  Tlie  percentage  efficiency    =-75     x     100    =    75% 

/  \  rr-L    TT  r>  j      7       7  T    J7.    ,  Work  put  in  per  minute 

(7)  The  H.P.  developed  by  the  two  men  =  — 

33,000 

-u-  v        8000  ft.-lbs.      1  i 

^•.H.r.  =  -  =  —  bare,  or  «  of  a  horse-power  per  man. 

*  It  is  evident  that— 

P  x  i  turn  of  handles     Number_of  teeth  in  the  driver 
WTx  i  turn  of  barrel  =  NumbeTof  teeth  irTthelollower. 
\)r,     .         .         P  x  27rK  :  W 


DOUBLE-PURCHASE  WINCH  OR  CRAB. 


143 


Double-Purchase  Winch  or  Crab. — It  will  be  observed,  from 
an  inspection  of  the  accompanying  photographic  view  of  a  "  Double- 
purchase  Crab,"  that  the  chief  difference  between  it  and  the  single- 
purchase  one  is,  that  it  has  another  pinion  and  wheel,  with  a  view 
of  increasing  the  actual  or  the  working  advantage,  and  thus 
enabling  the  same  manual  force  to  lift  a  greater  load,  although 
by  taking  a  longer  time.  It  is  also  larger,  heavier,  and  stronger. 


DOUBLE-PURCHASE  WINCH  OB  CRAB. 
By  Messrs.  London  Bros.,  Glasgow. 

As  will  be  seen  from  the  figure,  it  may  be  used  as  a  single-purchase 
winch  by  simply  lifting  the  claw-pawl  hinged  on  the  top  stay,  and 
pushing  the  handle  shaft  forward  until  its  left-hand  pinion  gears 
with  the  large  spur  wheel,  and  then  letting  the  pawl  drop  on  to 
bearing  to  the  right  hand  of  the  two  collars  on  this  shaft.  By  so 
doing,  the  right-hand  pinion  or  first  driver  (when  in  double-pur- 
chase gear)  is  freed  from  the  first  follower,  and  both  are  inactive 
during  the  time  it  is  used  in  single  purchase,  but  the  second 


144 


LECTURE   XIII. 


driver  is  still  in  gear  and  is  turned  round  by  the  spur  wheel.  The 
brake  strap  pulley  is  keyed  to  the  second  shaft  (carrying  the  first 
follower  and  second  driver),  and  can  be  used  for  lowering  the  load 
without  the  handles  coming  into  action  (as  described  in  the  pre~ 
vious  case)  by  placing  the  claw-pawl  between  the  two  collars  in  the 
first  motion  shaft.  When  the  pawl  is  in  this  position,  both  of  the 
pinions  on  this  shaft  are  out  of  gear.  The  machine  may  be  locked 
arid  the  load  left  suspended  by  dropping  the  ratchet  into  the 
ratchet-wheel  cast  on  the  right-hand  end  of  the  barrel  in  the  same 
way  as  with  the  single-purchase  crab.  A  triple-purchase  winch 
was  illustrated  in  Lecture  XII.,  and  the  student  should  again 
refer  to  the  plan  and  the  side  elevation  of  its  gearing. 

EXAMPLE  II.  —  Four  men  exert  a  force  of  20  Ibs.  each,  on  the 
handles  of  a  double-purchase  crab,  which  are  15"  long.  The 
driving  pinions  have  12  teeth  each,  the  followers  24  and  48  teeth 
respectively,  and  the  diameter  of  the  barrel  is  10".  Find  the 
weight  that  can  be  raised  if  25  per  cent,  of  the  work  put  in  be 
absorbed  in  overcoming  friction. 

ANSWER.  —  Here  P  =  4X2o  =  8o  Ibs.  ;  R  =  1  5"  ;  nDL  =  1  2  ;  nDt  = 
12;  7iFi  =  24;  nF>  =  48;  r=s". 

By  the  formula  deduced  in  the  previous  lecture  from  the 
principle  of  work  (neglecting  friction), 

P  x  R  x  nf  x  np  =  WT  x  r  x  nD  xnD 


324 
After  cancelling,  we  get  — 

8ox3X     2  x  4  =  WT=  1920  Ibs. 

If  1920  Ibs.  of  work  be  expended  by  the  men  and  25  per  cent. 
of  this  be  lost  work,  there  remains  75  per  cent,  as  useful  work. 

Or,      .         .         100  :   75   ::    1920  Ibs.  :  WA. 

Weight  actually  raised  =  WA  =  1440  Ibs. 

Wheel  Gearing  in  Jib  Cranes.  —  In  Lecture  VIII.  the  side 
view  of  a  jib  crane  was  given  for  the  purpose  of  exemplifying  the 
stresses  on  the  jib,  tie-rods,  and  central  pillar.  We  now  illustrate 
A  swing  jib  crane  on  a  bogie  and  rails,  to  show  that  the  frame- 
work and  lifting  gear  are  simply  those  of  an  inverted  double- 
purchase  crab  with  the  toothed  wheels  placed  outside  the  standards 
instead  of  inside  as  in  the  ordinary  winch.  The  snatch  block 
pulley  (previously  referred  to  in  Lecture  VII.),  to  the  hook  of 
which  the  load  is  attached,  doubles  the  theoretical  purchase  or 
advantage  of  the  winch  gearing,  and  therefore  one,  two  or  more 
men  can  lift  nearly  double  the  weight  by  aid  of  this  simple 
addition  to  the  machine.  Large  crones  of  th'S  description  are 


WHEEL    GEARING  IN   JIB   CRANES. 


145 


fitted  with  slewing  or  horizontal  turning  gear,  to  enable  the  load 
when  lifted  to  be  swung  round  before  depositing  it  in  a  truck. 


hold  of  a  ship,  or  on  a  machine  tool.     This  latter  gear  consists  of 
a  horizontal  wheel  on  the  top  of  the  vertical  central  cast-iron 


146  LECTURE 

supporting  boss,  with  which  is  geared  a  bevel  pinion,  actuated  by 
aid  of  a  lever  handle. 

In  order  to  prevent  the  whole  machine  being  capsized  by  a 
heavy  load,  there  is  a  back  balance  weight,  and  further  the  bogie 
wheels  can  be  clamped  to  the  rails.  The  back  balance  weight  also 
tends  to  cancel  the  severe  right  angle  stress  on  the  central  pillar 
which  was  specially  taken  notice  of  in  Lecture  VIII.  We  will 
defer  the  description  of  heavy  steam  power  cranes,  tripods  and 
shear  legs  to  our  Advanced  Course. 


LBCTUEB  XIII.— QUESTIONS— (continued) 

10.  What  do  you  understand  by  the  efficiency  of  a  machine,  and  how  is 
it  measured?     In  a  single  purchase  crab,  the  pinion  has  12  teeth  and  the 
wheel  has  78  teeth,  the  diameter  of  the  barrel  being  7  inches,  and  the 
length  of  the  lever  handle  14  inches.     It  is  found  that  the  application  of  a 
force  of   15   Ibs.  at  the  end  of  the  handle  suffices  to  raise  a  weight  of 
280  Ibs.     Find  the  efficiency  of  the  machine.     Ans.  072  ;  or  72  per  cent. 

jo.  In  a  crane  an  effort  of  122  Ibs.  just  raises  a  load  of  3265  Ibs.  What 
is  the  mechanical  advantage  1  If  the  efficiency  be  60  per  cent.,  what  is 
the  velocity  ratio  1  Ans.  Mech.  Adv.  2676  :  i  ;  Vel.  Ratio  44-6  :  I. 

(B.  of  E.,  1903.) 

11.  In  a  crane,  a  force  of  3  Ibs.  applied  at  the  handle  is  found  to  raise  a 
weight   of  42  Ibs.,  and  a  force   of  8  Ibs.  a  load  of    120  Ibs.      If   the 
relation  between  the  force  applied  and  the  weight  raised  is  represented  by 
the  straight  line  law,  obtain  the  equation   expressing  the  relationship 
between  them  ;  and  if  the  velocity  ratio  between  the  force  applied  and 
the  weight  raised  is  18,  estimate  the  efficiency  of  the  crane  when  lifting  a 
load  of  200  Ibs.    Am.  F=5W/78  +  4/i3  ;  84*6%. 

(0.  &  G.,  1904,  0.,  Sec.  A.) 

12.  In  an  electrically  driven  overhead  crane  a  weight    of  5    tons   is 
raised  at  the  rate   of  90  feet  per  minute.     What  is  the  horse-power? 
Convert  this  into  watts.     The  motor    drives    through    gearing    whose 
efficiency  is  70  per  cent.     How  many  amperes  of  current  must  be  supplied 
to  the  motor  at  a  voltage  of  220  if  the  efficiency  of  the  motor  is  87  per 
cent.  ?    Ans.  30-5  H.P.  22,787  watts  ;  170  amperes. 

[Note  that  i  horse  power  =  746  watts  ;  and  i  ampere  multiplied  by  i  volt 
is  i  watt.J  (B.  of  E.  1905.) 


LECTUEE  Xm. — QUESTIONS. 

LECTUBE  XIII.— QUESTIONS. 

1.  Where  wheelwork  is  employed  to  modify  motion,  as  in  a  crane,  or  in 
the  double-geared  headstock  of  a  lathe,  how  is  the  change  of  motion 
calculated  ?    Write  down  the  formula  employed. 

2.  Sketch  a  side  elevation  and  end  view  of  a  single  purchase  crab,  and 
describe  the  same  by  aid  of  an  "index  to  parts."    Apply  the  principle  of 
work  in  solving  the  following  question : — The  lever  handle  of  a  crab  is 
three  times  the  diameter  of  the  drum,  and  the  wheelwork  consists  of  a  pinion 
of  16  teeth  driving  a  wheel  of  80  teeth ;  what  weight  will  be  lifted  by  a 
force  of  30  Ibs.  acting  at  the  end  of  the  lever  handle  1    An*.  900  Ibs. 

3.  Describe,  with  a  freehand  sketch,  a  single  purchase  lifting  crab.    The 
leverage  of  the  handle  of  the  crab  is  16  in.,  and  there  is  a  pinion  of  20  teeth 
driving  a  wheel  of  100  teeth,  the  diameter  of  the  barrel  being  8  in.    Assign 
the  relative  proportions  of  the  working  parts,  and  estimate  the  theoretical 
advantage.    What  weight  would  be  raised  by  a  man  exerting  a  force  of 

15  Ibs.  on  the  lever  handle,  neglecting  friction  ?    Ans.  300  Ibs. 

4.  A  weight  of  4  cwt.  is  raised  by  a  rope  which  passes  round  a  drum 
3  feet  in  diameter,  having  on  its  shaft  a  toothed  wheel  also  3  feet  in 
diameter.     A  pinion,  8  inches  in  diameter,  and  driven  by  a  winch-handle 

1 6  inches  long,  gears  with  the  wheel.     Find  the  force  to  be  applied  to  the 
winch-handle  in  order  to  raise  the  weight.    Ans.  112  Ibs. 

5.  In  a  lifting  crab  the  lever  handle  is  14  inches  long,  the  diameter  of  the 
drum  is  6  inches,  and  the  wheel  and  pinion  have  57  and  n  teeth  respect- 
ively.   Find  the  weight  in  pounds  which  could  be  raised  by  a  force  of  50 
Ibs.  applied  to  the  lever  handle,  friction  being  neglected.     Ans.    1209  Ibs. 

6.  In  a  crane  there  is  a  train  of  wheelwork,  the  first  pinion  being  driven 
by  a  lever  handle  ;  and  the  last  wheel  being  on  the  same  axis  as  the  chain 
barrel  of  the  crane.     The  wheelwork  consists  of  a  pinion  of  1 1  gearing 
with  a  wheel  of  92,  and  of  a  pinion  of  12  gearing  with  a  wheel  of  72,  the 
diameter  of  the  barrel  being  18  inches  and  that  of  the  circle  described  by 
the  end  of  the  lever  handle  being  36  inches  ;  find  the  ratio  of  the  pull  to 
the  weight  raised,  friction  being  neglected.     Ans.  1 1  :  1 104. 

7.  In  a  3O-ton  crane  the  tension  of  the  chain  as  it  runs  on  the  winding 
barrel  is  7$  tons,  the  barrel  is  2  feet  in  effective  diameter,  and  the  spur 
wheel  connected  with  it  is  4  feet  in  diameter  on  the  pitch  line ;  what 
pressure  will  come  upon  the  teeth  of  the  spur  wheel,   supposing  such 
pressure  to  act  on  the  pitch  line  (friction  is  neglected)  ?   A  n.«.  375  tons. 

8.  The  crank  of  an  engine  is  2'  long,  and  the  diameter  of  the  fly-wheel 
is  icf ;  also  the  fly-wheel  has  teeth  on  its  rim,  and  drives  a  pinion  3'  in 
diameter.      If  the  mean  pressure  on  the  crank  pin  be  7^  tons,  what  is 
the  mean  driving  pressure  on  the  teeth  of  the  pinion  1    Ans.  3  tons. 

9.  Draw  to  scale  a  side  elevation,  end  view  and  plan  of  a  double  purchase 
crab,  and  describe  the  same  by  aid  of  an  "  index  to  parts."     If  four  men 
each  exert  a  constant  force  of  15  Ibs.  on  the  handles  of  such  a  crab  ;  if  the 
handles  have  a  leverage  of  1 6  inches  whilst  the  barrel  is  16  inches  diameter, 
and  if  the  drivers  have  12  teeth  each  while  the  followers  have  24  and  60  teeth 
respectively ;    find    the  weight  which    they  could    balance    neglecting 
friction.     If  30  per  cent,  of  the  work  put  in,  be  taken  up  in  overcoming 
friction,  what  load  can  they  lift?      State    (i)    theoretical    advantage; 
(2)  working  advantage  ;  (3)  work  put  in  when  lifting  the  load  i  foot  ; 
(4)  the   work  got   out ;    (5)   the  percentage    efficiency ;    (6)  the  height 
through  which  they  would  lift  the  load  in   i  minute  if  each  man  de- 
veloped i  H.P.     AM.  1200  Ibs.  ;  840  Ibs.;  (i)  20 :  i  ;  (2)  14: 1    (3)  1200 
ft. -Ibs.  ;  (4)  840ft.-lbs-,  (5)  70  per  cent.;  (6)  13- 75  ft. 


(     '48    ) 


LECTURE  XIV. 

CONTENTS.— Screws— The  Spiral,  Helix,  or  Ideal  Line  of  a  Screw  Thread 
— The  Screw  viewed  as  an  Inclined  Plane— Characteristics  and  Con- 
ditions to  be  Fulfilled  by  Screw  Threads— Different  Forms  of  Screw 
Threads — Whitworth's  V-Threads — Whitworth's  Tables  of  Standard 
V-Threads,  Nuts  and  Bolt  Heads— Seller's  V-Thread— The  Square 
Thread— The  Hounded  Thread— The  Buttress  Thread— Right  and 
Left-hand  Screws— The  Screw  Coupling  for  Railway  Carriages  — 
Single,  Double  and  Treble  Threaded  Screws — Backlash  in  Wheel  and 
Screw  Gearings — Questions. 

Screws. — Every  one  is  more  or  less  familiar  with  the  form  and 
uses  of  the  screw  nail  for  securing  pieces  of  wood  together,  and  of 
the  bolt  with  its  nut  for  fixing  metal  plates  m  position  ;  but  every 
one  is  not  so  familiar  with  the  principle  upon  which  screws  are 
generated  and  act,  or  with  the  best  shape  to  be  given  to  a  screw 
under  different  circumstances.  We  shall  therefore  endeavour 
in  this  Lecture  to  explain  these  points  in  an  elementary  manner, 
instancing  a  few  examples  of  the  practical  applications  of  screws, 
but  reserving  for  the  following  Lecture  questions  on  the  work 
done  by  screws  and  their  efficiency. 

The  Spiral,  Helix,  or  Ideal  Line  of  a  Screw  Thread.— 
A  very  good  idea  of  the  form  of  a  screw  is  obtained  from  the  accom- 
panying figure,  which  represents  one  means  of  elevating  or  trans- 


SPIRAL  OK  SCREW  FOR  MOVING  GRAIN. 

ferring  grain,  flour  or  other  powdered  substances  from  one  part 
of  a  milling  works  to  another.  It  consists  of  a  steel  band  twisted 
around  a  cylindrical  shaft  in  a  continuous  and  uniformly  pitched 
spiral.  This  shaft  and  screw  are  placed  in  a  trough,  tube  or 
pipe.  The  grain  or  powdered  substance  is  fed  in  at  one  end 
of  the  pipe,  and  by  rotating  the  screw  with  a  wheel  or  lever  fixed 


THE  SCREW. 


149 


to  one  end  of  the  shaft,  the  loose  material  is  gradually  pressed 
forward  until  it  reaches  the  other  end,  from  which  it  may  be 
dropped  into  sacks  or  put  through  another  process.  It  is  evident 
from  an  inspection  of  the  figure  that  as  the  screw  is  turned  round 
by  the  lever,  the  particles  of  matter  are  forced  along  the  face  of 
the  continuous  inclined  plane  formed  by  the  spiral  steel  band. 

The  principle  upon  which  the  screw  acts  is,  therefore,  a  combination 
of  tfw  inclined  plane  and  the  lever. 

To  bring  this  view  of  the  case  still  more  forcibly  before  the 
student,  take  a  cylinder  and  fix  along  the  side  thereof  parallel  to 


FORMING  A  SCREW  THREAD  ON  A  CYLINDER, 

its  axis  (by  gum  or  drawing  pins)  a  rectangle,  ACDE,  of  paper 
or  white  cloth,  having  its  sides,  AC  and  DE  exactly  equal  to  the 
circumference  of  the  cylinder.  Then,  when  the  envelope  is  wound 
round  the  cylinder  by  the  turning  of  the  handle,  H  (in  the  direction 
shown  by  the  arrow  at  P),  it  exactly  covers  its  cylindrical  surface. 
On  the  outside  of  this  rectangle  when  unfolded,  draw  any  con- 
venient number  of  parallel  inclined  black  lines,  AB,  &c.,  equi- 
distant from  each  other  as  shown  by  the  figure,  and  again  wrap 
it  round  the  cylinder.  These  lines  will  be  found  to  form  a  con- 
tinuous spiral,  helix,  or  screw-thread  Line  from  one  end  cf  the 
cylinder  to  the  other.  And  the  side  AC  of  the  right-angled 
triangle  ACB  forms  the  circumference,  BC  the  pitch,  AB  the 
length  of  the  thread  (for  one  complete  turn  of  the  cylinder),  and 
the  angle  BAC  is  the  inclination  or  angle  of  the  screw. 

The  Screw  Viewed  as  an  Inclined  Plane. — Take  another 
cylinder  having  an  evenly  pitched  screw-thread  line  drawn  upon 
it.  Cut  a  sheet  of  flexible  cardboard  into  the  form  of  a  right- 
angled  triangle  with  its  height  BC  or  h  equal  to  the  pitch  (or  dis- 


LECTURE    XIV. 


tance  between  two  consecutive  threads  when  measured  parallel 
to  the  axis  of  the  cylinder) ;  AC  or  b  equal  to  the  circumference 
of  the  screw  and  wrap  it  round  the  cylinder,  taking  care  to  keep 
BC  parallel  to  the  axis.  Then  the  hypothenuse  AB  or  length  I 
of  the  inclined  plane  will  coincide  with  the  contour  of  the  screw- 
thread  for  one  complete  turn,  and  BAG  or,  a,  is  the  angle  of  the 
thread  to  the  plane  at  right  angles  to  the  axis  of  the  cylinder. 

Now  conceive  this  screw-thread  instead  of  being  a  mere  line 
to  be  an  inclined  plane  of  known  breadth,  as  in  the  case  of  the 
grain  elevator.*  Let  the  total  weight  of  material  being  urged 


FIGURE  TO  PROVE  THAT  A  SCREW  THREAD  is  AN  INCLINED  PLANE. 

forward  or  upwards  by  the  turning  of  the  screw  be  W  Ibs.,  and 
let  the  resistance  due  to  this  load  be  uniformly  distributed  along 
the  screw  thread  or  inclined  plane.  Then,  comparing  the  first  and 
the  third  figures,  it  is  evident  that  any  small  portion  of  the  load 
having  a  weight  ~YV2  Ibs.  will  have  a  corresponding  reaction  R2  Ibs., 
and  will  require  a  part  P2  Ibs.  (of  the  total  force,  P,  applied  to 
turn  the  screw  at  the  radius  at  which  this  portion  is  situated)  to 
move  it  along  the  screw-plane  against  the  frictional  resistance  Fr 
Imagine  the  work  done  to  be  transferred  to  the  inclined  plane, 
AB,  then  any  portion  of  the  load  having  a  weight  Wt  Ibs.  will 
have  a  corresponding  reaction  Rx  Ibs.,  and  will  require  a  part 
Px  Ibs.  (of  the  total  force,  P,  applied  parallel  to  the  base  to  pull 
the  whole  load  up  the  inclined  plane)  to  move  it  along  the  plane 
against  the  frictional  resistance  Fr  Now,  these  forces  act  in 
identically  the  same  way  as  the  second  case  of  the  inclined  plane, 
which  was  discussed  in  Lecture  IX.,  consequently — 


W 


R 


AC 

AC 

b 


CB 

CB 

h 


AB 

AB 

I 


*  Or,  that  the  screw-thread  has  a  certain  depth  as  measured  radially 
from  the  axis  of  cylinder. 


SCREW   THREADS.  I$I 

Or  P  __  C  B  _  h  eight  _  h  _         pitch  of  thread 

W     AC       base      b     circumference  of  screw. 

We  therefore  see  that  a  screw  may  be  treated  as  an  inclined 
plane  where  the  force  turning  the  screw — i.e.,  overcoming  the 
resistance  to  motion — acts  parallel  to  the  base  of  the  incline  The 
same  reasoning  may  be  applied  to  any  screw  turning  in  a  nut  or 
to  a  nut  turning  on  a  screw. 

Characteristics  of  and  Conditions  to  be  Fulfilled  by 
Screw  Threads. — The  essential  characteristics  of  a  screw-thread 
are  its  pitch,  depth,  and  form. 

The  principal  conditions  to  be  fulfilled  by  a  screw-thread  are : 
(i)  efficiency;  (2)  strength;  (3)  durability. 

(1)  The  efficiency  depends  on  the  pitch  and  the  friction,  and 
hence  on  the  pitch  and  form  of  thread. 

(2)  The  strength  depends  upon  the  form  or  the  shearing  thick- 
ness and  depth,  or  area  of  the  cross  section  parallel  to  the  axis. 

(3)  The  durability  depends  chiefly  on  the  depth — that  is,  upon 
the  extent  of  bearing  surface. 

Different  Forms  of  Screw  Threads.— Sir  Joseph  Whitworth, 
the  famous  tool  and  gun  manufacturer,  was  so  impressed  with 
the  great  inconvenience  and  loss  of  money  which  arose  from  the 
use  of  different  pitches  and  forms  of  threads  for  screws  and  nuts, 
that  he  published  the  following  tables  giving  the  dimensions  of 
what  has  now  become  known  as  the  Whitworth  standard.  Prior 
to  1841,  the  year  in  which  Whitworth  proposed  the  adoption  of 
standard  sizes  for  screws,  and  for  several  years  afterwards,  differ- 
ent engineering  works  in  this  country  not  only  used  different 
pitches  for  screws  of  the  same  diameter,  but  it  was  no  uncommon 
thing  to  find  a  want  of  uniformity  in  the  same  shop.  Now, 
every  one  in  Great  Britain  and  her  colonies  uses  the  Whitworth 
standard  sizes  for  V-threaded  bolts  and  nuts  of  J-inch  and  upwards, 
and  the  British  Association  standard  for  smaller  screws  in  electrical 
and  philosophical  instruments. 

Whitworth's  V  Thread.* — The  following  figures  of  a  Whit- 
worth thread  and  nut,  together  with  the  tables,  will  serve  to 
give  full  information  regarding  the  number  of  threads  per  inch 
for  different  diameters  of  screw-bolts,  nuts  and  bolt-heads,  <fcc. 

The  angle  between  opposite  sides  of  the  threads  and  of  the 
intervening  spaces  is  55°.  One-sixth  of  the  depth  of  the  thread 
is  rounded  off  at  both  the  top  and  the  bottom  for  the  purpose  of 
preventing  a  sharp  nick  at  the  bottom  (which  would  weaken  a 

*  For  a  description  of  Whitworth's  screw-taps,  plates,  stocks,  dies  and 
combs,  see  "  Workshop  Appliances  "  bj  Professor  Shelley.  And  for  a  table 
of  the  B.  A.  Standard  for  Small  Screws,  see  Munro  and  Jamieson's  Electrical 
Rules  and  Tables,  i6th  ed.,  p.  67. 


LECTURE  XIV. 


WHITWORTH'S  STANDARD  FOR  SCREWS  WITH  ANGULAR  THREADS. 


-8 

ITS 

T3 

,-g 

CO 

.g 

1| 

Old  Sizes, 
Inches. 

New  Standa 
Decimals  of 
Inch. 

No.  of  Threa 
per  Inch. 

Old  Sizes, 
Inches. 

New  Standai 
Decimals  of 
Inch. 

No.  of  Threa 
per  Inch. 

Old  Sizes, 
Inches. 

New  Standar 
Decimals  of 
Inch. 

48 

O'lOO 

12 

0'600 

4 

2f 

2*375 

40 

£ 

0*125 

1  1 

1 

0-625 

4 

22 

2-500 

32 

0*150 

II 

0*650 

4 

2i 

2*625 

24 

0*175 

II 

0-675 

3£ 

2j 

2*750 

24 

O'2OO 

II 

0-700 

21 

2-875 

24 

0-225 

IO 

1 

0-750 

3i 

3 

3'OOQ 

20 

4 

0-250 

10 

0-800 

34 

3^5 

2O 

0-275 

9 

1 

0-875 

31 

3^ 

3-50 

18 

0-300 

9 

0-900 

3 

31 

375 

18 

0-325 

8 

I 

I'OOO 

3 

4 

4*00 

18 

0-350 

7 

Is1 

1*125 

2j 

44 

4-25      , 

16 

1 

0-375 

7 

ii 

1-250 

2§ 

4-50 

16 

0-400 

6 

'1 

1-375 

2| 

4f 

475 

14 

0*425 

6 

ii 

1-500 

2| 

5 

5-00 

14 

0-4^0 

5 

if 

1*625 

2| 

54 

14 

0*475 

5 

if 

1*750 

2| 

5*50 

12 

£ 

0-500 

44 

Ig 

1*875 

5S 

575 

12 

0-525 

4£ 

2 

2-000 

23 

6 

6'oo 

12 

0-550 

4i 

2§ 

2*I25 

12 

0-575 

4 

24 

2-250 

WHITWORTH  VEE  THREAD. 


Angle  of  thread  =  55°.  One -sixth 
of  depth  is  rounded  off  at  top  and 
bottom. 

Number  of  threads  to  the  inch  in 
square  threads  =  J  number  of  those  in 
angular  threads. 

Depth  of  threads  =  0*64  pitch  for 
angular  =  0*475  pitch  for  square 
threads. 


WHITWORTH'S  GAS  THREADS. 


Diameter  in  Inches. 

I 

i 

t 

i 

1 

i 

ij 

i* 

ij 

2 

No.  of  threads  per  inch 

28 

19 

19 

M 

14 

II 

ii 

ii 

ii 

II 

WHITWORTH'S   STANDARD  NUTS  AND-  BOLTS.  153 


WHITWORTH  SCREW  NUT. 


WHITWORTH'S  STANDARD  NUTS  AXD  BOLT-HEADS. 


_^ 

1 

^ 

ll 

^ 

1 

•3 

*i 

i 

5 

fc 

m 

3 
CQ 

S 

£ 

•§  * 

11 

ll 

1 

H 

i* 

H 

ll 

i 

|| 

1^ 

"S  ~ 

~Sn 

£3 

•S  £ 

ts5 

—  "** 

-S  S 

§ 

g 

•£ 

•g 

ES 

1 

g 

•^ 

1 

|3 

5 

c- 

H 

5* 

.S 

a 

g 

g 

l» 

1 

0-338 

I 

0-1093 

0-0929 

ii 

1*8605 

il 

0^43 

0-942 

T* 

0-448 

l'* 

0*1640 

0*1341 

ii 

2*0483 

i| 

1-0937 

•o67 

i 

0-525 

i 

0-2I87 

0*1859 

if 

2*2146 

if 

I*203I 

•1615 

A 

0*6014 

A 

0-2734 

0-2413 

i-l 

2-4134 

i% 

I-3I25 

•2865 

1 

0-7094 

1 

0-3281 

0*2949 

i| 

2-5763 

i| 

I*42l8 

•3688 

A 

0*8204 

TB 

0-3828 

0*346 

if 

27578 

if 

1*5312 

•4938 

i 

0-9191 

i 

0-4375 

0*3932 

is 

3-0183 

U 

1*6406 

•5904 

T^ 

I  'OH 

I9? 

0-4921 

Q-4557 

2 

3'I49I 

2 

175 

7154 

1 

•101 

1 

0-5468 

0*5085 

-: 

3-337 

2g 

i  -8593 

•8404 

H 

•2O  I 

H 

0-6015 

0-571 

2i 

3'546 

2i 

1-9687 

1*9298 

1 

•3012 

I 

0-6562 

0*6219 

2| 

375 

2| 

2*0781 

2*0548 

H 

'39 

H 

0-7109 

0*6844 

2j 

3-894 

2| 

2-1875 

2*1798 

i 

•4788 

| 

0*7656 

07327 

2| 

4-049 

2| 

2-2968 

2*3048 

H 

'5745 

II 

0*8203 

07952 

2| 

4*181 

2| 

2-4062 

2-384 

i 

I  -6701 

i 

0-875 

0*8399 

3 

4-53I 

3 

2-625 

2-634 

154 


LECTURE  XIV. 


bolt  or  a  nut),  as  well  as  for  ease  in  manufacturing  them,  since  it 
would  be  practically  impossible  to  maintain  such  perfectly  sharp 
edges  in  the  stocks  and  dies  or  in  the  combing  tools  with  which 
such  bolts  and  nuts  are  generally  screwed.  Besides,  it  would  be 
most  inconvenient  to  handle  such  sharp-pointed  screws  if  they  had 
edges  tapering  right  off  to  55°,  and,  moreover,  it  would  serve  no 
useful  purpose,  for  such  a  thin  edge  cannot  materially  add  to  the 
strength  of  a  screw-thread. 

The  Whitworth  thread  is  stronger  than  any  other,  except 
the  buttress  one,  since  its  thickness  at  the  bottom  of  the  thread 
is  nearly  equal  to  the  pitch  of  the  screw.  The  compression  or 
grip  is  considerably  greater  than  with  the  square  thread,  because 
the  pitch  is  only  half  as  much  for  the  same  size  of  bolt.  The 
efficiency  of  the  Whitworth  V-thread  as  a  means  of  transmitting 
motion  is,  however,  small,  since  the  reaction  being  at  right  angles 
to  the  face  of  the  thread,  a  large  part  of  the  force  employed  in 
turning  the  screw  is  expended  in  tending  to  burst  the  enveloping 
nut.  This  very  inefficiency,  however,  adds  to  its  utility  as  a 
binder  for  all  kinds  of  machinery,  since  a  properly  fitted  nut  when 
once  screwed  down,  will  not  run  back  or  overhaul,  unless  the  pitch 
be  very  great  and  the  threads  be  well  oiled. 

Seller's  V-Thread.— In  the  United  States  of  America,  Seller's 
V-thread  is  used.     It  differs  from  the 
Whitworth  V-thread  in  that  the  angle 
between  the  opposite  sides  of  the  thread 
and  between  the  spaces  is  60°  instead 
of  55°,  also  the  depth  is  reduced  by  a 
sharp  flat  top  and  bottom,  equal  to  one- 
eighth    of  the  total    depth,  instead  of 
being  rounded.     This  is  rather  a  curious 
divergence  from  the  usual  American  practice,  where  almost  all 
other    parts    of   their   excellent    machine  tools   are   beautifully 
rounded  off  by  symmetrical  curves. 

The  Square  Thread — Since  the  bearing  surface  in  this 
thread  is  very  nearly  at  right  angles  to  the  direction  of  pressure 
and  resistance  it  is  much  used  for  transmitting  motion.  Of  the 


— P--*? 


SELLER'S  V  THREAD. 


SQUARE  THREAD. 


SCREW    THREADS. 


155 


force  applied  to  turn  this  screw  there  is  only  a  small  percentage 
dissipated  in  tending  to  burst  the  nut ;  consequently,  its  efficiency 
is  greater  than  that  of  the  V-thread.  As  will  be  seen  from  the 
accompanying  figure,  the  thickness  of  the  thread  and  the  width  of 
the  space  are  made  equal,  in  single- threaded  screws,  therefore  the 
shearing  thickness  is  greatly  reduced,  and  consequently  its  strength 
is  less  than  the  V-thread.  The  durability  is,  however,  greater 
than  in  any  other  form  of  screw,  for  there  is  a  larger  bearing  sur- 
face presented  in  the  best  manner  to  resist  pressure. 

The  Rounded  Thread  — This  form  is  simply  a  modification 
of  the  square  thread,  in  order  to  facilitate  the  quick  engaging  and 

Fif-H 


ROUNDED  THREAD 

disengaging  of  a  leading  motion  screw  by  its  nut  in  machine 
tools,  or  where  a  screw  has  to  be  subjected  to  rough  usage.  Its 
efficiency  and  durability  are  less  than  the  square  thread,  but  its 
strength  is  much  greater,  since  the  shearing  thickness  is  greatly 
increased  by  the  fillets  at  the  bottoms  of  the  thread. 

The  Buttress  Thread. — In  such  cases  as  the  raising  and 
lowering  of  heavy  guns  for  the  purposes  of  sighting  and  loading 
them,  where  the  pressures  are  always 
in  one  direction,  then  this  form  of 
thread  is  adopted,  because  its  strength 
is  a  maximum,  the  loss  due  to  friction 
is  a  minimum,  and  there  is  very  little 
tendency  to  burst  the  nut.  The 
efficiency  is  quite  equal  to  that  of  the 
square  thread,  although  the  durability 
is  lessened  by  the  fact  that  a  certain 
amount  of  wear  would  diminish  the  depth  of  the  thread.  The 
strength  is,  however,  nearly  double,  since  the  shearing  thick- 
ness is  double.  It  therefore  possesses  the  advantages  of  the  V 
and  the  square  thread  where  pressures  have  to  be  applied  in  one 
direction. 

A  slight  toodificaiion  of  the  buttress  thread  is  used  for  wood 
screws.  These  bolts  take  a  very  firm  hold  of  any  material  into 
which  they  can  be  screwed.  Consequently,  they  are  used  for 
screwing  thick  planks  of  wood  together,  and  binding  down  plates  or 


BUTTRESS  THREAD. 


156 


LECTURE    XIV. 


other  planks  where  vibration  and  stresses  would  start  and  lessen 
the  grip  of  the  ordinary  V-thread.     They  are  much  used  by  ship 


COACH  OB  WOOD  SCREW  WITH  SEMI- BUTTRESS  THREAD. 

carpenters  and  erectors  of  light  scaffolding,  and  are  sometimes 
called  holding-down  bolts. 

Right-  and  Left-hand  Screws. — A  right-hand  screw,  when 
being  turned  forward  or  into  a  nut,  rotates  in  a  right-handed 
way  or  in  the  direction  of  motion  of  the  hands  of  a  watch,  whereas 


RIGHT  HANDED  SCREW 


LEFT  HANDED  SCREW 

a  left-hand  screw  moves  in  the  opposite  or  left-handed  direction, 
as  shown  by  the  direction  of  the  circular  arrows  in  the  above 
figure. 

The  Screw-coupling  for  Railway  Carriages  is  a  very  good 


SCREW  COUPLING  FOR  RAILWAY  CARRIAGES, 


SCREW-COUPLING   FOR   RAILWAY  CARRIAGES.  157 

example  of  the  use  of  right-  and  left-hand  screws.  When  two 
carriages  are  brought  together,  the  free  link  hanging  from  the 
hook  of  one  of  them  is  placed  on  the  hook  of  the  other  one.  The 
porter  then  turns  the  central  lever  by  rotating  the  ball  in  a  circle, 
thereby  screwing  lx>th  the  right-  and  the  left-hand  screws  into 
their  respective  nuts,  which  consequently  draws  the  hooks  to  ward 
each  other,  and  couples  the  carriages  tightly  together. 

EXAMPLE.  —  If  the  pitch  of  each  screw  is  J",  the  length  of  the 
lever  arm  or  distance  from  the  axis  of  the  screw  to  the  centre  of 
the  ball  is  14";  and  if  the  railway  porter  pulls  the  ball  with  a 
force  of  40  Ibs.  when  the  carriages  are  brought  tightly  together, 
what  will  be  the  tension  on  the  screw  threads  ? 

ANSWER.—  Here  ;?=  J";  6  =  27rll  =  2  x  —  x  14"  =  88";  P  =  4olbs. 

The  formula  for  the  ratio  of  P  to  W  in  the  case  of  a  single 
screw  given  in  this  Lecture  is 


But  there  are  two  screws,  and  for  every  complete  turn  made  by 
P,  the  stress  W  would  be  moved  through  twice  tho  pitch  of  one 
screw  or  through  2  x  J"  =  i". 

P  x  b     40  x  88 
.-.  W  =  -  =  -       -  =  3520  Ibs. 

NOTE.  —  We  may  answer  this  question  directly  from  the  "  Principle  of 
Work."  Students  'should  be  trained  to  work  out  each  question  from  first 
principles  rather  than  from  formulae  ;  for,  by  a  too  free  use  of  formulae  thej 
are  apt  to  lose  sight  of  principles. 

Let  the  lever  make  one  complete  turn,  then  eacli  nut  will  advance  along 
its  own  screw  a  distance  equal  to  thepitch.  Therefore  the  two  nuts,  and  con- 
sequently the  two  carriages,  will  be  brought  nearer  by  a  distance  equal  to 
twice,  the"  pitch,  or,  =  2  x  p. 

By  the  principle  of  work,  and  neglecting  friction  — 

Work  got  out  =  Work  put  in 
Or,       .        .         Wx2/>  =  Px2irR 
_Px2irR 


Or, 


Single,  Double,  and  Treble-threaded  Screws.—  As  has  been 
previously  stated,  both  the  efficiency  and  the  forward  distance 
traversed  in  a  single  turn  of  a  screw  are  directly  as  the  pitch  of 


158  LECTURE  XIV. 

the  thread,  but  the  strength  is  proportional  to  the  area  of  its  cross 
section.  Now,  if  for  any  purpose  requiring  a  rapid  movement  of 
the  nut  or  of  a  screw,  the  pitch  must  be  increased ;  and  if  the 
screw  consisted  of  a  single- threaded  square  one,  where  the  depth, 
thickness  of  the  thread,  and  the  width  of  the  groove  are  each 
equal  to  half  the  pitch,  the  strength  of  the  shaft  upon  which  the 
screw  is  cut  would  be  unnecessarily  reduced.  If  the  groove  be 
made  shallower  and  narrower,  then  two  threads  with  two  spaces 
having  the  same  pitch  as  the  single  one,  can  be  cut  upon  it  so  as 
to  present  about  the  same  area  of  bearing  surface  to  the  pressure 
and  at  the  same  time  afford  quite  as  great  a  shearing  thickness 
without  interfering  with  the  velocity  ratio.*  If  a  very  great 
velocity  ratio  should  be  required,  then  three  or  more  threads  with 
corresponding  grooves  may  be  cut  in  the  shaft  and  nut. 

Backlash  in  Wheel  and  Screw-Gearings. — Backlash  is  the 
slackness  between  the  teeth  of  wheels  in  gear  or  between  a  screw 
and  its  nut.  Suppose  that  two  wheels  are  in  gear,  and  that  you 
move  one  of  them  in  a  certain  direction  until  it  turns  the  other, 
and  then  reverse  the  motion  ;  if  you  can  now  move  the  pitch  circle 
through,  say,  |  inch,  before  the  second  wheel  responds,  this  distance 
is  the  amount  of  backlash.  In  the  same  way,  suppose  you  turn  a 
screw  in  one  direction  until  its  nut  moves,  and  then  reverse  the 
motion,  the  angle  or  proportion  of  a  turn  which  you  can  now 
make  before  the  nut  responds,  is  the  backlash  of  the  screw  and  its 
nut.  If  a  great  amount  of  backlash  be  present  in  wheel-gearing, 
it  causes  vibration  and  a  disagreeable  rattling  noise ;  and  where 
severe  stresses  and  sudden  stoppages  are  common,  the  teeth  are 
liable  to  be  stripped.  It  can  only  be  thoroughly  prevented  by 
cutting  the  teeth  most  accurately  of  the  best  rolling  contact  form 
by  a  tooth-cutting  machine.  All  screws  and  nuts  that  are  much 
worked  are  liable  to  backlash  as  they  become  worn,  although  when 
new  they  may  have  been  very  free  from  it,  so  that  the  best  way 
of  taking  up  the  slack  is  to  form  the  nut  in  two  parts  with  flanges 
connected  by  screw-bolts,  which  may  be  tightened  from  time  to 
time  so  as  to  take  up  the  wear,  and  thus  keep  one  side  of  the 
threads  in  one  half  of  the  nut,  bearing  hard  against  one  side  of  the 
threads  of  the  screw,  and  those  m  the  other  half  against  the  other 
Bide. 

*  The  screw  of  the  fly-press,  figured  on  p.  249,  is  a  double-threaded  one. 


LECTUBE  XIV. — QUESTION 


LECTUEB  XIV.— QUESTIONS. 

1.  Explain  how  a  screw  is  a  combination  of  the  lever  and  inclined 
plane,  and  illustrate  your  remarks.    Find  the  theoretical  advantage  or 
ratio  of  W  to  P  in  the  case  of  a  screw  of  I  inch  pitch  and  3-2  inches 
diameter  ;  if  the  lever  or  spanner  key  be  7  feet  long.    Ans.  528  : 1. 

2.  Given  a  cylinder  and  a  sheet  of  paper  of  sufficient  size  to  cover  the 
cylindrical  surface,  show  how  you  would  trace  an  evenly  pitched  spiral  or 
screw  line  on  the  cylinder.     Mark  on  your  sketch  the  pitch,  circumference, 
•\nd  angle  of  the  screw-thread. 

3.  Trace  a  screw-thread  line  on  a  cylinder.    Draw  a  triangle  to  repre- 
sent  the  pitch,  circumference  and  angle  of  the  thread,  and  show  the 
direction  of  all  the  forces  on  the  supposition  that  there  is  a  total  pressure, 
Wlbs.,  on  the  end  of  the  cylinder  acting  parallel  to  its  axis  and  balanced  by 
a  force,  P  Ibs.,  acting  at  its  circumference  in  a  plane  at  right  angles  to  the 
axis,  with  a  total  friction  of  F  Ibs.  on  the  screw-thread. 

4.  What  are  the  essential  characteristics  of  a  screw-thread  ?     Upon 
which  of  these  do  (i)  the  efficiency,  (2)  the  strength,  (3)  the  durability  of  a 
screw  depend? 

5.  Sketch  and  describe  all  the  forms  of  screw-threads  which  you  have 
seen  in  practice.     State  their  representative  advantages  and  disadvantages, 
and  for  which  kind  of  work  each  kind  is  most  suitable. 

6.  Define  the  pitch  of  a  screw.    In  the  Whitworth  angular  screw-thread, 
what  is  the  angle  made  by  opposite  sides  of  the  thread  ?    To  what  extent 
is  the  thread  rounded  off  at  the  top  and  bottom  ?    Distinguish  between  a 
tingle  and  a  double-threaded   screw ;  in  what  cases  should  the  latter  be 
used  ?    Why  are  holding  down  bolts  made  with  angular  threads  ? 

7.  Distinguish  between  a  right-handed  and  a  left-handed  screw.    Sketch 
the    screw-coupling  which   is   commonly  used    to  connect    two  railway 
carriages,  and  explain  the  action  of  the  combined  screws.     If  the  pitch  of 
each  acrew  is  f  inch  and  the  lever-arm  from  the  axis  of  the  screw  to  the 
centre  of  the  ball  is  12  inches,  with  what  force  will  the  carriages  be  pulled 
together  by  a  force  of  50  Ibs.  applied  to  the  ball  on  the  end  of  the  arm  ? 
Ans.  5028  Ibs. 

8.  Draw  a  single,  double,  and  treble  square-threaded  screw  to  a  iVth 
scale,  where  the  outside  diameter  of  the  screw-thread  is  10  inches  and  the 
pitch  6  inches.     Explain  the  advantages  of  using  a  double  or  treble  thread 
instead  of  a  single  one  for  transmitting  rapid  motion  against  a  considerable 
resistance. 

9.  Why  is  the  angular-threaded  Whitworth  or  Seller's  screw  better 
adapted  than  the  square,  rounded,  or  buttress  thread  for  the  bolts  which 
are  used  to  bind  pieces  of  machines,  &c.,  together  ? 

10.  What  is  meant  by  backlash?    How  may  backlash  be  prevented  in  a 
crew,  and  in  wheel  gearing  2 


LECTUEE  XV. 

CONTENTS. — Efficiency,  &c.,  of  a  Combined  Lever,  Screw,  andPulleyGear — 
Example  I. — Bottle  Screw- Jack  —  Example  II.  —  Traversing  Screw- 
Jack — Screw  Press  for  Bales — Screw  Bench  Vice — Example  III. — 
Endless  Screw  and  Worm- Wheel — Combined  Pulley,  Worm,  Worm- 
Wheel  and  Winch  Drum— Worm- Wheel  Lifting  Gear— Example  IV. — 
Questions. 

Efficiency,  &c.,  of  a  Combined  Lever,  Screw,  and  Pulley 
Gear. — Construct  an  apparatus  of  the  following  description, 
having  a  horizontal  Whitworth  V-screw  of,  say,  p"  pitch,  with 
cylindrical  ends  and  flanges  supported  by  bearings,  so  that  the 
screw  cannot  move  longitudinally,  but  with  a  nut  free  to  travel 
from  one  end  of  the  screw  to  the  other,  along  a  slide  or  guide 


APPARATUS  FOR  DEMONSTRATING  THE  ACTION  AND 
EFFICIENCY  OF  SCREW  GEAR, 

INDEX  TO  PARTS. 

P  represents  Pull  on  pulley  rope 


W  represents  Weight  to  be  lifted. 
GP         „          Guide  Pulley. 
N  Nut. 


fci 


Screw. 


Radius  of  pulley. 
Base  or  support 


SCBZT,  LEV23,    ASL  PULLEY  GEAR.  l6l 

which  prevents  it  from  turning  round.  Apply  a  force,  P,  to  a 
rope  passed  over  the  V-grooved  pulley  of  radius,  R,  keyed  to  the 
end  of  the  screw  shaft,  until  it  moves  the  nut  with  the  hook, 
rope,  and  weight,  W,  attached  thereto,  ao  shown  by  the  accom- 
panying side  elevation,  plan  and  end  view  of  the  apparatus.* 

EXAMPLE  I. — If  the  radius,  R,  of  tho  turning-pulley  be  12",  the 
pitch,  p,  of  the  screw  i",  and  the  gross  pull,  P,  required  to  lift  a 
weight  of  TOO  Ibs.  be  4  Ibs. :  find  (i)  the  velocity  ratio;  (2)  the 
theoretical  advantage ;  (3)  the  working  advantage ;  (4)  the  work 
put  in  to  lift  W  i  foot ;  (5)  the  work  got  out ;  (6)  the  percentage 
efficiency. 

ANSWER. — We  have  got  in  this  question  all  the  necessary  data  required 
to  find  the  various  answers  except  n,  the  number  of  turns  which  the  screw 
will  have  to  make  in  order  to  lift  W  i  foot.  Since  the  pitch  of  the  screw 
is  i",  each  turn  thereof  will  elevate  or  lower  the  weight  i",  according  as  it 
is  turned  the  one  way  or  the  other  ;  consequently,  if  the  screw  makes 
12  turns,  the  nut  and  the  weight  will  move  through  12",  therefore  n=i2 
turns. 


*  It  is  evident  that,  in  addition  to  the  friction  between  the  screw  and 
the  nut,  there  is  friction  at  the  several  bearings,  at  the  nut  slide,  and  in 
the  bending  of  the  ropes.  Consequently,  if  the  student  were  to  place  in 
fucceftsion  weights  at  W  of,  say,  10,  20,  30,  40  Ibs.,  &c.,  and  ascertain  by  aid  of 
a  Sailer's  spring  balance  (hooked  into  the  rope  which  passes  round  the  turn- 
ing-pulley), the  corresponding  pulls  required  to  lift  these  several  weights, 
and  to  plot  down  the  results  on  squared  paper  with  the  weights  as  abscissa 
and  the  pulls  as  ordinates,  and  then  to  draw  a  line  through  the  inter- 
sections of  the  vertical  and  horizontal  lines  drawn  through  the  correspond- 
ing values,  he  would  obtain  a  characteristic  curve  for  the  friction  of  the 
machine  as  a  whole.  If  he  took  the  precaution  to  balance  the  initial 
friction  of  the  machine  (when  there  was  no  weight  attached  at  W)  by 
hanging  such  a  small  weight  at  P  as  would  just  move  the  nut  towards 
the  turning-pulley,  he  would  find  upon  repeating  the  above  experiments 
(keeping  the  small  additional  weight  on  all  the  time)  and  replotting  the 
results  as  now  recorded  by  the  spring  balance,  that  the  second  frictional 
curve  would  approach  much  nearer  to  a  straight  line  than  the  former  one. 
In  fact,  its  deviation  therefrom  would  simply  prove  that  the  friction  of  the 
movable  bearing  surfaces  was  not  directly  proportional  to  the  load.  To 
arrive  at  the  characteristic  friction  curve  for  the  screw  alone,  he  would 
have  to  find  out  by  trial  the  proportion  of  the  several  pulls  applied,  which 
were  spent  in  overcoming  friction  at  all  other  points  except  between  the 
screw  and  the  nut.  To  those  students  who  have  the  time  and  opportunity 
for  carrying  out  experiments  in  applied  mechanics,  the  apparatus  illustrated 
above  will  prove  interesting  and  instructive.  The  figures  are  drawn  from 
the  machine  constructed  in  the  author's  engineering  workshop  for  the 
purpose  of  enabling  his  students  to  make  similar  tests  to  those  suggested 
above.  A  square,  or  a  rounded,  or  a  buttress-thread  may  be  substituted 
for  the  V-  Whit  worth  one,  and  sound  information  may  thus  be  obtained 
about  different  forms  of  screws,  which  will  make  a  stronger  and  more 
lasting  impression  on  some  students  than  merely  studying  books. 


1 62  LECTURE  XY. 

By  the  principle  of  work  : — 

/  \  m-i    rr  j    •*     T>  *  -  P  s  distance  in  i  turn  of  driving  pulley 

(1)  The  Velocity  Ratio _,    ,.  . .  .  ... jfP^- * 

v  '  W  s  distance  in  the  same  tune 

~  _  0°*  of  pulley  _  2?rB.  _  75  '4 

pitch  of  screw       p          1 

(2)  The  Theoretical  Ad-\  _  Weight  lifted  if  there  were  no  friction 

vantage      .         .     I  Pull  applied 


^        p          1 

(3)  The  Working  Advan-  \  _  W  =  100  Ibs.  =  25 

tage  .  .     J       P        4  Ibs.         1 

(4)  The  Work  Put  in  to] 


y  X  I  2 

(5)  The  Wart  0*  va  «|  =  w       ,_         lbs        ,.„„  fl  . 

raising  W  i  /ooi       j 


(6)  31  -    _  Effid 

cieTicy  .     J  J 

Work  got  out 
w  "  Work  put  in  X 

IPO  ft.-lbs.  # 

301-56  ft,ibs:= 

Bottle  Screw-Jack.  —  The  importance  of  the  screw  as  a 
simple  machine  for  exerting  great  pressures,  is  very  well  ex 
emplified  by  the  screw-jack.  This  tool  is  used  for  replacing 
locomotives  and  railway  carriages  upon  their  rails,  for  elevating 
heavy  girders  into  position,  or  for  overcoming  any  great  resistance 
through  a  small  space  which  cannot  be  effected  by  a  labourer  and 
a  lever.  As  will  be  seen  from  the  accompanying  figure  it  consists 
of  a  strong  hollow  bottle-shaped  casting,  with  a  projecting  handle 
for  facilitating  the  carrying  of  the  tool  from  one  place  to  another. 
In  the  upper  end  of  the  casting  a  square-threaded  screw  is  cut 

*  It  is  evident  that  with  such  a  low  percentage  efficiency  the  weight 
when  hanging  from  the  rope  will  not  be  able  to  overhsrul  the  machine. 
The  student  can  calculate  what  pitch  of  screw  would  be  required  with  the 
same  co-efficient  of  friction  before  overhauling  could  take  place. 


BOTTLE    SCREW-JACK,  x  163 

parallel  with  the  axis,  and  into  this  nut  there  is  fitted  a  steel 

screw  terminating  in  a  spherical  head,  having  two  holes  bored 

through  it  at  right  angles  to  each  other.     Into 

one  or  other  of  these  holes  an  iron  lever  bar 

is  fixed,  so  that  by  pulling  or  pushing  on  the 

outer  end  of  the  bar  the  screw  is  turned,  and 

thus  the  head  is   gradually   raised  from  the 

base.     To  avoid  the  tearing,  grinding  action 

that   would  ensue  between  the  head  and  the 

object  acted  upon,  the  former  is  provided  with 

a  loose  crown  fitted  on  a  central  pin  projecting 

from  the  round  head. 

Let  L  =  Length  of  the  lever  arm  in  inches 
from  centre  of  jack  to  where  the 
force  is  applied. 

„      p  =  Pitch  of  screw  in  inches. 

„     P=^Pull  or  push  applied  at  radius  L. 

ft  W  =  Weight  lifted  or  resistance  over- 
come. 

Then,  by  the  Principle  of  Work,  and  neglect- 
ing friction,  we  have  in  one  turn  of  lever — 

P  x  its  distance  =  W  x  its  distance 
!*P~"Wxp 

•     •    -C    —  ' 


BOTTLE  SCKEW- 
JACK. 


EXAMPLE  II. — A  weight  of  10  tons  has  to  be  lifted  by  a  screw- 
jack,  in  which  the  pitch  of  the  screw  is  J".  What  length  of  lever 
will  be  required  if  a  force  of  70  Ibs.  be  applied  at  the  end  of  it  ? 
(i)  Neglecting  friction ;  (2)  if  the  modulus  or  efficiency  of  the 
tool  is  only  '4. 

ANSWER. — (i)  By  the  previous  formula  (neglecting  friction) 


1120 


L 


X27T 


/?0X#X22         22 


(2)  Taking  friction  into  account  we  see  from  the  question  that 
the  efficiency  is  =  -4,  therefore  the  percentage  efficiency  is  40,  or 
60  per  cent,  of  the  work  put  in  is  lost  work  required  to  overcome 
friction  between  the  screw  and  its  nut.  But  as  the  length  of  the 
lever  is  directly  proportional  to  the  work  put  in,  the  theoretical 
length  of  the  lever  found  above  is  only  40  per  cent,  of  the  actual 
or  working  length  required. 


164 


LECTURE  XV. 


.-.  40:  1001:25-45  :L3 
L      100x25-45  =  63.6,, 

40 

Traversing  Screw-Jack. — It  is  very  often  convenient,  when 
using  a  strong  heavy  screw-jack,  to  be  able  to  move  the  head  a 
short  distance  to  one  side  or  the  other,  when  near  the  object  to 
which  it  is  to  be  applied ;  or,  after  having  raised  a  load  with  one 
or  more  jacks,  to  be  able  to  traverse  the  jacks  forward  or  back- 
ward through  a  short  distance  until  the  load  is  brought  into 


TRAVERSING  SCREW-JACK  WITH  KATCHET-LEVERS. 
(By  P.  &  W.  MacLellan,  Glasgow. ) 

the  desired  position.  These  movements  may  be  effected  with  a 
jack  of  the  form  shown  by  the  accompanying  figure.  Further, 
this  jack  is  provided  with  a  side  foot-step  attached  to  and  pro- 
jecting from  the  lower  end  of  the  vertical  screw.  This  foot- step 
can  be  placed  under  the  flange  of  a  low  beam  or  rail,  where  it  would 
be  inconvenient  or  perhaps  impossible  to  get  the  top  head  under- 
neath the  same.  The  nut  of  the  horizontal  traversing  screw  is 


SCREW   PRESS   FOR   BALES.  165 

formed  in,  or  fitted  to  the  bottom  of  the  vertical  casting,  and  this 
screw  is  turned  by  a  ratchet-lever  which  may  be  slipped  on  to  one 
or  other  of  the  squared  ends  of  its  shaft.  The  upward  and  down- 
ward movement  of  the  vertical  screw  is  also  affected  by  a  ratchet- 
lever,  and  in  this  case  without  turning  the  screw,  for  the  ratchet- 
wheel  is  fixed  to  the  nut  of  its  screw.  The  pawl  of  the  ratchet 
may  be  locked  on  one  side  or  the  other,  so  as  to  enable  the  ratchet- 
wheel  and  the  vertical  screw-nut  to  be  turned  round  in  either 
direction  for  elevating  or  lowering  the  load. 

Screw  Press  for  Bales. — When  soft  goods  or  hay  have  to  be 
transported  they  may  be  squeezed  into  small  bulk  by  means  of  a 


SCREW  PEESS  FOB  BALES. 
(By  London  Bros.,  Glasgow.) 

screw  press,  and  bound  firmly  when  under  the  press,  by  strips  of 
hoop-iron  passed  round  them  and  then  riveted  before  the  pressure 
is  relieved.  The  bound  bundle  is  then  termed  a  bale.  The 
operation  will  be  understood  by  an  inspection  of  the  accompany- 
ing figure.  The  loose  material  is  placed  in  the  space  between  the 
rigid  base  and  the  movable  plate  of  the  press,  the  doors  are  closed 
and  locked,  the  pressman  applies  himself  to  the  end  of  the  lever 
with  a  force,  P,  thereby  turning  the  nut  of  the  screw  and  forcing 
the  movable  plate  downwards  with  a  pressure,  W,  until  the 


i66 


LECTURE  XV. 


desired  compression  of  the  goods  has  been  attained.  The  doors 
are  then  opened  and  the  strips  of  hoop  iron  (which  were  previously 
or  are  now  placed  in  the  grooves  of  the  base  and  upper  plate)  are 
brought  together  and  riveted.  The  lever  is  then  turned  in  the 
opposite  direction  to  relieve  the  pressure,  and  the  bale  is  removed 
to  the  store  or  ship  to  make  room  for  another  quantity  of  goods 
being  subjected  to  a  similar  action. 

The  same  formula  as  we  used  for  the  screw-jack  and  for 
Example  I.  in  connection  with  the  combined  lever,  screw  and 
pulley  gear,  naturally  applies  to  this  press,  and  to  any  similar 
appliance,  such  as  a  letter-copying  press.* 

Screw  Bench  Vice. — A  bench  vice  is  essentially  an  instru- 
ment for  seizing  and  holding  firmly  any  small  object  whilst  it  is 
being  acted  upon  by  a  chisel,  file,  drill,  saw,  or  emery  cloth,  &c. 
Looking  at  the  figure  which  illustrates  the  following  example,  it 
will  be  seen  that  the  vice  is  a  combination  of  two  levers,  a  square- 
threaded  screw,  and  a  nut.  The  object  0  to  be  gripped  is  placed 


END  VIEW.  SIDE  VIEW. 

SCEEW  BENCH  VICE. 

between  the  serrated  jaws  JJ.  The  lever  handle  H,  on  being 
turned,  forces  the  screw  S  into  its  long  nut,  and  thereby  presses 
forward  the  outer  jaw  upon  the  object,  by  aid  of  the  flange  on  the 
screw-head.  This  jaw  is  a  lever,  having  a  fulcrum  at  F,  and  there- 
fore the  pressure  on  the  object  is  less  than  that  on  the  screw- 
collar  in  the  proportion  of  SF  to  OF.  The  bent  flat  spring 
between  the  limbs  of  the  fixed  and  movable  jaws  serves  to  force 
the  movable  jaw  away  from  the  fixed  one  when  the  screw  is  turned 
backwards,  and  thus  relieves  the  object  without  having  to  pull 

*  Kefer  to  index  for  page  where  the  illastration  of  the  Fly-press  occurs.. 
The  statical  pressures  produced  by  this  machine  when  used  for  punching 
holes,  &c.,  may  be  treated  in  the  same  way. 


BENCH  YICE.  *  167 

this  jaw  back  by  the  hand.  It  will  be  observed  that  the  fixed 
jaw  should  have  been  continued  to  the  floor  level  by  a  vertical 
supporting  leg,  in  the  case  of  such  a  big  vice  intended  for  rough 
heavy  engineering  work. 

EXAMPLE  III. — Sketch   an  ordinary  bench  vice.     Apply  the 
principle  of  work  to  find  the  gripping  force  obtained  when  a  man 
exerts  a  pressure  of  20  Ibs.  at  the  end  of  a  lever  18  inches  long, 
the  screw  having  four  threads  per  inch,  the  length   from  the 
hinge  to  the  screw  being   18  inches,  and  the  length  from  the 
hinge  to  the  jaws  being  24  inches.     (S.  <fe  A.  Exam.  1892.) 
ANSWER. — Let  P  represent  Pull  on  end  of  handle  H  =  20  Ibs. 
„    Q         ,,         Resistance  offered  by  screw  at  S. 
„    B         „         Reaction,  or  gripping  force,  exerted 

on  object  at  O. 

„    L         „         Length  of  handle  H=  18  inches. 
„    p          „         Pitch  of  screw  S  =  J  inch. 

Suppose  the  handle,  H,  to  make  one  complete  turn  under  the 
action  of  a  constant  force,  P,  at  the  extremity  thereof,  against  a 
constant  resistance,  Q,  acting  along  the  axis  of  the  screw. 

[The  student  will  observe  that  we  suppose  the  forces  P  and  Q  to  be  con- 
stant, which  is  not  correct  for  such  a  large  movement  as  a  complete  turn 
of  the  handle,  but  which  may  be  assumed  here  for  the  sake  of  simplicity. 
The  reason  for  this  is,  that  the  resistance,  R,  will  vary  with  the  com- 
pression produced  on  the  object  at  O.  However,  the  ratio  between  P 
and  R  will  remain  a  constant  quantity.] 

The  work  done  by  P  during  one  turn  of  handle  =  P  x  2irL. 
And        „  on  Q,  during  the  same  time         =Q  xp. 

But,  by  the  Principle  of  Work — 

Work  done  by  P  =  Work  done  on  Q 

.*.   Px  2?rL  =  Q  Xp 

Substituting  the 
numerical  values — 
22 

20X2X XlS"  =  QxJ'  K* 

.-.  Q=20X2X272Xl8x4~9°S'-43  "». 
But  by  the  Principle  of  Moments — 


24  4 

t.«.,     B  =     x  9051-43  =  6788-67  Ibs. 


1 68  LECTURE   XV. 

Endless  Screw  and  Worm-Wheel.*  —  When  a  screw  is 
rotated  between  fixed  bearings  so  that  it  cannot  move  longi- 
tudinally, it  is  called  an  endless  screw,  because  the  threads  of  the 
screw  seem  to  travel  onwards  without  ending.!  When  such  a 
screw  gears  with  a  toothed  wheel,  having  its  teeth  set  obliquely 
at  the  same  angle  as  the  threads  of  the  screw  so  as  to  bear  evenly 
thereon,  the  wheel  is  termed  a  worm-wheel.  The  endless  screw  is 
sometimes  called  the  worm,  no  doubt  from  its  resemblance  to  that 
well-known  humble  animal  which,  when  coiled  up  for  rest,  would 
not  turn  upon  any  one  unless  trod  upon. 

By  this  arrangement,  motion  may  be  transmitted  from  one  shaft 
to  another  at  right  angles  to  each  other,  without  any  possibility 
of  the  machine  overhauling  ;  for  although  the  velocity  ratio  is  very 
great,  the  efficiency  is  comparatively  small — considerably  under 
50  per  cent,  with  single-threaded  screws — owing  to  the  friction 
between  the  worm  and  the  wheel.  J 

It  is  most  important  for  the  student  to  comprehend  that  if  the 
screw  be  a  single-threaded  one,  it  must  make  as  many  turns  as  there 
are  teeth  on  the  wheel,  for  every  revolution  of  the  latter.  If  the 
screw  is  a  double-threaded  one,  then  for  each  revolution  thereof  it 
drives  the  wheel  through  a  distance  equal  to  the  distance  between 
tu-o  teeth  on  the  pitch  circle,  and  if  treble-threaded  through  the 
pitches  of  three  teeth.  Thus,  if  N  equal  the  number  of  teeth  in 
the  worm-wheel,  then,  with  a  single-threaded  screw,  for  every 

turn  of  the  same,  the  wheel  will  move  a  distance  of  ^ ;  with  a 

double-threaded  worm  ^,   and  with    a    treble-threaded   one  j* 

and  so  on. 

The  endless  screw  and  worm-wheel  is  used  in  a  very  great 
variety  of  circumstances,  from  the  turning  of  a  big  marine  engine 
when  in  port,  to  the  delicate  movements  in  a  telescope  or  a  micro- 
scope. 

Combined  Pulley,  Worm,  Worm-wheel  and  Winch 
Drum. — This  combination  is  shown  by  the  accompanying  end 
and  side  views  drawn  from  an  experimental  piece  of  apparatus  it 

*  Refer  to  the  next  figure. 

t  The  term  perpetual  screw  would  express  more  exactly  its  action,  foj 
when  in  motion,  it  continually  screws  the  worm-wheel  round. 

+  The  greater  the  diameter  of  the  screw  and  the  smaller  its  pitch  is,  the 
better  will  be  its  bearing  on  the  teeth  of  the  wheel,  but  then  the  efficiency 
will  be  so  small  that  there  will  be  no  chance  of  overhauling.  This  is  the 
condition  to  be  observed  when  the  screw  is  intended  to  drive  the  wheel. 
If,  however,  it  should  be  required  to  drive  the  screw  by  the  wheel,  or 
necessary  that  overhauling  should  take  place,  then  the  screw  must  be  small 
In  diameter,  its  pitch  very  great,  and  either  double  or  treble  threaded. 


COMBINED   PULLEY,    WOKM,   WORM-WHEEL,   ETC. 


169 


the  Author's  Laboratory,  which  is  used  by  the  students  for  ascer- 
taining the  efficiency  of  the  machine,  and  for  finding  the  co-efficient 
of  friction  between  the  endless  screw  and  worm-wheel. 


END  VIEW.  SIDE  VIEW. 

PULLET,  WORM,  WORM-WHEEL  AND  WINCH  DRUM.  * 
INDEX  TO  PARTS. 


P  represents  Pull    applied    to 


R 

Wm 

WW 


Radius  of  pulley. 
Worm  or  endless 

screw. 
Worm  wheel. 


N  repre?ents  Number  of  teeth  in 

WW. 
D          „          Drum,  or  diameter  of 

winch  barrel. 

r          „  Radius  of  drum,  D. 

W  Weight  to  be  lifted. 


By  the  Principle  oj   Work  (neglecting  friction),  if  the  drum, 
makes  one  turn,  and  if  the  worm  be  a  single-threaded  sciew, 

P  x  its  distance  =  "W  x  its  distance 
Or,  P  x  2?rRN  =  W  x  2?rr 

(divide  both  sides  by  zv) 

P  x  RN  =  W  x  r 


_ 
*  W~~RN 

*  Find  out  what  is  wrong  with  the  above  drawing. 

t  It  will  be  evident  to  the  student  that,  given  any  four  of  these  five 
values,  he  can  change  this  formula  so  as  to  find  the  fifth  one ;  and,  that 
he  can  experiment  with  this  machine  in  precisely  the  same  way  as  has 
been  already  explained  in  the  case  of  the  wheel  and  axle,  block  and  tackle, 
Weston's  pulley  block  and  screw,  &c.,  to  ascertain  its  working  advantage, 
co-efficient  of  friction  and  efficiency. 


LECTURE  XV. 

Worm-wheel  Lifting  Gear.  —  The  accompanying  figure 
shows  a  practical  application  of  the  endless  screw  and  worm- 
wheel  for  the  same  purpose  as  the  Weston's  differential  block  is 
used — viz.,  the  lifting  of  weights  without  fear  of  the  tackle  over- 
hauling. A  light-driving  endless  chain  passes 
over  a  V-grooved  pulley  having  ridges  or  teeth 
on  the  inner  sides  of  the  grooves,  so  as  to  fit 
the  pitch  of  the  links  of  the  chain.  This 
pulley  is  keyed  to  the  outer  end  of  a  worm 
spindle,  whose  screw  gears  with  a  worm- 
wheel  fixed  to  or  cast  along  with  a  second 
V-grooved  ridged  pulley  or  drum,  over  which 
is  passed  the  movable  end  of  a  heavier  lifting 
chain  after  it  has  been  reeved  under  a  snatch- 
block  pulley.  In  fact,  it  is  simply  the  previous 
experimental  apparatus  in  a  handy  and  com- 
pact form. 

EXAMPLE  IY. — If  in  lifting  tackle  of  the 
above  description  the  driving  pulley  has  a 
radius  B  =  5",  the  number  of  teeth  in  the 
worm-wheel  N  =  20,  and  the  driven  pulley  a 
radius  r  =  5"  ;  what  weight  suspended  from 
the  snatch-block  hook  could  be  lifted  by  a 
force  of  10  Ibs.  applied  to  the  forward  side  of 
the  light  chain — (i)  Neglecting  friction,  (2)  if 
the  modulus  or  efficiency  of  the  whole  appa- 
ratus were  only  "25  ? 

ANSWER. — (i)  Applying  the  previous  formula,  and  taking 
account  of  the  fact  that  the  lifting  chain  is  combined  with  a 
snatch-block,  we  have — 

PxRxN       2X10X0X20 


(2)  Owing  to  friction,  weight  of  chain  and  snatch-block,  the 
actual  result  obtainable  is  only  '25,  or  25  per  cent,  of  this  theo- 
retical value ;  consequently 

100  :  25  :  400  :  x 
x= £5x400  =  1Q01bs 

IOO 


LECTURE  XV. — QTTESTIONS.  I? 1 


LECTURE  XV.— QUESTIOITS. 

1.  A  horizontal  screw,  of  I  inch  pitch,  is  fitted  to  a  sliding  nnt  which  is 
pulled  horizontally  by  a  cord  passing  over  a  fixed  pulley,  and  having  a 
weight,  W,  attached  to  it.     To  the  free  end  of  the  screw  there  is  fixed  a 
pulley  of  20  inches  diameter,  from  the  circumference  of  which  a  weight, 
"°,  hangs  by  a  cord.     Find  the  ratio  of  P  to  W.     Ans.  i  :  62-8. 

2.  In  a  set  of  combined  lever,  screw,  and  pulley  gear,  like  that  illustrated 
before  Example  I.  in  this  Lecture,  R  =  6",  P  =  2  Ibs.,  W  =  50  Ibs.,  and 
the  pitch  of  the  screw  is  such  that  there  are  2  threads  to  the  inch  ;  find  (i) 
velocity  ratio,  (2)  theoretical  advantage,  (3)  working  advantage,  (4)  work  put 
in  to  lift  W  i  ft.,  (5)  work  got  out,  (6)  percentage  efficiency.  Ans.  (i)  75-4  :  i ; 
(2)  75-4  :  I  ;  (3)  25  :  I ;  (4)  150-8  ft. -Ibs. ;  (5)  50  ft.-lbs.  ;  (6)  33-1  per  cent. 

3.  Describe,  with  sketches,  the  construction  of  an  ordinary  lifting  jack 
in  which  the  weight  is  lifted  by  means  of  a  screw  and  nut.     If  the  screw 
be  i  inch  pitch,  the  lever  20  inches  long,  and  the  pressure  applied  at  the 
end  of  the  lever  be  30  Ibs.  ;  what  weight  can  be  lifted  (neglecting  friction)  t 
(Take  IT  —  3'i4i6.)    Ans.  3770  Ibs. 

4.  In  a  screw-jack,  where  a  worm-wneel  is  used,  the  pitch  of  the  screw  is 
|  inch,  the  number  of  teeth  on  the  worm-wheel  is  16,  and  the  length  of 
the  lever  is  10  inches;  find  the  gain  in  pressure.     Ans.  P  :  W  :  :  i :  1609. 

5.  What  practical  objection  is  .there  to  the  use  of  screw  gear  of  any 
description  for  obtaining  great  pressure  ?    Take  for  example  the  case  of 
the  screw-lif  ting  jack.     Sketch  in  vertical  section  and  plan,  and  describe,  a 
traversing  one  to  lift  say  20  tons.     Explain  how  the  screw  of  the  jack  is 
raised  and  lowered  without  being  turned  round. 

6.  Sketch  and  describe  the  construction  and  action  of  a  screw  press  for 
pressing  goods  so  as  to  make  them  into  bales  for  transport.     What  force 
must  be  applied  at  the  end  of  a  screw  press  lever  8'  4"  in  length,  in  order 
to  exert  on  the  goods  a  total  pressure  of  22,000  Ibs.  when  the  pitch  of  the 
screw  is  i" ?    If  60  per  cent,  be  lost  in  friction,  what  pressure  would  result 
from  the  application  of  this  force  on  the  lever?    Ans.  35  Ibs. ;  8800  Ibs. 

7.  Sketch  an  ordinary  bench  vice.     Apply  the  principle  of  work  to  find 
the  gripping  force  obtained  when  a  man  exerts  a  pressure  of  15  Ibs.  at  the 
end  of  a  lever  15  inches  long,  the  screw  having  5  threads  per  inch,  the 
length  from  the  hinge  to  the  screw  being  12  inches,  and  the  length  from 
the  hinge  to  the  jaws  being  16 inches.     Ans.  5303-6  Ibs. 

8.  Explain,  with  a  sketch,  the  manner  in  which  the  principle  of  work  is 
applied  in  determining  the  relation  of  P  to  W  in  the  case  of  the  endless 
screw  and  worm-wheeL     The  lever  handle  which  works  the  screw  being 
14"  long,  the  number  of  teeth  in  the  worm-wheel  20,  and  the  load  being  a 
weight  of  icoo  Ibs.  hanging  upon  a  drum  12"  diameter  on  the  worm-wheel 
shaft,  find  the  force  to  be  applied  at  the  end  of  the  lever  handle  in  order 
to  support  the  weight.     Ans.  21-43  ^DS- 

9.  Explain  the  mechanical  advantage  resulting  from  the  employment 
of  an  endless  screw  and  worm-wheel.    The  lever  handle  which  turns  an 
endless   screw  is   14"  long,  the  worm  has  32  teeth,   and  a  weight  W, 
hangs  by  a  rope  from  a  drum  6"  diameter,  whose  axis  coincides  with  that 
of  the  worm-wheel.     If  a  pressure  P  be  applied  to  the  lever  handle,  find 
the  ratio  of  P  to  W.     Ans.   P  :  W  :  :  3  :  448.     If  in  this  question  the 
worm  oe  changed  to  (i)  a  double,  and  (2)  a  treble-threaded  screw,  what 
Trill  be  the  respective  ratios  of  P  to  W?    Ans.  (i)  i  :  747  ;  (2)  i  :  49-8. 


172  LECTURE    XV. — QUESTIONS. 

10.  Describe,  with  the  aid  of  a  sketch,  how  the  pressure  upon  the  book 
is  obtained  in  an  ordinary  copying-press.     What  should  be  the  length  of 
the  double-ended  lever,  supposing  that  the  force  be  always  applied  simul- 
taneously to  both  ends  of  the  lever,  in  order  that  with  a  screw  having 
6  threads  to  the  inch,  the  combination  may  have  a  mechanical  advantage 
of  216?     Ans.  Length  of  lever  6  inches. 

11.  Sketch  in  vertical  section  the  common  screw  or  bottle  lifting  jack. 
The  lever  in  such  a  jack  is  single  ended,  and  measures  24  inches  in  length, 
the  pitch  of  the  screw  is  f  inch.     What  force  applied  at  the  end  of  the 
lever  would  be  required  to  raise  a  load  of  22  cwt.,  the  effect  of  friction 
being  neglected  1     Ans.  6  '12  Ibs. 

12.  Describe,  with  a  sketch,  the  construction  of  an  ordinary  screw-jack 
with  a  lever  handle  and  screw.     If  the  pitch  of  the  screw  be  f  inch,  the 
length  of  the  lever  handle  29  inches  ;  what  load  could  be  lifted,  neglecting 
friction,  by  a  force  of  19  Ibs.  applied  to  the  end  of   the  lever  handle  ? 
Ans.  2  tons. 

13.  Describe  either  a  screw-jack  (pitch  of  screw  ^",  handle  19"  long)  or  a 
simple  winch  for  lifting  weights  up  to  I  ton  by  one  man.     What  is  the 
mechanical  advantage  neglecting  friction?     Describe  what  sort  of  trial 
you  would  make  to  find  its  real  mechanical  advantage  under  various  loads, 
and  what  sort  of  result  would  you  expect  to  find  ? 

14.  The  diameter  of  the  safety  valve  of  a  steam  boiler  is  3  inches.     The 
weight  on  the  end  of  the  lever  is  55  Ibs.,  and  the  distance  from  the  centre 
of  the  valve  to  the  fulcrum  is  4-5  inches.     What  must  be  the  length  of  the 
lever  from  the  centre  of  the  valve  to  the  point  of  suspension  of  the  weight, 
in  order  that  the  valve  will  just  lift  when  the  pressure  of  steam  in  the 
boiler  is  80  Ibs.  per  square  inch  ?     Neglect  the  weight  of  lever  and  valve. 
Ans.  41 '3  inches. 

15.  Make  a  correct  draughtsmanlike  drawing  of  the  next  to  the  last 
figure  in  this  lecture.     Point  out  distinctly  what  is  wrong  with  the  end 
view  if  the  side  view  be  taken  as  correct. 

16.  Describe  how  you  would  proceed  to  determine  experimentally  for  a 
screw-jack  (i)  the  velocity  ratio,  (2)  the  actual  effort  required  to  lift  a 
given  load.  43.  of  E.,  1905.) 


HOTES  AND   QUESTIONS. 


(      '74    ) 


LECTURE  XYI. 

CONTENTS. — General  Idea  of  the  Mechanism  in  a  Screw-cutting  Lathe — 
Motions  of  the  Saddle  and  Slide  Rest — Velocity  Ratio  of  the  Change 
Wheels— Rules  for  Calculating  the  Required  Number  of  Teeth  in 
Change  Wheels— Examples  I.  II.— Movable  Headstock  for  a  Common 
Lathe— Descriptions  of  a  Screw-cutting  Lathe  and  of  an  Electrically 
Driven  Hexagon  Turret  Lathe,  with  Frontis-Plates  and  complete  sets 
of  Detail  Drawings — Questions. 

General  Idea  of  the  Mechanism  in  a  Screw-cutting 
Lathe. — We  will  devote  this  Lecture  to  giving  a  general  idea  of 
the  mechanism  by  which  screws  are  cut  in  lathes,  and  the  velocity 
ratio  of  the  screw  to  be  cut  to  the  leading  screw,  together  with  a 
description  of  a  complete  set  of  illustrations  prepared  from  work- 
ing drawings  of  a  new  self-acting  screw-cutting  lathe. 

Referring  to  the  following  figure,  and  to  the  general  view  of  the 
6-inch  centre  screw-cutting  lathe  (further  on),  it  will  be  seen  that 
the  round  metal  bar  on  which  the  screw  is  to  be  cut  is  placed 
between  the  steel  centres  of  the  fixed  and  movable  headstocks  of 
the  lathe.  This  bar  has  an  eye-catch  on  its  end  next  to  the  fixed 
headstock,  which  engages  with  a  driving-stud  connected  to  the 
face-plate.  In  order  to  obtain  the  necessary  force  to  cut  the 
screw,  and  to  reduce  the  speed  of  the  workshop  motion  shafts  (in 
the  case  of  a  power  lathe,  or  of  the  treadle  shaft  in  the  case  of  a 
foot  lathe)  to  the  required  velocity,  the  fixed  headstock  is  sup- 
plied with  back  motion  gearing.  The  back  wheels  may  be  put 
into  or  out  of  gear  with  the  lathe  spindle  wheels  at  pleasure,  by  a 
simple  eccentric  motion  (in  the  case  of  the  lathes  herein  illus- 
trated), or,  as  is  sometimes  effected,  by  sliding  the  back  shaft 
forward,  so  that  its  wheels  clear  those  on  the  lathe  spindle,  and 
then  fixing  it  there,  by  a  tapered  pin  fitting  through  a  hole  in  the 
framing  and  a  groove  cut  in  the  shaft.  But,  when  the  back 
motion  is  required  for  the  purpose  of  making  a  slow  heavy  cut, 
the  follower  F,  is  thrown  out  of  gear  with  the  stepped  cone 
pulley,  so  that  the  driver  DL  (which  is  keyed  to  the  cone)  may 
turn  the  follower  Fj ;  and  the  driver  Ds  (which  is  keyed  to  the 
same  spindle  as  Ft)  rotate  the  follower  F,  (which  is  keyed  to  the 
lathe  spindle),  and  hence  revolve  the  face-plate  and  the  bar,  out  of 
which  the  screw  is  to  be  formed. 


MECHANISM   IN   A   SCREW-CUTTINa  LA.THK. 


175 


On  the  back  extension  of  the  lathe  spindle  there  is  fixed  a 
change  wheel  or  small  driver,  d,  which  gears  with  a  follower,  f 
(keyed  to  the  left-hand  end  of  the  leading  or  parent  screw),  either 
direct  in  the  case  of  the  cutting  of  a  very  finely  pitched  left- 
handed  screw;  or,  through  the  intervention  of  a  transmitting,  or 


Isc 

JjQS 

CHILD 

\\\\\\\\ 

i\\U\U\\\\\\\v\\ 

13 

PLAN  SHOWING  CUTTING  A  SCREW. 


END  VIEWS  SHOWING  CHANGE  WHEELS. 
FOE  RIGHT-HANDED  SCREWS.  FOE  LEFT-HANDED  SCREWS. 

GENERAL  IDEA  OF  MECHANISM  IN  A  SCREW-CUTTING  LATHE. 


SC  represents  Screw  fro  be  cut 
or  child. 


LS 
SR 


Leading    screw, 

or  parent. 
Slide  rest. 
Drivers  of  fixed 

headstock. 


INDEX  TO  PARTS. 

F,f  F2  represents  Followers  of  fixed 

headstock. 

d,  f  „  Driver  and  fol- 
lower of  change 
wheels. 

IP          „          Idle      pinion      of 
change  wheels. 


what  is  technically  termed  an  idle,  pinion,  IP,  in  the  case  of  a 
medium-pitched  right-hand  screw.  (See  also  the  end  views  of  the 
change  wheels  above.) 

It  will  therefore  be  seen  that  there  are  two  independent  motions 
to  be  considered — (i)  the  reducing  gear  from  the  speed  of  the 


176  LECTURE  XVL 

driving  cone  to  that  of  the  lathe-spindle  or  bar  to  be  operated 
upon ;  and  (2)  the  multiplying  or  reducing  gear  between  the 
lathe-spindle  and  the  leading  screw.  The  former  of  these  will  be 
at  once  understood  from  the  figures,  and  from  what  was  said  in 
regard  to  wheel-gearing  in  Lecture  XII. 

We  shall  now  consider  the  second  motion.  Remembering  that 
the  pitch  of  the  parent  or  leading  screw  is  fixed  and  unalterable, 
and  that  on  its  truth  depends  to  a  large  extent  the  accuracy  with 
which  the  child,  or  screw  to  be  cut,  can  be  formed,  it  will  be  clear 
that  we  have  only  to  connect  these  two  parallel  shafts  with  suit- 
able gearing  in  order  to  transmit,  by  aid  of  the  "  copying  prin- 
ciple "  the  characteristics  of  the  parent  to  the  child.*  This  may 
be  done  in  an  equal  or  magnified  or  diminished  degree,  according 
as  the  pitch  of  the  screw  to  be  cut  is  equal  or  greater  or  less  than 
that  of  the  leading  screw. 

Motions  of  the  Saddle  and  Slide  Best.— The  base  of  the 
slide  rest,  or  the  saddle  as  it  is  technically  termed,  bears  upon  and 
is  guided  by  the  truly-planed  shears  (or  upper  framing  of  the  lathe) 
parallel  to  the  line  joining  the  centres  of  the  fixed  and  movable 
heads.  In  turning  a  right-handed  screw  the  saddle  is  moved  from 
the  movable  headstock  towards  the  fixed  one,  or  from  right  to  left,  by 
clasping  it  to  the  leading  or  guiding  screw  with  a  split  nut  attached 
to  the  under  side  of  the  saddle.  In  cutting  a  left-handed  screw 
the  saddle  is  moved  by  the  same  means,  but  in  the  opposite  direc- 
tion,— i.e.,  from  left  to  right.  In  other  words,  it  travels  in  the  direc- 
tion towards  which  the  threads  of  the  screw  to  be  cut  are  inclined 
forward. 

To  the  upper  side  of  the  saddle  is  bolted  the  slide-rest  sur- 
mounted by  the  tool-holder.  The  rest  is  provided  with  two  in- 
dependent sliding  motions,  each  actuated  by  a  hand-turned  screw, 
and  guided  by  a  true  plane  surface  with  dovetailed  sides.  These 
motions  (for  the  purposes  of  turning  parallel  work  and  screws) 
are  fixed  at  right  angles  to  each  other,  the  lower  one  being 
parallel  to  the  centre  line  of  the  lathe,  and  the  upper  one  at  right 
angles  thereto.  Both  motions  are  therefore  independent  of  each 
other  and  of  the  sliding  motion  of  the  saddle.  The  turner  is 
thereby  enabled  to  adjust  the  cutting  tool  with  great  delicacy  and 
accuracy  with  reference  to  the  job  to  be  operated  upon,  irrespective 
of  the  automatic  travel  of  the  supporting  saddle. 

Velocity  Ratio  of  the  Change  Wheels. — As  has  been  men- 
tioned already,  the  change  wheels  are  interposed  between  the 

*  It  is  reported  that  Sir  Joseph  Whitworth,  feeling  the  importance  of  a 
thoroughly  true  leading  screw,  spent  an  immense  deal  of  money  upon  the 
scraping  and  finishing  of  a  parent  screw  for  a  first-class  lathe,  from  which 
many  of  the  best  screws  in  this  country  have  been  copied. 


CHANGE  WHEELS.  177 

back  end  of  the  lathe  spindle  and  the  leading  screw,  for  the  pur- 
pose of  transferring  motion  to  the  saddle,  and  determining,  that 
the  cutting  tool  shall  be  moved  through  a  definite  pitch  for  each 
rotation  of  the  cylinder  to  be  turned  or  screwed.  Every  turn  of 
the  leading  screw  moves  the  saddle  and  cutting  tool  through  a 
distance  equal  to  its  pitch,  and  consequently  if  the  bar  to  be 
screwed,  turns  at  the  same  rate  as  the  leading  screw,  the  pitch  ot 
the  screw  cut  upon  it,  will  be  the  same  as  that  of  the  leading 
screw.  If  it  moves  faster  than  the  leading  screw,  the  pitch  will 
be  less;  and  if  slower,  the  pitch  will  be  correspondingly  greater. 
It  therefore  follows  as  a  matter  of  course,  that  if  we  fit  wheels  on 
the  lathe  spindle  and  on  the  leading  screw  of  the  same  diameter, 
or  having  the  same  number  of  teeth,  the  screw  being  cut  will 
have  the  same  pitch  as  the  leading  screw.  If  we  fix  a  small 

Einion,  or  one  with  few  teeth,  on  the  lathe  spindle  ai^d  a  wheel  of 
irge  diameter,  or  many  teeth  on  the  leading  screw,  the  pitch  of 
the  screw  to  be  cut  will  be  small,  compared  with  that  of  the  leading 
screw.  Or,  if  the  number  of  turns  per  minute  of  the  leading 
screw  be  greater  than  that  of  the  screw  being  cut,  the  pitch  of 
the  latter  will  be  greater  than  that  of  the  former,  and  vice  versa* 
Rules  for  Calculating  the  Required  Number  of  Teeth 
in  Change  Wheels. — The  following  rules  simply  express  the 
previous  reasoning  in  the  form  of  proportion.  In  applying  them, 
the  student  should  again  refer  to  the  end  views  of  the  change 
wheels  in  the  first  figure  of  this  Lecture. 

Pitch  of  ncrrw  to  be  cut_  No.  of  teeth  in  ist  driver  x  No.  in  2nd  driver. 
1'itch  of  guiding  screw     No.  of  teeth  in  ist  follower  x  No.  in  2nd  follower. 
Let  pe       =  Pitch   of  screw  to  be  cut  in  inches,  or  fraction  of 

inch,  between  two  threads. 

„   pa       =  Pitch  of  guiding  screw  »».»».        » 

„  dr  d,  =  Diameters  or  number  of  teeth  in  drivers. 
»>    Juft  =  Diameters  or  number  of  teeth  in  followers. 

Then, 
Or, 

*  What  was  said  in  Lectures  XII.  XIII.  and  XIV.  enables  the  student  to 
pee  clearly  the  velocity  ratio  between  the  cut  screw  and  the  leading  screw. 
We  need  scarcely  remind  the  student  that  the  above  statements  refer  to 
the  pitch  of  a  screw  as  the  distance  between  two  consecutive  threads,  and  not  to 
the  number  of  threads  per  inch.  If  the  number  of  threads  per  inch  of  its 
length  are  taken  as  the  pitch,  instead  of  the  distance  between  two  threads, 
the  reverse  ratio  will  hold  good.  Since  a  pitch  of  £"  means  4  threads  to 
the  inch,  a  pitch  of  $"  means  3  threads  to  the  inch,  and  a  pitch  of  £"  means 
2  threads  to  the  inch.  Or,  the  number  of  threads  per  inch  is  inversely 
proportional  to  the  distance  between  two  consecutive  threads  of  the  screw. 


LECTURE  XVL 

When  the  train  of  wheels  is  a  compound  one,  as  in  this  case,  the 
two  intermediate  multiplying  or  reducing  wheels,  /  and  dv  are 
fixed  to  any  outstanding  movable  arm  or  quadrant  at  the  left-hand 
end  of  the  lathe,  so  as  to  bring  them  into  gear  with  dl  and  /,.  (See 
second  view  of  the  previous  figure.) 

If  the  train  of  wheels  is  a  simple  one,  as  in  the  first,  third,  and 
fourth  views  referred  to  above,  where  there  is  only  one  driver,  d, 
and  one  follower,/,  with,  when  necessary,  one  or  more  idle  pulleys, 
IP,  simply  for  the  purpose  of  connecting  d  and  /and  of  giving  J 
the  desired  direction  of  rotation,  then  — 


**-7, 
JV  / 

Should  the  pitch  of  a  screw  be  expressed  by  the  number  of 
threads  per  inch  of  its  length  —  as  is  usually  the  case  in  tables  of 
screws  and  change  wheels  —  then  you  can  either  convert  this 
number  into  the  pitch  proper,  by  taking  its  reciprocal  —  (i.e.,  by 
making  the  number  of  threads  per  inch  the  denominator  of  a 
fraction,  with  i  for  the  numerator)  or  you  may  say  — 

Let  tc  =  Threads  per  inch  of  screw  to  be  cut. 
„   ^  =  Threads  per  inch  of  guiding  screw. 

Then,  since  the  number  of  threads  per  inch  are  inversely 
proportional  to  the  distance  between  any  two  consecutive  threads, 


*0        PC 

Or,         .        .     t0  x  ^  x  da  =  t0  x  /,  x  ft 

If  the  train  is  a  simple  one,  then 

i-=/;or,^  x  d-iff  x/ 
la       a 

EXAMPLE*  I.  —  The  lathe  illustration  further  on,  has  a  guiding 
screw  of  J"  pitch,  or  4  threads  to  the  inch.  Calculate  the 
number  of  teeth  in  the  change  wheel  to  be  fixed  to  the  end  of  the 
guiding  or  leading  screw  in  order  to  cut  a  screw  of  8  threads  td 
the  inch  when  the  driver  on  the  lathe-spindle  has  40  teeth. 

Compare  the  answer  with  the  change-wheel  table  printed  above  the 
general  view  of  the  screw-cutting  lathe,  further  on  in  this  Lecture. 

ANSWER.  —  Here  te  =  8  ;  t  —  4  ;  d  =  40  ;  and  you  are  required 
to  find/. 


NUMBER  OF  TEETH  IN   CHANGE  -WHEELS.  179 

By  above  formula, 

!«  =/;     or,  *     -  £    .*./~8  x  *°  =  80  teeth. 

tg       d  '  4          40  4 

By  using  the  previous  formula,  we  have  pc  =  J"  and  pg  =  J" 
...  &       *s     0r,|=^.../=i44o  =  8_X 

P<7          /  4  /  8  4 

It  is  at  once  evident  from  this  example  that  you  avoid  having  to 
multiply  and  divide  by  sometimes  awkward  fractions  if  you 
consider  the  number  of  threads  per  inch  as  the  measure  of  the 
pitch  of  the  screw,  instead  of  the  distance  between  two  threads. 

EXAMPLE  II.  —  The  guiding  screw  of  a  lathe  is  %'  pitch,  and 
you  are  required  to  cut  screws  of  •£$"  and  ^V'  pitch  respectively. 
Determine  the  number  of  teeth  in  the  follower,  given  the  use 
of  a  driver  having  20  teeth. 

ANSWER.  —  For  a  screw  of  ^"  pitch,  or  10  threads  per  inch,  and 
using  a  driver  of  20  teeth,  we  get  by  the  above  formula  for  a 
simple  train, 

i_  =  /;      or    IO-IOX  I0-100-/ 
tg       d'         '22       10      20       d 

For  a  screw  of  •£§"  pitch  the  number  of  threads  per  inch  will  be 
20,  and  using  a  driver  of  20  teeth,  we  find  from  the  formula 
for  a  compound  train  — 

i.         '_x/t 

<  d, 


20   x      40 

Here  we  multiplied  numerator  and  denominator  by  20,  in  order  to 
obtain  suitable  wheels,  of  which  dv  will  have  20  teeth.  (See  in 
the  previous  figure  the  second  of  the  end  views  showing  change 
wheels.) 

Movable  Headstock  for  a  Common  Lathe.  —  Before 
describing  a  complete  screw-cutting  lathe  we  will  explain  the  use 
and  construction  of  this  part  of  a  common  small  lathe  for  ordinary 
work.  As  will  be  seen  from  the  accompanying  rough  sketch,  it 
consists  of  a  cast-iron  poppet-head  planed  on  its  under  side,  so  as 
to  engage  the  breadth  of  the  top  of  the  shears.  It  may  be  bolted 
thereto  in  any  desired  position  (along  the  length  of  the  bed)  by 
an  underneath  iron  plate  placed  across  the  shears,  and  a  single 
vertical  bolt.  The  upper  portion  of  the  head  is  cylindrical,  and 
is  bored  for  about  seven-eighths  of  its  length  to  receive  a  round 


i8o 


LECTURE  XVI. 


hollow  steel  mandril,  M,  and  for  the  remaining  one-eighth  to 
receive  the  spindle  S.  The  mandril  is  fitted  in  front  with  a 
tapered  centre,  0,  and  behind  with  a  screw  nut,  N".  The  centre 
is  for  carrying  one  end  of  the  job  to  be  operated  upon  by  the 
turning  tool,  and  the  nut  is  for  engaging  the  screwed  part  of  the 
spindle  S.  On  the  back  end  of  the  spindle  there  is  a  collar,  c 
(kept  in  position  by  a  larger  collar  or  guard,  G,  with  small  screws), 
and  a  hand-wheel,  H\V.*  Consequently,  by  turning  this  wheel  in 


MOVABLE  HEADSTOCK  FOR  A  COMMON  LATHE. 


one  direction  the  mandril  and  its  centre  are  forced  forward,  and 
when  moved  in  the  opposite  direction  they  are  screwed  backwards. 
To  prevent  the  mandril  turning  round,  it  is  fitted  with  a  longi- 
tudinal slot  on  its  underside,  into  which  fits  the  flattened  or 
rounded  end  of  a  small  screw,  s.  A  fixing  stud,  FS,  with  a 
handle,  enables  the  mandril  to  be  clamped  to  the  head  when  it 
has  been  adjusted  by  the  hand  wheel  and  screwed  spindle. 

Description  of  a  Screw-cutting  Lathe. — By  the  favour  of 
Messrs.  John  Lang  &  Sons  we  are  enabled  to  give  a  general  view, 
with  a  complete  set  of  reduced  working  drawings,  carefully 
indexed  to  every  detail,  of  the  very  strong  and  superior  6-inch 
centre  screw-cutting  lathe,  lately  presented  to  the  Author's  Elec- 
trical Engineering  Laboratory  and  Engineering  Workshop  by 
Mr.  Andrew  Stewart,  of  Messrs.  A.  &  J.  Stewart,  and  Clydesdale. 
This  lathe  weighs,  with  all  its  chucks  and  supernumerary  parts, 
over  1 5  cwt.  It  has  a  bed  6  feet  long,  and  admits  a  bar  3  feet 

*  This  arrangement  of  collar  and  guard  is  neither  good  nor  strong, 
although  frequently  adopted  in  the  case  of  small  foot-lathes.  The  collar 
should  be  inside  the  bored  head,  behind  the  nut  N. 


MOVABLE   HEADSTOCK   OF  A  SCBEW-CUTTING   LATHE.    l8l 


2  inches  between  its  centres.  The  bed  is  9^  inches  broad  and 
6  J  inches  deep.  The  gap  is  9  inches  wide  and  6  inches  deep ; 
consequently  the  lathe  can  swing  a  job  of  20  inches  diameter  clear 
of  the  leading  screw,  and  one  of  24  inches  diameter  when  this 
screw  is  withdrawn  from  its  bearings.  The  speed-cone  has  three 
pulleys,  each  2  J  inches  broad,  the  diameter  of  the  largest  being 
8  inches  and  that  of  the  smallest  4  inches. 

The  makers  have  planed  and  scraped  the  bed  to  a  true  bearing 
surface,  and  have  so  fixed  the  gap  piece  that  it  cannot  wear  loose 
or  spring  the  bed. 


k 


_EH_ 


N\\\\\\\Vy\\\\\\\\\\\V  •'          , ,     ._!. 

— *-y  •  •     -    '  -j---^  ^ , 


END  VIEW.  LONGITUDINAL  SECTION. 

MOVABLE  HEADSTOCK  OP  SCREW-CUTTING  LATHE. 
INDEX  TO  PARTS. 


JBP  represents  Bottom  part. 
TP          „          Top  part. 

S          „          Spindle,  or  mandril. 
SC          „          Steel  centre. 
SS          „          Steel  screw. 
HW  Hand  wheel. 


BH  represents  Binding  handle. 
AS  Adjusting  screw. 


ES 

EH 

B 

P 


Eccentric  spindle. 
Eccentric  handle. 
Bolt  for  clamping. 
Plate  under  B. 


The  movable  or  loose  headstock  is  gripped  to  the  bed  by  an 
eccentric  motion  worked  by  a  handle,  so  that  it  may  be  instantly 
clamped  in  position  without  the  trouble  of  finding  a  key  to  fit 
the  usual  nut,  and  then  screwing  it  gradually  home.  The  upper 
part  of  this  head,  which  carries  the  mandril  or  spindle,  has  a  side 
adjustment  by  means  of  a  side  screw,  whereby  the  steel  centre 
may  be  truly  aligned  with  the  corresponding  centre  of  the  fast 
headstock,  or  it  may  be  moved  to  the  one  side  or  to  the  other  in 
the  'case  of  taper  turning.  A  small  oil-holder  is  cast  on  the  back 
side  of  the  head  to  facilitate  the  oiling  of  the  steel  centre  without 
having  to  look  for  an  oil-can. 


182 


LECTURE  XVI. 


FRAMING,  ETC.,  FOE  A  SCREW -CUTTETO  LATHE.          183 


184 


FAST   HEADSTOCK   OF  SCREW-CUTTING   LATHE. 


H- 

•8 


1 

g^l. 

o  1S&I 


»a  »  j  M 

su  cc    >-  J5    « 

.3  3  ?,  |  > 
«^ 


•&mi«  * 

•ilil:-  * 


3   I  so 

ipr^i==i!i  t^ii 
Mliiidi^^K-|Uil] 

rrTr^  r^        C3        H  &-        Ld  h-J        WWW 


DESCRIPTION  OF  A  SCREW-CUTTING  ..LATHE. 


I8S 


The  spindle  of  the  fast  headstock  is  made  of  hard  crucible  steel 
ground  accurately  cylindrical,  where  it  fits  into  parallel  gun- 
metal  bearings.  These  bearings  are  of  extra  diameter  and  length. 
This  spindle  is  bored  hollow  for  1 2  inches  of  its  length,  in  order 
to  admit  small  rods  for  making  terminals  and  screws  in  electrical 
engineering  work.  The  speed-cone  is  turned  inside  and  outside, 
and  properly  balanced.  A  specially  strong  and  simple  reversing 
gear  has  been  fitted  to  the  back  end  of  this  headstock,  whereby 
the  machine-cut  steel  pinions  for  turning  right  and  left  hand 
screws  may  be  put  into  or  out  of  gear  by  simply  depressing  or 
elevating  a  reversing  handle.  The  back-motion  gear  is  actuated 
by  means  of  a  handle  and  eccentrics  on  each  end  of  the  back- 
motion  shaft ;  whilst  the  front  wheel  (or  last  follower,  F2,  as  we 
have  symbolled  it  in  the  formula)  is  locked  to  the  cone  or  thrown 
out  of  gear  therewith  in  the  usual  way — viz.,  by  a  bolt  fitting 
into  a  sliding  slot  in  the  cone  and  a  projecting  nut  on  the  side  of 
the  toothed  wheel. 

The  saddle  has  T  slots  on  its  upper  side  for  the  purpose  of 
bolting  work  to  it  that  requires  boring  out,  and  which  necessitates 
the  removal  of  the  slide  rest.  A  quick  hand  traverse  motion  is 
provided  for  the  saddle  by  means  of  a  rack  and  pinion  motion, 
quite  independent  of  the  sliding  motion  of  the  leading  screw. 
The  leading  screw  is  turned  to  the  standard  pitch  of  J  inch,  or 
four  threads  to  the  inch.  The  engaging  nut  is  made  in  halves, 
so  that  it  may  grip  the  leading  screw  fairly  at  the  top  and  bottom 
of  the  threads.* 

*  In  order  to  make  the  construction  and  action  of  the  split  nut  which 
engages    the     leading    screw- 
clearer,  we  show  here  an  en- 
larged view  with  the  halves  of 

the    nut,    N< *N,   slightly 

apart,  and  the  disc  handle  re- 
moved, so  as  to  bring  into 
full  view  the  two  eccentric 
slots,  ES,  which  guide  the  two 
steel  pins,  P  and  P,  fixed  on 
N  and  N.  By  comparing  this 
view  with  the  others  under 
heading  "  Saddle  and  Slide," 
the  student  will  see  how,  by 
merely  turning  the  disc  handle 
DH  the  disc  D  is  moved  round 
through  nearly  a  quarter  of  a 
circle,  and  the  eccentric  slots 
ES  cause  the  pins,  P,  P,  to 
move  closer  to  or  further  away  ENLAEGED  VIEW  OF  SPLIT  NUT  FOB 
from  the  centre  of  the  disc  D,  LEADING  SCREW,  &c. 

and   consequently    move    the 
two  parts  of  the  nut,  N,  N,  in  or  out  of  gear  with  the  leading  screw. 


I  86      SADDLE  AND  SLIDE  FOR  A  SCREW-CUTTING  LATHE. 


»    1". 

.  oo  -H  ^3  be  far, 

02  -g   be  3   fl 

H-IJM  .«§'§! 

^  S#!3A.S|  |  I 

iieisi£J5d 


PH 


g  «3  fr  £  o     . 

2  ^s,||l||- 

g    ^^-a-g  S).s|.S 

g       0§§^*1*3^ 

p    EH  tt  EH  H  h  t-5  ^5  ^ 

S 


,§    a_r 


CHANGE-WHEEL  GEAR. 


i87 


*"  r-3     02  ^^ 

~"  A~*  rC      ^ 

be  ticS  t>o  fe  i 

flC         S3     i    60 

.2  »i-i   ^-**  CD   zr 

^     W     E?  QQ     vU  C 


^iliii 


fe  ft 


1 88  LECTURE   XVI. 

A  compound  slide  rest  is  fitted  to  the  top  of  the  saddle,  having 
large  bearing  surfaces  with  adjustments  for  taking  up  the  wear, 
and  a  swivel  arrangement  for  conical  boring. 

All  the  toothed  wheels,  including  the  change-wheels,  have  had 
their  teeth  cut  directly  from  the  solid  casting,  by  the  makers' 
special  tool  for  that  purpose,  so  that  back-lash,  and  consequently 
noise  and  vibration  arising  from  fast-speed  driving  may  be  mini- 
mised as  far  as  possible. 

The  driving  shaft  has  anti-friction  steel  roller-bearings.  It  is 
connected  to  the  foot-treadle  at  each  end  by  a  pulley,  chain,  and 
crank.  The  driving-cone  is  so  stepped  that  the  belt  has  equal 
tension  on  any  corresponding  pair  of  driving  and  driven  pulleys. 
It  is  sufficiently  heavy  to  act  as  a  fly-wheel.  It  is  balanced  along 
with  the  treadle  to  secure  an  easy,  steady  drive.  A  power-drive 
may  be  applied  if  desired,  but  the  author  believes  that,  as  students 
should  work  in  pairs  or  in  sets  of  three  in  a  laboratory,  they  will 
take  a  deeper  interest  in  their  experiments  if  they  have  turned 
out  everything  by  their  own  skill  and  labour,  than  if  motive  power 
were  freely  supplied  to  them. 

Of  heavy  chucks  there  are  a  very  complete  set,  including  a  four- 
jaw  expanding  chuck,  clement  driver,  drill  chucks  for  both  the 
fast  and  loose  head  spindles,  <fec. 

The  student  should  now  go  over  each  drawing  most  carefully 
by  aid  of  the  corresponding  index  to  parts,  and  compare  the 
drawings  with  an  actual  screw-cutting  lathe. 

Hexagon  Turret  Lathe. — Before  completing  this  section  it 
will  be  necessary  to  illustrate  one  of  the  latest  and  most  important 
labour-saving  appliances.  The  hexagon  turret  lathe  is  principally 
used  for  repetition  work,  and  it  is  generally  found  in  large  en- 
gineering shops  (where  this  class  of  work  is  carried  on  to  a  large 
extent)  to  pay  the  firm  to  spend  some  time  in  designing  suitable 
cutting  tools.  The  lathe  with  hexagon  turret  shown  in  frontis- 
plate,  and  the  several  detail  views  of  turret  head  and  saddle  as 
designed  by  Messrs.  Alfred  Herbert,  Ltd.,  Coventry,  who  have 
kindly  given  me  assistance  in  supplying  the  necessary  drawings 
and  photos  from  which  these  figures  were  reproduced.  The 
special  features  of  this  lathe  are — (i)  the  ease  with  which  the 
speeds  and  feeds  can  be  changed,  also  the  feeds  reversed  5(2)  the 
greatly  improved  set  of  tool  holders  on  the  turret  head ;  (3)  the 
improved  lever  operated  chuck  for  gripping  the  bar ;  and  (4)  the 
new  stop  motion,  by  which  each  tool  has  its  own  longitudinal  and 
transverse  stops. 

Description  of  Lathe. — Fig.  i  on  page  189  shows  a  direct- 
geared  motor-driven  headstock  for  this  lathe.  The  motor  is 


HEXAGON  TURRET  LATHE. 


189 


Front  Elevation  and  Plan  of 
Back  Friction  Clutch 


FIG.  i.— SECTIONAL  PLAN  OP  AN   ELECTRICALLY-DRIVEN    HEXAGON   TURRET 
LATHE,  SHOWING  ARRANGEMENT  OF  GEARS  AND  FRICTION  CLUTCHES. 


M  represents  Electric  motor. 


MS  „  Motor  spindle. 

\  P6  ,,  Pinions. 

\-F6  „  Followers  or  wheels. 

IS  ,,  Intermediate  gear  stud. 

Lj-Ko  „  Keys  fixed  in  shafts. 

JSj,  CS2  „  Friction  clutch  sleeves. 

lv  R.J  „  Rollers. 

P  „  Plunger  or  wedge. 

FS  „  Friction  screw. 

FP  >,  Friction  plate. 

AP  „  Adjusting  pin. 

FC  „  Friction  clutch  centre. 

FBS  „  Friction  J^ack  shaft. 

Cf  „  Collar  or  ruff  on  shaf  tFBS 

SS  „  Sliding  shaft. 

^2,  L3  „  Levers  for  working  SS. 


INDEX  TO  PARTS. 

PP  represents  Pulley  for  working  force 

pump. 
LS          ..        Lathe  spindle. 


RCj,  RC2  ,,  Friction  ring  carriers. 

Glf  G2  „  Gluts. 

TO  „  Toggle  carrier. 

T  „  Toggle. 

S15  S2  „  Studs. 

FRj,  FRo  „  Friction  rings. 

S3,  S4  „  Shoes    for    friction   r 
and  toggle  carrier. 

Lj  „  Lever  for  actuating  R 

and  RC2 

BS  „  Back  gear  shaft. 

B!,  B2  „  Bearings  for  LS. 

B3,  B4  „  Bearings  for  BS. 


LECTURE  XVI. 


«J£ 
2  .•§ 


(«£So202WOo26o^HOWcc 


isli: 


tn 


c5    .  ^pq 


*       £ 

HOwl 


^SN  ,    ** 

*«*#%*##?  * 

nti  \     /      if      Jj&J  <(n,      / 


HEXAGON  TUKRET  LATHE. 


LECTURE  XVI 


^  J  c/2  ^ 

02  PQ  PQ  m  «  02  PH  O  02  W 


OF   THE 

UNIVERSITY 

OF 


HEXAGON     TURRET     LATHE. 


ST£AD¥  SL/DE 
ADJUST/NO  SCREW 

FUNNEL  FOR  1MB  ft J GATING 
3T£ADy  £LIQ£  CLAMP 


ffACK  P/N/ON  SMAfT 

STOP  CLAMP 

\ 

WORM  StiA, 


CUTTER  CLAMP/MG  SCP£W* 
CUTTER  SL/DE 
TOP  iNCLIME 


BODV 


ELEVATING 


SCREWS  •*£-• ItJ- 


SUPPORT 
FACE 

CUTTER  SL/DE  G/B  SCff£HS& 
CUTTER  SL/DE   CLAMP 


THE     PATENT     ROLLER     STEADY    TURNER. 


The  following  improvements  have  been  made  in  this  lathe  : — • 

1. — The  motor  is  mounted  on  a  hinged  base  plate,  by  which  it  can  be 
elevated  and  depressed,  and  it  drives  the  headstock  by  means  of  a  belt.  The 
swinging  movement  of  the  motor  serves  to  adjust  the  tension  of  the  belt. 

2. — The  geared  headstock  provides  sixteen  speeds  within  itself,  thus 
enabling  a  constant  speed  motor  to  be  used,  and  renders  the  lathe  equally 
applicable  to  direct  or  alternating-current  motors. 

3. — The  driving  pulley  of  the  lathe  contains  an  epicyclic  reverse  gear  for 
giving  the  spindle  speeds  in  every  direction. 

4.— The  feed  motion  now  gives  nine  feeds  instead  of  three  as 
formerly. 

5. — The  turning  tool  holders  are  fitted  with  roller  steadies,  enabling 
much  higher  cutting  speeds  to  be  used  than  formerly. 

6. — A  scale  and  adjustable  pointer  are  fitted  to  the  bed  to  enable  direct 
measurements  to  be  made  without  using  a  rule. 


CHUCKS — TURRET-HEAD  AND  SADDLE.  1 93 

of  8  B.H.P.  It  is  a  semi-enclosed  motor  with  speed  variation 
obtained  by  inserting  resistance  in  the  field  coils  from  a  60- 
point  shunt  controller.  Consequently  any  desired  speed  is 
easily  got  between  1150  and  1850  revolutions  per  minute.  The 
motor  M  and  spindle  MS  is  attached  to  the  rawhide  pinion  P,. 
This  pinion  drives  by  means  of  the  compound  gear  train  Fv  P,, 
F2  with  P3,  the  first  shaft  FBS.  As  will  be  seen  from  the  sec- 
tional plan  the  shaft  FBS  carries  two  spur  pinions  P3  and  P5, 
either  of  which  can  be  put  in  or  out  of  gear  with  follower  wheels 
F3  and  F5  by  means  of  the  friction  clutches  CS^  CS2.  The  levers 
LS,  L3  with  central  spindle  SS  actuate  the  friction  clutches 
CS1?  CS2.  Hence,  for  every  speed  of  the  motor  this  shaft  FBS 
can  give  two  different  speeds  to  the  sleeve  or  followers  F3  and  F4 
on  the  lathe  spindle  LS  which  is  driven  from  shaft  FBS.  These 
follower  wheels  correspond  to  the  cone  pulley  on  an  ordinary 
lathe,  and  drive  the  spindle  LS  direct  through  the  friction  ring 
carrier  RC2,  which  slides  along  the  key  K,  fixed  in  the  lathe 
spindle  LS ;  or,  through  double  gearing  P4,  F4,  BS,  P6,  F6  and 
friction  ring  carrier  RCj  connected  to  lathe  spindle  LS  by  key 
K2,  the  change  from  the  one  to  the  other  is  made  by  means 
of  the  lever  Lr  The  friction  clutches  on  the  shaft  FBS  are  of 
the  expanding  ring  type  operated  by  means  of  wedge  surfaces  P 
and  rollers  R1?  R2,  whilst  the  clutches  RC1?  R03  on  the  spindle 
LS  are  also  of  the  expanding  ring  type,  but  actuated  by  toggles  T. 
(See  Index  for  toggle  joints.) 

The  lathe  shown  by  the  frontispiece  is  of  a  later  type. 
It  is  driven  by  a  constant  speed  motor  mounted  on  a  hinged 
base  plate.  This  base  plate  provides  for  the  necessary  tightening 
of  an  endless  belt. 

Chucks. — The  lathe  may  be  fitted  either  with  an  automatic 
or  universal  chuck.  The  former  chuck  is  recommended  where  the 
bars  to  be  worked  are  practically  straight  and  cylindrical,  whilst 
the  latter  is  used  on  bars  which  are  badly  out  of  round  or  not 
straight. 

Looking  at  the  front  outside  view  of  lathe,  it  will  be  seen  how 
that  the  bar  to  be  turned  is  gripped  by  an  automatic  chuck 
worked  by  the  longer  handle  parallel  to  and  beside  Lr 

The  automatic  chuck  may  be  opened  and  closed  whilst  the 
machine  is  running,  and  it  has  the  advantage  of  holding  finished 
work  without  bruising. 

Turret-Head  and  Saddle. — It  will  be  seen  from  the  frontis- 
plates  and  the  accompanying  views,  Figs.  2  and  3,  that  the  saddle 
can  be  moved  either  by  hand  or  power,  as  in  the  ordinary  lathe. 
In  the  former  method  of  working,  the  saddle  is  moved  by  nv»ans 


194 


LECTURE  XVI. 


of  the  pilot  handle  PH  acting  through  wheels  SPj,  SW:,  RP  to  the 
rack  R;    whilst,  by  the  second  method,  motion  is  taken  from  the 


FIG.  5. — THE  "COVENTRY"   SELF-OPENING  Dix  HEAD — WITH 
ROUGHING  AND  FINISHING  ATTACHMENT. 


FIGt  6. — TURNING  TOOLS— VIEW  SHOWING  RIGHT   AND  LEFT-HAND 

CUTTEES. 

feed  or  traverse  shaft  FS  through  the  spur  wheels  SP8  and  worm 
W,  worm  wheel  WW,  and  wheels  SPp  SW1?  RPX  to  the  rack  R. 


CHANGE  WHEEL  GEAR,   ETtt  195 

The  worm  wheel  is  carried  in  a  cradle  CH,  hinged  round  the 
shaft  WS,  and  held  in  position  by  the  stop  or  trip  lever  TL  and 
spring  SS2.  Whenever  the  hand  traverse  is  to  be  used,  the 
traverse  worm  W  is  released  by  the  cradle  dropping  due  to  its 
own  weight,  after  releasing  the  catch  or  trip  lever  TL. 

When  the  handle  TL  is  pressed  to  the  right,  the  bar  connected 
to  it  turns  on  a  fulcrum,  and  causes  the  worm  cradle  handle  CH 
to  be  dropped,  thus  stopping  any  further  traverse  of  the  saddle 
along  the  bed  of  the  lathe.  This  traverse  may  also  be  stopped 
automatically  by  means  of  the  trip  lever  TL  coming  into  contact 
with  the  stop  S,  which  is  set  at  the  desired  position  on  the  stop 
rod  SR,  and  actuating  the  feed  trip  lever  FTL.  The  hexagon 
head  can  be  rotated  into  any  position  by  turning  the  handle  H, 
plunger  pinion  PP,  and  plunger  P2,  so  that  P,  is  freed  from  the 
cast  steel  bush  B. 

Bolted  to  the  faces  of  the  hexagon  head  are  the  specially 
shaped  tools,  while  boring  bars  may  be  passed  through  the 
circular  holes  shown  in  the  hexagon  head  and  held  firmly  by 
bolts. 

Change  Wheel  Gear. — It  will  be  seen  from  the  illustrations 
in  Fig.  4  that  the  traverse  or  clutch  shaft  CS  is  rotated  from  the 
mandril  through  the  feed  pulley  FP,  and  the  pairs  of  wheels  SW^ 
SW6.  Any  pair  of  wheels  may  be  put  into  gear  by  means  of  the 
hand  lever  HL,  which  acts  upon  the  internal  sliding  rod  SR,  and 
slides  a  cotter  key  SO  so  as  to  fix  SW4,  SW5  or  SW6  to  the  shaft 
SR.  The  handle  or  reversing  lever  RL  puts  the  claw  clutch  RC 
in  gear  with  bevel  pinions  BPX  or  BP,  to  give  either  a  forward  or 
backward  rotation  or  stoppage  of  the  shaft  CS.  Consequently, 
three  variations  of  speed  for  cutting  purposes  may  be  obtained 
by  altering  the  position  of  the  vertical  hand  lever  HL,  while  the 
direction  of  rotation  of  the  feed  shaft  may  be  changed  by  turning 
the  reversing  lever  RL. 

Turning  Tool  Holders  as  supplied  with  the  lathe  are  one  of 
its  most  important  features.  A  general  idea  of  the  different 
tools  on  the  turret  or  hexagon  head  is  obtained  from  the  frontis- 
plate.  When  another  tool  is  required  to  be  fetched  up  to  the 
work,  this  is  effected  by  turning  the  short  handle  H  seen  near  top 
of  saddle  downwards,  thus  releasing  the  turret,  which  can  now  be 
turned  as  desired,  afterwards  bringing  the  handle  back  to  its 
normal,  horizontal  position,  and  firmly  fixing  the  hexagon  head. 
Separate  views  of  the  turning  tool  holder,  cut-off  tool  rest  and 
self-opening  die  head  for  the  hexagon  head  are  also  given,  see 
Fig.  6  on  page  194  and  plate  facing  page  193. 

Rapid  Turning  with  High  Speed  Steel. — The  student- 
will  be  interested  to  learn  something  regarding  the  recent  im- 


196 


LECTURE  XVI. 


provements  which  have  been  made  in  rapidly  turning  out  work 
with  the  most  modern  electric-driven  lathes,  and  the  new  kinds 
of  high  speed  steel.  Consequently,  we  have  made  a  short 
abstract  from  three  papers  read  recently  before  Engineering 
Societies  where  these  matters  were  specially  dealt  with  and 
discussed.* 


FIG.  7.— CUT-OFF  TOOL  REST  SHOWING  STEPPED  CUT-OFF  TOOL. 

Description  of  Lathe. — The  lathe  on  which  all  the  tests 
were  made  was  supplied  by  Armstrong,  Whitworth  &  Co.,  and  was 
on.e  of  their  15"  centre  screw-cutting  lathes,  taking  in  9'  6" 
between  the  centres,  but  for  these  experiments  18"  headstocks 
were  fitted.  The  fast  headstock  had  both  double  and  treble  back 

*  See  Proc.  Inst.  Eng.  and  Ships,  in  Scotland,  vol.  xlvii.  1904,  for  Mr. 
Charles  Day's  Paper  on  "  Experiments  with  Rapid  Cutting  Steel  Tools." 
Also  see  Mr.  J.  M.  Gledhill's  paper  on  "High  Speed  Tool  Steel,"  read 
before  the  Coventry  Engineering  Society  in  March  1904,  and  Mr.  P.  V. 
Vernon's  paper  on  "  Speeds  of  Machine  Tools,"  read  before  the  Manchester 
Association  of  Engineers  on  November  14,  1903. 


EESULTS  I9/ 

gears,  the  ratios  being  14.9  to  i  and  42.5  to  i.  The  headstock 
was  specially  fitted  with  a  3-step  cone  suitable  for  a  6"  belt.  The 
lathe  was  driven  by  a  direct  current  shunt-wound  motor  of  120 
E.H.P.,  with  a  large  air-cooled  rheostat.  The  speed  of  the  motor 
could  be  varied  between  150  and  300  revolutions  per  minute  at 
no  load  on  the  lathe,  and  from  60  to  300  revolutions  with  heavy 
cuts  by  means  of  the  rheostat.  The  lathe  was  driven  by  two 
intermediate  countershafts  having  10"  belts. 

Results. — The  diagrams  show  the  maximum  cutting  speeds 
successfully  used  in  each  experiment  made  with  rapid  cutting 
steel  tools,  and  the  curves  show  the  average  speeds  during  each  set 
of  trials.  Figs.  8  and  9  show  the  average  results  obtained  from 
those  tools  which  finished  in  such  condition  as  to  warrant  atten- 
tion, whilst  the  dotted  lines  in  the  figures  show  the  maximum 
results  obtained  with  any  tool  which  finished  in  a  satisfactory 
condition. 

It  was  found  that  no  single  make  of  steel  proved  to  be  superior 
to  all  others  in  every  respect,  but  it  would  appear  that  the 
average  curves  are  those  which  may  be  taken  as  standards  of  all 
round  comparison  for  use  in  general  engineering  shops.  The 
following  empirical  formulae  give  approximately  the  cutting  speeds 
which  may  be  adopted  for  different  areas  of  cut  upon  different 
materials,  and  the  curves  show  the  results  obtained  therefrom  : 

For  soft  steel          S *  g6      +  1 2 

A  x  0.013 

„    medium  steel  S  =     *'  23     +  5 
Ax  0.015 

,    hard  steel        S=  A    T'77 — +5 
Ax  0.027 

„    soft  cast  iron  S  = +  20 

A  x  0.02 

i-5 


Ax3 

»  hard  »    »  S= 


Where:  —  S  =  Cutting  speed  in  ft.  per  minute. 

A  =  Area  of  section  of  cut  in  sq.  inches,  i.e.,  traverse 
in  inches  multiplied  by  the  depth  of  cut  in  inches, 


198 


LECTUKE  XVI. 


FIG.  8. — VARIATION  OF  CUTTING  SPEED  WITH  AREA  OF  CUT  STEEL. 


3      1 


FIG.  9.— VARIATION  OF  CUTTING  SPEED  WITH  AREA  OF  CUT  CAST  IRON. 


POWEB  BECORDS— CUTTING  SPEED  RESULTS,  ETC.    199 

Power  Records. — Very  careful  records  were  taken  of  the 
power  used  for  the  various  cuts  and  spaces.  Also,  data  were 
obtained  of  the  forces  brought  to  bear  on  the  cutting  tools. 

Cutting  Speed  Results. — Table  I.  gives  the  average  results 
obtained  from  the  tools  which  finished  in  good  condition.  The 
horse-powers  given  are  the  gross  horse-powers  as  calculated  from 
the  readings  of  electrical  instruments  attached  to  the  motor,  and 
include  the  motor  losses  with  any  countershaft  friction.  The  nei 
horse-powers  required  to  overcome  the  resistance  to  cutting  are 
not  given  in  the  Table.  These  are  only  required  for  determining 
the  cutting  force  on  the  tool  point. 

Endurance  Results. — Table  II.  gives  the  average  result* 
obtained  from  soft  forg<  d  steel  and  medium  cast  iron,  which 
maintained  their  average  cutting  edge  in  fair  condition  foi 
60  minutes  or  longer. 

Comparison  of  Results. — For  the  purposes  of  comparing 
results  which  may  be  obtained  with  the  new  steels  against  those 
obtainable  with  ordinary  Mushet  steel  and  ordinary  water- 
hardened  steel,  tools  made  of  the.se  materials  were  tested,  and 
the  average  results  are  also  given  in  Table  II.  It  will  be  noted 
that  the  new  steels  give  decidedly  improved  results,  and  that 
with  them  the  cutting  speed  can  be  about  twice  as  fast  as  with 
ordinary  Mushet  steel,  and  three  or  four  times  as  fast  as  with 
ordinary  water-hardened  steel. 

An  item  of  interest  which  may  be  mentioned  here  is  that  the 
ordinary  Mushet  steel  can  be  very  greatly  improved  by  treating 
it  in  the  same  manner  as  the  new  steels  when  tempering.  This 
is  a  point  of  value,  as  it  enables  greatly  improved  results  to  be 
obtained  from  existing  tools. 

It  will  be  seen  from  an  inspection  of  Table  I.  that,  when  much 
metal  has  to  be  removed,  this  may  be  done  not  only  more  quickly, 
but  also  with  a  less  expenditure  of  energy  per  Ib.  of  material 
removed,  if  a  heavy  cut  is  taken  at  a  comparatively  low  speed  in 
preference  to  a  lighter  one  at  a  high  speed. 

Cutting  Forces  and  H.P.  for  Lathes. — The  figures  showing 
the  cutting  force  on  tool  points  should  prove  of  great  service  to 
machine-tool  designers. 

The  information  regarding  the  horse-powers  is  worthy  of 
special  attention,  for  it  is  this  element  which  perhaps  will  form 
the  greatest  difficulty  in  the  way  of  using  existing  lathes 
efficiently  with  the  new  high-speed  cutting  tools.  A  lathe  on 
which  a  cut  of  f "  x  J"  on  soft  steel  can  be  taken  is  by  no  means 


2OO 


LECTURE  XVI. 


- 

Ifi- 


ON  O  •^•1^*1-*  •**•  0*^00  Tj-QsO  t-».  c\  1-1  vo  O  oo  vn  u~>  »o  ro  »ooo 
•*l-  ON  *o  N  r^oo  ON  ro  ^  "-i  vo  >->  co  in  o\  to  >-i  •rj-O  <o  TJ-  r^  T^- 

OO   10  rt  T}-OO   r^O    <<t  N    I--  ON  N    ^  Q    LO  .-,  00    O    »^  O  00  CO  OO 
N   IO  H          N    10  O   *->   CO  C^  •*•         i-<    0»    •<*•         N   COVO          »-i   CO 


Tf 

w 


cf 


N    O   t-^OO   »O  rj-  N    r^OOOOOVO    N 


4J  -O        . 

£  u  c    • 

•»ls£ 

^C    S   v-   c 


O    N   w   OOO  vO   10  ON  1-1   •*  O 


i-t  N  ro  10  O  >-t 


O  10  10  t^  O  O\OO  Tf-  »O   t^OO  rf  •->  r^  N-  O  ONOO  »O  •*•  N  vo 
CO^O  ^d"  *O  CO  IO  (O  d  N  ^O  C^  *O  ^"VO  LO  W  N  CO  W  w  M  C<  M 


JA|' 


vo  >-osO 


vo  >-osO  Tj-iomioroM  N  rriM  LOTJ-VO  IOM  N  row  M  M  N  M 

d  d  d  d  d  c>  o*  o*  d  o*  d  o"  d  d  d  c5  o"  6  d  o*  d  d  d  d 


B.2 


ooqoqooqqqoo-qqooooooooo 

6 d d d d d dodddddoddoo d d d d 6 


H.S 


VOvOMMvOvONN    CNVQ    O)    CJ  vO  'O    N    NOVO    M    NvDvO    M 

qo«Mqq«Moo«H-,qoMMqo-'«ooM 
ddddddddddddddddddddddd 


's 

P.-S 


oo  vc 

>O  t^OO  OOVO'«J-lOON        >-iiOWr».__ 
tovo  VO   ^O  1OCO   4s**  COVO   ON  t^\O  O   ONVO 

ddddddddddddddddddddddd 


f*}      10 

wi-^i-iMTj-  csoo 

t^  »Ot^vo   10  10  t-^ 


rj-oo  oo  vo  co  >-i  oo  oo  co  rooo  ON<N  COT^M  qvqoo  NVO 

OOOVO   lOU-jrJ-cON 


at, 

•o  5  Ja 

o  rt  o 

^H.S 

ll-s 


vovo 

LC.<92.80. 


xxxxxxxxxxxxxxxxxxxxxxxx 

VOVOVOOOVOVOVOOOVOVOVOOOVOVOVOOOVOVOVOOOVOVOVOOO 


d 

o    ,. 


AVERAGE   OF  RESULTS   OBTAINED. 


201 


§  , 

S^.S 

* 

^ 

83 

£** 

ON 

M 

2* 

~ 

ON 

OO 

M  w 

C   g 

1sJ  i 

^ 

8 

o 

s 

o 

o 

•j! 

1*3  1 

^ 

CO 

$ 

ON 

0 

8 

1" 

III 

ON 

* 

C& 

N 

ON 

00 

ON 

£ 

gt.g 

en 

w 

d 

- 

O 

d 

d 

d 

-* 

•c.E" 

CN 

0 

S 

CO 

* 

10 

OO 

H 

-  — 

^-       o* 

^ 

M 

*H 

o 

X 

<^ 

-n  £  w 

d 

d 

d 

d 

6 

d 

o 

a 

ex  — 

c 
c 

"S  £ 

co 

^ 

10 

« 

| 

H'l 

§ 

00 

o* 

5 

10 

vO 

- 

V 

. 

r- 

ON 

* 

rr 

CO 

f- 

00 

- 

a 

O 

O 

o 

O 

0 

o 

o 

•~ 

<  cr 

d 

0 

0 

0 

o 

o 

d 

- 

c 

^J 

H 

O 

10 

10 

10 

vo 

10 

10 

10 

•- 

s 

(S| 

o» 

fvj 

C4 

N 

ri 

"3 

u   •/; 

VD 

VO 

VO 

vo 

O 

VO 

vO 

(S 

Ss 

0 

d 

d 

C 

d 

d 

d 

^ 

< 

H 

1 

•s  . 

00 

N 

M 

OO 

M 

OO 

c? 

CO 
OO 

i? 

- 

- 

2  ~ 

d 

d 

d 

d 

d 

d 

d 

- 

?- 

•s  s 

D-,  = 

VO 

0 

_ 

CO 

>o 

to 

w 

5  ^ 

P  5  Ci 

N 

ro 

ro 

rf 

ON 

N 

^ 

O   ~ 

a.  u  c 

ON 

W 

fO 

M 

X 

^•C 

w  S."" 

M 

OJ 

: 

's 

7) 

: 

: 

"S 

- 

S 

C 
0) 

c 
o> 

o 

c 

0) 

a 

W 

t3 

IE 

'O 

1 

oi 

"S 

5 

^ 

s 

>— 

M 

H 

•3 

•S 

3 

1 

'5 

0> 

/a 

3 

- 

1 

- 

O 

y 

M 

P 

2 

^ 

^ 

a 

Q. 

o 

>« 

K 

0> 

t>» 

>^ 

r_j 

Jj 

OD 

rt 

c5 

CO 

oi 

ec 

u 

^ 

£j 

& 

(^ 

a 

a 

Q 

to 

T3 

'O 

to 

^ 

T3 

w 

0 

0 

B 

O 

0 

§ 

: 

: 

: 

g 

1 

00 

s 

s 

r 

jj 

-•^ 

S. 

o 

I 

X 

s 

s 

a 

3 

s. 

s 

s 

S 

i 

S 

202  LECTURE  XVI. 

an  abnormal  one,  and  this  duty  can  be  carried  out  on  most  good 
lathes  of,  say,  12"  centres,  but  the  driving  cones,  the  countershaft, 
and  the  belts  connected  with  few  such  lathes  would  be  suitable 
for  24  h.p.  Further  than  this,  the  line  shafts  in  most  engineering 
shops  are  too  light  to  drive  many  lathes  using  20  h.p.  each,  or 
anything  approaching  that  figure. 

Example. — As  a  small  example  of  what  the  Herbert  Turret 

Lathe  (see  previous  figs.)  has  done  with  this  new  high-speed  steel, 

we  quote  the  following  from  the  Proceedings  of  the  Coventry 

•  Enginering  Society,  as  found  in  the  paper  read  by  Mr.J.  M. 

Gledhill  on  March  4,  1904 : 

Sample  No.  i  was  a  i"  bolt  6"  long  in  shank,  with  i£"  round 
head,  i^"  deep,  and  with  the  point  screwed  for  2". 

This  was  finished  complete  in   5  minutes   28  seconds,  using 
"  A.  W. "  tools.     The  following  are  the  details : 
Reducing  i-J-  in.  bar  to  i  in.,  6in. 

long,  96  cuts  per  inch      .         .     4  min.    o  sees. 
Screwing    .....  40     „ 

Cutting-  off          ....  28     „ 

Idle  movements,  etc.  .         .         .  20     „ 

Total       .         .  5  min.  28  sees. 

The  above  time  does  not  include  the  facing  of  the  back  of  the 
head,  which  would  require  J  minute. 

This  bolt,  made  in  the  ordinary  way  with  ordinary  tool-steel, 
with  a  good  operator,  requires  15  minutes. 

Forging  and  Hardening  the  Tools. — In  forging,  annealing, 
and  hardening  crucible  steels  it  is  essential  that  the  most  suitable 
temperatures  should  be  found  for  all  of  these  processes,  and  then 
accurate  means  be  taken  to  ensure  such  temperatures  being 
actually  obtained  as  near  as  practically  possible.  This  can  only 
be  effected  by  the  skilful  use  of  pyrometers  or  other  scientific 
heat-recorders,  for  to  work  on  the  old-fashioned  lines  of  judging 
by  the  eye  is  no  criterion  of  actual  temperature,  and  is  no  longer 
advisable.  It  is  now  known  that  every  composition  of  steel  has 
its  own  definite  temperature  that  is  best  suited  for  obtaining 
from  it  the  most  satisfactory  results,  and  the  nearer  this  can  be 
worked  to  the  better,  any  deviation  from  the  correct  temperature, 
up  or  down,  involving  a  corresponding  difference  in  the  efficiency 
of  the  steel.* 

*  Students  may  refer  here  to  the  illustrated  descriptions  of  these  Pyro- 
meters in  the  Author's  Elementary  and  Advanced  Steam  Books. 


FORGING  AND  HARDENING  THE  TOOLS  203 

Having  obtained  a  bar  of,  say,  the  Armstrong- Whitworth  ot 
"  A.  W."  brand  of  tool  steel,  it  is  necessary  to  cut  off  the  required 
lengths ;  and  this  must  be  done  at  a  forging  heat.  The  lengths 
must  not  be  broken  off  cold,  as  this  tends  to  cause  cracks  in  the 
bars.  For  forging,  the  steel  should  be  placed  in  the  fire,  and 
slowly  but  thoroughly  heated,  taking  care  that  the  heating  has 
penetrated  to  the  centre  of  the  bar,  and  then  forged  at  a  bright 
red  heat.  Whilst  forging,  the  bar  should  not  be  allowed  to  get 
lower  than  a  good  red.  After  the  tool  is  forged  it  should  be  laid 
down  in  a  dry  place  and  allowed  to  cool  slowly.  To  harden  the 
tool,  the  nose  only  should  be  raised  to  a  white  melting  heat  and 
then  cooled  with  an  air  blast. 

To  obtain  the  maximum  efficiency  from  this  "  A.  W.  "  steel  it 
is  essential  that  the  nose  of  the  tool  shall  be  raised  to  a  white 
melting  heat  as  described,  for  if  during  this  heating  the  point 
of  the  nose  becomes  fused  or  melted,  no  harm  whatever  has  been 
done.  The  tool  is  then  ready  for  use  after  grinding  on  a  wet 
stone. 

Another  method  which  may  be  described  of  preparing  the  tools 
is  as  follows : 

Forge  the  tool  as  before,  and  when  cold  roughly  grind  to  shape 
on  a  dry  stone  or  dry  emery  wheel.  The  tool  then  requires 
heating  to  a  white  heat,  just  short  of  melting,  and  cooling  in  the 
air  blast.  This  method  also  lends  itself  for  cooling  the  tools  in 
oil;  before  which  the  temperature  has  to  be  lowered  from  the 
white  heat  to  a  good  red  heat  (about  1600°  F.)  either  by  the  air 
blast  or  in  the  open,  and  the  tool  then  quenched  in  oil.  Tools 
hardened  by  the  latter  method  are  specially  good  where  the 
retention  of  a  sharp  edge  is  a  desideratum,  as  in  finishing  tools, 
capstan  and  automatic  lathe  tools,  brass  workers'  tools,  <fec. 
Nothing  has  yet  been  found  to  be  so  good  for  air  hardening 
steels  as  the  wet  sandstone. 

Specific  Gravity  Test. — Where  engineering  firms  buy  large 
quantities  of  various  steels,  it  is  advisable  to  have  samples  cut  off, 
to  ascertain  the  various  specific  gravities  and  tabulate  them.  The 
specific  gravity  of  the  steel  of  any  one  particular  brand  and 
maker  is  found  to  be  fairly  constant.  Consequently  the  firm  will 
be  able  to  identify  from  amongst  any  number  the  manufacturer  of 
the  steel  which  they  find  by  trial  to  be  most  suitable  for  their 
purpose  by  this  method. 


204  LECTURE  XVI. — QUESTIONS. 


LECTURE  XVI.— QUESTIONS. 

1.  Sketch  the  fast  headstock  of  a  double-geared  lathe,  and  explain  the 
contrivance  for  increasing  or  diminishing  the  speed  of  the  mandril.     In 
the  headstock  of  a  lathe  a  pinion  of  20  teeth  drives  a  wheel  of  60,  and  a 
second  pinion  of  20  drives  another  wheel  of  60;  compare  the  rates  of 
rotation   of   the   first  driving  pinion  and  of  the  mandril  of  the  lathe. 
Ans.  9  :  i. 

2.  Why  is  a  lathe  often  back-geared  ?     Sketch  a  section  through  the 
headstock  showing  the  arrangement.     If  the  two  wheels  have  63  and  63 
teeth  respectively,  and  each  pinion  has  25  teeth,  find  the  reduction  in  the 
velocity  ratio  of  the  lathe  epindle  due  to  the  back-gear.     Ans.  6*35  :  i. 

3.  Make  a  vertical  longitudinal  section  through  the  movable  or  loose 
headstock  of  a  lathe,  showing  precisely  the  manner  in  which  a  screw  anc 
nut  are  applied  to  produce  the  necessary  movement  of  the  centre  which 
supports  the  work.     Name  the  materials  of  which  the  several  parts  are 
made. 

4.  What  is  the  use  of  the  guide-screw  in  a  lathe  ?    Where  is  ifc  usually 
placed  ?    Show  by  sketches  the  precise  manner  in  which  the  slide  rest  is 
connected  with  or  disengaged  from  the  guide-screw. 

5.  Describe  and  show  by  sketches  the  means  by  which  the  slide  rest  of 
a  lathe  may  be   connected   with  the  leading  screw.     If  the  slide  rest 
traverses  the  bed  at  the  rate  of  i^  feet  when  the  leading  screw  makes  56 
revolutions,  what  is  the  pitch  of  the  screw  thread  1    Ans.  %  inch. 

6.  Sketch  and  describe  the  mechanism  by  which  the  saddle  of  a  screw- 
cutting  lathe  can  be  made  to  travel  automatically  in  either  direction  along 
the  lathe  bed  while  the  speed  pulleys  run  always  in  the  same  direction. 

7.  How   is   the   copying  principle  applied   in   a    screw-cutting   lathe  ? 
Describe  a  method  of  throwing  a  self-acting  screw-cutting  lathe  in  and 
out  of  gear,  and  of  reversing  it  by  means  of  a  belt  and  overhead  pulleys. 
(See  Fig.  5  in  Lecture  XI.) 

8.  Explain  the  use  of  the  quadrant  for  change  wheels  in  a  screw- cutting 
lathe  by  making  a  sketch  showing  it  in  its  position  on  a  lathe  with  the 
wheels  in  gear.     (See  the  general  and  the  end  views  of  the  6"  screw 
cutting-lathe  bed,  and  "  Index  to  Parts  "  for  the  part  marked  CP.) 

9.  Explain  the  mode  in  which  change  wheels  are  employed  in  a  screw- 
cutting  lathe.     The  leading  screw  being  of  J-inch  pitch,  give  a  sketch 
of  the  arrangement  of  change  wheels  required  for  cutting  a  screw  of  15 
threads  to  the  inch,  marking  the  numbers  oE  the  teeth  on  each  wheel. 

10.  Sketch  and  describe  the  mechanism  for  cutting  a  screw  with  five 
threads  to  the  inch  in  a  lathe  where  the  guide  screw  has  three  threads 
to  the  inch.    Assign  suitable  numbers  to  the  wheels  which  you  would 
employ. 

11.  The  leading  screw   of  a  lathe  is  |-inch   pitch  and  right-handed. 
Sketch  and  describe  the  arrangement  whereby  you  would  employ  the  lathe 
for  cutting  a  screw  of  £-inch  pitch,  and  left-handed^. 

12.  Describe  the  operation  of  cutting  a  screw  in  a  lathe,  showing  the 
wheels  required,  and  how  they  are  placed  to  cut  a  right-handed  screw 
with  eight  threads  to  the  inch  in  a  lathe  whose  leading  screw  is  of  J-inch 
pitch. 


LECTURE  XVI. — QUESTIONS. 

13.  Explain  the  use  of  change  wheels  in  a  screw-cutting  lathe.    It  is 
desired  to  cut  a  screw  of  f-inch  pitch  in  a  lathe  with  a  leading  screw  of 
four  threads  to  the  inch,  using  four  wheels.    If  both  screws  be  right- 
handed,  what  wheels  would  you  employ  ? 

14.  The  leading  screw  in  a  self-acting  lathe  has  a  pitch  of  £  inch  ;  show 
an  arrangement  of  change  wheels  for  cutting. a  screw  of  f-inch  pitch. 

15.  You  are  required  to  cat  a  left-handed  screw  of  five  threads  to  the 
inch  in  a  lathe  fitted  with  a  right-handed  guide  screw  of  $-inch  pitch. 
Show  clearly  by  the  aid  of  sketches  the  change  wheels  which  you  would 
employ  for  the  purpose,  indicating  how  they  would  be  respectively  carried, 
and  the  number  of  teeth  in  each  wheel. 

1 6  What  do  you  understand  by  a  single-geared,  a  double-geared,  and  a 
treble-geared  lathe  ?  Give  such  sketches  as  will  show  clearly  the  arrange- 
ment of  the  headstock  in  each  of  these  cases. 

17.  Given  a  screw-cutting  lathe  with  aright-handed  leading  screw  with 
four  threads  per  inch — sketchan  arrangement  for  cuttinga  left-hand  thread 
of  eleven  threads  per  inch.     What  gear  wheels  would  be  required  ? 

1 8.  A  driving  shaft  runs  at  100  revolutions  per  minute,  and  carries  a 
pulley  22  inches  in  diameter  from  which  a  belt  communicates  motion  to  a 
pulley  12  inches  in    diameter  carried   upon  a  counter-shaft.     On   the 
counter-shaft  is  also  a  cone  pulley  having  steps,  8,  6,  and  4  inches  in 
diameter  respectively,  which  gives  motien  to  another  cone  pulley  with 
corresponding  steps  on  a  lathe  spindle.     Sketch  the  arrangement  in  front 
and  end  elevation,  and  find  the  greatest  and  least  speeds  at  which  the 
lathe  spindle  can  revolve.     Ans.  366'6  revs,  per  minute,  and  91 '6  revs,  per 
minute. 

19.  Describe  and  sketch  the  arrangement  of  the  mechanism  by  which 
the  saddle  of  a  lathe  is  traversed  by  hand  along  the  bed. 

If  the  slide  rest  of  a  screw-cutting  lathe  when  in  gear  with  the  leading 
screw  moves  along  the  bed  fora  distance  of  14",  while  the  leading  screw 
makes  56  revolutions,  what  must  be  the  pitch  of  the  thread  on  the  leading 
screw  1  Ans.  £  inch. 

20.  Give  free-hand  sketches  of  the  front  and  back  outside  views  of  the 
Hexagon  Turret  Lathe  described  in  this  Lecture  with  index  to  parts. 

21.  Give  a  sectional  plan  with  index  to  parts  of  the  electrically-driven 
Hexagon  Turret  Lathe,  showing  the  detailed  arrangement  of  motor  and 
gears  with  friction  clutches  for  the  fixed  headstock. 

22.  Give  a  plan  with  front  and  end  elevations,  together  with  an  index  to 
parts,  showing  the  general  arrangement  of   Saddle  and  Turret  for  the 
Hexagon  Lathe. 

23.  Give  a  plan  with  front  and  end  elevations  of  the  general  arrange- 
ment of  the  apron  for  the  Hexagon  Turret  Lathe.     Indicate  each  part  by 
letters  and  an  index. 

24.  Give  a  front  and  end  elevation  with  index  to  parts  of  the  general 
arrangement  of  the  feed  motion  of  the  Hexagon  Turret  Lathe. 

25.  What  is  meant  by  "  rapid  turning  high  speed  tool  steel "  1    Indicate 
the  variations  of  cutting  speed  with  area  of  cut-steel  and  cast-iron  by 
diagrams  plotted  on  squared  paper  with  an  example  of  the  h.-p.  required 
under  certain  conditions. 

26.  Describe  the  process  of  forging  and  hardening  rapid-cutting  tool 
steel,  and  state  how  it  differs  from  ordinary  tool  steeL 


2O6  NOTES    AND    QUESTIONS. 

27.  Describe,  with  sketches,  the  mechanism  for  giving  an  automatic 
feed  to  the  cutting  tool  of  a  lathe  or  shaping  machine,  and  how  it  is  put 
in  or  out  of  action,  and  the  amount  of  feed  varied.  (B.  of  E.,  1902.) 

28.  On  the  headstock  spindle  of  a  lathe  is   keyed  a  speed  cone,  the 
greatest  and  least  diameters  of  which  are  10  ins.  and  5^  ins.  respectively. 
It  is  driven  from  a  similar  speed  cone  keyed  to  a  counter-shaft  which  makes 
350  turns  a  minute.  The  back  gearing  is  of  the  usual  type,  the  spur-wheels 
concentric  with  the  headstock  spindle  having  62  and  30  teeth,  gearing  with 
wheels  having  18  and  50  teeth  respectively  on  the  back  spindle.     Sketch 
and  describe  the  arrangement,  and  find  the  greatest  and  least  speeds  at 
which  the  headstock  spindle  can  run.  (C.  &  G.,  1903,  0.,  Sec.  A.) 

Ans.  Without  gearing:  6j6'$6  revs,  per  minute,  and    I92'5   revs,   per 

minute. 
With  gearing  :  1107  revs,  per  minute,  and  33*5  revs,  per  minute. 

29.  The  gearing  of  a  capstan  engine  is  arranged  as  follows  :  Fixed  on  the 
crank  shaft  is  a  double-threaded  worm,  which  gears  with  a  worm-wheel  of 
50  teeth  ;  keyed  to  the  worm-wheel  shaft  is  a  spur-wheel  (A)  of  22^  ins. 
diameter  pitch  circle,  and  very  approximately  3^-in.  pitch,  driving  another 
wheel  of  40  teeth,  which  is  keyed  to  the   same  shaft  as  the  holder  round 
which  the  cable  passes.    Find  the  number  of  teeth  in  the  \\  heel  A ;  and  if 
the  effective  diameter  of  the  cable-holder  is  24  in^.,  find  the  number  of 
revolutions  the  engine  must  make  to  heave  in  90  ft.  of  cable. 

(C.  &  G.,  1904,  O.,  Sec.  A.) 

Ans,   Nnmber  of  teeth  on  wheel  A  =  20.     Revs,  of  engine  required  to 
heave  in  90  ft.  of  cable  =  715. 

30.  It  is  required  to  transmit  a  velocity  ratio  of  80  to   I  by  a  train  of 
toothed  wheels  ;  no  pinion  of  this  train  of  wheels  is  to  have  less  than 
16  teeth,   and  no  wheel  is  to  have   more  than  90  teeth.     Determine   a 
suitable  train  of  wheels,  and  the  number  of  teeth  in  each  wheel. 

(B  of  E.,  1905,) 
.  _64x8ox64_8o 

!~l6xi6xi6~T* 

31.  In  the  gear  for  an  electrically-driven  turret,   the  motor  shaft   is 
provided  with  a  single-threaded  worm  gearing  with  a  worm- wheel  of  about 
23^  ins.  diameter,   and  i^-in.  pitch,  keyed  to  a  spindle,  which  carries  a 
wheel  of  12  teeth,  which  gears  with  a  wheel  of  .about  30^  ins.  diameter 
and  4-in.  pitch.     Keyed  to  the  spindle  of  the  latter  is  a  spur-wheel  of 
15  ins.  diameter,  gearing  with  a  circular  rack  on  the  turret,  and  which  is 
15  ft.  diameter.     It  is  found  that  the  shortest  and  longest  times  to  turn 
the  turret  through  270°  are  52  seconds  and  21  minutes  respectively.     Find 
the  corresponding  revolutions  per  minute  of  the  motor. 

(C.  &  G.,  1905,  0.,  Sec,  A.) 
Ans.  Number  of  teeth  on  worm-wheel  =  49  ; 
,,         ,,         ,,  spur-wheel    =24; 

Revs,  of  motor  per  minute.  1018  and  42. 


(         207        ) 


LECTURE  XYII 

CONTENTS. — Hydraulics — Definition  of  a  Liquid— Axioms  relating  to  a 
Liquid  at  Rest— Transmission  of  Pressure  by  Liquids— Pascal's  Law 
— "Head"  or  Pressure  of  a  Liquid  at  Different  Depths — Total  Pres- 
sure on  a  Horizontal  Plane  immersed  in  a  Liquid — Lord  Kelvin's 
Wire-testing  Machine— Total  Pressure  on  any  Surface  immersed  in  a 
Liquid — Examples  I.  II.— Questions. 

Hydraulics. — Hitherto  the  student's  attention  has  been  con- 
fined to  solid  bodies,  which  were  supposed  to  remain  perfectly 
rigid  and  unchanged  when  acted  upon  by  forces.  We  shall  now 
direct  his  consideration  to  the  properties  and  applications  of 
another  great  division  of  matter — viz.,  liquids — which  possess  the 
marked  opposite  character  of  mobility  under  the  action  of  forces. 
In  nature  we  do  not  meet  with  either  perfectly  solid  or  perfectly 
liquid  bodies;  and  consequently  the  practical  engineer,  when 
applying  the  formulae  of  the  physicist  to  his  machines  and 
hydraulic  works,  has  to  make  certain  allowances  according  to 
circumstances,  with  the  aid  of  constants  predetermined  by  experi- 
ence and  experiment. 

The  most  common  and  the  most  useful  liquid  with  which  the 
engineer  has  to  deal  is  that  of  water.  Hence  the  term  "  hydraulic 
engineer,"  as  applied  to  persons  who  direct  and  guide  the  action 
of  waters,  as  in  the  case  of  the  water  supply  for  a  town,  or  for 
navigation  purposes,  or  for  the  transmission  of  force  and  power. 
The  term  hydraulics,  therefore,  comprehends  hydro- statics y  which 
is  the  science  of  liquids  in  equilibrium^  and  hydro-kinetics,  the 
science  of  liquids  in  motion.  We  shall  only  have  space  in  this 
manual  for  an  elementary  inquiry  into  the  former  of  these  two 
divisions  of  hydraulics.* 

Definition  of  a  Liquid. — A  liquid  is  a  collection  of  particles 
which  are  perfectly  movable  about  each  other.  In  consequence  of 
this  property,  a  liquid  requires  some  external  force  or  resistance 
to  keep  its  particles  together,  such  as  the  sides  of  a  vessel ;  for  its 
molecules  can  be  displaced  by  the  smallest  force,  and  are  readily 
divided  from  each  other  in  any  direction.f 

*  For  Viscous  Fluids,  see  p.  224. 

t  The  late  Prof.  Clerk  Maxwell  distinguished  solids  from  liquid*  in  the 
following  manner: — "Bodies  which  can  sustain  longitudinal  pressure, 

o 


208 


LECTURE    XVII. 


a  v2ss 


Axioms  relating  to  a  Liquid  at  Rest. — It  follows  directly 
from  the  above  definition,  that  when  equilibrium  exists — 

(1)  l\efree  surface  of  a  liquid  at  rest  is  horizontal ; 

(2)  Any  surface  of  a  liquid  at  rest  is  everywhere  perpendicular  to 
the  force  which  acts  upon  it  ; 

(3)  A  liquid  at  rest  acted  on  by  a  force  presents  a  surface  whijh 
is  everywhere  perpendicular  to  the  direction  of  the  force  ; 

(4)  A  surface  supporting  a  liquid  at  rest  reacts  everywhere  per- 
pendicularly to  the  pressure  of  the  liquid  ; 

(5)  In  all  cases  of  pressure  on  or  from  liquids  at  rest,  action  and 
reaction  are  equal  and  opposite. 

If  such  were  not  the  ..case,  equilibrium  could  not  exist,  and 
motion  of  the  liquid  would  take  place. 

Transmission  of  Pressure  by  Liquids. — Take  a  tight  vessel 

filled  with  a  liquid  and  fitted 
with  four  f rictionless  piston- valves, 
Y15  Y2,  V3,  Y4,  of  the  same  area. 
Let  the  outward  pressure  on  these 
valves  be  balanced  by  spiral 
springs,  arranged  so  that  they 
indicate  the  forces  applied  to 
them.  Now  apply  an  inward 
force  of,  say,  i  or  5  or  10  Ibs.  to 
the  spiral  spring  of  valve  Yx,  then 
instantly  the  other  three  springs 
TEANSMISSION  OF  PEESSUEE  of  the  other  valves  will  register 
BY  LIQUIDS.  an  outward  pressure  of  the  same 

(Horizontal  Section.)  amount  as  that  applied.      If  the 

other  valves  had  been  of  different  areas  from  valve  Yp  their 
springs  would  have  registered  pressures  corresponding  with  the 
ratio  of  their  areas  to  the  area  of  valve  Yr  Or  the  pressure  per 
square  inch  on  valve  Yx  is  communicated  throughout  the  liquid 
to  the  other  valves,  and  to  every  square  inch  of  the  internal  sur- 
face of  the  vessel,  with  undiminished  effect. 

Pascal's  Law. — Fluids  transmit  pressure  equally  and  in  all 
directions*  In  the  case  of  solids  pressure  is  only  transmitted 

"however  small  that  pressure  may  be,  without  being  supported  by  lateral 
pressure,  are  called  solids,  and  those  which  cannot  are  termed  liquids."  A 
perfect  liquid  is  therefore  one  in  which  there  is  absolutely  no  resistance  to  a 
change  of  shape,  although  there  may  be  practically  an  infinite  resistance  to 
change  of  volume.  We  say  practically  because,  although  liquids  are  more 
or  less  compressible  to  a  very  small  extent,  yet  the  amount  is  so  small  as 
to  be  negligible  in  the  case  of  most  engineering  problems. 

*  Here  the  word  fluid  hag  been  used  instead  of  liquid,  as  being  more 
general,  since  the  term  fluid  includes  both  liquids  and  gases.  Refer  to 
p.  2,  Lecture  I.,  for  the  distinction  between  a  liquid  and  a  gas, 


PRESSURES   DUE  TO   LIQUIDS.  2  09 

along  the  line  of  its  action,  and  therefore  we  have  in  this  law  an 
exemplification  of  the  fundamental  distinction  between  solids  an<S 
fluids.  In  Lecture  J1X.  we  will  explain  several  machines  that 
depend  upon  the  principle  enunciated  by  Pascal's  law  for  their 
action. 

Head  or  Pressure  of  a  Liquid  at  Different  Depths. — 
Imagine  a  very  small  horizontal  area,  a  (for  instance,  a  square 
inch),  situated  at  a  depth  or  height,  h,  inches  from  the  free  surface 
of  a  liquid,  and  that  the  vertical  column  from,  a,  to  the  surface 
becomes  solidified  -without  in  any  way  disturbing  equilibrium;  It 
is  evident  that  the  horizontal  and  the  vertical  forces  on  the  solid 
column  must  be  separately  in  equilibrium,  otherwise  motion  would 
ensue.  But  the  only  vertical  forces  are  the  weight  of  the  column 
downward  and  the  pressure  of  the  surrounding  liquid  upwards 
on  the  base,  a.  Therefore, 

The  pressure  upwards  =  weight  of  the  prism. 

Or,  .         .        */*      p  =  haw. 

Where,  «?,  is  the  weight  of  every  inch  of  its  height  or  the  weight 
of  a  cubic  inch  of  the  column.  But  the  area,  a,  and  the  weight,  10, 
are  constant  quantities  for  any  particular  unit  of  area  and  kind 
of  liquid.  Hence — 

Pressure  varies  directly  as  the  depth  from  the  free  surface. 

Or,  p  ec  h. 

The  technical  term  "  head  "  expresses  the  above  fact  in  a  single 
word.  For,  when  speaking  of  the  working  pressure  per  square 
inch  due  to  a  supply  of  water  for  a  mill  wheel  or  turbine,  we  say 
it  has  i o  or  20  or  30  feet  of  head,  meaning  thereby  the  pressure 
due  to  a  difference  of  level  of  so  many  feet,  from  the  free  surface 
of  the  water  as  it  enters  the  supply  pipe  to  the  free  surface  of 
the  tail  race  or  discharge  pipe.  Since  every  foot  of  "  head  "  of 
water  gives  in  round  numbers  a  pressure  of  i  Ib.  per  square  inch, 
we  might  have  said  that  the  pressure  was  5  or  10  or  15  Ibs 
respectively  per  square  inch.  Consequently, 

Pressure  varies  directly  as  the  head. 

Total  Pressure  on  a  Horizontal  Plane  immersed  in  a 
Liquid. — Take  a  vessel  of  any  shape  having  a  horizontal  base, 
and  fill  it  with  a  liquid  to  any  known  height.  Then  from  the 
above  rule  it  follows  that, 

height  in  inches  from  base  to  sur- 


The  Total  Pressure  on  the  lose  =  . 


face  x  area  of  base  in  square 


inches  x  weight  of  a  cubic 
inch  of  the  liquid. 
For,  pressure  per  sq.  in.,  p  =  hawt  when,  a=  i  square  inch. 


210 


LECTUEE   XVII. 


Consequently,  if  the  total  area  of  the  horizontal  plane  be  equal 
to,  a,  square  inches,  instead  of  i  square  inch. 

The  Total  Pressure  =  haw. 

This  shows  that  the  shape  of  the  vessel  containing  the  liquid,  and 
the  total  weight  of  water  in  the  vessel,  do  not  in  any  way  affect  the 
total  pressure  on  the  base.  For,  it  depends  solely  on  the  difference  of 
level  between  the  base  (or  immersed  plane)  and  the  free  surface,  on 
the  area  immersed,  and  on  the  weight  per  unit  volume  or  specific 
gravity  of  the  liquid. 

This  property  results  in  what  used  to  be  termed  the  hydrostatic 
paradox,  which  is  very  well  illustrated  by  Lord  Kelvin's  apparatus 
for  testing  the  tensile  strength  and  percentage  elongation  of  the 
sheathing  wires  used  for  covering  and  protecting  the  insulated 
conductors  of  submarine  cables. 

Lord  Kelvin's  Wire-testing  Machine,  or  Hydrostatic 
Paradox. — W  represents  the  wire  to  be  tested,  which  is 
fixed  to  the  clips  04  Cr  HB  is  a  circular  hydrostatic  bellows, 

3'  diameter,  with  india-rubber 
sides.  WD2  is  the  bottom  wooden 
disc  attached  by  bolts  to  an  iron 
tripod  T,  which  is  connected  at  its 
centre  to  the  clip  C2 ;  while  WDX  is 
an  upper  wooden  disc  rigidly  fixed 
to  the  wooden  framing  WF.  H  is 
a  handle  keyed  to  the  screwed 
spindle  S.  HS  is  a  hydrostatic 
scale,  fixed  behind  the  vertical  glass 
tube  which  is  fitted  into  a  short 
brass  cylinder  passing  through 
WDt  and  into  HB.  ES  is  the  scale 
for  measuring  the  percentage  elon- 
gation. The  upper  end  of  this  scale 
is  fixed  to  the  wire  W,  and  the 
lower  end  is  free.  There  is  a  clip 
pointer  P  which  is  affixed  to 

each  wire  before  testing  it,  and   moved  up  or  down  until  it  is 
opposite  to  the  zero  of  the  scale  ES. 

Method  of  Testing  Wire  by  this  Machine. — (i)  Turn  the  handle 
H  backwards  until  Ct  is  as  far  down  as  it  can  get.  (2)  Fix  wire 
in  clips,  and  attach  the  pointer  P  so  as  to  be  opposite  the  zero  of 
scale  ES.  (3)  Turn  the  handle  H  forward,  thus  lifting  WD3,  and 
stretching  the  wire,  by  forcing  water  up  the  glass  tube  in  front  of 
HS.  This  gives  the  necessary  "  head,"  A,  or  pressure  due  to  the 
difference  in  level  between  the  free  surface  in  the  glass  tube  and 
the  bottom  of  the  wooden  base  WB2.  The  area  in  square  inches 


THOMSON'S  HYDROSTATIC 
WiEE-TESTING  MACHINE. 


TOTAL   PRESSURE    DUE   TO   LIQUIDS.  211 

of  this  base  gives,  #,  and  hence  the  total  pull  on  the  wire  is  =  haw. 
(4)  Note  the  elongation  by  the  scale  ES,  and  the  total  tensile  stress 
by  the  scale  HS,  at  the  moment  the  wire  breaks.  WDa  falls  upon 
stops,  so  as  not  to  injure  the  india-rubber  hydrostatic  bellows 
HB. 

This  machine  was  used  in  1872-73  by  the  Author  and  others 
in  testing  all  the  sheathing  wire  for  the  Western  and  Brazilian 
Company's  cables.  The  homogeneous  wire  gave  an  average  of 
55  tons  per  square  inch. 

In  this  machine  we  see  that,  owing  to  the  quaqua  versus  principle 
enunciated  above  a  few  pounds  weight  of  water  can  produce  a 
stress  of  many  hundreds  or  even  thousands  of  pounds  by  simply 
giving  it  "head"  through  a  small  tube  in  connection  with  an 
enlarged  area. 

When  the  sides  of  a  vessel  taper  towards  the  top,  as  in  the  case 
of  a  wine  bottle,  the  liquid  pressing  vertically  upwards  upon  them 
produces  a  reaction  on  the  base,  which  makes  up  for  the  want  of 
weight  of  liquid  which  would  be  naturally  due  to  direct  vertical 
pressure  in  the  case  of  a  cylindrical  vessel. 

Total  Pressure  on  any  Surface  immersed  in  a  Liquid. 
— Let  a  surface  of  any  shape  be  immersed  in  a  liquid  of  any  kind 
to  any  depth,  as  illustrated  by  the  following  figures.  Then,  by 
applying  the  previous  proofs,  and  a  property  of  the  "centre  of 


END  VIEW.  SIDE  VIEW. 

PEESSURE  ON  ANY  SURFACE  IMMERSED  IN  A  LIQUID. 
gravity  "  (which  affirms  that  the  mean  perpendicular  distance  from 
any  plane,  is  equal  to  the  distance  from  the  e.g.  of  the  surface  to  that 
plane),  we  find,  that  the  total  pressure  on  the  immersed  surface  is 
represented  by  the  following  equation  : 
P  =  HAW.* 
Where   P  =  Pressure  (total)  in  Ibs. 

„       H  =  Height  from  e.g.  to  free  surface  in  feet.* 
,,       A  =  Area  in  square  feet.* 
„      W  =  Weight  of  a  cubic  foot  of  the  liquid.* 
*  The  student  will  observe  that  we  have  suddenly  jumped  from  height* 


212 


LECTURE   XVII. 


==^^    3ft. 


EXAMPLE  I. — Find  the  total  pressure  on  the  bottom  of  a 
cubical  tank  having  a  bottom  4'  x  4'  and  filled  with  water  to  a 
depth  of  4'. 

ANSWER. — By  the  above  formula — 
P  =  HAW. 
P  =  4'  x  (4'  x  4')  x  62-5  Ibs.  =  4000  Ibs.  =  1-8  tons. 

. — We  may  here  remark  that  62*3  Ibs.  is  the  weight  of  a  cubic  foot 
of  fresh  water  at  65°  F  (see  Appendix  for  Useful 
Constants),  whereas  62-5  Ibs.  is  the  value  at  the 
maximum  density  of  water,  or  39°  F. 

EXAMPLE  II. — A  rectangular  tank  for 
holding  water   has  a  vertical  side   whose 
dimensions  are  3  feet  vertical  by   4  feet 
^P3j_^  horizontal.     An  open  pipe  is  inserted  into 

I  £,  |§  the  cover  of  the  tank,  and  water  is  poured 

JL'<>  iiF  in  until  the  level  in  the   pipe   is  7  feet 

^  ' '  above  the  base  of  the   tank.      Find   the 

pressure  on  the  vertical  side  and  the  reduc- 
tion of  pressure  when  the  water  in  the 
pipe  is  allowed  to  sink  ij  feet.  (The 
weight  of  a  cubic  foot  of  water  =  62.5  Ibs.) 
(S.  and  A.  Exam.  1890.) 

ANSWER. — In  the  first  case, 
Height  from  e.g.  of  side  to  free  surface  =  Ht  =  5.5'. 
Area  of  this  vertical  side  in  sq.  ft.          =  A   =  3'  x  4"=  12  sq.  ft. 
Weight  of  a  cubic  foot  of  water  =  W  =62.5  Ibs. 

By  the  above  formula, 
The  total  pressure  P^H^AW. 

.-.?!  =  5.5' x  12x62.5  =  4125  Ibs. 

In  the  second  case,  when  the  free  surface  is  lowered  by  i  J  ft., 
everything  remains  the  same  except  the  H,  which  is  now  reduced 
from  Hj  to  H2  =  4'. 
By  the  formula, 

P3  =  H2AW. 

.'.  P2  =  4X  12  x  62.5  =  3000  Ibs. 

Consequently,  the  reduction  in  pressure  is  the  difference  between 
these  pressures. 

Or  (Pt  -  P2)  -  4125  Ibs.  -  3000  Ibs.  -1125  Ibs. 

in  inches  to  those  in  feet,  areas  in  square  inches  to  those  in  square  feet, 
and  weights  of  cubic  inches  to  those  of  cubic  feet.  This  is  because  the 
usual  units  of  measurement  in  hydraulics  are  feet,  square  feet,  and  cubic 
feet.  Before  attempting  the  more  difficult  questions  on  page  213,  he 
ihould  study  a  few  pages  of  the  next  Lecture. 


LECTURE   AVJLL.  —  QUESTIONS,  2 1 3 


LECTURE  XVII.— QUESTIONS. 

1.  Define  the  terms  liquid,  hydro-statics,  hydro-dynamics,  and  hydraulics. 

2.  Give  the  chief  properties  of  a  liquid,  stating  wherein  it  differs  from  a 
solid  and  a  gas. 

3.  Describe  and  illustrate  any  experiment,  other  than  the  one  referred  to 
in  this  Lecture,  to  prove  the  law  of  transmission  of  pressure  by  liquids. 
State  Pascal's  law. 

4.  Describe  the  nature  of  fluid  pressure.    A  mass  of  stone  when  in  water 
appears  to  be  lighter  than  when  it  is  situated  in  the  open  air.     Will  you 
explain  the  cause  of  this  fact,  and  state  the  difference  of  weight  per  cubic 
foot  of  water  displaced  ? 

5.  What  is  meant  by  "  head"  in  relation  to  water  supplies  for  developing 
power  t    Give  an  example. 

6.  Explain  how  the  pressure  on  the  base  of  a  vessel  depends  solely  upon 
the  area  of  the  base  and  its  depth  from  the  free  surface.     Illustrate  your 
remarks  by  showing  a  series  of  connected  vessels  of  very  different  shapes, 
but  with  each  of  their  bases  of  the  same  size  and  on  the  same  level,  and 
filled  with  water  to  the  same  height. 

7.  Sketch  and  describe  Sir  Wm.  Thomson's  wire-testing  machine,  and 
explain  how  such  a  great  force  is  obtained  thereby  from  such  a  small 
quantity  of  water. 

8.  How  is  the  pressure  of  water  on  a  given  area  ascertained  ?    A  tank, 
in  the  form  of  a  cubical  box,  whose  sides  are  vertical,  holds  4  tons  of  water 
when  quite  full ;  what  is  the  pressure  on  its  base,  and  what  is  the  pressure 
on  one  of  its  sides  ?    Ans.  4  tons  ;  2  tons. 

9.  A  water  tank  is  13  feet  square  and  4  feet  6  inches  deep  ;  find  the  pres- 
sure upon  one  of  the  sides  when  the  tank  is  full.    Ans.  8226-56  Ibs. 

10.  State  approximately  the  increase  of  pressure  to  which  a  diver  would 
be  exposed  when  working  at  a  depth  of  50  feet  below  the  surface  of  fresh 
water.     Ans.  About  22  Ibs.  per  square  inch. 

11.  In  the  vertical  plane  side  of  a  tank  holding  water,  there  is  a  rectan- 
gular plate  whose  depth  is  I  foot  and  breadth  2  feet,  the  upper  edge  being 
horizontal,  and  8  feet  below  the  surface  of  the  water  ;  find  the  pressure  on 
the  plate.     Ans.  1062-5  Ibs. 

12.  The  base  of  a  rectangular  tank  for  holding  water  is  a  square,  16 
square  feet  in  area.     The  sides  of  the  tank  are  vertical,  and  it  holds  250 
gallons  of  water  when  quite  full.     Find  the  depth  of  the  tank  and  th« 
pressures  on  each  side  and  on  the  base  when  quite  filled  with  water. 

Am.  2-5  feet ;  781-25  Ibs.  ;  2500  Ibs. 

13.  A  rectangular  tank  for  holding  water  has  a  vertical  side  whose 
dimensions  are  4  feet  vertical  by  5  feet  horizontal.     An  open  pipe  is  in- 
serted into  the  cover  of  the  tank,  and  water  is  poured  in  until  the  level  in 
the  pipe  is  10  feet  above  the  base  of  the  tank.    Find  the  pressure  on  the 
vertical  side  and  the  reduction  of  pressure  when  the  water  in  the  pipe  is 
allowed  to  sink  2  feet.     Ans.  10,000  Ibs.  ;  2500  Ibs. 

14.  A  gauge  in  a  water  pipe  indicates  a  pressure  of  water  equal  to  40  Ibs. 
on  the  square  inch.      What  is  the  depth  of  the  point  below  the  free 
surface  ?    Sketch  and  explain  the  action  of  some  form  of  gauge  suitable 
for  the  above  purpose.         Ant.  92*16  ft. 


FRESH 


LECTURE  XVIII. 

CONTENTS. — Useful  Data  regarding  Fresh  and  Salt  Water — Examples 
I.  II.  III.  IV.— Centre  of  Pressure— Immersion  of  Solids— Law  of 
Archimedes — Floating  Bodies — Example  V. — Atmospheric  Pressure — 
The  Mercurial  Barometer — Example  VI.— Low  Pressure  and  Vacuum 
Water  Gauges— Example  VII. — The  Siphon — Distinction  between 
Solids,  Liquids  and  Gases — Definitions  of  perfect,  viscous,  and  elastic 
Fluids — Cohesion — Questions. 

tTseful  Data  regarding  Fresh,  and  Salt  Water. — We  will 
commence  this  Lecture  by  giving  some  useful  data  regarding  the 
weights,  <fcc.,  of  fresh  and  salt  water,  and  then  work  out  a  few 
more  examples  for  the  pressures  on  immersed  surfaces,  finishing 
with  the  immersion  of  solids  in  fluids,  &c. 

Specific  gravity  *  =  i . 

cubic  foot  weighs  62-5  Ibs.,  or  1000  oz. 

gallon  weighs  10  Ibs.,  or  160  oz. 

ton  occupies  35-84  cubic  feet. 

atmosphere  =  1 4' 7  Ibs.  per  sq.  in.  =  29*92  in. 

mercury  =  33*9  (say  34)  ft.  head  of  water. 

foot  of  head  =  -43  Ib.  on  sq.  in. 

Ib.  on  the  sq.  in.  =  2-308  ft.  head. 
H.P.   in  a  waterfall  =  cubic  ft.  per  minute  x 

head  x  62-5  -f-  33,000. 

Specific  gravity  *  =  1-026. 

i  cubic  foot  weighs  64  Ibs. 

i  gallon  weighs  joj  Ibs. 

I  ton  occupies  35  cubic  ft.,  or  2iSj  gallons. 

EXAMPLE  I. — A  cubical  box  or  tank 
with  a  closed  lid,  the  length  of  a  side  of 
which  is  4  feet,  rests  with  its  base  hori- 
zontal, and  an  open  vertical  pipe  enters 
one  of  its  sides  by  an  elbow.  The  tank 
is  full  of  water,  and  the  pipe  contains 
water  to  the  height  of  i  foot  above  the 
top  of  the  tank.  What  are  the  pres 
sures  of  water  on  the  top,  bottom,  and 
sides  of  the  tank  ?  (Given  the  weight  of 
a  cubic  foot  of  water  =  62  J  Ibs.)  (S.  and 
A.  Exam.  1887.) 

*  Specific  gravity  is  the  ratio  of  the  weight  of  a  given  bulk  of  a  sub- 
8tar.ce,  to  the  weight  of  the  same  bulk  of  pure  water.  ~ 


SALT  WATER 


1 


EXAMPLES    OF   PRESSURE  OX    IMMERSED   SURFACES.      215 

ANSWER. — (i)  For  the  pressure  on  the  top — 

The  depth  of  e.g.  of  the  top  from  free  surface  =  H  =  i'. 

.• .    Total  pressure   on  top  =  HAW  =  i'  x  (4'  x  4')  x  62-5  Ibs.  = 
1000  Ibs. 

(2)  For  the  pressure  on  the  bottom — 

The  depth  of  e.g.  of  the  bottom  from  the  free  surface  =  H  =  5*. 

.•.  Total  pressure  on  bottom  =  HAW  =  5'  x  (4'  x  4')  x  62-5  Ibs.  = 
5000  Ibs. 

For  the  pressure  on  e  ich  of  the  sides — 

depth  of  e.g.  of  each  side  from  the  free  surface  =  H  =  3'. 

. • .  Total  pressure  on  each  side  =  HAW  =  3'  x  (4'  x  4')  x  62 -5  Ibs. 
=  3000  Ibs. 

EXAMPLE  II. — A  cylindrical  vessel,  30 
inches  long  and  6  inches  in  diameter,  is 
sunk  vertically  in  water,  so  that  the  base, 
which  is  horizontal,  is  at  a  depth  of  25 
inches  below  the  surface  of  the  water. 
Find  the  upward  pressure  in  pounds  on 
the  base  of  the  vessel  The  weight  of  a 
cubic  foot  of  water  is  62 1  Ibs.,  and  TT  = 
3-1416.  (S.  and  A.  Exam.  1889.) 

ANSWER. — The  depth  of  e.g.  of  the  base  from  the  free  surface 
is  =  H  =  -^  =  2j^-'=  2*083  feet. 

7T  -  2  2  , 

Ti.e  area  of  the  base  =  A  =  -«  =  ——('5  x  -5  )  =  T90  sq.  tt. 

4        7x4 

The  weight  of  a  cubic  foot  =  W  =  62*5  Ibs. 

.•.  The  total  pressure  on  base  =  HAW  =  2-083  x -196  x  62-5  = 
25-5  Ibs. 

EXAMPLE  III. — A  water  tank,  8  feet  long  and  8  feet  wide,  with 
an  inclined  base,  is  12  feet  deep  at  the  front  and  6  feet  deep  at 
the  back,  and  is  filled  with  water.  Find  the  pressure  in  Ibs.  on 
each  of  the  four  sides  and  on  the  base  ;  water  weighing  62 J  Ibs. 
per  cubic  foot. 

ANSWER. — In  answering  a  question  of  this  kind  the  student 
will  find  it  best  to  draw  a  figure  representing  the  water  tank  and 
the  positions  of  the  centres  of  gravity  of  each  side  and  of  the  base 
in  the  manner  shown  by  the  accompanying  illustration.  The  only 
point  that  presents  any  difficulty  is  the  e.g.  of  the  side  DEFC 
and  of  the  correspondingly  opposite  one.  This  might  be  done  by 
first  finding  the  e.g.  of  the  D  DEFH,  viz.,  G2 ;  second,  of  the  A 
HFC,  viz.,  G3 ;  third,  by  joining  these  two  points  with  a  line  G»G3, 


2l6 


LECTUEE   XVIII. 


and  taking  a  distance  along  it  from  G2  towards  G3  inversely  pro- 
portional to  the  areas  of  the  n  DEFH  and  the  A  HFC  ;  this 
would  give  a  point  G6  the  e.g.  of  the  whole  side  =  4*6'  from  surface. 
But  it  will  evidently  be  easier  to  treat  the  pressures  on  the  Q 
and  A  separately,  and  then  to  add  them  together  in  order  to 
obtain  the  total  pressure  on  the  whole  side  DEFC. 


/ 

4 

&. 

1  i 

SHOWING  POSITIONS  OF  THE  CENTEBS  OF  GRAVITY. 

G,  represents  centre  of  gravity  of  area  ABCD 
G2  „  „  „  DEFH 

G3  „  „  „  HFC 

G  „  „  „  ENMF 


Let  Hj,  H2,  &c.,  represent  depths  of  G1?  G2,  &c. 
Then  H!  =  JDC  =  6'  ;  H2  =  JEF  =  3'. 

G3  is  J  of  HC  below  the  line  HF  (see  Lecture  III.,  re  position  of 
e.g.  of  certain  areas). 


G5  is  at  a  d^pth  below  the  surface  =  the  mean  between  the  edges 
BC  and  FM  of  the  base  BCFM. 


Total  pressure  on  area  — 

ABCD  =  H1A1W=  6'  x  (i  2'  x  8')  x  62-5    =  36,000  Ibs. 
3/x(6/    x  8')  x  62-5  =   9,000  Ibs. 


CENTRE    OF   PRESSURE. 


2I7 


H3A3W  =  8'x(-,    x8')x62-5  =  12000  Ibs. 

DEFC  =  DEFH  +  HFC  -  9000  +  1 2000  =  21000  „ 
H4A4W  =  3'x(6'  x  8')  x  62-5  =  9000  „ 
H5A5W  =  9'x(io'x8')x62-5  =45000  „ 


EXAMPLE  IY. — A  sluice  gate 
is  4  feet  broad  and  6  feet  deep, 
and  the  water  rises  to  a  height 
of  5  feet  on  one  side  and  2 
feet  on  the  other  side.  Find 
the  pressure  in  pounds  on  the 
gate. 

ANSWER. — The  net  pressure 
on  the  sluice  gate  is  evidently 
equal  to  the  difference  of  the 
pressures  on  the  two  sides. 

Total  pressure  on — 


NET  PRESSURE  ON  SLUICE  GATE. 


Back  side  =  H,A,W  =  2*5' x  (4' x  5')  x  62.5  =  3125  IDS- 
Front  side  =  H2A3W  =  i'     x  (4'  x  2')  x  62.5  =    500    „ 

Subtracting  the  front  from  the  back)  _oftOK  Ib 
pressure  we  fret  the  net  pressure     / 


Centre  of  Pressure. — In  the  case  of  a  plane  area  immersed 
in  a  liquid,  the  "  centre  of  pressure  "  is  the  point  at  which  the  re- 
sultant of  all  the  pressures  of  the  fluid  acts.  If  the  plane  be 
horizontal,  the  resultant  naturally  acts  at  the  centre  of  the  figure, 
and  therefore  the  centre  of  pressure  agrees  with  the  centre  of 
gravity  of  the  figure.  In  the  case  of  a  vertical  rectangle,  having 
one  of  its  edges  in  the  surface  of  liquid,  like  a  dock-gate  or  a 
sluice,  the  centre  of  pressure  will  be  at  a  point  f  of  the  depth  from 
the  free  surface  and  at  the  middle  of  the  breadth  of  the  immersed 
portion.  We  will  have  to  prove  this  in  our  Advanced  Course, 
and  perhaps  refer  to  the  position  of  the  centre  of  pressure  in 
other  cases. 

Immersion  of  Solids. — Archimedes'  Discovery. — If  a  solid 
be  immersed  in  any  fluid  (whether  liquid  or  gas),  it  displaces  a 
quantity  of  that  fluid  equal  to  its  own  volume.  This  is  evident 
from  the  principle  of  impenetrability — viz.,  "  two  bodies  cannot 
occupy  the  same  space  at  the  same  time." 

Hence  we  have  a  simple  method  of  determining  the  volume  of 


2l8  LECTURE  XVIII. 

any  irregular  body  by  plunging  it  into  a  liquid,  and  noting  the 
cubic  contents  of  the  liquid  displaced,  by  letting  it  run  into  a 
measure  of  known  volume,  such  as  a  graduated  jar.  This  prin- 
ciple was  first  discovered  by  Archimedes,  a  philosopher  of  Syra- 
cuse, in  the  year  250  B.C.  The  story  of  this  discovery  is  related 
by  Yitruvius,  who  states  that  Hero,  a  king,  sent  a  certain  weight 
of  gold  to  a  goldsmith  to  be  made  into  a  crown.  Suspecting  that 
the  workman  had  kept  back  part  of  the  gold,  he  weighed  the 
crown,  but  found  that  it  was  the  same  as  the  weight  of  the  gold 
previously  sent  by  him  to  the  goldsmith.  He  was,  however,  not 
satisfied  with  this  test,  so  he  consulted  Archimedes,  and  asked 
him  whether  he  could  find  out  if  the  crown  was  adulterated. 
Not  long  afterwards  the  philosopher,  on  going  into  his  bath 
(which  happened  to  be  full  of  water),  observed  that  a  quantity  of 
the  water  was  displaced.  He  immediately  conjectured  that  the 
water  which  ran  over  must  be  equal  to  the  volume  of  the  immersed 
part  of  his  body.  He  was  so  overjoyed  at  the  discovery  that  he 
jumped  out  of  the  bath  and  ran  naked  to  the  king,  exclaiming, 
EvprjKa  I  fvpTjKa  I  (I  have  discovered  !  I  have  found  out !)  He  then 
began  to  experiment  with  the  crown  by  taking  a  quantity  of  pure 
gold  of  the  same  weight,  and  observed  its  displacement  in  water. 
Next  he  ascertained  by  the  same  process  the  volume  of  the  same 
weight  of  silver,  and  finally  the  volume  of  the  crown,  which 
actually  displaced  more  water  than  its  equivalent  weight  of  pure 
gold.  In  this  interesting  manner  the  fraud  of  the  artificer  was 
detected,  to  his  great  astonishment  and  chagrin,  and  a  Law  of 
Nature  was  discovered. 

Floating  Bodies. — A  body  is  said  to  float  in  a  fluid  when  it  is 
entirely  supported  by  the  fluid.     In  order  that  a  body  may  float, 

the  forces  acting  upon  it  must 
be  in  equilibrium.  Now,  as 
may  be  seen  from  the  case 
illustrated  by  the  accom- 
panying figure,  there  are 
only  two  forces  to  be  con- 
sidered— viz.,  the  weight  of 
the  body  acting  vertically 
downwards  through  its  centre 
of  gravity  Glf  and  the  pres- 

CONDITIONS  OF  EQUILIBRIUM  IN  THE         8Ure   °f  the  fluid  acting  VOT- 

CASE  OF  A  FLOATIN-G  BODY.  tically  upwards  through  the 

centre  of  gravity  G,  of  the 

displaced  fluid.  The  horizontal  pressures  of  the  fluid  on  the 
body  are  in  equilibrium  by  themselves,  and  simply  tend  to  com- 
press it  so  that  they  do  not  aflect  the  question.  The  upward 


FLOATING   BODIES. 


2I9 


pressure  of  the  fluid  must  be  equal  to  the  weight  of  the  body, 
otherwise  the  body  would  rise  or  sink 

Also,  the  weight  of  tne  body  must  be  equal  to  the  weight  of 
the  fluid  displaced.  This  will  be  evident  when  we  remember  that 
the  total  upward  pressure  of  the  fluid  on  the  surface  ACB  is 
equal  to  the  weight  of  the  fluid  which  formerly  filled  that  space. 
But,  since  equilibrium  still  exists  when  the  body  is  floating,  it  is 
clear  that  the  weight  01  the  body  must  also  be  equal  to  the 
weight  of  the  fluid  displaced.  If  the  body  be  wholly  immersed  it 
will  be  pressed  upwards  with  a  force  equal  to  the  weight  of  the 
fluid  which  it  displaces.  Hence  the  statement  known  as  the — 

Principle  of  Archimedes. —  When  a  body  is  wholly  or  partially 
immersed  in  a  fluid  it  is  pressed  vertically  upwards  by  the  fluid 
with  a  force  equal  to  the  weight  of  the  fluid  which  it  displaces  and 
this  force  may  be  taken  to  act  at  the  e.g.  of  the  displaced  fluid. 

As  a  natural  deduction  from  the  above  proof  we  eonclude  that 
a  body  cannot  float  in  a  liquid  of  less  specific  gravity  than  itself. 
A  solid  glass  or  metal  ball  will  float  in  mercury,  but  not  in  water. 
If  the  specific  gravity  of  a  body  be  the  same  as  that  of  a  liquid,  it 
will  float  totally  submerged.  If  the  body  and  the  liquid  are  each 
incompressible,  the  body  will  float  indifferently  at  any  depth.  If 
the  body  be  incompressible,  but  be  placed  in  a  compressible  fluid, 
such  as  air,  the  body  will  rise  or  fall  until  it  finds  a  place  where 
its  mean  specific  gravity  is  the  same  as  that  of  the  displaced  gas. 
This  is  exemplified  by  the  case  of  a  balloon  filled  with  a  gas  lighter 
than  air.  It  rises  until  it  arrives  at  a  height  from  the  earth  where 
the  combined  weight  of  the  machine  and  the  gas  contained  therein 
are  equal  to  the  weight  of  the 
same  volume  of  air.* 

EXAMPLE  V. — A  rectangular 
tank,  4  feet  square,  is  filled 
with  water  to  a  height  of  3  feet. 
A  rectangular  block  of  wood, 
weighing  125  Ibs.,  and  having 
a  sectional  area  of  4  square  feet, 
is  placed  in  the  tank,  and  floats 
with  its  sides  vertical  and  with 
this  section  horizontal  How 
much  does  the  water  rise  in  the 
tank,  and  what  is  now  the 
pressure  on  one  vertical  side 
of  the  tank?  (S.  and  A.  Exam.  1892.) 

ANSWEB. — Let  AjBx  be  the  original  surface  of  the  water  in  the 
tank  before  the  block  was  immersed,  A,B,  the  surface  after 
immersion  of  the  block. 


*  We 

Cours*. 


must  leave  the  subject  of  m«tacentres,  fee.,  to  our  Advanoad 


220  LECTURE   XVIII. 

Let  Vj  =  volume  represented  by  A^CD. 
»    V2  =      »  „       .   „    A2B2CD. 

„      z>  =       „       of  water  displaced  by  block, 

represented  by  abed. 
„      x  =  amount  by  which  the  water  rises  in  the  tank 

when  the  block  is  immersed. 
Then  clearly,  V2  =  "V\  +  v. 
Or»  V.-V^v. 

N  ow  Va  -  Vx  =  volume  represented  by  A^BjBjA,, 

=  cross  sectional  area  of  tank  x  xt 
=  42xa;=i6aj  cub.  ft. 

•••  i6x  =  v  or  x=  ^-  ft. 

16 

But,  by  the  principle  of  Archimedes  we  know  that 

The,  weight  of  water  displaced  by  block  =  The  weight  of  the  block. 

s.  =  i25  Ibs. 

v  =         =  2  cub.  ft. 


x  =  -^  =  —  -  £  ft.  =  1  J  inches. 

Next,  we  have  to  find  ^e  pressure  on  one  of  the  vertical  sides 
of  the  tank.  Here  the  depth  of  the  centre  of  gravity  of  the  area 
of  the  side  subjected  to  pressure  below  the  free  surface  of  the 
water  is 


Total  pressure  on  side  =  P  =  HAW 


Or,  ...     P  =  1220-7  Ibs. 

Atmospheric  Pressure.  —  Surrounding  the  earth's  surface 
there  is  a  deep  belt  of  air,  which  gets  rarer  and  lighter  the 
higher  we  rise  from  the  earth.  If  we  consider  the  case  of  a  com- 
plete vertical  column  of  this  air,  we  find  that  it  produces  an 
average  pressure  on  the  earth's  surface  of  about  15  Ibs.  ;  or,  in- 
other  words,  we  say  that  the  atmosphere  produces  an  average 
pressure  of  1  5  Ibs.  on  the  square  inch,  for  we  find  that  it  will 
balance  a  vertical  column  of  mercury  of  about  30  inches,  or  a 
vertical  column  of  water  of  34  feet.  We  do  not  experience  any 
inconvenience  from  this  normal  pressure  of  the  atmosphere,. 
because  we  are  so  constituted  as  to  be  able  to  resist  it.  Should  wer 
however,  enter  the  closed  compressed  air-chamber  of  the  under- 
ground workings  of  a  railway  tunnel  (such  as  those  in  operation- 


THE  MERCURIAL  BAROMETER.  221 

at  the  present  time  for  the  construction  of  the  London  Tube 
Railway),  or  the  caissons  of  a  great  bridge  while  they  are  being 
sunk  (as  in  the  case  of  the  Forth  Bridge),  or  go  down  into  the  sea 
in  a  diving-dress  or  diving-bell,  then  we  do  feel  a  most  uncom- 
fortable sensation  in  our  ears,  eyes,  <fcc.  Or,  if  we  climb  a  very 
high  mountain,  or  rise  far  into  the  air  in  a  balloon,  we  have  a  some- 
what similar  sensation,  but  due  to  an  opposite  effect — viz.,  a 
diminution  from  the  normal  pressure  to  which  we  are  accustomed. 

The  Mercurial  Barometer. — The  pressure  of  the  atmosphere 
is  usually  measured  by  a  mercurial  barometer,  which  consists  of 
a  vertical  tube  of  glass  about  33  inches  long,  of  uniform  calibre, 
hermetically  sealed  at  the  top  end,  into  which  has  been  carefully 
introduced  mercury  freed  from  air.  The  lower  end  dips  into  an 
open  dish  containing  a  quantity  of  that  liquid  metal.  Conse- 
quently the  pressure  of  the  atmosphere  acting  on  the  mercury  in 
the  open  dish  forces  it  up  inside  the  tube  to  a  height  directly 
proportional  to  its  pressure,  since  there  is  supposed  to  be  a 
perfect  vacuum  between  the  upper  surface  of  the  mercury  and 
the  closed  end  of  the  glass  tube. 

EXAMPLE  VI. — Suppose  the  height  of  mercury  as  registered  by 
a  mercurial  barometer  is  30  inches,  and  that  the  specific  gravity 
of  mercury  be  taken  as  13-6,  what  would  be  the  height  in  feet  of 
a  water  column  which  would  support  the  same  atmospheric 
pressure  ? 

ANSWER. —          i :  13-6  : :  30  inches :  x 

.*.  x  =  30  x  13-6  =  408"  =  34  feet. 

Low  Pressure  and  Vacuum  Water  Gauges.* — It  is  often 
necessary  for  the  engineer  to  measure  low  pressures  or  vacuums  of 
gases.  For  example,  in  the  supply  of  illuminating  gas  to  a 
town,  or  in  the  pressure  of  air  feeding  a  boiler  furnace  by  natural 
or  forced  draught,  or  the  vacuum  produced  by  a  chimney- 
stalk;  or,  in  the  case  of  the  vacuum  in  a  coal  mine  produced 
by  a  furnace  below  the  earth,  or  by  a  guibal  fan  situated 
near  the  upcast  shaft,  <fcc.  In  such  cases,  as  well  as  in  many 
others  where  low  pressures  have  to  be  observed,  the  force  is  not 
reckoned  by  pounds  per  square  inch,  or  by  inches  of  mercury 
sustained  in  a  vertical  column,  but  by  the  number  of  inches  of 
water  which  the  pressure  will  support  or  which  the  vacuum  will 
detract  from  the  atmospheric  pressure. 

The  accompanying  figure  illustrates  the  apparatus  usually 
employed  in  determining  such  low  pressures.  It  consists  of  a 

*  For  a  description  of  mercurial  pressure  and  vacuum  gauges,  as  well  aa 
Bourdon's  high-pressure  and  vacuum  gauges,  refer  to  the  Author's  Elemen- 
tary and  Advanced  Books  on  "  Steam  and  Steam  Engines." 


222 


LECTUKE  XVIII. 


W 


simple  bent  U  glass  tube  with  a  scale  between  the  vertical  legs  of 
the  U,  divided  into  inches  and  tenths  of  an  inch,  so  that  either 
the  pressure  or  the  vacuum  may  be  read  off  in  inches  of  water 
pressure,  according  as  the  forward  pressure  from  the  point  of 
supply  is  positive  or  negative  in  respect  to  the  pressure  of  the 
atmosphere.  For  example,  let  the  leg  of 
the  U  tube  next  the  cock  be  connected 
to  the  gas  pipe  of  a  house,  then  the 
pressure  of  the  gas  supply  acts  on  the 
water  in  the  right-hand  leg  of  the  tube, 
and  forces  it  downwards,  whilst  the 
water  in  the  other  leg  rises  correspond- 
ingly. The  reading  observed  on  the 
scale  S,  below  or  above  the  zero  or 
equilibrium  line,  has  of  course  to  be 
doubled  in  order  to  ascertain  the  exact 
total  pressure  in  inches  of  water.  If 
the  U  tube  be  connected  to  a  vacuum 
or  negative  pressure,  then  the  water 
rises  in  the  inner  leg  of  the  U  tube, 
owing  to  the  greater  pressure  of  the 
atmosphere  on  the  outer  limb,  and  the 
inches  of  water  representing  the  amount 
of  the  vacuum  are  accordingly  read  off 
in  the  same  way.  For  example,  if  the 
apparatus  be  connected  to  the  base  of 
a  steam  boiler  chimney,  or  to  the  inlet  of  a  guibal  fan  creating 
a  draught  in  a  coal  mine,  then  the  suction  produced  forms  a 
vacuum  which  requires  the  supply  of  atmospheric  air,  and  con- 
sequently the  air  presses  on  the  open  water  of  the  outer  limb  of 
the  U  tube,  and  forces  it  downwards.  The  vacuum  is  therefore 
observed  and  recorded  by  adding  the  inches  of  water  below  and 
above  the  zero  line. 

EXAMPLE  VII. — A  difference  of  level  is  observed  of  4  inches 
between  the  outer  and  inner  limbs  of  a  U  tube  water-gauge. 
What  is  the  pressure  of  the  gas  supply  in  Ibs.  per  square  inch  ? 

ANSWER. — A  vertical  column  of  34  feet  of  water  corresponds  to 
15  Ibs.  pressure  on  the  square  inch.  Consequently, 

(34'  x   12"):  4"::  15  Iks.:* 


GAS  PRESSURE  GAUGE. 

INDEX  TO  PARTS. 

SC  represents  Steam  or 

gas  cock. 

GT          „          Glass  tube. 
S  Scale. 

W  Water. 


34 


12 


=  —  >  or  nearly  -{-  of  a  Ib.  per  sq.  in. 


The  Siphon  is  simply  a  bent  tube  for  withdrawing  liquids 
from  a  higher  to  a  lower  level  by  aid  of  the  atmospheric  pressure. 
It  is  used  in  chemical  laboratories  and  works  for  emptying  acids 


THE    SIPHON.  ,  223 

from  carboys,  in  breweries  and  distilleries  for  extracting  beer 
from  vats  and  spirits  from  casks,  in  the  crinal  glass  tube  of 
Lord  Kelvin's  recorder  for  conveying  ink  from  the  ink-pot  to 
the  telegraph  message-paper ; 
and  on  a  large  scale  for  draining 
low-lying  districts,  such  as  the 
fens  of  Lincolnshire. 

The  conditions  for  the  success- 
ful working  of  a  siphon  are, 
that— 

1.  The  liquid  shall  be  carried          THE  SIPHON. 
by  the  outer  limb  of  the  tube  to 

a  lower  level  than  the  surface  of  the  supply. 

2.  The  vertical  height  from  the  free  surface  of  the  liquid  being 
drained  to  the  top  of  the  bend  of  the  siphon  shall  not  be  greater 
than  the  height  of  the  water  barometer  at  the  time— say  only  30  feet 

on  account  of  the  necessary  deduction  of  3  or  4  feet  to  be  made 

from  the  full  height  of  34  feet,  due  to  having  to  overcome  the 
friction  of  the  pipe. 

3.  The  end  of  the  siphon  dipping  into  the  liquid  to  be  drained, 
shall  not  become  uncovered. 

To  start  the  siphon,  either  the  tube  must  be  filled  with  liquid, 
the  ends  closed,  and  the  siphon  inverted,  with  the  shorter  limb 
under  the  fluid  to  be  drained,  before  uncovering  the  ends ;  or, 
whilst  the  end  of  the  shorter  limb  is  in  the  liquid  a  partial 
vacuum  must  be  formed  in  the  siphon  tube  by  extracting  the  air 
from  the  end  of  the  longer  leg. 

The  principle  upon  which  the  siphon  acts  is  as  follows  :— 
A  vacuum  having  been  formed  in  the  tube,  the  pressure  of  the 
atmosphere  acting  on  the  free  surface  of  the  liquid  to  be  drained, 
forces  it  up  the  shorter  limb,  and  having  turned  the  highest 
point  of  the  n  the  liquid  descends  the  longer  limb  by  the  action 
of  gravity  with  a  velocity  proportional  to  the  J 'difference  of  levels 
between  the  outlet  and  the  free  surface  of  the  source  of 
supply.  The  outflowing  liquid  is  always  acting  as  a  water-tight 
piston  at  the  bend  of  the  f|,  and  in  this  way  keeping  up  the  vacuum 
there,  until  either  the  inlet  and  the  outlet  free  surfaces  come  to  a 
level  (when  the  siphon  stops  for  want  of  "  head  "),  or,  when  the 
difference  of  level  between  the  free  surface  of  the  supply  and  the 
top  of  the  bend  exceeds  the  height  supportable  by  the  atmosphere, 
when  it  stops  for  want  of  breath  or  atmospheric  pressure. 

p 


224  LECTURE   XVIII. 

.Distinction  between  Solids,  Liquids,  and  Gases.— At  the 

very  commencement  of  this  book  we  referred  to  the  fact  that  Matter 
exists  under  three  conditions. 

(i)  Solids  ;   (2)  Liquids ;   (3)  Gases.     We  shall  now  define  and  distin- 
guish concisely  between  the  three  states  of  matter. 

(1)  A   Solid  is  matter  in  such  a  condition,  that  the  molecules  cannot 
move  freely  amongst  themselves,  and  consequently  it  retains  its  shape 
and  volume  unless  acted  upon  by  a  force. 

(2)  A  Liquid  is  a  collection  of  inter-mobile  particles  of  matter,  which 
offer  great  resistance  to  change  of  volume,  but  little  to  change  of  shape. 

(3)  A  Gas  is  matter  in  its  most  subdivided  state,  and  which  readily 
yields  to  the  slightest  force  tending  to  change  its  shape  or  its  volume. 

We  thus  see  that  the  chief  characteristic  distinctions  between  these 
three  states  of  matter  are,  that — 

(1)  A  Solid  resists  both  change  of  shape  and  of  volume. 

(2)  A  Liquid  only  resists  change  of  volume. 

(3)  A  Gas  resists  neither  change  of  shape  nor  of  volume. 

(4)  A  Fluid  may  be  either  a  liquid  or  a  gas. 

(5)  A    Viscous  Fluid  is  a  liquid  which  offers  more  or  less  resistance  to 
motion  amongst  its  particles,  e.g.,  treacle,  tar,  and  heavy  oils,  &c. 

(6)  An  Elastic  Fluid  is  a  gas  whose  volume  will  increase  indefinitely. 

(7)  Cohesion  is  a  property  of  matter  common  to  both  solids  and  liquids. 
It  causes  more  or  less  resistance  to  the  separation  of  the  molecules  of 
matter. 


LECTURE  XVIII. — QUESTIONS.  225 


LECTURE  XVIII.— QUESTIONS. 

1.  What  are  the  respective  specific  gravities  and  the  weights  per  cubic 
foot  and  per  gallon  of  fresh  and  of  salt  water  ? 

2.  A  cylindrical  vessel,  120  inches  long  and  10  inches  in  diameter,  is 
sunk  vertically  in  water,  so  that  the  base,  which  is  horizontal,  is  at  a  depth 
of  100  inches  below  the  surface  of  the  water.     Find  the  upward  pressure 
in  pounds  on  the  base  of  the  vessel.     Ans.  284-2  Ibs. 

3.  A  cubical  box  or  tank  with  a  closed  lid,  the  length  of  a  side  of  which 
is  5  feet,  rests  with  its  base  horizontal,  and  an  open  vertical  pipe  enters 
one  of  its  sides  by  an  elbow.     The  tank  is  full  of  fresh  water,  and  the  pipe 
contains  water  to  the  height  of  10  feet  above  the  top  of  the  tank.     What 
are  the  pressures  of  water  on  the  top,  bottom,  and  sides  of  the  tank  1 
Ans.  15,625  Ibs. ;  23,437.5  Ibs.  ;  19,531-25  Ibs. 

4.  A  water  tank  10'  long,  10'  wide,  with  an  inclined  base  10'  deep  at  one 
end  and  5*  at  the  other  end,  is  filled  with  fresh  water.    Find  the  pressure 
in  pounds  on  each  of  the  four  sides  and  on  the  base.     Ans.  31,250  Ibs.  ; 
7,812-5  Ibs.  ;  18,229-16  Ibs.  ;  52,500  Ibs. ;  10,421  *S  Ibs.* 

5.  A  lock  gate  is  12  feet  wide,  and  the  water  rises  to  a  height  of  8  feet 
from  the  bottom  of  the  gate.    What  pressure  in  pounds  does  it  sustain  ? 
The  weight  of  a  cubic  foot  of  water  is  62^  Ibs.    Ans.  24,000  Ibs. 

6.  A  vertical  rectangular  sluice  gate,  measuring  2  feet  horizontal  by  3  feet 
vertical,  is  immersed  so  that  its  upper  side  is  4  feet  below  the  surface  of  the 
water  pressing  on  it.     Find  the  pressure  on  tbe  gate  :  you  are  required  to 
explain  the  reasoning  on  which  your  calculation  is  founded., 

Ant.  2062*5  Ibs. 

7.  What  is  meant  by  the  "  centre  of  pressure "  in  the  case  of  a  plane 
surface  immersed  in  &  liquid  ?    If  the  plane  be  a  horizontal  circle,  where 
does  the  centre  of  pressure  act  ?    If  it  be  a  vertical  rectangle  10  feet  wide 
and  6  feet  deep,  immersed  in  water  so  that  the  upper  edge  of  the  rectangle 
Is  flush  with  the  surface  of  the  water,  where  does  the  "  centre  of  pressure  " 
act  ?    Ans.  at  the  centre  of  the  circle  ;  4  feet  below  surface  of  water. 

8.  State  the  law  discovered  by  Archimedes,  and  the  conditions  for  a  bodj 
In  equilibrium  floating  in  a  liquid.    A  cy Under  10  feet  long  and  2  feet  in 
diameter  floats  in  fresh  water,  with  2  feet  prelecting  from  the  surface  ;  find 
tbe  weight  of  the  cylinder.    Ans.  1571  Ibs. 

9.  A  rectangular  tank,  5  feet  square,  is  filled  with  water  to  a  height  of 
7  f  feet.    A  rectangular  block  of  wood,  weighing  312-5  Ibs.,  and  having  a  sec- 
tional area  of  5  square  feet,  is  placed  in  the  tank,  and  floats  with  its  sH*»s 
vertical  and  with  its  section  horizontal.     How  much  does  the  water  rise 
in  the  tank,  and  what  is  now  the  pressure  on  one  vertical  side  of  the  tank  I 
Ans.  2-4  inches  ;  9875-4.^)8. 

10.  The  mercurial  barometer  registers  31";  calculate  the  height  of  columns 
Df  fresh  and  of  salt  water  that  will  balance  the  corresponding  pressure. 
Ans.  35-13  ft.,  34-24  ft. 

n.  Sketch  and  describe  a  mercurial  barometer.  State  how  it  is  made, 
and  how  it  acts  as  a  register  of  the  pressure  of  the  atmosphere. 

12.  Describe  some  simple  form  of  gauge  which  would  enable  you  to 
measure  the  pressure  at  which  gas  is  supplied,  and  explain  the  principle  on 
which  it  is  constructed. 

*  In  the  answers  given,  ^12$  is  assumed  to  be  11*3. 


226  LECTURE  XVIII.— QUESTIONS. 

13.  Sketch  and  explain  the  action  of  the  siphon,  and  give  a  few  practical 
examples  of  its  use.    Also  state  under  what  circumstances  it  fails  to  work. 

14.  The  bottom  of  a  water-tank  measures  f  in  length  and  3'  4"  in  width. 
When  the  tank  contains  900  gallons  of  water,  what  will  be  the  depth  of 
the  water,  and  what  would  be  the  pressure  on  the  bottom,  on  each  side 
and.  end  of  the  tank  respectively  ?     One  gallon  of  water  weighs  10  Ibs. 
One   cubic  foot  weighs  62-3  Ibs.     Ans.   Pressure  on   bottom   of  tank  = 
9000  Ibs.     Pressure  on  each  side  =  8382  Ibs.     Pressure  on  one  end  of  tank  - 
3991  Ibs.     Depth  of  centre  of  pressure  is  6*2  feet. 

15.  Draw  the  diagram  of  water-pressure  on  the  side  of   a  tank  wi  h 
vertical  sides,  12  feet  high,  and  filled  with  water.     Deduce  the  vertical 
depth  of  the  centre  of  pressure  below  the  top  edge  of  the  tank. 

16.  Name  the  chief  physical  properties  of  a  liquid,  and  show  in  what 
respect  a  liquid  differs  from  a  gas  and  from  a  solid.     How  is  the  pressure 
of  water  on  the  vertical  sides  of  a  tank  calculated  ? 

A  water-tank  is  10'  long,  10'  wide,  and  10'  deep.  When  it  is  filled  with 
water,  what  will  be  the  force  with  which  the  water  acts  on  one  side  of  the 
tank  1  Ans.  31,250  Ibs. 

17.  Describe  how  you  would  carry  out  an  experiment  to  determine  the 
discharge  of  water  through  a  sharp  edged  circular  orifice  on  the  sid«  of  a 
water  tank.     (B.  of  B.,  1904.) 


LECTURE  XIX. 

CONTENTS. — Hydraulic  Machines— The^Common  Suction  Pump — Example 
I. — The  Plunger,  or  Single-acting  Force  Pump— Example  II. — Force 
Pump  with  Air  Vessel — Continuous-delivery  Single-acting  Force 
Pump  without  Air  Vessel — Combined  Plnnger  and  Bucket  Pump — 
Double-acting  Force-Pump — Example  III. — Centrifugal  Pumps — 
Example  IV. — Questions. 

Hydraulic  Machines. — The  Common  Suction  Pump  consists 
of  a  bored  cast-iron  barrel  PB,  terminating  in  a  suction  pipe,  SP, 
fitted  with  a  perforated  end  or  rose  R,  which  dips  into  the  well 
from  which  the  water  is  to  be  drawn.  The  object  of  the  rose  is 
to  prevent  leaves  or  other  matter  getting  into  the  pump,  that 
might  clog  and  spoil  the  action  of  the  valves.  At  the  junction 
between  the  barrel  and  suction  pipe  there  is  fitted  a  suction  valve 
SY,  of  the  hinged  clack  type  faced  with  leather.  The  piston  or 
bucket  B  is  worked  up  and  down  in  the  barrel  of  the  pump  by 
the  force  P,  applied  to  the  end  of  the  handle  H,  being  commu- 
nicated to  it  through  the  connecting  link  of  the  hinged  piston- 
rod  PR.  In  the  centre  and  at  the  top  of  the  bucket  is  fixed  the 
clack  delivery  valve  DY,  which  is  also  faced  with  leather  in  order 
to  make  it  water-tight.  The  bucket  is  sometimes  packed  with 
leather ;  but,  as  shown  by  the  figure,  a  coil  of  tightly  woven  flax 
rope  wrapped  round  the  packing  groove  would  be  more  suitable 
in  the  present  instance. 

Action  of  the  Suction  Pump. — (i)  Let  the  barrel  and  the  suc- 
tion pipe  be  filled  with  air  down  to  the  water-line,  and  let  the 
bucket  be  at  the  end  of  the  down  stroke.  Now  raise  the  bucket 
to  the  end  of  the  up-stroke  by  depressing  the  pump  handle.  This 
tends  to  create  a  vacuum  below  DY ;  therefore  the  air  which  filled 
the  suction  pipe  opens  SY,  expands,  and  fills  the  additional  volume 
of  the  barrel.  Consequently,  according  to  Boyle's  law,  its  pres 
sure  must  be  diminished  in  the  inverse  ratio  to  the  enlargement 
of  its  volume.*  This  enables  the  pressure  of  the  atmosphere 

*  The  student  may  refer  to  Lecture  XII.  of  the  Author's  Elementary 
Manual  on  "  Steam  and  the  Steam  Engine,"  for  an  explanation  and  demon- 
stration of  Boyle's  law  ;  where  it  is  shown  that  if  p=the  pressure  of  a 
gas  and  t>=its  volume,  then  at  a  uniform  temperature  pv=o  constant,  or 
p  varies  as—. 


228 


LECTURE   XIX. 


(which  acts  constantly  on  the  surface  of  the  water  in  the  well)  to 
force  a  certain  quantity  of  water  up^the  suction  pipe,  until  the 
weight  of  this  column  of  water  and  the  pressure  of  the  air 
(between  it  and  the  delivery  valves)  balance  the  pressure  of  the 
outside  atmosphere. 


COMMON  SUCTION  PUMP. 
INDEX  TO  PARTS. 


H  represents  Handle. 

P         „          Push  or  pull  at  A. 

F          „          Fulcrum  of  H. 
PR          „          Plunger  rod. 
IB          „          Pump  barrel. 

S          „         Spout. 


SP  represents  Suction  pipe. 
R  Rose. 


SV 

B 

DV 


Suction  valve. 
Bucket  or  piston. 
Delivery  valve. 


(2)  In  pressing   the   bucket  to  the  bottom  of  the  barrel  by 
elevating  the  handle,  the  suction  valve  closes  and  the  delivery 
valve  opens,  thereby  permitting  the  compressed  air  in  the  barrel 
to  escape  through  the  delivery  valve  into  the  atmosphere. 

(3)  Raise  and  depress  the  piston  several  times  so  as  to  produce 
the  above  actions  over  again,  and  thus  gradually  diminish  the 
volume  of  the  air  in  the  pump  to  a  minimum.     Then  water  will 
have  been  forced  by  the  pressure  of  the  atmosphere  up  the  suction 
pipe  and  into  the  pump,  if  the  bucket  and  the  valves  are  tight, 


SINGLE    ACTING   POECE   PUMP.  2  29 

and  if  the  delivery  valve  when  at  the  top  of  its  stroke  be  not 
more  than  the  height  of  the  hydro-barometric  column  above  the 
water  line  of  the  well.* 

(4)  The  bucket  now  works  in  water  instead  of  in  air.  In  fact, 
the  machine  passes  from  being  an  air-pump  to  be  a  water  one. 
During  the  down-stroke  water  is  forced  through  the  delivery 
valve.  During  the  up-stroke  this  water  is  ejected  through  the 
spout  ;  at  the  same  time  more  water  is  forced  up  through  suction 
pipe  and  valve  to  supply  the  place  of  the  vacuum  created  by  the 
receding  piston.  The  water  is  therefore  discharged  only  during 
the  up-stroke  in  the  case  of  the  pump  illustrated  by  the  figure. 
Should  it,  however,  be  fitted  with  an  air-tight  piston-rod  and  pump 
cover,  and  should  the  pump  handle  be  moved  rapidly,  more  water 
will  be  taken  into  the  barrel  than  can  escape  from  the  spout 
during  the  up-stroke.  Consequently,  the  compression  of  the  pent- 
up  air  between  the  surface  of  the  water  in  the  barrel  and  the 
cover,  will  cause  the  water  to  flow  out  in  a  more  or  less  continuous 
stream  during  the  down-stroke.  In  other  words,  the  top  cover 
and  the  portion  of  the  pump  above  the  spout  may  be  converted 
into  an  air  vessel,  the  precise  action  of  which  will  be  explained 
later  on. 

EXAMPLE  I.  —  If  the  cross  area  of  the  bucket  of  a  suction  pump 
be  20  sq.  in.  and  if  water  be  raised  24  ft.  from  its  surface  in  the 
well,  what  is  the  pull  on  the  pump  rod  ? 

ANSWER.  —  The  pull  P  on  the  pump  rod  is  evidently  equal  to 
the  weight  of  a  column  of  water  of  height  H=24  ft.,  and  the 
area  of  the  bucket  in  sq.  ft.  =  A=  204-  144.  Therefore,  by  the 
formula  employed  for  the  pressure  of  a  liquid  on  a  surface  in 
Lectures  XVII.  and  XVIII.— 


P  =  24'x—  x  62-5  =  208  Ibs. 

144 

The  Plunger,  or  Single-acting  Force  Pump.  —  The  upper 
or  outer  end  of  the  barrel  of  this  pump  is  provided  with  a  stuffing- 
box  and  gland,  through  the  air-tight  packing  of  which  the  solid 
pump  plunger  works. 

During  the  up  or  outward  stroke  of  the  plunger  a  vacuum  is 

*  Theoretically,  such  a  pump  should  be  able  to  lift  water  from  a  depth 
of  34  feet  below  the  highest  part  of  the  stroke  of  the  delivery  valve,  but 
practically,  owing  to  the  imperfectly  air-tight  fitting  of  the  piston  and  the 
valves,  it  is  not  used  for  withdrawing  water  from  wells  more  than  20  to  25 
feet  below  this  position  of  the  delivery  valve.  In  fact,  such  a  pump  fre- 
quently requires  a  bucket  or  two  of  water  to  be  poured  into  it  above  the 
delivery  valve  in  order  to  make  it  work  at  all,  if  it  should  have  been  left 
for  some  time  without  being  worked. 


230 


LECTURE   XIX. 


created  in  the  pump  barrel,  and  consequently  air  is  expanded 
into  it  from  the  suction  pipe.  This  pipe  is  attached  to  the  flange 
of  the  suction  valve-box.  During  the  down  or  inward  stroke 
the  suction  valve  closes,  and  the  pent-up  air  in  the  barrel  is  forced 
through  the  delivery  valve.  This  action  goes  on  precisely  in  the 
manner  just  explained  in  the  case  of  the  suction  pump,  until  the 
water  rises  into  the  barrel.  Then  the  inward  stroke  of  the  plunger 
drives  water  through  the  delivery  valve  to  any  desired  height  (or 
against  any  reasonable  back  pressure,  as  in  the  case  of  a  feed 


THE  PLUNGER  FORCE  PUMP. 


SV  represents  Suction  valve. 
DV          „          Delivery  valve. 


INDEX  TO  PARTS. 

PB 
PP 


Ch 


Checks  for  valves. 


SB  and  G 


represents  Pump  barrel. 
,         Pump  plunger. 
,          Stuffing  box 
and  gland. 


pump  for  a  steam  boiler)  consistent  with  the  strength  of  the  pump 
and  the  power  applied. 

The  eye  of  the  plunger  may  be  attached  to  a  connecting-rod 
actuated  by  a  hand  lever,  as  in  the  case  of  the  suction  pump,  or  it 
may  be  worked  from  one  eccentric  or  crank  revolved  by  a  steam 
engine  or  other  motor. 

By  whichever  way  it  is  worked,  the  force  applied  to  the  plunger 
must  be  sufficient  to  overcome  the  friction  between  the  plunger 


FOECE    PUMP    WITH  AIR  VESSEL. 


231 


and  the  packing,  the  resistance  due  to  sucking  the  water  from  the 
source  of  supply,  and  of  driving  the  same  up  to  the  place  where 
it  is  delivered. 

With  this  pump  (as  in  the  case  of  the  suction  pump),  the  water 
is  only  delivered  during  one  out  of  every  two  strokes  of  the 
plunger,  and  consequently,  in  an  intermittent  or  pulsating  fashion. 
In  order  to  make  the  supply  continuous  we  have  to  use  one  or 
other  of  the  devices  about  to  be  described. 

EXAMPLE  II. — In  a  single-acting  plunger  force  pump  the  cross 
area  of  the  plunger  is  10  sq.  in.,  and  its  distance  from  the  surface 
of  the  water  in  the  well,  when  at  the  end  of  its  outv*  a~d  or  suction 
stroke,  is  20  ft.  During  the  inward  stroke  the  water  is  pumped 
up  to  a  height  of  100  ft.  above  the  end  of  the  plunger.  What 
forces  are  required  to  move  the  pump  plunger  during  (i)  an  "  out," 
and  (2)  an  in-stroke  (neglecting  the  forces  to  overcome  friction). 


ANSWER.— (i)  Px  =  HXAW  =20' 


10 

x —  x  62*5  =  86-8  Ibs.  pull. 
144 

10 


(2)  P8  =  H,AW  =  ioc/  x  —  x  62-5  =  434  Ibs.  pressure. 

Force  Pump  with  Air  Vessel. — In  the  following  figure  of 
a  force  pump  the  only  points  of  difference  worth  noticing  between 


FOECE  PUMP  WITH  AIR  VESSEL. 
INDEX  TO  PARTS. 


SP  represents  Suction  pipe. 


SV 
B 

PB 
G 


Suction  valve. 
Barrel  of  pump. 
Plunger  barrel. 
Packing  gland. 
Plunger  rod. 


DV  represents  Delivery  valve. 


S 
AV 
AC 
DP 


Stop  for  DV. 
Air  vessel. 
Air  cock. 
Delivery  pipe. 


232  LECTURE    XIX. 

it  and  the  previous  one  are  : — (i)  The  plunger,  instead  of  being 
solid,  is  a  hollow  trunk  or  barrel,  with  the  connecting  rod  fixed 
to  an  eye-bolt  at  its  lower  end. 

(2)  The  suction  and  the  delivery  valves  are  both  at  one  side, 
instead  of  being  fixed  on  opposite  sides  of  the  pump. 

(3)  There  is  an  air  vessel. 

Action  of  the  Air  Vessel. — During  the  inward  or  delivery  stroke 
of  the  plunger,  part  of  the  water  forced  from  the  barrel  goes  up 
the  delivery  pipe,  and  the  remainder  enters  the  air  vessel,  and 
consequently  compresses  the  air  in  AV.  During  the  out- 
ward or  non-delivery  stroke  of  the  plunger  the  compressed 
air  in  the  air  vessel  presses  the  rest  of  the  water  into  the  delivery 
pipe.  In  this  simple  way  a  continuous  flow  of  water  is  main- 
tained in  the  delivery  pipe,  and  with  far  less  shock,  jar,  and  noise 
than  in  the  previous  case.  Where  very  smooth  working  is  re- 
quired, an  air  vessel  is  also  put  on  to  the  suction  side  of  the 
pump.  Should  the  air  in  the  air  vessel  become  entirely  absorbed 
by  the  water,  the  fact  will  be  noticed  at  once,  by  the  noise  and 
the  intermittent  delivery.  Then  the  pump  should  be  stopped, 
the  air  cock  AC  opened,  and  the  water  run  out.  When  the  air 
vessel  is  full  of  air,  the  air  cock  should  be  shut  and  the  pump 
started  again. 

Continuous-delivery  Pump  without  Air  Vessel.  —  A 
fairly  continuous  delivery  may  be  obtained  by  making  the  plunger 
of  the  piston  form,  and  the  pump  rod  exactly  half  its  area,  as 
shown  by  the  accompanying  figure.  During  the  down  stroke, 
half  the  water  expelled  by  the  piston  from  the  under  side  of 
the  pump  barrel  goes  up  the  delivery  pipe,  and  the  other  half 
is  lodged  above  the  piston,  to  be  in  turn  sent  up  the  delivery  pipe 
during  the  up-stroke.  Where  very  high  pressures  are  required, 
such  as  in  the  filling  of  an  accumulator  ram,  pumps  working  on 
this  principle,  but  of  the  following  form,  are  frequently  used. 
The  *>-  '  ,</n  is  precisely  the  same  as  in  the  one  just  described,  and 
th.8  £  ime  index  letters  have  been  used,  so  that  the  student  will 
l;ave  no  difficulty  in  understanding  the  figure  ;  more  especially 
as  the  directions  of  motion  of  the  piston  and  of  the  ingoing  and 
outflowing  water  have  been  marked  by  straight  and  feathered 
arrows  respectively.  Where  sea  or  acid  water  is  used  it  may  be 
necessary  to  fit  the  pump  barrel,  PB,  with  a  brass  liner,  L,  to 
prevent  corrosion. 

In  accumulator  and  other  kinds  of  high-pressure  work  it  is  not 
advisable  to  use  air  vessels,  because  you  cannot  prevent  the  water 
which  enters  the  vessel  absorbing  air  and  carrying  the  same  with 
it  to  the  hydraulic  machines,  where  its  presence  would  be  most 
objectionable,  and  because  with,  say,  750  to  1000  or  more  Ibs. 


CONTIGUOUS    DELIVERY    FORCE    PUMP. 


^33 


CONTINUOUS-DELIVERY  FORCE  PUMP  WITHOUT  AIR  VESSEL. 
INDEX  TO  PARTS. 


IP  represents  Inlet  pipe. 

SV         „         Suction  valve. 

CC  Cover  and  check  to 

SV. 

P         „         Piston. 
PR         „         Pump-rod. 


DV  represents  Delivery  valve. 
CO         „          Cover  and  check  to 

DV. 

DP         „         Discharge  pipe. 
SB  Stuffing-box. 

G  Gland. 


CONTINUOUS-DELIVERY  FORCE  PUMP. 
used  ID   Connection  with  the  Armstrong  Accumulator. 
(See  Indexes  to  previous  Figures.) 


234  LECTURE   XIX. 

pressure  per  square  inch,  you  would  require  a  very  large  and  very 
strong  air  vessel  before  it  could  be  of  any  service.  If  a  pressure 
of  only  750  Ibs.  per  square  inch  were  used,  then,  since  the  normal 
pressure  of  the  atmosphere  is  15  Ibs.  per  square  inch,  the  air  in 
the  air  vessel  would  be  compressed  to  -f^,  or  -pyth  of  its  original 
volume,  in  accordance  with  Boyle's  law.  Consequently,  with  an 
%ir  vessel  of  50  cubic  feet  internal  volume,  there  would  be  only 
i  cubic  foot  of  air  in  it,  when  the  pump  was  in  full  action. 

Combined  Plunger  and  Bucket  Pump. — We  have  already 
seen  that  a  suction  pump  discharges  water  during  the  outward 
stroke,  and  that  a  plunger  pump  discharges  water  during  the 
inward  stroke ;  consequently,  by  combining  these  two  kinds,  we 
get  a  double-acting  pump.  By  making  the  cross  area  of  the 
plunger  half  that  of  the  barrel,  half  the  water  raised  by  the 
bucket  during  the  up-stroke  goes  into  the  delivery  pipe,  whilst 
the  other  half  fills  the  space  left  by  the  receding  plunger.  During 
the  down-stroke  the  plunger  forces  the  latter  half  up  the  delivery 
pipe.  We  do  not  happen  to  have  a  figure  with  which  to  illus- 
trate these  remarks,  but  if  the  student  will  first  of  all  sketch  a 
complete  vertical  section  of  a  suction  pump  like  that  shown  by 
the  first  figure  in  this  lecture,  and  then  draw  a  solid  plunger,  with 
stuffing-box  and  gland,  like  that  in  the  second  figure,  in  place  of 
the  pump  rod  and  open  cover  in  the  suction  pump,  it  will  form  a 
useful  exercise  in  the  designing  of  such  a  pump. 

Double-acting  Force  Pump. — The  pumps  which  we  have 
hitherto  considered  are  all  single-acting  in  this  sense,  that  they 
do  not  both  suck  and  discharge  water  during  every  stroke.  This 
can,  however,  be  accomplished  by  having  two  sets  of  suction  and 
delivery  valves  placed  at  each  end  of  the  pump  barrel,  as  shown 
by  the  accompanying  figure.  Then,  during  the  outward  stroke  of 
the  piston  the  pump  draws  water  from  the  source  of  supply 
through  the  inlet  pipe  and  suction  valve  SVj.  At  the  same  time 
the  piston  forces  the  water  in  front  of  it  through  the  delivery 
valve  DV,  and  outlet  pipe.  During  the  inward  stroke,  suction 
takes  place  through  SV,  and  discharge  through  DVX,  all  as 
clearly  shown  by  arrows  in  the  drawing.  The  valves  are  pro- 
vided with  india-rubber  cushions,  IR,  to  ease  the  shock  and  mini- 
mise the  jarring  noise  due  to  their  reaction  and  natural  reverbe- 
ration when  they  are  suddenly  opened  and  closed. 

EXAMPLE  III. — In  a  double-acting  force  pump  the  vertical 
height  from  the  surface  of  the  well  to  the  point  of  delivery  is 
TOO  feet.  If  the  area  of  the  piston  equal  i  square  foot,  what  is 
the  stress  on  the  piston-rod  during  each  stroke  ? 

ANSWER. — Here  we  need  not  distinguish  between  the  force 
required  during  suction  and  delivery,  for  both  actions  take  place 


DOUBLE   ACTING  FORCE  PUMP. 


235 


during  each  stroke.    We  have  only  to  deal  with  the  net  force 
required  to  elevate  a  column  of  water  to  a  height  of  100  feet. 


DOUBLE  ACTING  FORCE  PUMP.* 
INDEX  TO  PARTS. 


SV,  SV2  represent   Suction  valves. 
DV,  DV2         „          Delivery  valves. 
IK         „         India-rubber 

cushions. 


B  represents  Barrel  (liner). 
P         „         Piston  (solid). 
PR  Piston-rod. 


Neglecting  friction,  the  stress  on  the  piston  rod  will  therefore 
be  the  weight  of  a  column  of  water  of  height  100'  and  cross 
area  =  i  sq.  ft. 

.'.  P-  HAW=  100'  x  i'  x  62-5  =  6250  Ibs.  pull  and  push. 
If  30  per  cent,  of  the  force  applied  be  spent  in  overcoming 
friction,  what  will  then  be  the  stress  on  the  pump- rod.     Here 
6250  is  only  70  per  cent,  of  the  whole  stress,  for  30  per  cent,  of 
the  whole  is  lost  force. 

.'.  70  :  TOO  : :  6250  :x 
x  =  6250  x    100  = 

*  We  are  indebted  for  the  above  figure  to  Professor  H.  Robinson's  book 
on  "Hydraulic  Machinery,"  published  by  Messrs.  Charles  Griffin  &  Co. 
Students  should  refer  to  Lecture  XXIV.  of  the  Author's  Elementary 
Manual  on  "  Steam  and  the  Steam  Engine  "  for  detailed  illustrations  and 
description  of  the  air  and  circulating  pumps  of  the  SS.  "  St.  Rognvald." 


LECTURE   XIX. 


Centrifugal  Pumps. — The  following  illustration  shows  one  ok 
these  pumps  or  reversed  water  turbines.  They  are  often  used  in 
preference  to  the  reciprocating  pumps  previously  described,  when 
large  quantities  of  water  have  to  be  quickly  elevated  through  a 
small  height,  such  as  emptying  graving  docks  and  holds  of  vessels 


sc 


THE  BON-ACCORD  CENTRIFUGAL  PUMP. 
Designed  and  made  by  Drysdale  &  Company,  Glasgow. 

INDEX  TO  PARTS. 


SF  for  Supporting  flanges. 
PC   „  Pump  casing. 
CP  „       „      door. 
SP  »    Suction  pipe. 
V  „   Suction  water  guide. 


IW  for  Impeller  wheel. 

S  „         „         shaft. 

N  „  „  nut. 
DP  „  Delivery  pipe. 
->  „  Direction  of  flow. 


or  circulating  the  cooling  water  through  the  condenser  tubes  of 
steam-engines,  as  well  as  for  dredging  soft- bottomed  rivers. 

The  original  type  of  centrif  ugafpump  had  straight  radial  blades, 
but  it  has  been  found,  that  if  these  are  curved  in  the  direction 
and  manner  shown  by  the  accompanying  figure,  there  is  less 
shock  due  to  the  quick  flowing  water  and  greater  efficiency.  They 
may  be  driven  by  belts,  direct  coupled  steam-engfnes,  turbines, 
or  electric  motors  as  preferred. 


V 

BON-ACCORD  CENTRIFUGAL  PUMP.          237 

Details. — As  will  be  seen  from  the  vertical  cross-section  and 
side  view,  the  chamber  consists  of  a  snail-like  outer  pump  casing 
P  C,  supported  upon  two  flanged  feet  S  F,  connected  to  a  suction 
pipe  S  P,  and  a  delivery  pipe  D  P.  In  the  centre  is  fitted  the 
shaft  S,  which  carries  an  impeller  wheel  I  W,  that  rotates 
between  the  tapered  inside  faces  of  the  pump  casing  P  C,  and 
a  removable  side  cover  S  C. 

Action  when  circulating  cold  water  through  a  steam  condenser. — 
Should  this  pump  be  situated  below  the  level  of  the  supply  water, 
the  air  is  driven  out  of  the  pump  and  its  pipes  by  this  head  of 
water.  In  such  a  case,  the  pump  can  be  started  straight  away 
by  its  motor.  But,  where  the  pump  is  situated  above  the  suction 
supply,  then  the  mere  rotation  of  the  impeller  wheel  I  W  does 
not  produce  a  sufficient  vacuum  to  make  the  water  rise  into  it ; 
and  consequently,  the  pump  casing  has  either  to  be  filled  with 
water  through  the  nipple  hole  (beside  the  lifting  eye-bolt)  or  a 
steam  ejector  with  a  sluice  valve  are  added  in  certain  cases. 
Supposing  that  the  pump  is  fairly  started,  then  the  mere  rota- 
tion of  I  W  inside  the  water-tight  casing  not  only  gives  the  kinetic 
energy  and  pressure  to  the  water  contained  therein,  to  force  the 
same  right  through  the  delivery  pipe  D  P  and  the  condenser  tubes, 
butalso  tokeepup  the  necessary  vacuum  in  the  purnp,so  as  to  ensure 
a  continuous  feed  of  water  through  the  suction  pipe.  It  will  be 
observed,  thf.t  the  incoming  water  is  divided  by  the  sharp,  knife- 
edged  portion  of  cast  iron,  at  the  volute  V,  and  that  it  flows 
equally  up  each  side  to  the  centre  of  the  wheel,  whereby  the 
same  is  subjected  to  balanced  side  pressures. 

Should  the  interior  of  the  pump  require  to  be  inspected,  the 
attendant  may  first  open  the  cleaning  door  C  D,  but  if  he  finds 
that  any  adjustment  is  required,  then  he  can  take  off  the  side 
cover  S  C.  When  this  is  removed,  he  will  obtain  a  clear  view 
of  the  whole  of  I  W,  and  he  may  remove  the  same  from  the 
tapered  end  of  the  shaft  S,  by  unscrewing  the  nut  N. 


238  LECTURE   XIX. 

EXAMPLE  IV. — A  centrifugal  pump  is  to  lift  6'2  cubic  feet  of 
water  per  second  to  a  height  of  7  feet ;  how  much  horse-power  must 
be  supplied  to  it  if  its  efficiency  is  45  per  cent  ? 

It  is  direct-driven  by  a  continuous  current  electro-motor  whicv 
works  at  200  volts.  How  many  amperes  of  current  must  be  supplied 
to  the  motor,  if  its  efficiency  is  85  per  cent  ?  (B.  of  E.,  1904.) 

Useful  work  done)      TTT  •  i  ,     * 

by  pump  in  lifting  L  =  Weight  of  w^r ln,lbs'  x  dlstance  m  ft-  throuSV 
the  water       J         which  it  is  raised. 

„         „  =6-2  x  62-5  x  7  =  2712-5  ft.-lbs. 

But,  Efficiency  of  \  _  Useful  work  done  by  centrifugal  pump  in  ft.-lbs. 
Pump,  ijf        J     Total  work  done  in  driving  the  centrifugal  pump. 
45  )        27I2'5  ft.-lbs. 

100  J  ~~  Total  work  done. 

.-.Total  work  done  j      2?I2.5X 

in  driving  Cen-  j-  =  —      — -  ft.-lbs. 

trifugal  Pump  J  45 

And  this  work  is  done  per  second. 

.*.  H.P.    required  J      27i2'r  x  100 

to  drive  the  Cen- ]-=— —         =10'96H.P. 

trifugal  Pump  J 

Also  i  H.P.  =  746  watts,  and  i  watt  =  i  volt  x  i  ampere. 
Hence.     Watts)  , 

givenoutby Motor)  =  Io'96  *  746-  watts. 

And,  Efficiency  of)      Watts  given  out. 
Motor  J  ~~  Watts  taken  in. 

85  ^         10-96  x  746 

100  J  ^  Watts  taken  in. 

.*.  Watts  taken  in)      10-96  x  746  x  100 
by  Motor  /"  "~8j~ 

But,  Watts  taken)      n  -n  -rr  ^ 

in  by  Motor     )  =  Currenfc  m  Amperes  x  Pressure  in  Yolte. 

.%  Current  in  am-)  _  Watts  taken  in  by  Motor, 
peres  J  ~~         Pressure  in  Yolts, 

}_  io-96  x  746  x  100 
85  x  200 

4088 

-5 — =48-1  Amperes. 


LECTUEE  XIX. — QUESTION*.  239 


LECTURE  XIX.— QUESTIONS. 

1.  Explain  the  manner  in  which  the  pressure  of  the  atmosphere  Is  mada 
serviceable  in  the  case  of  the  common  suction  pump.     Sketch  and  explaip 
by  an  index  the  details  of  this  pump. 

2.  Describe,  with  a  sketch,  an  ordinary  suction  or  lifting  pump,  and  ex» 
plain  its  action.    If  the  diameter  of  the  bucket  is  4",  and  the  spout  is  20? 
above  the  free  surface  of  the  well,  what  is  the  tension  on  the  pump-rod  in 
the  tsp-stroke  ?    Arts.  109  Ibs. 

3.  Sketch  and  describe  a  force  pump,  drawing  a  section  so  as  to  show 
the  packing  of  the  plunger  and  the  construction  of  the  valves.     How  is  an 
air-vessel  applied  to  such  a  pump  ?    Why  is  the  air-vessel  dispensed  with 
when  pumping  water  into  an  accumulator  ? 

4.  Explain  the  use  of  an  air-vessel  in  connection  with  a  force  pump. 
Sketch  a  section  through  a  double-acting  force  pump,  showing  the  valves 
and  the  connection  of  the  pump  with  the  air-vessel,  and  explain  the  action 
of  the  pump.    Water  is  forced  «p  to  100  feet  above  the  air-vessel ;  what 
proportion  of  the  volume  of  the  air-vessel  is   occupied  with  water,  and 
what  is  the  pressure  of  the  air  therein?    Ans.  74*6  per  cent.  ;  43*35  Ibs. 
per  sq.  in.  above  the  atmospheric  pressure. 

5.  The  leverage  to  the  end  of  the  handle  of  a  common  force  pump  is  five 
times  that  to  the  plunger,  ana  the  area  of  the  plunger  is  5  square  inches ; 
what  pressure  at  the  end  of  the  lever  handle  will  produce  a  pressure  of 
45  Ibs.  per  square  inch  on  the  water  within  the  barrel  1    Ans.  45  Ibs. 

6.  A  force  pump  is  used  to  raise  water  from  a  well  to  a  tank.     The 
piston  has  a  diameter  of  r6",  and  is  20'  above  the  free  surface  of  the  water 
in  the  well,  and  40'  below  the  mouth  of  the  delivery  pipe  leading  into  the 
tank.     Find  the  force  required  to  work  the  pump — (i)  Neglecting  friction; 
(2)  when  30%  is  spent  in  overcoming  friction  ;  (a)  when  sucking,  (6)  when 
forcing,  (c)  what  is  the  work  put  in  and  got  out  per  double  stroke  of  6"  t 
Ans.  (a)  (i)  17-45  Ibs-  5  (2)  24-93  Ibs.  ;  (&)  (i)  34'9  Ibs. ;  (2)  49-86  Ibs.  ;  (c) 
37-39  ft.-lbs.  ;  26-17  ft. -Ibs. 

7.  What  is  the  difference  between  a  double-acting  and  a  single-acting 
pump  ?    The  area  of  the  plunger  of  a  force  pump  being  3  square  inches, 
find  the  pressure  upon  it  when  water  is  forced  up  to  a  height  of  2of. 
Ans.  26-04  Ibs. 

8.  Describe,  with  a  sketch,  some  form  of  pump  which  will  deliver  half 
the  contents  of  the  barrel  at  each  respective  up-stroke  and  down-stroke  of 
the  pump-rod.    Name  the  valves. 

9.  Sketch  and  describe  a  "double-acting  force  pump."    If  the  diameter 
of  the  piston  be  12",  the  stroke  3',  the  distance  from  pump  to  well  2d, 
from  pump  to  position  for  delivering  the  water  40',  and  if  the  number  of 
strokes  per  minute  be  40,  what  is  (i)  the  theoretical  horse-power  required 
to  work  the  pump,  (2)  the  actual,  if  30  per  cent,  of  the  power  be  spent 
against  friction.     Ans.  (i)  1071  ;  (2)  15-3. 

10.  What  is  the  difference  between  a  single  and  a  double  acting  pressure 
pump  ?     Sketch  in  section  a  double-acting  force  pump  for  working  at  high 
pressure,  showing  the  arrangement  of  valves,  and  indicate  of  what  material 
the  several  parts  should  be  constructed. 

11.  Sketch  and  describe  the  construction  and  action  of  some  form  of 
pump  by  which  you  could  raise  water  from  a  well  where  the  level  of  the 

Q 


240  LECTURE  XIX. — QUESTIONS. 

water  is  45  feet  below  the  surface  of  the  ground.  Explain  fully  where  you 
would  fix  the  pump,  and  give  reasons  for  the  arrangement  which  you 
propose  to  adopt. 

12.  Sketch  in  section  and  describe  the  action  of  the  ordinary  lifting 
pump.    In  such  a  pump  the  pump -rod  is  f  inch  in  diameter,  and  the  pump 
barrel  is  5  inches  in  diameter,  while  the  spout  at  which  the  water  is 
delivered  is  20  feet  above  the  surface  of  the  pump  bucket  when  the  latter 
is  at  its  lowest  point ;  what",  would  be  the  maximum  tension  on  the  pump 
rod  in  the  upstroke  of  the  pump,  neglecting  the  weight  of  the  pump  rod 
and  the  pump  bucket,  also  the  weight  of  water  below  bucket  in  suction 
pipe  (the  weight  of  a  cubic  foot  of  water  is  62*5  Ibs.)  ?    -4ns.  166*6  Ibs. 

13.  Describe  the  construction  and  action  of  an  ordinary  suction-pump 
for  raising  water  from  a  well.     If  200  gallons  of  water  are  raised  per  hour 
from  a  depth  of  20  feet,  and  if  the  efficiency  of  the  pump  is  60  per  cenk, 
wfcat  horse- power  is  being  given  to  the  pump  ?    Ans.  -034. 


LECTURE  XX 

CONTENTS.—  Bramah's  Hydraulic  Press— Bramah's  Leather  Collar  Pack 
ing— Examples  I.  II. — Large  Hydraulic  Press  for  Flanging  Boiler 
Plates — The  Hydraulic  Jack— Weem's  Compound  Screw  and  Hydraulic 
Jacfc — Example  III. — The  Hydraulic  Bear  or  Portable  Punching 
Machine — The  Hydraulic  Accumulator — Example  IV. — Questions. 

Bramah's  Hydraulic  Press. — This  useful  machine  was  in- 
vented by  Pascal,  but  he  could  not  make  the  moving  parts  water- 
tight. Bramah,  about  the  year  1 796,  discovered  a  means  by  which 
this  difficulty  was  effectually  overcome ;  and  thus  the  instrument 
has  been  handed  down  to  us  under  his  name.  As  may  be  seen 
from  the  following  figure,  it  consists  of  a  single-acting  force  pump 
in  connection  with  a  strong  cylinder  containing  a  plunger  or  ram, 
which  is  forced  outwards  from  the  cylinder  through  a  tight  collar 
by  the  pressure  of  the  water  delivered  into  the  cylinder  from  the 
force  pump. 

From  what  was  said  in  Lecture  XIX.  about  force  pumps,  we 
need  not  particularise  about  this  part  of  the  machine,  except  to 
gay  that  the  suction  and  delivery  valve  boxes  can  be  disconnected 
from  the  pump,  and  the  valve  cover-checks  removed  at  any  time 
for  the  purpose  of  examining  the  parts,  or  of  legrinding  the 
valves  into  their  seats.  The  plunger  extends  through  a  stuffing- 
box  and  gland  filled  with  hemp  packing,  and  is  guided  by  a  cen- 
trally bored  bracket  bolted  to  the  top  flange  of  the  pump.  The 
lever  fits  through  a  slot  in  this  guide-bar,  whereby  it  has  an  easy 
free  motion,  when  communicating  the  force  applied  through  it  to 
the  pump  plunger.  The  relief- valve  RV  has  a  loaded  lever,  ad- 
justed like  the  lever  safety  valve  in  Lecture  IV.,  so  as  to  rise  and 
let  the  water  escape  when  the  pressure  exceeds  a  certain  amount. 
It  may  also  be  used  for  taking  the  pressure  of  the  object  under 
compression,  or  for  lowering  the  ram  R  by  simply  lifting  the 
little  lever  and  pressing  down  the  table  T,  when  the  water  flows 
easily  from  the  cylinder,  and  out  of  DP  by  the  relief  valve.  The 
delivery  pipe  DP  is  made  of  solid  drawn  brass,  and  the  ram 
cylinder  is  carefully  rounded  at  the  bottom  end,  instead  of  being 
flat,  in  order  that  it  may  be  naturally  of  the  strongest  shape.* 

*  Lo  the  case  of  largp  cylinders  for  very  great  pressures,  the  lowtr  oz 


LECTUEE   XX. 


242 

The  guide  pillars  are  securely  bolted  to  the  base  B  by  nuts  and 
iron  wash«rs,  not  shown.  The  cup  leather  packing  CL  deserves 
special  attention,  because  it  formed  the  chief  improvement  by 


VERTICAL  SECTION  OF  A  BRAMAH  HYDRAULIC  PRESS, 
Made  in  the  Engineering  Workshop  of  The  Glasgow  Technical  College. 


INDEX  TO  PARTS. 


L  represents  Lever. 


DV  represents  Delivery  valve. 


p 

„         Pressure  on  L  at  A. 

EV 

,          Eelief  valve. 

F 

„          Fulcrum  of  L. 

DP 

,         Delivery  pipe. 

B 

„          L's  connection  with 

EC 

,          Earn  cylinder. 

plunger's    guide- 

E 

,          Earn  or  plunger. 

rod. 

CL 

,          Cup  leather  packing. 

PP 

,,          Pump  plunger. 

T 

,          Top,  table,  or  T  piece. 

Q 

,,          Keaction  or  stress 

W 

,          Weight  lifted,  or 

on  plunger  PP. 

total  pressure  on  E. 

PB 

,,          Pump  barrel. 

CO 

,          Cross  girder. 

IP 

,,          Inlet  pipe. 

GP 

,          Guide  pillars. 

SV 

,,          Suction  valve. 

BB 

,          Base  block. 

inner  end  of  the  cylinder  should  be  carefully  rounded  off,  both  inside  and 
outside.  For,  if  left  square,  or  nearly  square,  the  crystals  formed  in 
the  casting  of  the  metal  naturally  arrange  themselves  whilst  cooling  in 
such  a  manner  as  to  leave  sn  initial  stress,  and  consequent  weakness,  invit- 
ing fracture  along  the  lines  joining  the  inside  to  the  outside  corners 
of  the  cylinder  end.  The  severe  shocks  and  stresses  to  which  this  weak 
line  of  division  is  subjected  during  the  working  of  the  press  would  sooner 
or  later  force  out  the  end  of  the  cylinder,  in  the  shape  of  the  frustum  of 
a  cone,  unless  the  cylinder  had  been  made  unnecessarily  thick  and  strong 
at  the  bottom  end. 


BRAMAH'S  LEATHER  COLLAR  PACKING.  243 

Bramah  on  Pascal's  press.  It  consists  of  a  leather  collar  of  fl 
section,  placed  into  a  cavity  turned  out  of  the  neck  of  the  cylinder, 
and  kept  there  by  the  gland  of  the  cylinder  cover.  The  following 
figure  shows  an  enlarged  section  of  Bramah's  packing  suitable  for 
a  huge  press,  where  the  desired  shape  of  the  leather  collar  LC 
is  maintained  by  an  internal  brass  ring,  BR,  and  an  outside  metal 
guard  ring  GE.,  resting  on  a  bedding  of  hemp  H.  It  will  be 
observed  at  once,  from  an  inspection  of  this  figure,  that  the  water 
which  leaks  past  the  easy  fit  between  the  plunger  or  ram  R,  and 
the  cylinder  C,  presses  one  of  the  sharp  edges  of  the  leather  collar 
against  the  ram,  and  the  other  edge  against  the  side  of  the  bored 
cavity  in  the  neck  of  the  cylinder,  with  a  force  directly  propor- 
tional to  the  pressure  of  the  water  in  the  cylinder.  By  this  simple 
automatic  action,  the  greater  the  pressure  in  the  cylinder  the 
tighter  does  the  leather  collar  grip  the  ram  and  bear  on  the 
cylinder's  neck. 

Bramah's  Leather  Collar  Packing. — This  collar  is  made 
from  a  flat  piece  of  new  strong  well-tanned  leather,  thoroughly 
soaked  in  water,  and  forced  into  a  metal  mould  of  the  requisite 


ENLARGED  VIEW  OP  BRAMAH'S  LEATHER  COLLAR  FOB  A 
BIG  HYDRAULIC  PRESS. 


INDEX  TO  PARTS. 


B  represents  Ram. 
C          „          Cylinder. 
G          „          Gland  of  C. 
LC  Leather  collar. 


BR  represents  Brass  ring. 
GR          „          Guard  ring. 
H         „          Hemp  bedding. 


size  and  shape  until  it  has  assumed  the  form  of  a  \J  collar.  The 
central  or  disc  portion  of  the  leather  is  then  cut  out,  and  the  cir- 
cular edges  are  trimmed  up  sharp  in  the  bevelled  manner  shown 
by  the  above  figure. 


U5CTTJRE   XX. 

Formula  for  the  Pressure  on  the  Ram  of  a  Bramah  Press. 

—Referring  again  to  the  first  figure  in  this  Lecture,  it  will  be 
found  that  by  taking  moments  about  the  fulcrum  at  F,  we  obtain 
the  pressure  or  reaction  Q  on  the  plunger  of  the  force  pump 
Therefore,  neglecting  weight  of  lever  and  friction,  we  get  — 

PxAF-QxBF.     .-.  Q-^ 


Further,  by  Pascal's  law  for  the  transmission  of  pressure  by  liquids, 
enunciated  in  Lecture  XVII.,  we  know  that  the  statical  pressure 
Q  is  transmitted  with  undiminished  force  to  every  corresponding 
area  of  the  cross  section  of  the  ram. 

Or,          ,         .     Q  :  W  :  :  area  of  plunger  :  area  of  ram. 

.*.  W  x  area  of  plunger  =  Q  x  area  of  ram. 
Wx7rra  =  Qx7rRa 

Where  r  =  radius  of  plunger,  and  R  =  radius  of  ram,  both  in  the  same  unit^ 
Substituting  the  previous  value  for  Q,  and  dividing  each  side  of  the  equa- 
tion by  TT,  we  get-— 


Since  the  radius  of  a  circle  is  directly  proportional  to  its  diameter,  we 
may  write  the  formula  thus,  where  D  is  the  diameter  of  the  ram  and  d  the 
diameter  of  the  plunger,  both  in  the  same  unit — 

PxAF    I? 

EXAMPLE  I. — In  a  small  Bramah  press,  P=  50  Ibs.,  AF=  20  in., 
BF=  2  in.,  area  of  plunger  =  i  sq.  in.,  whilst  area  of  rani  =  14  sq. 
in.  Find  W,  neglecting  friction  and  weight  of  lever. 

ANSWER. — By  the  above  formula — 

w_PxAF    7rRa 

BF      X^r7* 
Substituting  co  x  20      IA 

values,  we  get—        W  =  ^ x  —  =  7000  Ibs. 

2  T 

EXAMPLE  II. — In  Bramah's  original  press  at  South  Kensington 
the  plunger  is  3"  in  diameter,  and  it  acts  at  a  distance  of 
6  inches  from  the  fulcrum,  which  is  at  one  end  of  a  lever  10  feet 
3  inches  long,  carrying  a  loaded  scale-pan  at  the  other  end. 
What  should  be  the  pressure  of  the  water  in  the  press  in  order  to 
lift  a  weight  of  3  cwt.  in  the  scale-pan,  neglecting  the  weight  of 
the  lever  ?  Make  a  diagram  of  the  arrangement.  (S,  and  A« 
Exam.  1892.) 


HYDRAULIC   PRESS.  245 

ANSWER.  —  Here  d  =  3  in.,  consequently  the  area  of  the  plunger 
/'x"  =    s.  in.     BF  =  6";  AF=io'  3"  =  i2"; 


P  =  3  cwt.  =  3x112  =  336  Ibs.  ;  and  we  have  to  find  the  pressure 
per  sq.  in.  on  the  ram  that  will  balance  P,  acting  with  the  stated 
advantage,  since  the  area  of  the  ram  is  not  given.  By  the  formula  — 

w_PxAFxareaof  1  sq.  in._336x  123"  x  I  sq.  i".-9811bg 
BF        area  of  plunger  6"          7  sq.  in. 

.'.  Pressure  per  sq.  inch  on  ram  of  press  =  984  Ibs. 

Large  Hydraulic  Press  for  Flanging  Boiler  Plates,  &c.  — 
As  an  example  of  the  practical  application  of  the  Bramah  press 
to  modern  boiler-making,  the  accompanying  illustration  shows 
the  form  which  it  takes  when  worked  by  a  high  -pressure  water 
supply  derived  from  a  central  accumulator,  which  may  at  the  same 
time  be  used  to  work  cranes,  punching,  riveting,  and  other  similar 
machine  tools,  in  the  same  works. 

The  operation  of  flanging,  say  the  end  tube-plates  of  the  cylin- 
drical barrel  of  a  locomotive  boiler,  is  carried  out  in  the  following 
manner  :  —  The  ram  B,  is  lowered  to  near  the  bottom  of  the 
hydraulic  cylinder  HC,  thus  leaving  room  to  place  the  boiler 
plate  (which  has  been  heated  all  round  the  outside  edge)  on  the 
movable  table  Tr  High-pressure  water  is  then  admitted  from 
the  central  accumulator  to  the  auxiliary  cylinders  ACj  thus  forc- 
ing the  side  rams  SB,  SB,,  with  their  table  Tr  and  the  plate  P, 
vertically  upwards,  until  the  upper  surface  of  the  plate  bears  hard 
against  the  bearers  B,  B,  or  internal  part  of  the  dies.  Water 
from  the  same  source  is  now  admitted  into  the  hydraulic  cylinder 
HC,  which  forces  up  the  ram  B-^  with  its  table  Tlf  supporting 
columns  SO,  SO,  and  the  external  part  of  the  dies  D,  D,  until 
the  latter  has  quietly  and  smoothly  bent  the  hot  edge  of  the 
plate  round  the  curved  corner  of  the  internal  bearer  B,  B.  The 
ram  B,  is  now  lowered,  carrying  with  it  the  table  Tx  and  dies 
D,  by  letting  out  water  from  HC,  and  then  the  table  T,,  with 
the  flanged  plate,  are  lowered  by  letting  out  water  from  AC.  The 
plate  is  removed  from  its  table,  allowed  to  cool,  placed  in  position 
in  the  barrel  of  the  boiler,  marked  off'  for  the  rivet  holes,  drilled 
and  riveted  in  the  usual  manner.  The  student  will  now  under- 
stand what  a  useful  and  powerful  servant  a  hydraulic  press  is  to 
the  engineer  in  the  hands  of  a  skilful  workman,  for  it  can  be 
made  to  do  work  in  the  manner  indicated  above  in  far  less  time, 
and  with  far  greater  certainty  of  uniformity  and  exactitude,  than 
the  boiler-smith  could  turn  out,  with  any  number  of  hammermen 
to  help  him.  It  is  fast  replacing,  the  steam-hammer  for  crossing. 
work,  and  the  steam  or  belt-driven  punching  and  riveting 


246 


LECTURE   XX. 


LARGE  HYDRAULIC  PRESS  FOR  FLANGING  BOILER  PLATES.* 


*  The  above  figure  is  a  reduced  copy  of  one  from  Prof.  Henry  Kobinson's 
book  on  "  Hydraulic  Machinery,"  published  by  Messrs.  Charles  Griffin  & 
Co.,  bnt  it  has  been  indexed  according  to  the  Author's  style  of  symbols, 
and  described  in  an  elementary  manner. 


THE    HYDRAULIC   JACK.  247 

INDEX  TO  PABTS. 

HO  represents  Hydraulic  cylinder. 

R         „  Ram  of  HC. 

0, 0         „  Columns  supporting  Y. 

Y         „  Yoke  or  cross- head. 

BB         „  Bearers  of  the  internal  die  ring. 

P  Plate  to  be  flanged. 


DD 
80 


8R 
AO 


Dished  die  or  external  die  ring. 
Supporting  columns  for  DD. 
T-pu-ce  or  movable  table  for  DD. 
T-piece  or  movable  table  for  P. 
Side  rams  for  T 


e  rams   or    r 
Auxiliary  cylind 


ers. 


machines,  the  steam  screw  and  wheel-gear  worked  cranes,  screw 
and  wheel-gear  hoists,  as  well  as  the  screw  press  for  making  up 
bales  of  goods  mencioned  in  Lecture  XV.  For  with  it,  you  can 
bring  to  bear  a  force  of  a  few  pounds  on  the  square  inch  or  as 
many  tons,  by  merely  turning  the  handle  of  a  small  cock,  and 
with  a  certainty  of  action  unattainable  by  any  other  means. 

The  Hydraulic  Jack  is  a  combined  hydraulic  press  and  force 
pump,  arranged  in  such  a  compact  form  as  to  be  readily  portable, 
and  applied  to  lifting  heavy  weights  through  short  distances. 
It  therefore  effects  the  same  objects  as  the  screw-jack  described 
in  Lecture  XV.,  but  with  less  manual  effort  or  greater  mechanical 
advantage. 

The  base  on  which  the  jack  rests  is  continued  upwards  in  the 
form  of  a  cylindrical  plunger,  so  as  to  constitute  the  ram  of 
the  hydraulic  cylinder  HC.  Along  one  side  of  this  ram  there  is 
cut  a  grooved  parallel  guide  slot  GS,  into  which  fits  a  steel  set 
pin,  screwed  through  the  centre  of  a  nipple  cast  on  the  side  of  the 
cylinder  (not  shown  in  the  drawings)  for  the  purpose  of  guiding 
the  latter  up  and  down  without  allowing  it  to  turn  round.  The 
top  of  the  ram  is  then  bolted  with  a  water-tight  cup  leather  CL, 
by  means  of  a  large  washer  and  screw-bolt. 

The  action  of  this  cup  leather  is  precisely  the  same  as  the 
leather  collar  in  the  cylinder  of  the  Bramah  press  already  de- 
scribed ;  but  it  has  only  to  be  pressed  by  the  water  in  one  direction 
— viz.,  against  the  sides  of  the  truly-bored  cast-steel  cylinder, 
instead  of  against  both  the  ram  and  the  cylinder  neck,  as  in  the 
previous  case.  The  head  11  and  upper  portion  of  the  machine  is 
of  square  section,  and  is  screwed  on  to  the  hydraulic  cylinder  in 
the  manner  shown  by  the  figure.  It  contains  a  water  reservoir 
WR,  which  may  be  filled  or  emptied  through  a  small  hole  by 
taking  out  the  screw-plug  SP.*  In  the  centre  line  of  the  head- 

•  This  screw  plug  SP  is  slackened  back  a  little  to  let  the  air  in  or  out 


248 


LECTURE   XX. 


THE  HYDRAULIC  JACK. 


THE   HYDRAULIC   JACK. 


249 


piece  there  is  placed  a  small  force  pump,  the  lower  end  of  which 
is  screwed  into  the  centre  of  the  upper  end  of  the  hydraulic  cylin- 
der. This  pump  is  worked  by  the  up-and-down  movement  of  a 
handle  placed  on  the  squared  outstanding  end  of  the  turned  crank 
shaft  CS.  To  the  centre  of  the  crank  shaft  there  is  fixed  a  crank 
0,  which  gears  with  a  slot  in  the  force-pump  plunger  P,  and  thus 
the  motion  of  the  handle  is  communicated  to  the  pump  plunger 
in  a  reduced  amount,  corresponding  to  the  inverse  ratio  of  the 
lengths  of  the  handle  and  the  crank  from  the  fulcrum  or 
centre  of  the  crank  shaft.  By  comparing  the  right-hand  section 
of  the  water  reservoir,  and  the  section  on  the  line  AB,  with  the 
vertical  left-hand  section  of  the  jack,  it  will  be  seen  where  the 
inlet  and  delivery  valves  IV  and  DV  are  situated.  On  raising 
the  pump  plunger  P,  water  is  drawn  from  WE,  into  the  lower 
end  of  the  pump  barrel  through  IV,  and  on  depressing  the 
plunger  this  water  is  forced  through  the  delivery  valve  DV  into 
the  hydraulic  cylinder,  thus  causing  a  pressure  between  the  upper 
ends  of  the  cylinder  and  the  ram,  and  thereby  forcing  the  cylinder, 
with  its  grooved  head  H,  and  footstep  S,  upwards,  and  elevating 
whatever  load  may  have  been  placed  thereon.  Both  the  inlet  and 
outlet  valves>are  of  the  kind  known  as  "  mitre  valves."  They 
have  a  chamfer  cut  on  one  or  more  parts  of  their  turned  spindles, 
so  as  to  let  the  water  in  and  out  along  these  channels.  The  yalves 
are  assisted  in  their  closing  action  by  small  spiral  springs  SS, 
bearing  in  small  cups  or  hollow  centres,  as  shown  more  clearly  in 
the  case  of  DV  by  the  enlarged  section  on  AB. 

Weems'  Compound  Screw  and  Hydraulic  Jack. — This  is  a 
jack  combining  some  of  the  advantages  of  the  ordinary  screw-jack 
with  those  of  the  hydraulic  one.  It  is  often  desirable  to  be  able 
to  bring  the  head  or  footstep  into  trial  contact  with  the  load 
before  applying  the  water  pressure.  This  can  easily  be  done  by 
turning  the  nut  at  the  foot  of  the  screw,  cut  on  the  ram  of  the 
jack.  The  arrangement  will  at  once  be  understood  from  the 
figure.  It  will  be  observed  that  the  load  may  also  be  lowered  by 
turning  this  nut,  or  by  the  screw-tap  which  permits  water  to  flow 
from  the  cylinder  back  into  the  cistern,  as  in  the  previous  case. 
The  bottom  nut  may  be  screwed  hard  up  to  the  foot  of  the 
hydraulic  cylinder,  so  as  to  sustain  the  whole  load,  and  thus 
prevent  overhauling  through  leakage  of  the  water. 

When  it  is  necessary  to  lower  the  load  or  the  head  of  the  jack, 

of  the  top  of  the  water  reservoir  when  working  the  jack.  There  is  gene- 
rally another  and  separate  screw  plug  opening  (as  will  be  seen  by  the 
following  figure  of  Weems'  patent  jack)  for  filling  or  emptying  the  water 
reservoir,  quite  independent  of  the  above-mentioned  one,  which  is  used  in 
tfci»  caj»e  lor  both  purposes. 


250 


LECTURE   XX. 

the  relief  valve  or  lowering  screw,  is 
turned  so  as  to  permit  the  water  to 
escape  from  the  hydraulic  cylinder 


back  into  the  water  reservoir,  as 
clearly  shown  by  the  drawing. 
This  may  be  done  very  gently  by 
simply  giving  this  screw  a  very 
small  part  of  a  complete  turn  ; 
in  other  words,  by  throttling  the 
passage  between  the  hydraulic 
cylinder  and  the  water  reservoir. 
Or  it  may  be  done  quickly  by 
turning  it  through  one  or  more 
revolutions.  This  passage  can 
then  be  closed  by  screwing  the 
plug  home  on  its  seat. 

Mr.  Croydon  Marks,  in  his 
book  on  "  Hydraulic  Machinery," 
illustrates  and  describes  another 
method  of  lowering  the  jack-head 
(first  introduced  by  Mr.  Butters, 
of  the  Royal  Arsenal,  Woolwich), 
where,  by  a  particular  arrange- 
ment, the  inlet  and  delivery  valves 
are  acted  upon  by  an  extra  de- 
pression of  the  handle,  and  conse- 
quent movement  of  the  pump 
plunger.  He  also  gives  the  main 
dimensions,  with  a  drawing,  of 
the  standard  4-  ton  pattern  as  used 
by  the  British  Government,  where 
the  ram  has  a  diameter  D  =  2", 
the  pump  plunger  a  diameter 
d  =  i"  ;  and  the  ratio  of  the  lever- 
age of  the  handle  to  the  crank  is 
1  6  to  i.  Therefore  from  the 
previous  formula  we  find  that, 

The  Theoretical  Advantage  = 


WBEMS'  COMPOUND  SCREW  AND 
HYDRAULIC  JACK. 


WAF 


_ 

~  i  Xi2 


THE   HYDEAUUC   BKAB.  251 

And  he  instances  two  trials  by  Mr.  W.  Anderson,  the  Inspector- 
general  of  Ordnance  Factories,  to  determine  the  efficiency  of  these 
jacks,  where,  with  a  pressure  on  the  end  of  the  working  handle  of 
76  Ibs.,  the  theoretical  load  should  have  been  76  Ibs.  x  theoretical 
ad  vantage  =  76x64  =  4864  Ibs.,  instead  of  which  it  was  only 
3738  Ibs.; 

•••        .        .    4864  Ibs.  :  3738  Ibs.  :  100  :  x 


Or,        .        .         x=  =  77  %  efficiency 


In  a  second  trial,  a  load  of  1064  ^DS-  required  a  pressure  of 
22  Ibs.  on  the  handle,  and  consequently  the  efficiency  at  this 
lighter  load,  as  might  be  expected,  was  less,  or  only  74  %  . 

EXAMPLE  III.  —  With  a  hydraulic  jack  of  the  dimensions  given 
above,  and  of  77  %  efficiency,  it  is  desired  to  lift  a  load  of 
4  tons  ;  what  force  must  be  applied  to  the  lever  handle  ? 

ANSWER.  —  By  the  previous  theoretical  formula, 

w  _  P   x   AF      D> 

-IF"   '"? 

.  p  _  W  x  BF       # 

~sr~  "E5 

p_4X   224°  x   I   X^ 
16  2 

But   the   efficiency  of  the  machine  is  only   77%:   consequently 
140  Ibs.  is  77  per  cent,  of  the  force  required  — 

.•.  77  :  100  ::  140  Ibs.  ixlba. 

x=I40XIOO=  181-81  Ibs. 

77 

The  Hydraulic  Bear,  or  Portable  Punching  Machine.  — 
This  is  another  very  useful  application  of  the  hydraulic  press  and 
force  pump.  It  is  used  in  every  iron  or  steel  shipbuilding-yard 
and  bridge-building  works.  By  comparing  the  drawing  with  the 
index  to  parts,  and  taking  into  consideration  the  fact  that  its 
construction  and  action  are  so  very  similar  to  the  hydraulic  jack 
already  described  in  full  detail,  we  need  say  nothing  more  than 
direct  the  student's  attention  to  the  action  of  the  raising  cam, 
and  to  the  means  by  which  the  apparatus  is  lifted  and  suspended. 
In  order  to  raise  the  punch  for  the  admittance  of  a  plate  between 
it  and  the  die  D,  the  relief  valve  B-Y  must  first  be  turned  back- 
wards, and  the  lever  L  depressed.  This  causes  the  corner  of  the 
raising  cam  RC  to  force  the  hydraulic  ram  HB  upwards,  and  the 
water  from  the  hydraulic  cylinder  HC  back  into  the  water 


LECTURE   XX. 


reservoir  \VR.  The  relief  valve  may  now  be  closed  and  the  plate 
adjusted  in  position.  Then  the  pump  lever  can  be  worked  up 
and  down  until  the  punch  P  is  forced  through  the  plate,  and  the 
punching  drops  through  the  die  D  and  the  hole  in  the  metal 
frame  MF,  on  to  the  ground,  or  into  a  pail  placed  beneath  to 
receive  it. 


SIDE  VIEW  AND  SECTION.  END  VIEW  AND  SECTION. 

THE  HYDEAULIC  BEAK,  OK  PORTABLE  PUNCHING  MACHINE. 


INDEX  TO  PARTS. 


PL  represents  Pump  lever. 
CS  Crank  shaft. 


C 

PP 
WR 
IV 
DV 
BV 


Crank. 

Pump  plunger. 
Water  reservoir. 
Inlet  valve. 
Delivery  valve. 
Belief  valve. 


HC  represents  Hydraulic  cylinder. 
CL  Cup  leather. 


HB 

BC 

L 

P 

D 

MF 


Hydraulic  ram. 
Baising  cam. 
Lever  for  BC. 
Punch. 
Die  ring. 
Metal  frame. 


The  whole  bear  is  suspended  by  a  chain  (worked  by  a  crane  or 
other  form  of  lifting  tackle)  attached  to  a  shackle,  whose  bolt 
passes  through  a  cross  hole  in  the  back  of  the  metal  frame  MF, 
just  above,  but  a  little  to  the  front  of  the  centre  of  gravity  of  the 
machine.  This  hole  and  shackle  are  not  shown  in  the  drawing, 
but  the  student  can  easily  understand  that  the  hole  would  be 
bored  a  little  above  where  the  letters  RC  appear  on  the  side  view, 


TL8   HYDRAULIC   ACCUMULATOR.  253 

and  that  the  chain  would  pass  clear  of  the  pump  Jever,  since  it 
works  well  to  the  right-hand  side  of  the  bear. 

The  Hydraulic  Accumulator. — The  demand  for  hydraulic 
power  to  work  elevators,  cranes,  swing  bridges,  dock  gates,  presses, 
punching  and  riveting  machines,  <fcc.,  being  of  an  intermittent 
nature — at  one  moment  requiring  a  full  water  supply  at  the 
maximum  pressure,  and  at  another  a  medium  quantity,  whilst  in 
many  cases  all  the  machines  may  be  idle — it  is  evident  that  if 
an  engine  with  pumps  were  devoted  to  supplying  this  demand  in 
a  direct  manner,  the  power  thereof  would  have  to  be  equal  to  the 
greatest  requirements  of  the  plant,  and  would  have  to  instantly 
answer  any  and  every  call  from  the  same.  In  the  case  of  a  low- 
pressure  supply,  as  for  lif ts,  this  difficulty  is  best  overcome  by 
placing  one  tank  in  an  elevated  position  at  the  top  of  the  hotel  or 
building  where  the  lift  is  required,  and  another  tank  below  the 
level  of  the  lowest  flat.  Then  a  small  gas  engine  working  a  two- 
or  three-throw  pump,  or  a  Worthington  duplex  steam  pump,  may 
be  used  to  elevate  the  water  more  or  less  continuously  from  the 
lower  to  the  higher  tank.  The  "  head  "  of  water  in  the  elevated 
tank  will,  if  sufficient,  work  the  lift  at  the  required  speed,  and 
the  discharged  water  from  the  hydraulic  cylinder  will  enter  the 
lower  tank,  to  be  again  sent  round  on  the  same  cycle  of  operations. 
Should  the  lift  be  stopped  for  any  considerable  time,  then  a  float 
in  the  upper  tank,  connected  by  a  rope  or  chain  with  the  shifting 
fork  for  the  belt-driven  pumps  (in  the  case  of  the  gas  engine) 
will  force  the  belt  over  on  to  the  loose  pulley,  or  shut  off  the  steam 
from  the  Worthington  pump.  And  when  the  water  falls  in  the 
upper  tank,  the  float  will  cause  a  reverse  movement  of  the  rope 
and  shift  the  belt  to  the  tight  pulley,  or  open  the  steam  valve,  and 
so  start  the  pumps.  When  the  pressures  required  are  great,  such 
as  for  cranes,  <kc.,  where  700  Ibs.  on  the  square  inch  is  considered 
a  very  medium  pressure,  an  elevated  tank  would  be  out  of  the 
question,  for  it  would  have  to  be  fully  1600  feet  high  in  order  to 
exert  this  force  and  to  overcome  friction.  Under  these  circum- 
stances recourse  is  had  to  a  very  simple  and  compact  arrangement 
called  an  accumulator,  of  which  a  lecture  diagram  is  herewith 
illustrated,  without  any  details  of  cocks  or  valves,  and  automatic 
stopping  and  starting  gear.  A  steam  engine  or  other  motor 
works  a  continuous  delivery  pump,  of  the  combined  piston  and 
plunger  type,  without  the  aid  of  an  air  vessel,  as  illustrated  by 
the  fourth  and  fifth  figures  in  Lecture  XIX.  The  water  from  the 
pump  enters  the  left-hand  branch  pipe  leading  into  the  foot  of 
the  accumulator  cylinder,  and  forces  up  the  accumulator  ram  with 
its  cross  head  or  top  T  piece,  and  the  attached  weight  or  dead 
load,  until  the  ram  has  reached  nearly  to  the  end  of  its  stroke.  Then 


254 


LECTUKE   XX. 


the  top  of  the  T  piece  or  a  projecting  bracket  on  the  side  of  the 
wrought-iron  cylinder  containing  the  dead  load,  engages  with 
and  lifts  a  small  weight  attached  to  a  chain  passing  over  a  pulley 
fixed  to  the  guide  frame  or  to  the  wall  of  accumulator  house. 
This  chain  is  connected  directly  to  the  throttle  valve  of  the  steam 

engine  supply  pipe,  or  to  the 
belt  shifting  gear  (if  the 
pump  is  driven  by  belt  gear- 
ing), and  being  provided  with 
a  counter-weight,  the  motor 
and  pump  are  automatically 
stopped  by  the  raising  of  the 
weight  and  the  chain  in  the 
accumulator  house.  Should 
the  water  which  has  been 
forced  into  the  accumulator 
cylinder  be  now  used  by  a 
crane  or  other  machine,  the 
load  on  the  ram  causes  it  to 
follow  up  and  keep  a  constant 
pressure  per  square  inch  on 
the  water.  The  starting 
weight  naturally  falls  as  the 
receding  T  piece  or  bracket 
descends,  thus  pulling  the 
starting  chain,  and  opening 
the  steam  engine  throttle 
valve,  or  shifting  the  belt 
from  the  loose  to  the  fixed 
pulley,  and  again  setting  the 
pump  to  work.  Should  the 
hydraulic  machines  be  work- 
ing continuously,  the&  the 
pump  is  kept  going,  for  the 

C4/1  111  UJtti.        V;  V  illlUCi        \J±.  VVlWlltlllU  n  •,  •»•  -| 

iron  and  suspended  from  the    water  from  li  P^ses  directly 
top  of  T-piece  or  crosshead.       on  to  the  machines,  and  only 

the  surplus  water  finds  its 
way  into  the  accumulator  cylinder  if  the  pump's  supply  exceeds 
the  demand  of  the  machines  for  water. 

The  annular  cylinder  of  wrought  iron  is  generally  filled  with 
scrap  iron,  iron  slag,  or  sand,  or  other  inexpensive  weighty 
material.  The  accumulator  cylinder  AC  has  a  stuffing- box  and 
gland  at  its  upper  end.  A  coil  of  hemp  woven  into  a  firm  rect- 
angular section  and  smeared  with  white  lead  is  placed  in  the 
bottom  of  the  stuffing-box.  The  gland  is  screwed  down  on  the  top 


THE  HYDRAULIC  ACCUMULATOR. 

INDEX  TO  PARTS. 
AC  for  Accumulator  cylinder. 


AP 
W 


Accumulator  plunger  or  ram. 
Weight  or  load  contained  in  an 
annular  cylinder  of   wronght 


THE  HYDRAULIC  ACCUMULATOR.          255 

of  this  packing  until  the  normal  pressure  of  the  water  in  the 
cylinder  cannot  leak  past  it.  Cup  leather  packing  is  seldom  used 
for  this  simple  form  of  accumulator  ;  just  the  ordinary  packing 
that  would  be  used  for  pump  rods  is  found  to  answer  all  re- 
quirements. This  is  the  simplest  form  of  accumulator  which 
we  have  described,  but  it  requires  the  greatest  load  for  a  certain 
hydraulic  pressure  per  square  inch.  There  are  several  other  forms 
of  accumulators,  and  several  most  interesting  appliances  such  as 
capstans,  cranes,  bridges,  punching  and  riveting  machines,  &c., 
are  worked  by  them,  which  we  would  have  liked  to  have  described 
here,  but  the  limits  of  our  space  and  the  complexity  of  tbeii 
construction  necessitate  our  deferring  this  pleasure  to  our  Ad 
vanced  Course. 

EXAMPLE  IY.  —  Describe  and  sketch  in  section  a  hydraulic 
accumulator,  showing  how  the  ram  is  kept  tight  in  the  cylinder. 
A  hydraulic  press,  having  a  ram  16  inches  in  diameter,  is  in  con- 
nection with  an  accumulator  which  has  a  ram  8  inches  in  diameter 
and  is  loaded  with  50  tons  of  ballast  ;  what  is  the  total  pressure 
on  the  ram  of  the  press  ?  (S.  and  A.  Exam.  1892.) 

ANSWER.  —  The  first  part  of  the  question  is  answered  by  the 
previous  figure  and  by  the  text. 

By  Pascal's  Law  the  pressure  per  square  inch  in  the  accumulator 
is  equal  to  the  pressure  per  square  inch  in  the  hydraulic  press. 
Consequently  — 

Total  Pressure  on  Press  Cross  Area  of  Press 

Total  Load  on  Accumulator     Cross  Area  of  Accumulator 


qo  x  16  x  16 
'*'  P=        8x8  -  =  20°  tona 


256  LECTTJRE   XX. — QUESTIONS. 


LECTURE  XX. — QUESTIONS. 

1.  Draw  a  section  through  a  hydrostatic  press,  showing  the  cylinder, 
ram,  and  force  pump,  together  with  the  valves.     Why  is  the  base  of  the 
cylinder  of  a  large  press  rounded  instead  of  being  flat  as  in  a  steam 
cylinder  ?     If  the  diameter  of  the  ram  is  9  times  that  of  the  force  pump, 
and  if  Q  be  the  pressure  on  the  pump,  what  is  the  pressure  exerted  by  the 
ram,  neglecting  friction  ?    Ans.  81  Q. 

2.  Explain  by  aid  of  a  sketch  the  mode  of  packing  the  ram  of  a  hydraulic 
press  and  explain  how  it  acts.     The  force  which  actuates  the  force  pump 
is  applied  at  the  end  of  a  lever  giving  a  mechanical  advantage  of  14  to  i, 
and  the  area  of  the  plunger  of  the  pnmp  is  i  square  inch.     What  pressu/e 
must  be  applied  to  the  end  of  the  lever  to  produce  a  pressure  of  i  ton  per 
square  inch  on  the  water  enclosed  in  the  press  ?    Ans,  160  Ibs. 

3.  In  the  force  pump  of  a  press  the  area  of  the  plunger  is  £  of  a  square 
inch,  the  distance  from  the  fulcrum  of  the  lever  handle  to  the  plunger  is 
2  inches,  and  the  distance  from  the  fulcrum  to  the  other  end  of  the  lever 
is  2  feet  ;  what  pressure  per  square  inch  is  exerted  on  the  water  under- 
neath the  plunger,  when  a  weight  of  20  Ibs.  is  hung  at  the  end  of  the  lever 
handle  ?    Ans.  720  Ibs.  per  square  inch. 

4.  In  what  way  do  you  estimate  the  theoretical  advantage  gained  by  the 
use  of  the  hydraulic  press  ?    In  a  small  press  the  ram  is  2  inches  and  the 
plunger  ^  inch  in  diameter ;  the  length  of  the  lever  handle  is  2  feet,  and 
the  distance  from  the  fulcrum  to  the  plunger  is   i£  inches.     Find  the 
pressure  exerted  on  the  ram  when  10  Ibs.  is  hung  at  the  end  of  the  lever. 
Ans.  2560  Ibs. 

5.  In  an  hydraulic  press  with  two  pumps  the  plungers  are  2^  and  i  inch 
in  diameter,  and  each  is  worked  by  a  similar  lever,  which  is  acted  on  by 
the  same  force.     When  the  larger  pump  alone  is  at  work  the  pressure  on 
the  ram  is  40  tons  ;  what  will  it  be  when  the  smaller  plunger  is  only  work- 
ing ?    Ans.  250  tons. 

6.  An  hydraulic  press,  which  is  used  for  making  lead  pipes,  has  a  ram 
20  inches  in  diameter,  while  the  ram  which  presses  the  lead  is  5  inches  in 
diameter.     Find  the  pressure  per  square  inch  on  the  lead  when  the 
hydraulic  gauge  indicates   i  ton  per  square  inch.     Sketch  a  sectional 
elevation   of  the  press,  and  show  the  packing  of    the  hydraulic  ram. 
An*.  1 6  tons. 

7.  How  is  the  pressure  taken  off  the  object  under  compression  when 
required,  in  a  hydraulic  press  1     Sketch  the  arrangement.     What  is  the 
proportion  of  the  diameters  of  the  plunger  and  ram  when  the  theoretical 
advantage  gained  thereby  is  TOO  to  i,  neglecting  friction  ?    Ans.  i  to  10. 

8.  Make  a  rough  sketch,  and  write  a  short  description  of  the  hydraulic 
lifting  jack.     It  may  be  arranged  on  any  system  that  you  are  acquainted 
with.     Show  clearly  how  the  valves  act  and  how  the  jack  is  lowered. 

9.  Sketch  and  describe  the  hydraulic  bear  or  portable  punching  machine. 
Explain  how  the  punch  is  raised  and  how  the  tool  is  handled. 

10.  Sketch  and  describe  the  construction  of  a  vessel  suitable  for  storing 
up  a  supply  of  water  under  pressure,  and  intended  for  actuating  hydraulic 
machinery.     If  the  plunger  of  this  vessel  be  17  inches  in  diameter,  what 
load  will  bring  the  pressure  of  the  water  to  700  Ibs.  per  square  inch  f 
JLns.  1 58,950  Ibs. 

11.  Sketch  and  describe  the  hydraulic  accumulator  for  storing  up  water 


LECTURE  XX. — QUESTIONS. 

under  pressure.     If  the  ram  of  the  accumulator  be  6  inches  in  diameter, 
what  load  will  be  required  to  produce  a  water  pressure  of  500  Ibs.  on  the 
square  inch  ?    To  what  head  of  water  would  this  pressure  correspond  ! 
Ans.   14, 142-8  Ibs.  and  1152  feet. 

12.  A  hydraulic  accumulator,  with  a  ram  of  16  inches  in  diameter,  is 
connected  with  a  hydraulic  press  whose  ram   is  26  inches  in  diameter. 
The  load  on   the  accumulator  is  80  tons  ;  what  force  would  the  press 
exert  ?    Make  a  vertical  section   through  the  accumulator,  showing  its 
construction.     Ans.  21 1*25  tons. 

13.  Make  a  sectional  sketch  of  a  hydrostatic  press  suitable  for  giving 
a  pressure  of  100  tons,  showing  the  valves  and  pump  and  by  what  con- 
trivance the  leakage  of  water  is  prevented. 

The  pump  for  such  a  press  has  a  cylindrical  plunger  I  inch  in  diameter 
with  a  lever  of  10  to  I,  what  should  be  the  least  diameter  of  the  ram 
which  would  give  100  tons  pressure  when  a  force  of  56  Ibs.  was  applied  at 
the  end  of  the  pump  lever  ?  What  form  is  most  suitable  for  the  base  of 
the  ram  cylinder,  and  for  what  reason  is  a  special  form  adopted  ? 

Ans.  20  inches. 

14.  Sketch  and  describe  any  tool  used  by  riveters  and  worked  by  water 
pressure.     (S.  E.  B.  1902.) 

15.  The  pressure  of  water  in  a  hydraulic  company's  main  is  750  Ibs.  per 
square  inch,  and  the  average  flow  is  25  cubic  feet  per  minute.     What 
horse-power  does  this  represent  ?     If  the  charge  for  the  water  is  twopence 
per  100  gallons,  what  is  the  cost  per  horse-power  hour  ?     (S.  E.  B.  1902.) 
Ans.  8r8;  2'3d. 

1 6.  Distinguish  between  the  velocity  ra1  io  and  the  mechanical  advantage 
of  a  machine.     In  a  hydraulic  lifting  jack  the  ram  is  6"  in  diameter,  the 
pump  plunger  is  $"  diameter,  the  leverage  for  working  the  pump  is  io  to  I. 
What  is  the  velocity  ratio  of  the  machine  ?    Experimentally  we  find  that 
a  force  of  23  Ibs.  applied  at  the  end  of  the  lever  lifts  a  weight  of  8500  Ibs. 
on  the  and  of  the  raui.  What  is  the  mechanical  advantage  of  the  machine  ? 

Ans.  470;  425. 

17.  A  hydraulic  crane  is  supplied  with  water  at  a  pressure  of  700  Ibs. 
per  square  inch,  and  uses  2  cubic  feet  of  water  in  order  to  lift  4  tons 
through  a  height  of  12  feet.     How  much  energy  has  been  supplied  to  the 
crane  ?  and  how  much  has  been  converted  into  useful  work  ? 

Ans.  201,600  ft. -Ibs.  ;  107,520  ft.-lbs. 

1 8.  Sketch  and  describe  the  construction  and  working  of  any  hydraulic 
accumulator  with  which  you  are  acquainted.     If  an  accumulator  has  a  ram 
20"  diameter  with  a  lift  of  15',  and  the  gross  weight  of  the  load  lifted  is 
130  tons,  what  is  the  pressure  of  water  per  square  inch  and  the  maximum 
energy  in  ft.  -Ibs.  stored  in  the  accumulator,  neglecting  friction  7     (S.  E.  B. 
1900.)     Ans.  (i)  927  Ibs.     (2)  4,368,000  ft.-lbs. 

19.  A  single-acting  hydraulic  engine  has  three  rams,  each  of  3  inches 
diameter  ;  common  crank  3  inches  long  ;  pressure  of  water  above  that  of 
exhaust  100  Ibs.  per  square  inch  ;  100  revolutions  per  minute  ;  no  slip  of 
water.    What  is  the  horse-power?    If  this  engine  does  2-15  horse-power 
usefully  by  means  of  a  rope,  what   is  the  efficiency?    (S.  E.  B.   1901.) 
Ans.  3 '2  horse-power,  and  efficiency  "67. 

20.  Water  at  a  pressure  of  700  Ibs.  per  square  inch  is  supplied  to  a 
hydraulic  crane,  and  n  cubic  feet  are  used  in  lifting  15  tons  through  a 
height  of  1 8  feet.     How  much  energy  has  been  given  to  the  crane  ?    How 
much  energy  has  been  wasted?  (B.  of  E.  1903.) 
Ans.  Energy  given  to  crane =  i,  ioS,8co  ft.-lbs. 

Energy  which  is  wasted  =  504,000  ft.-lbs. 


258  LECTURE  XX. — QUESTIONS. 

21.  The  ram  of  a  hydraulic  accumulator  is  4  inches  in  diameter  ;  wfiat 
is  the  total  weight  of  the  ram  and  the  load  upon  it  in  Ibs.   if  the  desired 
water  pressure  in  the  accumulator  is  i^  tons  per  square  inch,  neglecting 
friction?     If,  owing  to  the  friction  of  the  ram  against  the  cup  leathers, 
5  per  cent,  of  the  load  is  wasted,  what  load  would  be  necessary  to  produce 
the  required  pressure  ?  (B.  of  E.  1905.) 

Ans.  (i.)  18-85  tons  ;(  ii.)  19-8  tons. 

22.  Convert  a  Horse-Power-Hour  into  foot-pounds  per  minute. 

If  water  under  the  pressure  of  700  Ibs.  per  sq.  inch  acts  upon  a 
piston  or  ram  I  square  foot  in  area  through  a  distance  of  I  foot,  what 
work  is  done  ?  Vv  hat  work  is  done  per  gallon  of  water  ? 

If  the  Hydraulic  Company  charges  1 8  pence  for  1000  gallons  of 
such  water,  how  much  is  this  per  horse-power-hour  ?  (B.  of  E.  1904.) 
.  i   H.P.  =  33,000  ft. -Ibs.  per  minute.  M  H. P. -hour  =  33,000  x  60 
ft. -Ibs.     Work  done  on  ram  =  100,800  ft.  -Ibs.     Work  done  per  gallon 
of  water=  16,128  ft. -Ibs.,  and  cost  per  H.-P.-hour=2'23  pence. 


259 


LECTURE  XXI. 

CONTENTS. — Morton  and  Velocity — Uniform,  Variable,  Linear,  and  Angular 
Velocity — Unit  of  Velocity — Acceleration — Unit  of  Acceleration — 
Acceleration  due  to  Gravity — Graphic  Representation  of  Velocities — 
Composition  and  Resolution  of  Velocities — Newton's  Laws  of  Motion 
— Formulae  for  Falling  Bodies— Formulae  for  Linear  Velocity— with 
Uniform  Acceleration — Atwood's  Machine  with  Experiments — Results 
and  Formulae— Galileo's  and  Kater's  Pendulum  Experiments — The 
Path  of  a  Projected  Body — Centrifugal  Force  due  to  Motion  in  a  Circle 
—Centrifugal  Force  Machine— Experiments  I.  II.  III. — Example  I. — 
Balancing  High-speed  Machinery — Centrifugal  Stress  in  the  Arms  of 
a  Fly- wheel -Exam  pie  1 1. — Enetgy— Potential  Energy— KineticEnergy 
— Accumulated  Work — Accumulated  Work  in  a  Rotating  Body — The 
Fly-wheel— Radius  of  Gyration— Example  III.— The  Fly  Press- 
Example  IV. — The  Energy  Stored  in  a  Rotating  Fly-wheel — Motion 
on  Bicycle  and  Railway  Curves — Momentum — Examples  VI.  to  IX. — 
Questions. 

Motion  and  Velocity. — (i)  Motion  is  the  opposite  of  rest,  foi 
it  signifies  change  of  position. 

(2)  Velocity  is  the  rate   at  which  a   body  move?!,   or  rate  of 
motion.     It  is  considered  absolute  when  it  is  measured  from  some 
fixed  point,  and  relative  if  it  refers  to  another  body  in  motion  at 
the  same  time. 

(3)  Uniform  Velocity  takes  place  when  the  rate  of  motion  does 
not  change — i.e.,  when  the  body  moves  over  equal  distances  in 
equal  times. 

(4)  Variable   Velocity  takes   place   when   the   rate  of   motion 
changes — i.e.,  when  a  body  moves  with  either  a  constantly  in- 
creasing or  decreasing  velocity.     For  example,  a  stone  pitched 
into  the  air  rises  with  a  gradually  decreasing  velocity,  but  falls 
with  a  gradually  increasing  rate  of  motion.. 

(5)  The  Unit  of  Velocity  is  the  velocity  of  a  body  which  moves 
through  unit  distance  in  unit  time.     The  British  unit  of  velocity 
is  therefore  i  foot  in  i  second.     In  physical  problems  velocity  is 
generally  expressed   in  feet  per  second,  but  for  convenience  the 
engineer  reckons  the  piston  speed  of  engines  in  feet  per  minute, 
and  the  public  speak  of  the  speed  of  a  man  walking,  of  a  horse 
trotting,  or  of  a  train,  in  miles  per  hour. 

(6)  Linear  Velocity  is  the  rate  of  motion  in  a  straight  line,  and 
is  measured,   as  we  have  just  stated,   in  feet  per  second  or  per 
minute,  or  in  miles  per  hour. 

If  v  =  the  velocity ;  I  =  the  distance  ;  and  t  =  the  time. 

I  I 

Then  0  =  7;  or  I  =  vt ;  or  t  =  -• 


26O  LECTURE   XXI 

(7)  Angular  Velocity  is  the  rate  at  which  a  body  describes  an 
angle  about  a  given  point — for  example,  the  number  of  revolu- 
tions per  minute  of  a  pulley ;  but  angular  velocity  may  also  be 
measured  by  the  feet  per  second  or  per  minute  which  a  point  at 
a  known  distance  from  the  centre  of  motion  moves. 

(8)  Acceleration. — In  the  case  of  variable  velocity,  the  rate  of 
change  of  the  velocity  is  termed  the  ace  deration,  and  may  be  either 
positive  or  negative — i.e.,  it  may  be  an  increasing  or  a  decreasing 
rate. 

(9)  The  Unit  of  Acceleration  is  that  acceleration  which  imparts 
unit  change  of  velocity  to  a  body  in  unit  time  ;  or  in  this  country 
it  is  an  acceleration  of  i  foot  per  second  in  one  second. 

(10)  The  Acceleration  due  to  Gravity  is  considerably  greater 
than  the  above  unit,  and  varies  at  different  places  on  the  earth's 
surface.     At  Greenwich  it  is  32-2  feet  per  second  in  one  second. 
In  Elementary  Applied  Mechanics  questions  we  will  indicate  it  by 
the  symbol  g,  and  consider  #  =  32  feet  per  second  in  one  second. 

Graphic  Representation  of  Velocities. — The  linear  velocity 
of  a  point  (such  as  the  e.g.  of  a  body)  may  be  represented  in  the 
same  way  as  we  have  hitherto  represented  a  force.  A  line  drawn 
from  a  point  with  an  arrow-head  indicates  the  direction  of  motion, 
and  the  length  of  the  line  to  scale  the  magnitude  of  the  velocity. 
(See  p.  3,  Lecture  I.) 

Composition  and  Resolution  of  Velocities. — Velocities 
may  be  compounded  and  resolved  in  exactly  the  same  way  as  we 
treated  forces  by  the  parallelogram  and  triangle  of  forces,  &c., 
in  Lecture  VIII. 

Newton's  Laws  of  Motion. — I.  A  body  in  motion,  and  not 
acted  on  by  any  external  force,  will  continue  to  move  in  a  straight 
line  and  with  uniform  velocity. 

II.  WJien  a  force  acts  upon  a  body  in  motion,  the  change  produced 
in  the  quantity  of  motion  is  the  same,  both  in  magnitude  and 
direction,  as  if  the  force  acted  on  the  body  at  rest. 

The  change  in  the  quantity  of  motion  is  therefore  proportional  to 
the  force  applied,  and  takes  place  in  the  direction  of  that  force* 

III.  If  two  bodies  mutually  act  upon  each  other,  the  quantities  of 
motion  developed  in  each  in  the  same  time  are  equal  and  opposite* 

Or,  Action  and  reaction  are  equal  and  opposite. 

These  three  laws  were  first  stated  clearly  by  Sir  Isaac  Newton 
as  the  result  of  inductive  reasoning.  Having  observed  certain 
facts,  he  set  about  investigating  what  would  be  the  consequence 
if  his  conjectures  as  to  these  facts  were  applied  to  particular 

•  Here  *  quantity  of  motion  "  means  "  momentum,"  or  mafas  x  Telocity, 
.  • .  Quantity  of  motion  or  momentum  =  Wv/g. 


LINEAR  VELOCITY.  26  1 

cases.  Finding  that  his  estimate  of  the  probable  result  came 
true,  he  formulated  a  general  law  in  accordance  with  his  observa- 
tions and  reasonings. 

The  student  has  already  conceived  the  truth  of  the  first  and 
third  laws  in  the  reasonings  and  applications  of  force  to  matter, 
treated  of  in  the  previous  Lectures.  We  will  now  give  in  as 
brief  a  form  as  possible  the  formulae  for  falling  bodies,  because 
they  naturally  lead  on  to  the  formulae  for  "  centrifugal  force  "  on 
a  rotating  body,  and  to  the  "  energy  stored  "  up  in  a  moving  body, 
both  of  which  are  of  great  interest  and  importance  to  the  young 
engineer.  The  experimental  and  algebraical  proofs  of  these 
formulae  are  given  in  most  books  on  Theoretical  Mechanics,  so 
that  we  might  assume  that  the  student  had  studied  these,  yet  the 
following  will  be  refreshing. 

FornrulSB  for  Falling  Bodies.  —  If  a  body  falls  freely  in  vacua 
under  the  action  of  gravity  from  rest  through  a  height  h  feet  ; 
then  (since  gravity  produces  a  constant  acceleration  in  the  velocity 
of  the  body)  at  the  end  of  each  successive  second  the  velocity  of 
the  body  will  be  increased  by  gr,  or  32  feet.  Let  v  be  the  velocity 
of  the  body  at  the  end  of  t  seconds, 

Then,      .         .         t?—  gt;  but  tf—2gh 


2g 

* 


And,       .         .          t-2   -^-    /!* 

9          9       V    g 


"Formulae  for  Linear  Velocity  with  Uniform  Accelera- 
tion. —  Suppose  that  instead  of  the  uniform  accelerating  force  of 
gravity  we  have  any  other  constant  force  of  P  Ibs.  acting  on  a 
body,  and  if  this  force  moves  the  body  through  a  distance  of  ~l  feet 
along  a  perfectly  smooth  /horizontal  plane,  the  above  formulas 
naturally  become*  — 

Then,     .         .         v=*at;  but  i?  —  2al 


And, 


*  We  intentionally  use  the  letter  I  for  length  or  distance,  and  a  fbi 
acceleration.  Most  writers  nse  the  word  "  space  "  for  distance  and  the 
symbol  s;  but  space  is  of  three  dimensions,  and  involves  the  idea  ol 
volume.  It  cannot  therefore  be,  strictly  speaking,  used  to  represent  dis- 
tance or  length,  which  is  only  of  one  dimension.  The  letter /is  also  often 
used  for  acceleration ;  but/  naturally  represents  a  force,  so  we  prefer  to 
use,  a,  for  acceleration,  in  order  to  be  consistent  with  our  notation. 


262  LECTURE  XXI. 

Atwood's  Machine.  *  General  Description.  —  This  machine  is 
much  used  in  Physical  Laboratories  by  teachers  and  students,  to  prove 
by  experiment  the  previously  mentioned  Laws  and  Formulae  for  the  rela- 
tion between  time,  distance,  velocity  and  acceleration.  The  results  have 
many  important  applications  to  practical  mechanics  which  will  bedealt  with 
later  on  in  this  Manual,  and  in  the  author's  more  advanced  text-book  on 
Applied  Mechanics,  vol.  ii. 

In  the  latest  form,  (see  fig.  p.  263)  it  is  provided  with  both  vertical  and 
horizontal  adjustments  for  the  scale  and  plumb-bob,  and  may  be  fixed  to 
the  wall  of  a  class-room  by  its  brackets.  The  graduated  brass  upright  is 
8  feet  long,  and  carries  on  its  upper  end  a  light,  accurately  adjusted  and 
balanced  aluminium  wheel,  which  runs  freely  in  agate  bearings  to  minimise 
the  friction.  The  inertia  of  this  wheel  is  equivalent  to  that  of  a  known 
mass  situated  at  its  circumferencef.  Two  sliding  platforms,  one  of  which 
is  a  sliding  ring;  afine,  strong, flexible  silk  thread ;  cylindrical  weights  with 
circular  overweights  of  the  same  diameter  Oi  and  riders  —  r>—  are  also  pro- 
vided. 

The  release  of  the  left-hand  weight  is  gently  effected  by  aid  of  a 
pneumatic  pipe  and  ball,  as  shown  at  the  foot  of  the  figure.  This  permits 
the  other  right-hand  weight  with  its  rider  — O—  to  start  falling  freely 
without  either  jerk,  swing  or  vibration.  A  correction  for  the  friction  of 
the  wheel  bearings  may  be  made  by  putting  a  little  fine  wii'e  on  the 
descending  weight  when  two  equal  cylindrical  weights  are  used,  until  no 
slowing  down  is  observable  after  setting  these  weights  in  motion.  The 
weight  of  the  cord  or  thread  passing  over  the  pulley  may  be  balanced  by 
a  length  of  the  same  kind  and  size,  attached*  to  the  bottoms  of  the  right 
and  left  weights  ;  or,  by  hanging  therefrom  loose,  vertical  lengths  of  such 
thread.  Either  of  these  plans  enables  the  experimenter  to  raise  and  lower 
the  weights  into  position  for  adjustment  or  starting  an  experiment. 

Method  of  Usiny  the  Machine. — If  the  two  weights  attached  to  the  ends 
of  the  cord  are  equal,  then  no  motion  will  ensue,  since  the  downward  force 
of  gravity  acting  upon  one  weight  balances  its  downward  force  upon  the 
other  through  their  tensions  on  the  connecting  thread.  The  balanced 
wheel  is  unaffected  by  the  action  of  gravity.  But,  if  the  right-hand 
weight  be  made  heavier  than  the  left-hand  weight,  by  placing  a  very  small 
wire  weight  to  balance  the  friction  of  the  wheel  bearings,  plus  a  rider  — o— 
upon  it,  then,  it  will  be  set  in  motion,  solely  due  to  the  action  of  gravity 
upon  the  rider,  until  the  rider  is  caught  by  the  ring.  The  ring,  although 
intercepting  the  rider,  permits  the  weight  with  the  little  wire  weight  to 
pass  clear  through  it  without  touching.  The  stop-stand  may  be  clamped 
at  any  desired  position  on  the  vertical  scale. 

Now,  referring  to  the  two  following  figures,  let  us  suppose  that  only  a 
rider  —  ^—  be  placed  on  the  right-hand  weight  whose  bottom  is  level  with  the 

*  George  Atwood  w.is  born  in  1745,  educated  at  Cambiidge,  where  he 
became  a  Fellow  and  Tutor  of  Trinity  College.  He  published  a  few 
treatises  ou  Mechanics  and  Engineering,  and  died  in  1807.  The  author 
first  experimented  with  the  old  Atwood's  Machine  in  1868-9,  made  by 
Professor  Copeland  in  1796,  which  belonged  to  the  Natural  Philosophy 
Department  of  Aberdeen  University,  and  again  with  an  identically  similar 
machine  at  the  College  of  Science  and  Arts,  Glasgow,  from  1880-87.  He 
had  it  fitted  with  automatic  electrical  time  starting  and  recording 
apparatus. 

t  This  mass  may  be  arrived  at  by  trial  as  explained  under  the  following 
case  VI. 


IMPROVED  AT  WOOD'S  MACHINE. 


263 


LINE  DIAGRAM  TO  REPRESENT 
THE   DISTANCES,    TIMES,   AND 
THE  VELOCITIES  OF  THE  RIGHT- 
HAND  FALLING  WEIGHT. 


WEIGHTS,  RIDERS,  AND 
OVERWEIGHTS. 


IMPROVED  ATWOOD'S  MACHINE. 
By  E.  E.  Becker  of  London. 


264  LECTURE  XXI. 

zero  of  scale  and  top  level  with  A  on  the  line  diagram.  Then  if  th« 
ring  intercepts  this  rider  at  B  one  second  after  the  weights  have  begun 
moving,  the  motion  during  that  time  has  been  uniformly  accelerated, 
and  the  velocity  is  represented  to  scale  by  line  BE.*  But,  when 
the  right-hand  weight  passes  the  ring,  its  motion  will  thereafter  be 
uniform  until  its  base  is  arrested  by  the  stop-stand  with  the  velocity 
which  it  had  acquired  when  its  top  just  passed  through  the  ring,  viz.,  CD 
which  is  equal  to  BE.  We  may  thus  find  the  velocity  gained  in  the  first 
second,  and,  by  changing  the  mass  of  the  weights  and  riders,  the  positions 
of  the  ring  and  of  the  stop-stand,  we  can  try  the  following  and  other  ex- 
periments, wherein  the  rider  may  be  replaced  by  one  or  other  of  the 
circular  overweights  which  pass  clear  through  the  ring,  and  cause  uniform 
acceleration  until  they  come  to  the  stop-stand. 

Noting  the  Times  in  Seconds. — These  may  be  done  by  aid  of  a  simple 
pendulum  beating  distinct  seconds,  or  a  laboratory  clock  or  a  split  seconds 
stop-watch.  It  is  simplest  and  best  to  start  the  right-hand  weight  with 
its  lower  surface  at  zero  upon  the  beat  of  the  sixtieth  second  of  a  minute  ; 
to  so  set  the  ring  that  it  catches  the  rider  at  an  exact  number  of  seconds 
from  the  start,  and  that  the  base  of  the  right-hand  weight  is  arrested  by 
the  stop-stand  S,  at  an  exact  number  of  seconds  from  the  time  of 
starting.  This  saves  any  confusion  and  trouble,  arising  from  noting  parts 
of  a  second,  and  ensures  that  the  two  known  distances  passed  through  by 
the  lop  of  the  right-hand  weight,  viz.,  from  A  to  B  and  from  A  to  C,  are 
accomplished  in  exact  whole  seconds.  The  distance  CS  is  simply  equal  to 
the  height  of  the  right-hand  weight. 

I.  To  prove,  for  uniform  velocity,  that  the  velocity  is  equal  to  the  dis- 
tance passed  through  divided  by  the  time  ;  that  is,  v  =•  h/t ;  or,  h  =  vt. 

II.  To  prove,  that  the  distances  described  from  rest  are  proportional 
to  the  squares  of  the  times  ;  that  is,  h  oc  &. 

III.  To  prove,  that  the  distances  from  rest  during  acceleration  are  half 
those  described  in  the  same  timet  after  the  motion  has  become  uniform. 

IV.  To  prove,  that  the  acceleration  (or  increase  of  velocity  per  second) 
is  equal  to  the  velocity  (at  any  instant)  divided  by  the  time  from  rest ; 
that  is,  a  =  vjt ;  or,  v  =  at. 

V.  To  prove,  that  the  distance   moved  through  in  a  certain  time  is 
equal  to  half  the  acceleration  multiplied  by  the  square  of  the  time  ;  that 
is,  A  =  i  a  «2  ;  or,  a  =  2A/£2  ;  and  v^  =  2ah  =  2al. 

VI.  To  prove,  that  when  a  force  produces  uniformly  accelerated  motion 
in  a  body,  or  a  system  of  bodies,  then  the  acceleration  is  directly  propor- 
tional to  the  force  and  inversely  proportional  to  the  total  mass  moved ; 

•ra 

or,   a  =  -T, ;  but  F  =  M  a  =  m  g 

*  The  scientific  meaning  of  the  term  acceleration  is  an  increase  of 
velocity  per  unit  of  time.  And,  since  velocity  is  a  rate  or  distance  per 
unity  of  time,  we  see  that  acceleration  means  an  increase  of  distance 
passed  over  by  a  body  per  unit  of  time  per  unit  of  time ;  or,  an  increase  of 
distance  per  second  per  second.  Whereas,  when  traffic  managers  of  rail- 
ways or  shipping  companies  speak  or  write  of  the  speeds  of  their  trains  or 
ships  being  accelerated,  they  simply  mean  that  their  average  velocity  has 
been  increased  so  as  to  cover  their  normal  distance  between  certain 
stations  or  ports  in  less  time  than  before.  It  is  only  during  the  times  of 
getting  up  to  their  full  speed  that  the  train  or  ship  has  ( + )  or  positive 
acceleration,  and  when  slowing  down  that  ( - )  or  negative  acceleration  is 
experienced. 


EXPERIMENTS  WITH  ATWOOD'S  MACHINE.  265 

RESULTS  OF  EXPERIMENTS  WITH  AN  ATWOOD'S  MACHINE. 


No.  of 
Experiment. 

Position  of 
Ring  B  from 
zero  A  in  ins. 

Position  of 
stop-stands 
from  zero  A 
in  inches. 

Time  in  seconds 
to  ring  B  from 

rest. 

Time  in  seconds 
to  stop-stand  8 
from  rest. 

I 

If 

8| 

I 

3 

2 

7 

21 

2 

4 

3 

I5l 

47i 

3 

6 

J 

CASE  I. —  When  accelerating  force  is  removed  then   the  velocity  become* 
uniform  and  is  equal  to  the  distance  passed  through  •£•  the  time  ;  that  w, 

v  =  ~  ;  or,  h  =  v  t 

Exp.  (i)  Distance  from  ring  to  stand  =  8f  -  if  =  7  inches. 
Time  taken  to  move       „  „         =3—1=2  seconds. 

.•.  «j  =— l  =     =  3 "5"  =  velocity  acquired  in  1  second. 

Exp.  (2)  Distance  from  ricg  to  stand  =  21  —7  =  14  inches. 
Time  taken  to  move      „  „        =4  —  2=2  seconds. 

.•.  Vt  =-1  =  —  =  7"  =  velocity  acquired  in  2  seconds. 

Exp.  (3)  Distance  fron*  ring  to  stand  =  47 £  -  isf  =  3ii  inches* 
Time  taken  to  move       ,  „         =6    —    3=3  seconds. 

.•.  v3  =  —  = — s^»=  10  "5"  =  velocity  acquired  in  3  seconds. 
ts         6 

CASE  II.— The  distances  described  from  rest  are  proportional  to  the  square* 
of  the  times. 


Exp.  (i)  Distance  in  ist  second  =  if  inches  =  £  =  f  x  i,  or  : 
Exp  (2)  „          2nd      „      =7        „       =  7  =  |  x  4,  „  : 

Erp.(3)  „          3rd      „      =i5f     „       =*^=  £  x  9, 


32 


CASE  III. — The  distances  from    rest  during  acceleration  are    half  those 
described  after  the  motion  becomes  uniform,  in  the  same  time. 

Exp.  (i)  Distance  passed  through  from  rest  in  i  second  =  (If  x  1)  inches. 
Distance  after  motion   becomes  uniform  in  (3—1 

=  2  seconds)  is  8f— if =7  „ 

.  * .  Distance  in  I  second  after  motion  becomes  uniform 

=  3^  inches .     =  (If  x  2)      „ 

Exp.  (2)  Distance  passed  through  from  rest  in  2  seconds  =(7xl)        „ 
Distance  after  motion  becomes  uniform  in  (4  —  2 

=  2  seconds)  is  21-7  ...  .         .     =  14  „ 

Distance  in  2nd  second  after  motion  becomes  uniform  =(7x2)       t, 
CASE  IV. —  Uniform  acceleration  is  equal  to  velocity  -r  time  ;  or,  a  =  vjt. 
From  the  detailed  working  out  of  Case  I.  we  see,  that  the  change  of 
Telocity  =  (vx-  ve)  =  (t^-t^)  =  (t>,-»a)  =  3-5"  per  MO. 


266  LECTURE   XXI. 

Hence,  since  the  change  of  velocity  in  3  seconds  is  10-5"  ;  we  get, 
-  =  I0  5  =  3-5  =  a  the  acceleration. 

CASE  V.  —  Distance  moved  through  =  \  acceleration  X  time  2. 
Or,  h  =  i  a  *2. 

Exp.  (3)  Distance  h  moved  through  in  time  (t)  3  seconds  =  15!". 
And,  from  Case  IV.  the  acceleration  a  =  3-5"  per  sec.  per  sec. 

Hence,  by  substituting  known  values  in  the  equation  — 
w  A  =  £  a  <2. 

<e  get,    A  =  ^  x  3>5  x  32  =  I5|  inches> 

Now,  from  Case  IV.  v  =  a  t,    .  •.    v*  =  a2  £2  ;  or,    f2  =  v2/a2, 

But   from  Case  V.    h  =  £  a  £2  ...................  •.    £2  —  2A/a. 

Hence,  2A/a  =  t>2/a2. 

Or,  2  a  A  =  1>2. 

Or  generally,  t>2  =  2  a  Z  where  Z  is  the  distance  passed  through  by  a  body 
having  an  acceleration  a. 

Corollary  to  Case  V.  —  If  a  body  has  a  certain  initial  velocity  u  before  it 
becomes  uniformly  accelerated,  then  the  final  velocity  v,  will  be  the  sum 
of  the  initial  and  accelerated  velocities. 

From  the  previous  definition  of  uniform  acceleration,  viz.,  that  it  is 
the  increase  of  velocity  per  unit  of  time  from  the  commencement  of  the 
acceleration,  we  see  that  — 


u  +  a 


.  . 

The  average  or  mean  velocity  will  be  half  the  initial  and  final  velocities, 

Or     i[u  +  (u  +  a  «)]  =  u  +  \  a  t. 

And,  the  distance  I  moved  through  or  height  h  through  which  the  body 
passed  will  be  tlds  mean  velocity  multiplied  by  the  time  of  its  motion  from 
the  instant  that  the  noted  acceleration  a  began. 

That  is,  I  =  h  =  (average  velocity)  x  (time  of  motion)  j 
Or,  h  =  (u  +  %  a  t)  x  t. 
.'.  h  =  ut  +  %  a  t%. 

The  above  refers  to  positive  acceleration  or  getting  up  speed  :  but  the 
same  applies  to  negative  acceleration  or  retardation  of  a  body's  initial 
velocity  if  we  apply  the  —  sign  instead  of  the  +  sign. 

Here,     I  =  ut  -  \  a  &, 
Hence  generally,    /  =  ut  +  %  a  I2, 

And,  if  M  =  o,  or  the  body's  motion  is  started  from  rest,  we  get  as  in 
Case  IV. 


To  prove  the  above  by  the  Atwood's  machine  : 

First,  place  an  overweight  and  then  a  rider  upon  the  right-hand  weight; 
_jid  bring  lower  surface  of  the  latter  opposite  the  zero  mark  of  the  scale. 

Second,  set  the  ring  at  say  15!"  below  zero  A,  so  that  the  rider  is  caught 
at  B,  the  end  of  the  3rd  second  and  calculate  the  velocity  with  which  the 
whole  moving  mass  is  going  at  the  instant  the  rider  is  caught  by  the  ring  ; 
let  this  velocity  =  u  inches  per  second. 

Third,  note  the  time  t  in  seconds  and  the  acceleration  a  per  second  per 
second  which  the  overweight  O  now  causes  the  remainder  to  possess  in 
passing  through  any  convenient  height  h  inches  between  the  upper  edge  of 
the  ring,  and  the  upper  side  of  the  right-hand  weight  when  its  bottom  is 
arrested  by  the  stop-stand  S. 

Fourth,  check  the  average  of  an  odd  number  of  trials  by  substituting 
the  values  for  h,  u,  t  and  a  in  the  formula  — 
h  =  ut  +  i  a  t*. 


EXPERIMENTS   WITH   ATWOOD^S   alA  CHINE.  267 

E  VI.  —  When  a  constant  force  produces  uniform  acceleration  of  a 
body  or  system  of  bodies,  the  acceleration  is  proportional  to  the  force  and 
inversely  proportional  to  the  total  mass  being  moved  by  the  force. 
In  the  previous  Experiments  with  the  Atwood's  machine  : 

If       m      =-  Mass  of  the  rider  =  ^. 

g 

,,         w      =  Weight  of  the  rider  =  |  oz.  or  '5  oz. 
„         F      =  Force  produced  by  gravity  on  rider  =  mg  =  *5  oz. 
,,         M      =  Total    mass  moved  =  (right  +  left   weight  +  thread  -4- 
equivalent  mass  active  at  circumference  of  wheel  to 
overcome  its  inertia  +  wire  weight  to  balance  friction 

W  -f  to 
of  wheel  bearings  +  mass  of  rider)  =  - 

„       W       =  Weight  due  to  whole  moving  mass  M  (minus  mass  m  of 

weight  %•)  =  3  Ib.  6|  oz.  =  5475  oz. 

Then  „  W  +  w  =  Total  weight  in  motion  up  fo  position  of  ring. 
And  a      =  Acceleration  produced  on  Mass  M  by  gravity  acting  on 

m  =  3  '5  inches  or  (3*5  -f  12  feet)  per  second  per 
second. 

=  Value  of  the  acceleration  produced  by  the  force  of 
gravity  upon  a  freely  falling  body  (to  be  found  from 
the  result  of  the  previous  experiments). 

Hence,  from  the  above  statement  we  see  that  the  following  relations 
should   hold  good,  viz.  :  — 

F  F 

a=B;andy=-ir  , 

Or,  Ma  =  F  =  mg 

•   M  -  A  .  or  g  -    a  ¥  ^  a  (W   +   M)  4  g  _  a  (W  +  w) 
'ma'  m  w  -J-  g  10 

Inserting  the  values  obtained  from  the  previous  experiment*. 

We  get, 


,  =  a  =  33-*3  «.  per  se,  per  sec. 

It  has  been  proved  by  the  most  careful  experiments  carried  out 
at  Greenwich  Observatory,  that  the  force  of  gravity  there,  when  reduced 
to  sea  level,  produces  an  acceleration  of  32*1912  ft.  per  sec.  per  sec.  But, 
Aberdeen,  where  the  above  experiments  were  made,  is  nearer  the  centre 
of  gravity  of  the  earth,  the  value  may  well  be  nearer  32*2  which  is  con- 
sidered the  usual  average  value,  although  32  is  often  taken  when  only 
approximate  results  are  desired. 

Regarding  Inertia  of  the  Wheel.  —  The  value  of  the  small  weight  x  which 
forms  part  of  W  in  the  last  equation  and  which  balances  the  inertia  of 
the  aluminium  wheel,  may  be  found  from  two  experiments  producing 
two  different  accelerations  cij  and  a%  by  two  different  riders  TOJ  and  tn., 
respectively,  and  substituting  their  known  values  in  the  previous  equation, 
until  it  is  found  that  uniform  velocity  is  obtained  in  each  of  these  trials 
after  the  riders  m^  and  m^  have  been  caught  by  the  ring. 

Let,  W  =  W  +  Xy  Where,  W  =  the  combined  weight  of  the  left  and 

right-hand  equal  weights  +  the  small  wire  on 

the  latter  which  is  required  to  balance  the  friction 

of  the  axle  in  its  bearings 
And  x  =  the  weight  required  to  balance  the  inertia 

of  the  wheel. 


268  LECTURE  XXI. 

Then,  since  gravity  has  a  constant  value  v  e  get 

W+w  (W+x  +  wJ, 

9  =  a  —  ^—  •  or  in  Exp.  (i)  g  =  ax  --  ^— 

(W+X  +    Wo) 

and  in  Exp.  (2)g  -  az  --  —  — 
Hence  in  Exp.  (i)-^p1  =  (TT+  x  +^)  and  in  Exp.  (z)-~  =  (W  +  x  +  wj 

Or  in  Exp.  (i)  a;  =  -^-1--  (W  +  wj  and  in  Exp.  (2)  x  =  -^p  =  (W  +  wa) 

When  Experiments  (i)  and  (2)  give  the  same  value  for  x  under  the  above 
conditions  it  balances  the  inertia  of  the  wheel  in  both  cases. 

It  will  now  be  interesting  to  find  from  the  foregoing  data  what  should 
be  the  ratio  of  the  mass  of  a  rider  to  the  whole  mass  moved  in  order  to 
produce  an  acceleration  of  i  foot  per  second  per  second  ;  since  the 
dynamical  unit  of  force  which  is  sometimes  called  the  "BritL-h 
Absolute  Unit  of  Force  "  called  the  Poundal,  is  that  force  which,  acting 
for  i  second  upon  a  mass  of  i  lb.,  imparts  to  it  a  velocity  of  I  foot  per 
second. 

Here  M  =  (W  +  ").=  lib. 

9 
„      a  =  i  ft.  per  sec.  per  sec. 

}i    m  —  -^-,  and  w  has  to  be  found  T 

9 
From  Case  VI.  we  see  that  — 


m       a  i 

But,  since  the  masses  are  proportional  to  the  weights—  < 
We  get,  M  _  W  +  w  _  32*2 

m  w  i     * 

And,  since      (W  +  w)  =  i  lb. 

«  =  f-L-\  lb.  =  i  oz.  (nearly). 

Or,  a  rider  of  ^  lb.,  i.e.,  £  oz.  would  be  required  to  be  placed  upon  the 
right-hand  weight  of  an  Atwood's  machine  —  where  the  sum  of  all  the 
moving  weights  reckoned  as  before,  was  equivalent  to  i  lb.  —  in  order  to 
produce  an  acceleration  of  i  foot  per  second  per  second. 

Hence,  speaking  generally,  the  value  of  a  poundal  (or  so-called  absolute 

dynamical  unit  of  force)  is  equal  to  ^  or-  lb.  in  the  ordinary  engineers 

gravitation  unit  of  force  ;  and  a  force  of  i  pound  weight  is  equal  to  32  or 
g  poundals.  But,  we  shall  not  bother  the  elementary  student  in  this  book 
with  poundals,  since,  as  we  said  before,  the  gravitation  unit  of  force  or 
force  which  will  sustain  a  weight  of  i  lb.  of  matter  is  sufficiently  absolute 
for  any  particular  place  and  for  our  purpose. 

It  will  at  once  be  seen  from  the  following  proportion,  that  no  rider, 
however  heavy,  if  it  were  placed  upon  any  weight,  however  small  (whjen 
the  latter  was  balanced  against  the  force  of  gravity)  could  produce  an 
acceleration  of  32*2  feet  per  second  per  second.  For,  the  impressed 
acceleration  a  would  also  have  to  be  equal  to  g  or  32*2,  when  we  get 


a          W          32'2I 

That  is,  W  could  have  no  weight  became  u> 


GALILEO'S  AND  KATER'S  EXPERIMENTS, 


269 


The  application  of  the  foregoing  explanations,  formula?  and  reasoning 
to  ordinary  applied  mechanics  questions  will  be  seen,  when  we  come  to 
deal  with  constant  forces  acting  in  any  direction  upon  masses  of  matter 
for  a  known  time  and  producing  a  certain  acceleration  ;  such  as  the 
acceleration  produced  upon  the  piston  and  piston  rod,  &c.,  of  a  steam 
engine  by  the  pressure  of  the  steam  up  to  the  point  of  cut  off.  Or,  upon 
the  fly  wheel  due  to  the  mean  force  upon  the  crank  pin  during  so  many 
revolutions  ;  or  upon  the  starting  or  the  stopping  of  railway  trains,  centri- 
fugal machines,  dynamos,  and  many  other  prime  movers,  or  motors  where 
it  is  advisable  to  raise  their  speed  as  quickly  as  possible  to  a  permissible 
maximum  or  normal  velocity  and  to  stop  them  quickly. 

Galileo's  Experiments. — More  than  100  years  before  Atwood  was 
born,  the  famous  Italian  philosopher  Galileo  (about  1583-1630)  experi- 
mented upon  falling  bodies  of  different  kinds  and  weights  by  letting  them 
fall  simultaneously  but  separately  from  the  top  of  the  leaning  tower  at 
Pisa  and  found  that  they  reached  the  ground  at  the  same  instant,  or  from 
the  same  height  in  practically  the 
same  time.  He  also  let  spherical 
balls  roll  down  a  straight,  smooth, 
inclined  plane  (where  the  friction 
between  the  ball  and  the  plane 
was  a  minimum)  and  found  that 
the  distances  through  which  the 
ball  passed  in  successive  seconds 
varied  directly  as  the  square  of 
the  times  from  rest ;  and  that  the 
velocities  acquired  by  the  ball 
varied  directly  as  the  times  from 
rest ;  or, 

Times  in  seconds  varied  as 
I    :    2:3    :  t 

Distances  passed  varied  as 
12  :  22  :  32  :  ^gt2 

Speeds  acquired  varied  as 
i    :2    :3    :gt. 

Pendulum  Experiments. 

— Galileo  (about  1583),  Huyghens 
(about  1650-70),  and  Captain 
Kater,  an  Englishman  (about 
1813-32),  experimented  with  pen- 
dulums to  determine  not  only  the 
length  of  the  seconds  pendulum 
but  also  to  ascertain  the  value 
of  the  force  of,  and  acceleration 
due  to  gravity  at  different  places. 
The  investigation  of  -this  subject 
must  be  deferred  until  harmonic 
motion  has  been  explained.  It 
may,  however,  be  stated  that  for 
a  seconds  pendulum — 

The  length    l  =  (^~) 

=  39*13983  inches  at  Greenwich 
(reduced  to  sea  level)  where  g  = 
32'i9i2feet  persecondper  second. 


EATER'S  REYEESTBLE 


270 


LECTURE   XXI. 


Rater's  Reversible  Pendulum.— This  pendulum  consists  of  a 
vertical  bar  carrying  a  brass  weight  near  each  end,  together  with  two 
similar  boxwood  weights,  between  the  brass  weights  which  are  so  attached 
as  to  compensate  for  the  air  displacement  of  the  brass  ones.  The  pendulum 
may  be  hung  from  one  or  other  of  two  movable  steel  knife  edges  working 
on  agate  planes.  We  now  test  to  see  if  the  lower  weight  has  been  cor- 
rectlj  placed,  because,  the  time  taken  by  the  pendulum  to  perform  say  100 
swings  about  the  top  movable  steel  knife  edge  should  be  equal  to  the  time 
taken  to  perform  100  swings  about  the  other  knife  edge.  If  the  times  are 
not  quite  equal  then  one  of  the  brass  weights  or  its  movable  knife  edge 
must  be  readjusted  until  the  time  of  a  swing  when  suspended  from  either 
end  must  be  exactly  equal.  The  length  of  an  isochronous,  simple  equiva- 
lent pendulum  is  precisely  equal  to  the  final  adjusted  distance  between  the 
knife  edges.  Therefore,  knowing  the  length  of  the  pendulum  arid  its  time 
of  oscillation,  the  acceleration  due  to  gravity  can  be  f..und  from  the 

gt2 

previously  stated  formula  I  =  — -  •     With  proper  care  Kater's  pendulum  can 

be  made  to  give  a  very  accurate  result  for  the  value  of  the?  acceleration 
due  to  gravity. 

The  velocity  attained  by  a  body  sliding  down  any 
straight  inclined  plane  varies  directly  as  the  square  root 
of  the  height.* 

Let  AB  be  a  smooth  inclined  plane  of  length  I  making  an  angle  6  with 
the  horizontal,  as  in  the  figure,  Eesolve  the  weight  of  the  smooth-faced 


To  PEOVE  THAT  THE  VELOCITY  ATTAINED  BY  A  BODY  SLIDING  DOWN 
ANY  INCLINED  PLANE  DEPENDS  UPON  THE  HEIGHT  OF  THE  PLANE. 

body  W  as  represented  to  scale  by  the  dotted  vertical  line  ab,  into  two 
components,  one  cb  parallel  to  the  plane  and  the  other  ac  perpendicular 
to  it. 

Now,  the  latter  component  force  ac  has  no  effect  on  the  motion  of  the 
body,  since  its  surface  and  that  of  the  plane  are  supposed  to  be  smooth, 
and  therefore  the  former  component  cb  is  the  only  force  which  causes  the 
body  to  move  down  the  plane. 

The  magnitude  of  this  force  F  =  ab  sin  0  =  W  sin  6. 

*  It  has  been  proved  in  the  Author's  more  Advanced  Text-Book  on 
Applied  Mechanics  and  Mechanical  Engineering,  Vol.  II.,  Lecture  XXII.  > 
that  although  the  kinetic  energy  of  a  spherical  ball,  or  of  a  cylinder  of  a 
Certain  mass,  after  it  falls  or  slides  or  rolls  down  any  path  through  a 
definite  height  is  of  the  same  value  ;  yet  the  velocity  and  momentum  are  of 
different  values  for  the  cases  of  rolling  and  of  sliding. 


VELOCITY,  ETC.,  IMPARTED  TO  A  FALLING  BODY. 

Also,   it    has   been  shown  from  Case   VI.   in    connection  with  the 
Atwood's  machine,  &c..  that  the  acting  force  =  mass  moved  x  accelera- 

W 

tion  attained  by  body ;  that  is,  F  =  —  x  a 

Or,    Wsin0x# =Wa 

Further,  it  was  shown  in  connection  with  Case  V.  that  t>2  =  2  a  I. 
Now,  substituting  the  values  for  a  and  I  in  this  equation  and  figure,  we 
get v2  =  2g  sin  0  x  BA. 

But,  ||=  sin  0,  or   -?5=  BA. 

•.^      =  20sin0  x  j^-e  =  2<?BC =  2gh. 

That  is,  r2  oc  h  the  vertical  height  of  plane.    Or,  v  oo  Vh. 
(Then  multiplying  both  sides  by  W  and  dividing  them  by  zg.), 

We  get, — —  (the  kinetic  energy  or  stored  work)  =  W^P°tential>i 
2g  \  energy.  / 

The  Velocity,  the  Momentum,  and  the  Kinetic  Energy 
attained  by  a  body  in  passing  from  a  given  height  along 
any  path  are  each  respectively  the  same,  when  the  friction 
between  the  path  and  the  body  is  negligible.— The  following 
figure  illustrates  an  arrangement  to  prove,  that  the  velocity  acquired  by  a 


APPARATUS  TO  PROVE  THAT  THE  VELOCITY,  THE  MOMENTUM  AND 
THE  KINETIC  ENERGY  IMPARTED  TO  A  FALLING  BODY  is  THE 

SAME  WHICHEVER  PATH   IT  FOLLOWS,  IF  FRICTION  BE 
NEGLIGIBLE    OR   NEGLECTED. 

body  falling  freely  down  any  curved  path  is  the  same  whatever  be  the 
shape  of  the  path.  It  shows  one  straight  and  two  curved  paths.  Each 
path  is  made  of  a  smooth  glass  plate,  or  a  pair  of  parallel  polished  metal 
rods,  along  which  a  truly  spherical  billiard  ball  may  be  allowed  to  roll 
down  from  a  platform.  The  ball  when  it  reaches  the  lowest  position, 
i>  allowed  to  travel  up  an  adjustable  and  very  smooth,  plate  glass,  inclined 
plane. 

It  will  be  noticed  that  in  each  instance,  the  ball  ascends  to  the  same 
height  on  this  plane.  Consequently  we  conclude  from  the  results  of  the 
experiment  that  the  velocity,  the  momentum  and  the  kinetic  energy  (or 
energy  of  motion)  of  the  ball  when  commencing  to  ascend  this  inclined 
plane  must  be  respectively  the  same.  Further,  if  the  ball  experienced  no 
frictional  resistance  along  its  pajth,  then  it  would  run  up  the  inclined  plane 
until  it  reached  exactly  the  same  level  from  which  it  started  with  no 


272 


LECTURE  XXI. 


initial  velocity  on  the  other  side.  The  resistance  of  the  air  and  any  slight 
want  of  smoothness  in  the  paths  make  this  ideal  condition  unattainable 
in  practice.  But,  with  care,  the  results  are  sufficiently  accurate  to  prove, 
that  if  A,  the  height  from  start  to  foot  of  path,  W  the  weight  of  the 
ball,  and  g  the  acceleration  due  to  gravity  remain  constant  throughout  the 
experiment,  and  since  the  ball  ascends  along  the  incline  always  to  the 
same  height  for  a  given  angle  of  inclination,  the  kinetic  energy  attained 
at  the  foot  of  each  path  is  the  same,  for  nothing  else  could  send  it  to  the 
same  height  unless  the  kinetic  energy  ^  mvz ;  the  momentum  mv ;  and 
velocity  v  were  the  same  in  each  case.  Seeing,  however,  that  the  body 
started  from  rest  at  a  height  A  above  the  lowest  point  of  its  path,  its 
potential  energy  before  starting  =  WA,  and  since  no  energy  is  assumed  to 
have  been  lost  we  get  WA  oc  Wt>2/2£.  Hence,  v2  oc  2  gh,  or  t;2  oc  A  since 
2  g  is  a  constant,  i.e.t  v  oo  ^/h.  Consequently  under  these  circum- 
stances, the  velocity,  momentum  and  kinetic  energy  attained  by  a  body 
falling  down  or  sliding  over  any  inclined  or  curved  path  is  solely  dependent 
upon  the  vertical  height  of  the  fall  and  quite  independent  of  the  shape  or 
length  of  the  path  taken  by  the  body  when  friction  is  eliminated  or 
neglected. 

Comparison  of  Dynamical  Formulae  for  Linear  and 
Angular  motion  when  due  to  any  Force  producing  a 
constant  Acceleration. 

KINEMATIC  FORMULA. 
Or,  formulae  dealing  with  pure  motion  in  the  abstract. 

Linear,  Angular. 


v=at 


KINETIC  FORMULAE 

Or,  formulae  dealing  with  the  models  of  real  bodies  under  the  action 
of  actual  forces. 

F      _  F  _va-v1  _T.M. 

•  —    T»r/  -    ™Tir    ~ 7~~"«  •«»•••••«••••••. tt  —  — j 

(     Angular     )  _  Twisting  Moment 
(Acceleration}  "Moment  of  Inertia 
_  Twisting  moment 
"Mass  x  (radius)3 
T.M.XI 


Momentum    =    Im- 
pulse or  Force  x 

Angular    ^ 
Momentum 

time  it  acts 

or  moment 

. 

of  momen- 

tum of  the 

rotating 

1M^-F* 

body. 
....ilw5 

Kinetic  Energy  =  Stored  work. 


Or, 


fMoment^ 

i    of    r 

I  Impulse  J 


:T.M..x0 


Kinetic  Energy = Stored  Work. 


PATH   OF  A   PROJECTED   BODY.  273 

The  Path  of  a  Projected  Body  which  then  falls  under 
the  Action  of  Gravity  is  a  Parabola.— The  accompanying  illustra- 
tion shows  a  suitable  form  of  apparatus  for  observing  the  path,  which  a 
body,  projected  horizontally,  describes  under  the  combined  actions  of  the 
projecting  force  and  of  gravity.  The  whole  apparatus  should  be  firmly 
secured  to  the  wall,  and  the  ball  should  always  be  allowed  to  roll  lown 
from  the  top  of  the  concave  quadrant  "  race,"  in  order  that  the  path  traced 
on  the  blackboard  may  be  definitely  followed. 

When  a  body  has  a  certain  velocity  imparted  to  it,  such  as  a  stone 
thrown  at  an  object,  or  water  issuing  under  pressure  from  a  pipe,  or  a 
bullet  fired  from  a  rifle  in  any  direction,  other  than  that  of  a  truly  vertical 
one,  it  has  two  different  motions  imparted  to  it.  The  one  direction  and 
motion  is  the  result  of  the  initial  velocity,  while  the  other  is  due  to  the 
earth's  attractive  force,  termed  the  force  of  gravity.  In  the  same  way, 
the  ball,  when  it  just  leaves  the  lower  end  of  the  quadrant  "race,"  has 
both  a  horizontal  and  a  vertical  motion.  The  resultant  motion  is  deter- 
mined from  the  relative  values  of  these  component  motions,  by  the  prin- 
ciple of  the  parallelogram  of  velocities.  The  ball  will  not  move  along  a 
straight  line,  but  will  describe  a  parabolic  curve,  as  shown  by  the  dotted 
curved  line  in  the  blackboard  of  the  figure.  If  the  ball  had  left  the 
quadrant  with  a  velocity  in  any  direction  (other  than  that  of  a  vertical  one) 
it  would  still  have  described  a  parabolic  path. 

Now  referring  to  the  two  upper  left  and  right  figures  of  a  cone,  we  see 
that  the  parabolic  curve  BVC  is  the  outline  of  the  section  of  the  cone  GAL, 
by  a  plane  passing  through  VO  and  parallel  to  the  opposite  generating 
line  AL  of  the  cone. 

Generally  speaking,  a  parabola  is  a  curve  traced  by  a  point,  which 
moves  in  such  a  manner,  that  the  distance  from  the  point  at  any  instant 
to  the  focus  of  the  curve  is  always  equal  to  the  normal  distance  of  the 
same  point  from  the  directrix  of  the  curve. 

Directrix  and  Focus. — If  a  point  so  move  that  the  ratio  of  its  distance 
from  a  given  fixed  point,  to  its  perpendicular  distance  from  a  fixed  straight 
line  be  a  constant,  it  describes  a  conic  section,  of  which  the  fixed  straight 
line  is  termed  the  directrix  and  the  fixed  point  the  focus.  The  constant 
ratio  referred  to  is  termed  the  eccentricity,  and  its  magnitude  determines 
the  nature  of  the  conic.  Thus,  if  in  the  figure  DOD  be  the  directrix  and 
F  the  focus,  and  if  the  point  P  moves  so  that  the  ratio  of  its  distance  frocr 
if,  is  to  its  distance  PN  from  DOD,  be  a  constant,  then  P  will  trace  out  a 
conic  section  which  will  be  a  parabola,  an  ellipse  or  a  hyperbola,  according 
as  the  ratio  in  question  is  equal  to,  less  or  greater  than  unity  That  is 
as  PF  is  equal  to,  less  than  or  greater  than  PN  or  as  FV  =  ,  <or>  VO. 

Proof  of  Equation  to  the  Parabola. — In  the  right-hand  upper  figure,  let  P 
be  a  point  on  the  curve  BVC,  then  the  distance  of  P  from  the  focus  F  is 
equal  to  the  length  of  the  perpendicular  line  PN,  let  fall  from  the  same 
point  P  upon  the  directrix  line  DOD. 

Any  conic  section  made  by  a  plane,  such  as  VE,  which  cuts  the  two 
extreme  generating  lines  such  as  AG  and  AL  of  the  cone  GAL  is  called  an 
ellipse.  An  example  of  an  ellipse  is  shown  by  the  curve  BLCGB,  of 
which  F!  and  F2  are  the/oc*. 

Any  conic  section  made  by  a  plane  such  as  VH,  parallel  to  the  axis  AC 
IB  called  a  hyperbola. 

Referring  to  the  right  hand  top  figure,  the  equation  of  a  parabola,  viz,. 
ya=4ax  may  be  proved  as  follow  :— 

C; 


274 


LECTURE  XXI. 


INDEX  OF  PARTS. 

A  for  apex  of  cone. 
AQ    ,,    axis        ,, 
VC    „    section  „ 
BVC    „    parabola  at  VC. 
DOD   ,,    directrix  of  para 

bola. 

V    ,,    vertex. 
F    „    focus. 


OUTLINE  VIEW  OF  CONE 
WITH  SECTION  PLANE. 
VC  FOR  PARABOLA  ; 
VH  FOR  HYPERBOLA  ; 
AND  VE  FOR  ELLIPSE. 


PROJECTED  VIEW  OF 
CONE  SHEWING 
PARABOLIC  CURVE 
BVC,  AND  ELLIPSE 
BLCGB. 


APPARATUS  FOR  DETERMINING  THE  PATH  OF  A  BO.JY  PBOJECT£D 
HORIZONTALLY  ASD  FALLING  FREELY. 


CENTRIFUGAL  FORCE.  2/5 

By  Euclid  I. -4 7. 

FP2  =  FM2  +  MP2 

Or,  MP2  =  FP2  -  FM2 
That  is,  MP2  =  P]ST2  -  FM2 

Or,  MP2  -  (PT  +  TN)2  -  (VM  -  VF)2 

Tluit  i-,  MP2  =  2TX  x  2PT      (But   MP  =  y: 

\  Al-o  PT   =  x 
.-.       T,2  =  4aa;  I  And  TN  =  a. 

Centrifugal  Force  due  to  Motion  in  a  Circle.— EXPERI- 
MENT I. —  When  a  body — such  as  a  stone — is  attached  to  a  cord 
and  whirled  round  and  round  in  a  circle,  the  hand  experiences  a 
pull  in  the  direction  of  the  string,  which  is  in  tension  under 
the  action  of  a  force,  and  the  faster  the  body  is  moved  the 
greater  becomes  the  stress  in  the  string,  just  as  David  of  old 
must  have  felt  it  before  he  let  go  that  pebble  from  his  sung  which 
went  so  straight  for  Goliath's  brow.  The  stone  is  constantly 
tending  to  fly  off  at  a  tangent,  and  is  only  kept  moving  in  the 
<  ircular  path  by  the  reaction  pulling  it  towards  the  centre  of 
motion.  The  pull  from  the  centre  of  motion  is  called  the  centri- 
fugal or  centre-flying  force,  and  the  exactly  equal  and  opposite 
reaction  is  termed  the  centripetal  or  centre-seeking  force.  It  may 
be  proved  by  geometry  that  each  of  these  forces  is  equal  to 


EXPERIENCING  THE  EFFECT  OP  CENTRIFUGAL  FORCE. 

the  weight  of  the  body  x  the  square  of  the  velocity  4-  the 
acceleration  due  to  gravity  x  the  radius  of  the  circle  described 
by  the  body.* 

*  At  present  the  student  mast  accept  the  above  formula  as  correct. 
We  shall  have  occasion  to  deduce  the  formula  by  aid  of  geometry  in  the 
Advanced  Course.  (See  Text-Book  on  "  Applied  Mechanics  and  Mechanical 
Engineering,"  Vol.  II.,  lectures  xxii.  and  xxiii.) 


276 
Or, 


LECTURE  XXI. 


p  - 


gr 


Ibs. 


"Where  P  =  Pull  on  the  cord,  or  the  centrifugal  force  in  Ibs. 
M      W  =  Weight  of  the  body  in  Ibs. 
w       v  =  velocity  of  the  body  in  feet  per  second, 
>*       9  —  gravity's  acceleration  =  32'  per  second  in  one  second, 
M       r  =  radius  from  centre  of  motion  to  e.g.  of  body  in  feet, 

Centrifugal  Force  Machine. — General  Description. — This  machine 
enables  the  student  to  systematically  experiment  and  prove  the  laws  con- 
nected with  centrifugal  force. 

A  polished  wooden  beam  about  2  ft,  in  length  is  rotated  by  means  of 


CENTRIFUGAL  FORCE  EXPERIMENTAL  MACHINE. 

the  pulley  at  any  speed.  Attached  to  the  centre  of  this  beam  is  an 
accurately  machined  box  with  one  side  flexible.  The  box  is  filled  with 
mercury  or  coloured  alcohol.  An  upright  glass  tube  is  fitted  into  the 
centre  of  this  box,  and  exactly  in  the  axis  of  rotation,  so  that  the  height 
of  the  mercury  can  be  measured  whatever  be  the  speed  of  the  machine. 
The  height  of  the  mercury  in  the  tube  can  be  adjusted  by  means  of  a 
screw  at  the  side  of  the  box.  Fastened  to  the  centre  of  the  flexible  side 
of  the  box  is  a  long  brass  rod,  which  in  turn  is  attached  to  one  end  of  a 
flat  brass  rod,  whilst  the  other  end  of  the  rod  is  connected  to  the  end  of 
the  beam.  This  arrangement  of  supporting  the  long  brass  rod  allows  it  to 


CENTRIFUGAL  FORCE  MACHINE.  2/7 

•move  backward  and  forward  with  less  friction  than  if  it  were  made  to 
elide  on  a  bearing.  This  long  rod  is  graduated,  and  a  sliding  weight  can 
be  clamped  at  any  position  from  the  box.  Hence,  we  see  that  if  the 
centre  of  the  flexible  side  of  the  box  be  pulled  or  pushed,  the  mercury  will 
rise  or  fall  in  the  vertical  glass  tube  ;  although,  we  cannot  observe  the 
yielding  of  the  flexible  side.  With  a  fixed  vertical  scale  alongside  the 
glass  tube  it  is  easy  to  measure  the  rise  and  fall  of  the  mercury  in  the 
latter.  The  weight  shown  on  the  left-hand  side  can  be  used  for  balancing 
the  weight  on  the  movable  rod,  and  in  this  case  it  should  be  clamped  at 
the  same  distance  from  the  axis  of  the  instrument. 

Prior  Adjustments. — Before  commencing  any  experiments  it  is  necessary 
to  pull  the  end  of  the  long  movable  brass  rod  with  a  force  of  i,  2,  or  3  Ibs., 
&c.,  by  means  of  a  spring  balance,  to  note  the  respective  heights  of  the 
column  of  mercury  in  the  glass  tube.  Hereby  we  can  afterwards  tell  the 
value  of  our  scale  measurements.  We  can  also  make  a  number  of 
experiments  with  the  sliding  weights  removed  from  the  rods,  to  find  the 
centrifugal  force  of  the  remaining  parts  of  the  machine  when  run  at 
different  speeds,  in  order  that  these  readings  may  be  subtracted  from 
subsequent  observations  to  get  true  results. 

Working  of  the  Centrifugal  Machine. — One  experimenter  turns  the  handle 
of  a  separate  pulley,  which  drives  the  pulley  shown  in  the  figure  by  means 
of  a  rope.  At  the  same  time  he  keeps  his  eye  upon  the  height  of  the 
mercury  column  in  order  to  keep  the  speed  constant,  and  he  counts  the 
number  of  revolutions  per  minute  which  his  hand  makes. 

It  is  easy  with  this  instrument  to  test  the  law  which  is  usually  given, 
that  if  a  body  be  compelled  to  move  in  a  curved  path  it  exerts  a  force 
directed  outwards  from  the  centre  of  motion,  and  the  amount  in  pounds  is 
found  by  multiplying  the  mass  of  the  body  by  the  square  of  the  velocity 
in  feet  per  second,  and  dividing  by  the  radius  of  the  curved  path  we  get 

Wv^ 

the  centrifugal  force =P= —  .    Or,  if  we  multiply  the  mass  by  the  radius 

of  the  circle  and  by  the  square  of  the  angular  velocity  in  radians  per  second, 
we  get  «ir»2=P,  the  centrifugal  force  urging  the  weights  from  the  centre 
,of  motion  in  a  radial  direction. 


278  LECTURE    XXI. 

EXPERIMENT  II.—  Take  a  pail  and  half  fill  it  with  water. 
Attach  a  rope  to  the  centre  of  the  handle,  and  swing  it  round 
and  round  your  head.  The  water  does  not  fall  out,  even  if  you 
swing  it  in  a  vertical  plane,  if  the  velocity  be  sufficient  to  cause 
the  centrifugal  force  to  be  greater  than  the  force  of  gravity. 

EXAMPLE  I.  —  A  small  tin  pail,  containing  i  Ib.  of  water,  with  a 
rope  attached  to  its  handle,  is  to  be  whirled  in  a  vertical  circle. 
If  the  distance  from  the  hand  or  centre  of  motion,  to  the  surface 
of  the  water  be  2  feet,  what  is  the  least  number  of  revolutions 
per  minute  that  you  can  give  it  in  order  not  to  spill  any  of  the 
water  ? 

ANSWER.  —  Here  P  must  be  at  least  equal  to  i  Ib.,  for  W  =  i  Ib. 
and  r  =  2  feet,  whilst  g  =  32. 
By  the  formula  — 


Or,  v  =  8  x  60  =  480  ft.  per  minute. 

Now  a  circle  of  2  feet  radius  =  12*56  feet  circumference. 

480 

.*.  -  7  =  38*2  revolutions  per  minute. 
12*56 

Consequently,  if  you  whirl  the  pail  at  40  revolutions  per 
minute,  there  will  be  no  fear  of  any  water  coming  out  of  it  even 
when  it  is  upside  down  at  the  highest  part  of  the  circle. 

EXPERIMENT  III.—  Turn  a  disc  of  wood  with  a  small  barrel  on 
one  side  of  the  centre.  Fit  the  wheel  and  the  barrel  so  truly 
with  a  turned  axle  that  when  the  axle  is  supported  by  eye  hooks 
at  each  end  for  bearings,  a  cord  wound  round  the  barrel  and 
then  pulled  sharply,  will  cause  the  wheel  to  revolve  freely  at  a 
high  speed  without  vibration  or  oscillation.  Now  bore  a  hole 
through  the  disc  near  its  circumference,  and  run  in  molten  lead 
into  this  hole.  Again  spin  the  wheel  rapidly,  when  it  will  be 
found  to  hobble  to  such  an  extent  as  to  shake  itself  almost  out 
of  the  bearings. 

The  centrifugal  force  due  to  the  unbalanced  piece  of  lead 
asserts  itself  so  thoroughly  that  when  it  reaches  the  highest 
position  of  its  revolution  round  the  axis,  it  overcomes  gravity,  and 
lifts  the  whole  wheel  and  barrel  clean  out  of  the  bearings.  It 
thereby  creates  such  a  disturbance  as  to  leave  a  distinct  impres- 
sion on  the  mind  of  the  student. 

Next  bore  another  hole  through  the  disc  of  the  same  size  as  the 
former  one,  and  at  the  same  distance  from  the  axle,  but  diametri- 
cally opposite  to  the  front  hole,  and  run  in  the  same  weight  of 


BALANCING    HIGH-SPEED    MACHINERY.  2/9 

lead  into  it.  Again  spin  the  wheel,  and  it  will  be  found  to  run 
smoothly. 

This  experiment  conveys  to  the  young  engineer  a  most  useful 
lesson,  for  it  not  only  shows  him  the  effect  of  centrifugal  force 
due  to  want  of  balance,  but  it  also  gives  him  an  idea  how  to  rectify 
the  evil. 

Balancing  High-speed  Machinery.  —  All  high-speed  machi- 
nery, whether  revolving  or  reciprocating,  should  as  far  as  possible 
be  most  carefully  balanced,  in  order  to  prevent  centrifugal  force 
coming  into  play  and  creating  that  horrid  vibration  and  noise 
with  which  it  is  always  more  or  less  accompanied.  There  is 
nothing  tends  so  much  to  the  heating  of  bearings,  and  to  the  quick 
wearing  out  of  brasses  and  other  bearing  surfaces  as  unbalanced 
moving  parts  ;  besides  which,  at  very  high  velocities  they  become 
actually  dangerous,  and  have  frequently  been  known  to  cause 
destruction  to  life  and  property.* 

Centrifugal  Stress  in  the  Arms  of  a  Ply-Wheel.  —  If  the 
arms  of  a  fly-wheel  or  pulley  are  not  properly  proportioned  to 
resist  the  centrifugal  force  due  to  the  mass  of  the  revolving  rim  ; 
or,  if  tthe  casting  has  been  carelessly  cooled,  so  as  to  set  up  internal 
stresses  between  the  arms  and  the  boss  or  the  rim,  the  wheel  may 
give  way.  In  fact,  there  is  no  fly-wheel  or  pulley  made  that  would 
not  burst,  under  the  very  great  stress  of  centrifugal  force,  if  you 
only  ran  it  fast  enough.  The  student  will  observe  from  the 
formula  that  the  centrifugal  force  or  stress  in  the  arms  of  a  fly- 
wheel is  directly  proportional  to  the  square  of  the  velocity,  so 
that  by  merely  doubling  the  number  of  revolutions  per  minute 
you  quadruple  the  stress  in  the  arms,  and  if  the  speed  be  increased 
three  times,  the  stress  becomes  nine  times  as  great. 

EXAMPLE  II.  —  Each  segment  of  a  fly-wheel,  with  its  correspond- 
ing arm  to  which  it  is  attached,  weighs  1000  Ibs.,  and  the  mass 
may  be  taken  as  collected  at  a  distance  of  4  ft.  from  the  axis  of 
the  wheel.  If  each  arm  has  a  breaking  stress  of  100,000  Ibs., 
what  is  the  maximum  number  of  revolutions  per  minute  that  the 
fly-wheel  could  be  run  at  without  breaking  the  arms,  neglecting 
the  binding  strength  of  the  rim  of  the  wheel  ? 

ANSWER.  —  By  the  previous  formula  for  centrifugal  force  — 


1000    X    V2 
100,000   --=  --      .•.  tf    =12,800 

32     x  4 

*  See  Mr.  C.  A.  Matthey's  paper  and  the  discussion  on  "  The  Mechanics 
of  the  Centrifugal  Machine,"  in  the  Transactions  of  the  Institution  of 
Engineers  and  Shipbuilders  of  Scotland  for  Session  1898-99. 


28O  LECTURE  XXI. 


—  113  ft.  per  second,  fully. 

=  113  x  60  =  6780  ft.  per  minute. 


.*.  v  = 

Or,  v 
Now,  the  circumference  of  a  circle  of  4'  radius  =  25  ft. 


6780  .   ,. 

,\  —  f  —  =  271  revolutions  per  minute. 

Energy.  —  In  applied  mechanics  energy  means  the  capability 
of  doing  work.* 

Potential  Energy  is  that  form  of  energy  which  a  body  possesses 
in  virtue  of  its  position  or  its  condition.  For  example,  when  a 
body  of  10  Ibs.  is  lifted  10  ft.  high,  it  has  a  potential  energy  of 
100  ft.-lbs.  ;  for  it  takes  that  amount  of  work  to  lift  the  10  Ibs. 
through  the  10  ft.  ;  and  if  then  allowed  to  fall,  it  would  naturally 
give  out  the  same  quantity  of  work,  either  in  overcoming  friction, 
or,  if  it  fell  freely,  it  could  be  usefully  employed  to  that  amount 
and  no  more. 

Potential  energy  may  also  be  due  to  a  condition  of  a  body,  such 
as  the  potential  energy  in  the  coiled  spring  of  a  watch  or  clock, 
which  when  wound  up  does  work  in  moving  the  mechanism. 
We  have  also  the  case  of  potential  energy  in  a  lump  of  coal  which 
when  burned  gives  out  heat,  that  will  raise  steam  to  be  used  in 
a  steam  engine  for  doing  work.  Or,  in  the  case  of  an  electric 
battery,  where  plates  of  copper  and  zinc  are  respectively  placed  in 
solutions  of  sulphate  of  copper  and  zinc,  and  on  being  suitably 
connected  by  wires  to  an  electric  motor,  will  give  out  electrical 
energy,  which  may  be  converted  into  mechanical  work  by  the 
motor,  and  thereby  effect  some  useful  purpose. 

Kinetic  Energy  (E^)  is  energy  due  to  motion.  For  example,  in 
the  first  instance  of  potential  energy  the  weight  of  10  Ibs.,  in 
falling  freely  down  through  10  ft.,  had  stored  up  in  it,  due  to  its 
motion,  an  amount  of  accumulated  work  equivalent  to  100  ft.-lbs. 

Accumulated  Work  (E,).  —  If  a  body  of  weight  W  Ibs.  be 
raised  to  a  height  h  feet  above  the  earth 

The  potential  energy  stored  up  E^=»W&  (ft.-lbs.) 

Now,  if  the  body  be  allowed  to  fall  freely,  under  the  action  of 
gravity,  through  h  feet,  it  would  have  a  velocity  at  the  end  of 
time  t  seconds  of  v  feet  per  second. 

Referring  back  to  the  formulas  for  falling  bodies  previously 
given  in  this  lecture  we  see  that  — 


h  -  --  .-.  Wh  =  —  ft.-lbs. 
*g  *g 

*  We  have  specially  avoided  using  this  term  hitherto,  as  students  are 
liable  to  confuse  it  with  force,  work,  and  power. 


ACCtTMTTLATED   WORK.  281 

Therefore  the  kinetic  energy  or  accumulated  work  stored  up  in  a 
moving  body  is  expressed  by  the  formula  — 


If  a  body  of  weight  W  Ibs.  were  impressed  forward  along  a 
perfectly  smooth  plane  for  a  distance  of  I  feet,  by  a  force  F  Ibs., 
causing  an  acceleration  of,  a,  feet  per  second  ;  then  the  previous 
set  of  formula?  for  linear  velocity  would  apply  when  the  reaction 
from  the  plane  cancelled  the  force  of  gravity. 

Here,         .        .        F-«/  and  I  - 


But  the  Wvrk  Done  through  distance  I  =  F  x  I  =  E 


/. 


Therefore  in  this  case  the  accumulated  work  stored  up  in  the 
moving  body  would  be  expressed  by  the  formula  — 


Accumulated  work  in  a  Hotating  Body.  —  If  a  body  of  W 
Ibs.  be  concentrated  at  a  distance  of  r  feet  from  the  centre  of 
motion,  and  be  rotated  so  that  it  has  a  velocity  of  v  feet  per 
second,  then 


The  Accumulated  Work  =-  -  ft.  -Ibs. 
29 

The  Energy  of  a  Rotating  Ply  -wheel  is  a  good  example  of 
accumulated  work.  If  the  pressure  of  steam  in  the  cylinder  and 
the  point  of  cut-off  be  kept  constant,  and  if  one  or  other  of  the 
machines  which  are  being  driven  by  the  engine  be  thrown  out  of 
circuit  —  or,  in  other  words,  if  the  belt  be  moved  to  the  loose  pulley  — 
the  load  on  the  engine  will  be  lessened,  and  the  engine  will  have  a 
tendency  to  increase  in  speed.  If,  however,  it  be  provided  with 
a  very  heavy  fly-wheel,  the  surplus  power  of  the  engine  will  be 
stored  up  in  the  fly-wheel,  so  that  the  increase  of  speed  will  not 
be  so  great  as  if  it  had  a  light  one,  or  none  at  all.  If  a  machine 
should  be  suddenly  brought  into  circuit  again  after  a  short  time, 
then  the  load  on  the  engine  will  be  as  quickly  increased  ;  but  the 
etored-up  energy  in  the  fly-wheel  will  enable  it  to  overcome  this 
sudden  demand  for  power,  so  that  the  speed  of  the  engine  will  not 
be  greatly  altered.  The  fly-wheel,  therefore,  acts  as  a  regulator 
of  speed,  not  only  for  alterations  of  load,  but  also  for  the  variable 
pressures  which  exist  in  the  cylinder  of  an  engine.  This  ia 


282  LECTURE   XXI. 

particularly  noticeable  in  the  case  of  gas  engines,  where  the 
almost  instantaneous  explosion  of  gas  in  the  cylinder  at  the 
beginning  of  a  stroke  creates  an  immense  force,  which  would 
urge  the  piston  forward  at  lightning  speed,  if  it  were  not  for 
the  very  heavy  fly-wheel  with  which  the  engine  is  provided.  The 
fly-wheel  stores  up  some  of  this  sudden  force  and  gives  it  out 
again  during  the  intervening  strokes  when  there  is  no  explosion, 
thus  tending  to  a  uniformity  of  speed  which  would  be  conspicuous 
by  its  absence  if  the  gas  engine  had  only  a  light  fly-wheel,  or  none 
at  all.  In  fact,  the  motion  of  gas  engines  would  be  so  erratic 
without  fly-wheels  as  to  prevent  their  application  to  many  pur- 
poses for  which  they  are  admirably  adapted  when  aided  by  very 
heavy  ones. 

Radius  of  Gyration.* — It  will  be  evident,  almost  without  ex- 
planation, that  in  the  case  of  a  fly-wheel  or  a  rotating  disc,  those 
parts  which  are  furthest  from  the  centre  of  motion  must  accu- 
mulate more  energy  than  those  of  the  same  weight  which  are 
nearer  to  that  centre,  because  they  move  at  a  greater  velocity. 
There  is,  however,  for  every  body  a  mean  radius  of  rotation,  termed 
" radius  of  gyration"  (k)  which  is  at  such  a  distance  from  the 
centre  of  motion,  that  if  the  whole  mass  of  the  body  were  concen- 
trated there,  the  same  kinetic  energy  or  accumulated  work  would 
be  developed  at  the  same  speed  or  number  of  revolutions  per 
minute.  The  length  of  this  mean  radius  varies  with  the  shape 
of  the  rotating  body,  and  requires  a  knowledge  of  higher  mathe- 
matics for  its  computation ;  so  we  will  assume  that  in  the  case  of 
a  fly-wheel  it  is  at  the  e.g.  of  the  rim,  or  that  the  distance  is  given 
in  any  question  requiring  solution. 

EXAMPLE  III. — A  fly-wheel  weighing  10,000  Ibs.  has  a  mean 
radius  of  rotation,  k=*r=  5  feet,  and  turns  normally  at  100  revolu- 
tions per  minute.  Owing  to  the  load  being  diminished,  the  speed 
increases  to  no  revolutions  per  minute  ;  what  reserve  energy  is 
stored  up  in  the  fly-wheel,  which  is  fit  to  overcome  any  sudden 
increase  of  load  ? 

ANSWER. — Let  vl  =  the  velocity  in  feet  per  second,  at  the  nor- 
mal speed  n^  revolutions  per  minute, 

And  v2  =  the  velocity  at  the  increased  speed  na  revo- 
lutions per  minute ; 

2  x  22  x  q  x  100 

Vj  =  271-7*^!  = ^ =  52*4  ft.  per  sec. 

7  x6o 

= 2 =  57*6  ft.  per  sec. 


5  The  radius  of  gyration  is  called  the  swing  radius  by  some  engineers. 


THE  FLY-PRESS: 

Stored  energy  at  speed  n^  = L 


r>  7  j 

Reserved  stored  energy  =  -  —  2  _ 
2g 


W 


2          , 

'  -  v 


=  80,375  ft -Ibs. 

The  Ply  press. — This  machine  is  used,  in  the  form   shown  by 
the  figure,  either  for  embossing  or  stamping  pieces  of  metal  with 


THE  FLY-PRESS. 
INDEX   TO   PARTS. 


D  represents  Disc  supporting  M. 
M          „          Metal  to  be  stamped. 

„          Punch  or  die. 
F  Frame  of  machine. 


S  represents  Screw. 
N          „          Nut  for  S. 
L          „          Lever  arms. 
B          „          Balls  or  weights. 


some  design,  or  for  punching  thin  metal  plates.  The  piece  of 
met :il  M,  to  be  embossed  or  punched,  is  laid  on  a  disc  D,  and  the 
die  or  punch  P  is  caused  to  come  down  on  M  with  a  large 
amount  of  stored-up  energy,  due  to  the  operator  taking  hold  of 


284  LECTURE    XXI. 

one  or  other  of  the  heavy  balls  B,  and  giving  them  a  very  rapid 
turn  round.  The  result  of  this  movement  is  to  send  the  quickly 
pitched  square-double-threaded  screw  rapidly  through  its  nut  N, 
thereby  forcing  the  guided  square  carrying  the  punch  straight 
downwards,  and  causing  the  latter  to  overcome  the  resistance  of 
the  hard  metal.  Neglecting  friction  at  the  screw  and  the  guide, 
and  considering  the  combined  weight  of  the  two  balls  as  =  W  Ibs., 
and  v  =  their  velocity  in  feet  per  second  at  the  instant  the  punch 
meets  the  metal  M,  then  — 

The  stored  energy,  or  energy  of  the  blow,  =  —  -  ft.-lbs. 

If    .    1=  Length  the  punch  or  die  goes  into  the  metal  in  feet, 
And  E,  =  Ilesiotance  overcome  (mean)  in  Ibs., 


Then      .....  1^  =          ft.-lbs. 

2g 

EXAMPLE  IV.  —  Distinguish  between  energy  and  power.  What 
is  the  unit  of  power  in  this  country  ?  In  a  fly-press  two  balls, 
each  weighing  60  Ibs.,  are  moving  with  a  linear  velocity  of  15 
feet  per  second,  what  is  the  measure  of  the  energy  existing  in  the 
balls  (take  ^  =  32)  ?  What  is  the  power  required  to  raise  6600 
gallons  of  water  up  150  feet  in  30  minutes!  A  gallon  of  water 
weighs  10  Ibs.  (S.  and  A.  Exam.  1893.) 

ANSWER.  —  (i)  Energy  is  the  capability  of  doing  work  which  a 
body  may  possess  on  account  of  its  position,  or  condition,  or 
motion.  Power  is  the  rate  of  doing  work,  or  the  work  done  in  a 
given  time.  The  unit  of  power  in  this  country  is  the  horse-power 
and  is  the  rate  of  doing  work  equivalent  to  33,000  ft.-lbs.  per 
minute. 

(2)  Here  W  =  combined  weight  of  the  two  balls  =120  Ibs. 
v  =  linear  velocity  of  balls  =  15  ft.  per  sec. 

Then  energy  existing  in  balls  =  — 

421-87  ft.-lb8. 


2x32 
(3)  Weight  of  water  raised          6600  x  10 

per  minute  ~^  --  =  220°  lbs- 

.".  Work  done  per  minute  =  2200  x  150'  =  330,000  ft.-lbs. 
,".  Power  required  «  33°>000  _  10 


ENERGY  STORED   IN  A   ROTATING   BODY. 


285 


To    find    Experiment  ally    the    Energy    Stored    in    the 

Rotating  Mass  Of  a  Fly-wheel.— Description  of  the  Experimental 
Machine. — The  accompanying  figure  shows  a  fly-wheel  mounted  on  an  axle. 
This  axle  is  supported  on  ball  bearings,  thereby  reducing  the  friction 
to  a  minimum.  The  diameter  of  the  fly-wheel  is  18  inches  and  weighs 
about  100  Ibs.  If  the  centre  of  gravity  of  the  fly-wheel  is  not  exactly  in 
the  axis,  then  it  is  better  to  place  the  wheel  as  in  the  figure.  One  end  of 
a  cord  is  looped  over  a  pin  on  the  axle,  and  after  being  wound  several 
times  round  the  axle,  the  other  end  is  led  over  an  aluminium  pivoted 
pulley,  and  attached  to  the  7-lb.  weight. 

Object  of  Experiments. — The  object  of  this  experiment  is  to  illustrate 


,    EXPERIMENTAL  APPARATUS  FOE  DETERMINING  THE  ENERGY 
STORED  UP  IN  A  ROTATING  FLY-WHEEL. 

the  "Principle  of  Work"  or  the  law  of  the  "Conservation  of  Energy." 
Since  energy  cannot  be  generated  or  destroyed,  the  quantity  given  to  a 
machine  can  be  traced  in  its  transmission  through  the  machine,  as  clearly 
pointed  out  in  Lecture  V.  Hence  the  amount  of  energy  given  to  the  fly- 
wheel and  the  attached  weight  in  this  experiment  is  measured  by  the  pull 
of  the  earth  on  the  7-lb. weight  multiplied  by  the  distance  through  which 
the  latter  falls. 

Conservation  of  Energy. — Part  of  this  energy  is  stored  up  in  the  fly-wheel 
as  kinetic  energy,  part  of  it  is  used  to  turn  the  wheel  against  the  friction 
of  the  bearings,  and  part  is  stored  as  kinetic  energy  in  the  falling  weight. 
The  last  two  items  of  the  total  energy  are  converted  into  heat  energy,  and 


286 


LECTURE   XXI. 


gt  neral  formula. 

fEnergy  given  to^ 
the  apparatus 
•!       during  tha 

[  Kinetic  } 
energy  E* 
-  =  •!    stored 

•  +- 

Energy  " 
converted 
into  heat 

falling  of  the 
[        weight.         , 

in  the 
^fly-  wheel.; 

by 

friction. 

the  several  sub-divisions  of  the  total  energy  may  be  connected  under  one 

f  Kinetic  energy  in 
falling  weight  at 
1  the  instant  when 
I  the  cord  is  released 
(^  from  the  axle. 

Determination  of  Speed  of  Machine  by  the  Fly-wheel. — After  setting  the 
mark  upon  the  rim  of  the  fly-wheel  opposite  the  fixed  mark,  and  with  the 
7-lb.  weight  on  the  cord  to  keep  it  taut,  turn  the  wheel  by  hand  to  see  as 
near  as  possible  how  many  turns  the  wheel  makes  in  falling  a  certain 
number  of  feet  Let  this  distance  through  which  the  weight  is  to  fall  in 
the  experiment  be  h  feet,  and  let  n  be  the  number  of  turns  made  by  the 
fly-wheel  during  this  fall.  Now  determine  by  aid  of  a  stop-watch  how  long 
it  takes  the  7-lb:  weight  to  fall  through  h  feet  whilst  turning  the  fly-wheel, 
and  tabulate  your  results  as  follows  : 


Weight 

Height 

Turns 

Time  taken 

Velocity  of 

Revolutions 

on  cord 
in  Ibs. 

of  fall 
in  feet 

of  fly- 
wheel (n) 

foi   weight 
to  fall  (h) 

weight  at  the 
instant  when 

per  second  of    1 
fly-wheel  at 

feet 

cord  leaves  the 

the  instant 

axle  in  feet 

when  cord 

per  second 

leaves  the  axle. 

Final  Calculations. — As  we  have  found  the  total  number  of  revolutions 
made  by  the  wheel  until  the  cord  drops  off  its  pin,  and  also  observed  the 
time  taken  in  seconds.  Then,  as  we  know  that  the  speed  increases 
uniformly  during  tMs  interval  of  time,  the  mean  speed  is  just  half  the 
speed  at  the  end  of  the  time  interval ;  consequently,  if  we  divide  t lie  number 
of  revolutions  by  the  number  of  seconds  in  which  they  were  performed,  and 
multiply  this  quotient  by  2,  we  will  get  the  number  of  revolutions  per  second  made 
by  the  wheel  when  the  weight  just  ceases  to  act.  You  can  test  the  accuracy 
of  your  result  by  counting  the  number  of  revolutions  of  the  fly-wheel  from 
the  time  that  the  cord  drops  off  until  the  wheel  comes  to  rest  due  to  its 
own  friction  and  dividing  by  the  time  which  has  elapsed,  then  multiply 
this  quotient  by  2  and  this  will  give  you  the  required  ag«ed  of  the  fly- 
wheel. 


MOTION  ON  BICYCLE  AND  RAILWAY  CUHVES.  287 

Motion  on  a  Curved,   Inclined,   or  "Banked"  Track.— 

Take  as  an  example  a  cycling  track,  and  let  us  suppose  that  the 
bicycle  is  moving  with  uniform  velocity  of  v  feet  per  second  round  a 
smooth  circular  course  of  radius  OB  equal  to  r  feet.  Then  it  is  necessary 
to  find  at  what  angle  to  the  horizontal  plane  the  track  should  be  inclined 
or  "  banked  "  in  order  that  the  bicycle  may  keep  in  its  circular  path. 
We  see  from  the  diagram  that  two  forces  are  acting  on  the  bicycle — 


Of<- 0  B  =  Radius  r  feet 


j        Sectional  Elevation  of  Bicycle,  \  Course. 


Oj<- -0  B= -Radius  r  feel  - 


Plan  of  above 
Bicyde  Course. 


SECTIONAL  ELEVATION  AND  PLAN  OF  A  BICYCLE  TRACK. 


(i)  Its  own  weight  W  ;  and  (ii)  the  reaction  R,  which  is  perpendicular 
to  the  smooth  track.     These  two  forces  have  a  resultant  horizontal  force 


P  =    —  ,  which  is  acting  towards  the  centre  0  of  the  horizontal  circle  in 

which  the  bicycle  moves. 

Hence,  if  we  take  any  vector  line  to  represent  the  weight  W  and  from 
the  upper  end  thereof  draw  a  horizontal  line,  and  from  the  lower  end  draw 
another  vector  line  inclined  at  an  angle  6  to  the  vertical  until  it  meets  the 
horizontal  line  in  a  point.  The  sides  of  the  triangle  will  represent  in 

direction  and  magnitude  the  forces  W,  R,  and  P,  and  tan  0  =  =  = 

f  _  Li  ^rj  —v-,  from  which  we  can  quite  easily  obtain  the  required 

angle  of  the  inclination  0  of  the  track  and  the  height  h  for  the  desired 
width  of  the  same. 

Railway  Curves.  —  If  the  lines  of  a  railway  curve  be  laid  at  the 
same  level,  then  the  centripetal  thrust  of  the  rails  on  the  wheel  of  trains 
passing  round  curves  act  on  the  flanges  of  the  wheels,  and  the  centri- 
fugal thrust  of  the  wheel  on  the  track  would  tend  to  push  it  sideways  out 

T 


288  -,LECTURE  XXI. 

of  its  place.  Bat,  in  order  to  have  this  action  and  reaction  normal  to  the 
track,  the  outer  rail  is  raised  and  the  track  thereby  inclined  to  the  hori- 
zontal. The  amount  of  this  super-elevation  which  is  suitable  for  a  par- 
ticular speed  is  quite  easily  calculated. 


SECTIONAL  ELEVATION  OF  A  RAILWAY  CUEVE 

Let  G  =  gauge  in  inches  between  the  rails. 

„    v  =  velocity  in  feet  per  second  of  the  train. 

„    r  =  radius  of  curve  in  feet. 

„    h  =  height  of  super-elevation  of  one  rail  above  the  other  in  inches. 

„    6  =  the  angle  of  "  banking  "  or  inclined  plane. 
Now  draw  the  triangle  ABC  to  represent  the  three  forces,  W,  K,  and  P 
as  indicated  in  the  previous  case. 

Then  AC  =  AB  sin  0  =  AB  tan  6,  since  6  is  always  very  small. 

<y2 

h  =  G—  inches. 

gr 

EXAMPLE  V.  —  A  motor-car  moves  in  a  horizontal  circle  of  300  ft.  radius 
at  30  miles  per  hour,  what  is  the  ratio  of  its  centrifugal  force  to  its 
weight  ?  This  is  the  tangent  of  the  angle  at  which  the  track  ought  to  be 
inclined  sideways  to  the  horizontal  if  there  is  to  be  absolutely  no  tendency 
to  side-slip  ;  find  this  angle.  (B.  of  E.  1904.) 
Answer.  — 

Let  v  =  velocity  of  the  car  in  feet  per  second. 
„  P  =  centrifugal  force  in  Ibs. 
„  W  =  weight  of  the  car  in  Ibs. 

„  r  =  radius  of  horizontal  circle  of  car's  motion  in  feet. 
„  g  =  acceleration  due  to  gravity  =  32  ft.  per  sec.  per  sec. 
Then  the  forces  which  are  acting  at  the  centre  of  gravity  of  the  car 
are  as  follows  : 

(i)  The  weight  of  the  car,  W  Ibs.  acting  vertically  downwards. 


(ii)  The  centrifugal  force  P,  equal  to  —  -  Ibs.  acting  in  a  horizontal 

direction. 

(a)  To  Find  the  Ratio  of  the  Centrifugal  Force  to  the  Weight  of  the  Car.— 
Magnitude  of  centrifugal  force       P       Wv2   .  w  _  ^ 
Weight  of  the  motor-car         =  W  =  ~gr^  ~        ~  gr 

-Z      30  x  5280  x  30  x  5280 
W  ~~  60  x  60  x  60  x  60  x  32  x  300 
P      121  1 

=      =  ;  oraboufc5' 


(b)  To  find  the  inclination  of  the  angle  6  at  which  the  track  ought  to 
be  inclined  to  the  horizontal  if  there  is  to  be  absolutely  no  tendency  to 
side-slip. 


MOMENTOTL  289 

Momentum  is  the  quantity  of  motion  possessed  by  a  body. 
It  is  measured  by  the  quantity  of  moving  matter  (i.e.,  its  mass) 
multiplied  by  its  velocity.* 

Or,  Momentum  =  Mass  x  Velocity 

M          —     m     x         v 

The  momentum  of  a  body  w,  therefore,  that  constant  forct 
which,  acting  for  unit  time,  would  stop  the  body. 

A  force  of  2  Ibs.  acting  for  unit  time,  or  one  second,  on  a  body, 
•will  produce  a  certain  amount  of  momentum;  it  is  therefore 
obvious  that  twice  the  force  acting  for  half  the  time,  i.e.,  4  Ibs. 
acting  for  half  a  second,  would  produce  the  same  momentum. 

EXAMPLE  VI. — A  hammer  head  of  2\  Ibs.  moving  with  a  velocity 
of  50  feet  per  second  is  stopped  in  -ooi  second.  Find  the  average 
force  of  the  blow. 

ANSWER. — Momentum  of  Hammer  Head  =  mass  x  velocity. 

„     =  (2i/32)x  50  =  3-906  lbs.-sec. 

Average  Force  (F)  =  ma  =  —  ( ^y^1 )  =  ~f  ( 'o-ooi )  =  39°6  lbS' 

The  negative  signs  (before  va  and  50)  show  that  the  acceleration  (a)  is  in 
this  case  a  retardation.  The  momentum  3*906  represents  the  force  in  Ibs. 
which,  acting  for  one  second,  would  stop  the  hammer  head.  The  average 
force  cf  the  blow  is  therefore  (3 '906-:- '001)  =  3906  Ibs. 

EXAMPLE  VIL — A  ship  of  2000  tons,  moving  at  3  knots,  is 
stopped  in  one  minute ;  what  is  the  average  retarding  force  ? 
Neglect  the  motion  of  the  water.  One  knot  is  6080  feet  per 
hour. 

ANSWER. — Here  we  may  obtain  the  retarding  force  in  tons  and 
reduce  the  speed  in  knots  to  feet  per  second. 

Now,  3  knots  =  3  x  6080  feet  per  hour;  or, ^—        -  ft.  per  sec. 

_           ,                                            ...        2000     3  x  6080 
Hence  the  momentum  =  mass  x  velocity  «= x  ^ 7—  units. 

*  The  mass  of  a  body  is  its  weight  in  Ibs.  divided  by  the  acceleration 
due  to  gravity.  Hence,  if  a  body  is  W  Ibs.  in  weight,  and  if  g  repre- 
sents the  acceleration  due  to  gravity,  or  32  ft.  per  sec.  per  sec.  then 

TTT 

the  mass  m  =  —  .     For  example,  if  a  body  weighed  3-2  Ibs.,  then  its  mass 

is  3- 2 -7-32  or  •  i  unit  of  mass.  If  this  body  is  moving  with  a  velocity  of 
10  feet  per  second  its  momentum  is  *i  x  10  or  i.  If  this  momentum  be 
created  or  destroyed  by  a  force  acting  for  one  second  only  on  the  body, 
the  force  must  have  a  value  of  i  Ib.  If  it  be  created  or  destroyed  in  ten 
seconds,  then  the  force  is  •  i  of  a  Ib. ;  if  in  ^  second,  its  value  would  be 
10  Ibs. 


29O  LECTURE    XXI. 

r> 
Since  the  retarding  force  acts  for  one  minute,  or  sixty  seconds . 

„-•    The  average  retarding  force  =  change  of  momentum  -r  60 

2000     3  x  6080 

*•*•»  »  *  n  32    x  60  x  60 

Or,  „  „  »      =  5'27  tons. 

EXAMPLE  VIII. — A  railway  train  starting  from  rest  along  a 
level  line  acquires  a  speed  of  30  miles  per  hour  in  five  minutes. 
What  has  been  the  mean  pull  between  the  engine  and  the  train  ; 
the  resistances  to  motion  being  taken  at  10  Ibs.  per  ton,  and 
weight  of  train  exclusive  of  engine  150  tons? 

ANSWER. — Constant  pull  due  to  resistances  =10x150=1 500  Ibs. 

ico  x  2240 
Mass  of  tram  =  m  =  — j— ^--150  x  70 

Velocity  of  train*. 0  =  30  miles  per  hour  =  3°  x  S28°  feet  Por 

60  x  60    second. 

„  „      =*  v  =  44  feet  per  second. 

Momentum  of  train  =-m  x  v**- 150  x  70  x  44  =  462,000. 
And  the  time  taken  to  produce  this  momentum  =  5  x  60  seconds. 
Average  pull  for  change"!  __  (mv9  -  mvA  _  462,000  _  n 

nf  t.h  A  TvmmAnt.iim  f  ~  *~1       ~ TTTTT  ~  J54°  lbs- 


of  the  momentum          J      y         $         I        5  x  60 

.-.  Total  mean  pull  between  the)  ,, 

engine  and  carriages  j  =  15°°  +  '54°  =  3°4°  Ibs. 

Mass  is  defined  as  the  quantity  of  matter  in  a  body.  It  is 
measured  by  the  weight  of  the  body  in  pounds  at  London  divided 
by  the  acceleration  of  gravity  at  the  same  place — i.e.,  Mass  = 
t0-r-<7;  or,  w  in  Ibs.  4-  32 '2. 

Inertia  is  defined  as  that  property  of  matter  whereby  it  tends  to 
remain  in  a  condition  of  rest  or  of  uniform  motion.  Hence,  we 
have  Newton's  first  law  of  motion  as  stated  in  the  second  page 
of  this  Lecture,  and  which  is  sometimes  termed  the  "  Law  of 
Inertia." 

NOTE. — The  force  in  Ibs.  is  the  distance-rate  at  which  work  is  done  in 
foot-pounds,  and  it  is  also  the  lime-rate  at  which  momentum  is  produced 
or  destroyed. 

Moment  of  Momentum. — Unit  moment  of  momentum  or  angular 
momentum  is  unit  momentum  at  unit  perpendicular  distance. 


WORK  DONE  BY  A  VARIABLE  FORCE. 


29I 


EXAMPLE  IX.— A  body  weighing  3220  Ibs.  was  lifted  vertically  by  rope, 
there  being  a  damped  spring  balance  to  indicate  the  pulling  force  of  .Fib.  on 
the  rope.  When  the  body  had  been  lifted  x  ft.  from  its  position  of  rest, 
the  pulling  force  was  automatically  recorded  as  follows : 


X 

0 

18 

43 

60 

74 

95 

in 

130 

F 

7700 

7680 

743° 

713° 

6770 

5960 

5160 

39/0 

Find  approximately  the  work  done  on  the  body  when  it  had  risen  115  ft. 
How  much  of  this  is  stored  as  potential  and  how  much  as  kinetic 
energy  ?  What  is,  then,  the  velocity  of  the  body  ?  (S.  E.  B.  1900.) 

Answer. — Use  squared  paper,    and  plot  the  values   of   x   along   the 


8000 


00        10       20       30      40       50       60      70       80       90       100      110      120        C 


3000 


DIAGRAM  OP  WORK  DONE  BY  A  VARIABLE  FORCE. 

abscissae  OC,  with  the  corresponding  values  of  F  as  ordinates.  Then  draw 
a  curve  AB  through  the  several  points.  Dmde  OC  into,  say,  15  equal 
parts,  and  from  the  middle  point  of  each  division  read  off  upon  the  vertical 
scale  the  corresponding  values  of  F.  The  sum  of  these  ordinates  divided 
by  15  will  give  the  mean  force  F  in  Ibs.  This  force  multiplied  by  the 
distance  through  which  it  acts,  viz.,  130  ft.  gives  the  total  work  done  by 
the  force  or  the  area  of  the  diagram  OAJBC  in  ft. -Ibs. 


LECTURE  XXL 
Total  work  done  by  force    F=a,rea,  of  diagram  OABO  in  ft.  Ibs. 


Work    done    by  force  F  in\_  f  f. 

passing  through  115  ft.        J~     "  " 

="6951  xus       =799,  360  ft.  -Ibs. 

But,  work  done  in  lifting  the\_w,  -vwisonft  Ih* 

weight  W  through  h  feet  j-WA-322ox  115  -370,300  ft.-lbs. 

Hence,  work  done  by  force]  ("Work  done  in^l  C  K'netic  energy,  "\ 
Fibs,  in  passing  through  [  =  \  lift.ng  weight  j-  -f  -j  E^,  in  the  body  at  V 
115  ft.  J  [through  1  15  ft.  J  I  end  of  the  lift.  J 

Or,  799'36o     =370,300 


•£—  =  799.  36°  ~  370.300=429,060  ft.-lbs. 
,_  429,  060  xzg    429,060x64 
W  3220       ' 

^=8528. 

v=  ^8528  =  92*3  feet  per  second. 

Poten'ial  Energy  E^  in  body  at  height  of  115  ft.  =  370,  300  ft.-lbs. 
Kinetic  Energy  E^  in  body  at  height  of  115  ft.  =4=29,060  ft.-lbs. 


LECTURE   XXI.— QUESTIONS.  293 


LECTURE  XXL— QUESTION!. 

1.  A  body  moves  in  a  circle  with  a  uniform  velocity ;  show  that  It  must 
be  acted  on  by  a  constant  force  tending  towards  the  centre,  and  find  the 
magnitude  of  the  force  in  terms  of  the  radius  of  the  circle,  and  of  the  mass 
and  velocity  of  the  body. 

2.  A  body  weighing  2\  Ibs.,  fastened  to  one  end  of  a  thread  4  feet  long, 
is  swung  round  in  a  circle,  of  which  the  thread  is  the  radius ;  what  will  be 
its  velocity  when  the  tension  of  the  thread  is  a  force  of  20  Ibs.  (^=32)  ! 
Ans.  32  feet  per  second. 

3.  When  an  unbalanced  wheel  is  set  in  rapid  rotation,  a  considerable 
amount  of  shake  and  vibration  is  experienced.    You  are  required  to  explain 
this  result  from  first  principles,  and  to  state  the  mechanical  laws  which 
appear  to  be  at  work.    How  would  you  calculate  the  amount  of  pull  that 
this  unbalanced  weight  exerts  ? 

4.  What  primary  law  in  mechanics  asserts  itself  when  some  revolving 
piece  of  machinery  moves  at  a  high  velocity,  and  is  unbalanced  f    A  weight 
of  i  Ib.  is  placed  on  the  rim  of  a  wheel  2  feet  in  diameter,  which  revolves 
upon  its  axis  and  is  otherwise  balanced.     The  linear  velocity  of  the  rim 
being  30  feet  per  second,  what  is  the  pull  on  the  axis  as  caused  by  the 
weight  of  i  Ib.  1    Ans.  28*1  Ibs. 

5.  A  segment  of  a  fly-wheel,  with  the  arm  to  which  it  is  attached,  weighs 
3500  Ibs.,  and  the  mass  of  the  portion  may  be  taken  as  collected  at  a  dis- 
tance of  8  feet  from  the  axis  of  the  wheel,  which  makes  40  revolutions  per 
minute.    What  is  the  force  tending  to  pull  away  the  segment  and  arm 
from  the  boss  of  the  wheel  f    Ant.  15,365  Ibs. 

6.  Define  kinetic  energy.    How  does  it  differ  from  potential  energy  ?    If 
a  velocity  of  300  ft.  per  second  is  impressed  on  a  weight  of  10  Ibs.,  what 
is  the  measure  of  the  energy  now  imparted  to  the  weight. 

Ans.  14,062-5  ft.-lbs. 

7.  State  the  rule  for  finding  the  amount  of  work  stored  up  in  a  given 
weight  when  moving  with  a  given  velocity.    A  weight  of  6  cwt.  moves 
with  a  velocity  of  20  feet  per  second ;  how  many  units  of  work  are  stored 
up  in  it  ?     Ans.  4200  ft.-lbs. 

8.  Write  down  the  formula  for  the  amount  of  energy  stored  up  in  a  given 
weight  when  moving  with  a  given  velocity.    Describe,  with  a  sketch,  the 
action  of  a  fly-press.     If  each  ball  of  the  press  weighs  50  Ibs.,  and  the  work 
stored  up  in  the  balls  is  400  ft.-lbs.,  find  the  velocity  with  which  they  are 
moving.    Take  the  number  32  to  represent  g.    Ans.  16  feet  per  second. 

9.  Account  for  the  storing  up  of  energy  in  a  rotating  fly-wheel.     If  the 
weight  of  the  rim  be  doubled  while  the  rate  of  rotation  remains  unchanged, 
how  much  is  the  energy  increased  ?    Ans.  Twice. 

10.  State  the  formula  for  the  energy  stored  up  in  a  fly-wheel,  on  the  sup- 
position that  the  whole  of  the  material  is  collected  in  a  heavy  rim  of  given 
mean  radius.    Apply  the  formula  to  show  (i)  the  effect  of  doubling  the 
number  of  revolutions  per  minute ;  (2)  the  effect  of  doubling  the  weight ; 
(3)  the  effect  of  increasing  the  mean  radius  in  the  proportion  of  3  to  2. 

11.  A  fly-wheel  weighs  2j  tons,  and  its  mean  rim  has  a  velocity  of  40  feet 
per  second,    If  the  wheel  gives  out  10,000  ft.-lbs.  of  energy,  how  much  is 
its  velocity  diminished  t    An*,  x  -455  feet  per  second. 


294 


LECTURE  XXI. QUESTIONS. 


12.  Explain  the  use  of  the  fly-wheel  in  any  machine  with  which  yon  are 
acquainted.     To  what  class  of  machines  is  such  a  wheel  usually  applied  T 
What  is  the  kinetic  energy  in  a  wheel  revolving  at  150  revolutions  per 
minute,  if  the  wheel  loses  5000  ft.-lbs.  of  energy  when  its  speed  is  reduced 
to  147  revolutions  per  minute  ?  Ans.  126,263  ft.-lbs. 

13.  A  fly-wheel  of  a  shearing  machine  has  150,000  foot-pounds  of  kinetic 
energy  stored  in  it  when  its  speed  is  250  revolutions  per  minute ;  what 
energy  does  it  part  with  during  a  reduction  of  speed  to  200  revolutions 
per  minute  ?    Ans.  54,000  ft.-lbs. 

If  82  per  cent,  of  this  energy  given  out  is  imparted  to  the  shears  during 
a  stroke  of  2  inches,  what  is  the  average  force  due  to  this  on  the  blade  of 
the  shears  ?  (S.  E.  B.  1902.)  Ans.  265,680  Ibs. 

14.  A  fly-wheel  is  required  to  store  12,000  ft.-lbs.  of  energy  as  its  speed 
increases  from  98  to  102  revolutions  per  minute ;  what  is  its  kinetic  energy 
at  100  revolutions  per  minute  ?    (S.  E.  B.  1900.)    Ans.  150,000  ft.-lbs. 

15.  A  machine  is  found  to  have  300,000  foot-pounds  stored  in  it  as 
kinetic  energy  when  its  main  shaft  makes  100  revolutions  per  minute  ;  if 
the  speed  changes  to  98  revolutions  per  minute,  how  much  kinetic  energy 
has  it  lost  1    (S.  E.  B.  1901.)    Ans.  11,880  ft.-lbs. 

16.  What  do  you  understand  by  work,  potential  and  kinetic  energy? 
A  bullet  weighing  i  oz.  leaves  the  muzzle  of  a  rifle  with  a  velocity  of 
1350  feet  per  second  ;  what  is  the  kinetic  energy  of  the  bullet  in  ft.-lbs.  ? 

Ans.  1780  ft.-lbs. 

17.  If  a  gun  delivers  400  bullets  per  minute,  each  weighing  0*5  oz.,  with 
2000  feet  per  second  horizontal  velocity  ;   neglecting  the  momentum  of 
the  gases,  what  is  the  average  force  exerted  upon  the  gun  ?   (S.  E.  B.  1900.) 
Ans.  12*94  Ibs. 

1 8.  A  bullet  of  O'l  lb.,  with  a  speed  of  2200  feet  per  second,  is  fired 
into  the  middle  of  a  block  of  wood  of  30  Ibs.,  which  is  at  rest  but  free  to 
move ;  find  the  speed  of  the  block  and  bullet  afterwards.     What  is  the 
loss  of  kinetic  energy  in  foot-pounds?    'VS.  E.  B.  1902.)    Ans.  7-3  ft.  per 
sec. ;  7537  ft.-lbs. 

19.  A  man  and  his  bicycle  weigh  170  Ibs.  ;  he  has  a  speed  indicator  (not 
a  mere  counter).    When  going  at  10  miles  an  hour  on  a  level  road  he 
suddenly  ceases  to  pedal,  and  in  15  seconds  finds  that  his  speed  is  8  miles 
an  hour.    What  is  the  force-resisting  motion  ?    (S.  E.  B.  1901.)    Ans.  I  lb. 

20.  A  car  weighing  z\  tons  and  carrying  40  passengers,  the  average 
weight  of  each  of  them  being  145  pounds,  is  travelling  on  a  level  rail  at 
the  rate  of  6  miles  an  hour.     What  is  its  momentum  in  engineer's  units  ? 
If  the  propelling  force  be  withdrawn,  what  average  force  in  pounds  must 
be  exerted  to  bring  the  car  to  rest  in  two  seconds  ?  and  supposing  the 
force  to  be  constant,  what  distance  would  the  car  travel  before  it  came  to 
rest?  Ans.  3135  Ibs. -ft. -sees.  ;  1567-5^8.;  8*8  ft. 

21.  A  car  is  drawn  by  a  pull  of  P  Ibs.,  varying  in  the  following  way 
t  being  seconds  from  the  time  of  starting  : 


P 
t 

1020 

980 

882 

720 

702 

650 

713 

722 

805 

0 

2 

5 

8 

10 

13 

16 

19 

22 

The  retarding  force  of  friction  is  constant  and  equal  to  410  lb.  Plot 
P  — 410,  and  the  time  t,  and  find  the  time  average  of  this  excess  force. 
What  does  this  represent  when  it  is  multiplied  by  22  seconds  ?  (S.  E.  B. 
1902.)  Ans.  366  Ibs. 


LECTUEE   XXI. — QUESTIONS, 


295 


22.  A  body  weighing  1610  Ibs.  was  lifted  vertically  by  a  rope,  there  being 
a  damped  spring  balance  to  indicate  the  palling  foro«  F  Ib.  of  the  rope. 
When  the  body  had  been  lifted  x  feet  from  its  position  of  rest,  the  palling 
force  was  automatically  recorded  as  follows : 


X 

0 

ii 

20 

34 

45 

55 

66 

76 

F 

4010 

39i5 

3763 

3532 

3366 

3208 

3100 

3007 

Find  approximately  the  work  done  on  the  body  when  it  has  risen 
70  feet.  How  much  of  this  is  stored  as  potential  energy,  and  how  much 
as  kinetic  energy  1  Ans.  247,000  ft.-lbs.  ;  1 12,700  ft. -Ibs.  5134,300  ft.-lbs. 

(3.  E.B.  1901.) 

23.  A  tramcar,  weighing  15  tons,  suddenly  had  the  electric  current  cut 
off.     At  that  instant  its  velocity  was  16  miles  per  hour.     Reckoning  time 
from  that  instant,  the  following  velocities,  V,  and  times,  t,  were  noted : — 

V. — Miles  per  hour    •        16        14        12        10 
t.  — Seconds      -        -          o       9*3        21         35 
Calculate  the  average  value  of  the  retarding  force  and  find  the 
average  value  of  the  velocity  from  t  =  o  to  t  =  35.     (B.  of  E.  1903.) 
Ans.  Retarding  Force=264  Ibs.;    Average  Velocity  =1274  miles 
per  hour. 

24.  A   projectile    has    kinetic  energy  =  1,670,000  foot-pounds  at    a 
velocity  of  3000  feet  per  second.     Later  on  its  velocity  is  only  2000  feet 
per  second,  how  much  kinetic  energy  has  it  lost  ?    What  is  the  cause  of 
this  loss  of  energy  1    Ans.  927,778  ft. -pounds.  (B.  of  E.  1903.) 

25.  A  man  weighing  160  Ib.  is  in  a  lift  which  starts  to  descend  with  an 
acceleration  of  2  feet  per  second  per  second.    What  force  is  exerted  by  the 
man  upon  the  floor  of  the  lift  ?    What  would  the  force  be  if  the  lift  were 
descending  at  a  uniform  speed  ?    Ans.  150  Ibs. ;  160  Ibs.       (B.  of  E.  1903.) 

26.  In  a  gun,  of  which  the  internal  diameter  is  6  inches,  a  projectile 
weighing  100  Ib.  has  imparted  to  it  in  a  distance  of  12  feet  a  velocity  of 
2500  feet  per  second.     Find  the  average  pressure  of  the  gases  on  the  base 
of  the  projectile  up  to  the  time  it  leaves  the  gun.    (Neglect  friction  and 
the  energy  of  rotation  of  the  projectile.) 

Ans.  Average  Pressure  =  7200  Ibs.  per  sq.  inch.  (B.  of  E.  1903.) 

27.  A  weight  of  120  Ib.  falls  to  the  ground  from  a  height  of  18  feet  and 
just  rebounds.      If  the  time  of  contact  between  weight  and  ground  be 
the  energy  of  rotation  of  the  projectile.) 

Ans.  Average  Pressure  =  28, 800  Ibs.  per  sq.  inch.  (B.  of  E.  1903.) 

28.  A  train  weighing  250  tons  is  moving  at  40  miles  per  hour,  what  is  its 
momentum  in  engineers'  units  ?     If  this  momentum  is  destroyed  in  ten 
seconds,  what  is  the  average  force  acting  on  the  train  during  these  ten 
seconds.    Define  what  is  meant  by  force  by  people  who  have  to  make  exact 
calculations.     Ans.  Average  Force  =  456  tons  ;    Momentum  =  14  670  ton- 
feet  seconds  ?  (B.  of  E.  1904.) 

29.  A  fly  wheel  weighs  24,000  Ib.,  its  mean  radius  (or  rather  radius  of 
gyration)  is  10  feet,  it  revolves  at  75  revolutions  per  minute   what  is  its 
kinetic  energy  ? 

If  suddenly  disconnected  from  its  engine,  in  how  many  revolutions 
will  it  come  to  rest,  if  we  know  that  in  each  revolution  the  energy 
wasted  in  overcoming  friction  is  3000  foot-pounds  ? 
Ans.  E*  =  2,315,051  ft.-lbs.  ;  and  772  revs.     (B.  of  E.  1904.) 


296 


LECTURE   XXI. — QUESTIONS. 


30.  An  ordinary  steam  engine  has  a  stroke  of  18  ins.,  and  the  connecting 
rod  is  36  ius.  long.     The  crank  shaft  makes  400  turns  per  minute.     Find 
the  velocity  of  the  piston,  in  feet  per  minute,  when  it  has  moved  through 
one-quarter  of  the  stroke,  reckoned  from  the  back  end. 

Ans.   ¥^=1760  feet  per  minute.     (C.  &  G.,  1905,  0.,  Sec.  A.) 

31.  A  cricket  ball,  weighing  0-28  lb.f  reaches  the  batsman  when  it  is 
travelling  horizontally  at  96  feet  per  second ;  what  is  its  momentum  in 
engineers'  units  ?     The  batsman  drives  the   ball  straight  back  to  the 
bowler  with  the  same  speed ;   what  has  been  its  change  of  momentum  ? 
If  the  time  of  the  blow  is  one-thirtieth  of  a  second,  find  the  average 
magnitude  of  the  force  exerted  by  the  bat  upon  the  ball.    (B.  of  E.  1905.). 

Ans.  Change  of  momentum  =  i'68  units  ;  Average  Force  =  50*4  Ibs. 

32.  A  casting  is  bolted  to  an  angle  plate  on  the  face  plate  of  a  lathe. 
The  casting,  angle  plate,  and  bolts  are  equivalent  to  75  Ibs.  at  a  radius  of 
4^  inches.    In  what  position  must  a  weight  of  20  Ibs.  be  fixed  to  the  face 
plate  to  effect  a  balance  1    Ans.  i6|  inches  from  centre  of  plate. 

(B.  of  E.  1905.) 

33.  The  angular  position  D  of  a  rocking  shaft  at  any  time  t  is  measured 
from  a  fixed  position.     Successive  positions  at  intervals  of  1/50  second 
have  been  determined  as  follows  : — 


Time  t,      ) 
seconds     j 

o"oo 

0*02 

0-04 

o'o6 

o'o8 

O'lO 

0*13 

0-14 

o'i6 

0-18 

Position  D,  ) 
radians      J 

o'io6 

o"2o3 

o'337 

0-487 

0*651 

0-819 

0-978 

I'm 

I  '201 

1-222 

Find  the  change  of  angular  position  during  the  first  interval  from 
t  =  o-o  to  t  =  o-o2  ;  calculate  the  mean  angular  velocity  during  this 
interval  in  radians  per  second,  and  set  this  up  on  a  time  base  as  an 
ordinate  at  the  middle  of  the  interval.  Eepeat  this  for  the  other 
intervals,  tabulating  the  results,  and  drawing  the  curve  showing  approxi- 
mately angular  velocity  and  time.  Kead  off  the  angular  velocity  when 
t  =  0-075  second.  (B.  of  E.  1905.) 


297 


LECTUKE  XXII. 

CONTENTS. — Some  Properties  of  Materials  employed  by  Mechanics — Essen- 
tial Properties — Extension — Impenetrability — Contingent  Properties 
— Divisibility — Porosity — Density —  Cohesion  —  Compressibility  and 
Dilatability  —  Rigidity  —  Tenacity  —  Malleability  —  Ductility —  Elas- 
ticity—Fusibility— Load,  Stress,  and  Strain— Total  Stress  and  Inten- 
sity of  Stress — Tensile  Stress  and  Stress — Example  I. — Compressive 
Stress  and  Strain — Example  II. — Limiting  Stress  or  Ultimate  Strength 
— Safe  Loads  and  Elasticity — Limit  of  Elasticity — Hooke's  Law — 
Factors  of  Safety — Modulus  of  Elasticity — Ratio  of  Stress  to  Strain- 
Examples  III.-V. — Resilience  or  Work  Done  in  Extending  or  Com- 
pressing a  Bar  within  the  Elastic  Limit — Examples  VI.-IX. — Single 
Riveted  Lap  Joints — Example  X. — Questions. 

Some  Properties  of  Materials  employed  by  Mechanics. 
— The  properties  of  matter  are  almost  innumerable,  but  they  may 
be  divided  into  two  classes:  (i)  Essential  properties;  (2)  Con- 
tingent properties.  The  essential  properties  are  those  without 
which  matter  cannot  possibly  exist.  The  contingent  properties 
are  those  which  we  find  matter  possessing,  but  without  which  we 
could  conceive  it  to  exist. 

Essential  Properties — 1.  Extension  means  that  property  by 
which  every  body  must  occupy  a  certain  bulk  or  volume.  When 
we  say  that  one  body  has  the  same  volume  as  another,  we  do  not 
mean  that  it  has  the  same  quantity  of  matter,  but  only  that  it 
occupies  the  same  space.* 

2.  Impenetrability  means  that  every  body  occupies  space  to 
the  exclusion  of  every  other  body,  or  that  two  bodies  cannot 
exist  in  the  same  space  at  the  same  time. 

Contingent  Properties. — 1.  Divisibility  means  that  matter 
may  be  divided  into  a  great  but  not  an  infinite  number  of  parts. 
The  ultimate  particles  of  matter  are  termed  atoms,  derived  from  a 
Greek  word  signifying  indivisible. 

2.  Porosity  signifies  that  every  body  contains  throughout  its 
mass  minute  spaces  or  insfcerstices  to  a  greater  or  less  extent. 
This  has  been  proved  to  be  the  case  with  every  known  substance. 
These  spaces  are  supposed  to  be  filled  with  a  highly  elastic  fluid 
called  ether. 

For  example,  when  the  steel  or  cast-iron  cylinder  of  a  hydraulic 

*  For  Simple  Rules  of  Mensuration  see  the  Author's  Elementary  Manual 
on  "Steam  and  the  Steam  Engine,"  Lectures  I.,  II.,  III. 


9  8  LECTURE  xxn. 

press  is  subjected  to  enormous  pressure,  water  will  ooze  through 
the  metal  from  the  interior  to  the  outside. 

3.  Density  is  that  property  by  which  one  body  differs   from 
another  in  respect  of  the  quantity  of  matter  which  it  contains.  * 

Let  Mp  M,  =  Masses  of  two  bodies 
Let  Vj,  V,  =  Volumes  of  two  bodies. 
Let  Dj,  Ds  =  Densities  of  two  bodies. 

If  V,  -  V,,  then  gi  =  S ;  if  D,  =  D,,  then       - 

,,       M. 
If  both  vary,  then  ^ 

4.  Cohesion  is  that  property  by  which  particles  of  matter 
mutually  attract  each  other  at  insensible  or  indefinitely  small  dis- 
tances.    It  is  therefore  different  from  gravitation,  since  the  latter 
acts  at  all  distances.     It  is  evident  that  without  this  property 
we  could  not  have  a  solid,  for  if  a  solid  body  be  lifted  by  one 
part,  the  remainder  sticks  to  it,  and  the  whole  is  kept  together  by 
cohesion. 

5.  Compressibility  and  Dilatability  are  properties  common 
to  all  bodies,  by  which  they  are  capable  of  being  compressed  like 
a  sponge  or  extended  like  a  piece  of  india-rubber  in  a  greater  or 
less  degree. 

6.  Rigidity  signifies  the   stiffness  to  resist  change  of  shape 
when  acted  on  by  external  forces.     Unpliable  materials  which 
possess  this  property  in  a  large  degree  are  termed  hard,  whilst 
uhose  which  readily  yield  to  pressure,  without  disconnection,  are 
called  soft.     Substances  which  cannot  resist  a  change  of  shape 
without  breaking  are  termed  brittle,  whilst  those  that  do  resist 
and  at  the  same  time  change  their  form  are  said  to  be  tough. 

7.  Tenacity  is  the  resistance  (due  to  cohesion)  which  a  body 
offers  to  being  pulled  asunder,  and  is  measured  by  the  tensile 
strength  in  Ibs.  per  square  inch  of  the  cross  section  of  the  body. 
We  will  consider  this  property  in  the  case  of  metals,  &c.,  when 
lealinsr  with  stress  and  strain. 

8.  Malleability  is  that  property  by  which  certain  solids  may 
be  pressed,  rolled,  or  beaten  out  from  one  shape  to  another  with- 
out fracture.     It  is  therefore  a   property  depending   upon    the 
softness,  toughness,  and  tenacity  of  the  material.     Gold  possesses 
this  property  in  a  higher  degree  than  any  other  metal,  and  con- 

*  The  Density  of  a  substance  is  either  the  number  of  units  of  mass  in  a 
unit  of  volume,  in  which  case  it  is  equal  to  the  heaviness  (i.e.,  weight  of 
unit  volume  of  substance  in  unit  weight)  ;  or  it  is  the  ratio  of  the  mass  of 
a  eriven  volume  of  the  substance  to  the  mass  of  an  equal  volume  of  water, 
m  which  case  it  is  equal  to  the  specijic  gravity. 


PROPERTIES   OF  MATERIALS.  299 

Bequently  sheets  of  gold  are  procurable  of  less  than  one-thousandth 
of  an  inch  in  thickness.  Copper  is  one  of  the  most  useful  of  the 
malleable  metals,  and  it  may  be  beaten  out  into  most  elaborate 
shapes  from  the  solid  ingot.  The  Swedish  iron  of  which  horse- 
shoe nails  are  made  is  also  very  malleable,  and  is  therefore  highly 
prized  by  the  blacksmith.  Lead,  although  possessing  softness,  is 
not  sufficiently  tenacious  to  be  considered  a  very  malleable  metal, 
but  still  it  finds  one  of  its  most  useful  applications  in  the  form  of 
rolled  lead  sheathing  for  roofs  of  houses  and  interiors  of  water 
tanks,  «fec. 

9.  Ductility*  is" that  property  by  which  some  metals  may 
be  drawn  down  through  a  die-plate  into  wire  or  tubes.  This  pro- 
perty depends  chiefly  on  toughness  and  tenacity.  For  example,  we 
find  that  the  very  fine  pianoforte  wire  used  with  Lord  Kelvin's 
deep-sea  sounding  machine  is  both  hard  and  rigid,  but  possesses 
great  toughness  and  tenacity.  The  copper  wire  used  for  electrical 
conductors  becomes  harder  and  harder  as  it  gets  drawn  down  to 
smaller  and  smaller  sizes,  and  it  has  therefore  to  be  annealed  in 
order  to  comply  with  the  many  bendings  and  unbendings  which  it 
has  afterwards  to  undergo  in  winding  and  unwinding  it  upon 
bobbins  whilst  twisting  it  into  a  stranded  conductor  or  in 
covering  it  with  a  dielectric  of  cotton,  silk,  gutta-percha,  or 
india-rubber,  <kc.  Solid-drawn  copper  pipes  are  frequently  used 
for  conveying  steam  and  liquids  where  a  sound  light  job  is  required 
to  resist  great  pressures.  This  flowing  property  of  metals  is  now 
taken  great  advantage  of  by  the  engineer  in  a  variety  of  ways. 
For  example,  lead  and  tin,  when  subjected  to  great  hydraulic 
pressure,  and  properly  guided  through  a  die,  can  be  squirted  into 
long  continuous  rods  or  pipes,  or  squeezed  on  to  insulated  electric 
light  conductors,  so  as  to  form  a  water-tight  protecting  sheathing 
thereto,  just  as  if  these  metals  were  composed  of  so  much  plastic 
dough.  In  fact,  all  you  have  to  do  in  order  to  cause  many  harder 
and  stronger  metals,  such  as  copper,  iron,  and  steel,  to  flow  cold 
into  almost  any  shape  of  mould,  is  to  apply  sufficient  pressure  and 
to  give  sufficient  time  for  them  to  retain  their  natural  homogeneous 
and  isotropic  structure,  or  to  adopt  means  for  rp^toriner  thp 
structure  should  they  have  departed  therefrom  during  any  part 
of  the  process.  A  metal  is  said  to  be  homogeneous  when  it  is  of 
the  same  density  and  composition  throughout  its  mass.  It  is 
isotropic  when  it  has  the  same  elastic  properties  in  all  directions. 

*  Refer  to  the  description  of  the  Lever  Testing  Machine,  illustrated  in 
Lecture  IV.,  and  to  Lord  Kelvin's  Hydrostatic  Wire  Testing  Machine, 
illustrated  in  Lecture  XVII.,  as  examples  of  machines  whereby  the 
comparative  ductility  of  certain  materials  may  be  ascertained  by  their 
percentage  elongation. 


300 


LECTURE    XXII. 


1O.  Elasticity  is  that  property,  possessed  by  different  solids  in 
a  greater  or  less  degree,  of  regaining  their  original  size  and  shape 
after  the  removal  of  the  force  which  caused  a  change  of  form. 
We  shall  see  later  on  that  there  are  limits  of  plasticity  oeyond 
which  the  bodies  will  not  regain  their  exact  normal  size  or  shape. 

12.  Fusibility  is  that  property  whereby  metals  and  many  other 
substances,  such  as  resins,  tallows,  &c.,  become  liquid  on  being 
raised  to  a  certain  temperature.  The  following  table  shows  in 
round  numbers  the  melting-points  of  a  few  of  the  commoner 
metals : — 

MELTING  POINTS  OF  METALS  IN  DEGREES  FAHRENHEIT. 


Mercury       . 
Tin       .        . 

.        .        .    -   38 

Copper 
German  silver      . 

200O 
2OOO 

Bismuth       . 
Lead    . 

...       500 

•        ,       600 

Gold    . 
Cast  iron      .        • 

2OOO 
22OO 

Zinc     .         . 

Steel    . 

25OO 

Antimony    • 

.        .       800 
.        •        .     1800 

Nickel,  also  AluminiuE 
\Vrouerht  iron 

a           2800 

23OO 

Platinum 

3SOO 

Load,  Stress,  and  Strain. — When  force  is  applied  to  a  body 
so  as  to  produce  either  elongation  or  compression,  bending,  torsion, 
shearing,  or  a  tendency  to  any  of  these,  the  force  applied  is 
termed  the  load,  the  corresponding  resistance  or  reaction  in  the 
material  is  termed  the  stress  due  to  the  load.  Any  alteration  pro- 
duced in  the  length  or  shape  of  the  body  is  termed  the  strain. 

DEFINITIONS. — Load  is  the  force  or  forces  applied  to  the  body. 
/Stress  is  the  reaction  in  the  body  duo  to  the  load. 
Strain  is  the  alteration  in  shape  as  the  result  of  the  load. 

The  load  is  called  a  dead  load  when  it  produces  a  steady  or  a 
gradually  increasing  or  diminishing  stress.  For  example,  the 
weight  of  a  roof  on  the  walls  of  a  building  is  a  steady  or  dead 
load.  The  gradually  increasing  pull  produced  on  the  specimen 
in  the  lever-testing  machine,  illustrated  by  the  fourth  figure  in 
Lecture  IV.,  is  also  a  dead  load. 

The  load  is  termed  a  live  load  when  it  varies  from  instant  to 
instant.  For  example,  a  regiment  of  soldiers,  or  a  series  of 
vehicles,  or  a  train  passing  over  a  bridge  creates  a  live  load  on  the 
bridge. 

Total  Stress  and  Intensity  of  Stress.— The  total  stress  is 
the  total  reaction  due  to  the  total  load.  The  intensity  of  stress, 
or  simply  the  word  stress,  expresses  the  reaction  per  unit  area  of 
the  cross  section.  Thus,  if  P  be  the  total  force  applied  in  lbs.r 
and  A  be  the  total  cross  section  in  square  inches,  then  the 

p 
Mean  Intensity  of  Stress  on  the  section  =  ~-r  Ibs.  per  square  inch. 


STRESS   AND    STEAIN.  30 1 

Tensile  Stress  and  Strain. — If  the  line  of  action  of  a  load 
oe  along  the  axis  of  a  bar,  tie-rod,  or  beam,  so  as  to  tend  to  elon- 
gate the  same,  the  reaction  per  square  inch  of  cross  section  is 
termed  the  tensile  stress,  and  the  elongation  per  unit  of  length  is 
called  the  tensile  strain. 

EXAMPLE  I. — A  wire  ^  square  inch  in  cross  section,  and 
10  feet  long,  is  fixed  at  its  upper  end.  A  load  of  1000  Ibs.  is 
hung  from  the  lower  end,  and  then  the  wire  is  found  to  stretch 
i  inch,  (i)  What  is  the  stress  ?  (2)  What  is  the  strain  ? 

ANSWER. — (i)  Here  P=  1000  Ibs.,  and  A  =  T^  sq.  in. 

Let  p  «=  stress  or  pull  per  square  inch  in  Ibs. 

p  / 

.-.   The  stress,  or  p  =~^=  Ioo°  /  TV  =  10>000  lbs-  P61"  SQ-  ^h- 

(2)   Original  length    =  L  «=  10'  =  120",  and  the   increase   of 
length  =  I  =  i". 

Let  e  •*  strain  or  extension  per  unit  of  length,  i.e.,  per  inch  in 
this  case, 

increase  of  length     I         i" 

.-.    The  Strain,  or  e  =  —          i   i     Jh    =f=  "  "  *0083 

original  length       L      120 

Compressive  Stress  and  Strain. — If  the  line  of  action  of  a 
load  be  along  the  axis  of  a  bar,  shore,  strut,  or  pillar,  so  as  to 
tend  to  compress  or  shorten  the  same,  the  reaction  per  square 
inch  of  cross  section  is  termed  the  compressive  stress,  and  the 
diminution  per  unit  of  length  is  called  the  compressive  strain. 

EXAMPLE  II. — A  vertical  support  in  the  form  of  a  hollow 
pillar,  having  2  square  inches  cross  section  of  metal,  is  10  feet  long. 
With  a  load  of  10,000  Ibs.  resting  on  the  top,  it  is  found  to  be 
compressed  ^  of  an  inch  in  length,  (i)  What  is  the  stress? 
(2)  What  is  the  strain  ? 

ANSWER. — (i)  Here  P  =  10,000  Ibs.,  and  A  =*  2  sq.  inches. 
Let  p  =  stress  or  compression  per  sq.  in.  of  cross  section  in  Ibs. 

P       10,000 

.•.  The  stress,  or  p  =  -r-  = =  5000  per  square  inch. 

A.  2 

(2)  Original  length  =  L  =  10'  =    120",  and  the  diminution  of 
length  =  J  =  A" 

Let  e  =-  strain  or  compression  per  unit  of  length,  t.e.,  per  inch 
in  this  case, 

diminution  in  length        -i" 
or  .  -       origiDaI  lengtlf     =  ^  -  -00083 

Limiting  Stress  or  Ultimate  Strength. — For  every  kind 
of  material  and  every  way  in  which  a  load  is  applied,  there  must 
be  a  value,  which,  if  exceeded,  causes  rupture  or  fracture  of  tn« 


302 


LECTURE     XXII. 


body.  The  greatest  stress  which  the  material  is  capable  of 
withstanding  is  called  the  limiting  stress  or  ultimate  strength  per 
square  inch  of  cross  section  of  the  substance,  for  the  particular 
way  in  which  the  load  is  applied. 

Factors  of  Safety. — The  ratio  of  the  ultimate  strength  or 
limiting  stress  to  the  safe  working  load  is  called  the  factor  of  safety. 
This  factor  of  necessity  varies  greatly  with  different  materials, 
and  even  with  the  same  material,  according  to  circumstances. 
For  materials  which  are  subjected  to  oxidation  or  to  internal 
changes  of  any  kind,  the  factor  of  safety  must  of  necessity  be 
larger  than  in  those  which  are  always  kept  dry  or  are  well  painted 
and  carefully  handled.  There  is  no  condition  in  engineering 
structures  which  requires  a  more  careful  calculation,  or  estimate 
of  the  necessary  factors  of  safety,  than  that  of  railway  bridges, 
which  are  exposed  to  all  sorts  of  weathers  and  to  extremely 
variable  live  loads.  The  skill  of  the  engineer  is  therefore  brought 
out,  when  he  designs  structures  so  as  to  include  all  possible 
circumstances  to  which  they  may  be  subjected,  and  so  proportions 
the  material  at  his  disposal,  that  there  shall  be  a  minimum  of 
internal  stress  and  strain,  with  a  maximum  resistance  to  dead  or 
live  loads  for  a  minimum  cost  of  material  and  workmanship.* 
TABLE  OF  ULTIMATE  STRENGTH  AND  WORKING  STRESS  OP  MATE- 
RIALS WHEN  IN  TENSION,  COMPRESSION,  AND  SHEARING. 


i 

Ultimate  Strength. 

Working  Stress. 

Tons  per  sq.  inch. 

Tons  per  sq.  inch. 

Materials. 

Ten- 

Com- 

Shear- 

Ten- 

Com- 

Shear- 

sion. 

pression. 

ing. 

sion. 

pression. 

ing. 

Cast  iron     . 

7-5 

45 

14 

i-5 

9 

3 

Wrought-iron  bars 

25 

20 

20 

5 

3-5 

4 

Steel  bars     . 

45 

70 

3° 

9 

9 

5 

Copper  bolts 

15 

25 

3 

5 

Brass  sheet  .         . 

14 

—  — 

3 

— 

Safe  Loads  and  Elasticity. — As  a  rule,  however,  the  object 
of  the  engineer  is  not  to  put  such  a  stress  on  his  materials  of 
construction  as  will  cause  rupture  or  destruction,  but  rather  to 

*  For  other  tables  relating  to  the  Strength  of  Materials  in  Engineering 
Constructions,  Factors  of  Safety,  &c.,  refer  to  Rankine's  "Rules  and 
Tables,"  Molesworth's  "Pocket  Book  of  Engineering  Formulae,"  D.  K. 
Clarke's  "Rules  and  Tables,"  "The  Practical  Engineer's  Pocket  Book;" 
and  for  Electrical  Engineering  Materials  to  Munro  and  Jarnieson's  "  Pocket 
Book  of  Electrical  Rules  and  Tables." 


LIMIT    OF    ELASTICITY — HOOKE'S    LAW.  303 

make  machines  and  raise  structures  that  will  withstand  all  rea- 
sonable forces  likely  to  be  brought  to  bear  upon  them.  Conse- 
quently, he  is  quite  as  much  interested  in  what  may  be  termed 
safe  loads  as  in  ultimate  or  destructive  ones.  He  therefore 
requires  to  know  what  loads  can  be  safely  applied  to  materials 
under  different  circumstances,  so  as  to  comply  with  that  most 
useful  property  termed  elasticity,  which  we  again  define  as  the 
capability  of  regaining  their  original  size,  shape,  and  even  strength, 
after  the  removal  oftht  forces  which  caused  a  change  of  form  in 
them. 

Limit  of  Elasticity — Hooke's  Law.  —  So  long  as  the 
stress  or  reaction  per  square  inch  of  cross  section  does  not  exceed 
a  certain  limit,  called  the  limit  of  elasticity,  then  the  material 
will  return  to  its  original  shape,  size,  and  strength,  after  the 
removal  of  the  load.  This  limit  has  been  ascertained  for  most 
materials  of  construction  by  elaborate  experiments,  which  are  to 
be  found  tabulated  in  the  Proceedings  of  the  Institutions  of  the 
Civil  and  Mechanical  Engineers,  and  in  such  books  as  Rankine's 
"  Rules  and  Tables,"  Molesworth's  "  Pocket  Book  of  Engineering 
Formula?,"  and  D.  K.  Clark's  "  Rules  and  Tables."  For  example, 
with  a  bar  of  good  wrought  iron  the  elastic  limit  is  only  reached 
after  a  stress  of  24,000  Ibs.  per  square  inch  has  been  brought  to 
bear  upon  it,  and  in  a  similar  degree  every  other  material  has  a 
corresponding  limit,  beyond  which  it  is  not  safe  to  stress  it,  for 
tear  that  it  should  be  overstrained,  and  thus  lose,  to  a  certain 
extent,  its  property  of  recuperation  or  restitution,  or  take  a 
permanent  set. 

Within  this  limit,  Hooked  Law  holds  good  for  metal  bars  under 
the  action  of  forces  tending  to  elongate  or  compress  them.  This 
law  states  that : 

(1)  The  amount  of  extension  or  compression  for  the  same  bar 
is  in  direct  proportion  to  the  stress. 

(2)  The  extension  or  compression  is  directly   proportional  to 
the  length. 

(3)  The  extension  or  compression  is  inversely  proportional  to 
the  cross  sectional  area ;  consequently,  if  the  area  be  doubled  the 
extension  or  compression  will  be  halved,  or  the  resistance  to  the 
load  will  be  doubled. 

Let  P  =  Pull,  push,  or  load  in  Ibs.  on  the  bar. 

„  A  =  Area  of  cross  section  of  the  bar. 

„    L  =  Length  of  the  bar  before  the  load  was  applied. 

n    I    =  Length  by  which  the  bar  is  extended  or  compressed. 

w  p   =  Stress  or  load  per  square  inch  of  cross  section  =  P/A. 

P 

Then,  so>  long  as  Y-  does  not  exceed  the  elastic  limit,  I  varies  directly 

U 


JO4  LECTURE   XXIL 

I  P 

as  P  for  the  same  bar ;  or  ^  varies  directly  as  -^>  for  different  bari 

of  the  same  material  and  subjected  to  the  same  conditions. 

In  other  words,  so  long  as  the  stress  does  not  exceed  the  elastic 
limit,  the  strain  will  be  proportional  to  the  stress. 

Modulus  of  Elasticity,  or  Ratio  of  Stress  to  Strain. — As 
we  have  just  indicated,  by  HOOKE'S  LAW,  if  a  metal  under  test  be 
gradually  subjected  to  a  stress,  and  if  the  load  does  not  exceed 
the  limits  of  elasticity  of  the  material,  the  strain  will  be  in  pro- 
portion to  the  load. 

Consequently,  the  ratio  of  the  stress  to  the  strain  is  a  constant 

Quantity  for  eaoh  particular  substance  within  the  limits  of  Hooke  s 
^aw,  and  is  termed  the  Modulus  of  Elasticity  of  the  substance. 

But         .         .         Stress  oc  Strain 

Stress  =  Ex  Strain.* 

Where  E  represents  a  constant  number  or  modulus  depending  on 
the  natural  elasticity  of  each  material — 

JP 

.    F       Stress        ^       PL^ 
Strain   "    l_    ''    Al 
L 

».«.,     ;     .     .     .     PL  =  AZE 

Or,  imagine — it  is  pure  imagination — that  a  substance  could 
be  elongated  to  double  its  length  or  compressed  to  zero  by  sub 
jecting  it  to  a  certain  load,  we  should  then  have  an  index  value, 
or  constant  number,  or  modulus,  by  which  we  could  compare  it 
with  every  other  substance  which  behaved  likewise  under  similar 
circumstances.  This  imaginary  value  is  termed  the  Modulus  of 
Elasticity. 

For  example,  take  a  bar  of  wrought  iron  of  i  square  inch  cross 
section,  which  is  found  to  stretch  2  4  a  o^  o  o  o  Par^  °^  ^s  length 
under  a  stress  of  i  lb.,  and  consequently  by  Hooke's  Law  twice 
that  amount  under  a  stress  of  2  Ibs.,  and  so  on  ;  then  this  number 
(24,000,000)  is  called  the  Modulus  of  Elasticity  of  the  iron  bar. 
For,  if  the  elasticity  of  the  bar  were  perfect,  it  is  evident  that  a 
stress  of  24,000,000  Ibs.  would  produce  a  strain  or  elongation 
equal  to  the  length  of  the  bar,  or,  fj£°o™o  =  i.  In  other 
words,  the  length  of  the  bar  would  be  doubled  under  this  stress. 
Consequently,  we  have  the  following  definition, 

*  Since  stress  is  reckoned  by  so  many  Ibs.  per  square  inch  of  cross  section 
of  a  material,  and  strain  is  simply  an  abstract  number,  it  follows  that  the 
Modulus  of  Elasticity  (E)  must  also  be  reckoned  by  so  many  Ibs.  per 
square  inch. 


MODULUS    OF    ELASTICITY. 


30S 


DEFINITION. — The  Modulus  of  Elasticity  of  any  substance  is 
that  load  which  would  double  its  length  on  the  supposition  that  the 
elongation  was  proportional  to  the  stress,  and  that  the  cross  section 
t>f  the  bar  was  of  unit  area,  or  one  square  inch,  and  supposing  the 
bar  to  remain  perfect  during  the  operation. 

From  this  we  again  see  that — 

Mod  ulus  of  Elasticity  =  8tres8  =  E  =  ?  /  * 
strain  A/  L 

Or,    ....        PL  =  AZE 

MODULI  OP  ELASTICITY  TO  STRETCHING. 
(See  Rankine's  Rules  and  Tables  for  complete  Data.) 


Material 

Modulus  of 
Elasticity  in 
Ibs.  per  sq.  in. 
in  round 
numbers. 

Material. 

Modulus  of 
Eia.-<iicity  in 
Ibs.  per  sq.  in. 
ih  round 

numbers. 

(Mean  values.) 

(Mean  values.) 

Wood,  Elm    .     .     . 

I.OOO.OOO 

Lead  (sheet)  .     .     . 

700.000 

„      Larch     .     . 

1,100.000 

„      (wire)  .     .     . 

I.OOO.OOO 

„      Beech     .     . 

1,300.000 

Brass  (cast)    .     .     . 

9.000,000 

ff      Birch       .     . 

1,400.000 

„       (wire).     .     . 

14.000.000 

„      Mahogany  . 

I,4OO,OOO 

Copper  (cast)      .     . 

15.000.000 

„      Oak    ... 

1,500,000 

(wire)     .     . 

17,000,000 

„     Pine  (yellow) 

1,600.000 

Cast  Iron  .... 

i8.ooo,coo 

M      Ash    .     .     . 
Teak 

I.600,000 
2.OOO.OOO 

Wrought  Iron     .     . 
Steel     

25,000.000 
35.000,000 

n       a 

EXAMPLE  III. — A  steel  bar  5'  long  and  2\  sq.  in.  in  cross  section 
is  suspended  by  one  end  ;  what  weight  hung  on  the  other  end  will 
lengthen  it  by  -016  inch,  if  the  modulus  of  elasticity  of  steel  is 
30,000,000  Ibs.  per  square  inch?  (S.  and  A.  Exam.  1877.) 

ANSWER. — First  ask  what  is  wanted  ?     Viz.,  stress. 

Now  the  universal  rule  is  Modulus  of  Elasticity  = — 

strain 

Or,  stress  =  modulus  x  strain. 

For,  The  strain  is  the  elongation  per  unit  of  the  length. 
-016"  -016 


Consequently,  e  * 


5'x  12' 


6o 


=  •00026. 


3o5  LECTURE  xxn. 

.•.     Tho  Stress  =  Modulus  x  strain 

»  =  30,000,000  x  -00026  =  8000  Ibs.  per  sq.  in. 

And,  The  Total  Stre88  =  Sooo  Ibs.  x   2-25  sq.  in.  =  18,000  Ibs. 

Or,  we  might  have  applied  the  formula  previously  deduced  — 
Hz., 


where  P  is  the  total  pull  required  in  Ibs. 


.. 

L  5x  12 

EXAMPLE  IV.  —  What  do  you  understand  by  stress  and  strain 
respectively  ?  If  an  iron  rod,  50  feet  long,  is  lengthened  by  J  inch 
under  the  influence  of  a  stress,  what  is  the  strain  ?  (S.  and  A. 
Exam.  1892.) 

ANSWER.  —  Stress  is  the  reaction  per  unit  area  of  cross  section 
due  to  the  load.  Let  P  =  the  total  tension  acting  on  area  A  ; 

P 
Then  stress  —P  =  ~T 

Strain  is  the  ratio  of  the  increase  or  diminution  of  length  or 
volume  to  the  original  length  or  volume.  Let  L  =  original  length 
of  a  bar  of  the  material,  I  =  amount  by  which  the  length  is  in- 
creased or  diminished  •,  then,  when  the  bar  is  subjected  to  stress, 

The  strain  =  e  =  ^ 
L 

In  the  example  given,  L  =  50'  x  1  2"  =  600  inches  ;  and  1=  J  inch. 

.«.  Strain,  e=i  =  -i-  =  —?—  ='00083 
L     ooo     i  200 

EXAMPLE  V.  —  From  the  above  question  and  answer  determine 
the  modulus  of  elasticity  of  the  iron  of  which  the  rod  is  composed, 
if  the  load  was  4366  Ibs.,  and  the  cross  section  of  the  rod  2  square 
inches. 

Total  load 
ANSWER.  —  (i)  Stress  =/^  — 

x  '  Cross  area 

P      4366  Ibs 
Or,  ,         .         .          p  =  —  -  =  li     -  =2183  Ibs.  per  sq.  in. 

A.  2 

(2)  Modulus  of  Elasticity    =  -  -  !— 

Strain 

Or,          ...          E  =  P-=    2T83.  =25,000,000 

6        '00083 

.'.  A  load  of  25,000,000  Ibs.  would  elongate  a  rod  of  the  iron 
to  double  its  length  by  tensile  stress. 


RESILIENCE   WITH  EXAMPLES.  307 

Resilience  or  Work  done  in  Extending  or  Compressing  a 
Bar  Within  the  Elastic  Limit.—  Definitions  of  Resilience.  —  (i)  When 
a  bar  is  strained  within  the  elastic  limits  either  by  a  compressive  or  a 
tensile  force,  then  the  work  done  in  extending  or  compressing  it  is  equal  to 
the  amount  of  compression  or  extension  multiplied  by  the  mean  stress 
which  produces  the  strain.  The  amount  of  work  thus  done  when  tho 
stress  just  reaches  the  elastic  limit  is  termed  resilience. 

(2)  Resilience  may  also  be  defined  as  half  the  product  of  the  stress  into 
the  strain,  where  the  stress  and  the  strain  are  those  produced  when  the 
elastic  limit  is  reached. 

Let  P  =  push  or  pull  in  Ibs.  applied  gradually. 

„     A  =  area  of  bar  in  square  inches  of  cross-section. 

„      L  =  length  of  bar  in  inches. 

„       1=  elongation  of  the  bar  in  inches  due  to  the  force  P. 

,,      p=  stress  per  square  inch  when  the  elastic  limit  is  reached,  =P/A. 

„      E  =  resilience  of  the  bar. 

I.  —  Where  the  Load  is  Gradually  Applied.  —  If  the  load  be  gradually  in- 

p 
creased  from  o  up  to  P  Ibs.,  its  mean  value  will  be  —  ,  and  the  work 

•p 
done  or  resilience,  R=:|  I  inch-lbs. 

But  P=pA=EAJL 

BAP       i   /P\  aAL       I    a  AL 
.  •  .  work  done  or  res^ence  R=  -~  =-         —  =  ~  P*  -» 


2 

„    pa     volume  ofbar.   rt,  1Via 
>.        i.  „          R=EX  -  2  -  mch-lbs. 

Work  done  per  unit^  _  p1 
of  volume        j~2E 

This  latter  equation  gives  the  strain  energy  stored  in  the  6ar,  since  the 
material  is  still  elastic. 

II.—  When  the  Load  is  Suddenly  Applied  without  Initial  Velocity.—  Let  I 
be  the  elongation  or  shortening  of  the  bar  when  the  load  P  is  suddenly 
applied,  but  without  initial  velocity. 

Then,  the  work  done  by  the  external  load  must  be  equal  to  the  energy 
•tored  up  in  the  bar. 

Let  P  max  be  the  maximum  stress  per  square  inch  which  is  produced  in 

the  bar. 

•p 

The  me*n  stress  will  be     max  as  its  initial  value  was  o. 

Hence,  the  work  done  on  the  bar       =  -  fnax    =      -.  , 
and  the  work   done   by  external  load  =  PZ. 
Therefore,  ^J^^^^pf, 

2  2L 

I  P 

or,  P,^=c2P;  andEj-  =  2-^. 

Consequently,  the  maximum  intensity  of  stress  induced  in  a  bar  by  the 
sudden  application  of  a  constant  load,  without  initial  velocity,  is  double 
the  intensity  of  the  stress  produced  by  the  load  itself. 

III.  —  When  the  Load  is  Suddenly  Applied  and  with  an  Initial  Velocity.  — 
Let  a  weight  of  W  Ibs.  be  dropped  from  a  height  of  h  inches  upon  a  bar, 
and  that  the  maximum  stress  produced  in  the  bar  was  P  max  Ibs.  per  square 
inch,  whilst  the  elongation  or  compression  was  I  inches. 


3O8  LECTURE   XXII. 

Work  done  by  the  falling  weight  =  W  (A  +  J)  =  energy  stored  up  in  the  bar. 

P     J,    EAZ2 
Therefore,  -^=—  -  =W  (h  +  l)±=Wh  if    he  value  of  I 

is  small. 
If  v=velocity  of  load  P  at  moment  of  impact  in  ft.  per  sec., 

then  kinetic  energy,  E*  =  g^  =  ^  =  W(h  +  *)  ft,lbs. 
2y         2  A.  1  2 

EXAMPLE  VI.—  Calculate  the  resilience  of  a  steel  tie-bar,  i  inch  in 
diameter  and  4  feet  long  if  the  elastic  limit  is  reached  under  a  load  ot 
20  tons,  and  modulus  of  elasticity  =  13,000  tons  per  square  inch. 

Answer.  — 

Work   done  or  resilience,  11  = 


=  -  inch-tons. 


EXAMPLE  VII.—  A  round  bar  of  steel  is  20  feet  long  and  i  inch  in 
diameter.  Find  the  tensile  load  which,  if  suddenly  applied,  would  cau^e 
an  instantaneous  elongation  of  the  bar  of  'I  inch. 

Taking  £  =  13,000  tous  per  square  inch. 
.  — 

T7I   \  J 

P=—  =-»  where  symbol  letters  represent    the  values  stated 
2Ll        in  the"  text. 

p  _  1  3,000x785  ix  'I 

2  X  2O  X  1  2 


EXAMPLE  VIII.  —  Determine  the  greatest  weight  that  can  be  dropped 
from  a  height  of  i  foot  on  a  bar  of  steel  which  is  i  inch  in  diameter  and 
10  feet  long.  The  modulus  of  elasticity,  £=13,000  tons  per  square  inch, 
and  the  elastic  limit  18  tons  per  square  inch.  Also,  find  the  alteration  in 
the  length  of  the  bar. 

Answer.  — 

p   m 

P=A=-i7 

13000x2 


RESILIENCE  WITH  EXAMPLES.  309 

EXAMPLE  IX.—  In  a  tensile  test  of  a  piece  of  flat  wrought  iron  bar 
the  following  results  were  obtained  : 

(i)  Original  dimensions  of  cross-section,  2*02  inches  by  0-51  inch. 
(ii)  Final  dimensions  of  cross-section  at  point  of  fracture.  I  -49  inches 

by  0*39  inch. 

(iii)  Gross  load  at  limit  of  elasticity,  36,000  Ibs. 
(iv)  Gross  load  at  fracture,  59,000  Ibs. 
(v)  Total  final  extension  on  a  length  of  10  inches,  1*63  inches,  extension 

when  load  was  22,000  Ibs.,  -0075  inch. 

Find  from  the  above   data,  (a)  the  modulus  of  elasticity,  (6)  limit  of 
elasticity,  and  tenacity  in  Ibs.  per  square  inch  ;  also  (c)  the  reduction  of 
area   per  cent.,  and    (d)  the  approximate  work  done   in  fracturing  this 
specimen.     (L.U.B.Sc.  Eng.  1903.) 
Answer.  — 

(a)  Young's]  pL 
modulus  of  V  E=-T-J 
elasticity  J  ** 

_        22,OOOX  10  220,000 


2  '02  X5  1  X  -0075  ~  -007726 

„          E  =  28,473,440  Ibs.  per  square  inch. 

(b)  Stress      an 

elastic  limit  I  Load  at  elastic  limit  _  36,000 
in  tons  per  j  ~~  Original  area  ~~  i  -0302 
square  inch  J 

„  =34,944  Ibs.  per  square  inch. 

(c)  Percentage]      Originalarea-final  area 
contraction  of  >-  =  —     ,.  .   .  —  :  -    -  •  x  100 
area  J  Original  area 

^(2-02  x  -51)  -(i  '49  x  -39)     IOQ 

2'O2X  '51 


(d)  Work  done^ 

fnch  in0  frac°    =Work  dope  in  stretching  the  test-piece 
taring      ^e  |  Volume  of  the  test-piece 

specimen        J 

/Mean  load  between  starting-point  and  point  of\ 

_\  fracture  x  distance  moved  through  in  inches  / 

""/Area  in  square  inches  x  length  of  test-piece  in) 

\  inches  / 


IO 

_  Final  length  -  original  length 
on"  io&  inches  /  ~"  Original  length 

*I-^^°=16-3  per  cent, 


310 


LECTURE  XXII. 


Single-Riveted  Lap  Joints.    (See  the  author's  text-book  on  Steam, 
«.,  re  Riveted  Joints.) 
Let  p  =  pitch  of  rivets  in  inches. 
„    d  =  diameter  of  rivet  hole  in  inches. 

t  =  thickness  of  plate  in  inches. 
\    ft  =  tearing  resistance  or  tensile  stress  of  the  plate  in  Ibs.  per 

square  inch. 

„  /8  ^  shearing  resistance  or  shear  stress  of  the  rivet  in  Ibs.  pel 
square  inch. 


n 


SINGLE-RIVETED"  T^AP  JOINT. 

Then  for  a  single-riveted  lap  joint, 

the  area  of  plate  under  tensile  stress    =  (p  -  d)t, 

Trd~ 
and  the  area  of  rivet  under  shear  stress  =  - 

4 
Hence,  for  equal  strength,  the  following  equation  must  be  true  : 


In  practice,  d  =  i  '2  y  £,  or  £  =  —  -  is  usually  taken  for  boilers. 


ft(p-d)  =/•  —  x  i'44  =  "36  x 


X/r 


Or, 


The  values  to  be  taken  for/t  and/s  in  any  given  case  depends  upon  the 
number  of  rows  of  rivets  ;  upon  the  material  which  is  used  (iron  or  steel), 
and  on  whether  the  holes  have  been  punched  or  drilled.  But,  in  practice 
ft  ranges  from  35,000  to  67,000  Ibs.  per  square  inch,  and  /«  from  43,000  to 
53>°oo  Ibs.  per  square  inch.  Whilst  for  iron  plates  and  iron  rivets,  with 

drilled  holes,  the  ratio— may  be  taken  as  equal  to  '94;  which  would  give 
for  the  single-riveted  lap  joint  (p-d)o*94=  1*131. 


SINGLE-RIVETED  LAP  JOINT.  311 

EXAMPLE  X.—  In  a  single-riveted  lap  joint  the  thickness  of  the  plate  ia 
|  inch,  and  the  diameter  of  the  rivet  is  I  inch.  If  the  tearing  resistance  of 
the  plate  is  60,000  Ibs.  per  square  inch,  and  the  shearing  resistance  cf 
the  rivet  is  50,000  Ibs.  per  square  inch,  find  the  proper  pitch  of  the  rivets. 

(C.  &  G.,  1905,  O.,  Sec.  B.) 

Answer.  —  Substituting  the  numerical  values  given  by  the  question  in 
the  above  formula,  we  get  : 

(p-<Dr=  1-131 


50,000 

1-131  x  50,000  .    . 
J*-1  =  --  60^55  --  =?  95 

j>  =  '95  +  i  =  1'95  inches. 


312  LECTURE   XXII. — QUESTIONS. 


LECTURE  XXII.—  QUESTIONS. 

1.  State  and  define  the  essential  and  contingent  properties  of  matter, 
and  give  the  names  of  those  engineering  materials  with  which  you  happen 
to  be  practically  acquainted,  that  best  exemplify  each  property. 

2.  What  is  the  meaning  of  the  term  ductility  as  applied  to  wrought  iron  ? 
Describe,  with  sketches,  some  apparatus  for  testing  a  piece  of  metal  as  to 
ductility.     If  a  uniform  bar  of  iron  10  inches  long  is  found  to  stretch 
i£  inches  at  the  time  of  fracture,  what  is  the  measure  of  the  ductility  of 
the  material  of  the  bar  1  Ans.  15  per  cent. 

3.  Give  the  approximate  breaking  tensile  stress  for  a  bar  of  cast  iron  of 
one  square  inch  sectional  area,  and  the  same  for  a  bar  of  wrought  iron  1 
What  is  the  meaning  of  the  term  ductility  as  applied  to  wrought  iron,  and 
how  is  the  ductility  of  iron  measured  ? 

4.  What  must  be  the  diameter  in  inches  of  a  round  rod  of  wrought  iron 
in  order  to  sustain  a  load  of  50  tons  ?    It  is  given  that  a  bar  of  iron  i  square 

inch  in  section  will  just  support  a  load  of  25  tons.     Ans.d=\/   --  —  =  i'6". 


5.  What  is  the  modulus  of  elasticity  of  a  substance  ?  A  round  bar  of  iron, 
12  feet  long  and  i^  square  inches  in  sectional  area,  is  held  at  one  end  and 
pulled  by  a  force  till  it  stretches  £  inch  ;  find  the  force,  the  modulus  of 
elasticity  being  30,000,000,  Ans.  39,063  Ibs. 

6.  A  round  bar  of  steel  i"  in  diameter  and  10  feet  long,  is  fi.xed  at  its 
upper  end,  and  a  load  is  applied  to  the  bottom  end  and  stretches  it  -05". 
Find  the  load  if  the  modulus  of  elasticity  is  30,000,000.     Ans.  9817  '5  Ibs. 

7.  Find  the  dimensions  of  a  transverse  section  of  a  square  rod  of  fir  to 
sustain  a  suspended  load  of  10  tons,  the  rod  being  held  vertically.     The 
breaking  load  of  a  rod  of  fir  one  square  inch  in  section  is  6  tons.    Ans. 
1-29  inches. 

8.  Find  the  extension  produced  in  a  bar  of  wrought  iron  4  feet  long  and 
2  square  inches  in  section  by  a  suspended  weight  of  4!  tons,  the  modulus 
of  elasticity  of  the  material  being  29,000,000   pounds   per  square  inch. 

Ans.  -009  inch. 

9.  What  do  you  understand  by  the  terms  stress,  strain,  and  modulus  of 
elasticity  ?    A  tie-rod,  100'  long  and  2  square  inches  cross  area,  is  stretched 
.75"  under  a  tension  load  of  32,000  Ibs.     What  is  the  intensity  of  the  stress, 
the  strain,  and  the  modulus    of  elasticity   under  these  circumstances? 

Ans.    16,000  Ibs.  per  square  inch;  0-000625; 
25,600,000. 

10.  Define  what  is  meant  by  "  dead  load,"  "  live  load,"  "  limiting  stress," 
"limit  of  elasticity,"  and  "  factors  of  safety." 

11.  What  do  you  understand  by  stress  and  strain  respectively?    If  an 
iron  rod,  50  ft.  long,  is  lengthened  by  £  in.  under  the  influence  of  a  stress, 
what  is  the  strain  ?     If  the  rod  is  2  sq.  in.  in  section,  and  the  load  1  1,000  Ibs., 
what  is  the  modulus  of  elasticity  ?    Ans.  '000417  ;  13,200,000. 

12.  Find  the  stress  produced  in  *  pump-rod  4"  diameter,  lifting  a  bucket 
28"  diameter  if  the  pressure  on  the  top  of  the  bucket  be  6  Ibs.  per  square 
inch  in  addition  to  the  atmosphere,  and  the  vacuum  below  the  bucket  be 
26"  by  gauge.  Beckon  each  2"  of  vacuum  =  i  Ib.    Ans.  931  Ibs.  per  sq.  inch. 

13.  If  the  rod  in  question  is  5'  long,  find  its  extension  if  the  modulus  of 
elasticity  =  9,000,000.     Ans.  '006  inch. 


LECTURE   XXII. — QUESTIONS.  313 

14.  What  do  you  understand  by  the  terms  tensile,    compressive  and 
thearing  strength  respectively  of  any  material?    Define    "modulus    of 
elasticity."     If  a  wrought-iron  bar  of  I  square  inch  sectional  area  just 
breaks  under  a  tensile  stress  of  60,000  Ibs.,  what  would  be  the  area  of  the 
section  of  a  tie-rod  which  would  just  support  a  load  of  20  tons? 

Ans.  75  sq.  inch. 

15.  A  wrought-iron  tie  bar,   f  inch  in  diameter,   has  a  modulus  of 
elasticity  of  28,000,000  Ibs.  per  square  inch.     Its  length  is  23  inches  ;  find 
the  load  under  which  the  bar  will  extend  '015  of  an  inch.     Find  also  the 
stress  per  square  inch. 

Ans.  8067*6  Ibs.  and  18,261  Ibs.  per  sq.  inch. 

1 6.  How  would  you  find  out  for  yourself  the  behaviour  of  steel  wire 
loaded  in  tension  till  it  breaks  ?    What  occurs  in  the  material  ?     Use  the 
words  "stress"  and  "strain"  in  their  exact  senses. 

17.  An  iron  rod,  of  i  inch  diameter  and  12  feet  in  length,  stretches 
3/32-inch  under  a  load  of  6  tons  suspended  at  its  extremity.     Determine 
the  stress,  strain,  and  modulus  of  elasticity  of  the  bar.     Ans.  17, 112*3  Ibs. 
per  sq.  in.  ;  0.00065  ;  E  =  26,526,615  Ibs.  per  sq.  in. 

1 8.  What  do  you  mean  by  Stress,  Strain,  and  Modulus  of  Elasticity  T 

A  wire  10'  long  and  |  sq.  inch  in  sectional  area  is  hung  vertically,  and 
a  load  of  450  Ibs.  is  attached  to  its  extremity,  when  the  wire  stretches 
•015"  in  length.  What  are  the  stress  and  strain  respectively?  And  also 
the  modulus  of  elasticity?  Ans.  3600  Ibs.  per  sq.  inch;  -000125; 
E  =  28,800,000  Ibs.  per  sq.  inch. 

19.  An  iron  wire  is  loaded  with  gradually  increasing  tensile  loads  till  it 
breaks.     We  want  to  know  its  modulus  of   elasticity,  its  elastic  limit 
stress,  and  its  breaking  stress.     What  measurements  and  calculations  do 
we  make?     (S.  E.  B.  1900.) 

20.  Sketch  apparatus  and  describe  a  laboratory  experiment  by  which 
you  could  find  E,  Young's  modulus  of  elasticity,  for  an  iron  wire  10  feet 
long  and  0*05  inch  diameter.     How  would  you  secure  the  upper  end  of  the 
wire  ?     How  apply  the  load  ?    And  how  measure  the  elongation  ?     How 
would  yon  plot  your  results  and  how  deduce  the  value  of  E? 

(S.  E.  B.  1902.) 

21.  Describe  an  experiment  by  which  you  could  determine  E,  Young's 
modulus  of  elasticity,  by  stretching  an  iron  wire.     (B.  of  E.  1903.) 

22.  A  bolt  z\  inches  diameter  has  a  tensile  load  of  30  tons,  what  is  the 
stress  ?    What  is  the  strain  if  Young's  modulus  of  elasticity  is  3  x  io7 
pounds  per  square  inch  ?    What  is  the  elongation  of  a  part  which  when 
unloaded  was  102  inches  long  ?  (B.  of  E.  1904.) 

Ans.  Stress  =  13,689*8  Ibs.  per  sq.  inch.     Strain  =  '000456. 
Elongation  =  '0465  inch. 

23.  Two  bars  of  equal  length,  both;of  rectangular  section  but  of  different 
materials,  are  firmly  riveted  together  at  their  ends  and  subjected  to  a  pull 
so  that  they   are   compelled  to   stretch   the  same    amount.     If  A1?  A3 
represent  their  sectional  areas,  and  Elf  E2  the  values  of  Young's  modulus 
for  the  two  materials,  show  that  when  the  pull  is  P  Ibs.  the  intensities  of 
stress  induced  in  the  two  bars  are 

PE,  PE2 


and 


A!  Ej  +  A2  Ba  Ax  Ej  +  A2  E2 

respectively — the  limits  of  elasticity  being  not  exceeded. 

(C.  &  G.,  1004,  0.,  Sec.  B.) 


3*4 


NOTES  AND   QUESTION'S. 


24.  A  bar  of  mild  steel,  of  rectangular  section,  is  2  inches  wide  and 
J  inch  thick,  and  is  10  inches  long.   If  Young's  modulus  is  12,500  tons  per 
square  inch,  find  the  amount  the  bar  stretches  when  the  load  on  it  is 
10  tons.     How  much  work  is  then  stored  up  in  the  bar  ? 

Ans.  I  =  -008  inch  ;  work  stored  up  in  bar  =  89*6  in.-lbs. 

(C.  &  G.,  1905,  0.,  Sec.  B.) 

25.  An  iron  column  is  12  inches  in  external  diameter,  and  the  metal  is 
i ^  inch  thick.     The  load  on  the  column  is  125  tons.     What  is  the  com- 
pressive  stress  in  the  metal  ?    By   what   amount   will   the  column   be 
shortened,  if  its  length  is  15  feet,  and  if  Young's  modulus  of  elasticity  is 
12,500,000  pounds  per  square  inch? 

Ans.  fc  =  6630  Ibs.  per  square  inch  ;  I  =  -095  inch.  (B.  of  E.  1905.) 

26.  The  following  results  were  obtained  during  a  tensile  test  of  a  mild 
gteel  bar  f  inch  in  diameter  : 


Total     load    on^| 

the      bar     in  V 

0-88 

176 

2-64 

S-Sa 

4-40 

5-28 

6-16 

7-04 

tons     .        .  J 

Elongation     on"\ 
a    length    of  1 
8  inches  stated  j 

0-0012 

O'O024 

0-0035 

0-0047 

0-0061 

0-0075 

0-0088 

0-0102 

in  inches      .  J 

(a)  Plot  a  curve  on  squared  paper,  going  evenly  through  the  points,  to 
show  the  relation  between  the  load  and  the  elongation. 

(b)  Find  the  load  necessary  to  cause  an  elongation  of  0*0040  inch. 

(c)  Find  the  total  work  done  in  inch-tons  upon  this  8"  length  of  the  bai 
during  the  test.  (B.  of  E.  1905.) 

Ans.  (b)  Load  =  3-0^  tons. 

(c)  Total  work  done -0-036  inch-ton. 


NOTES    AND    QUESTIONS. 


LECTURE  XXIII. 

CONTENTS. — Stresses  on  Chains — Shearing  Stress  and  Strain — Example  I. 
—  Torque  or  Twisting  Moment  —  Torsion  of  wires  —  Table  giving 
the  strength,  moduli  of  Elasticity  and  Kigidity  of  various  materials — • 
Strength  of  Solid  Kound  Shafts— Example  II. — Table  giving  the 
Horse-Power  which  steel  shafting  will  transmit  at  various  speeds — 
Strength  of  Hollow  Kound  Shafts— Kelation  between  the  Twisting 
Moment  and  Horse-Power  transmitted  by  shafting,  as  well  as  the 
diameter  necessary  to  transmit  a  given  Horse-Power — Examples,  III. 
IV, — Questions. 

IN  this  Lecture  we  will  continue  the  subject  of  "  strength  of  mate- 
rials," and  finish  the  course  with  reasons  for  the  shapes  generally 
given  to  sections  of  cast  iron,  wrought  iron,  and  steel  girders. 
^  Stresses  on  Chains.— The  only  stress  to  which  the  sides  of  the 
links  of  chains  are  subjected  under  ordinary  circumstances,  is  that 
of  tension.  This  stress  tends  to  bring  the  sides  of  the  links  closer 
together,  and  consequently  we  find  that  large  chain  cables  for 
mooring  ships  (where  very  sudden  and  severe  stresses  are  encoun- 
tered) have  a  cast-iron  stud  or  wedge  fitted  between  the  inner 
sides  of  the  links.  These  studs  most  effectually  keep  the  sides  of 
the  links  apart,  and  prevent  any  link  jamming  a  neighbouring 
one.  They  add  materially  to  the  strength  of  the  chain,  for  they 
are  in  compression  whilst  the  sides  of  the  links  are  in  tension. 
Being  composed  of  cast  iron,  which  offers  the  immense  resistance 
to  compression  of  fully  45  tons  per  square  inch,*  there  is  not 
much  fear  of  their  giving  way  before  the  sides  of  the  links. 

The  strength  of  a  stud-link  may  be  taken  as  equal  to  double 
the  strength  of  a  rod  of  wrought  iron,  of  the  same  diameter  and 
quality  of  material  as  that  of  which  the  chain  is  composed,  whereas 
the  strength  of  an  open-link  chain  is  only  about  70  per  cent,  of  this 
amount,  even  with  perfect  we! ding. f 

In  Molesworth's  "  Pocket-Book  of  Engineering  Formulae,"  the 
student  will  find  at  page  54  a  formula  for  the  safe  load  on  chains 
viz. — 

W=7.i^ 

Where  W  =  Safe  load  in  tons. 
„         d  =  Diameter  of  iron  in  inches. 

*  See  Table  of  the  Ultimate  Strengths  and  Safe  Working  Loads  given 
In  Lecture  XXII. 
t  Some  well-known  authorities  give  less  than  70  per  cent 


SHEARING    STRESS    AND    STRAIN.  317 

Now,  Btich  a  formula  is  very  easy  of  application,  but  the  student 
should  never  rest  content  until  he  finds  out  how  the  constants 
have  been  arrived  at,  and  what  relation  the  various  symbols  have 
towards  each  other.  If  he  refers  back  to  the  short  table  of 
"  Ultimate  Strengths  and  Working  Loads  "  given  in  the  previous 
Lecture,  he  will  find  opposite  wrought-iron  bars  and  under  tension, 
the  value  5  tons  per  square  inch  as  the  safe  working  load.  Con- 
sequently, applying  what  was  said  above  about  perfect  stud-link 
chains,  he  will  see  that — 

rtwice  the  load  of  a  rod  of  the  same  diameter  and 
quality  as  that  of  which  the  chain  is  composed. 

.».  W  =  2  x  5  x  cross  area  of  the  chain  iron. 

W=»2  x  5  x-d*=2*  5x^x^^  =  7-8^ 
4  7 

This  is  near  enough  to  the  constant  given  by  the  above  empirical 
formula  to  enable  him  to  see  how  it  has  been  obtained. 

Chains  which  are  subjected  to  many  sudden  jerks  (such  as  lift- 
ing chains  for  cranes  and  slings)  become  in  time  crystalline,  or 
short  in  the  grain,  and  consequently  brittle  and  unsafe.  The 
best  precaution  to  adopt  in  order  to  periodically  remove  this  en- 
forced internal  condition,  is  to  draw  them  once  a  year  very  slowly 
through  a  fire,  thus  allowing  them  to  become  heated  to  a  dull  red, 
and  then  to  cool  them  slowly  in  a  heap  of  ashes.  This  method 
is  followed  at  Woolwich  Arsenal  and  some  other  Government 
works. 

Shearing  Stress  and  Strain. — The  action  which  is  produced 
by  shearing  and  punching  machines  on  iron,  steel,  or  copper  plates, 
&c.,  is  to  force  one  portion  of  the  metal  across  an  adjacent  portion. 
The  shearing  stress  is  the  reaction  per  square  inch  opposing  the 
load  or  pressure  applied  to  the  shears  or  punch,  and  the  shearing 
strain  is  the  deformation  per  unit  length  or  volume.  Rivets 
holding  boiler  plates  together,  fulcra  of  levers,  the  pins  of  the 
links  of  the  chain  of  a  suspension  bridge,  the  cotter  keys  of  a 
pump  rod,  are  all  subjected  to  shearing  stresses  and  strains.  The 
ultimate  and  the  working  shearing  stresses  for  a  few  engineering 
materials  were  given  in  a  table  in  Lecture  XXII. 

In  the  case  of  loaded  beams  (which  we  will  consider  shortly  in 
connection  with  bending  moments)  the  shearing  force  at  any  point 
or  any  transverse  section  thereof  is  equal  to  the  algebraical  sum 
of  all  the  forces  on  either  side  of  the  point  or  section. 

EXAMPLE  I. — A  steel  punch  i"  diameter  is  used  in  a  large 
shipyard  punching  machine  to  make  holes  in  steel  plates  i"  thick. 
What  will  be  the  total  shearing  stress  or  least  pressure  required  ? 


3r8 


LECTUKE   XXI1T. 


ANSWER. — Referring  to  the  table  in  last  Lecture,  we  see  that  the 
ultimate  shearing  strength  or  shearing  stress  for  steel  bars  (which 
we  will  assume  to  be  the  same  as  for  plates)  is  30  tons  per  square 
inch.  Now  a  hoi  3  i"  diameter  has  a  circumference  =  ird 
=  3*  14",  and  since  the  plate  is  i"  thick,  the  area  of  the  resisting 
section  must  be  the  circumference  of  the  hole  x  its  depth,  or 
=  3' 14"  x  i"  =  3'i4sq.  in.,  and  the  total  pressure  required  =  30  tons 
x  3-14  =  94-2  tons. 

Torque,   or  Twisting  Moment.* — In  the  case   of  a   shaft 
having  a  lever,  pulley,  or  wheel  fixed  to  it  with  a  force  P  Ibs., 
applied  at  radius  R  feet  from  the  centre  of  the  shaft,  then 
The  twisting  moment  T.M.  is  =  P  x  R  Ibs.-feet. 

Or,        The  torque  =  P  x  R  x  12  Ibs.-inches. 

*  The  term  torque  was  devised  by  the  late  Professor  James  Thomson,  of 
Hlasgow  University,  to  signify  the  twisting  or  torsional  moment.  The 
Ibs. -feet  of  torque  must  not  be  confused  with  ft. -Ibs.  of  work  ;  or  with 
resilience  (which  is  the  work  done  in  straining  a  body,  as  measured  by  the 
elongation  or  compression  in  feetxthe  mean  load  causing  the  strain).  It 
will  therefore  save  confusion,  if  we  take  the  force  applied  at  the  end  of 
the  arm  in  Ibs.,  and  the  leverage  or  arm  in  inches,  and  then  multiply  them, 
so  as  to  get  the  torque  in  Ibs. -inches. 


APPARATUS  FOR  SHOWING  THAT  THE  ANGLE 

OF  TORSION  IN  ROD  is  PROPORTIONAL  TO 

ITS  LENGTH,  AND  THE  VARIATIONS 

IN  THE  RIGIDITY  OF  DIFFERENT 

MATERIALS. 


TORSION  OF  WIRES. 

Torsion  in  Bods.— It  will  be  seen  from  the  previous  figure,  that  the 
front  puller  carries  a  cord  with  known  weights  suspended  from  it.  The 
spindle  of  this  pulley  moves  in  ball  bearings,  and  carries  at  its  inner  end  a 
three-jaw  chuck.  This  chuck  is  for  holding  one  end  of  the  rod  under  test, 
whilst  the  «ther  end  is  for  tightly  clamping  it  to  the  back  bracket  of  the 
machine.  The  torsional  couple  is  applied  by  means  of  weights  hung  from 
the  afore-mentioned  cord,  and  the  torsion  in  degrees  is  read  off  on  the  dial. 
To  eliminate  errors  the  weight  should  be  hung  first  on  one  side  of  the 
pulley,  then  on  the  other  side,  and  the  mean  of  the  two  readings  taken. 
It  will  be  noticed,  that  both  the  pointer  and  the  scale  can  be  readily  moved 
to  any  distance  from  the  fixed  end  of  the  rod. 

It  can  thus  be  shown  that  the  angle  of  torsion  in  a  rod  is  proportional 
to  its  length  and  that  different  materials  have  different  rigidities. 

Torsion  of  Wires. — In  the  figure,  AB  represents  a  wire 
held  firmly  at  the  top  end  of  a  supporting  rod,  which  is  8  feet 
long.  A  pulley  is  fixed  firmly  to  the  wire  at  B,  and  this  pulley 
is  acted  upon  by  two  cords  which  tend  to  turn  it  without  moving 
its  centre  sideways,  i.e.,  they  act  on  the  pulley  with  a  turning 
moment  only.  But,  the  pulley  can  only  turn  by  giving  a  twist 
to  the  wire.  Hence,  if  a  light  pointer  be  fastened  to  the  wire  at 
0,  the  former  moves  over  a  dial,  and  the  angle  turned  through  by 
the  wire  and  the  pointer  is  called  the  total  angle  of  twist  at  0. 
Similar  pointers  fixed  to  the  wire  at  D  and  E  give  the  angle  of 
torsion  at  these  points.  The  dials  at  D  and  E  are  supported  upon 
adjustable  sliders,  so  that  they  may  be  moved  up  or  down  the 
vertical  rod  in  order  that  the  torsion  of  any  length  of  rod  may  be 
measured. 

(i)  If  the  length  AE  be  i  foot,  and  the  distance  between  the 
dials  E  and  D,  D  and  C,  be  also  i  foot,  then  we  should  find  that  the 
angles  of  twist  at  E,D  and  0  are  as  i  :  2  :  3  respectively.  That 
is,  the  angle  of  twist  is  proportional  to  the  length  of  wire  twisted. 

(ii)  By  varying  the  twisting  moment  and  noting  the  angle  of 
twist  which  is  produced  at  each  variation,  you  can  prove  that  the 
angle  of  torsion  or  twist  is  proportional  to  the  twisting  moment. 

(iii)  If  you  try  different  sizes  of  wires  of  the  same  material  and 
length,  and  apply  to  each  of  them  the  same  twisting  moment,  then 
you  will  find  that  the  amount  of  twist  produced  in  them,  will  be 
inversely  as  the  fourth  power  of  the  diameter  of  the  wires. 

(iv)  If  you  take  wires  of  the  same  diameter  and  length,  but  of 
different  materials,  and  apply  the  same  twisting  moment  to  them, 
then  you  will  find  that  the  amount  of  twist  will  be  inversely  pro- 
portional to  the  modulus  of  rigidity  of  the  material. 

Note,  the  Modulus  of  Rigidity  of  a  Material  in  Ibs.  per  square 
inch  may  be  defined  as  the  ratio  of  shearing  stress  to  shearing 
strain.  This  constant  or  coefficient  of  shearing  elasticity  for  each 
particular  material  is  indicated  in  the  following  table  by  the 
letter  0. 


320 


LECTUEE  XXIII. 


EXPEEIMBNTAL  APPAEATUS  FOB  MEASURING  TH] 

TOESION  OF  WIRES. 


STEENGTH   OF  SOLID  ROUND   SHAFTS. 


321 


BTBBNGTH,  MODULI  OF  ELASTICITY,  AND  RIGIDITY  OP  VARIOUS 
MATERIALS. 


Materials. 

Breaking  strength 
to  resist  tension, 
n  tons  per  square 
inch. 

Modulusof  elasticity, 
E,  in  tons  per 
square  inch. 

Modulus  of  rigidity. 
C,  in  tons  per 

square  inch. 

Aluminium   -  bronze 

) 

(90%    copper    Und 

40 

6500 

2500 

10%  aluminium)     . 

J 

Brass  wire       .     .     • 

20  to  25 

5000  to  6500 

2OOO  tO  23OO 

Oast-iron    .... 

5  {o  15 

4500  to  7000 

1700  to  2700 

Charcoal-iron  (hard- 

V    "?S  to  40 

drawn)      .... 
Charcoal-iron 

I         OJ              *T 

\ 

12,500  to  13,500 

5000  to  5500 

(annealed)         .     . 

}          30 

Copper  (cast)  .     .     . 

8  to  12 

5000  to  6000 

1900  to  2300 

(rolled)   .     . 

13  to  16 

5500  to  7500 

2100  tO  2900 

„   wire  (annealed) 

18  to  20 

— 

— 

„      „  (hard-drn.) 

26  to  30 

— 

— 

Delta  Metal  (forged) 

22  tO  24 

6350 

234O 

German-silver  wire  . 

30 

4800 

l8OO 

Gun-metal  (90%  cop- 
per and  10%  tin)    . 
Muntz  metal  (rolled 
or  forged)     .     .     . 

V      12  tO  17 

\      22 

5000 

6350 

IQOO 
2340 

Phosphor  bronze 

16  to  18 

) 

„              „     wire 
(hard-drawn) 

J-     45  to  70 

>•    6000  to  7000 

2300  to  2700 

Platinum    .... 

— 

10,500 

4000 

Steel  (ordinary)  .     . 
„     (annealed)  .     . 

7o 

IOO 

\  13,000  to  14,000 

5200  to  5700 

Strength  of  Solid  Round  Shafts. — It  is  evident  trom  the 
above,  that  a  shaft  subjected  to  a  twisting  moment  must  offer  a 
sufficient  resistance  thereto,  otherwise  it  would  be  twisted,  or 
sheared,  or  ruptured  through  by  the  torque.  It  may  be  proved 
that  in  the  case  of  solid  round  shafts  their  resistance  to  torsion 
is  directly  proportional  to  the  cubes  cf  their  diameters  when  made  of 
the  same  material  and  quality.* 

*  This  is  evident  from  the  fact  that  the  shaft  must  offer  a  moment  of 
resiitance,  or  shearing  moment,  equal  to  the  twisting  moment  at  the  instant  of 
rupture.  Now,  the  area  to  be  sheared,  is  the  cross  area  of  the  shaft 

=!LDa.  where  D  is  the  diameter  of  the  shaft.     The  mean  arm  or  leverage  at 

4 

which  this  resistance  acts  is  equal  to  half  the  radius  of  the  shaft,  for  at  the 
centre  the  arm  is=o,  and  at  the  circumference  it  is=r,  the  radius  of  the  shaft. 


The  mean  arm  is  therefore=  — 


And,  if  the  shearing  resistance  per 


tauwre  inch  of  cross  section  of  the  material  be=/,  the  product  of  these 
three  quantities  will  be  the  total  thearing  moment,  and  must  equal  the 
twitting  moment — viz.  =P  x  R,  where  P  is  the  force  applied  at  the  end  of  the 


322  LECTURE   XXIII. 

Let  Dj,  !>„  D,  -  Diameters  of  three  shafts,  i",  2",  and  3"  dia- 

meter respectively. 
Tj,  Tr  T3  —  Torques  which  they  will  respectively  resist 

when  stressed  to  the  same  extent. 
Then,        .        .        T^T.rT.iiD/iD/.-D/ 
Or,    .        .        .        T^T,:!,::  i«  :  2'    :  3« 

::i    :  8   :  27. 

In  -other  words,  the  strengths  of  the  three  solid  shafts  will  be  as 
1:8:27. 

A  good  wrought-  iron  shaft  of  i"  diameter  has  been  found  to 
withstand  a  torque  of  800  lbs,-ft.,  or  9600  Ibs.-inches,  which  means 
that  they  will  resist  800  Ibs.  force  at  i  foot,  or  12"  leverage,  or 
400  Ibs.  at  2  feet,  or  24",  and  so  on. 

Or,         .     P  x   R'    =     800  Ibs.-feet  of  torque 
i.e.,        .     P  x  R"  =  9600  Ibs.  -inch  torque. 
EXAMPLE  II.  —  On  the  above  basis,  what  force  acting  at  the 
circumference  of  a  pulley  20"  diameter  will  break  a  wrought- 
iron  shaft  2"  diameter? 

ANSWER.  —  By  the  above  rule  we  have  the  proportion  : 

TI:T,  -D^iD/ 

But  Tj  -  P!  x  R/'  =  800  Ibs  x  i  a" 
AndT,=  P8  x   R8"  -  P,  x   10" 

...    P^:  P2R,  ::  J>*:J>f 
i.e.,  PaRa  x  D^  =  P^  x  D,1 

P.R.  x  D,8     800  x  12"  x  8 

<0r>    •    •     p.  -  iy  x  R,°  -nr^"  '  76801bs 

lever  or  circumference  of  the  pulley,  and  K  the  length  of  the  arm  or  radius 
of  the  wheel  or  pulley. 

Consequently,  P  x  E  =-J^(  ^-D2  x  -  \  =fg  "-  '  W 

But  S  is  a  constant  quantity  for  any  particular  material.  Also,  TT  and  16 
are  constants.  .•.  P  x  R  vary  as  D3. 

At  the  instant  of  rupture  the  strength  of  the  shaft  just  balances  or  is 
equal  to  the  twisting  moment  P  x  R. 

.*.  The  strength  of  shaft  varies  as  D8. 

This  is  the  same  as  the  general  statement  in  the  text  above.  Without 
Borne  such  algebraical  explanation,  students  are  sorely  puzzled  how  the 
cube  of  the  diameter  crops  up  ;  or  still  more  so  when  they  see  the  following 
which  appears  in  some  text-books. 


:Such  a  statement  is,  however,  quite  evident  after  the  above  analysis.  (We 
must  leave  the  consideration  of  hollow  shafts,  tubes,  &c..  to  our  Advanced 
Course.) 


H.P.  TRANSMITTED  BT  STEEL  SHAPTa 


323 


§ 


5-1     C? 


02 


I 


JOd  'SAag 


O    W    Tj-vO  OO    O    CM    'i-vO  OO    O    CM    •**-$  00    OvoOvoQVOOvoOvoO    VO  < 
MMMMIH<ieteiaCICQCOCOCOrO*»Ttn  vovO  VO    tx  1x00  00    Ov  OM 


ONVOMVO    WOO    TfO   VOM    fx  CO  ON  •*}-  O  vO    O   vo  ON  r*-  ON  COOO    C*    tx,  C1  <O    O 

££8?  5,3,8  £#cT2  3S3  |&lf  i&£SfH§f5' 

MMtHHMHMMOOIOiOIOIMCOCO^^-^LOVO  vovO  vO  vO    Cx 


•*  tx  O\  CI  1-v£>  O\  H  -<j-\ 
IH  M  M  CH  W  M  CJ  COCO 
O  tvOO  CS  O  i-i  «  CO  ^*- 


00    M    CO  "ICO 


•<J-  OvO  C1  00    -<1-  O  VO    «  00    •<*•  O 

OvO    M  K.WOO    •^•ON'OO^O    01 

CO  »OOO  O    CO  lOOO    O    COVO  OO    H 

«  «  oi  cocococoTi-^-Ti-Ttvn 


CO  C1  O  Ov  CxVO  vo  CO  «  O  Ov  fxvO  VO  CO  «  CO  vovO  00  O\  M  C1  TJ-  LO  CxOO  O 
•*MOO  *•  MOO  VOW  0\vO  N  ONO  COO  CXTj-MOO  VON  O  tx-t-MOO  VOCO 
CO  TJ-  TJ-  vovO  vO  txOO  OOO\OOH«COCOvo  C^.00  O  N  Tj-iOtxOvO  N  J* 


aize. 


"g     I  "8 


\r>  O   lOO   ")O   »OO   *O  O   »OO   lOO    lOO    CO  «OOO    O   CO  U10O    O    CO  lOOO    O 

ci  tot^O  w  lotxo  M  iotxO  oi  lOt^OO  woo  »o  M  r^cogvb  o»oo  ^o 

M    H    M    CI    «    0*    01    COCOCOCO-'i-Ti-rl-Tj-iO  lOvO  \O    txCO  00    Os  O    O    w    HI    M 


'«  « 


Ot   01   COCOCO^-^trfvO' 


& 


Scl 


•3 

1 

I 
a 

\ 


vp  op  ON  p  «  ci  ^  jovp  oo  p\  p  M  co  t^  yo  co  M  ON  txvp  .""t-  w  p  op  vp  ^  ji 

VOOO    M   vooo    M   Vf  Vx  O    COvb    O    COvb    ON  C1    O  00   vo  CO  HI    O\  Cx,  vo  W    O  00  vb 
M    HI    C1    «    01    COCOCO^^-^vovovo  vovO    tx  txOO    ONO    O    HI    Ot    COrfrfvo 


ONM   ti 


'  .M  f*  y^^p  <p  P  f  P  i°  p  7*"°°  r°  r*  j1  *p  p  >°  °^  ?*& 


For 


co^yoyovp  fxoo  ON  ONO  M  H  «  p  -<f  ^ M  oo  joeioo  yoei  ONVO  «  ON\p 

*O<O   "^00    ON  O    HI    C1    CO  vovb    txOO    ON  O    M   Vfvb   ON  01   V  Vx  O    C1   VOOO    O   CO 
SMMMMMI-IIHHIOI««««COCOCO^-^^TJ-VOVO 


fO  O  Jx  jf  O  tx  ;<•  M  t .  rj-  M  op  JO  «  00  JO  «   ONvp  CO  ONvp  «  O  Cx  .•*  M  t* 
CO  *•  <*•  vovfl  VO    txOO  WONOOMC1C1CO  vovO  00    O    HI   CO  VO  1x00    O    Ct  CO 


S  8 


18 


324  LECTURE  XXIII. 

Strength  of  Hollow  Bound  Shafts.—  The  shafts  of  large  land 
and  marine  engines  are  sometimes  made  hollow.  The  ratio  between  the 
torsional  strength  of  a  solid  round  shaft  and  a  hollow  one  i§  as  follows  :— 

Let  D  =  outer  diameter  of  either  shaft. 

d    =  inner  diameter  of  the  hollow  shaft. 

D4-d4 
Then,  their  strengths  are  as  D8  :  -  jj-  , 

and  their  weights  are  as  D«  :  Da  -  &. 

Relation  between  the  Twisting  Moment  and  Horsa 
Power  Transmitted  by  Shafting,  as  well  as  the  Diameter 
Necessary  to  Transmit  a  given  Horse  Power. 

Let  P=the  force  of  the  twisting  couple,  either  constant  or  the 
„  mean  value  if  variable,  in  pounds. 

„     R=the  length  of  lever  arm  of  the  twisting  couple  in  feet. 
„     N  =  revolutions  of  shaft  per  minute. 
,,T.M.  =  mean  twisting  moment  =  PxR  Ib.-feet. 
„     /,  =  shearing  stress  per  square  inch  on  cross-section  of  shaft. 

Then,  T.M.  =  resisting  moment  =       dsfs  =  -i<)6d%. 


But,    work  done  per  minute  =  P  x  27rRN  =  T.M.  x  2irN  ft.  -Ibs. 
Also,  work  done  per  minute  =  H.P.  x  33,000  ft.  -Ibs. 
.-.   T.M.X27rN  =  H.P.  x  33,000 

Or,          T.M.  =  H.P.x  33.000  ^^  =  12  x  33,000  xH.P.  lbB<.inoheB< 

2?rN 


Hence,    T. M.  =  63, 030*^1  Ibs.-inches  =  63°3°  x  H'TP-  =  28  H'F>  ton-inches. 
N  2240  x  N  N 

But,  T.M.=^|&. 

Hence,        ^'=63,03^ 


If  we  assume  the  safe  values  of  the  stress  ft  to  be  as  follows  :  cast-Iron  • 
3600  Ibs.  per  square  inch  ;  wrought-iron  =  9000  Ibs.  per  square  inch  ; 
and  steel  *  13,500  Ibs.  per  square  inch,  then  the  diameter  in  inches  for  a 
round  shaft  in  terms  of  the  horse-power  to  be  transmitted  is,  for  Oast-iron 


Of  course,  the  twisting  moment  is  here  assumed  to  remain  constant  at  its 
mean  value.  In  practice  the  twisting  moment  varies  in  many  cases,  and 
to  allow  for  this  it  is  usual  to  take  the  maximum  twisting  moment  from 
i  -3  to  1*5  times  the  mean  twisting  moment,  thus  the  values  of  diameter  d 
as  found  above  are  slightly  increased. 

It  should  also  be  borne  in  mind  that  shafts  in  practice  are  subjected  to 
bending  as  well  as  twisting,  owing  to  the  loads  due  to  the  weights  of 


STRENGTH    OF   SHAFTS.  325 

pulleys  and  the  pulls  of  the  belts.     Hence,  if  the  usual  rule  for  the  dia-  \ 

3    /TT     p 

meter  of  a  wrought-iron  shaft  is  <*=3'3/y  — ^7- -  when  torsion  only  is  con- 
sidered, then  it  wiU  be  d=ex3'3/y  S^1  when  tending  is  taken  into 
account.  Some  values  of  the  coefficient  c  are  given  in  the  following  table : 


Kind  of  Shaft. 

Value  of  c. 

Propeller  shafts  of  steamships,  and  shafts  with  similar 
load      ... 

ria 

Line  shafting  in  mills,  etc  

i  '3 

Crank-shafts  and  shafting  subjected  to  shocks,  such  as 

i  -42 

EXAMPLE  III.—  (a)  Find  the  diameter  of  a  solid  steel  propeller  shaft  to 
transmit  12,000  H.P.  at  80  revolutions  per  minute.     (6)  If  the  shaft  is  to 
be  hollow,  find  its  external  diameter,  from  strength  considerations,  when 
its  internal  diameter  is  two-thirds  of  its  external  diameter. 
Answer.—  Let    D1=r  diameter  of  the  tolid  steel  propeller  ihaft. 

„      D«=  outer  diameter  of  the  hollow  steel  propeller  shaft. 
„        a  =  inner  diameter  of  the  hollow  steel  propeller  shaft. 
„        N=  number  of  revolutions  per  minute  of  shaft. 
(a)  Keferring  to  the  previous  article  on  the  relation  between  the  dia- 
meter of  shaft  necessary  to  transmit  a  given  horse-power,  we  deduced  the 
following  formula  for  a  solid  round  steel  shaft,  when  the  stress  //   was 
assumed  as  13,500  Ibs.  per  square  inch. 


Substituting  the  numerical  values  given  by  the  question,  we  get  — 
.  =2-9  //?=2-9/i5o=  15-4  inches. 


But,  to  allow  for  variations  in  stress,  bending  moments,  and  shocks  to 
which  the  propeller  shaft  may  be  submitted,  we  find  from  the  table  that 
the  value  of  the  coefficient  c  is  1*13. 

Hence  Dl  =  cd=  1-13  x  15*4=  17*4  inches  for  the  solid  round  shaft. 

II  P 
(6)  From  the  above,  we  see  that  <P=(2-g)*  -        Also,  from  the  relation 


of  the  strength  of  solid  to  hollow  round  shafts,  we  get,  d?  : 


~ 

~    •  =  (2-9?  5^-  for  hollow  steel  shafts  subjected  to  torsion  onfy~> 
D4-d4  H.P. 


326  LECTURE  XXIII. 

Jfow,  substituting  the  values  given  by  the  question,  we  get— 


Or,         iDa*= 

».«.    Pa8= 3x2158-5       =6475-5. 

.  • .  Da=  #6475-5       =18-64  inches. 
If  bending  is  taken  into  account,  then 

D2=  1-13 X  18-64    =21  inches  for  the  hollow  round  shaft. 

EXAMPLE  IV. — The  screw  shaft  of  a  marine  engine  is  10  ins.  diameter, 
and  the  revolutions  100  per  minute.  It  is  replaced  by  twin  screw  shafts 
rotating  500  times  a  minute.  If  the  total  horse-power  developed  in  the 
two  cases  be  the  same,  and  the  working  stress  is  also  the  same  in  the  twin 
screw  shafts  as  in  the  single  screw  shaft,  find  the  proper  diameter  of 
shafts  in  the  second  case  and  compare  their  weights. 

(C  .&(*.,  1905,  O.»3a,fr, 

Answer. — 

Let  D!  =  diameter  of  single  screw  shaft. 
n    D2  =  diameter  of  one  of  the  twin  screw  shafts. 
,,    N!  =  number  of  revs,  per  minute  of  single  shaft. 
„    N2  =         „  „  „  twin  screw  shafti. 

H.P.  =  total  horse-power  to  be  developed  in  each  case. 
ft.  =  working  stress  in  Ibs.  per  square  inch. 

Now,  Dj  =  \/ $'i  x  ^ ; ~  * 

and,  D,  =  //„  x  "  x  33-o°°  J*f. 


5-1  x  12  x  33,000  x  H.P. 

.-.          IV  =    ZTTNj/ 

Da8      5-1  x  12  x  33,000  x  H.P. 


Or 


in" 

i.e.,     Dx  =  1-442  D2.  .  •.  Da=  ~  =  7" 

But,  the  weights  of  shafts  are  proportional  to  their  cross  sectional  areas 
if  their  lengths  are  equal. 

|  Dja      (1-442  D2)a  _  2-08      1-04 

Hence,  ^-,  =  -^r~       ~T      ~  ' 

Therefore,  the  weights  of  the  shafts  in  the  two  cases  are  approximately  the 
same,  but  the  diameter  of  th^  single  screw  shaft  is  nearly  i£  times  the 
diameter  of  one  of  the  twin  screw  shafts. 


LECTURE   XXm. — QUESTIONS.  327 


LECTURE  XXIII.— QUESTIONS. 

1.  An  open  link  chain  is  constructed  of  round  wrought-iron  rod,  §  inch 
in  diameter  ;  calculate  what  is  the  probable  breaking  load  of  the  chain. 
Wrought-iron  chains  are  liable  to  deterioration  by  constant  use  ;  what 
change  do  they  undergo,  and  what  precaution  is  taken  to  prevent  their 
breaking  1    Ans.  io£  tons. 

2.  A  steel  punch  £  inch  in  diameter  is  employed  to  punch  a  hole  in  a 
plate  f  inch  in  thickness.     What  will  be  the  least  pressure  necessary  in 
order  to  drive  the  punch  through  the  plate  when  the  shearing  strength 
of  the  material  is  35  tons  per  square  inch  ?    An*.  51*56  tons. 

3.  Define  what  is  meant  by  **  shearing  etress  and*  strain,"  "torqifc.  n- 
j~?1«tiDg  moment."    Show  by  an  example  that  a  shaft  subjected  to  torqu* 
iaars  a  shearing  stress  tending  to  sever  it  afe  right  angles  to  its  axis. 

4.  What  is  meant  by  the  twisting  moment  "  of  a  shaft  ?    If  a  wrought 
iron  shaft  i  inch  in  diameter  breaks  in  torsion  by  a  force  of  800  Ibs.  at  the 
end  of  a  lever  i  foot  long,  what  force  at  the  end  of  a  lever  2  feet  long 
will  break  a  shaft  of  the  same  material,  but  2  inches  in  diameter  1    Find 
also  the  diameter  of  a  wrought-iron  shaft  to  resist  a  force  of  2  tons  at  a 
distance  of  18  inches  from  its  centre.     Ans.  3200  Ibs.  2  inches  full. 

5.  If  a  shaft,  2  inches  in  diameter,  is  found  equal  to  the  transmission  of 
4  horse-power,  what  amount  of  power  can  be  transmitted  by  a  shaft  4 
inches  in  diameter,  all  other  questions  remaining  the  same  ? 

Ans.  32  horse-power. 

6.  If  a  revolving  shaft,  which  is  2  inches  diameter,  is  found  sufficiently 
strong  to  transmit  4  horse-power,  how  much  power  may  be  transmitted 
by  a  shaft  which  is  3  inches  in  diameter,  supposing  all  the  other  conditions 
to  be  the  same,  and  that  the  iron  of  both  shafts  is  subjected  to  the  same 
stress?    Ans.  13*5  H.P. 

7.  If  8co  Ibs.  at  the  end  of  a  12-inch  lever  be  a  safe  stress  to  apply  to  a 
wrought-iron  bar  one  square  inch  in  section,  find  the  effort  which  a  shaft 
2  inches  in  diameter  can  transmit  at  the  circumference  of  a  pulley  one 
foot  in  diameter,  and  making  300  revolutions  per  minute.     Find  also  the 
horse-power  transmitted.     Ans.  8893  Ibs.  ;  254  H.P. 

8.  If  a  wrought-iron  shaft  of  i  inch  diameter  is  broken  by  the  torsion  of 
a  load  of  800  Ibs.  acting  at  the  end  of  a  12-inch  lever,  find  the  weight 
which,  when  applied  to  the  end  of  the  same  lever,  would  break  a*shaft  of 
the  same  material,  but  3  inches  in  diameter.    State,  in  general  terms,  the 
reasoning  by  which  you  arrive  at  the  result.     Ans.  21,600  Ibs. 

9.  Suppose  that  a  shaft  of  i  inch  diameter  may  be  safely  subjected  to 
a  torque  of  2000  Ib. -inches;   what  torque  will  a  2j  inch  shaft   safely 
resist  ?    Calculate  the  horse-power  which  may  be  safely  transmitted  by  the 
latter  shaft  if  its  speed  is  150  revolutions  per  minute.         (B.  of  E.,  1902.) 

Ans.  22,780  Ib.  inches;  54  horse-power. 

10.  A  wire  of  Siemens'  steel  O'i  inch  diameter  is  to  be  twisted  till  it 
breaks.     Sketch  the  arrangement  and  show  how  the  angle  of  twist  and 
the  twisting  moment  are  measured,  how  the  results  may  be  plotted  on 
squared  paper,  and  the  sort  of  results  that  may  be  expected.     In  what 
way  may  a  wire  of  twice  this  diameter  be  expected  to  behave  f 

(B.  of  E.  1901.) 


328  LECTURE  XXIII. — QUESTIONS. 

n.  If  a  shaft  4  inches  in  diameter  will  safely  withstand  a  torque  of 
120,000  lb. -inches,  what  torque  would  a  g-inch  shaft  take?  What  H.P. 
would  the  former  transmit  at  200  revolutions  per  minute,  and  what  would 
the  latter  transmit  at  50  revolutions  per  minute  ?  (B.  of  E.,  1903). 

Ans.  Torque  =  1,367,000  lb. -inches  ;   H.Pa  =  3847  and  H.P.2  =  uio. 

12.  Compare  the  strengths   and  weights  of  a  solid  wrought-iron  shaft 
and  a  hollow  steel  shaft  of  the  same  external  diameter — assuming  the 
internal  diameter  of  the  hollo  w  shaft  half  the  external,  the  working  stress 
of  steel  ij  times  that  of  iron,  and  the  densities  of  wrought-iron  and  steel 
to  be  the  same.  (C.  &  G-.,  1903,  O. ,  Sec.  B.). 

^ns Strength  of  solid  W.I.  shaft_32_   i 

Strength  of  hollow  steel  shaft  ~  45  ~  1^4* 

Weight  of  solid  W.I,  shaft  _  4  =  ij 
Weight  of  hollow  steel  shaft"  3     T~" 

13.  The  propeller  shaft  of  a  vessel,  whose  engines  develop  1000  horse- 
power at  60  revolutions  per  minute,  is  8"  dia.     Assuming  the  shaft  sub- 
jected to  pure  torsion,  and  that  the  maximum  twisting  moment  on  the 
•haft  is  I J  times  the  mean,  estimate  the  maximum  shear  stress  induced  in 
the  shaft  (C.&G.,  1904,0.,  Sec.  B.) 

Ans.  Max  shear  stress  =  13, 050  Ibs.  per  sq.  inch. 

14.  The  figure  shows  the  skeleton  mechanism  of  a  direct-acting  steam- 
engine.      A   is  the  cross -head,   B  is  the  connecting-rod,  and  C  is  the 
crank. 

The  connecting  rod  is  4  cranks  long,  and  in  the  position  shown  in  the 
figure,  the  crank  has  turned  through  an  angle  of  45°  from  the  dead  centre 
in  a  clock-wise  direction. 

-  A  force  F,  due  to  the  steam  pressure  on  the  piston  of  12,000  pounds, 
acts  upon  the  cross-head.  Find  graphically,  or  in  any  other  way,  the 
thrust  in  the  connecting  rod,  and  the  magnitude  of  the  force  It  between 
the  cross-head  and  slide  bar.  All  friction  to  be  neglected. 

(B.  of  E.  1905.) 


Ans.  F  =  ^12,200  Ibs.  ;  B  =  12,430  Ibs.  ;  and  B  =  2220  Ibs. 

15.  Describe,  with  a  sketch  of  the  apparatus,  how  you  would  experi- 
mentally determine  the  law  connecting  the  twisting  moment  and  the  angle 
of  twist  for  a  mece  of  steel  wire.  (B.  of  E.  1905.) 


LECTURE  XXIY. 

CONTENTS.— Hooke's  Coupling  or  Universal  Joint— Double  Hooke's  Joint 
— Sun  and  Planet  VTheels  —  Cams— Heart  Wheel  or  Heart-shaped  Cam 
— Cam  for  Intermittent  Motion — Quick  Return  Cam— Example — 
Pawl  and  Ratchet  Wheel— Reversible  Pawl — Masked  Ratchet— Silent 
Feed— Watt's  Parallel  Motion— Parallel  Motion— Questions. 

IN  this  and  the  following  Lecture  we  shall  examine  a  few  of  the 
many  devices  for  transmitting  circular  motion  and  for  converting 
it  into  rectilinear  motion,  or  vice  versd,  together  with  other 
miscellaneous  mechanisms. 

Hooke's  Coupling  or  Universal  Joint.— This  is  a  contrivance 
sometimes  used  for  connecting  t\vo  intersecting  shafts.     Each  of 

the  shafts  ends  in  a  iork,  F1?  F,, 
which  embraces  two  arms  of  the 
crosspiece,  O.  The  four  arms  of 
this  cross  are  of  equal  length.  As 
Cj  rotates,  Fx  and  F2  describe 
circles  in  planes  perpendicular  to 
their  respective  axes.  Since  these 
planes  are  inclined  to  each  other 
the  angular  velocity  of  C2  at  any 

HOOKE'S  JOINT.  instant  is  different  from  that  of  C1, 

but  the  mean  angular  velocities  are 

equal  to  one  another,  because  at  one  instant  C2  goes  faster  than  Ct, 
and  at  another  slower.  This  joint  will  not  M  ork  when  the  two 
shafts  are  inclined  at  90°,  or  any  smaller  angle,  to  each  other. 

Double  Hooke's  Joint. — The  variable  velocity  ratio  obtained 
with  a  Hooke's  joint  may  be  obviated  by  the  use  of  two  joints 
instead  of  one.  The  forks  are  connected  by  an  intermediate  link, 
C2,  which  must  be  carried  on  corresponding  arms  of  the  two 
crosses,  as  shown  in  the  next  figure.  If  the  intermediate  shaft 
be  equally  inclined  to  the  other 
two  shafts,  the  irregularities  caused 
in  the  motion  by  its  transmission 
through  the  first  coupling  are 
exactly  neutralised  by  the  equal 
and  opposite  ones  caused  by  the 
second  joint.  The  first  and  third 


DOUBLE  HOOKE'S  JOINT. 


330 


LECTURE   XXIV. 


shafts,  therefore,  revolve  with  the  same  velocity  at  every  instai  t. 
The  double  joint  works  equally  well  whether  the  two  extrer  e 
axes  are  inclined  as  shown  in  the  figure,  or  are  parallel  to  ea  h 
other  but  not  in  line. 

Both  the  single  and  double  Hooke's  joint  are,  as  a  rule,  used 
only  for  light  work,  such  as  for  astronomical  instruments. 

Sun  and  Planet  Wheels. — This  device  was  invented  by  Watt 
to  convert  the  oscillatory  motion  of  the  beam  in  his  engines  into 
the  circular  motion  of  the  flywheel.  As  will  be  seen  from  the 


SUN  AND  PLANET  WHEELS. 

first  figure,  it  consists  of  a  wheel  D,  rigidly  fixed  to  the  connect- 
ing rod  D  B,  and  kept  in  gear  with  another  wheel  C,  by  the 
link  DEC.  The  wheel  C,  is  keyed  to  the  flywheel  shaft.  As 
the  beam  oscillates  up  and  down,  the  connecting-rod  pulls  D  up 
one  side  of  C,  and  pushes  it  down  the  other.  It  thereby  causes 
C  to  rotate,  and  with  it  the  shaft  and  flywheel.* 

Cams. — Cams  are  usually  of  the  form  of  discs  or  cylinders. 
They  rotate  about  an  axis,  and  give  a  reciprocating  motion  to 

*  See  Vol.  I.,  Lecture  XIX.,  of  the  author's  text-book  on  "Applied 
Mechanics  "  for  a  description  of  epicyclic  trains  and  the  application  of  the 
formula  to  this  case.  Watt  first  applied  this  motion  to  his  "  Double  Acting 
Steam  Engine"  in  1784.  See  Lecture  XVIII.  of  the  author's  elementary 
manual  on  "Steam  and  the  Steam  Engine." 


HEART   WHEEL   OR   HEART-SHAPED   CAM. 


331 


Borne  point  in  a  rod  by  means  of  the  form  of  their  periphery  or 
surface,  or  by  grooves  in  their  surface. 

The  cam  generally  revolves  uniformly  round  its  axis,  whilst  the 
reciprocating  motion  may  be  of  any  nature,  depending  on  the 
shape  of  the  cam,  and  may  be  in  a  plane  inclined  at  any  angle  to 
the  axis  of  rotation.  In  the  following  examples,  uniformity  of 
rotation  is  assumed  in  the  case  of  the  cam,  and  the  motion  of  the 
reciprocating  piece  takes  place  in  a  plane  perpendicular  to  the 
axis. 

Heart  Wheel  or  Heart-shaped  Cam. — Suppose  that  it  is 
required  to  give  a  uniform 
reciprocating  motion  to  a 
bar  moving  vertically  be- 
tween guides,  and  in  a  line 
passing  through  C,  the  centre 
of  motion  of  the  cam  plate. 

Let  the  sliding  bar  be  at 
its  lowest  position,  as  shown, 
and  when  in  its  highest 
position  let  its  extremity  be 
at  the  point  6.  The  distance 
thus  moved  is  called  the 
travel  and  will  be  passed  over 
during  one-half  revolution 
of  the  cam.  The  required 
curved  outline  may  be  ob- 
tained in  the  following 
manner  : — "With  centre  C, 
describe  circles  passing 
through  the  extreme  posi- 
tions of  the  end  of  the  rod. 
Divide  the  travel  into,  say, 
six  equal  parts  at  the  points 

i,  2,  3,  (fee.  Divide  the  semi-circumference  into  the  &ame  number 
of  equal  parts  by  radial  lines  C  i',  C  2',  <fec.  Then  with  centre  C, 
draw  the  concentric  arcs  i,  i';  2,  2';  <fcc.,  intersecting  these  radii 
in  the  points  i',  2',  3',  &c.  The  dotted  line  drawn  through  these 
points  will  represent  the  required  curve. 

If  the  end  of  the  sliding  bar  rests  on  this  curve  it  is  clear,  that 
for  equal  angles  turned  through  by  the  cam,  the  bar  will  move 
outwards  through  equal  distances,  and  consequently,  will  have 
uniform  linear  motion  imparted  to  it.  The  return  motion  will 
evidently  be  obtained  by  the  similar  and  equal  curve  i",  2",  3", 
<fec.,  on  the  opposite  side  of  the  cam. 


,  6" 
HEART-SHAPED  CAM. 


332 


LECTURE   XXIV. 


A  cam  so  formed  would  impart  the  required  motion  to  a  point. 
If  the  end  of  the  sliding  bar  be  provided  with  a  roller  in  order  to 
diminish  the  friction,  then  the  shape  of  the  cam  must  be  altered 
so  that  the  centre  of  the  roller  shall  move  over  the  outline  of  the 
cam  as  traced  above.  To  accomplish  this,  we  must  draw  a  curve 
inside  the  original  one  by  describing  small  arcs  with  centres  on 
the  original  curve  as  at  i',  2',  3',  &c.,  with  a  radius  equal  to  that 
of  the  roller,  and  then  by  drawing  a  smooth  curve  touching  these 
arcs,  as  shown  by  the  heavy  line  in  the  figure. 


CAM  GIVING  AN  INTERVAL  OF  RUST. 


CAM  GIVING  A  QUICK  RETURN. 


Cam  for  Intermittent  Motion. — Sometimes  the  motion 
imparted  by  a  cam  is  intermittent.  For  instance,  a  common 
form  of  lever  punching  machine  is  fitted  with  a  cam  which  gives 
the  punch  an  upward  movement,  then  a  period  of  rest,  nnd 
finally  a  downward  movement  during  each  revolution.  As  an 
example  of  this,  let  us  set  out  a  cam  to  impart  vertical  motion 
to  a  bar,  so  that  the  latter  shall  be  raised  uniformly  during  the 
first  half  revolution,  remain  at  rest  during  the  next  one-sixth, 
and  descend  uniformly  during  the  remainder  of  the  revolution. 

As  before,  suppose  the  reciprocation  to  be  in  a  line  passing 
through  C,  the  centre  of  motion  of  the  cam  plate.  Then,  with 
centre  C,  draw  circles  passing  through  the  extreme  positions  of 
the  end  of  the  bar.  Divide  the  circumference  into  three  parts 
corresponding  to  the  periods  of  one-half,  one-sixth,  and  one-third 
revolution,  by  drawing  radial  lines  making  angles  of  180°,  60°, 
and  120°.  Since  the  motion  is  to  be  uniform,  divide  the  travel 


QUICK   RETURN   CAM. 


333 


into  a  convenient  number  of  equal  parts,  say  twelve ;  and  the 

circumference  into  the  same  number  of  equal  parts  by  radial  lines. 

Draw  the  concentric  arcs  2,  2";  4,  4";  &e.,  and  3,  3';  6,  6'; 

as  shown.     The  curves  through 

the    points    so   determined  will 

give  the  required  motions.     The 

interval  of  rest  will  evidently  be 

given    by   the    circular    portion 

from  12"  to  12'.     The  complete 

outline    is    represented    by   the 

heavy  line  in  the  diagram. 

Quick  Return  Cam.  —  The 
student  will  readily  understand 
from  the  right-hand  figure,  that 
if  two-thirds  of  a  revolution  be 
occupied  in  raising  the  motion 
bar  and  the  remainder  in  lower- 
ing the  same,  the  return  stroke 
will  be  performed  in  half  the 
time  of  forward  stroke.  The 
curves  of  this  cam  are  found  in 
the  same  way  as  in  the  previous 
examples. 

EXAMPLE.  —  A  vertical  bar, 
moving  in  guides,  is  driven  by 
a  circular  cam  plate  having  a 
centre  of  motion  in  the  centre 
line  of  the  bar.  The  distance 
from  the  centre  of  motion  to  the 
centre  of  the  plate  is  2  inches, 
and  the  bar  exerts  a  pr.  ssure  of 
10  Ibs.  when  rising,  but  falls  by  ccp 
its  own  weight.  Find  the  work 
done  in  100  revolutions  of  the 
olate. 


VB 

SR 


CP 
CM 


CIRCULAR  CAM  PLATE. 

INDEX  TO  PARTS. 

G  represents  Guides. 

Vertical  bar. 
Sliding  roller. 
Circular  cam  plate. 
Centre  of  plate. 
Centre  of  motion. 


ANSWER. — Since  the  distance  between  the  roller  S  R,  and  the 
centre  of  the  plate  C  P,  remains  constant  as  the  plate  revolves, 
it  is  evident  that  the  bar  will  move  as  if  it  were  actuated  by  a 
crank  of  length  equal  to  the  distance  between  C  M  and  C  P,  and 
a  connecting-rod  of  length  equal  to  the  radius  of  the  plate.  Hence, 
the  stroke  of  the  bar  will  be  4  inches,  or  J  foot — i.e.,  twice  the 
length  of  the  equivalent  crank.  Neglecting  friction,  the  work 
done  in  raising  the  bar  by  one  revolution  of  the  plate,  will  be :— 


334  LECTURE   XXIV. 

Pressure  x  distance  moved  —  10  x  J  ft.-lbs. 
.*.   Work  done  in  100  revolutions  =  100  x  ioxj  =  338»3  ft.-lbs. 

Pawl  and  Ratchet  Wheel. — A  toothed  wheel  which  is  acted 
upon  by  a  vibrating  piece,  termed  a  click  or  pawl,  is  called  a  ratchet 
•wheel.  Ratchet  wheels  are  made  in  many  different  forms,  and  an* 


PAWL  AND  KATCHET. 


used  for  a  variety  of  purposes.  For  instance,  clocks  and  watches 
are  usually  provided  with  ratchet  wheels  to  allow  the  spring  or 
weight  to  be  wound  up,  without  disturbing  the  rest  of  the  works, 
and  they  are  used  to  drive  the  feeding  arrangements  of  many 
machines.  When,  as  in  the  latter  case,  the  click  or  pawl  drives 
the  ratchet  wheel,  it  is  carried  on  a  vibrating  arm.  In  the  first 
figure,  A  B  is  the  vibrating  bar  which  drives  the  ratchet  wheel, 
by  means  of  the  click  B  C,  and  teeth  C  C,  when  moving  in  the 
direction  shown  by  the  arrow.  When  A  B  moves  back  to  A  B', 
the  click  slides  over  the  top  of  the  next  tooth  and  drops  behind  it. 
It  is  then  ready  to  drive  the  wheel  through  the  space  of  another 
tooth  when  A  B  again  moves  forward.  While  the  pawl  is  moving 
back  from  B  to  B',  the  wheel  is  prevented  from  moving  with  it 
by  another  pawl  or  detent,  b  C.  In  this  case,  the  vibrating  bar  is 
on  the  same  axis  as  the  ratchet  wheel ;  but  this  is  not  always 
430.  The  reactions  between  the  teeth  and  the  pawl  keep  ihern  in 


REVERSIBLE   PAWL. 


335 


contact  with  each  other.  The  resultant  pressure  of  the  teeth  on 
the  pawl  must  therefore  be  such,  that  its  moment  tends  to  turn  the 
pawl  towards  A,  the  centre  of  the  ratchet  wheel.  This  condition 
evidently  is  satisfied  if  C  D,  the  direction  of  the  resultant  pressure 
at  C,  passes  between  A  and  the  axis  B,  about  which  the  pawl 
turns.  Similarly,  the  moment  of  the  resultant  pressure  on  the 
detent  must  trend  to  turn  it  towards  A,  but  its  direction,  d  C  (not 
C  d),  must  lie  outside  A  6,  because  this  detent  ends  in  a  hook* 


REVELSIBLE  CLICK. 


Both  pawls  might  have  been  like  B  C,  which  acts  by  pushing,  or 
both  hooks,  which  act  by  pulling,  like  b  C.  The  pawls  are  pressed 
against  the  ratchet  by  their  own  weight,  or  by  springs,  according 
to  circumstances.  When  a  ratchet  wheel  is  used  only  to  prevent 
the  recoil  of  the  axis  on  which  it  is  fixed,  the  vibrating  arm  is,  of 
course,  not  required,  and  only  the  detent  is  used. 

Reversible  Pawl. — The  above  figure  shows  a  form  of  click  used 
in  the  feed  motion  of  shaping  and  other  machines.  The  ratchet 
wheel  is  here  an  ordinary  toothed  wheel,  and  the  click  B  C  is  so 
shaped  as  to  be  able  to  drive  it  either  way.  When  the  click  is  in 


336  LECTURE  XXIV. 

the  position  shown  in  full  lines,  it  drives  the  ratchet  wheel  in 
the  direction  of  the  arrow.  When  the  wheel  is  required  to  rotate 
the  other  way,  the  click  is  lifted  over  to  the  dotted  position  ;  and, 
if  it  be  desired  to  stop  the  feed  motion  without  stopping  the 
machine,  the  click  is  put  in  an  upright  position.  A  portion  of 
the  pin  at  B,  which  turns  with  the  click,  is  triangular  in  section, 
A  spring  presses  on  this  part  and  so  keeps  the  click  in  any  one 
of  its  three  positions.  The  ratchet  wheel  is  keyed  to  A,  the  axis 
of  the  screw  which  moves  the  slide  carrying  the  cutter,  and  the 
friction  between  this  screw  and  its  nut  is  sufficient,  without  any 
detent,  to  prevent  the  ratchet  from  moving  back.  The  vibrating 
arm  A  B,  which  carries  the  click  is  driven  by  a  small  eccentric 
or  crank.  The  pawl  may,  of  course,  be  made  to  move  the  ratchet 
more  than  one  tooth  at  a  time  by  adjusting  the  angle  through 
which  A  B  vibrates. 

Masked  Ratchet. — In  numbering  machines  it  is  often  necessary 
to  print  the  same  number  twice,  as  in  cheques  and  their  counter- 
foils. The  ratchet  which  shifts  the  type  wheels  must  therefore 
be  moved  at  every  alternate  back-stroke  of  the  printing  machine. 
This  may  be  accomplished  by  putting  a  second  ratchet,  running 
free  on  the  shaft,  alongside  the  driving  one  and  making  the  pawl 
broad  enough  to  move  both.  The  second  ratchet  has  the  same 
number  of  teeth  as  the  other,  but  its  teeth  are  made  alternately 
deep  and  shallow.  It  is  also  a  little  larger  than  the  driving 
ratchet,  so  that  the  pawl  passes  over  the  top  of  the  teeth  of  the 
latter,  without  moving  it,  when  in  a  shallow  tooth.  Next  stroke 
the  pawl  drops  into  a  deep  tooth.  This  allows  it  to  catch  the 
teeth  of  the  main  ratchet  and  so  shift  the  type  wheel.  This 
arrangement  is  called  a  masked  ratchet. 

Silent  Peed. — A  ratchet  wheel  is  always  more  or  less  noisy  in 
action,  and  the  wear  caused  by  the  sudden  drop  of  the  pawl  ig 
considerable.  To  avoid  this,  a  friction  catch  is  sometimes  sub* 
gtituted  for  the  pawl  and  a  grooved  wheel  for  the  toothed  one. 
The  pawl  and  ratchet  then  becomes  a  silent  feed*  The  action  of 
this  arrangement  will  be  easily  understood  by  a  reference  to  the 
figures.  E  C  is  an  eccentric  cam  tapered  at  its  edge  to  fit  the 
groove  in  the  grooved  wheel  G  W.  When  E  0  moves  in  the 
direction  of  the  arrow  the  friction  causes  it  to  turn  about  its  axis, 
and,  since  the  axis  is  not  concentric  with  the  circular  part  of  its  rim, 
it  gets  wedged  in  the  groove.  Hence,  for  the  rest  of  the  stroke,  the 
lever  carries  G  W  round  with  it.  At  the  beginning  of  the  return 
stroke,  E  C  turns  in  the  opposite  direction,  and  so  gets  released 
from  the  groove.  A  detent  E  D,  precisely  similar  to  E  C,  but 
carried  on  *  fixed  arm,  prevents  the  wheel  from  moving  back- 


VERTICAL  SAWING  MACHINE. 


337 


.VERTICAL  SAWING  MACHINE,  BY  JOHN  M'DOWAL  &  SONS  OP 
JOHK  STONE,  SHOWING  SILENT  FEED. 


338  LECTURE  XXIV. 

wards.  The  lever  L  L,  is  worked  by  an  eccentric,  and  the  length 
of  its  stroke  may  be  adjusted  by  altering  the  position  of  the  end  of 
the  eccentric  rod  E  R,  in  the  slot.  The  full-page  illustration  shows 
a  sawing  machiDe,  with  this  feed  motion  at  the  right-hand  side. 


CW 


EC 


INDEX  TO  PARTS. 

G  W  for  Grooved  wheel. 
EC,,    Eccentric  cam. 

L    „    Lever. 

ED    ,,   Eccentric  detent. 
E  R    „   Eccentric  rod. 


WORSSAM'S  SILENT  FEED. 

Watt's  Parallel  Motion.  —  Referring  to  the  illustration  of 
s 


WATT'S  APPROXIMATE  STRAIGHT- LINE 
MOTION. 


CONSTRUCTION  FOR  LENGTHS 
OP  LINKS. 


Watt's  double-acting  engine,  previously  mentioned  in  this  Lecture, 
the  student  will  notice  that  the  beam  and  piston-rod  are  connected 


PARALLEL  MOTION.  339 

by  a  set  of  links.  This  system  of  links  has  been  called  Watt's 
Parallel  Motion.  The  first  figure  will  serve  to  show  the  principle 
on  which  an  approximate  rectilinear  motion  is  obtained.  Part 
of  the  beam  of  the  engine  is  shown  in  three  different  positions, 
C  Tj,  C  Ta,  and  C  T,.  The  point,  T,  in  it  is  connected  by  the 
link,  T  t,  to  the  end  of  a  lever  or  radius  rod,  c  t,  pivoted  at  c. 
In  their  mid  positions,  C  Tr  c  tv  these  two  levers  are  usually 
parallel  to  each  other,  and  perpendicular  to  the  line  Pj  P,  Pr 
The  point,  T,  describes  an  arc  of  a  circle  round  C,  and  t  round  c. 
As  these  arcs  curve  in  opposite  directions,  we  should  expect 
some  intermediate  point  on  the  link  T  £,  to  curve  in  neither 
direction,  but  to  describe  an  approximate  straight  line.  This 

"P  t         O  T1 
point  P  may  be  found  by  making  -p-™  =  — — .     The  actual  path 

of  P  is  like  the  figure  8,  and  the  parts  which  cross  are  very  nearly 
exact  straight  lines  for  a  short  distance  on  either  side  of  the 
crossing. 

Prof.  Rankine  gives  the  following  construction  for  the  lengths 
of  the  links  in  his  Machinery  and  Millwork  . — Let  A  be  the  centre 
of  the  beam,  G  D  the  centre  line  of  the  piston-rod's  motion,  and  B 
the  mid  position  of  its  end.  Draw  A  D  perpendicular  to  G  D. 
Make  D  E  equal  to  one-fourth  of  the  stroke,  and  join  A  E.  Draw 
E  F  perpendicular  to  A  E,  and  meeting  A  D  in  F.  A  F  is  the 
length  of  the  beam.  If  G  be  the  point  where  the  radius  rod  cuts 
G  D,  draw  G  K  at  right  angles  to  G  D,  and  make  D  H  equal  to 
G  B.  Join  A  to  H,  and  F  to  B,  and  produce  A  H  and  F  B  to 
meet  G  K  in  K  and  L,  Then,  F  L  is  the  connecting  link,  K  L  is 
the  radius  rod,  and  B  is  the  point  on  the  link  F  L,  to  which  the 
piston-rod  must  be  attached. 

Parallel  Motion. — In  the  accompanying  figure  A  B  T  t  is  a 
parallelogram,  and  c  is  a  point  in  A  t  produced.  In  the  meantime 
we  will  leave  the  links  C  T  and  B  D  out  of  account  and  consider 
the  parallelogram  only.  Join  B  c  and  we  have  two  similar 
triangles,  B  A  c  and  P  t  c.  ' 

s4  =  T^.         Or,  P  *  =  B  A  ¥-  =  a  constant. 
13A      Ac  Ac 

That  is,  in  every  position  of  the  parallelogram,  the  point  P 
remains  in  one  fixed  position  in  the  link  T  t.     Moreover  the  ratio 
Be 
p-^  is  constant,  and,  therefore,  whatever  patk  P  traces  out,  B  will 

trace  out  a  similar  one  This  is  the  principle  of  the  pantograph, 
which  is  used  for  enlarging  or  reducing  drawings.  N<m,  we  have 


340 


LECTURE   XXIV. 


just  seen  how  we  may  make  P  move  in  an  approximate  straight 
line  by  the  link  C  T.  B  will,  therefore,  also  move  in  an  approxi- 
mate straight  line.  We  might  have  guided  B  instead  of  P  with 
a  radius  rod,  but  this  would  have  necessitated  longer  and  heavier 
links  and  would  have  occupied  more  space. 

In  applying  this  motion  to  his  engine,  Watt  made  A  t  c  the 
beam,  and  attached  the  piston-rod  to  B  and  the  air  pump-rod  to 


---V 


PARALLEL  MOTION. 


PARALLEL  MOTION  FOR  RICHARD'S 
INDICATOR. 


P.  The  lengths  A  c,  t  c  were,  therefore,  proportional  to  the  strokes 
of  the  piston  and  pump  bucket  respectively.  Sometimes  a  third 
link  was  added  so  as  to  get  a  second  parallelogram  and  a  second 
point  moving  parallel  to  P,  and  this  was  used  to  drive  the  feed- 
pump. 

The  right-hand  figure  shows  the  parallel  motion  of  Richard's 
steam  engine  indicator.*  The  student  will  at  once  see  that  it  is 
a  modification  of  Watt's  parallel  motion.  In  this  case  the  piston- 
rod  P  E,  is  guided  by  a  collar  so  as  to  move  vertically,  and  is 
attached  by  the  link  E  F  to  the  bar  C  D,  between  D  and  the 
centre  C.  The  motion  of  p,  to  which  the  pencil  is  attached  is, 
therefore,  a  magnified  copy  of  the  piston's  motion. 


*  See  Lecture  XVI.  of  the  author's  "Elementary  Manual  of  Steam  and 
the  Steam  Engine  "  for  a  description  of  this  indicator. 


LECTURE   XXIV.— QUESTIONS  341 


LECTUBE  XXIV.— QUESTIONS. 

1.  Describe  Hooke's  joint  for  connecting  two  axes  whose  directions  meet 
in  a  point. 

2.  Sketch  and  describe  the  double  Hooke's  joint,  and  explain  why  it  is 
used  in  certain  cases  in  preference  to  the  single  joint. 

3.  Explain  the  manner  in  which  Watt  used  the  so-called  Sun  and  Planet 
Wheels  as  a  substitute  for  a  crank  and  connecting-rod,  and  account  for  the 
result  which  he  obtained. 

4.  Sketch  a  cam  for  giving  a  bar  a  uniform  reciprocating  motion,  and 
explain  how  you  find  the  form  of  its  periphery. 

5.  Set  out  a  form  of  cam  which,   when  acting  on  a  bar  by  uniform 
rotation,  will  cause  the  backward  and  forward  motion  of  the  bar  to  have 
an  interval  of  rest  between  each. 

6.  Describe,  by  the  aid  of  the   necessary  sketches,  how  the  circular 
motion  of  the  driving  pulley  is  converted  into  the  recipiocating  motion  of 
the  punch  in  an  ordinary  machine  for  punching  holes  in  metal  plates. 
Calculate  the  approximate  maximum  pressure  in  pounds  at  the  end  of  a 
punch  in  cutting  a  hole  I  inch  in  diameter  through  a  steel  plate  |  inch 
thick,  the  resistance  of  the  plate  to  shearing  being  taken  as  50,000  Ibs.  per 
square  inch  of  section.  Ana.  98,175  Ibs. 

7.  Describe  the  nature  of  a  cam,  and  give  any  examples  you  know  of  for 
which  it  is  used. 

8.  What  is  a  cam  ?    Show,  with  a  sketch,  how-to  obtain  by  means  of  a 
cam  a  motion  in  a  direction  parallel  to  the  axis  of  rotation. 

9.  What  is  a  cam  ?    For  what  purposes  in  mechanism  are  cams  generally 
used  ?    Sketch  and  describe  the  construction  and  actual  form  of  a  cam  in 
use  in  any  machine  with  which  you  are  acquainted.     Sketch  a  cam  which 
would  give  a  slow  forward  and  quick  return  motion  to  a  reciprocating 
piece,  with  an  interval  of  rest  between  the  two  motions. 

10.  How  is  a  cam  in  the  form  of  a  heart  set  out,  so  that  when  the  cam 
rotates  uniformly  it  may  cause  a  sliding  piece  to  move  to  and  fro  with  a 
uniform  velocity  ? 

11.  Determine  a  form  of  cam  which,  by  rotating  uniformly,  will  com- 
municate a  reciprocating  movement  to  a  sliding  bar,  but  with  an  interval 
of  rest  at  the  beginning  and  end  of  each  stroke. 

12.  Sketch  a  pawl  and  ratchet  wheel  as  used  for  preventing  the  recoil  of 
the  gear. 

13.  Describe  a  ratchet  wheel,  and  the  manner  in  which  it  is  held  and 
driven.     Show  its  application  in  a  ratchet  brace  where  it  is  combined  with 
a  lever  and  screw.     Sketch  the  contrivance,  and  point  out  the  manner  in 
which  the  drill  is  advanced. 

14.  Sketch  a  ratchet  feed  motion,  such  as  is  suitable  for  a  planing 
machine,  and  explain  the  manner  in  which  the  amount  of  feed  is  regulated. 

15.  What  is  a  ratchet  wheel,  and  how  is  it  driven  T    In  what  way  can  it 
be  arranged  that  a  ratchet  wheel  shall  advance  half  the  space  of  a  tooth  at 
each  stroke  of  the  driver  ? 

1 6.  By  what  contrivance  may  a  ratchet  wheel  with  60  teeth  be  made  to 
-act  as  if  it  had  120  teeth  ? 


3  P  LECTURE  XXIV. — QUESTIONS. 

17.  Sketch  and  describe  some  form  of  pawl  which  will  drive  a  ratche 
wheel  during  both  the  forward  and  backward  strokes. 

1 8.  It  is  sometimes  useful  to  advance  a  ratchet  wheel  at  every  alternate 
forward  stroke  of  the  driver,  instead  of  at  every  stroke,  as  is  commonly 
the  case ;  describe  and  sketch  a  mechanical  contrivance  which  will  give 
such  a  movement. 

19.  Describe,  with  the  necessary   sketches,  gome  form  of  silent-feed 
arrangement  commonly  used  instead  of  a  ratchet  wheel,  for  advancing  the 
timber  in  sawing  machines.     Explain  the  principle  of  the  friction  grip  upon 
which  such  a  contrivance  depends. 

20.  A  pinion  with  20  teeth  works  in  a  straight  rack,  the  distance  from 
centre  to  centre  of  the  teeth  in  the  pinion  being  |  inch.     The  pinion  is 
driven  by  a  ratchet  wheel  with  40  teeth  fixed  on  the  pinion  shaft  ;  find 
the  advance  of  the  rack  in  inches  for  each  tooth  moved  through  by  the 
ratchet  wheel.  Ans.  '25  inch. 

21.  Sketch  and  describe  a  vertical  sawmill,  showing  how  the  silent  feed 
is  applied. 

22.  Explain  the  principle  of  Watt's  approximate  straight-line  motion, 
commonly  called  a  "parallel  motion  "     By  what  combination  of  linkwork 
I*  an  exact  straight-lime  motion  obtained  ? 

23.  Sketch  and   describe  the  action   of  a  ratchet  wheel.     For   what 
purposes  are  such  wheels  used  in  machine  tools  ?     Show  how  a  ratchet 
wheel  of  30  teeth  can  be  used  to  give  a  feed  through  ^th  of  a  revolution 
at  each  stroke  of  the  arm  of  the  pawl.     (S.  E.  B.  1900.) 

24.  Design  a  cam  to  lift  vertically  A  sliding  piece  at  a  nniform  speed 
through  2  ins.,  the  return  motion  being  also  at  a  uniform  rate,  but  at  half 
the  speed  ;    having  given  that  the  line  of  stroke,  produced,  of  the  slider 
passes  through  the  axis  of  the  cam  shaft,  that  the  nearest  approach  of  the 
centre  of  the  roller  to  the  cam  centre  is  2  ins.,  and  that  the  diameter  of 
the  roller  is  £  in.  (C.  &  G.,  1904,  0.,  Sec.  A.) 

25.  Describe,  with  the  aid  of  a  sketch,  Watt's  parallel  motion,  and  show 
liow  to  find  the  position  of  the  point  in  the  coupler  which  most  nearly 
describes  a  straight  line.     Sketch    the   path  which  the   tracing   point 
describes  for  all  possible  positions.  (C.  &  Gr.,  1905,  0.,  Sec.  A.) 


(     343     ) 


LECTURE  XXV. 

CONTENTS.— Reversing  Motions— Planing;  Machine— Reversing  by  Friction 
Cones  and  Bevel  Wheels — Whitworth's  Reversing  Gear — Quick  Return 
Reversing  Motion — Whitworth's  Quick  Return  Motion — Whitworth'fc 
Slotting  Machine  —  Common  Quick  Return  —  Horizontal  Shaping 
Machine— Quick  Return  with  Elliptic  Wheels— Vertical  Slotting 
Machine — Questions. 

Reversing  Motions.— Planing  Machine.  —  In  Lecture  XI. 
we  illustrated  and  described  a  belt  reversing  motion  for  uniform 
forward  and  return  speeds  and  also  for  a  quick  return  as  applied 


HORIZONTAL  PLANING  MACHINE  WITH  QUICK  RETURN  BELT 
GEARING  BY  MESSRS.  J.  ARCHDALE  &  Co.,  BIRMINGHAM. 

to  planing  machines.     The  above  figure  will  serve  to  show  the 
manner  in  which  the  quick  return  and  slower  forward  motions 


344 


LECTURE  XXV. 


are  applied  to  a  modern  planing  machine.  Here,  the  smaller 
fixed  and  loose  pulleys  are  situated  to  the  extreme  left,  whilst  the 
larger  ones  are  placed  alongside  of  them.  The  pulleys  are 
connected  by  underneath  toothed  gearing  with  the  rack  of  the 
moving  table  upon  which  the  material  to  be  planed  is  bolted. 
The  under  side  of  this  table  has  two  straight,  truly  planed  and 
scraped  V-shaped  surfaces,  which  accurately  fit  corresponding 
V  grooves  on  the  upper  surface  of  the  strong  heavy  bed  plate. 
Two  vertical  standards,  with  planed  and  scraped  surfaces  on  their 
front  faces,  serve  to  guide  the  cross-piece  which  carries  two  tool- 
holders.  An  up  and  down  motion  is  given  to  this  cross-piece  by 
means  of  the  uppermost  handle,  spindle,  two  pairs  of  bevel 
pinions  and  vertical  screws  actuating  nuts  connected  to  the  back 
of  it.  A  horizontal  motion  is  given  to  the  tool-holders  by  the 
lower  right-hand  handle  and  horizontal  screws.  The  tool-holders 
themselves  are  adjustable  up  and  down  by  handles  and  screws  as 
shown.  They  may  also  be  so  set  as  to  cut  at  any  angle,  or  a 
horizontal  tool-holder  can  be  fixed  upon  one  or  other  of  the 
upright  standards  to  plane  vertical  surfaces.  All  the  above- 
mentioned  motions  may  be  actuated  from  either  side  of  the 
machine. 

We  shall  now  illustrate  and  describe  several  other  forms  which 
are  frequently  used  in  connection  with  machinery  of  different 
kinds. 

Reversing  by  Friction  Cones  and  Bevel  Wheels. — In  the 
first  of  the  two  following  figures,  the 
two  cones  B  and  C  are  fixed  to  the 
shaft  DD,  which  can  be  moved  up 
or  down  so  that  the  cones  B  or  C 

.           .,,.,. ,.  —  .....  maybe  alternately  brought  into  con- 

jiM  I  A  tact  with  the  other  cone  A,  which  is 

I  iiJL fixed  to  its  shaft.     Suppose  A  to  be 

10 llllj  I      g7   the  driver ;  then,  when  it  is  in  con- 
| IF3^    tact  with  B,  the  shaft  D  D  will  be 
.1  III  turned  in  one  direction,  and  when  in 

contact  with  C  it  will  be  rotated  in. 
the  opposite  way.  The  spindle  or 
shaft  D  D  may,  however,  be  driven 
by  a  belt  or  toothed  gearing,  and  con- 

FEICTION  CONE  REVERSING    sequently  when  A  is  in  contact  with 
GEAR.  B,  it  will  be  turned  in  one  direction 

and  when  in  contact  with  C  it  will 

be  rotated  in  the  opposite  way.  This  device  is  frequently  used 
in  connection  with  governors  for  engines,  in  order  to  lengthen  or 


WHITWORTH  S   REVERSING   GEAR. 


345 


shorten  the  rod  to  the  throttle  valve  or  cut-off  gear  and  thus 
enable  the  governor  balls  to  regain  their  normal  position,  without 
altering  the  quantity  of  steam  being  admitted. 

If  we  require  to  transmit  more  force  than  the  friction  between 
the  cones  will  effect  without 
slipping,  then  we  must  substi- 
tute toothed  bevel  wheels  for 
plain  cones.  In  such  a  case,  if 
the  speed  and  stress  were  con- 
siderable the  sudden  engagement 
of  the  wheels  on  one  side  or  the 
other  would  be  apt  to  damage 
the  teeth;  hence,  it  is  usual 
with  steam  cranes,  winches, 
windlasses,  screwing  gear,  &c., 
to  have  B  and  G  free  to  rotate 
upon  their  shaft  D  D,  and 
al  A-ays  in  gear  with  A,  as  shown 
by  the  second  figure.  The  re- 
versal in  this  case  is  effected  by 
means  of  the  clutch  E,  which 
can  be  slid  along  a  feather  on 
the  shaft  by  the  lever,  so  as  to 
engage  B  or  C  and  thus  make 
it  turn  with  the  shaft.  It  will 
be  easily  seen  that  the  direction  in  which  A  will  rotate  depends 
upon  whether  B  or  C  is  locked  to  the  shaft  D  D. 

Whitworth's  Reversing  Gear. — Another  modification  of  this 
gear  is  that  made  by  Sir  Joseph  Whitworth  &  Ob.  for  planing 
machines.  In  this  case,  the  reversal  is  effected  by  shifting  the 
driving  belt  by  the  fork  B  F,  from  the  forward  pulley  F  P  to  the 
backward  one  B  P.  The  latter  is  cast  in  one  piece  along  with  or 
keyed  to  the  boss  of  the  bevel  wheel  B  "Wj,  which  runs  loose  on 
the  shaft  S ;  whilst,  the  former  and  also  B  W2  are  rigidly  con- 
nected to  this  shaft.  A  loose  pulley  L  P  is  placed  between  the 
forward  and  backward  ones  in  order  to  facilitate  the  shifting  of 
the  belt  from  the  one  to  the  other  and  to  carry  the  belt  when  the 
machine  is  not  at  work.  The  table  T  upon  which  is  placed  the 
casting  or  other  material  to  be  operated  upon,  has  a  strong  nut 
fixed  upon  its  under  side,  and  is  moved  along  the  bed  or  slide  T  S 
by  the  driving  screw  DS.  This  screw  is  keyed  to  the  bevel 
wheel  B  W3  and  is  consequently  driven  in  a  forward  or  backward 
direction  according  as  the  belt  is  on  FP  or  BP.  In  order  to 
save  the  time  that  would  otherwise  be  wasted  if  the  cutting  tool 


BEVEL  WHEEL  AND  CLUTCH 
REVERSING  GEAR. 


346 


LECTUKE   XXV. 


was  so  fixed  as  to  cut  in  one  direction  only,  Sir  Joseph  Whitwoi  th 
designed  a  cylindrical  revolving  tool-holder,  which  automatically 
turns  the  tool  half  round  at  the  end  of  each  stroke  of  the  table 
The  reversal  of  the  motion  of  the  table  is  automatically  effected 
by  its  pushing  the  reversing  stops  R  St  (and  R  S.,  not  shown  in  the 
figure)  as  it  nears  the  end  of  a  stroke.  These  stops  are  fixed  iii 


r.s 


WHITWOETH  EEVERSING  GEAE. 

such  positions  upon  a  rod  connected  to  the  reversing  levers  R  L, 
as  to  shift  the  belt  fork  at  the  proper  time  and  thus  give  the 
required  length  of  stroke. 

Quick  Return  Reversing  Motion. — Another  form  of  re- 
versing motion,  used  for  planing  machines  which  only  cut  one 
way,  is  made  up  of  a  train  of  spur  wheels  and  a  rack  R.  As  in 
the  previous  case,  there  are  three  belt  pulleys,  each  of  the  outer 
ones  being  connected  to  a  separate  pinion.  One  of  these  pinions, 
D  P2,  drives  the  rack  on  the  planing  table  through  a  spur  wheel 
F2  and  pinion  P  fixed  on  an  intermediate  shaft ;  the  other,  D  Pp 
which  is  connected  with  F  P,  transmits  its  motion  through  another 
pair  of  wheels  Fx  and  Dr  This  will  cause  the  rack  and  table  to 
move  in  the  opposite  direction,  and  as  these  wheels  are  made  of 
unequal  sizes,  the  motion  is  also  slower  than  when  driving 
through  D  P2.  This  slower  motion  is  used  for  the  cutting  stroke 


WHITWORTH'S  QUICK  RETURN  MOTION. 


347 


and  the  quicker  one  for  the  return  stroke.  The  reversal  is 
effected  by  shifting  the  belt  from  FP  to  BP,  or  rice  versa,  as  in 
the  Whitworth  gear. 


B.R 


QUICK  RETURN  REVERSING  GEAR  FOR  PLANING  MACHINES. 


Whitworth's  Quick  Return  Motion. — In  a  shaping  or 
slotting  machine  the  table  carrying  the  work  is  fixed  and  the  tool 
moves  over  it,  cutting  in  one  direction  only.  In  such  a  case,  the 
tool  usually  obtains  a  reciprocating  motion  from  a  compound 
crauk,  so  arranged  as  to  give  a  quicker  return  stroke  in  order  to 


Return    Strok 


QUICK  RETURN  MOTION. 


save  time.  Looking  at  the  above  diagrammatic  figure,  A  and  B 
are  two  fixed  points  and  C  is  a  point  connected  to  the  tool-holder, 
so  guided  as  to  move  along  L  M  at  right  angles  to  A  B.  A  crank 


348  LECTURE   XXV. 

D  B  E,  centred  at  B,  has  its  outer  end  joined  to  C,  by  the 
connecting-rod  EC.  A  second  crank  A  D  rotates  round  A  and 
drives  the  first  by  having  a  pin  at  D  which  moves  in  a  slot  B  D. 
Now,  when  D  is  at  the  position  G,  C  will  be  at  the  extreme  left 
of  its  stroke ;  and  when  D  is  at  F,  C  will  be  at  the  other  end  of 
its  stroke.  Hence,  if  A  D  rotates  uniformly  in  the  direction  of 
the  arrows ;  0  will  make  its  cutting  stroke  from  left  to  r.ight 
while  D  is  moving  round  G  K  F  ;  but  during  its  return  stroke  D 
only  requires  to  move  round  F  H  G.  The  return  stroke  will 
therefore  occupy  less  time  than  the  cutting  stroke  in  the  same 
ratio  as  the  arc  F  H  G  is  less  than  the  arc  G  K  F. 

Sir   Joseph    Whitworth    applied    this    principle    to   shaping 
machines  in  the  manner  shown  by  the  next  figure,  which  has  the 


WHITWORTH'S  QUICK  RETURN  MOTION  FOIL  SHAPING  MACHINES. 

same  lettering  for  corresponding  parts  as  the  previous  one. 
Here,  the  crank  A  D  is  obtained  by  putting  a  pin  D  on  a  toothed 
wheel  which  rotates  freely  on  a,  fixed  shaft  S,  whose  centre  is  at  A ; 
and  the  crank  D  B  E  is  supported  by  a  pin  in  a  hole  bored  at  B 
in  the  end  of  this  shaft.  In  the  back  of  this  crank-piece  there  is 
a  slot  B  D  in  which  D  can  slide,  and  in  the  front  another  slot  B  E 
in  which  E  can  be  clamped  in  any  position  so  as  to  adjust  the 
length  of  B  E  and  thus  give  the  required  travel  to  the  tool.  The 
large  wheel  is  driven  uniformly  by  a  pinion  P  connected  with  the 
belt  pulley.* 

*  Students  who  desire  further  information  on  machine  tools  such  as 
drilling  and  milling  machines  or  on  measuring  appliances  should  refer  to 
Professor  Shelley's  "Workshop Appliances," Professor  Goodeve's  "Elements 
of  Mec'  anism,"  and  Lineham's  "  Text-booV  of  Mechanical  Engineering:,"  &c. 


WHITWORTH'S  SLOTTING  MACHINE. 


349 


Whitworth's  Slotting  Machine.*  —  The  following  side 
elevation  of  Sir  Joseph  Whit  worth  <fe  Co.'s  slotting  machine 
will  serve  to  show  how  their  quick  return  motion  is  practically 
applied.  The  driving  stepped  cone  D  C  receives  its  motion 
from  a  corresponding  overhead  cone  fixed  to  the  workshop 


WHITWORTH'S  SLOTTING  MACHINE. 

shafting.  Its  motion  is  in  turn  communicated  to  the  com- 
pound crank  E  B  D  (just  described  and  illustrated  by  the  two 
preceding  figures)  through  the  toothed  drivers  and  followers 
Dp  F!  ;  D2,  F2 ;  D3,  F3.  The  quick  up  and  slow  down  motion  of 
the  vertical  slide  bar  V  S,  with  its  tool-holder  T  H,  is  obtained 
from  the  compound  crank  through  the  connecting-rod  C  E.  The 
table  T,  upon  which  the  metal  to  be  slotted  is  bolted,  may  be 
shifted  by  hand  levers  or  automatically  moved  to  and  fro,  cross- 

*  The  above  figure  is  reduced  from  a  lithographed  drawing  which 
appears  in  Mr.  I.ineham's  book  on  "  Mechanical  Engineering,"  to  which 
students  may  refer  for  further  views  and  details. 


350 


LECTURE   XXV. 


wise  and  turned  at  pleasure,  by  the  feed  gearing  F  G,  actuated  by 
a  ratchet  at  B.  This  ratchet  has  a  reversible  click  of  the  kind 
illustrated  in  the  previous  lecture  and  it  is  driven  by  a  red  from 
the  feed  cam  F  0.  The  whole  of  the 
moving  parts  are  supported  by  a  strong 
heavy  bed  plate,  cast  in  one  piece  with 
the  upright  framing  so  as  to  prevent 
vibration  in  the  material  being  oper- 
at«ed  upon  or  chattering  of  the  cutting 
tool. 

Common  Quick  Return. — Another 
form  of  quick  return  very  often  used 
for  shaping  and  slotting  machines  is 
based  on  the  mechanism  of  the  oscillat- 
ing steam-engine.  A  crank  A  C  rotates 
uniformly  and  imparts  motion  to  a 
slotted  arm  B  F.  This  arm  will  have 
its  extreme  positions  at  B  H  and  B  K. 
It  will  therefore  make  its  forward  and 
back  swings  while  C  is  moving  round 
H  D  K  and  K  E  H  respectively,  and 
hence  it  has  the  quick  return  motion 
COMMON  QUICK  RETURN  desired  for  the  tool.  The  tool-holder 
FOE  SLOTTING  MACHINE,  is  connected  by  a  rod  to  such  a  point 

in  B  F  as  will  give  the  required  travel. 

Horizontal  Shaping   Machine. — The  following  illustration 
shows  a  shaping  machine  with  this  gear  for  driving  the  cutting 


SHAPING  MACHINE  WITH  QUICK  RETURN  BY  MESSES. 
SELIG  SONNENTHAL  &  Co.,  LONDON. 


QUICK   RETURN   WITH   ELLIPTIC   WHEELS. 


351 


tool.  The  work  is  fixed  in  a  vice  mounted  on  a  table  at  the  front 
of  the  machine,  and  this  table-  can  be  moved  up  and  down  by  a 
rack  or  screw,  and  traversed  along  the  bed  in  either  direction, 
backwards  and  forwards  by  ai  screw.  The  tool-holder  moves 
inwards  and  outwards  over  this  vice  and  is  provided  with  a 
screw  for  moving  the  tool  vertically-  and  a  worm  for  adjusting 
its  inclination.  The  gear  for  driving  the  tool  is  actuated  by  a 
sliding  pinion  on  a  shaft  which  lies  along  the  whole  length  of  the 
back  of  the  machin  and  is  driven  by  a  stepped  cone  pulley.  The 
feed  of  the  tool-holder  towards  the  left  or  right,  is  effected  by  a 
nut  and  screw  underneath.  This  screw  is  worked  by  a  ratchet 
seen  to  the  right,  in  a  similar  way  to  that  described  for  the 
previous  machine. 

Quick  Return  with  Elliptic  Wheels.— Vertical  Slotting 
Machine. — We  illustrate  another  slotting  machine  which  has  a 
different  method 

of     obtaining     a  ^^      °      "^\  D 

quick  return.  In 
this  machine,  the 
moving  bar  is  ac- 
tuated by  a  simple 
crank,  but  this 
crank  is  driven 
at  different  speeds 
for  the  cutting  and 
return  strokes  by 
the  following  de- 
vice. One  portion 
of  the  circumfer- 
ence of  the  wheel 
on  the  crank-piece 
is  part  of  an  ellipse 

,         B  D  A.  ELLIPTIC  WHEELS  FOB  QUICK  RETUEN. 

circular.  The  wheel  for  driving  this  has  an  elliptical  part  EOF, 
and  a  complete  circle  E  H  F  G.  Suppose  B  and  F  to  be  in  contact. 
Then,  as  the  lower  wheel  moves  round,  the  two  ellipses  keep  in 
contact  until  A  and  E  come  together.  The  two  shafts  thus 
make  about  half  a  revolution  in  the  same  time.  The  circular 
wheels  now  come  into  gear,  and  the  lower  wheel  must  make  one 
and  a  half  turns  while  the  upper  rotates  through  its  second  half. 
This  will  bring  B  and  F  aarain  into  contact  and  the  whole  process 
repeats  itself.  The  quicker  motion  with  the  elliptical  wheels  is 
used  for  the  upward  stroke  and  the  slower  motion  from  the 


352 


LECTURE   XXV. 


circular  wheels  for  the  downward  or  cutting  stroke.  By  looking 
at  the  side  elevation,  and  at  the  view  of  the  complete  machine, 
it  will  be  seen  that  the  circular  parts  of  the  wheels  D  JB  and  H  Gr 


VERTICAL  SLOTTING  MACHINE  BY  MESSES.  T.  SHANKS  &  Co., 
JOHNSTONE. 


are  placed  to  the  left  of  the  elliptical  parts,  B  C  and  C  F ;  and 
how,  at  each  junction  B  and  F  of  the  two  portions,  one  tooth 
stretches  right  across  .both  of  them  so  as  to  give  a  steady  connected 
motion.  Also,  how  the  smooth  part  A  K  B  of  the  upper  elliptic 
wheel  is  cut  away  to  clear  the  point  of  the  other  ellipse  during 


QUICK   RETURN   WITH   ELLIPTIC   WHEELSv  353 

thoso  revolutions  in  which  the  circles  are  in  gear.  From  what 
has  already  been  said  about  the  other  machines  the  student  will 
easily  understand  the  working  of  this  one.  He  should,  however, 
notice  that  the  moving  bar  has  a  counterbalance  to  relieve  the 
gearing  from  the  weight  of  the  sliding  bar,  also  to  reduce  the 
driving  force  required  for  the  upstroke  and  to  prevent  any  sudden 
drop  of  the  tool  on  the  work  which  may  be  placed  upon  the  table. 


354  LECTURE   XXV. — QUESTIONS. 

1.  Illustrate  and  explain  the  form  of  reversing  gear  you  would  employ 
for  a  steam  winch  in  which  the  engine  shaft  always  runs  one  way. 

2.  Show  a  method  of  applying  a  self-acting  motion  for  reversing  the 
motion  of  the  table  of  a  planing  machine  when  a  screw  is  employed  for 
driving  it. 

3.  Describe,  with  a  sketch  and  index  of  parts,  some  form  of  quick 
return  gearing   suitable  for  a  planing  machine,   the    movement    being 
obtained  by  a  combination  of  pulleys  and  spur  wheels. 

4.  Sketch  and  describe  an  arrangement  for  driving  the  table  of  a  planing 
machine  by  means  of  a  screw,  so  that  the  table  may  travel  50  per  cent, 
faster  in  the  return  than  in  the  forward  or  cutting  stroke.     Why  is  a 
square  threaded  screw  employed  in  such  a  machine? 

5.  Sketch  and  describe  an  arrangement  of  mechanism  for  reversing  the 
table  in  a  screw  driven  planing  machine.     In  what  way  can  a  quick  return 
of  the  table  be  obtained  in  such  a  machine  ? 

6.  Sketch  and  describe  a  good  form  of  slow  forward  and  quick  return 
for  a  shaping  machine. 

7.  Describe,  with  sketches,  a  planing,  a  slotting,  or  a  shaping  machine, 
showing  clearly  how  the  cutting  and  feeding  motions  are  effected. 

8.  Sketch  and  describe  the  mechanism   for  feed  motions: — (i)  In  a 
machine  where  there  is  a  reciprocating  movement,   as    in    a    planing 
machine  ;  (2)  where  there  is  a  continuous  movement,  as  in  a  machine  for 
boring  cylinders. 

9.  Sketch  and  describe  a  vertical  slotting  machine  with  quick  return 
elliptical  gear.    Give  a  separate  diagram  and  explanation  of  the  elliptical 
gear. 

10.  Sketch  and  describe  the  arrangement  of  mechanism  by  which  the 
tool  of  a  planing  machine  is  traversed  across  the  slide  of  the  machine  at 
each  stroke  of  the  table.    (B.  of  E.,  1900.) 

11.  Describe,  with  sketches,  the  mechanism  for  giving  an  automatic 
feed  to  the  cutting  tool  of  a  lathe  or  shaping  machine,  and  how  it  is  put 
in  or  oat  of  action,  and  the  amount  of  feed  varied.     (B.  of  E.,  1902.) 

12.  Describe,  with  sketches,  two  quick  return  motions  for  driving  the 
table  of  a  planing-machine — one  in  which  the  quick  return  is  obtained  by 
belting,  and  the  other  in  which  it  is  obtained  by  ordinary  gearing. 

(C.  &G.,  1903,  O.,  Sec.  A.) 

13.  The  mechanism  of  the  ordinary  direct-acting  engine  is  used  as  a 
quick  return  motion  for  a  small  shaping  machine  by  simply  placing  the 
crank  shaft  centre  below  the  line  of  stroke,  instead  of  in  the  line  of  stroke 
produced.     The  crank  radius  is  if  ins.,  the  connecting  rod  is  7  ins.  long, 
and  the  centre  of  the  crank  shaft  is  3^  ins.  below  the  line  of  stroke.     Find, 
graphically,  the  stroke  and  the  time  ratio  of  the  return  and  cutting  strokes 
—  the  crank  shaft  being  supposed  to  rotate  uniformly. 

Ans.  Length  of  stroke  =  4^  inches  ;  Time  ratio  =  i  to  i  -2. 

(0.  &  G.,  1903,  0.,  Sec.  A.) 

14.  Describe,  with  the  aid  of  sketches,  the  mechanism  known  as  the 
crank  and  slotted  lever  used  to  give  a  quick  return  motion  to  the  tool  of  a 
shaping  machine  ;   and  if  the  crank  radius  is  half  the  distance  between 
the  fixed  centres,  find  the  ratio  of  the  times  occupied  in  performing  the 
cutting  and  return  strokes.     The  crank  shaft  may  be  supposed  to  rotate 
uniformly,  and  the  obliquity  of  the  connecting  rod  to  the  line  of  stroke  of 
the  tool-box  neglected.    Ans.  (0.  &  G.,  1904,  0.,  Sec.  A.) 


(     355     ) 
LECTURE  XXVI. 

CONTENTS. — Measuring  Tools  and  Gauges — Limit  Ganges — Micrometer 
Screw  Gauge — Sir  Joseph  Whitworth's  Early  Realisations  of  Mechanical 
Accuracy — Improved  Equivalents  Micrometer  Gauge — A  New  Set  of 
English  Gauges — Whitworth  Millionth  Measuring  Machine — Improved 
Standard  Workshop  Measuring  Machine — Construction  and  Uses  of 
the  Tangentometer — Questions. 

Measuring  Tools  and  Gauges.—  A  few  years  ago?  a  pair  of 
calipers  and  a  foot-rule  graduated  to  ^-inch,  formed  an  outfit  for  the 
British  working  mechanic.  Anything  more  accurate  than  this  was  ex- 
pressed by  the  somewhat  elastic  term  full  or  bare !  As  far  as  each 
individual  piece  of  fitting  was  concerned,  the  work  turned  out  was  pro- 
bably of  a  good  sound  nature,  and  the  same  workman  was  able,  through 
practice,  to  make  a  number  of  parts  to  within  a  fair  degree  of  accuracy, 
when  we  consider  the  tools  which  were  at  his  disposal.  In  this  method 
of  working,  however,  discrepancies  due  to  individuality  occur,  and  modern 
practice  demands  the  suppression  of  these  as  far  as  possible.  Again, 
good  manual  labour  is  not  so  cheap  nowadays  as  it  used  to  be.  The  result 
is,  that  machines,  which  lessen  the  cost  of  production  and  turn  out  work 
of  a  well-finished,  accurate,  and  interchangeable  nature,  have  greatly 
superseded  the  old  rough-and-ready  methods  when  dimensions  were 
indicated  by  "sooking  fits,"  "  hair-breadths,"  and  suchlike  terms. 

The  introduction  of  the  micrometer  no  doubt  altered  things  to  some 
extent ;  but  this  instrument,  being  of  a  delicate  nature,  requires  careful 
handling,  and  that  means  lost  time  and  consequently  lost  money.  The 
micrometer  is  rather  a  tool-room  instrument  for  the  purpose  of  checking 
and  comparing  the  various  gauges  and  tools  used  in  the  shops. 

Limit  Gauges. — It  is  now  found  that  the  quickest  and  most  accurate 
method  of  turning  out  interchangeable  work  is  to  make  use  of  some 
system  of  limit  gauges.  In  reality,  a  limit  gauge  consists  of  two  gauges, 
one  of  which  is  larger  and  the  other  smaller  by  a  minute  fraction  than 
the  size  intended.  Thus,  if  we  employ  an  internal  limit  gauge  of  the  form 


not  go  In 


INTERNAL  LIMIT  GAUGE. 

shown  in  tne  figure,  for  measuring  a  Ig-ineh  hole,  and,  if  we  fin«1  that  the 
right-hand  end  goes  into  the  hole,  but  that  the  left-hand  end  does  not, 
then  we  know  for  certain  that  the  diameter  of  the  hole  cannot  possibly 
differ  from  ig-inch  by  more  than  -0012  inch  too  large,  or  '0004  inch  to* 
small. 


356 


LECTURE    XXVI. 


The  difference  limit  varies,  of  course,  with  the  class  of  work  which  it  is 
intended  to  produce.  It  is  quite  evident,  therefore,  that  for  a  machine 
which  runs  in  a  dust-laden  atmosphere,  a  much  greater  difference  limit 
will  be  required  in  the  bearings,  than  for  another  similar  machine  which  is 
designed  to  work  in  a  comparatively  clean  atmosphere. 

But,  no  matter  how  tight  or  how  slack  the  working  fits  may  be,  it  will 
be  found,  that  work  constructed  on  some  system  of  limit  gauges  will  have 
a  much  greater  all-round  efficiency  than  that  made  in  the  old  haphazard 
trial  and  error  fashion  with  an  ordinary  rule  and  calipers. 


EXTERNAL  LIMIT  GAUGE. 

The  above  or  second  figure  is  an  illustration  of  an  External  Limit 
Gauge  for  a  2|"  or  2-25  inch  shaft.  One  end  is  marked  to  "go on"  and  is 
only  o'OOoS"  or  8/10,000  of  an  inch  less  than  the  2*25  in.  shaft ;  whilst 
the  other  or  smaller  caliper  end  is  marked  to  "not  go  on,"  for  it  is  0*0014  in. 
or  i4/ro,ooO  of  an  inch  less  than  the  2^25  in.  shaft.  Consequently,  the 
maximum  difference  between  the  "go  on"  fit  and  the  "not  go  on"  fit  is 
(-2 -25  in.  -  0.0008 in. )  -  (2*25  in.  -  0-0014 in-)  or  simply  (0*0008  in  - O'OOi4in. ) 
r=o'OOo6in.,  that  is  6/10,000  in.,  or  say  1/1666  in.  which  is  less  than 
i/ioooth  of  an  inch.  If  the  wider  end  of  the  gauge  just  goes  on,  the  shaft 
will  be  a  "  tiyhljit,"  but  if  the  narrower  end  just  goes  on,  the  shaft  will  be 
a  "  running  f<,t  "  in  a  bush  bored  exactly  to  2-25  in.  diameter. 


STANDARD  INTERNAL  AND  EXTERNAL 
CYLINDRICAL  GAUGES. 


MICROMETER    SCREW    GAUGE. 


357 


The  third  figure  shows  a  Standard  Internal  and  External  Cylindrical 
Gauge.  This  type  of  gauge  is  chiefly  used  as  a  standard  o'f  reference. 
It  is  hardened,  ground,  lapped  by  hand,  and  is  accurate  to  within  -oooi 
inch. 

The  Micrometer  Screw  Gauge.— This  instrument  is  very  useful 
for  measuring  diameters  or  thicknesses  to  within  'ooi  inch.  On  turning 
the  milled  cap  shown  at  the  right-haud  side  of  the  illustration,  the 

operator  turns  the  screw  of  40 
threads  to  the  inch,  which  also 

i  |  Q  fr[  may  be  seen  from  the  figure.     It 

\  j    IJjgi  ~  is   clearly   evident,   that   if  one 

complete  turn  be  given  to  the 
milled  cap,  the  screw  will  advance 
or  recede  fa  inch.  Therefore,  if 
the  edge  of  the  sleeve  (which 
forms  part  of  the  cap)  be  divided 
into  25  equal  parts  along  its 
circumference,  and  the  cap  be 
rotated  by  a  part  of  a  revolution 
corresponding  to  one  of  these 
divisions,  the  screw  will  have 
advanced  by  -fe  of  fa  inch,  i.e., 

175*07  or  >o01  mcn 

It  will  be  seen  from  the  two  figures   that   micrometer  screw  gauges 


MICROMETER  SCREW  GAUGE. 

FOR  MEASURING  ALL  SIZES  LESS  THAN 

0-3  INCH  BY  THOUSANDTHS 

OF  AN  INCH. 


STARRETT  MICROMETER  GAUGE. 


are  made  of  various  ranges  and  styles.  If  a  mere  portable  gauge  is 
required  to  suit  different  sizes,  then  one  of  the  best  forms  is  that  made  by 
the  Starrett  Co.,  U.S.A.  In  which  the  position  of  the  movable  end  is 
determined  by  inserting  a  hardened  steel  tapered -pin  into  hardened 
steel  bushed  holes.  For  fixed  measuring  machines  of  great  accuracy  the 
Whitworth  Millionth  Measuring  Machine  is  still  considered  the  standard 
in  this  country. 


358  LECTURE   XXVI. 

Sir  Joseph  Whit  worth's  Early  Realisations  of  Me- 
chanical Accuracy. — "Whitworth's  Standard  Measuring  Machine 'r 
will  be  illustrated  and  described  near  the  end  of  this  Lecture.  But,  it  may 
be  mentioned  here,  that  the  whole  subject  of  accurate,  scientific,  mechanical 
measurement  and  its  standardisation  had  remained  in  great  confusion  and 
uncertainty  until  Whitworth  first  carefully  considered  and  then  made, 
about  the  year  1840,  mechanically  perfect  flat-surface  plates.  Second, 
he  made  standard  screw  threads,  screw  taps  and  dies,  as  well  as  parent 
leading  screws  for  lathes,  &c.  Lastly,  a  standard  or  parent  Measuring 
Machine,  which  was  to  be  an  instrument  of  such  extreme  precision  that  it 
could  detect  the  difference  of  one  one-millionth  of  an  inch  in  the  end 
measurement  of  short  standard  bars. 

These  three  early  steps  in  his  career  were  realised  in  succession.  In 
fact,  he  could  never  have  made  the  standard  measuring  machine  if  he  had 
not  previously  made  and  drilled  his  men  into  producing  the  two  former 
sets  of  tools. 

All  the  present-day  accurate  machine-shop  surface-plates,  screw  threads, 
standard  bars,  gauges  and  fixed  measuring  machines,  may  be  said  to  be 
the  outcome  of  Whitworth's  skill,  perseverance  and  forethought,  in 
systematising  the  production  of  standard  tools.  He  also  advertised  the 
results  and  sold  accurate  copies  of  his  correct  parent  tools  not  only 
amongst  British  engineers  but  throughout  the  civilised  world. 

Sir  Joseph  Whitworth,  before  commencing  the  afore-mentioned  difficult 
tasks,  was  satisfied  that  the  most  practical  means  of  workshop  measure- 
ment was  to  be  founded  on  the  truth  of  surface  and  the  sense  of  touch.  He 
maintained  to  his  dying  day,  that  the  most  delicate  sense  of  the  mechanic 
and  mechanician  was  that  of  touch.  In  confirmation  thereof,  he  showed 
that  when  a  piece  of  metal  had  parallel  end  faces  and  was  so  held  between 
the  two  fair-in  line  and  parallel  measuring  planes  of  his  machine,  that  the 
piece  being  tested  just  gravitated  slowly  downwards,  due  to  its  own  weight 
between  these  two  faces.  Then,  any  good  mechanic  could  at  once  detect 
the  difference  in  bringing  these  two  plane  parts  or  distance  faces  nearer 
together  or  further  apart  by  the  minute  difference  of  one  one-millionth  of 
an  inch  !  Of  course,  such  extreme  accuracy  is  not  required  in  ordinary 
tool-making  and  engine -building  works,  but  it  is  required  in  some  works. 

Improved  Equivalents  Micrometer  Gauge.— This  new  instru- 
ment has  certain  advantages  over  the  ordinary  screw  Micrometers.  By 
means  of  the  screw  on  the  hub  and  the  two  divided  discs,  readings  can  be 
taken  up  to  '300  inch  with  the  various  equivalents.  Or, measurements  may 
be  made  in  the  same  way  as  with  the  older  style  of  gauge  for  all  measure- 
ments less  than  one  inch  by  TTTOTT  °f  an  inch  between  the  jaws. 

On  one  side  of  the  disc  appear  decimals  of  an  inch,  decimals  of  mtlli-- 
metre  standard  wire  gauge  and  Stubs'  round  hole  wire  gauge.  On  the 
other  side,  fractions  of  an  inch  by  -fa  inch,  and  screwing  sizes  for  B.A. 
threads. 

Equivalents  to  other  gauges,  such  as  the  "British  Standard  Wire 
Gauge,"  "Stubs'  round  hole  wire  gauge,"  and  fractions  of  an  inch  on  the 
metric  standard,  can  be  read  off  without  the  necessity  of  reference  tables  ; 
whilst  at  the  same  time  the  jaws  are  set  to  give  an  exact  size  in  decimals 
of  an  inch.  It  will  thus  be  seen,  that  by  turning  the  disc  until  any 
decimal  or  other  number  appears,  several  equivalents  can  be  read 
simultaneously,  which  will  be  especially  convenient  for  telegraph  and 
electrical  engineers. 

The  hub  is  fitted  with  a  small  milled  thumb  piece,  projecting  beyond 
the  ordinary  hub,  by  means  of  which  the  speed  of  turning  the  screw 


IMPROVED    EQUIVALENTS    MICROMETER    GA-'GE. 


359- 


Tnay  be  increased.  The  anvil  end  of  the  Micrometer  is  flush  with  the 
screw  spindle,  thus  allowing  of  the  close  calipering  of  projections  or 
ledges. 

NOTE. — I  am  indebted  for  the  figure  and  description  of  this  improved 
gauge  to  the  patentees  and  makers,  Messrs.  Grimshaw  and  Baxter,  of 
29  Goswell  Road,  London. 


3'60  LECTUEE    XXVI. 

A  New  Set  of  English  Gauges  (Windless  Patents). — Descrip- 
tion of  the  Following  Six  Figures. — Fig.  I  shows  the  lower  half  of  a  pocket- 
case^"  x  5"  x  i £")  containing  a  complete  set  of  these  Caliper  Gauges. 
It  will  be  observed  that  both  the  external  and  the  internal  measuring  holders 
are  divided  in  the  centre,  to  allow  of  the  insertion  of  one  or  more  of  the 
steel  blocks  (illustrated  on  the  right  hand  of  the  case)  between  either  of 
these  two  holders. 

Fig.  II  indicates  the  arranging  of  the  External  Gauge  to  measure  £  inch. 
This  can  be  effected  in  a  very  short  time  by  means  of  the  "  quick-grip 
lock-nut  device." 

Fig.  Ill  gives  a  photographic  proof  of  the  perfectly  flat  surfaces  and 
the  very  superior  finish  of  the  faces  of  the  seven  cast-steel  blocks,  by  their 
clinging  together  after  having  been  wrung  together. 

Professor  Tyndall  was  the  first  scientist  to  prove  that*  perfectly  flat 
surfaces  adhere  together,  due  to  the  mere  molecular  attraction  between 
the  great  number  of  bearing  points  when  brought  into  close  contact.  He 
entirely  disposed  of  the  previously  held  theory  that  the  adherence  (of, 
say,  two  good  surface  plates)  was  due  to  the  exclusion  of  the  air  between 
them,  and,  therefore,  to  atmospheric  pressure. 

Fig.  IV  shows  the  internal  measuring  holder  arranged  to  measure  3! 
inches.  It  is  usually  finished  with  flat  ends,  but  it  can  be  supplied  with 
spherical  ends  having  a  radius  slightly  smaller  than  the  smallest  cylinder 
of  2\  inches  which  the  gauge  will  enter. 

Fig.  V  exemplifies  how  the  f ",  £",  ft*  y  and  &•  blocks,  when  wrung 
together,  are  used  to  check  the  i*  block  by  the  external  gauge.  This  process 
may  be  reversed  by  placing  the  i"  piece  between  the  jaws  and  the  five 
smaller  pieces  in  the  holder.  This  method  of  self -checking  is  one  of  the 
best  points  of  these  gauges. 

Fig.  VI  is  a  view  of  the  new  1910  pattern  of  a  "  Combined  Limit  and 
Double-ended  Caliper  Gauge."  When  the  jaws  marked  (+,  +)  and 
{ — »  — )  are  opposite  each  other,  as  in  the  photo,  the  gauge  is  set  for 
limit  measurements,  the  (+,  -f)  gap  being  slightly  larger  and  the 
{ — >  — )  gap  slightly  smaller  than  the  exact  measurement  aimed  at. 
But,  when  slackened,  turned  round  and  then  readjusted  (so  that  the 
(+>&  —  )  markings  come  opposite  each  other),  both  ends  become  exact 
caliper  or  snap  gauges.  They  now  indicate  the  precise  size  due  to 
the  number  and  breadths  of  the  round  steel  standard-sized  blocks, 
which  have  been  inserted  in  the  centre,  between  the  two  lock-nuts. 

It  may  be  mentioned,  that  all  the  parts  of  these  several  gauges  are  made 
of  the  best  British  cast-steel,  properly  hardened  and  finished. 

Range  of  Measurements  by  the  External  and  In- 
ternal Holders. — In  the  up-to-date  standard  sets  there  are  but 
seven  steel  blocks,  viz.,  i",  £",  f",  £",  ^  %'  and  TV*.  This  gives  the  ex- 
ternal measuring  holder  a  range  from  Ty  to  z\\  by  TV  of  an  inch,  or  forty 
different  sizes  ;  and  the  internal  measuring  holder  a  range  from  2%"  to  4", 
also  by  rVths  of  an  inch,  or  twenty-four  different  sizes.  In  addition  to  this, 
forty  end  sizes  from  rV"  to  2  J"  are  obtainable  by  wringing  the  blocks  together 
in  combinations.  Thus,  we  get  a  range  of  over  100  separate  standard  gauge 
sizes  with  one  small  pocket  instrument. 

Mr.  H.  M.  Budgett,  of  the  Crown  Works,  Chelmsford,  informs  the 
author  that  he  is  now  (1910)  adding  ^"  and  ^"  measuring  pieces  to 
the  above-mentioned  seven  blocks.  This  will  greatly  increase  the  range 
of  the  sizes  obtainable  with  this  instrument.  In  fact,  the  outside  gauge 
will  then  give  160  sizes  and  the  internal  one  96  sizes,  whilst  the  pieces 


FOR  DESCRIPTION  AND  DETAILS  SEE  PROFESSOR  JAMIESON'S  MANUAL 
OF  APPLIED  MECHANICS. 


FIG.  I. 

SHOWING  COMPLETE  SET  IN  CASE,  HALF  ACTUAL   SIZE. 
These  Gauges  are  made  by  the  Crown  Works.  Chelmsford,  Engla 


FIG.   II. 
SHOWING   EXTERNAL  GAUGE  BEING  ARRANGED  TO  MEASURE  --in. 


FIG.    Ill, 
SHOWING   HOW  THE  VARIOUS  PIECES   WILL  ADHERE  WHEN  WRUNG  TOGETHER 


FIG    IV. 

SHOWING  THE  INTERNAL  GAUGE  ARRANGED  TO  MEASURE  34  in. 


FIG.  V. 

SHOWING   HOW  THE  GAUGES  ARE  SELF-CHECKING, 


FOR  DESCRIPTION  AND  DETAILS  SEE  PROFESSOR  JAMIESON'S  MANUAL 
OF  APPLIED  MECHANICS. 


FIG.   VI. 
SHOWING  COMBINED  LIMIT  AND  DOUBLE    ENDED  CALIPER  GAUGE. 


WHITWORTH    MILLIONTH    MEASURING    MACHINE.         361 

themselves  will  give  163  sizes  ;  so  that  419  different  exact  sizes  could 
thereby  be  measured  ! 

Uses  of  the  Gauges  with  a  Surface-Plate.— In  every  up- 
to-date  tool-making  and  engineering  workshop  there  should  be  a  set  of 
standard  surface-plates.  Then,  the  previously  described  gauges  may  be 
used  in  many  ways  with  such  a  surface -plate.  For  example,  an  exact 
height  gauge  may  be  formed  by  wringing  one  arm  of  the  external  measuring 
holder  with  any  desired  number  of  blocks,  and  then  using  them  on  the 
surface-plate. 

The     National    Physical    Laboratory    Certificate.  —  We 

finish  the  description  of  these  British-made  gauges  in  reproducing  from  a 
photograph  the  Certificate  granted  in  August  1909  by  The  National  Physical 
Laboratory,  with  two  objects  in  view  : 

First,  to  show  the  extreme  accuracy  of  the  gauges,  where  none  of  the 
seven  blocks  had  an  error  of  one  one  hundred-thousandth  of  an  inch  ! 

Second,  to  bring  to  the  notice  of  Engineering  Students,  that  if  ever  they 
should  be  fortunate  enough  to  devise  anything  new  in  regard  to  ther- 
mometers, pyrometers,  barometers,  micrometers,  cyclometers,  galvano- 
meters, electrometers,  or  steam,  vacuum  and  mechanical  gauges,  &c., 
they  may  have  the  accuracy  of  their  invention  or  improvement  tested  by 
an  absolutely  impartial  and  reliable  judge  at  a  comparatively  small  cost. 

Certificate  of  Examination  of  Seven  End  Gauges. 
By  the  National  Physical  Laboratory,  Teddington. 

For :   H.  M.  Budgett,  Crown  Works,  Chelmsford. 
Form  :   Cylindrical,  i"  in  diameter  with  a  ±"  hole  through  the  centre. 
The  end  faces  are  perpendicular  to  the  axis  of  the  cylinder. 
These  gauges  have  been  compared  with  the  Laboratory  Standards,  and 
the  mean  lengths  at  62°  F.  have  been  found  to  be  : 

Gauge.        Length  at  62°  F.  Gauge.        Length  at  62°  F. 

t  0.062  50  inch.  £  0.25000  inch. 

0.12500     ,,  0.37500     „ 

T<V  0.18750     ,,  $  0.50000     „ 

Gauge  i  was  i.ooooo  inch  at  62°  F. 

Each  gauge  has  been  tested  at  several  points,  and  the  end  faces  in  each 
case  have  been  found  to  be  parallel  within  o".ooooi. 

J.  A.  HABKEB, 
August  25,  1909.  For  the  Director. 

The    Whitwprth   Millionth    Measuring    Machine.— From 

what  has  been  said  in  this  Lecture  about  Sir  Joseph  Whitworth's  early 
realisations  of  mechanical  accuracy,  the  student  will  be  anxious  to  learn 
how  he  accomplished  the  wonderful  task  of  constructing  a  machine  capable 
of  detecting  a  difference  of  only  one-millionth  of  an  inch  in  the  length 
of  two  bars.  Another  object  which  he  then  had  in  view  was  to  produce 
an  exact  fractional  part  of  the  "  British  Parliamentary  Standard  of  Length," 
viz.  the  legal  yard,  or  36  inches.  Consequently,  he  first  of  all  made 
a  machine  to  receive  between  its  two  measuring  noses  a  bar  of  only  i  inch 
long,  and  thereafter  he  constructed  the  machine  which  we  now  illustrate 
and  describe  to  measure  bars  of  3*  and  4"  in  length,  so  that  twelve  of  the 


362  LECTURE   XXVI. 

former  or  nine  of  the  latter  placed  end  to  end  would  make  up  the  British 
standard  yard. 

In  1851  he  was  awarded  the  Council  Medal  of  the  first  Great  Exhibition 
(which  had  been  proposed  and  successfully  carried  through  under  the 
patronage  of  the  late  Queen  Victoria  and  Prince  Consort)  for  a  precisely 
similar  machine  to  take  in  a  bar  of  40"  in  length. 

Referring  to  the  figure,  we  see  that  it  consists  of  a  strong,  rigid  cast- 
iron  framing  F,  which  stands  upon  three  legs,  two  at  the  right-hand  end 
and  one  leg  at  the  left-hand  end.  This  frame  F  not  only  forms  the  bed 
of  the  machine,  but  it  also  includes  the  bases  of  the  two  fixed  head-stocks, 
of  which  C1  and  C2  are  the  upper  or  removable  caps.  Precisely  in  the  centre- 
line of  the  two  head- stocks  the  casting  is  moulded  into  two  hollow  right- 
angled  or  V  grooves.  These  V  grooves  are  then  carefully  planed  and 
scraped  perfectly  fair  and  square  and  a  rectangular  steel  bar  O  is  so  very 
carefully  fitted  into  each  of  them  that  they  bear  and  can  be  moved  evenly 
throughout  the  length  of  each  end  head-stock  base,  along  the  centre  line. 
The  ends  of  these  two  steel  bars  of  o  or  square  section  are  then  faced  at 
right-angles  to  their  length,  and  a  truly  central  hole  is  bored  and  screwed, 
to  receive  a  long  steel  screw  at  their  outer  ends  and  shorter  sere  wed- in 
noses  Nj,  N2  as  shown  in  the  figure.  The  inner  faces  of  these  two  noses 
are  so  carefully  turned  and  scraped  that  they  are  "  dead  parallel "  to 
each  other.  It  is  between  these  noses  Nx  and  N2  that  the  short  standard 
bar  or  its  copy  is  to  be  placed  and  its  length  measured  in  the  following 
way. 

It  will  be  observed,  that  on  the  extreme  left  of  the  framing  a  graduated 
wheel  QWlt  has  been  fitted  to  the  outer  turned  end  of  the  screw  spindle 


GW 


WHITWORTH  MILLIONTH  MEASURING  MACHINE, 

whose  nose  is  Nj.  Since  the  pitch  of  the  screw  has  twenty  threads  to  the 
inch,  and  the  flat  periphery  surface  of  the  graduated  wheel  GW,  is  divided 
into  250  equal  parts,  with  a  fine  pointer  at  P,  it  will  be  readily  understood, 
that  if  this  wheel  GWa  is  so  turned  that  the  pointer  P  shows  a  movement 
through  one  division,  then  the  inner  face  of  the  nose  Nj  will  onlij  move 
one  five-thousandth  of  an  inch  forward  or  backward,  according  to  the 
direction  in  which  the  wheel  is  turned.  This  screw  and  wheel  are  there- 
fore only  suitable  for  comparatively  rough  to  and  fro  adjustments  of  the 
nose  Nlf  since  *Vx*iv=7<fo*th  of  an  inch. 


WHIT  WORTH    MILLIONTH    MEASURING  MACHIN7E         363 

Now  look  at  the  other  head-stock  where  the  screw  which  actuates  the 
nose  N2  has  also  a  pitch  of  twenty  threads  to  the  inch,  and  the  wheel  WW 
has  200  teeth.  This  wheel  gears  with  the  worm-screw  WS,  whose  spindle 
is  fixed  to  the  centre  of  the  large  graduated  wheel  GW2,  which  has  250 
equal  divisions  marked  upon  its  flat-faced  periphery  with  a  pointer  not 
shown  [in  [the  figure.  Consequently  (since  &  x  ?fa  X  ^  =  j^^, 
if  the  wheel  GW2  be  so  carefully  turned  that  its  pointer  shows  a  movement 
of  one  division,  the  inner  face  of  the  nose  N2  will  only  move  to  or  fro  by  the 
one -millionth  of  an  inch. 

Such  a  small  movement  might  bind  or  let  the  short  bars  i*,  2",  3*  or  4* 
in  length  drop  down  as  the  case  may  be,  and  thus  to  a  certain  extent  hamper 
the  "  finer  feelings  "  of  the  operator.  So  Whitworth  introduced  what  he 
called  a  "feeler  "  (which  consists  of  a  thin  parallel  slip  of  hardened  polished 
steel)  between  one  face  of  the  standard  bar  (or  the  end  of  the  bar  to  be 
measured  and  compared  with  the  standard  one)  and  the  flat  face  of  the 
nose  N!  or  N2.  When  the  adjustment  of  Nj  and  N2  is  considered  perfect, 
this  "  feeler  "  should  just  glide  down  gently  by  its  own  weight,  between 
one  of  these  nose  faces  and  the  nearest  end  of  the  bar  under  test. 

Such  an  accurate  machine  as  we  have  just  described  must  not  only  be 
brought  to  and  kept  at  a  fixed  standard  temperature,  say  62°  Fahr.  for 
British  measurements  and  70°  Fahr.  for  United  States  of  America  tests, 
but  care  must  be  taken  to  protect  it  from  dust,  moisture,  and  currents  of 
»ir. 

Improved  Standard  Workshop  Measuring  Machine.— 
For  engineering  workshops,  a  much  less  expensive  and  simpler  fixed  machine 
is  now  made.  It  is,  however,  sufficiently  accurate  to  detect  a  difference 
of  ^^  of  an  inch  in  the  length  between  one  bar  or  gauge  [and  another. 
It  consists  of  a  strong  frame  and  bed-  or  lathe-shears,  with  a  fixed  head- 
stock  at  one  end  and  another  one  which  is  movable.  The  latter  is 
moved  to  or  fro  by  means  of  a  screw  placed  longitudinally  between  the 
whole  length  of  the  shears.  This  screw  is  rotated  by  an  end  wheel,  and 
the  screw  gears  with  a  nut  fixed  into  the  tongue  of  the  movable  head- 
stock.  The  spindle  of  this  "  poppit "  or  movable  head  is  often  round  and 
parallel  throughout.  Consequently,  all  to-and-fro  adjustments  of  the 
right-hand  nose-face  N2  are  made  in  the  same  way  as  shown  by  the 
previous  figure  in  connection  with  the  fixed  head-stock, 
p  Now,  supposing  that  you  have  to  compare  the  length  of  a  bar  with  that 
of  a  standard  bar  (whose  length  had  been  previously  adjusted  between  the 
noses  N!  and  N2  and  the  pressure  thereon  noted),  you  place  the  new  bar  be- 
tween Nj  and  N2.  If  this  bar  is,  say,  longer  than  the  standard  bar,  the  free 
spindle  of  the  movable  head-stock  will  be  pressed  slightly  outwards.  Then, 
since  its  outer  end  now  presses  harder  against  the  elastic  centre  of  a  circular 
metal  case  containing  water  (with  a  vertical  graduated,  clear  glass  tube 
extending  therefrom),  the  water  will  rise  higher  in  the  tube  than  in  the 
former  case,  that  is  when  the  standard  bar  was  under  measurement. 

The  graduated  wheel  of  the  fixed  head-stock  is  now  turned  until  the 
height  or  "  head  "  of  water  in  the  glass  tube  is  brought  to  the  same  position 
as  in  the  first  case.  Then,  the  difference  between  the  two  readings  upon 
the  divided  wheel  at  once  indicates  the  difference  in  ten-thousandth  of  an 
inch  between  the  two  bars.  We  know,  that  the  pressure  upon  the  ends  of 
each  bar  was  the  same  because  the  hydrostatic  "  head  "  was  the  game.  Th 
replaces  the  personal  error  of  the  operator  with  his  feeler. 


3^4 


LECTURE    XXVI. 


Construction  of  The  Tan?3nt  >met9r.— This  instrument  con- 
§ists  of  a  horizontal  metre  rule,  divided  into  inches  and  parts  of  an  inch 
along  its  upper  scale,  and  into  centimetres  and  millimetres  along  its  lower 
scale.  At  the  centre  of  this  rule  there  is  attached  one  end  of  a  second 
rule,  half  a  metre  in  length  and  perforated  at  various  points.  A  small  slot 
is  cut  in  the  second  rule  to  enable  the  degrees  marked  on  the  semicircle 
to  be  read.  Attached  to  this  second  rule  by  means  of  a  pin,  are  (i)  a 
third  rule,  (2)  a  plumb-line.  The  third  rule  is  graduated  into  cm.  and  mm., 
and  has  a  fine  slot  cut  down  its  centre  to  enable  the  divisions  on  the  first 
or  horizontal  rule  to  be  read.  In  order  to  avoid  parallax  a  cursor  or  fine 
dividing  line  connects  the  vertical  with  the  horizontal  rules.  The  whole 
instrument  is  made  of  boxwood  and  fixed  on  a  firm  central  base. 


811k  thread 

Plumb-Line. 

Plumb 
Weight 


BERRIDGE'S  TANGENTOMETER. 
(As  made  by  Harris  &  Co,,  Ltd.,  Birmingham.) 


Uses  of  The  Tangentometer.— This  instrument  einblea  teachers 
to  explain  graphically  the  meanfng  of  the  several  trigonometrical  ratios  to 
technical  students.  It  also  enables  a  student  to  measure  for  himself 
various  angles  and  to  prove  his  answers  to  questions  by  noting  the  ratio 
of  the  actual  lengths  of  the  sides  of  a  right-angled  triangle.  He  can 
then  compare  his  results  with  the  Table  of  Trigonometrical  values  for 
"Functions  of  Angles  "  at  the  end  of  this  book. 

FOR  EXAMPLE,  taking  the  above  scale  figure  of  the  full-sized  instrument, 
we  see,  that  the  centre  line  of  the  second  rule  (which  is  50  cms.  long)  lies  at 
an  angle  of  48°  to  the  centre  line  of  the  first  or  horizontal  rule.  And,  the 
third  or  vertical  rule  hangs  at  right  angles  to  the  first  rule.  The  length 
of  the  plumb-line  of  the  third  rule  is  37  cms.  to  where  it  cuts  the  centre 
line  of  the  first  rule  at  right,  angles  at  33-5  cms.  from  its  centre  or  zero 
mark. 


USES    OF    THE   TAMJENTOMETER. 


365 


Hero,  rhe  first  or  horizontal  rule  is  called  the  base;  the  second  or  50  cm. 
rule  i-  th.j  hypothenuse  ;  and  the  third  or  vertical  rule  is  called  the  per- 
pend cid  r.  Then,  we  have  the  following  ratios  for  the  acuie  angle  of  48* 
in  the  lull-sized  instrument.* 


Sine  of  the 

Tangent 
Cotangent 
Secant 
Cosecant 


Perpendicular 
HyputbtiuUae 

_  5^  _ 
Hjputuenuse 

PerriPTidicnlar 


p  , 

Perpendicular       37  cms. 

Hvpofhennse        ^o  cms. 


Table  for    8* 


j 
Perpendicular       37  cms. 

i  Radian  =  Unit  angle  in  circular  measure  = =  57°'29. 

.-.  48° -i-  57° -29  =  0  8378  of  a  radian.     See  Table  for  the  radian  of  48*. 

*  When  resolving  a  force  into  its  two  component  forces,  the  student 
must  pay  particular  attention  to  the  ( +  )  or  {  —  )  sign  of  the  value  of  the 
angle.  See  Functions  of  Angles  in  Castle's  "Practical  Mathematics  for 
Beginners,"  chapter  xvi.,  or  other  book  on  Trigonometry. 

NOTE. — I  am  indebted  to  Ludw.  Loewe  and  Co.  Ltd.,  30  Farringdon 
Road,  London,  for  the  use  of  their  Electros  of  "  Limit  Gauges." 

I  have  to  thank  Mr.  H.  M.  Budgett,  of  the  Crown  Works,  Chelmsfordr 
for  six  excellent  views  of  his  company's  new  English  Gauges,  and  the 
Louis  Gassier  Co.,  Limited,  for  the  liberty  to  reproduce  the  Micrometer 
Gauges  and  the  W'hitworth  measuring  machine  which  appeared  in  Cassier's 
Magazine,  September  1901.  The  attention  of  Students  who  are  interested 
in  gauges  and  accurate  work  is  directed  to  "  The  Specification  of  the 
Engineering  Committee  on  Standards " ;  and  to  comments  upon  the 
same  in  The  Electrician  for  Aug.  1906. 


366  LECTURE    XXVI. QUESTIONS. 


LECTURE  XXVI. — QUESTIONS. 

1.  Describe  any  micrometer  screw  gauge  with  which  you  are  acquainted 
suitable  for  measuring  to  the  rgV*  of  an  inch.     Sketch  and  describe  care- 
fully the  method  of  graduation  and  the  position  of  the  gauge  when  set  to 
measure  -374  inch. 

2.  Sketch  and  describe  the  construction  and  use  of  external  and  internal 
workshop  gauges,  by  means  of  which  the  size  of  a  spindle  (say  2  inches 
diameter),  and  that  of  a  hole  into  which  it  fits,  may  be  ensured  within 
specified  limits  of  accuracy.     State  any  advantages  due  to  this  system  of 
working.  (B.  of  E.,  1902.) 

3.  Sketch  a  cylindrical  one  inch  external  gauge,  and  describe  generally 
the  measuring  machine  which  you  would  require  to  employ,  and  the 
manner  of  using  the  same,  in  order  to  construct  another  gauge  of  the  like 
kind,  but  measuring  i  -005  in  diameter.     How  is  the  gauge  worked  down 
to  the  right  size  and  finished  ? 

4.  Sketch  and  describe  the  construction  and  action  of  a  *'  Whitworth 
Millionth  Measuring  Machine."    For  which  purposes  have  you  seen  it 
used? 

5.  Sketch  and  describe  an  "  Equivalents  Micrometer  Gauge  "  and  state 
the  advantages  which  it  possesses  over  an  ordinary  gauge. 

6.  Sketch  and  describe  fully  an  "  External  Limit  Gauge  "  for  a  2}  inch 
shaft,  such  as  the  one  illustrated  by  the  second   figure  in  this  Lecture. 
Explain  concisely  and  clearly  how  you  would  use  such  a  gauge.     Show 
your  calculations  in  full  for  the  percentage  error  in  2^  inches  diameter,  if 
a  turner  just  makes  a  shaft  as  a  tight  fit  for  the  "go  on  "  end  of  the  gauge. 
Also  for  the  percentage  error  if  he  should  so  reduce  the  diameter  of  a 
shaft,  to  let  the  "not  go  on  "  end  of  the  gauge  fit  the  shaft. 

7.  Describe  by  aid  of  sketches  Sir  Joseph  Whitworth's  early  attempts 
towards  and  realisations  of  mechanical  accuracy  with  standard  surface- 
plates,  screws,  gauges,  and  measuring  machines. 

8.  Sketch  and  describe  the  new  set  of  English  Gauges  as  made  by  the 
Crown  Works,  Chelmsf ord,  according  to  Windley's  Patents. 

9.  Show  how  the  gauges  in  the  previous  question  are  self-checking. 
Also,  show  by  calculation,  the  exact  range  or  number  of  different  measure- 
ments which  can  be  made  with  seven  blocks  and  the  external  holders. 
If  two  extra  blocks,  -fa*  and  -fa",  were  added,  prove  how  many  more  measure- 
ments could  be  made. 

10.  Sketch  neatly  and  describe  concisely  Sir  Joseph  Whitworth's  Millionth 
Measuring  Machine.     State  the  pitch  of  screw,  number  of  teeth  on  worm- 
wheel,  and  number  of  divisions  on  the  graduated  wheel  to  measure  with 
such  accuracy. 

11.  Sketch,  index  and  describe,  any  fixed  good  workshop  measuring 
machine  and  state  how  you  would  use  it. 

12.  Find  the  sine,  cosine,  tangent,  cotangent,  secant  and  cosecant  of  an 
angle  in  a  right-angled  triangle  whose  base  is  40  cms,  and  perpendicular 
20  cms.     Also  express  27°  in  radian  measure. 


(     367     ) 


APPENDIX  A  (pages  368  to  370). 

(I)  General   Instructions  by  the  Board  of  Education  for  their  Examina- 
tions on  Applied  Mechanics,  Stage  1. 

(ii)  General  Instructions  by  the  City  and  Guilds  of  London  Institute  for 
their  Examination  on  Mechanical  Engineering,  Ordinary  Grade. 

(iii)  Rules  and  Syllabus  of  Examinations  by  the  Institution  of  Civil  Engi- 
neers for  Admission  of  Students. 


APPENDIX  B  (pages  371  to  401). 

Board  of  Education's  Exam.  Papers  in  Applied  Mechanics,  Stage  I. 
The  City  and  Guilds  of  London  Institute's  Ordinary  Exam.  Papers  in 
Mechanical  Engineering.  And  the  Institution  of  Civil  Engineers'  Exam. 
Papers  *  Elementary  Mechanics,  arranged  in  the  order  of  the  Lectures. 

APPENDIX  C  (pages  403  to  410). 

The  latest  Exam.  Papars  pertaining  to  Mechanics  and  set  by  the  govern- 
ing bodies  enumerated  under  Appendix  A. 

APPENDIX  D  (pages  411  to  413). 

(i)  Units  of  Measurement  and  their  Definitions ;  Practical  Electrical 
Units  and  their  Symbol  Letters. 

(ii)  Examination  Tables,  Useful  Constants,  Logarithm?,  Antilogarithra 
and  Functions  of  Angles. 


2  A 


(     368     ) 

Appendix  A. 

May  Examination  on  Subject  VII. 
APPLIED  MECHANICS.* 

BY  THE  BOARD  OF  EDUCATION,  SECONDARY  BRANCH, 
SOUTH  KENSINGTON,  LONDON 

Stage  1. 

GENERAL  INSTRUCTIONS. 

If  the  regulations  are  not  attended  to,  your  paper 
will  be  cancelled. 

Immediately  before  the  Examination  commences,  the  following 
REGULATIONS  ARE  TO  BE  READ  TO  THE  CANDIDATES. 

Before  commencing  your  work,  you  are  required  to  fill  up  the  numbered 
slip  which  is  attached  to  the  blank  examination  paper. 

You  may  not  have  with  you  any  books,  notes,  or  paper  other  than  that 
supplied  to  you  for  use  at  this  examination. 

You  are  not  allowed  to  write,  draw,  or  calculate  on  your  paper  of 
questions. 

You  must  not,  under  any  circumstances  whatever,  speak  to  or  com- 
municate with  another  candidate.  Those  superintending  the  examina- 
tion are  not  at  liberty  to  give  any  explanation  bearing  upon  the  paper. 

You  must  remain  seated  until  your  papers  have  been  collected,  and  then 
quietly  leave  the  examination  room.  No  candidate  will  be  allowed  to 
leave  before  the  expiration  of  one  hour  from  the  commencement  of  the 
examination,  and  none  can  be  re-admitted  after  having  once  left  the  room. 

All  papers,  not  previously  given  up,  will  be  collected  at  10  o'clock. 

If  any  of  you  break  any  of  these  regulations,  or  use  any  unfair  means, 
you  will  be  expelled,  and  your  paper  cancelled. 

Before  commencing  your  work,  you  must  carefully 
read  the  following  instructions. 

Put  the  number  of  the  question  before  your  answer. 

You  are  to  confine  your  answers  strictly  to  the  questions  proposed. 

Such  details  of  your  calculations  should  be  given  as  will  show  the 
methods  employed  in  obtaining  arithmetical  results. 

The  value  attached  to  each  question  is  shown  in  brackets  after  the 
question. 

A  table  of  logarithms  and  functions  of  angles  and  useful  constants  and 
fc  rriiulffi  is  supplied  to  each  candidate. 

The  examination  in  this  subject  lasts  for  three  hours, 

*  See  Appendix  C  for  the  latest  Exam.  Papers." 


APPENDIX   A.  369 


CITY  AND  GUILDS  OF  LONDON  INSTITUTE. 
DEPARTMENT  OF  TECHNOLOGY. 

Technological  Examinations. 
MECHANICAL  ENGINEERING.* 

OBDINARY  GRADE — PART  L 
(FIRST  YEAR'S  COURSE.) 

INSTRUCTIONS. 

No  Certificates  will  be  given  on  the  results  of  this  Examination  (First 
Year's  Course),  but  the  Candidates'  successes  will  be  notified  to  the  Centre 
where  they  were  examined. 

To  obtain  a  Certificate,  it  is  essential  that  Candidates  should  pass  both 
in  Part  I.  and  Part  II.  ;  the  Examination  in  Part  II.  witt  be  hdd  on 
Thursday,  May 3  Jot  7  p.m.  Candidates  may  take  both  Parts  I.  and  IL  in 
the  same  year. 

The  class  of  Certificate  and  the  order  of  Prize  will  be  determined  by  the 
results  of  the  Examination  in  Part  II.  only. 

The  maximum  number  of  marks  obtainable  is  affixed  to  each  question. 

The  number  of  the  question  must  be  placed  before  the  answer  in  the 
worked  paper. 

Three  how*  allowed  for  thit  paper. 

The  Candidate  is  at  liberty  to  use  divided  scales,  compasses,  set  squares, 
calculators,  slide  rules,  and  tables  of  logarithms. 

A  piece  of  squared  paper  to  be  given  to  each  Candidate,  if  required. 

The  Candidate  is  not  expected  to  answer  more  than  vine  question*, 
wSaich  must  be  selected  from  (too  Sections  only. 

*  See  Appendix  C  for  the  latest  Exam.  Papers. 

t  This  date  is  only  approximate,  and  subject  to  a  slight  alteration  eaoh 
year. 


APPENDIX    A. 


The  Institution  of  Civil  Engineers*  Rules  for 
Admission  of  Students. 

SYLLABUS  OF  THE  EXAMINATIONS.* 

1.  ENGLISH  (one  Paper,  time  allowed,  3  hours).     A  general  Paper  com- 
prising questions  in  Geography,  History  and  Literature. 

2.  MATHEMATICS  (two  Papers,  time  allowed,  3  hour*  for  each).    Papers 
comprise  questions  in  Arithmetic;    Algebra;   Geometry  (Euclid  I. -IV. )  '• 
and  Trigonometry. 

3.  Two  subjects,  to  be  selected  by  the  Candidate  from  the  following 
ten :  a  language  is  not  compulsory,  but  in  any  case  not  more  than  one 
language  may  be  taken  (time  allowed,  3  hours  for  each  Paper) : 

LATIN,  GREEK,  FRENCH,  GERMAN,  ITALIAN,  SPANISH. 

Elementary  Mechanics  of  solids  and  fluids,  t  ELEMENTARY 
PHYSICS,  including  heat,  light,  Electricity  and  Magnetism. 
ELEMENTARY  CHEMISTRY.  GEOMETRICAL  AND  FREEHAND  DRAWING. 

*  See  Appendix  C  for  the  latest  Exam.  Papers. 

f  My  Manuals  on  Applied  Mechanics,  Magnetism  and  Electricity,  are 
suitable  for  Young  Engineers  desiring  to  prepare  by  correspondence  or 
otherwise  for  these  subjects,  of  which  the  most  recent  examples  in  Mechanics 
are  given  in  Appendix  C,  whilst  those  on  Electricity  and  Magnetism  are 
printed  in  the  Appendix  of  the  Eighth  Edition  of  the  latter  work.  Candi- 
dates should  write  direct  at  once  to  the  Author  of  this  book  for  his  C.E. 
Prospectus,  which  gives  full  details  of  Tuition  for  these  Examinations. 

RULES  OF  THE  EXAMINATIONS. 

The  Examinations  are  held  in  London  in  February  and  October  annually, 
on  four  days  beginning  on  the  second  Tuesday  in  each  of  those  months. 
The  February  Studentship  Examination  may,  in  the  discretion  of  the 
Council,  be  held  also  in  Manchester,  Glasgow  and  Newcastle-on-Tyne. 

The  Council  will  consider  an  Application  from  a  person  who  is  duly 
recommended  for  Admission  as  a  Student  of  the  Institution,  to  present 
himself  for  the  Studentship  Examination. 

,  Applications  to  attend  the  Associate  Membership  Examination  will  be 
received  from  Students  of  the  Institution  who  are  not  less  than  21  years 
nor  more  than  26  years  of  age  on  the  last  day  for  entry. 

Arrangements  may  be  made  for  the  examination  of  Candidates  In  India 

1  or  in  the  Colonies,  after  submitting  duly  completed  proposals  for  Election. 
All  applications  for  Rules,  Forms  and  Admission,  &c.,  must  be  made 
through  the  Secretary,  the  Institution  of  Civil  Engineers,  Great  George 
Street,  Westminster,  London,  S.W. 


ORDINARY  QUESTIONS. 


371 


Appendix  B. 

See  Appendix  D  for  Practical  Electrical  Units 
LECTURE  II. — ORDINARY  QUESTIONS. 

1.  In  a  shale  mine  in  order  to  drain  one  of  the  pits  a  treble  ram  pump, 
driven  by  an  electric  motor,  is  employed.     The  rams  are  9^  inches  in 
diameter  by  1 2-inch  stroke,  they  each  make  3475  strokes  per  minute,  and 
the  height  to  which  the  water  is  lifted  is  393  feet. 

Fnd: 

(a)  How  many  gallons  of  water  this  pump  can  lift  per  minute. 
(6)  How  many  foot-pounds  of  useful  work  are  done  per  minute, 
(c)  The  useful  horse-power  when  the  pumps  are  running  steadily. 

(B.  of  E.,  1906.) 

2.  A  windmill  is  employed  to  drive  a  pump  which  has  to  lift  water  from 
a  well  and  deliver  it  into  an  overhead  tank.     It  was  found  that  when  the 
windmill  works  steadily  under  the  action  of  a  uniform  wind  for  a  period 
of  i  hour,  5000  gallons  of  water     are  raised  from  the  well  and  delivered 
into  the  tank — the  average  height  of  lif t  is  60  feet.     What  under  these 
conditions  is  the  useful  horse-power  of  the  windmill  ?         (B.  of  E.,  1907.) 

3.  An  electrical  hoist  is  employed  in  raising  coal  from  the  hold  of  a 
ship  and  delivering  it  into  railway  cars,  the  amount  of  lift  being  125  feet. 
If  the  coal  is  raised  at  the  rate  of  2400  Ibs.  per  minute,  what  is  the  useful 
horse-power  ? 

Convert  this  into  watts. 

If  the  current  is  supplied  at  a  voltage  of  250,  and  if  the  efficiency  of  the 
whole  arrangement  is  50  per  cent.,  how  many  amperes  of  current  must  be 
supplied  to  the  motor  working  the  hoist  ?  (B.  of  E.,  1907.) 

4.  Two  closely  coiled  spiral  springs  were  made  out  of  round  steel  wire, 
J-inch  diameter.     The  one  spring,  A,  had  a  mean  diameter  of  coil  of  4  inc^  es 

and  the  other,  B,  had  a  mean  diameter  of  coil  of  5  inches ;  both  springe 
had  12  complete  coils.  These  two  springs  were  tested  by  loads  extending 
them  axially,  and  the  results  of  the  tests  are  shown  in  the  table  below  : —  , , 


Axial  load    \ 
in  pounds      / 

2 

4 

6 

8 

10 

12 

H 

16 

18 

20 

Extension      "| 

of  the 
spring  A.      I 
Inches.      J 

0-26 

0-52 

079 

1*06 

1-32 

i'59 

1-86 

2'12 

2'39 

2-66 

Extension     ) 

of  the         I 
spring  B.      f 
Inches.        ) 

0-51 

I  '02 

i'53 

2-04 

2  '5  5 

3-06 

3'57 

4-09 

4'6o 

5-12 

Plot  the  results  on  squared  paper. 

Given  that  the  law  connecting  the  extension  of  these  springs 
with  their  mean  diameter  of  coil  is  of  the  form — 

Extension  of  B        /Mean  diameter  of  coil  of  B\n 
Extension  of  A  ~  \Mean  diameter  of  coil  of  A) 
what  is  the  probable  value  of  n  ?  (B.  of  E.,  1908.) 


APPENDIX  B. 


5.  Show  how  to  determine  the  work  done  by  a  variable  force[~moving"in 
its  own  direction.     A  cage  weighing  1200  Ibs,  is  raised  300  ft,  by  a  windlass 
having  a  wire  rope  weighing  i£  Ibs.  per  foot  run.     Show,  by  a  diagram  to 
scale,  the  work  done  at  any  stage,  and  mark  on  it  the  numerical  values  for 
a  lift  of  100,  200,  and  300  feet  respectively.         (C.  &  G.,  1909,  O.,  Sec.  A.) 

6.  An  engine  cylinder,  fitted  with  a  Joy  valve  gear,  has  a  stroke  of 
26  inches,  and  a  connecting-rod  74  inches  long.     In  the  accompanying 


LINE  DIAGEAMS  OF  PROBLEM  ON  JOY'S  VALVE  GEAR. 

sketch  the  link  AB  is  pivoted  to  the  connecting-rod  at  A,  and  to  the  free 
end  B  of  a  swinging  link  BC  centred  at  C.  The  motion  for  operating  the 
valve  is  taken  from  a  point  D  on  the  link  AB.  Draw  the  path  of  thefpoint 
D  for  a  complete  revolution  of  the  crank.  Take  a  scale  of  lin.  =  i  ft. 

(C.  &  G.,  1909,  0.,  Sec.  A.) 


LECTURE  IV.— ORDINARY  QUESTIONS. 

i.l  The  right-angled  bell  crank  lever,  centred  at  A,  shown[in  the[sketch 
is  attached  to  a  spring  by  one  of  its  arms,  and  to  another^lever,  centred^at 
B,  by  the  other  arm. 

If  the  spring  requires  a  direct  pull  of  20  Ibs.  hi  order  to  stretch  it  2[inches, 
find  what  force  P,  applied  as  shown,  will  stretch  the  spring  this  amount.C ! 

(B.  of  E.,  1906.) 


«  24"  ~~2-'-* 

22 

oj 

1 

J 

'--4'-*  A 

B 

? 

-> 
( 

nT* 

BELL  CRANK  LEVER  AND  SPIRAL  SPRING. 


ORDINARY  QUESTIONS. 


373 


2.  An  ordinary  bell-pull,  shown  in  the  sketch,  is  in  equilibrium.  Deter- 
mine in  any  way  you  please  the  magnitude  of  the  force  Q  and  the  magni- 
tude and  direction  of  the  resultant  thrust  upon  the  supporting  pin  A. 

(B.  of  E.,  1907.) 


P=10lbs. 


ORDINAEY  BELL-PULL  LEVEB. 

3.  The  figure  shows  the  mechanism  known  as  a  knuckle  joint.^A^force 
of  50  Ibs.  is  applied  at  the  point  A,  its  line  of  action  being  perpendicular 
to  the  line  BC.  Determine  graphically,  or  in  any  other  way,  the  vertica 
thrust  delivered  by  the  block  D.  Neglect  friction. 

How  will  this  thrust  vary  as  the  block  D  descends  ?      (B.  of  E.,  1908.) 


TOGGLE  OR  KNUCKLE  JOINT. 


3/4  APPENDIX  B. 

LECTURE  V.— ORDINARY  QUESTIONS. 

i.  Describe  how  you  would  determine  experimentally  the  coefficient 
of  sliding  friction  between  two  pieces  of  metal  of  any  convenient  size  when 
the  speed  of  rubbing  is  low.  (B.  of  E.,  1906.) 


LECTURE  VI.— ORDINARY  QUESTIONS. 

f  I.  Describe  any  one  form  of  lifting  tackle  with  which  you  are  acquainted 
and  explain  with  reference  to  it  the  terms  "  velocity  ratio,"  "  force  ratio," 
and  "  efficiency."  Explain  how  you  would  determine  their  numerical 
values  for  all  loads  up  to  the  full  capacity  of  the  tackle.  (n 

(C.  &  O.,  1908,  0.,  Seo.  A.) 


LECTURE  VII.— ORDINARY  QUESTIONS. 

I.  The  sheave  of  a  differential  pulley  block  consists  of  two  parts  which 
have  diameters  of  8  and  9  inches  respectively.  What  is  the  velocity  ratio 
of  the  mechanism  when  a  load  is  being  raised  ? 

If  the  mechanical  efficiency  of  the  pulley  is  32  per  cent.,  what  pull  must 
be  exerted  in  order  to  raise  a  load  of  2  tons  ?  (B.  of  E.,  S.  1, 1909.) 


LECTURE  VIII. — ORDINARY  QUESTIONS. 

1.  The  weight  of  a  span  of  telegraph  wire  is  127  Ibs.     At  one  end  the 
wire  makes  an  angle  of  5°  and  at  the  other  an  angle  of  7°  with  the  horizontal, 
what  are  the  pulling  forces  at  these  ends  ?  (B.  of  E.,  S.  i ,  1909. ) 

2.  A  simple  Warren  girder  is  as  sketched.     Loads  of  3  and  4  tons  are 
carried  at  the  two  joints  of  the  top  member.     Find,  analytically  or  other- 
wise, the  forces  in  the  different  members,          (C.  &  G.,  1906,  0.,  Sec.  B.) 

1 3  TONS  14  TONS 


SIMPLE  WARREN  GIRDER. 

3.  Explain  one  method  of  determining  the  stresses  in  the  members  of 
a  pin-jointed  frame,    j 


ORDINARY  QUESTIONS. 

\4  Tons 


375 


5  Tons 


^ «„  • _„__. 

LOADED  ROOF  TRUSS. 

Determine  the  stresses  in  all  the  members  of  the  roof  truss  loaded,  as 
shown  in  the  figure.  (C.  &.  G.,  1908,  O.,  Sec.  B.) 

4.  A  simple  crane  is  of  the  form  shown  in  the  diagram,  and  carries  5  tons 
at  C.  Determine  the  stress  in  AB,  AC,  and  BC,  and  find  the  magnitude 
of  the  balance  weight  W  so  that  there  shall  be  no  bending  moment  on  the 
post  BD.  (C.  &  G.,  1908,  O.,  Sec.  D.)  j 


5  Tons 


LINE  DIAGRAM  OF  A  SIMPLE  CRANE. 

5.  "A  crane  of 'the  form  shown  by  the  accompanying  sketch  carries  a 
load  of  3  tons.     The  reaction  at  the  upper  bearing  ia  horizontal.     Deter* 


Horizontal,  •* 

Reaction, 


LINE  DIAGRAM  OP  A  WALL  CRANE. 


376  APPENDIX  B. 

mine  the  resultant  pressure  on  the  footstep  bearing,  and  the  stresses  in  the 
members  of  the  crane,  assuming  that  all  the  connections  are  pin-joints. 

(C.  &  G.,  1909,  0.,  Sec.  D.) 


LECTUBE  X.—  ORDINARY  QUESTIONS. 

1.  A  machine  weighing  8  tons  is  dragged  slowly  along  a  horizontal  floor. 
If  the  coefficient   of   friction  between  the  base  of  the  machine  and  the 
floor  is  0*35,  find  in  pounds  the  magnitude  of   the  pull,  and  the  normal 
pressure  on  the  floor  when  (a)  the  line  of  pull  is  horizontal,  (6)  the  line  of 
pull  makes  an  upward  angle  of  30°  with  the  horizontal.     (B.  of  E.,  1907.) 

2.  The  length  of  a  journal  is  9  inches,  and  diameter  6  inches,  and  it  carries 
a  load  of  3  tons.     What  horse-power  is  absorbed  when  making  100  revo- 
lutions per  minute,  taking  the  coefficient  of  friction  as  "015,  and  how  many 
thermal  units  are  radiated  away  per  minute  when  the  temperature  of  the 
bearing  remains  constant  ?  (C.  &  G.,  1907,  0.,  Sec.  A.) 

3.  An  electric  locomotive  draws  a  train  of  700  tons  up  an  incline  of 
i  in  100  at  a  steady  speed  of  10  miles  per  hour.     If  the  frictional  resistances 
are  equal  to  15  Ibs.  per  ton,  what  is  the  total  pull  exerted  on  the  train  and 
what  is  the  horse-power  ?     Find  the  current  consumption  in  amperes,  if 
the  voltage  is  625,  and  if  60  per  cent,  of  the  electric  energy  supplied  to  the 
locomotive  is  spent  in  hauling  the  load.*]  (B.  of  E.,  S.  i,  1909.) 


LECTURE  XI.— ORDINARY  QUESTIONS. 

1.  Hemp  ropes  are  employed  to  transmit  power  from  the  engine  shaft  to 
the  driving  pulleys  on  the  different  floors  in  a  spinning  factory.     The 
maximum  tension  in  a  rope  is  twice  the  minimum  tension,  the  breaking 
strength  of  one  rope  is  5700  Ibs.,  and  it  is  desired  to  have  a  factor  of  safety 
of  30. 

Find  the  maximum  horse-power  which  can  be  safely  transmitted  by  one 
of  these  hemp  ropes  at  a  speed  of  70  feet  per  second.         (B.  of  E.,  1906.) 

2.  It  is  required  to  transfer  3  horse-power  to  a  pulley  16  inches  diameter 
by  belting.     The  revolutions  per  minute  are  100,  the  tension  in  the  tight 
side  of  the  belt  is  i£  times  that  in  the  slack  side,  the  thickness  of  the  belt 
is  -/TJ-  inches,  and  the  maximum  working  stress  allowable  is  320  Ibs.  per 
square  inch.     Find  the  least  width  of  belt.         (C.  &  G.,  1906,  0.,  Sec.  A.) 

3.  In  a  rope  brake  on  a  fly-wheel  8  feet  diameter,  the  ropes  being  i  inch 
diameter  the  load  is  500  Ibs.,  and  the  pull  on  the  spring  balance   varies 
from  10  to  20  Ibs.  during  a  test.     Find  the  brake  horse-power,  the  revolu- 
tions being  105  per  minute.  (C.  &  G.,  1907,  0.,  Sec.  A.) 

4.  Two  shafts,  which  are  not  parallel,  and  do  not  intersect,  are  to  be 
connected  by  a  belt  passing  over  suitably  placed  pulleys.     Explain  what 
are  the  necessary  conditions  to  be  observed  in  order  that  the  belt  shall 
remain  on  the  pulleys.     A  horizontal  shaft,  running  along  one  side  of  a 
machine  shop,  drives  another  horizontal  shaft  at  right  angles  to  the  first 
shaft  and  20  ft.  blow.      Sketch  a  suitable  arrangement  for  the  belt-drive 
if  both  shafts  are  to  revolv    at  the  same  speed.1 

(C.  &  G.,  1908,  0.,  Sec.  A.) 


ORDINARY  QUESTIONS. 
LECTURE  XII.— ORDINARY  QUESTIONS. 


377 


I.  An  engine  having  a  stroke  of  12  inches,  and  a  connecting-rod  24 
inches  long,  centre  to  centre,  makes  300  revolutions  per  minute.  Find 
graphically  the  velocity  of  the  piston  at  six  intermediate  positions  of  the 
stroke,  and  draw  a  curve  showing  the  velocity  of  the  piston  at  any  instant. 

(C.  &  G,,  1908,  O.,  Sec.  A.) 


LECTURE  XIII. — ORDINARY  QUESTIONS. 

I.  A  crane,  tested  in  the  usual  way,  and  in  which  the  velocity  ratio  is 
40,  gave  the  following  results  : 


Weight  lifted  (W) 

ICO 

300 

500 

700 

Force  applied  (P) 

8-5 

17-0 

25-6 

34  '2 

Plot  a  curve  showing  the  relation  between  P  and  W  on  a  W  base,  and,  OIL 
the  same  base,  plot  a  curve  of  efficiency.  (C.  &  G.,  1906,  0.,  Sec.  A.) 


LECTURE  XV.— ORDINARY  QUESTIONS. 

i.  Taking  the  mean  diameter  of  the  thread  of  a  i-inch  bolt  to  be  0*92- 
inches,  the  number  of  the  threads  to  the  inch  being  8,  and  the  coefficient 
of  friction  O'I7  ;  find  the  turning  couple  required  to  overcome  an  axial 
force  of  2i  tons,  and  the  efficiency  under  this  load. 

(C.  &  G.,  1906,  O.,  Sec.  A.) 


LECTURE  XVI.— ORDINARY  QUESTIONS. 

i.  To  do  the  cutting  work  in  a  small  screw  cutting  lathe  it  is  found: 
that  0-47  H.-P.  is  required,  and  that  the  frictional  losses  in  the  gearing, 
bearings,  &c.,  absorb  another  0*21  H.-P.  How  many  foot-pounds  of  work 
per  minute  is  the  driving-belt  giving  to  the  lathe  ?  j 

The  countershaft  is  driven  by  an  electric  motor,  and  the  countershaft 
and  belts  absorb  0-17  H.-P.  How  many  watts  must  the  motor  give  off  in 
order  to  keep  the  lathe  running  ? 

If  the  voltage  is  220,  how  many  amperes  will  the  motor  require,  assuming; 
that  its  own  efficiency  is  89  per  cent.  ? 

i  H.-P.  =  746  watts,  and  amperes  multiplied  by  volts  =  watts. 

(B.  of  E.,  1906.) 


3/8  APPENDIX  B. 

2.  The  table  of  a  drilling  machine  is  raised  by  a  hand- wheel,  to  the  spindle 
of  which  is  attached  a  single-threaded  worm  which  meshes  with  a  worm- 
wheel  having  40  teeth.     Compound  with  the  worm-wheel  is  a  spur-pinion, 
having  19  teeth  of  i-inch  pitch,  which  meshes  with  a  rack  on  the  frame  of 
the  machine.     Sketch  the  arrangement  and  find  how  many  turns  of  the 
handle  are  required  to  raise  the  table  through  2  feet. 

(C.  &  G.,  1906,  0.,  Sec.  A.) 

3.  In  the  feed  gear  of  a  drilling  machine,  in  which  a  rack  is  used  to  give 
the  traverse  of  the  spindle,  the  spindle  is  rotated  by  a  bevel- wheel  of  18 
teeth  keyed  on  the  driving  shaft,  gearing  with  one  of  32  teeth  on  the  sleeve 
surrounding  the  spindle.     The  greatest  and  least  diameters  of  the  pulleys 
•on  the  speed  cone  for  the  driving  shaft  are  7  inches  and  4  inches,  and  this 
cone  drives  a  similar  speed  cone  on  the  horizontal  speed  shaft.     On  this 
shaft  is  a  single-threaded  worm  which  gears  with  a  worm-wheel  of  45  teeth 
on  the  vertical  feed  shaft.     A  single-threaded  worm  on  this  shaft  gears 
with  a  wheel  of  30  teeth,  turning  on  a  horizontal  stud,  and  to  which  is 
attached  a  pinion  of  15  teeth  gearing  with,  a  rack  of  £-inch  pitch,  which 
gives  the  required  feed.     Find  the  least  and  greatest  number  of  revolutions 
of  the  drill  spindle  per  inch  of  feed.  (C.  &  G.,  1906,  O.,  Sec.A.) 

4.  The  traverse  shaft  of  a  lathe  is  driven  from  the  headstock  mandrel 
by  belting,  the  greatest  diameter  of  the  speed  cone  at  the  extremity  of  the 
mandrel  being  5  inches.     This  drives  a  similar  cone  on  the  transverse  shaft, 
and  the  smallest  diameter  is  2  inches.     A  worm  on  the  traverse  shaft  meshes 
with  a  single-threaded  worm-wheel,  having  40  teeth,  turning  on  a  stud 
carried  by  the  saddle.     At  the  front  end  of  this  spindle  is. a  spur-wheel  of 
15  teeth,  meshing  with  a  wheel  of  45  teeth,  which  turns  on  a  stud  carried 
by  the  apron ;   and  compound  with  this  last  wheel  is  a  pinion  of  12  teeth, 
which  meshes  with  the  rack  of  £-inch  pitch,  attached  to  the  lathe  bed. 
Sketch  the  mechanism  and  find  the  traverse  of  the  saddle  per  revolution 
of  the  headstock  mandrel.  (C.  &  G.,  1907,  0.,  Sec.  A.) 

5.  A  lathe  is  driven  by  a  belt  running  on  the  1 2-inch  diameter  pulley  of 
its  speed  cone,  which  then  revolves  at  200  revolutions  per  minute,  and  the 
back  gear  of  the  lathe  reduces  this  speed  in  the  ratio  of  9  to  i.     Under 
these  working  conditions  it  is  found  that  when  a  certain  cut  is  being  taken 

•off  a  bar,  6  inches  diameter,  the  horse-power  transmitted  by  the  belt  is 
0*60.  What  is  the  pressure  on  the  cutting  tool  in  a  direction  tangential 
to  the  turned  surface,  if  we  assume  that  75  per  cent,  of  the  power  trans- 
mitted through  the  belt  is  lost  in  frictional  and  other  wasteful  resistances  ? 

(B.  of  E.,  1908.) 

6.  Show  how  screws,  differing  in  pitch  from  the  leading  screw,  can  be 
cut  in  a  lathe.  If  the  leading  screw  of  a  lathe  has  three  threads  per  inch,  and 
is  right-handed,  what  arrangements  of  change  would  you  use  to  cut  (i)  a 
right-handed  screw  of  four  threads  to  the  inch,  (ii)  a  left-handed  screw  of 
eleven  threads  to  the  inch  ?     You  may  assume  that  you  have  a  set  of  change 
wheels  with  teeth  varying  from  20  to  100  by  differences  of  five  teeth 

(C.  &  G.,  1908,  0.,  Sec.  A.) 

7.  Describe,  with  the  help  of  neatly-drawn  sketches  which  should  be 
roughly  to  scale,  any  form  of  loose  head-stock  or  poppet-head  for  a  small 
lathe,  with  which  you  have  had  practical  experience.     Show  how  the 
spindle  or  poppet  is  advanced  or  withdrawn,  and  how  it  is  clamped. 

(B.  of  E.,  S.  i.,  1909.) 

!  8.  A  small  machine  tool  is  driven  direct  by  an  electric  motor.  How 
would  you  determine  the  horse-power  absorbed  in  the  process  of  cutting 
*he  material  ?  (B.  of  E.,  S.  i,  1909.) 


ORDINARY  QUESTIONS. 


379 


9.  The  countershaft  of  a  drilling  machine  makes  240  revolutions  per 
minute,  and  it  carries  a  stepped  pulley,  the  diameters  of  which  are  12,  9 
and  6  inches  respectively.  This  drives  an  intermediate  shaft  by  a  belt  and 
pulley  with  similar  steps.  The  intermediate  shaft  drives  the  drill  spindle 
by  a  bevel  wheel  of  30  teeth  gearing  with  one  of  40  teeth  on  the  drill  spindle. 
Calculate  the  possible  speeds  of  the  drill  spindle,  and  also  determine  the 
diameter  of  the  largest  drill  you  can  use  if  the  circumferential  cutting 
speed  is  limited  to  240  inches  per  minute. 

(C.  &  G.,  1909,  0.,  Sec.  A.) 


LECTURE  XVIII. — ORDINARY  QUESTIONS. 

I.  The  following  results  were  obtained  during  an  experiment  to  deter- 
mine the  quantity  of  water  which  would  be  discharged  through  a  small 
circular  orifice  in  the  side  of  a  tank.  The  diameter  of  the  orifice,  which 
had  sharp  edges,  was  I  inch. 


Number  of 
experiment 

Duration  of 
experiment 

Actual 
discharge 

Head  of  water  above 
centre  of  orifice 

Minutes. 

Lbs. 

Inches. 

I 

15 

576 

1-5 

2 

15 

660 

2'0 

3 

15 

733 

2'5 

4 

15 

827 

3'27 

5 

15 

915 

4-01 

6 

15 

1,011 

5*o 

7 

10 

737 

6-0 

8 

10 

788 

7'0 

jj-  Plot  on  squared  paper  a  curve  to  show  the  relation  between  the  dis- 
charge in  Ibs.  per  minute,  and  the  head  of  water  above  the  centre  of  the 
orifice. 

From  your  curve  determine  the  discharge  in  gallons  per  hour  when  the 
head  of  water  was  5^  inches.  (B.  of  E.,  1907.) 

2.  A  straight  balk  of  timber  is  20  feet  long  and  12  inches  square  in  cross- 
section  :  its  weight  per  cubic  foot  is  43*5  Ibs.  If  a  weight  of  112  Ibs.  is 
placed  on  the  centre  of  the  balk  when  it  is  floating  in  water,  find  the  depth 
to  which  the  balk  will  be  immersed.  (B.  of  E.,  S.  i,  1909.) 


LECTUEE  XIX — ORDINARY  QUESTIONS. 

I.  The  rim  of  a  turbine  is  going  at  50  feet  per  second ;  100  Ibs.  of  fluid 
enter  the  rim  each  second,  with  a  velocity  in  the  direction  of  the  run's 
motion  of  60  feet  per  second,  leaving  it  with  no  velocity  in  the  direction 
Of  the  wheel's  motion.  What  is  the  momentum  lost  per  second  by  the 
fluid  ?  This  is  force.  _^What  work  is  done  per  second  upon  the  wheel  ? 

(B.  of  E.,  1907.)    ' 


APPENDIX  B. 

2.  A  hydraulic  press  has  a  ram  6  inches  in  diameter :   water  is  supplied 
to  the  press  from  a  single-acting  pump,  which  has  a  plunger  i  inch  in 
•diameter  with  a  stroke  of   i£  inches.     Neglecting  frictional  and  other 
losses  in  the  pump  and  press,  find  the  average  rate  (in  foot-pounds  per 
minute)  at  which  the  pump  works,  if  it  makes  100  working  strokes  per 
minute,  while  the  press  is  exerting  a  force  of  70  tons.          (B.  of  E.,  1907.) 

3.  A  centrifugal  pump,  driven  by  an  electric  motor  directly  coupled  to 
it,  is  found  during  a  test  to  deliver  320  gallons  of  water  per  minute  into 
«,n  overhead  tank,  the  mean  height  of  lift  being  65  feet.     What  useful 
horse-power  is  the  motor  doing  ? 

If  during  the  test  the  electric  motor  takes  35  amperes  of  current  at 
440  volts,  what  is  the  combined  efficiency  of  the  whole  plant  ? 

(B.  of  E.,  1908.) 

4.  A  turbine,  which  gives  off  50  horse-power  to  a  belt  running  on  a  pulley 
on  its  shaft,  is  supplied  with  water  which,  as  it  enters  the  turbine,  is  under  a 
head  of  125  feet.     If  75  per  cent,  of  the  total  energy  of  the  entering  water 
is  thus  utilised,  what  work  is  done  per  pound  of  water,  and  how  many 
gallons  of  water  pass  through  the  turbine  per  working  day  of  10  hours  ? 

(B.  of  E.,  S.  i,  1909.) 


LECTURE  XX. — ORDINARY  QUESTIONS. 

i.  Describe,  with  the  help  of  neatly  drawn  sketches,  which  should  be 
roughly  to  scale,  a  hydraulic  jack,  showing  clearly  all  valves. 


(B.  of  E.,  1908.) 
:,  the  wa1 


2.  In  a  hydraulic  crane,  with  a  ram  8  inches  diameter,  the  water  pressure 
is  800  Ibs.  per  square  inch,  and  the  velocity  of  lift  is  increased  eight-fold 
by  the  use  of  a  four-sheaved  pulley  block.  What  load  can  this  crane  lift 
if  its  mechanical  efficiency  is  40  per  cent.  ? 

How  many  gallons  of  power  water  will  be  used  in  lifting  the  load  50  feet  ? 

(B.  of  E.,  1908.) 


LECTURE  XXI. — ORDINARY  QUESTIONS. 

1.  A  cycle  track  is  approximately  elliptical  in  shape,  the   maximum 
radius  of  curvature  being  1 50  yards  and  the  minimum  50  yards. 

Find  at  each  of  these  two  places,  the  ratio  which  the  centrifugal  force 
bears  to  the  weight,  if  the  speed  of  the  racing  cyclist  is  25  miles  per  hour. 
What  would  be  the  two  inclinations  of  the  track  to  the  horizontal  if  the 
track  is  laid  so  as  to  be  perpendicular  to  the  resultant  force  in  each  case  ? 

(B.  of  E.,  1906.) 

2.  A  railway  truck  weighing  10  tons  starts  from  rest  down  an  incline 
£  mile  long  of  i  in  250.     If  the  frictional  and  other  resistances  are  equiva- 
lent to  8  Ibs.  per  ton  weight  of  the  truck,  with  what  velocity  will  the  truck 
be  moving  when  it  gets  to  the  end  of  the  incline  ? 

How  far  would  it  then  run  along  a  level  stretch  of  the  line  before  coming 
to  rest  ?  (B.  of  E.,  1906.; 


ORDINARY  QUESTIONS. 


381 


3.  Two  adjacent  positions,  Glt  G2  of  the  centre  of  mass  G  of  a  balance 
weight  were  obtained  by  geometrical  construction  from  a  skeleton  diagram 
of  the  mechanism.  These  positions,  measured  in  feet  from  two  perpen- 
dicular axes,  were  found  to  be  as  follows  : 


X 

y 

01 

0-167 

0*078 

o, 

0-352 

0-146 

The  displacement  Ol  G2  took  place  in  1/50  second.     Find  the  x  and  y 
components  of  the  mean  velocity  of  G  for  this  interval. 

Plot  the  points  Glt  G.2,  on  squared  paper.  (B.  of  E.,  1906.) 

4.  A  traction  engine  travels  at  6  miles  per  hour ;    the  road  wheels  are 
6  feet  in  diameter  and  are  driven  through  5  to  i  gearing.     Find  the  angular 
velocity  in  radians  per  second  of  the  fly- wheel  on  the  engine  shaft. 

(B.  of  E.,  1907.) 

5.  The  rim  of  a  cast-iron  pulley  has  a  mean  radius  of  12  inches;   the 
rim  is  6  inches  broad,  and  £  inch  thick,  and  the  pulley  revolves  at  the  rate 
of  1 50  revolutions  per  minute  ;   what  is  the  centrifugal  force  on  the  pulley 
rim  per  inch  length  of  rim  ? 

One  cubic  inch  of  cast-iron  weighs  0-26  Ib.  (B.  of  E.,  1907.) 

7.  With  an  automatic  vacuum  brake  a  train,  weighing   170  tons  and 
going  at  60  miles  an  hour  on  a  down  gradiant  of  I  hi  100,  was  pulled  up  in 
a  distance  of  596  yards.     Find  the  total  resistance  per  ton  in  pounds,  and 
the  time  taken  to  stop  the  train.  (C.  &  G.,  1907,  0.,  Sec.  A.) 

8.  A  horizontal  jet  of  water  issues  at  a  velocity  of  20  feet  per  second 
fiom  the  2 -inch  diameter  nozzle  of  a  hose  pipe,  and  strikes  a  vertical  wall. 
What  is  the  mass  of  water,  in  engineers'  units,  which  strikes  the  wall  per 
second  ?     What  is  the  momentum  of  this  quantity  of  water  ?     What  is 
the  force  on  the  wall  ?     It  is  assumed  that  no  water  splashes  back. 

(B.  of  E.,  1908.) 

9.  A  man,  whose  weight  is  14  stone,  stands  on  the  floor  of  a  lift.     What 
force  does  he  exert  on  it  (i)  when  the  lift  is  stationary,  (ii)  when  it  is  des- 
cending with  an  acceleration  of  10  feet  per  second  per  second,  and  (iii)  when 
it  is  ascending  with  the  same  acceleration  ?  (B.  of  E.  1908.) 

10.  A  train  is  running  round  a  circular  curve  of  2000  feet  radius  at  a 
speed  of  50  miles  per  hour.     A  weight  is  suspended  by  a  thin  cord  from 
the  roof  of  one  of  the  carriages ;    at  what  inclination  to  the  vertical  will 
the  cord  hang  ?  (B.  of  E.,  1908.) 

11.  The  fly-wheel  of   a  punching  machine  weighs  i£  tons,  and  has  a 
radius  of  gyration  of  3  feet.     It  is  turning  at  the  rate  of  130  revolutions 
per  minute  when  the  punching  of  a  hole  is  started,  but  at  the  completion 
of  the  operation  of  punching  it  is  found  to  be  turning  at  the  rate  of  only 
125  revolutions  per  minute.     How  many  foot-pounds  of  work  have  been 
expended  in  punching  the  hole  and  overcoming  the  frictional  resistances 
of  the  machine  ? 

Note. — If  k  is  radius  of  gyration,  this  means  that  we  may  imagine  the 
mass  of  the  whole  wheel  to  be  at  the  distance  k  from  the  axis,  and  this 
enables  us  to  calculate  its  kinetic  energy.  (B.  of  E.,  1908.) 


382 


APPENDIX  B. 


12.  Explain  how  to  determine  the  velocity  of  a  moving  body  by  con- 
sidering the  space  described  in  a  given  time.  A  motor  car,  starting  from 
rest,  travels  a  distance  of  s  feet  in  t  seconds,  in  accordance  with  the  following 
table : 


t 

0 

I 

2 

3 

4 

5 

6 

7 

8 

8 

0 

5 

16 

33 

56 

85 

1  20 

161 

208 

Draw  a  curve  showing  the  distance  travelled  at  any  time  within  this  period, 
and  from  this  curve  determine  the  velocity  of  the  car  at  the  ends  of  the 
third,  fourth,  and  fifth  seconds  respectively.  (C.  &  G.,  1908,  O.,  Sec.  A.) 

13.  A  pulley,  3  feet  in  diameter,  has  a  peripheral  speed  of  2000  feet  per 
minute.     It  is  unbalanced  to  an  amount  which  may  be  represented  by  a 
mass  of  0*5  Ibs.  at  a  radius  of  I  foot.     Calculate  the  unbalanced  force  on 
the  pulley-shaft,  and  determine  the  positions  at  the  pulley-rim  of  two 
masses  of  0-4  Ibs.  to  give  a  perfect  balance.         (C.  &  G.,  1908,  O.,  Sec.  A.)  ^ 

14.  An  experiment  with  a  small  Pelton  water-wheel  gave  results  shown Jn 
the  annexed  table : 


Mean 
Revs, 
per  Min. 

Cubic  Feet 
of  Water 
passing 
through 
Wheel 
per  sec. 

Speed  of 
Jet.     Feet 
per  sec. 

V. 

Peripheral 
Speed 
of  Vane. 
Feet  per 
Sec. 

V. 

Ratio. 

V 

T 

Efficiency 
of  Wheel. 
Per  Cent. 

1090 

•0870 

44-0 

975 

•0824 

63*8 

885 

•0834 

72-4 

645 

•0840 

72-8 

540 

•0840 

66-8 

460 

•0847 

6i-5 

385 

•0834 

56-1 

-       265 

•0860 

40-1 

£   The  cross-sectional  area  of  the  nozzle  in  square  feet  was  0*001043.     Th 
mean  diameter  of  the  bucket  was  107  inches. 

Fill  in  the  third,  fourth,  and  fifth  columns  of  the  table. 

Plot  a  curve  to  show  the  variation  of  efficiency  with  variation  of  ratl 

y  taking  efficiency  in  vertical  ordinates  and  -y  in  horizontal  abscissae. 

T  (B.  of  E.,  S.  i,  1909.) 


ORDINARY  QUESTIONS. 


383 


15.  Define  the  term  "  horse-power."    The  tension  on  the  draw-bar  of  a 
locomotive  is  800  Ibs.  when  the  speed  is  45  miles  per  hour.     The  weight  of 
the  train,  excluding  the  locomotive,  is  200  tons.     If  the  efficiency  of 
the  locomotive  is  65  %,  find  the  horse-power  of  the  engine.      Also  find 
the  accelerating  force  exerted  at  the  draw- bar  to  change  the  speed  from 
45  to  50  miles  per  hour  in  one  minute.  (C.  &  G.,  1909,  O.,  Sec.  A.) 

16.  Define  the  terms  "  velocity  "  and  "  acceleration,"  and  show  how  to 
determine  the  acceleration  of  a  body  moving  in  a  straight  line  when  its 
velocity  is  known  at  each  instant.     The  accompanying  diagram  is  to  be 


6 


CUEVE  REPRESENTING  THE  VELOCITY  OF  BODY. 

drawn  so  that  each  unit  square  is  of  I  inch  side.  The  curve  ABC  represents 
the  velocity  of  a  body  during  a  given  interval  of  time,  it  is  circular  from 
A  to  B,  with  centre  at  D,  and  from  B  to  C  it  is  a  straight  line  parallel  to 
the  horizontal  axis.  Draw  to  scale  a  curve  showing  the  acceleration  of 
the  body  at  any  instant,  and  mark  on  it  the  numerical  values  of  the  ac- 
celeration at  the  end  of  each  second.  (C.  &  G.,  1909,  O.,  Sec.  A.) 

ft  17.  Show  how  the  resultant  velocity  of  a  body  may  be  obtained  when  the 
component  velocities  impressed  upon  it  are  known.  A  goods  engine 
moving  at  the  rate  of  20  miles  per  hour,  has  equal  coupled  wheels  54  inches 
in  diameter,  and  the  crank-pins  move  in  circles  24  inches  in  diameter. 
Determine  the  velocities  of  the  crank-pins  relative  to  the  rails  for  three 
equally  spaced  positions  reckoned  from  the  lowest  positions  of  the  crank- 
pins.  (C.  &  G.,  1909,  O.,  Sec.  A.) 

^  1  8.  Assuming  the  expression  for  the  law  of  centrifugal  force,  show  how  to 
obtain  a  formula  for  the  stress  /in  the  rim  of  a  fly-wheel  in  the  form 


where  w  is  the  weight  of  a  unit  volume  of  the  rim,  v  is  the  velocity,  and  g 
is  the  gravitation  constant.  Determine  the  limiting  speed  in  revolutions 
per  minute  of  a  cast-iron  fly-wheel  rim  having  a  mean  diameter  of  10  feet, 
when  the  allowable  stress  is  2400  Ibs.  per  square  inch.  The  weight  of  one 
cubic  inch  of  cast-iron  may  be  taken  as  0*28  Ibs. 

(C.  &  G.,  1909,  O.,  Sec.  A.) 


2  B 


APPENDIX  B. 


19.  Ar  hammer  head  weighing  3-22  Ibs.  moving  at  30  feet  per  second  is 
stopped  in  o'ooi  second  :  what  is  the  average  force  of  this  blow  in  pounds  ? 

(B.  of  E.,  S.  i,  1909.) 

20.  A  locomotive  is  travelling  at  60  miles  per  hour.     The  driving-wheels 
are  6  feet  6  inches  in  diameter.     What  is  the  angular  velocity  of  the  driving- 
wheels  in  radians  per  second  ?  ¥•' 

If  the  stroke  of  the  piston  is  26  inches,  what  is  the  mean  speed  of  the 
piston  relatively  to  the  cylinder  in  feet  per  minute,  assuming  there  is  no 
slip  of  the  wheels  ?  (B.  of  E.,  S.  i,  1909.) 

21.  A  fly-wheel,  which  weighs  18  tons,  when  mounted  on  its  axis  and 
rotated,  is  found  to  be  out  of  balance,  and,  in  order  to  bring  it  into  balance 
it  is  found  necessary  to  fix  a  counter-weight  of  420  Ibs.  to  the  wheel,  at 
a  distance  of  90  inches  from  the  axis  of  the  shaft.     What  was  the  distance 
of  the  centre  of  gravity  of  the  unbalanced  wheel  from  the  axis  of  the  shaft  ? 

Show  by  a  sketch  where  you  would  fix  the  counter-weight. 

(B.  of  E.,  S.  i,  1909.) 

22.  A  fly-wheel  weighs  8  tons,  its  radius  of  gyration  is  5  feet  5  inches, 
and  it  is  rotating  at  a  speed  of  90  revolutions  per  minute.     How  many 
foot-pounds  of  energy  are  stored  up  in  it  ?     (P-  » ' 

If  this  wheel  were  supported  in  two  bearings,  each  12  inches  in  diameter, 
and  if  the  coefficient  of  friction  were  0*01,  how  much  energy  is  wasted  hi 
overcoming  friction  in  one  revolution,  and  how  many  revolutions  would 
this  fly-wheel  make  before  coming  to  rest  after  the  turning  force  was  cut 
off  ?  '  (B.  of  E.,  S.  i,  1909.)  | 


LECTURE  XXII.— ORDINARY  QUESTIONS. 

1.  A  tie-bar  in  a  roof  is  made  of  steel  angle  bar  ;  the  section  of  the  steel 
angle  bar  is  4  inches  by  4  inches  by  £  inch,  and  the  tie- bar  when  finished 
in  the  workshop  is  20  feet  in  length.     When  in  position  in  the  roof  the  tie- 
bar  may  during  a  gale  have  to  resist  a  total  pull  of  22^  tons  ;   what  is  the 
tensile  stress  per  square  inch  in  the  metal  of  the  tie-bar  under  these  con- 
ditions, and  how  much  would  the  tie-bar  lengthen  under  this  load  ?  ,-, 

Young's  modulus  of  elasticity  is  12,500  tons  per  square  inch.      ^  T 

(B.  of  E.,  1906.) 

2.  A  knuckle  joint  is  required  to  withstand  a  tensile  force  of  10  tonsi 
The  safe  working  stress,  both  in  tension  and  shear,  may  be  taken  as  9000  Ibs. 
per  square  inch.      Find  the  diameters  of  the  rod  and  pin,   and  sketch 
the  joint,  roughly,  to  scale.  (C.  &  G.,  1906,  0.,  Sec.  B.) 

3.  A  rectangular  test-bar,  in  tension,  gave  the  following  results : 


Total  1 
load     > 
in  Ibs.  J 

8,000 

16,000 

24,000 

32,000 

34»ooo 

40,000 

48,000 

56,000 

60,000 

SS.ooo 

Exten-^ 

sionin  f 
inches.  J 

'002 

•0044 

•0070 

'0103 

'Oi6 

•Tgc 

P470 

r3fi 

2'S 

2-9 

Sketch  a  curve  showing  the  relation  between  force  and  extension  on  any 
suitable  scale — squared  paper  may  be  used — and  infer  the  stress  at  the 


ORDINAEY   QUESTIONS.  385 

elastic  limit  and  the  maximum  stress,  the  original  dimensions  of  the  bar 
being  1763  inches  by  '611  inches.  If  the  distance  between  the  gauge 
points  is  10  inches,  find  the  coefficient  of  elasticity  (E)  of  the  bar.^,  -  ,-- 

(C.  &  G.,  1906,  p.,  Sec.  B.)^? 

4.  Explain  what  you  mean  by  the  efficiency  of  a  riveted  joint,  and  point 
out  on  what  it  depends.     In  a  marine  boiler  the  diameter  is  12  feet,  the 
working  pressure  is  200  Ibs.  per  square  inch,  and  the  longitudinal  joints 
are  butt  joints  with  double  straps  treble  riveted.     If  the  ultimate  stress  is 
62,000  Ibs.    per  sq.  inch,  the  factor  of  safety  5,  and  the  efficiency  of  the 
joint  -8,  find  the  thickness  of  the  plate  required,  and  make  a  rough  sketch 
of  the  joint.  (C.  &  G.,  1906,  O.,  Sec.  B.) 

5.  In  order  to  connect  together  the  two  halves  of  a  long  tie-rod,  an 
eye  is  forged  at  the  end  of  one  half,  and  a  fork  (into  which  the  eye  enters 
at  the  end  of  the  other  half,  and  a  pin  is  passed  through  the  two  sides  of 
the  fork  and  through  the  eye.     If  the  total  pull  in  the  tie-rod  is  16  tons, 
and  if  the  shearing  stress  in  the  metal  of  the  bolt  is  not  to  exceed  8000  Ibs. 
per  square  inch,  what  diameter  would  you  make  the  pin  ? 

(B.  of  E.,  1907.) 

6.  A  strut  is  built  up  out  of  two  pieces  of  T-steel,  each  6  inches  by  3 
inches  by  f  inch,  riveted  back  to  back.     If  this  strut  supports  a  load  of 
22-3  tons,  what  is  the  compressive  stress  per  square  inch  ? 

If  a  total  load  of  105  tons  would  destroy  this  strut,  what  is  the  factor  of 
safety  ?  (B.  of  E.,  1907.)  £ 

7.  A  bar,  of  rectangular  section,  1 75  inches  wide  and  0*6 1  inches  thick, 
is  found  under  a  load  of  20,000  Ibs.  to  have  stretched  0*0056  inch.     Find 
the  stress  induced,  and,  if  the  length  be  10  inches,  find  Young's  modulus. 

(C.  &  G.,  1907,  O.,  Sec.  B.) 

8.  Find  the  thickness  of  the  plates  of  a  cylindrical  boiler  50  inches  in 
diameter  to  sustain  a  pressure  of  50  Ibs.  per  square  inch,  the  working  stress 
being  4000  Ibs.  per  sq.  inch  and  the  efficiency  of  the  joint  being  o-6o. 

(C.  &  G.,  1907,  O.,  Sec.  B.) 

9.  If  in  the  last  question  the  joint  is  a  lap  joint  double  riveted,  and  the 
diameter  of  the  rivets  is  f  inch,  find  the  pitch,  the  shear  stress  of  rivets, 
being  4000  Ibs.  per  square  inch.  (C.  &  G.,  1907,  0.,  Sec.  B.) 

10.  A  piece  of  steel  is  to  be  tested  in  tension ;    show  how  you  would 
proceed  to  make  a  test,  and  indicate,  by  means  of  a  diagram,  how  the 
force  and  extension  vary  with  each  other.          (C.  &  G.,  1907,  O.,  Sec.  B.) 

11.  A  copper  trolley  wire,  which  is  0*45  inch  in  diameter  and  60  feet  in 
length,  is  found  to  elongate  0-075  mch  under  a  certain  pulL     If  the    mod- 
ulus of  elasticity  (Young's  modulus)  of  this  quality  of  copper  is  known 
to  be  15,000,000  Ibs.  per  square  inch,  what  is  the  total  pull  in  the  trolley 
wire  ?  (B.  of  E.,  1908.) 

12.  Explain  the  meanings  of  the  terms  "  stress,"  "  strain,"  and  "  modulus 
of  elasticit  y,  "  by  reference  to  the  case  of  a  rod  under  tensional  stress.     A 
piece  of  bo  iler  plate,  2  inches  by  f  inch  in  cross-section,  has  a  load  of  12,000 
Ibs.  applied    to  it  in  a  testing  machine.     The  modulus  of  elasticity  of  the 
material  e  xpressed  in  inches  and  pounds  is  31,000,000.     Calculate  the 
values  of    the  stress  and  strain  and  determine  the  increase  of  length,in 
a  len  gth  of  12  inches,  due  to  the  applied  load.     (C.  &  G.,  1908,  0.,  Sec.  B.) 

I  3 .  Make  a  sketch  of  a  knuckle  joint  connecting  an  eccentric  rod  to  a 
val  ve  spindle,  and  assuming  that  the  total  load  on  the  latter  is  4000  Ibs., 
dete  nnine  the  dimensions  of  the  various  parts,  and  design  the  joint.  Show 
yo  ur  calculations  clearly,  and  state  what  working  stesses  you  have  assumed 

(C.  &  G.,  1908,  0.,  Sec.  B.) 


386  APPENDIX  B, 

14.  A  tension  member,  8  inches  by  J  inch  in  cross  section,  has  a  riveted 
butt-joint  with  cover  plates  on  each  side.    The  total  load  on  the  member 
is  72,000  Ibs.    Design  and  draw  a  joint  for  this  member,  and  show  all 
your  calculations.  (C.  &  G.,  1908,  O.t  Sec.  B.) 

1 5.  A  hollow  cylinder,  10  inches  mean  diameter,  10  feet  long,  and  ij  inches 
thick,  is  to  be  oast  with  its  axis  vertical.    Taking  the  specific  gravity  of  cast 
iron  as  7-5,  find  the  pressure  on  the  bottom  of  the  mould  when  it  is  full  of 
metal.     One  side  of  a  mould  for  a  cast-iron  casting  is  a  rectangle,  3  feet  deep 
by  2  feet  wide.     Find  the  whole  pressure  on  the  side  of  the  mould. 

(C.  &  G.,  1908,  0.,  Sec.  D.) 

1 6.  In  an  experiment  with  a  hollow  cast-iron  column,    6  feet  long,'*  5 
inches  in  external  diameter,  and   4   inches   in  internal  diameter,   it  was 
found  that  under  a  compressive  load  of  30  tons  the  column  shortened  by 
0*063  inch;    what  is  the   value   of  Young's   Modulus  (E)  in  pounds  per 
square  inch  for  this  cast-iron  ? 

J^When  the  load  was  increased  to  192  tons,  the  column  broke;  what  was 
the  compressive  stress  in  tons  per  square  inch  at  the  instant  of  fracture. 

(B.  of  E.,  S.  i,  1909.) 


LECTUEE  XXIII.  ORDINARY  QUESTIONS. 

1.  Describe  how  you  would  determine  experimentally  the  modulus  of 
rigidity  of  either  a  block  of  india-rubber  or  a  steel  rod.      (B.  of  E.,  1906.) 

2.  In  a  direct-acting  steam-engine  mechanism  the  stroke  of  the  piston 
is  2  feet  and  the  crank  shaft  makes  1 50  revolutions  per  minute. 

What  is  the  speed  of  the  crank  shaft  in  radians  per  second  ?  What  is 
the  speed  of  the  crank  pin  in  feet  per  second  ?  What  is  the  mean  speed 
of  the  piston  in  feet  per  minute  ?  (B.  of  E.,  1906.) 

3.  A  solid  cylindrical  shaft  is  5  inches  in  diameter.     Find  the  external 
diameter  of  a  hollow  shaft  of  same  material,  the  internal  diameter  of  which 
is  two-thirds  the  external  and  which  shall  have  the  same  strength.     Com- 
pare the  weights  in  the  two  cases.     If  the  safe  working  stress  be  4  tons  per 
square  inch,  and  the  revolutions  per  minute  zoo,  find  the  greatest  horse- 
power which  can  be  safely  transmitted.       (C.  &  G.,  1906,  O.,  Sec.  B.) 

4.  A  motor  having  a  turning  moment  T  is  coupled  directly  to  a  shaft 
making  N  revolutions  per  minute.     Show  how  to  calculate  the  work  trans- 
mitted by  the  shaft,  and  obtain  a  formula  for  the  horse-power  transmitted 
in^  terms  of  N  and  T  and  a  constant.     Determine  the  horse-power  trans- 
mitted by  a  shaft  making  800  revolutions  per  minute  if  the  turning  moment 

«  16,000,  measured  in  pounds  and  inches. 

(C.  &  G.,  1908,  0.,  Sec.  B.) 

5.  A  chain  is  to  be  used  for  lifting  a  load  of  5  tons.     Assuming  a  safe 
working  stress  on  the  chain  of  4  tons  per  square  inch,  find  the  diameter  of 
the  iron  of  the  chain.  (C.  &  G.,  1908,  0.,  Seo.  D.) 

6.  A  shaft  3  inches  in  diameter  transmits  a  twisting  moment  of  66,ooo-Ib. 
inches  and  the  flange  couplings  are  bolted  together  by  four  bolts  spacedi 
on  a  circle  of  5  inches  diameter.     Determine  the  nature  and  amount  of  stress 
on    each    bolt    and    determine    its    diameter    if    the    allowable    stress    is 

12,000  Ibs.  per  square  inch.  (C.  &  G.,  1909,  O.,  Sec.  B.) 

7.  Explain  what  kind  of  stress  is  produced  in  a  shaft  by  a  twisting 


OKDINARY  QUESTIONS.  387 

moment,  and  make  a  diagram  showing  how  the  stress  varies  across  the 
section  of  a  shaft.'*  A  piece  of  tubing,  2  inches  in  external  diameter,  and  £ 
inch  thick,  is'used  as  a  shaft.  Assuming  that  the  stress  upon  it  is  uniformly 
distributed,  determine  the  twisting  moment  it  will  transmit  if  the  allowable 
stress  is_ 1 2,000  Ibs.  per  square  inch.  (C.  &  G.,  1909,  0.,  Sec.  B.) 


LECTURE  XXIV. — ORDINARY  QUESTIONS. 

i.  Sketch~a"single  Hooke's  joint,  and  explain  in  general  terms/ how  the 
angular  velocity  ratio  varies  during  a  revolution.  What  is  the  object  of  a 
double  Hooke's  joint  ?  (C.  &  G.t  1906,  0.,  Sec.  A.) 


LECTUBE  XXV. — OBDINABY  QUESTIONS. 

1.  Sketch, ^andjdescribe  the  action  of,  the  pin  and  slot  mechanism  aa 
applied  to  shaping  machines. 

In  such  a  mechanism,  the  distance  between  the  two  centres  of  rotation 
is  3"inches,  and  the  time  ratio  has  to  be  two.  If  the  line  of  stroke  produced 
pass'through  the  centre  of  the  variably  rotating  crank,  and  is  perpendicular 
to' the  line  of  centres,  find  the  length  of  the  crank  radius,  and  also  of  the 
slotted-link,  for  a  stroke  of  10  inches.  (C.  &  G.,  1906,  O.,  Sec.  A.) 

2.  InTthe  four-bar  mechanism  shown  in  the  sketch,  the  bar  A  is"a  fixed 
bar ;  the*  bars  B  and  D  rotate  about  the  fixed  centres  OAB  and  OA  D,  and 
they  are  coupled  together  at  their  outer  ends  by  the  bar  C ;    the  bar  B 


FOUR  BAR  MECHANISM. 

revolves~with  uniform  velocity  round  its  fixed  axis" OAB  at  50  revolutions 
per  minute.     Find  in  any  way  you  please  the  position  of  the  bar  D  when 


388  APPENDIX  B. 

the  bar  B  is  turned  in  a  clockwise  direction  through  angles  of  30°,  60°,  and 
90°  from  the  position  shown  in  the  sketch. 

Prepare  a  table  similar  to  the  one  shown,  and  obtain  and  enter  up  the 
results  required  to  complete  the  table  : 


Angle  turned  through 
by  the  bar  B. 

Angle  turned  through 
by  the  bar  D. 

Mean  angular  velocity  of 
the  bar  D  in  radians  per 
second  during  each  interval  . 

30°.                    .          . 

60°. 

• 

90°. 

(B.  of  E.,  S.  i,  1907.) 

3.  Sketch  the  arrangement  in  a  planing  machine  in  which  bevel  gears 
are  used,  explaining  how  the  motion  is  reversed,  and  how  a  quick-return 
motion  is  obtained.  Sketch  also  a  "  shipper  "  mechanism  in  which  cams 
or  lugs  are  used.  (C.  &  G.,  1907,  0.,  Sec.  A.) 

P  4.  The  piston  of  a  vertical  engine  has  a  travel  of  18  inches,  and  the  con- 
necting-rod is  36  inches  in  length  between  centres  of  bearings.  The  line  of 
action  of  the  piston  cuts  the  horizontal  position  of  the  crank  4  inches  from 
the  centre  of  the  crank-shaft.  Draw  the  curve  of  position  of  the  mid-point 
of  the  connecting  rod  for  a  complete  revolution  of  the  crank.  Use  a  scale 
of  £th.  (C.  &  G.,  1908,  O.,  Sec.  A.) 

W$-  Describe,  with  the  help  of  neatly-drawn  sketches  which  should  be  roughly 
to  scale,  a  belt  gear  for  giving  a  slow  cutting  speed  and  a  quick  return 
motion  suitable  for  use  in  a  planing  machine,  the  table  of  which  is  traversed 
to  and  fro  by  a  screw  or  rack.  (B.  of  E.,  S.  i,  1909.) 


STUD.  INST.  C.E.  QUESTIONS.  389 

LECTUEE  II. — STUD.  I. C.E.  EXAM.  QUESTIONS. 

1.  Explain  the  use  of  diagrams  for  determining  the  results  of  experiment. 
Plot  the  following  experimental  values  of  P  and  W  and  obtain  the  relation 
between  them  : 

P  pounds          ...         8  16          24          40 

W    „  o        200        400        800 

(Stud.  I.  C.  E.,  Feb.  1906.) 

2.  Show  how  the  work  done  by  a  variable  force  can  be  represented 
graphically.     Assuming  that  the  resistance  of  a  spiral  spring",  is  propor- 
tional to  its  extension,  and  that  a  load  of  24  Ibs.  extends  the  spring 
0-25  inch,  determine  the  work  done  in  extending  the  spring  i  inch. 

(Stud.  L  C.  E.,  Oct.  1906.) 

3.  Define  the  terms  "  force  "  and  "  work" 

A  spring  is  compressed  and  the  relation  between  the  compressive  force 
and  the  compression  is  as  given  below.  Find  graphically  the  work  done 
in  the  process. 

Force  in  Ibs.  .         .    I    o    I    15    I    35    |    65    I    no    I    170    I    300 

Compression  in  inches     .    |   O    |      I    j      2    |      3    |        4    |        5     |        6 

(Stud.  I.  C.  E.,  Oct.  1907.) 

LECTURE  III. — STUD.  I. C.E.  EXAM.  QUESTIONS. 

1.  A  metre  rule  (weight  50  grams)  rests  on  the  edge  of  a  table  with  20 
centimetres  projecting  over  the  edge.     On  the  other  end  rests  a  2O-gram 
weight.     How  far  from  the  edge  of  the  table  may  a  5oo-gram  weight  be  hung 
before  the  rod  tilts  ?  (Stud.  I.  C.  E.,  Feb.  1905.) 

2.  Prove  that  the  moment  about  any  axis  of  three  forces  in  equilibrium 
is  zero,  and  extend  the  theorem  to  any  number  of  coplanar  forces  in 
equilibrium. 

Determine  the  tension  of  the  rope  wound  on  a  capstan  2  feet  in  diameter 
when  10  men,  each  weighing  12  stone,  are  pushing  horizontally  on  the 
capstan- bars  4  feet  from  the  deck  at  a  radial  distance  of  8  feet,  the  vertical 
through  a  man's  centre  of  gravity  overhanging  his  toes  a  distance  of 
I -foot.  (Stud.  I.  C.  E.,  Oct.  1905.) 

3.  Prove  that  the  centre  of  gravity  of  a  triangular  plate  of  uniform 
thickness  is  on  the  line  joining  an  apex  to  the  centre  of  the  opposite  side, 
and  at  a  distance  from  this  apex  of  two-thirds  of  the  length  of  this  median 
line.     Also  show  how  to  determine  by  experiment  the  centre  of  gravity  of 
an  irregular  plate  of  uniform  thickness.  (Stud.  I.  C.  E.,  Feb.  1906.) 

4.  Show  how  to  determine   the  resultant  of  two   parallel  forces.     A 
horizontal  bar,  6  feet  long,  is  supported  at  each  end  by  rings  depending 
from  spring  balances.     Determine  the  position  of  the  centre  of  gravity  of 
the  bar  if  the  spring  balances  indicate  loads  of  40  Ibs.  and  50  Ibs.  respectively. 

(Stud.  I.  C.  E.,  Oct.  1906.) 

5.  Explain  what  is  meant  by  the  moment  of  a  force.     Show  that  the 
sum  of  the  moments  of  two  forces  in  a  plane  with  respect  to  a  point  in 
that  plane  is  equal  to  the  moment  of  their  resultant.     Also  show  that  the 
moment  of  a  couple,  with  respect  to  any  axis  at  right  angles  to  the  plane 
of  the  couple  is  invariable.  (Stud.  I.  C.  E.,  Oct.  1906.) 

6.  Show  how  to  determine  experimentally  the  centre  of  gravity  of  an 
irregular  body,  and  in  particular  explain  how  you  would  proceed  to  deter- 
mine the  centre  of  gravity  of  a  metal  plate  shaped  to  the  section  of  a  tram- 
way rail  (Stud  I.  C.  E.,  Feb.  1907.) 


39°  APPENDIX  B. 

7.  What  do  you  understand  by  the  "  centre  of  gravity  "  of  a  body  ? 

A  balk  of  timber  weighs  800  Ibs.  One  end  rests  on  the  ground,and  the 
other  on  a  "  V  "  support  placed  on  a  weigh-bridge.  The  weight  recorded  is 
320  Ibs.  The  weigh-bridge  is  then  moved  so  that  the  "  V  "  is  I  foot  nearer 
the  end  that  is  resting  on  the  ground  and  the  weigh-bridge  registers  360 
Ibs.  Find  how  far  the  centre  of  gravity  of  the  balk  is  from  that  end. 

(Stud.  I.  C.  E.,  Oct.  1907.) 

8.  What  is  a  "  couple  "  ?     How  is  a  couple  specified,  and  how  can  it  be 
represented  ? 

A  pair  of  compasses  is  opened  so  that  the  legs  are  at  90°.  Couples  are 
applied  to  the  legs  whose  moments  are  respectively  3  Ib.-foot  units  and 
4  Ib.-foot  units,  and  they  twist  in  opposite  ways.  Find  what  couple  must 
be  applied  at  the  hinge  to  equilibrate  the  two,  and  the  axis  of  that  couple. 

(Stud.  I.  C.  E.,  Oct.  1907.) 

9.  Define  the  term  "  centre  of  gravity. "     A  cylindrical  vessel  is  5  feet  deep 
and  weighs  100  pounds ;   when  it  is  empty  its  centre  of  gravity  is  2  feet 
above  its  base.     It  is  gradually  filled  with  water.     Plot  to  scale  a  curve 
showing  the  relation  between  the  depth  of  water  in  the  vessel  and  the  height 
of  the  new  centre  of  gravity,  if  the  vessel  when  just  full  can  contain  500 
pounds  of  water.  (Stud.  I.  C.  E.,  Feb.  1908.) 

LECTURE  IV.— STUD.  I.C.E.  EXAM.  QUESTIONS. 

1.  Answer,  giving  reasons,  the  following  questions  on  the  balance  : 

(i)  What  conditions  must  be  satisfied  in  order  that  a  balance  may 

be  true  ? 

(ii)  If  the  scale-pan  knife-edges  are  above  the  middle  knife-edge 
show  that  the  sensitiveness  of  the  balance  increases  with  the 
load, 
(iii)  Why  must  the  scale-pans  be  suspended  freely  from  the  beam  ? 

(Stud.  I.  C.  E.,  Feb.  1905.) 

2.  Explain  the  action  of  one  form  of  lever  weighing-machine.     A  loo-ton 
testing  machine,  using  a  single  lever  for  weighing  the  pull  on  a  test-piece, 
is  arranged  so  that  the  line  of  action  of  the  pull  is  4  inches  distant  from 
the  fulcrum,  and  this  pull  is  balanced  by  a  weight  of  5000  Ibs.  on  the 
long  arm  of  the  lever.     Calculate  the  distance  the  weight  moves  from  its  zero 
position  to  balance  the  full  load  of  100  tons.         (Stud.  I.  C.  E.,  Feb.  1907. ) 

LECTURE  V.— STUD.  I.C.E.  EXAM.  QUESTIONS. 

i.  Explain  how  work  is  computed  when  a  force  moves  its  point  of  appli- 
cation in  any  direction. 

Find  the  work  done  per  minute  by  a  force  pulling  a  body  weighing 
400  Ibs.  over  a  rough  plane  at  the  rate  of  5  miles  an  hour,  if  the  coefficient 
of  friction  is  0-25.  (Stud.  I.  C.  E.,  Feb.  1906.) 

LECTURE  VI.— STUD.  I.C.E.  EXAM.  QUESTIONS. 

I.  Write  a  short  essay  on  the  use  and  principle  of  a  machine,  bringing 
in  the  meaning  of  the  terms  :  velocity  or  displacement  ratio,  effort,  load, 
advantage,  efficiency. 

Describe  some  experiment  you  have  made  with  a  machine,  and  illustrate 
the  meaning  of  the  above  terms  by  numerical  examples. 

(Stud.  I.  C.  E.,  Feb.  1905.) 


STUD.  INST.  C.E.   QUESTIONS. 


39* 


2.  Give  sketches  of  three  systems  of  pulleys,  and  state  their  mechanical 
advantage. 

Explain  which  system  is  preferable  for  a  long  pull,  as  in  hoisting  a 
weight ;  and  which  is  to  be  preferred  for  a  strong  pull,  as  in  setting  up  a 
backstay.  (Stud.  I.  a  E.f  Oct.  1905.) 


LECTURE  VTI. — STUD.  I.C.E.  EXAM.  QUESTIONS. 

I.  Explain  the  principle  of  action  of  the  "  Weston  "  or  differential 
pulley,  and  show  how  to  determine  the  displacement  or  velocity  ratio.  In 
such  a  machine  the  velocity  ratio  was  found  to  be  20,  and  in  order  to  lift 
a  weight  of  420  Ibs.  a  pull  of  40  Ibs.  was  exerted.  Determine  the  efficiency- 
of  the  machine  for  this  load.  (Stud.  L  C.  E.,  Feb.  1906.) 

LECTUBE  VITE. — STUD.  I.C.E.  EXAM.  QUESTIONS. 

1.  A  rod  AB,  the  weight  of  which  may  be  neglected,  is  hinged  at  A,  andl 
a  weight  of  14  Ibs.  is  suspended  from  its  middle  point.     A  string  is  fastened 
to  the  end  B,  and  when  the  system  is  at  rest  the  rod  is  inclined  at  30°  to- 
the  horizontal,  and  the  string  makes  an  angle  of  90°  with  the  rod.     Find 
the  pull  along  the  string.  (Stud  I.  C.  E.,  Oct.  1904.) 

2.  Two  rods  AC,  BC  are  freely  jointed  together  at  C,  and  a  load  of  28  Ibs. 
is  suspended  from  C.     The  two  ends  A  and  B  are  connected  by  a  horizontal 
string.     If  the  system  be  placed  vertically  with  A  and  B  on  a  smootk 


A  B 

Two  RODS  AC  AND  BC  WITH  STRING  AB. 

floor,  find  by  a  graphic  method  the  thrusts  along  AC,  BC,  and  the  pull  of 
the  string,  when  AB  =  5  feet,  AC  =  4  feet,  BC  =  3  feet.  *  I 

(Stud.  I.  C.  E.,  Feb.  1905.)    ' 

3.  Show  in  a  diagram  the  forces  which  maintain  equilibrium  in  a  book 
held  horizontally  between  a  finger  and  thumb,  and  draw  a  graphical  deter- 
mination of  their  magnitude. 

A  boat  is  propelled  by  two  sculls,  each  9  feet  long  and  6  feet  from  the 
rowlock  to  the  blade,  and  the  sculler  pulls  each  hand  with  a  force  of  20  Ibs. 
Prove  that  the  thrust  on  each  rowlock  is  30  Ibs.,  but  the  propulsive  torch- 
on the  boat  is  20  Ibs.,  and  that  the  boat  moves  about  double  as  fast  as  tee- 
hands  pull.  L  (Stud.  I.  C.  E.,  Oct.  1905.) 


392 


APPENDIX  B. 


^4.:  A  flexible  oord  is  carried  by  two  pegs  A  and  D  in  the  same  horizontal 
line  and  24  inches  apart.  A  weight  of  8  Ibs.  hangs  at  B  and  an  unknown 
weight,  at  C,J  thereby,  causing  the  cord  to  assume  the  form  shown  in  the 
tflgure.  ( 

A  *s—  6' — *~ tO' * 8" +D 


FLEXIBLE  CORD  WITH  ATTACHED  WEIGHTS. 

Find  by  a  graphical  construction  the  magnitude  of  the  unknown  weight 
srad  the  tensions  in  the  parts  AB,  BG  and  CD  of  the  cord. 

(Stud.  I.  C.  E.,  Feb.  1906.) 

5.  Show  how  to  determine  the  resultant  of  a  number  of  forces  meeting 
at  a  point.     Six  forces  acting  at  a  point  are  parallel  to  the  sides  of  a  regular 
hexagon  taken  in  order,  and  their  magnitudes  are  4,  6,  7,  9,  8  and  3  pounds 
respectively.     Find  the  resultant,  assuming  that  all  the  forces  are  directed 
towards  the  point.  (Stud.  I.  C.  E.,  Oct.  1906.) 

6.  Obtain  the  graphical  condition  for  the  equilibrium  of  a  number  of 
tf orces  acting  at  a  point.     A  jointed  frame  is  loaded  as  shown  in  the  figure. 
.Determine  the  stresses  in  the  members  of  the  frame. 

(Stud.  I.  C.  E.,  Feb.  1907.) 


,4000 


3000  Ibs 


A  JOINTED  LOADED  FRAME. 


STUD.  INST.  C.E.   QUESTIONS.  393 

7.  Give  the  conditions  that  forces  acting  on  a  rigid  body  may  be  hi 
equilibrium.  !    I  ' 

A  uniform  plate,  weighing  5  Ibs.,  is  made  in  the  form  of  a  right-angled 
area  whose  sides  are  3,  4,  and  5  feet.  It  is  hung  up  by  means  of  a  string 
•and  a  peg  so  that  the  5 -foot  side  is  horizontal  The  peg  is  at  the  angle 
joining  the  4-  and  5-foot  sides;  the  string  is  at  the  other  acute  angle  and 
makes  an  angle  of  60°  with  the  5 -foot  side.  Find  graphically  the  tension 
in  the  string.  (Stud.  I.  C.  E.,  Oct.  1907.) 

8.  What  condition  must  be  fulfilled  in  order  that  a  system  of  forces 
acting  on  a  body,  which  are  all  in  one  plane,  but  not  acting  at  one  point, 
may  be  hi  equilibrium       Show  how  the  construction  can  be  used  to  find 
the  supporting  forces  required  for  a  structure  when  loaded. 

(Stud.  I.  C.  E.,  Oct.  1907.) 

9.  If  three  forces  act  on  a  rigid  body,  what  conditions  must  be  fulfilled 
in  order  that  equilibrium  may  be  maintained  ? 

A  uniform  beam  of  timber  weighs  200  pounds.  One  end  rests  on  the 
ground,  the  other  has  a  cord  attached  to  it.  This  cord  is  pulled  till  the 
beam  makes  30°  with  the  horizontal,  and  then  the  cord  makes  60°  with  the 
horizontal  Find  (graphically  or  otherwise)  the  tension  hi  the  cord  and 
the  force  on  the  ground. — Ans.  :  Tension  in  cord  =  100  Ibs ;  reaction  = 
173  2  Ibs.  (Stud.  L  C.  E.,  Feb.  1908.) 


LECTURE  IX.— STUD.  I. C.E.  EXAM.  QUESTIONS. 

1.  Define  horse-power,  and  prove  that  a  locomotive  of  H  horse-power 
can  draw  a  train  of  W  tons  against  a  resistance  of  r  Ibs.  per  ton  at  a  speed 

of   f  iff  miles  an  hour. 

Calculate  the  horse-power  of  a  locomotive  drawing  a  train  of  200  tons 
up  an  incline  of  i  in  200  at  50  miles  per  hour,  taking  the  road  and  air  re- 
sistance at  this  speed  as  28  Ibs.  per  ton.  (Stud.  I.  C.  E.,  Oct.  1905.) 

2.  Define  the  terms  "  work  "  and  "  power."     A  car  weighing  3  tons  is 
running  at  20  miles  per  hour  up  a  uniform  slope  of  i  in  50.     The  frictional 
resistances  are  50  pounds  per  ton.     Find  the  H.-P.,  and  the  work  done  in 
20  minutes. — Ans.,  H.-P.  =  13*8;  work  done  in  20  mins.  =  9, 100,800  ft.  Ibs. 

(Stud.  I.  C.  E.,  Feb.  1908.) 


LECTURE  X. — STUD.  I.C.E.  EXAM.  QUESTIONS. 

i.  What  is  meant  by  coefficient  of  friction  ?  Show  how  the  coefficient 
of  friction  between  wood  and  wood  can  be  determined. 

(Stud.  I.  C.  E.,  Oct.  1904.) 

If  a  machine,  such  as  a  screw-jack,  does  not  overhaul,  show  that  the 
work  done  against  the  friction  must  be  more  than  50  per  cent,  of  the  total 
work  the  man  does.  (Stud.  I.  C.  E.,  Oct.  1904.) 


394  APPENDIX  B. 

2.  Define  the  coefficient  of  limiting  friction,  and  prove  that  it  is  the 
tangent  of  the  slope  of  the  incline  on  which  the  body  is  on  the  point  of 
sliding. 

Determine  geometrically  the  greatest  slope  on  which  a  four-wheeled 
carriage  can  be  held  by  the  brakes,  applied  to  the  hind  pair  of  wheels. 

(Stud.  I.  C.  E.,  Oct.  1905.) 


LECTUHE  XI. — STUD.  I.C.E.  EXAM.  QUESTIONS. 

1.  A  belt-pulley  has  a  diameter  of  5  feet,  and  it  is  delivering  12  H.-P.  to 
a  line  of  shafting.     If  the  pulley  make  120  revolutions  per  minute,  find  the 
force  along  the  tight  part  of  the  belt.  (Stud.  I.  C.  E.,  Oct.  1904.) 

2.  How  can  the  brake  horse-power  of  an  engine  be  determined.     If  the 
diameter  of  the  brake-pulley  is  3  feet,  speed  250  revolutions  per  minute, 
load  24  Ibs.,  and  the  spring-balance  which  takes  up  the  rope  at  the  slack 
end  registers  4  Ibs.,  find  the  horse-power.         (Stud.  I.  C.  E.,  Feb.  1905.) 

3.  Explain  the  meanings  of  the  terms  "  work  "  and  "  energy,"  and  define 
the  practical  unit  of  work  used  by  British  engineers. 

Find  the  brake  horse-power  of  an  engine  making  300  revolutions  per 
minute  if  the  tensions  on  the  tight  and  slack  sides  of  the  brake  strap  are 
72  Ibs.  and  6  Ibs.  respectively,  and  the  brake  wheel  is  5  feet  in  diameter. 

(Stud.  I.  C.  E.,  Feb.  1907.) 

4.  Define  the  term  "  horse-power." 

A  pulley  whose  diameter  is  4  feet  is  making  250  revolutions  per  minute. 
A  belt  is  put  on  the  pulley  and  a  weight  of  50  Ibs.  is  hung  from  one  end. 
A  spring  balance  attached  to  the  other  end  reads  12  Ibs.  Find  the  horse- 
power delivered  to  the  pulley.  If  the  angle  of  embrace  of  the  belt  is  120° 
find  the  extra  force  on  the  bearing  due  to  the  brake. 

(Stud.  I.  0.  E.,  Oct.  1907.) 


LECTURE  XIII.— STUD.  I.C.E.  EXAM.  QUESTIONS. 

P  I.  Describe  any  experiment  made  by  you  to  determine  the  efficiency  of 
a  machine. 

In  a  test  of  a  hand-crane,  with  gear  having  a  velocity  ratio  of  1 50  :  I ,  it 
was  found  that  an  effort  of  25  Ib.  at  the  handles  raised  a  load  of  I  ton. 
Determine  the  efficiency  of  the  machine  for  this  load. 

(Stud.  I.  C.  E.,  Feb.  1907.) 

2.  Explain  the  terms  "  velocity  ratio,"  "  force  ratio,"  and  "  efficiency," 
as  applied  to  a  machine.  A  lifting  crab  has  the  following  relation  between 
the  force  on  the  handle  in  pounds  and  the  weight  lifted  in  tons.  Draw 
curves  connecting  the  force  ratio  and  the  efficiency  with  the  load,  and  find 
the  efficiency  of  the  crab  at  loads  of  5  tons  and  1 5  tons.  The  velocity  ratio 
is  500. 

Force  in  pounds     .          .          .         30          60          90          120 
Load  in  tons  .         .         .         3-3  ^    7-4        12*5       17*5^. 

Ans.  ^Efficiencies  are  0*558  and  O'6987  respectively. 

(Stud.  I.  C.  E.,  Feb.  1908.) 


STUD.  INST.    C.E.  QUESTIONS.  395 

LECTURE  XVII. — STUD.  I.C.E.  EXAM.  QUESTIONS. 

1.  Show  that  the  total  pressure  upon  a  flat  plate  immersed  in  water  is 
proportional  to  the  depth  of  its  centre  of  figure  below  the  surface.     Calculate 
the  pressure  per  square  inch  on  a  horizontal  plate  at  a  depth  of  10  feet, 
assuming  that  a  cubic  foot  of  water  weighs  62*4  Ibs. 

(Stud.  I.  C.  E.,  Oct.  1906.) 

2.  Show  how  to  find  the  total  pressure  on  any  submerged  plane  sur- 
face. 

A  sluice  gate  is  4  feet  wide  and  6  feet  high ;  the  bottom  is  20  feet  below 
the.  surf  ace  of  the  water ;  find  the  total  pressure  on  the  gate. 

[One  cubic  foot  of  water  weighs  62$  Ibs.]    (Stud.  I.  C.  E.,  Oct.  1907.) 


LECTUBE  XVIII. — STUD.  I.C.E.  EXAM.  QUESTIONS. 

1.  At  the  bottom  of  a  barometer  there  is  a  bubble  of  air,  of  which  the 
density  is  0-0013  grams  per  centimetre,  and  diameter  o'3  millimetre.    If 
this  rises  up  the  tube  its  volume  increases.     Explain  why.     At  what  height 
will  its  volume  be  doubled?  (Stud.  I.  C.  E.,  Feb.  1905.) 

2.  Show  how  to  determine  the  total  pressure  and  the  centre  of  pressure 
of  a  vertical  rectangular  plate  immersed  in  water  with  one  edge  at  the 
surface.     Find  the  total  pressure  and  the  depth  of  the  centre  of  pressure 
for  a  plate  12  feet  square  forming  one  side  of  a  tank  full  of  water. 

(Stud.  I.  C.  E.,  Feb.  1906.) 

3.|  Define"  the  term  "specific  gravity,"  and  show  how  to  determine  the 
specific  gravity  of  (i)  a  solid,  (ii)  a  liquid.  (Stud.  I.  C.  E.,  Oct.  1906.) 

4.' Explain  how  you  would  proceed  to  demonstrate  experimentally  that 
a  body  floating  in  water  displaces  a  quantity  of  liquid  equal  to  itself  in 
weight. 

The  displacement  of  a  tug-boat  in  sea-water  was  found  to  be  1560  cubic 
feet.  Calculate  the  weight  of  the  boat,  assuming  that  a  cubic  foot  of  sea- 
water  weighs  64  Ibs.  (Stud.  I.  C.  E.,  Feb.  1907.) 

5.  Define  the  term  "centre  of  pressure"  for  a  surface  immersed  in  a 
fluid. 

A  rectangular  sectioned  water-channel  has  a  board  put  vertically  across 
it,  which  is  held  up  against  the  water-pressure  by  two  horizontal  bars, 
one  at  the  bottom  of  the  channel,  the  other  18  inches  up  the  sides.  Find 
the  depth  of  water  in  the  channel  which  will  just  overset  the  board,  and 
the  pressure  then  existing  on  the  board  if  the  breadth  of  the  channel  is 
5  feet. 

(Stud.  I.  C.  E.,  Feb.  1908.) 


LECTURE  XIX — STUD.  I.C.E.  EXAM.  QUESTIONSI 

1.  Describe  by  aid  of  a  sketch  the  action  of  an  ordinary  suction  pump 
for  raising  water  from  a  well.     Calculate  the  work  required  to  raise  80 
gallons  of  water  through  a  height  of  20  feet,  if  the  pump  has  an  efficiency 
of  64  per  cent.  (Stud.  I.  C.  E.,  Feb.  1907.) 

2.  Sketch  and  describe  a  force  pump  suitable  for  raising  water  from  a 
well. 


396  APPENDIX  B. 

It  is  required  to  raise  water  through  a  total  height  of  80  feet  by  a  pump 
of  6  inches  stroke  with  a  barrel  3  inches  in  diameter.  If  20  strokes  per 
minute  are  made,  find  the  gallons  pumped  per  hour,  and  the  horse-power 
required.  (Stud.  I  C.  E.,  Oct.  1907.) 

3.  Describe,  with  sketches,  some  kind  of    double-acting  force-pump. 

A  pump  is  delivering  water  into  a  boiler  in  which  the  pressure  is  120 
Ibs.  per  square  inch  above  atmospheric  pressure.  Find  the  work  done  in 
foot-pounds  per  pound  of  water  delivered  to  the  boiler.  Find  also  the 
horse-power  of  the  pump  if  it  delivers  2,000  gallons  per  hour  and  its  efficiency 
is  60  per  cent.  Ans.  :  Work  done  per  Ib.  of  water  delivered  =  276  ft.lbs.  ; 
and  H.-P.  of  pump  =  47.  (Stud.  I.  C.  E.,  Feb.  1908.) 


LECTURE  XX. — STUD.  I.C.E.  EXAM.  QUESTIONS. 

i.  The  pressure  of  water  in  a  high-pressure  main  is  700  Ibs.  weight  per 
square  inch.  A  load  of  2  tons  is  to  be  lifted  by  means  of  a  ram  driven 
from  the  main.  Find  the  sectional  area  of  the  ram,  and  the  cubic  feet  of 
water  used  per  H.-P.-  hour.  (Stud.  I.  C.  E.,  Oct.  1904.) 


|  LECTURE  XXI.— STUD.  I.C.E.  EXAM.  QUESTIONS. 

i.  What  is  the  difference  between  acceleration  and  velocity  ? 

Plot  to  scale  the  following  velocities  in  terms  of  the  time,  and  write 
down  the  times  from  the  start,  when  the  acceleration  is  zero ;  also  state 
the  period  during  which  the  acceleration  is  negative.  Find  by  the  method 
of  equidistant  co-ordinates  the  average  velocity. 

Time  in        Velocity  in  Feet  Time  in         Velocity  in  Feet 

Seconds.  per  Second.  Seconds.  per  Second. 


0  5-0 

1  I0'0 

2  13-0 

3  H-S 

4  IS'S 
5-  iS'8 


6  15-8 

7  IS'S 

8  167 

9  18-3 

10  20-3 

11  22-5 
(Stud.  I.  C.  E,  Oct.  1904.) 


2.  A  train  of  120  tons  is  found  to  increase  its  speed  from  20  miles  per 
hour  to  40  miles  per  hour  in  ten  minutes.     If  the  frictional  resistance  is 
2000  Ibs.  weight,  find  the  force  which  must  be  pulling  the  tram — assuming 
it  to  be  a  constant  force.     Find  average  and  final  horse-power. 

"  (Stud.  I.  0.  E.,  Oct.  1904.) 

3.  Water  is  projected  horizontally  from  a  nozzle.     If  the  point  at  which 
it  strikes  the  floor  is  6  feet  below  the  nozzle  and  5  feet  from  the  vertical 
line  drawn  through  the  nozzle,  find  the  velocity  with  which  the  water  is 
projected,     (g  =  32*2  feet  per  sec.  per  sec.) 

(Stud.  I.  C.  E.,  Oct.  1904.) 

4.  A  2-ton  fly-wheel  drops  in  speed  from  100  revolutions  per  minute  to 
90  revolutions  per  minute.    If  the  mean  radius  is  5  feet,  find  the  work  given 
up  by  the  fly-wheel.  (Stud.  I.  C.  E.,  Oct.  1904.) 


STUD.    INST.   C.E.  QUESTIONS.  397 

5.  A  lo-ton  truck,  moving  at  the  rate  of  4  feet  per  second  strikes  an 
8-ton  truck  which  is  standing  at  rest.     If  the  two  move  off  together  after 
the  impact,  find  the  velocity  they  start  with.     If  the  resistance  due  to- 
friction,  &c.,  is  200  Ibs.  weight,  find  how  far  they  will  run  before  coming: 
to  rest.  (Stud.  I.  C.  E.,  Oct.  1904.) 

6.  A  ball  weighing  10  Ibs.  is  making  50  revolutions  per  minute  in  a 
horizontal  circle  of  5  feet  radius.     Find  the  force  in  Ibs.  weight  acting: 
upon  it  towards  th*e  centre  of  the  circle. 

(Stud.  I.  C.  E.,  Oct.  1904.) 

7.  The  distances  passed  over  from  rest  in  I,  2,  3,  4,  5  ...  etc.,  units  of 
time  are  respectively  0-05,  0-3,  0-58,  0*95,  1-4,  2'O,  27,  3-5,  4-4,  5  35,  &S+ 
77,  9-0,  10-4,  i r6,  13-2,  14-9,  16-6,  18-4,  20-3,  22-3,  24-3,  267,  28-8,  31-2,. 
337  units  of  length. 

Plot  the  space-time  curve.  The  unit  of  time  is  T  fo  second,  and  the  unit 
of  length  i  centimetre.  Find  the  acceleration  and  show  how  far  it  is 
constant.  (Stud.  I.  C.  E.,  Feb.  1905.) 

8.  Give  examples  of  how  a  diagram  can  be  used — 

(i)  to  correct  the  observation  or  measurement  of  an  experiment 

(See,  for  example,  Question  i.) 
(ii)  to  find  the  relation  between  two  quantities,  e.g.,  the  effort  and 

load  in  a  machine, 
(iii)  to  find  the  average  value  of  a  quantity,  e.g.,  to  find  the  average 

velocity  of  a  body.  (Stud.  I.  C.  E.,  Feb.  1905.) 

9.  A  shot  is  projected  from  a  gun.     Explain  why — 

(i)  The  momentum  of  the  shot  is  equal  (under  certain  conditions) 

to  the  momentum  of  the  gun. 
(ii)  The  energy  of  the  shot  when  it  leaves  the  gun  is  greater  by  far 

than  the  energy  communicated  to  the  gun. 

Example  :  shot  100  Ibs. ;  gun  i  ton ;  velocity  of  shot  1,200  feet  per 
second.  Find  velocity  of  recoil  of  the  gun,  and  the  energy  of  the  gun. 

(Stud.  I.  C.  E.,  Feb.  1905.) 

10.  In  the  car  of  a  balloon  a  piece  of  iron  is  hung  from  a  spring  balance. 
The  balance  registers  5  Ibs.  when  the  car  is  at  rest.     What  will  it  register 
when  the  car  is  rising  with  an  acceleration  of  2  feet  per  second  per  second  ?• 

(Stud.  I.  C.  E.,  Feb.  1905.) 

11.  Give  some  account  of  the  advance  made  in  dynamics  by  either 
Galileo  or  Newton. 

Show  how  to  find  the  acceleration  towards  the  centre  of  a  circle  of  a 
body  moving  with  uniform  velocity  in  the  circle. 

(Stud.  I.  C.  E.,  Feb.  1905.) 

12.  Continuous  brakes  are  now  capable  of  reducing  the  speed  of  a  train 
of  3f  miles  an  hour  every  second,  and  take  2  seconds  to  be  applied ;  show 
in  a  tabular  form  the  length  of  an  emergency  stop  at  a  speed  of  3f ,  7^, 
1 5,  30,  45,  60  miles  an  hour.  • 

Compare  the  resistance  with  gravity ;  express  the  resisting  force  in  Ibs. 
per  ton ;  calculate  the  coefficient  of  adhesion  of  the  brake-shoe  and  rail 
with  the  wheel,  and  sketch  the  mechanical  arrangement. 

(Stud.  I.  C.  E.,  Oct.  1905.) 

13.  If  W  tons  is  transported  from  rest  to  rest  a  distance  «  feet  in  t  seconds, 
being  accelerated  for  a  distance  s1  and  time  ^  by  a  force  of  P1  tons  up  to 


398  APPENDIX  B. 

velocity  v  feet  per  second,  and  then  brought  to  rest  by  P2  tons  acting'f or 
Jt2  seconds  through  *2  feet,  prove  the  formulas — 

»3?-™- 
oo  T? 


s, 

(iii)V  =  2- 

Supposing  i  in  m  is  the  steepest  incline  a  train  can  crawl  up,  and  I  in  n 
Is  the  steepest  incline  on  which  the  brakes  can  hold  the  train,  prove  that 
the  quickest  run  up  an  incline  of  i  in  p  from  one  station  to  stop  at  the 
next,  a  distance  of  a  feet,  can  be  made  in 


V- 


( 

I  I\ 

m  +  n) 

V  second^ 

fc- 

5)  (5- 

i\  H 

f?     J 

Calculate  for  w  =  50,  n  =  5,  p  =  100,  a  =  5280. 

(Stud.  I.  C.  E.,  Oct.  1905.) 

14.  Determine  the  motion  of  a  circular  hoop  of  radius  a  feet,  whirling  in 
-a  vertical  plane  on  a  round  stick  held  horizontally,  if  released  when  the 
centre  is  moving  with  velocity  V  feet  per  second  at  an  angle  a  with  the 

horizon,  and  prove  that  it  will  make  -  -  revolutions  per  second  in  the 


air. 

V 
Prove   that  the  tension  in  the  hoop  will  be  the  weight  of  a  length — 

feet  of  the  rim.  (Stud.  I.  C.  E.,  Oct.  1905.) 

1 5.  Explain  how  velocities  may  be  compounded.     Determine  the  apparent 
velocity  and  direction  of  rain-drops  failing  vertically  with  a  velocity  of 
20  feet  per  second  with  reference  to  a  bicyclist  moving  at  the  rate  of  12 
miles  an  hour.  (Stud.  I.  C.  E.,  Feb.  1906.) 

1 6.  Show  that  if  a  body  starting  from  rest  and  moving  in  a  straight  line 
is  accelerated  /  feet  per  second  per  second  it  will  describe  a  distance  s  in 
t  seconds  expressed  by  the  formula 

s  =  \ft* 

A  train  starting  from  rest  receives  a  uniform  acceleration  of  0*25  foot  per 
.-a  econd  per  second  for  one  minute.  Calculate  the  distance  travelled. 

(Stud.  I.  C.  E.,  Feb.  1906.) 

17.  Define  potential  and  kinetic  energy. 

Find  the  gam  of  potential  energy  of  a  train  weighing  320  tons  after 
mounting  an  incline  4  miles  long  of  i  in  200,  and  find  its  kinetic  energy 
-when  moving  at  30  miles  an  hour. 

(Stud.  I.  C.  E.,  Feb.  1906.) 

1 8.  Explain  how  forces  are  measured,  and  distinguish  between  the  mass 
of  i  Ib.  and  the  weight  of  i  Ib. 

Determine  what  force  will  be  necessary  to  change  the  velocity  of  a  masa 
-of  400  Ibs.  from  15  to  25  feet  per  second  in  8  seconds. 

(Stud.  I.  C.  E.,  Feb.  1906.) 


STUD.   INST.  C.E.   QUESTIONS. 


399 


19.  Prove  that  the  acceleration  a   of  a  body  moving  with  velocity  v  in  a 
circular  path  of  radius  r  is  expressed  by  the  formula  • 


Calculate  the  force  required  to  constrain  a  locomotive  weighing  50  tons 
to  move  in  a  circle  of  400  feet  radius  when  its  velocity  is  30  miles  an  hour. 

(Stud.  I.  C.  E.,  Feb.  1906.) 

20.  Explain  the  terms  "  moment  of  inertia,"  and  "  radius  of  gyration," 
and  determine  their  values  for  the  case  of  a  circular  disk  of  mass  m  and 
radius  r,  when    rotating  about  an  axis  passing  through  the  centre  and 
perpendicular  to  the  plane  of  the  disk.     Give  numerical  values  when  the 
mass  is  20  Ibs.  and  the  radius  is  2  feet.  (Stud.  I.  C.  E.  ,  Feb.  1906.) 

21.  Define  the  terms  "  velocity  "  and  "  acceleration  "  in  the  case  of  a 
body  moving  in  a  straight  line.     A  motor-car  starting    from    rest    and 
uniformly  accelerated  acquires  in  2  minutes  a  velocity  of  30  miles  an  hour. 
Find  the  acceleration.  (Stud.  I.  C.  E.,  Oct.  1906.) 

22.  A  heavy  ball,  attached  to  a  string  30  inches  long,  is  whirled  round  in 
a  horizontal  circle  with  constant  velocity.     Make  a  diagram  showing  the 
forces  acting  on  the  ball,  and  calculate  the  velocity  when  the  string  is  BO- 
inclined  that  the  ball  moves  in  a  circle  of  24  inches  radius. 

(Stud.  I.  C.  E.,  Oct.  1906.)    | 

23.  Explain  what  is  meant  by  kinetic  energy  and  deduce  an  expression 
for  the  kinetic  energy  of  a  circular  disk,  rotating  about  an  axis  passing 
through  its  centre  of  figure  and  perpendicular  to  the  plane  of  the  disk. 
Calculate  the  kinetic  energy  of  a  disk  having  a  radius  of  2  feet  and  weighing; 
400  Ibs.  when  revolving  at  the  rate  of  240  revolutions  per  minute. 

(Stud.  I.  C.  E.,  Oct.  1906.) 

24.  Show  how  velocities  may  be  compounded.     A  stone  is  dropped  from 
a  balloon  80  feet  above  the  ground  and  moving  horizontally  at  the  rate  of 
12  miles  an  hour.     Determine  the  velocity  and  direction  of  the  stone  when 
it  strikes  the  ground.  (Stud.  I.  C.  E.,  Feb.  1907.) 

25.  Define  the  terms  "  velocity  "  and  "  acceleration."     A  tram-car  start- 
ing from  rest  covers  s  feet  in  t  seconds  in  accordance  with  the  following; 
Table  :— 


t 

s 

' 

2 

3 

4 

5 

6 

7 

8 

9 

10 

4 

II 

21 

34 

5Q 

69 

9i 

no 

144 

175 

Plot  the  space-time  curve  and  from  it  determine  the  velocity  of  the  body 
at  the  end  of  each  second,  and  show  your  results  to  scale  upon  a  time 
base.  Explain  how  to  determine  the  acceleration  from  this  latter  curve,, 
and  determine  its  value  at  the  end  of  the  fifth  second. 

(Stud.  I.  C.  E.,  Feb.  1907.) 

26.  Define  "  angular  velocity  "  and  "  angular  acceleration  "  for  a  body 
revolving  about  a  fixed  axis,  and  deduce  a  formula  for  the  angle^turned 
through  by  a  shaft  starting  from  rest  and  accelerated  uniformly. 

2C 


4OO  APPENDIX  B. 

The  spindle  of  a  dynamo  is  uniformly  accelerated,  and  in  10  seconds 
from  starting  it  is  found  to  be  revolving  at  the  rate  of  600  revolutions  per 
minute.  Find  the  acceleration  and  the  number  of  revolutions  it  has  made. 

(Stud.  I.  C.  E.,  Feb.  1907.) 

27.  Prove  that  the  acceleration  (a)  of  a  body  moving  in  a  circle  of  radius 

•y2. 
r  with  velocity  v  is  expressed  by  the  formula  a  =  —      A  pulley  is  found  to 

be  out  of  balance  to  an  amount  which  may  be  represented  by  a  mass  of 
4  oz.  at  a  radius  of  i  foot.     Determine  the  unbalanced  force  when  the  shaft 
is  "making  1200  revolutions  per  minute. 
'    £  (Stud.  I.  C.  E.,  Feb.  1907.) 

28.  Explain  what  you  understand  by  "  acceleration." 

A  railway  carriage  is  accelerating  at  3  feet  per  sec.  per  sec.  Find  the 
acceleration  possessed  by  a  stone  dropping  from  its  roof.  If  the  carriage 
is  8  feet  high,  find  the  time  taken  to  fall  and  the  distance  the  stone  travels. 

(Stud.  I.  C.  E.,  Oct.  1907.) 

29.  Define  "  angular  acceleration,"  and  show  how  it  is  related  to  linear 
acceleration. 

A  hoop  whose  diameter  is  3  feet,  is  rolling  along  the  ground  and  comes 
to  rest  in  10  seconds,  after  rolling  240  feet.  If  it  is  retarded  uniformly, 
£nd  the  value  of  the  angular  retardation.  (Stud.  I.  C.  E.  Oct.  1907.) 

30.  A  weight  is  suspended  by  a  string  and  rotates  in  a  horizontal  circle. 
Find  the  forces  acting  on  the  weight. 

Such  a  weight  rotates  at  20  revolutions  per  minute  when  the  radius  of 
its  circular  path  is  3  feet.  Find  the  length  of  the  suspending  string. 

(Stud.  I.  C.  E.    Oct.  1907.) 

31.  Define  "  kinetic  energy  "  and  "  potential  energy." 

A  steamer  weighing  2000  tons  is  proceeding  at  20*1  miles  per  hour. 
When  steam  is  cut  off  its  speed  drops  to  19*9  miles  per  hour  after  it  has 
moved  through  200  feet.  Find  the  mean  force  retarding  it.  If  it  speeds 
up  again  to  its  first  speed  in  half  a  minute,  find  approximately  the  work 
done  in  foot-tons,  and  the  horse-power  required. 

'r  £  (Stud.  I.  C.  E.,  Oct.  1907.) 

'  "1  3  2.  ^Explain  how  velocities  can  be  represented  and  combined. 
Y  Two  men,  A  and  B,  are  5  miles  apart,  A  being  due  west  of  B.  They 
start  walking  at  the  same  moment  :  A  walks  to  the  south-east  at  4  miles 
per  hour,  and  B  walks  at  3  miles  per  hour  in  such  a  direction  as  to  meet  A 
on  his  road.  Find  graphically  the  two  possible  times  taken  for  the  two 
men  to  meet.  —  Ans.  :  53  minutes  and  i  hour  46  minutes. 

(Stud.  I.  C.  E.,  Feb.  1908.) 

33.  Define  the  term  acceleration,  and  show  that  if  a  curve  be  plotted 
-connecting  the  velocity  of  a  body  and  the  time,  the  area  under  the  curve 
is  the  distance  traversed,  and  the  slope  of  the  curve  measures  the  accelera- 
tion. 

A  body  starts  from  rest  with  a  uniform  acceleration.  After  the  lapse  of 
a  certain  time  it  is  found  that  in  successive  intervals  of  5  seconds  and  7 
seconds  it  traverses  62^  feet  and  129*-  feet  respectively  :  find  the  accelera- 
tion at  the  above  time  —  Ans.  :  3  feet  per  sec.  per  sec. 

(Stud.  I.  C.  E.,  Feb.  1908.) 

34.  Explain  what  you  understand  by  kinetic  and  potential  energy. 
A  weight  of  420  Ibs.  is  lifted  by  a  force  which  varies  as  follows  : 


o    I    2    3    4    5    6    7 
Force  in  •  pounds      700  610  490  390  380  450  650  800 


STUD.  INST.  C.E.  QUESTIONS.  40 1 

Plot  a  curve  connecting  the  force  and  height,  and  hence  find  ^"poten- 
tial and  kinetic  energy  of  the  body,  and  the  work  done  by  the  force  when 
the  body  is  6£  feet  from  the  ground. — Ans.  :  EP  =  2730  ft-lb.  :  EK  = 
2359  ft.-lb.,  and  work  done  by  the  force  =  5089  ft.-lb. 

(Stud.  I.  C.  E.,  Feb.  1908.) 

35.  Explain  how  energy  is  stored  in  a  fly-wheel,  and  obtain  an  expression 
for  this  energy.     If  such  a  wheel  stores   1,000  foot-lb.  when  rotating  at 
1  revolution  per  second,  find  the  work  that  must  be  done  to  change  its 
speed   from   10  revolutions   per   second  to    20   revolutions   per   second. 
Ans. :  EK  =  *Iw2.   Work  done  in  changing  the  speed  =  300,000  foot-lb. 

(Stud.  I.  C.  E.,  Feb.  1908.) 

36.  Explain  what  you  understand  by  the  term  "  centripetal  force." 

A  weight  of  20  pounds  is  hung  by  a  string  10  feet  long.  It  is  pulled 
to  one  side  so  as  to  be  6  feet  horizontally  away  from  the  vertical  If  the 
weight  is  then  let  go,  find  from  the  energy  equation  the  velocity  of  the 
weight  at  the  moment  it  passes  through  the  lowest  point,  and  deduce  the 
total  tension  in  the  string  at  that  moment.  Ans.:  V  =  ii'32  feet  per 
sec.  ;  F  =  8  Ib.  (Stud.  I.  C.  E.,  Feb.  1908.) 


LECTURE  XXII. — STUD.  I. C.E.  EXAM.  QUESTIONS. 

1.  Prove  that  the  increase  of  pressure  per  foot  vertically  downwards  in 
a  liquid  of  specific  gravity  s  is  0^433  x  s  Ibs.  per  square  inch. 

Mining  in  ground  of  uniform  density  at  a  depth  of  h  feet,  determine  the 
percentage  of  coal  that  can  be  won,  leaving  sufficient  as  pillars  for  the 
support  of  the  roof,  supposing  the  coal  to  crush' under  its  own  weight  in  a 
column  k  feet  high.  (Stud.  I.  C.  E.,  Oct.  1905.) 

2.  Investigate  the  mechanical  advantage  of  the   smooth  screw,  and 
explain  generally  how  the  wind  drives  a  windmill,  and  a  screw  propeller 
propels  a  steamer. 

Prove  that  a  platelayer  who  can  apply  a  force  of  28  Ibs.  will  be  apt  to 
break  the  screw-bolts  if  provided  with  a  lever  more  than  3  feet  long ;  the 
screw  having  8  threads  to  the  inch,  and  the  breaking  tension  of  the  bolt 
being  30  tons  per  square  inch.  (Stud.  I.  C.  E.,  Oct.  1905.) 

3.  Explain  the  meanings  of  the  terms  "  stress,"  "  strain  "  and  "  modulus 
of  elasticity  "  with  reference  to  a  bar  in  tension.     A  tie-bar,  3*5  square 
inches  in  section  and  16  feet  long,  stretches  0*05  inch  under  a  load  of 
28,000  Ibs.     Find  the  values  of  the  stress,  strain,  and  modulus  of  elasticity. 

(Stud.  I.  C.  E.,  Oct.  1906.) 


LECTURE  XX1IL — STUD.  I. C.E.  EXAM.  QUESTIONS. 

i.  A  machine  is  operated  by  a  shaft  making  N  revolutions  per  minute 
and  transmitting  a  twisting  moment  T.  Deduce  an  expression  for  the 
horse-power  delivered  to  the  machine  and  calculate  its  numerical  value  if 
the  shaft  makes  1 10  revolutions  per  minute,  and  the  twisting  moment  is 
2000,  the  units  being  pounds  and  feet.  (Stud.  I.  C.  E.,  Feb.  1907.) 


4O2  NOTES    AND    QUESTIONS. 


B.    OF    E.    QUESTIONS,    MAY    igiO.  403 

May,  1910,  Examination  on  Subject  VII. 
APPLIED  MECHANICS. 

STAGE  I. 
GENERAL  INSTRUCTIONS.— See  APPENDIX  A. 

You  must  not  attempt  more  than  EIGHT  questions  ;  EITHER  No.  i  OR  No.  22 
:  must  be  one  of  these  eight,  but  not  both.     The  remaining  seven  questions  may 
be  selected  from  Nos.  2  to  21.      The  questions  in  Series  A  are  framed  to  be 
more  particularly  suitable  for  the  Building  Trades,  and  those  in  Series  B 
/or  Mechanical  Engineers. 

SERIES  A. 

1.  Describe,  with  the  help  of  good  sketches,  onlyonz  of  the  following,  (a), 
(6),(c),or(d): 

(a)  A  mortar-mixing  machine. 

(&)  The  method  of  securing  the  cutting  chisels  into  the  cutter  blocks 
of  a  wood-planing  machine. 

(c)  Any  form  of  friction  clutch  suitable  for  use  with  a  speed  cone  or 

reversing  pulleys. 

(d)  Any  form  of  vernier  calliper  suitable  for  measuring  the  dimensions 

of  a  test-bar  to  the  nearest  thousandth  of  an  inch. 

(B.  of  E.,  1910.) 

2.  Describe,  with  sketches,  an  apparatus  to  verify  the  rule  for  finding  the 
compressive  and  tensile  forces  in  the  jib  and  tie  of  a  crane.     Do  the  ex- 
perimental results  exactly  agree  with  the  rule,  and  if  not,  what  is  the 
probable  reason  ?  (B.  of  E.,  1910.) 

3.  Answer  only  one  of  the  following,  (a),  (6),  or  (c) : 

(a)  Two  of  the  tests  specified  in  order  to  determine  the  quality  of 
Portland  cement  are  the  determination  of  the  tensile  strength  of 
(i)  a  mortar  of  neat  cement,  (ii)  a  mortar  with  sand.  Describe 
carefully  how  the  specimens  would  be  made  and  tested. 

(&)  You  are  supplied  with  a  length  of  steel-wire  one-eighth  of  an  inch 
in  diameter.  You  are  asked  to  find  (i)  Young's  Modulus  for  the 
material,  (ii)  the  limit  of  elasticity,  (iii)  the  breaking  stress. 
Explain  how  you  would  carry  out  the  test.  (See  my  Adv.  Vol.  IT.) 

(c)  You  wish  to  know  the  strength  and  stiffness  of  a  large  timber 
beam.  It  is  to  be  built  in  at  the  ends  ;  to  be  about  20  feet  long 
between  supports ;  to  be,  say,  10  inches  broad  and  12  inches 
deep,  and  it  is  to  be  loaded  uniformly  all  over.  You  therefor 
test  a  small  beam  of  the  same  kind  of  timber ;  describe  exactly 
how  you  would  make  the  test,  and  how  would  you  use  your 
results  ?  (See  my  Adv.  Vol.  II.)  (B.  of  E.,  1910.) 


4°4 


APPENDIX    C. 


4.  A  wooden  beam  is  built  into  a  wall  at  one  end.     Eight  feet  from  the 
wall  there  is  a  hook  hi  the  beam,  and  from  this  hook  is  suspended  a  weight 
of  i  ton.     What  is  the  bending  moment  in  the  beam  (i)  at  the  wall,  (ii)  at 
3  feet  from  the  wall  ?     Describe  the  nature  of  the  compressive  and  tensile 
stresses  throughout  any  section.  (B.  of  E.,  1910.) 

5.  Let  the  length  of  a  strut  divided  by  the  diameter  of  its  section  be 
called  x.     W  is  the  maximum  load  carried.     Tests  were  made  on  a  set  of 
cast-iron  struts  all  of  the  same  section  but  of  different  lengths,  with  the 
following  results : 


X 

10 

15 

20 

25 

30 

W 

64,000 

53,500 

44,800 

33,700 

24,IOO 

Plot  a  curve  showing  how  the  strength  depends  upon  x.  What  is  the 
maximum  load  when  the  length  is  18  times  the  diameter  ?  (See  my  Adv. 
Vol.  II.)  (B.  of  E.,  1910.) 

6.  There  is  a  triangular  roof-truss  ABC ;  AC  is  horizontal  and  10  feet 
long.    The  angle  EC  A  is  25°  and  BAG  is  55°  ;   there  is  a  vertical  load  of 

5  tons  at  B.  What  are  the  compressive  forces  in  BA  and  BC.  What 
are  the  vertical  supporting  forces  at  A  and  C.  Find  these  answers 
any  way  you  please.  (B.  of  E.,  1910.) 

7.  A  man's  hand  on  the  handle  of  a  crane  moves  120  feet  when  the 
weight  is  lifted  i  foot ;    35  per  cent,  of  the  total  energy  given  by  the  man 
is  wasted  in  friction.     A  load  of  1.5  tons  is  being  lifted.     What  force  is 
being  exerted  by  the  hand  ?  (B.  of  E.,  1910.) 

8.  Roughly,  what  is  the  weight  of  a  cubic  foot  of  brickwork  ?     There  is 
a  brick  building  80  feet  long  and  50  feet  wide.     The  foundations  carry 
the  following  weight : — First,  the  volume  of  brickwork  is  24,000  cubic  feet, 
the  roof  and  floors  weigh  altogether  200  Ib.  per  horizontal  square  foot  of 
the  area ;    the  machinery  weighs  altogether  1 50  tons.     What  is  the  total 
weight  to  be  carried  ?     What  is  the  breadth  of  the  foundation  wall  at  the 
footings,  if  the  load  there  is  not  to  exceed  i£  tons  per  square  foot  ? 

(B.  of  E.,  1910.) 

9.  It  is  necessary  to  keep  the  "  surface  level "  of  water  in  a  shaft  at  a 
depth  of  30  feet.     When  left  to  itself  the  level  rises  4  feet  in  i  minute. 
The  shaft  is  circular,  6  feet  in  diameter.     What  is  the  weight  of  water 
entering  per  minute  ?     This  water  is  lifted  by  a  pump  whose  efficiency 
is  0.45.     What  horse-power  must  be  supplied  to  the  pump  ?  . 

(B.  of  E.,  1910.) 

10.  Answer  only  one  of  the  following  questions,  (a)  or  (6) : 

(a)  Describe  briefly,  sketches  are  hardly  needed,  how  Portland  cement 
is  manufactured.  Give  a  reason  for  each  part  of  the  process. 
What  is  your  notion  of  what  occurs  (i)  when  cement  sets,  (ii)  when 
it  slowly  hardens  as  it  gets  older  ? 

(6)  Describe  briefly,  sketches  are  hardly  needed,  how  any  kind  of 
steel  used  for  girders  is  manufactured.  Give  a  reason  for  each 
part  of  the  process.  (B.  of  E.,  1910.) 


B.    OF    E.    QUESTIONS,    MAY    19 IO. 


405 


11.  You  are  given  a  4-ton  screw-jack.     How  would  you  experimentally 
determine  its  efficiency  under  various  loads  ?     What  sort  of  results  would 
you  expect  to  obtain  ?  (B.  of  E.,  1910.) 

SERIES  B. 

12.  The  depth  of  water  outside  the  gate  of  a  dry  dock  is  25  feet.     What 
is  the  total  water  pressure  on  the  gate  if  the  width  of  the  gate  is  40  feet  ? 
The  weight  of  i  cubic  foot  of  salt  water  is  64  Ib.  (B.  of  E.,  1910.) 

13.  In  a  hydraulic  cylinder,  i  square  foot  in  cross-section,  the  piston 
moves  through  a  distance  of  i  foot.      The  pressure  of  the  water  is  1400  Ib. 
per  square  inch.     What  work  is  done  on  the  piston  ?      What  is  the  work 
done  per  gallon  of  water  used  ?  (B.  of  £.,1910.) 

14.  The  pull  in  the  draw-bar  between  locomotive  and  train  is  13  Ib.  per 
ton  when  on  the  level ;  the  train  weighs  200  tons,  what  is  the  total  force  ? 
If  the  tram  is  being  pulled  up  an  incline  of  i  in  80  (a  vertical  rise  of  i  foot 
in  a  rail  distance  of  80  feet),  what  is  the  additional  pull  required  ?     What 
is  the  total  pull  ?   The  speed  is  2,500  feet  per  minute.     What  is  the  horse- 
power exercised  in  drawing  the  train  ?  (B.  of  E.,  1910.) 

15.  As  a  particle  of  water  flows  without  friction,  its  height  h  feet  above- 
datum  level,  its  pressure  p  (in  Ib.  per  sq.  ft.),  and  its  speed  v  (in  feet  per 
second),  may  all  alter,  but  the  sum 


20         W       I 

remains  constant.  Here  g  =  32.2  and  w  =  62.3.  The  particle  flows  from 
a  place  A  where  v  =  O,  p  =  O,  and  h  =  80,  to  a  place  B  where  p  =  O,. 
and  h  =  40 ;  what  is  the  value  of  v  at  the  place  B.  (B.  of  E.,  1910.) 

16.  The  motion  of  a  body  of  3,220  Ib.  is  opposed  by  a  constant  frictional 
resistance  of  2,000  Ib.  It  starts  from  rest  under  the  action  of  a  varying 
force  F  Ib.  whose  value  is  here  given  at  the  instants  at  which  the  body 
has  passed  x  feet  from  rest : 


F 

S.HO 

2,870 

2,630 

2,700  , 

X 

0 

5 

IO 

15 

As  more  work  is  being  done  upon  the  body  than  what  is  being  wasted 
in  friction,  what  is  the  speed  of  the  body  when  it  has  moved  1 5  feet  from 
rest  ?  (B.  of  E.,  1910.) 

17.  An  electric  motor  is  employed  for  lifting  purposes.  In  lifting  80  tons 
of  grain  100  feet  high  it  is  found  that  20  Board  of  Trade  units  of  Energy 
have  to  be  paid  for.  A  Board  of  Trade  unit  is  i  kilowatt  for  i  hour,  and 
a  horse-power  is  0.746  kilowatt.  The  cost  is  twopence  per  unit.  What 
is  the  cost  of  i  horse-power  hour  usefully  done  ?  What  is  the  ratio  of  useful 
work  to  the  electric  energy  supplied  ?  (See  Appendix  D  of  this  book.)  t^, 

(B.  of  E.,  1901.) 


406  APPENDIX    C. 

1  8.  The  radial  speed  of  the  water  in  the  wheel  of  a  centrifugal  pump  is 
6  feet  per  second.  The  vanes  are  directed  backwards  at  an  angle  of 
35  degrees  to  the  rim.  What  is  the  real  velocity  of  the  water  relatively  to 
the  vanes  ?  What  is  the  component  of  this  which  is  tangential  to  the  rim  ? 

(B.  of  E.,  1910.) 

ip.'A  projectile  leaves  the  muzzle  of  a  gun  at  2,000  feet  per  second,  its 
path  being  inclined  at  20°  upwards.  What  are  the  horizontal  and  vertical 
components  of  its  velocity  ?  In  3  seconds  how  far  has  it  travelled  hori- 
zontally ?  What  is  its  vertical  height  above  the  gun  ?  Neglect  resistance 
of  the  atmosphere.  (B.  of  E.,  1910.) 

2O.*A  block  of  cast-iron,  3  inches  by  4  inches  by  3  inches,  is  fastened  to 
the  arm  of  a  wheel  at  the  distance  of  3  feet  from  the  axis.  The  wheel 
makes  2,000  revolutions  per  minute.  What  is  the  force  tending  to 
fracture  the  fastening  ?  One  cubic  inch  of  cast-iron  weighs  0.26  Ib. 

(B.  of  E.,  1910.) 


.,f  an  drives  air  vertically  downward  through  an  opening,  8  feet  in 
diameter,  with  a  velocity  of  30  feet  per  second  ?  The  air  weighs  0.08  Ib. 
per  cubic  foot. 

What  weight  of  air  is  driven  downward  per  second  ?    What  is  its  mo- 
mentum ?  (B.  of  E.,  1910.) 

22.  Describe,  with  sketches,  only  one  of  the  following,  (a),  (b),  (c), 
or  (d)  : 

(a)  The  shaft  bearing  of  any  modern  fast  running  machine  such  as  a 

water  turbine  or  dynamo  machine. 

(6)  An  hydraulic  appliance  in  use  by  hydraulic  companies  or  their 
customers,  such  as  an  accumulator,  or  a  motor,  or  a  force  pump. 

(c)  Any  form  of  sensitive  drilling  machine  of  the  "  pillar  "  type.     What 

are  the  advantages  of  this  type  of  machine  for  small  accurate  work  ? 

(d)  Any  form  of  quick  return  motion  suitable  for  use  on  a  shaping 

machine.     Show  how  the  stroke  of  the  tool  may  be  varied. 

(B.  of  E.,  1910.) 


STUD.   INST.   C.E,    QUESTIONS,   OCT.    1909, 


407 


Appendix  C. 

The  Institution  of  Civil  Engineers'  Examinations 
for  Admission  of  Students,  October  1909. 

(vii.)  ELEMENTARY  MECHANICS. 


Not  more  than  EIGHT  questions  to  be  attempted  by  any  Candidate. 

1.  Define  the  terms  relative  velocity  and  absolute  velocity. 

A  bicycle  has  28-inch  wheels,  and  is  being  ridden  at  20  miles  an  hour. 
Find  the  velocity  of  a  point  on  the  rim  14  inches  from  the  ground,  (i) 
relative  to  the  rider,  (2)  relative  to  the  ground.  (Stud.  I.  C.  E.,  Oct.  1909.) 

2.  Explain  the  term  acceleration,   and  show  how  it  is  to  be  measured 
when  non-uniform.     The  velocity  of  a  body  at  given  times  is  as  given  in 
the  schedule  below.     Draw  the  curve  connecting  the  two  quantities,  and 
find  the  space  the  body  has  moved. 

(Stud.  I.  C.  E.,  Oct.  1909.) 


Time  in  seconds.     . 

o 
o 

i 

2 

3 

4 

5 

6 

7 

8 

Velocity  in  feet  per'l 
second  .     .    .     .J 

0'95 

3'8o 

5-00 

4*60 

3-15 

1-65 

075 

o 

3.  Give  the  equations  for  the  motion  of  a  body  which  has  a  uniform 
acceleration. 

An  airship  is  travelling  in  a  horizontal  line  at  30  miles  per  hour  towards 
an  object  on  the  ground  on  which  it  is  desired  to  drop  a  shell.  The  ship 
is  1600  feet  above  the  ground.  Find  where  it  must  let  the  shell  go  in  order 
to  hit  the  object.  (Stud.  I.  C.  E.,  Oct.  1909.) 

4.  Show  how  to  find  the  acceleration  produced  by  a  force  acting  on  a 
body. 

A  tramcar  whose  weight  is  14  tons  is  being  pulled  horizontally  along  a 
track  by  a  force  of  2000  Ibs.  ;  the  track  friction  is  20  Ibs.  per  ton.  Find 
the  acceleration  produced  in  miles  per  hour  per  second. 

(Stud.  I.  C.  E.,  Oct.  1909.) 

5.  Enunciate  the  conditions  that  must  be  fulfilled  in  order  that  three 
non-parallel  forces  may  be  in  equilibrium. 

A  uniform  plank  AB  is  pivoted  smoothly  at  the  end  A,  and  has  a  rope 
attached  to  the  end  B.  The  plank  is  10  feet  from  A  to  B,  and  weight 
100  Ibs.  The  rope  is  8  feet  long  and  is  fixed  to  a  point  C,  such  that  C  is 


408  APPENDIX    C. 

at  the  same  height  as  A  and  the  angle  ABC  is  a  right  angle.     Determine 
graphically  the  tension  in  the  rope  and  the  pressure  on  the  pivot. 

(Stud.  I.  C.  E.,  Oct.  1909.) 

6.  Show  that  a  body  which  is  moving  with  uniform  angular  velocity  w 
in  a  circle  of  radius  r  has  an  acceleration  u?2>.  towards  the  centre  of  the 
circle. 

A  flywheel  has  an  internal  trough  turned  on  it  to  contain  cooling  water. 
Find  the  least  possible  number  of  revolutions  per  minute  that  will  permit 
the  retention  of  the  water  if  the  diameter  of  the  trough  is  8  feet. 

(Stud.  I.  C.  E..  Oct.  1909.) 

7.  What  is  the  moment  of  a  force  ?     A  timber  balk  is  42  feet  long  :  a 
small  cross-bar  is  placed  underneath,  and  2  feet  from  one  end  ;  the  force 
required  to  lift  the  other  from  the  ground  is  600  Ibs. :  the  cross-bar  is 
moved  to  6  feet  from  the  end,  and  the  force  is  then  500  Ibs. ;  find  the 
weight  of  the  balk,  and  the  distance  of  its  centre  of  gravity  from  the  end. 

(Stud.  I.  C.  E.,  Oct.  1909.) 

8.  Give  the  condition  that  a  body  may  stand  in  equilibrium  on  a  plane. 
A  triangle  ABC  is  cut  out  of  a  thick  board  :  the  side  AB  is  10  inches,  and 
the  angle  ABC  is  30°.     Find  the  other  sides  if  the  triangle  is  just  stable 
when  standing  on  the  side  AB  on  a  horizontal  plane. 

(Stud.  I.  C.  E.,  Oct.  1909.) 

9.  Define  the  terms  work,  power,  horse-power. 

A  car  weighs  i^  ton  and  its  engine  is  working  at  a  constant  horse-power, 
It  is  running  on  a  road  for  which  the  total  frictional  resistance  is  50  Ibs 
per  ton.  A  five-mile  run  takes  20  minutes,  and  in  the  5  miles  the  total  rise 
is  400  feet.  The  car's  speed  is  the  same  at  each  end  of  the  run.  Find  its 
HP.  (Stud.  I.  C.  E.,  Oct.  1909.) 

10.  What  is  the  meaning  of  the  efficiency  of  a  machine  ? 

A  screwjack  has  a  screw  whose  pitch  is  £  inch,  and  the  force  is  applied 
by  a  lever  15  inches  long.  It  is  found  that  5  tons  is  lifted  by  a  force  of 
56  Ibs.  applied  at  right  angles  to  the  lever.  Find  the  efficiency  of  the 
jack.  (Stud.  I.  C.  E.,  Oct.  1909.) 

1 1 .  What  is  the  specific  gravity  of  a  body  ? 

A  thin  metal  tube  is  32  inches  long,  and  one  square  inch  in  sectional 
area.  The  bottom  inch  is  filled  with  a  metal.  When  the  tube  is  put  in  a 
vessel  of  water,  it  floats  with  2  inches  out  of  the  water.  Find  the  specific 
gravity  of  the  metal  if  the  tube  weighs  0*6  Ib. 

(Stud.  I.  C.  E.,  Oct.  1909.) 

12.  A  plane  surface  is  immersed  in  a  fluid :  find  an  expression  for  the 
pressure  exerted  on  the  same. 

A  cube  of  3  feet  side  is  placed  in  water  4  feet  deep,  with  one  face  of  the 
cube  horizontal.  Find  the  pressure  on  each  side. 

(Stud.  I.  C.  E.,  Oct.  1909.) 


STUD.    INST.    C.E.    QUESTIONS,    FEB.     1910.  409 


Appendix  C. 

The  Institution  of  Civil  Engineers'  Examinations 
for  Admission  of  Students,  February,  1910. 

ELEMENTARY  MECHANICS. 


Not  mare  than  EIGHT  questions  to  be  attempted  by  any  Candidate. 

(The  weight  of  a  cubic  foot  of  water  may  be  taken  as  62%  Ibs.  and  g  as  32.) 

1.  Explain  how  two  velocities  can  be  combined. 

A  cyclist  is  riding  along  a  straight  road  which  runs  at  30°  to  a  straight 
piece  of  railway  line.  He  sees  an  engine  on  that  line  when  he  is  looking 
in  a  direction  making  a  constant  angle  of  45°  with  his  direction  of  motion. 
He  is  travelling  at  12  miles  an  hour  :  find  the  velocity  of  the  engine. 

2.  Establish  the  equation  for  the  space  traversed  by  a  body  moving 
with  constant  acceleration. 

A  balloon  has  a  total  weight  of  I  ton  and  is  at  rest  900  feet  above  the 
ground.  It  suddenly  lets  fall  i  cwt.  of  ballast :  neglecting  friction,  find 
how  high  it  will  have  risen  when  the  ballast  reaches  the  ground. 

3.  A  curve  is  drawn  connecting  time  and  velocity  of  a  moving  body : 
show  how  the  space  traversed  in  a  given  interval  can  be  found  from  it. 

Such  a  curve  is  drawn  with  a  horizontal  scale  of  I  inch  =  I  second  and 
a  vertical  scale  of  i  inch  =  i  foot  per  second.  It  is  a  semi-circle  of 
3  inches  radius  with  its  centre  at  the  3  seconds  point  on  the  horizontal 
line.  Find  the  total  space  traversed,  and  the  acceleration  at  the  end  of 
the  first  second. 

4.  Give  the  theorem  known  as  the  triangle  of  forces. 

A  uniform  heavy  bar  is  5  feet  long  and  weighs  20  Ibs.  Two  strings 
3  and  4  feet  long  are  attached  to  the  ends,  the  other  extremities  of  the 
strings  being  fixed  to  a  peg.  Draw  the  position  when  the  bar  hangs- 
freely  ;  determine  the  triangle  of  forces  and  the  tensions  in  the  strings. 

5.  If  a  body  he  moving  in  a  circular  path  of  radius  r  with  a  velocity  v,. 

t'2 

show  that  there  must  be  a  constraining  force  of  the  amount  —  acting: 

along  the  radius. 

A  cyclist  and  his  machine  together  weigh  12  stone;  he  is  turning  a 
corner  with  a  velocity  of  15  miles  per  hour  and  the  radius  is  80  feet. 
Find  the  force  acting. 

6.  What  is  the  condition  that  must  be  fulfilled  in  order  that  a  body- 
acted  on  by  parallel  forces  shall  be  at  rest  ? 


410  APPENDIX    0. 

Two  heavy  uniform  rods  are  fixed  at  an  angle  and  pivoted  at  the 
junction  point.  The  shorter  rod  is  6  feet  long  and  weighs  10  Ibs.  :  the 
longer  rod  is  8  feet  long  and  weighs  15  Ibs.  When  suspended  by  the 
pivot  the  shorter  rod  is  horizontal.  Find  the  angle  at  which  the  rods  are 
fixed  together. 

7.  Explain  the  meaning  of  the  terms  worlt  and  power,  and  give  the 
usual  units  in  which  they  are  measured. 

A  locomotive  is  steadily  pulling  a  train  weighing  500  tons  (total)  at 
25  milts  an  hour  up  a  slope  of  I  in  200.  The  frictional  resistance  is 
10  Ibs.  a  ton.  Find  the  HP.  exerted  in  traction. 

8.  Explain  the  meaning  of  the  term  momentum. 

A  vessel  of  2000  tons  is  starting  to  tow  one  of  1000  tons.  The  instant 
before  the  rope  becomes  taut,  the  vessels  are  moving  on  the  same  line 
with  respective  velocities  of  7  and  3  knots.  Find  the  common  velocity  at 
the  moment  the  rope  becomes  taut. 

9.  What  do  you  understand  by  the  velocity  ratio  and  the  efficiency  of  a 
machine  ? 

A  hand  crane  has  a  handle  radius  of  i£  foot.  It  is  found  that  forty-two 
turns  are  required  to  raise  the  load  one  foot,  and  that  a  tangential  force 
of  37  pounds  will  just  raise  5  tons.  Find  the  efficiency  of  the  crane. 

10.  Define  the  centre  of  gravity  of  a  body,  and  show  how  you  would 
experimentally  find  the  centre  of  gravity  of  a  lamina, 

A  triangle  has  sides  which  are  respectively  5,  4  and  3  inches.  To  each 
a  square  is  attached  externally,  and  in  the  plane  of  the  triangle,  the 
squares  being  all  cut  from  the  same  sheet.  Draw  the  figure  to  scale  and 
mark  on  it  the  centre  of  gravity  of  the  area  formed  by  the  three  squares. 

11.  Define  the  term  specific  gravity  and  show  how  you  would  measure 
it  for  a  body. 

A  yard  of  wire  weighs  70  grams  in  air  and  61  grams  in  water.  Find 
the  specific  gravity  and  the  section  of  the  wire  [yard  =  91  \  centimetres, 
i  cubic  centimetre  of  water  weighs  I  gram.] 

12.  How  would  you  find  the  pressure  exerted  by  a  head  of  water  on  a 
dam? 

A  dam  is  50  feet  high  and  has  a  vertical  internal  face.  Find  the  total 
pressure  per  foot  run  and  the  position  at  which  it  may  be  considered  to 
act. 


APPENDIX    D. 

THE    CENTIMETRE,    GRAMME,   SECOND,    OR    C.G.S.    SYSTEM    OF 
UNITS  OF  MEASUREMENT  AND  THEIR  DEFINITIONS.* 

I.  Fundamental  Units. — The    C.G.S.   and  the  practical    electrical 
units  are  derived  from  the  following  mechanical  units. 

The  Centimetre  as  a  unit  of  length  ;  the  Gramme  as  a  unit  of  mass  ;  and 
the  second  as  a  unit  of  time. 

The  Centimetre  (cm)  is  equal  to  0*3937  inch  in  length,  and  nominally 
represents  one  thousand-millionth  part,  or  I0001OOOOQ  of  a  quadrant  of  the 
earth. 

The  Gramme  (gm)  is  equal  to  15  '432  grains,  and  represents  the  mass  of  a 
cubic  centimetre  of  water  at  4°  C.  Also,  I  Ib.  of  16  oz.  is  equal  to  45  3  '6 
grammes.  Mass  (M)  is  the  quantity  of  matter  in  a  body. 

The  second  (s)  is  the  time  of  one  swing  of  a  pendulum  making 
86,i64'09  swings  in  a  sidereal  day,  or  the  1/86,400  part  of  a  mean  solar  day. 

II.  Derived  Mechanical  Units. — 

Area  (A  or  cm2). — The  unit  of  area  is  the  square  centimetre. 

Volume  (V  or  cms). — The  unit  of  volume  is  the  cubic  centimetre. 

Velocity  (v  or  cm/s)  is  rate  of  change  of  position.  It  involves  the  idea 
of  direction  as  well  as  that  of  magnitude.  Velocity  is  uniform  when  equal 
distances  are  traversed  in  equal  intervals  of  time.  The  unit  of  velocity  is 
the  velocity  of  a  body  which  moves  through  unit  distance  in  unit  time,  or 
the  velocity  of  one  centimetre  per  second. 

Momentum  (Mv,  or  gm  x  cm/s)  is  the  quantity  of  motion  in  a  body,  and 
is  measured  by  mass  x  velocity. 

Accleration  (a  or  cm/s2)  is  the  rate  of  change  of  velocity,  whether 
that  change  takes  place  in  the  direction  of  motion  or  not.  The  unit  of 
acceleration  is  the  acceleration  of  a  body  which  undergoes  unit  change  of 
velocity  in  unit  time,  or  an  acceleration  of  one  centimetre-per-second  per 
second.  The  acceleration  due  to  gravity  is  considerably  greater  than  this, 
for  the  change  of  velocity  imparted  by  gravity  to  falling  bodies  in  one 
second  is  about  981  centimetres  per  second  (or  about  32*2  feet  per  second). 
The  value  differs  slightly  in  different  latitudes.  At  Greenwich  the  value 
of  the  acceleration  due  to  gravity  is  g  =  981*17  ;  at  the  Equator  «/  =  978*1, 
and  at  the  North  Pole  g  =  983*1. 

*  The  Author  is  indebted  to  his  Publishers,  Charles  Griffin  and  Co.,  for 
liberty  to  abstract  the  following  pages  on  this  subject  from  the  latest 
edition  of  Munro  and  Jamieson's  "  Pocket-book  of  Electrical  Rules  and 
Tables  for  Electricians  and  Engineers,"  to  which  the  student  is  referred 
for  further  values  and  definitions,  and  values  of  Practical  Electrical  Unit* 
of  Measurement  and  Testing  Rules,  &c. — A.J. 


412  APPENDIX    D. 

Force  (F  or  /)  is  that  which  tends  to  alter  a  body's  natural  state  of  rest 
or  of  uniform  motion  in  a  straight  line. 

Force  is  measured  by  the  rate  of  change  of  momentum  which  it  produces, 
or  mass  x  acceleration. 

The  Unit  of  Force,  or  Dyne,  is  that  force  which,  acting  for  one  second  on 
a  mass  of  one  gramme,  gives  to  it  a  velocity  of  one  centimetre  per  second. 
The  force  with  which  the  earth  attracts  any  mass  is  usually  called  the 
*'  weight  "  of  that  mass,  and  its  value  obviously  differs  at  different  points 
of  the  earth's  surface.  The  force  with  which  a  body  gravitates  —  i.e.  its 
weight  (in  dynes),  is  found  by  multiplying  its  mass  (in  grammes)  by  the 
value  of  g  at  the  particular  place  where  the  force  is  exerted. 

Work  is  the  product  of  a  force  and  the  distance  through  which  its  acts. 
The  unit  of  work  is  the  work  done  in  overcoming  unit  force  through  unit 
distance,  i.e.  in  pushing  a  body  through  a  distance  of  one  centimetre 
against  a  force  of  one  dyne.  It  is  called  the  Erg.  Since  the  "  weight  " 
of  i  gramme  is  I  x  981,  or  981  dynes,  the  work  of  raising  one  gramme 
through  the  height  of  one  centimetre  against  the  force  of  gravity  is  981 
«rgs  or  g  ergs.  One  kilogramme-metre  =  100,000  (g)  ergs.  One  foot-pound 
as  13,825  (g)  ergs  =  1-356  x  10?  ergs. 

Energy  is  that  property  which,  possessed  by  a  body,  gives  it  the  capa- 
bility of  doing  work.  Kinetic  energy  is  the  work  a  body  can  do  in  virtue 
of  its  motion.  Potential  energy  is  the  work  a  body  can  do  in  virtue  of  its 
position.  The  unit  of  energy  is  the  Erg. 

Power  or  Activity  is  the  rate  of  working.  The  unit  is  called  the  Watt(Wp) 
=  io7  ergs  per  second,  or  the  work  done  at  the  rate  of  one  Joule  (J)  per 
second. 

One  Horse-power  (H.P.)  =  33,000  ft.-lbs.  per  minute  =  550  ft.-lbs.  per 
second.  But  as  seen  above  under  Work,  i  ft.  Ib.  =  i'356  x  10?  ergs,  and 
under  Power,  i  Watt  —  10?  ergs  per  second. 

Hence,  a  Horse-power  =  550  x  1*356  x  10?  ergs  per  sec.  =  746  Watts. 
If  E  =  volts,  0  =  amperes,  and  R  =  ohms.  ;  then,  by  Ohm's  Law  C  =  E/R, 
Also,  EC  =  C2R  =  E2  R  =  Watts. 


Therefore,  H.P.  =          =         =          . 

746      746     746R- 


UNITS  OF  MEASUREMENT  AND  THEIR  DEFINITIONS.        413 


PRACTICAL  ELECTRICAL  UNITS. 

1.  As  a  Unit  of  Resistance  (R),  th«  International  Ohm  (ohm  or  w), 
based  upon  the  ohm  which  is  10$  units  of  resistance  in  the  C.G.S.  system 
of  electro -magnetic  units,  is  represented  by  the  resistance  offered  to  an 
unvarying  electric  current  by  a  column  of  mercury  at  the  temperature^  of 
melting  ice,  14-4521  grammes  in  mass,  of  a  constant  cross-sectional  area 
and  of  the  length  of  106*3  centimetres. 

2.  As  a  Unit  of  Current  (C),  the  International  Ampere  (A),  which 
is  one-tenth  of  the  unit  of  current  of  the  C.G.S.  system  of  electro-magnetic 
units,  and  which  is  represented  sufficiently  well  for  practical  use  by  the 
unvarying  current  which,  when  passed  through  a  solution  of  nitrate  of 
silver  in  water,  and  in  accordance  with  the  International  specifications, 
deposits  silver  at  the  rate  of  o'OOinS  grammes  per  second. 

3.  As  a  Unit  of  Electro-motive  Force  (E)  the  International  Volt 
(V),  which  is  the   E.M.F.   that,  steadily  applied  to   a  conductor  whose 
resistance  is  one  International  Ohm,  will  produce  a  current  of  one  Inter- 
national Ampere,  and  which  is  represented  sufficiently  well  for  practical 
use  by  £$$$  of  the  E.M.F.  between  the  poles  or  electrodes  of  the  voltaic 
cell  known  as  Clark's  cell,  at  a  temperature  of  15°  Centigrade,  and  prepared 
in  the  manner  described  in  the  International  specification. 

4.  As  the  Unit  of  Quantity  (Q)  the  International  Coulomb  (A  xs), 
which  is  the  quantity  of  electricity  transferred  by  a  current  of  one  Inter- 
national Ampere  in  one  second. 

5.  As  the  Unit  of  Capacity  (K)  the  International  Farad  (Fd),  which 
is  the  capacity  of  a  conductor  charged  to  a  potential  of  one  International 
Volt  by  one  International  Coulomb  of  electricity. 

6.  As  a  Unit  of  Work  the  Joule  (J)  (or  Watt-second  (W  x  $),)  which 
is    ID?   units   of   work   in   the    C.G.S.    system,   and   which  is  represented 
sufficiently  well  for  practical  use  by  the  energy  expended  in  I  second  in 
heating  an  International  Ohm. 

7.  As  the  Unit  of  Power  (Pw) the  International  Watt  (WP  ),  which  is 
equal  to  10?  units  of  power  in  the  C.G.S.  system,  and  which  is  represented 
sufficiently  well  for  practical  use  by  the  work  done  at  the  rate  of  one  Joule 
per  second.     The  Kilowatt  (K.W.)  =  1000  Watts  =  i£  horse-power. 

8.  As  the  Unit  of  Induction  (L)  the  Henry  (Hj  ),  which  is  the 
induction  in  the  circuit  when  the  E.M.F.  induced  in  this  circuit  is  one 
International  Volt,  while  the  inducing  current  varies  at  the  rate  of  one 
ampere  per  second. 

g.  The  Board  of  Trade  Commercial  Unit  of  Work  or  (B.T.U.) 
is  the  Kilowatt-hour  (K.W.-hr.)  =  1000  Watt-hours  =  i£  H.P.  working  for 
one  hour.  Or  say  10  amperes  flowing  in  a  circuit  for  i  hour  at  a  pressure 
of  100  volts. 

Note. — For  further  simple  explanations  with  Examples,  see  7th  Edition 
of  Prof.  Jamieson's  "  Manual  of  Magnetism  and  Electricity,"  pp.  87  to  94, 
and  222  to  224.  Also  latest  Edition  of  Munro  and  Jamieson's  Electrical 
Pocket  Book,  both  published  by  Charles  Griffin  &  Co.  Ltd.,  London. 


414  APPENDIX  D. 

EXAMINATION   TABLES. 

USEFUL  CONSTANTS. 


1  Inch  a  25*4  millimetres. 

t  Gallon  =  -1606  cubic  foot  =  10  Ibs.  of  water  at  62*  F. 

I  Naut  =  6080  feet. 

I  Knot  =  6080  feet  per  hour. 

Weight  of  1  Ib.  in  London  =  445,000  dyne*. 

One  pound  avoirdupois  =  7000  grains  =  453-6  gramme*. 

1  Cubic  foot  of  water  weighs  62*3  Ibs.  at  65*  F. 

1  Cubic  foot  of  air  at  0*  0.  and  1  atmosphere,  weighs  *0807  Ib. 

1  Cubic  foot  of  Hydrogen  at  0*  0.  and  1  atmosphere,  weighs  -00557  Ib* 

1  Foot-pound  »  1-8562  x  10'  ergs. 

1  Horse-power-hour  =  83000  x  60  foot-pounds. 

1  Electrical  unit  «  1000  watt-hours. 

*»V.  Kquiv^t  to  .nit  B«gn«lf.  H,  I.    {,™  gj£  ;  }  ** 

1  Horse-power  =  38000  foot-pomnds  per  minute  =  746  watts. 
Volts  x  amperes  «=  watts. 

1  Atmosphere  =  14*7  Ib.  per  square  inch  =  2116  Ibs.  per  square  foot  a 
760  m.m.  of  mercury  =  10*  dynes  per  sq.  cm.  nearly. 

A  Column  of  water  2*3  feet  high  corresponds  to  a  pressure  of  1  Ib.  per 
square  inch. 

Absolute  temp.,  t  *  V  0.  +  273*-7. 

Regnaulfs  H  =  606'6  +  '305  6°  0.  m  1082  +  -805  8*  F. 

fu  1.0*46^479 

log  Ml>  =  61007  -f-£ 

where  log  ^B  =  3-1812,  log  WC  a  fi-0871, 

p  is  in  pounds  per  square  inch,  t  is  absolute  temperature  Centigrade^, 

u  is  the  volume  in  cubic  feet  per  pound  of  steam. 
»  =  3-1416. 

One  radian  =  57 '3  degrees. 

To  convert  common  into  Napierian  logarithms,  multiplj  by  2-3021 
The  base  of  the  Napierian  logarithms  is  •  =  2-7183. 
The  value  of  g  at  London. «  32-182  feet  per  sec.  per  sec. 


TABLE   OF  LOGARITHMS. 


415 


10 

0 

1 

2 

3 

4 

5 

6 

7 

8 

9 

123 

456 

789 

0000 

0043 

0086 

0123 

0170 

0212 

0253 

0294 

0334 

0374 

4  8  12 

17  21  25 

29  33  37 

11 

12 
13 

14 
15 
16 

17 
13 
19 

0414 
0792 
1139 

0453 
0328 
1173 

0492 
0864 
1206 

0531 
OS99 
1239 

0569 
0934 
1271 

0607 
0969 
1303 

0645 
1004 
1335 

0682 
1038 
1367 

0719 
1072 
1399 

0755 
1106 
1430 

4  8  11 
3  7  10 
3  6  10 

15  19  23  26  30  34 
14  17  21'24  28  31 
13  16  19J23  26  29 

1461 
1761 
2041 

1492 
1790 
2068 

1523 
1818 
2095 

1553 
1847 
2122 

1534 
1875 
2148 

1614 
1903 
2175 

1644 
1931 
2201 

1673 
1959 
2227 

1703 
1987 
2253 

1732 
2014 
2279 

369 
368 
358 

12  15  18 
11  14  17 
11  13  16 

21  24  27 
20  22  25 
18  21  24 

2304 
2553 

2788 

2330 
2577 
2810 

2355 
2601 
2833 

2380 
2625 
2356 

2405 
2648 
2878 

2430 
2672 
2900 

2455 
2695 
2923 

2480 
2718 
2945 

2504 
2742 
2967 

2529 
2765 
2989 

257 
257 
247 

10  12  15 
9  12  14 
9  11  13 

17  20  22 
16  19  21 
16  18  20 

20 

21 
22 
23 

3010 

3032 

3054 

3075 

3096 

3118 

3139 

3160 

3131 

3201 

246 

8  11  13J15  17  19 

3222 
3424 
3617 

3243 
3444 
3636 

3263 
3464 
3655 

3284 
34S3 
3674 

3304 
3502 
3692 

3324 
3522 
3711 

3345 
3541 
3729 

3365 
3560 
3747 

3385 
3579 
3766 

3404 
3393 
3784 

246 
246 

246 

8  10  1214  16  18 
8  10  1214  15  17 
7  9  11113  15  17 

24 
25 
26 

27 
28 
29 

30 

31 
32 
33 

3S02 
3979 
4150 

3820 
3997 
4166 

3833 
4014 
4183 

3356 
4031 
4200 

3S74 
4048 
4216 

3392 
4065 
4232 

3909 
40S2 
4249 

3927 
4099 
4265 

3945 
4116 
4281 

3962 
4133 
4298 

2.45 
235 
235 

7  9  11  12  14  16 
7  9  10'l2  14  15 
7  8  1011  13  15 

4314 
4472 
4624 

4330 
4487 
4639 

4346 
4502 
4654 

4362 
4518 
4669 

4378 
4533 
4683 

4393 
4548 
4698 

4409 
4564 
4713. 

4425 
4579 
4728 

4440 
4594 
4742 

4456 
4609 
4757 

23568  9J11  13  14 
23568  9>11  12  14 
1  3  4  1  6  7  9  10  12  13 

4771 

4786 

4800 

4814 

4S29 

4843 

4S57 

4871 

4886 

4900 

134 

679 

678 
578 
568 

10  11  13 

10  11  12 
9  11  12 
9  10  12 

4914 
5051 
5185 

4923 
5065 
5193 

4942 
5079 
5211 

4955 
5092 
5224 

4969 
5105 
5237 

4933 
5119 
5250 

4937 
5132 
5263 

5011 
5145 
5276 

5024 
5159 
5289 

5038 
5172 
5302 

134 
134 
134 

34 
35 
36 

5315 
5441 
5563 

5328 
5453 
5575 

5340 
5465 
55S7 

5353 
5478 
5599 

5366 
5490 
5611 

5373 
5502 
5623 

5391 
5514 
5635 

5403 
5527 
5647 

5416 
5539 
5658 

5428 
5551 
5670 

134 
124 
124 

568 
567 
567 

9  10  11 
9  10  11 
8  10  11 

37 
33 
39 

40 

41 
42 
43 

56S2 
5798 
5911 

5694 
5809 
5922 

57  ;. 

5821 
5933 

5717 
5832 
5944 

5729 
5843 
5955 

5740 
5855 
5966 

5752 
5866 
5977 

5763 
5877 
5988 

5775 
5888 
5999 

5786 
5899 
6010 

123 
123 
123 

5  6  7 
567 
467 

8  9  10 
8  9  10 

s  a  10 

6021 

C031 

6042 

6053 

6064 

6075 

6035 

6096 

6107 

6212 
6314 
6415 

6117 

123 

456 

8  9  10 

6128 
6232 
6335 

6138 
6243 
6345 

6149 
6253 
6355 

6160 
6263 
6365 

6170 
6274 
6375 

6180 
6284 
6385 

6191 
6294 
6395 

6201 
6304 
6405 

6222 
6325 
6425 

123 
123 
123 

456 
456 
456 

789 
789 
789 

44 
45 
46 

6435 
6532 
6628 

6444 

6542 
6637 

6454 
6551 
6646 

6464 
6561 
6656 

6474 
6571 
6665 

6484 
6580 
6675 

6493 
6590 
6634 

6503 
6599 
6693 

6513 
66C9 
6702 

6522 
6618 
6712 

123 
123 
123 

456 
456 
456 

7  8  9 
789 
778 

47 
43 
49 

50 

6721 
6812 
6902 

6730 
6821 
6911 

6739 
6330 
6920 

6749 
6839 
6928 

6753 
6848 
6937 

6767 
6857 
6946 

6776 
6866 
6955 

6785 
6875 
6964 

6794 
6884 
6972 

6803 
6S93 
6981 

123 
123 

123 

455 
445 
445 

678 
678 
678 

6990 

6998 

7007 

7016 

7024 

7033 

7042 

7050 

7059 

7067 

123 

345 

678 

51 
Pi  -7 
53 

7076 
7lf:0 
7243 

7324 

7034 
7168 
7251 

7093 
7177 
7259 

7101 
7185 
7267 

7348 

7110 
7193 
7275 

7118 
7202 
7284 

7126 
7210 
7292 

7135 
7218 
7300 

7143 
7226 
7308 

7152 
7235 
7316 

123 
122 
122 

345 
345 
345 

678 
677 
667 

7332 

7340 

7356 

7364 

7372 

7380 

7388 

7306 

122 

345 

667 

416 


TABLE   OP   LOGARITHMS. — Continued. 


0 

ir 

1  2 

3 

4 

5 

6 

7 

8 

9 

1  2  3 

456 

789 

55 

7404 

7412 

7419 

7427 

7435 

7443 

7451 

7459 

7466 

7474 

122 

345 

567 

56 
57 

58 

7482 
7559 
7634 

7490 
7566 
7642 

7497 
7574 
7649 

7505 
7582 
7657 

7513 
7589 
7664 

7520 
7597 
7672 

7528 
7604 
7679 

7536 
7612 
7686 

7543 
7619 
7694 

7551 
7627 

77a 

122 

1  2  2 
112 

345 

345 
344 

567 
567 
567 

59 
60 
61 

7709 

7782 
7S53 

7716 

7789 
7860 

7723 
7796 
7868 

7731 

7S03 
7875 

7738 
7810 

78S2 

77-^5 
7818 
7889 

7752 
7825 
7896 

7760 
79.J3 

7767 
7C39 
7910 

7774 
78-16 
79.  7 

112 
112 
112 

344 
344 
344 

567 
566 
566 

62 
63 
64 

7924 
7993 

80G2 

7931 

8000 
8069 

7938 
8007 
8075 

7945 
8014 

80S2 

7952 

8021 
80i9 

7959 
£028 
8096 

7966 
80  5 
8102 

7973 
8041 
8109 

7980 
8048 
8116 

7917 
8055 
8122 

112 
112 

112 

334 
334 
334 

566 
556 
556 

65 

8129 

8136 

8142 

8149 

8156 

8162 

8169 

8176 

8182 

81  9 

112 

334 

556 

66 
67 
68 

8195 
8261 
8325 

8202 
8267 
8331 

8209 
8274 
8338 

8215 
8280 
8344 

8222 
8287 
8351 

8228 
8293 
8357 

8235 
8299 
8363 

8241 
83C6 
8370 

8248 
8312 
8376 

SC54 
8319 
8382 

112 

112 
112 

334 
334 
334 

556 
556 
456 

69 
70 
71 

8388 
8451 
8513 

8395 
8457 
8519 

8401 
8403 
8525 

8407 
8470 
8531 

8414 
8476 
8537 

8420 
8482 
8543 

8426 
8488 
8549 

8432 
8494 
8555 

8439 
8500 
8561 

8445 
85C6 
8567 

1  1  2 

112 
112 

234 
234 
234 

456 
456 
455 

72 
73 

74 

8573 
8633 
8692 

8579 
8G39 
8698 

8585 
8645 
8704 

S591 
8651 
8710 

8597 
8657 
8716 

S603 
8663 
8722 

8609 
8669 
8727 

8615 
8675 
8733 

8621 
8681 
8739 

8627 
86S6 
8745 

112 
112 
112 

234 
234 
234 

455 
455 

455 

75 

8751 

8756 

8762 

8768 

8774 

8779 

8785 

8791 

8797 

8802 

112 

233 

455 

76 

77 

78 

8808 
8S65 
8921 

8814 
8871 
8927 

8820 
8876 
8932 

8S25 
8S82 
8938 

8831 
SS87 
£943 

8837 
8;93 
8949 

8842 
8899 
8954 

8848 
8904 
8960 

8854 
8910 
8965 

S8f.9 
8915 
8971 

112 
112 
112 

233 
233 
233 

455 
445 
445 

79 
SO 
81 

8976 
9031 
9085 

C9S2 
C036 
9090 

S9S7 
9042 
9096 

8993 
9047 
9101 

89C8 
90C3 
9106 

9004 
90C3 
9112 

9009 
90C3 
9117 

9015 
9069 
9122 

9020 
9074 
9128 

9025 
9079 
9133 

112 
112 
112 

233 
233 
233 

445 
445 
445 

82 
S3 
84 

9138 
9191 
9243 

9143 
9196 
9248 

9149 
9201 
9253 

9154 
92C6 
9258 

9159 
9212 
9263 

9165 
9217 
9269 

9170 
9222 
9274 

9175 
9227 
9279 

91  SO 
9232 
9284 

9186 
9238 
9289 

112 
112 
112 

233 
233 
233 

445 
445 
445 

85 

9294 

9299 

9304 

9309 

9315 

9320 

9325 

C330 

9335 

9340 

112 

233 

445 

86 

87 
88 

9345 
9395 
9445 

9350 
9400 
9450 

9355 
9405 
9455 

9360 
9410 
9460 

9365 
9415 
9465 

9370 
9420 
9469 

9375 
9425 
9474 

C3GO 
9430 
9479 

9385 
9435 

9484 

9390 
9440 
94S9 

112 
Oil 
Oil 

233 
223 
223 

445 
344 
344 

89 
90 
91 

9494 
9542 
9590 

9499 
9547 
9595 

9504 
9552 
9600 

9509 
9557 
9605 

9513 
9562 
9609 

9518 
9566 
9614 

9523 
9571 
9619 

9528 
9576 
9624 

9533 
9581 
9628 

9538 
95S6 
9633 

Oil 

Oil 
Oil 

223 
223 
223 

344 
344 
344 

92 
93 
94 

9088 
96S5 
9731 

9643 
9689 
9736 

9647 
9694 
9741 

9652 
9699 
9745 

9657 
9703 
9750 

9661 
9708 
9754 

9666 
9713 
9759 

9671 
9717 
9763 

9675 
97-22 
9768 

9680 
9727 
9773 

Oil 
Oil 
Oil 

223 
223 
223 

344 
344 
344 

95 

9777 

9?82 

9786 

9791 

9795 

9800 

9805 

9809 

9814 

9818 

Oil 

223 

344 

96 

97 
98 

9S23 
9868 
9912 

9827 
9872 
9917 

9832 
9877 
9921 

9836 
9881 
9926 

9341 

9886 
9930 

9845 
9890 
9934 

9S50 
9894 
9939 

9854 
9899 
9943 

9859 
9903, 
9948 

9S63 
9908 
9952 

Oil 
Oil 
Oil 

223 
223 
223 

344 
344 
344 

99 

9956 

9961 

9965 

9969 

9974 

9978 

9983 

9987 

9991 

9996 

Oil 

223 

334 

TABLE   OF   ANTILOGARITHMS. 


417 


0 

1 

2 

3 

4 

5 

6 

7  1 

8 

9 

1  2  3 

456 

789 

00| 

1000 

1002 

1005 

1007 

1009 

1012 

1014 

1016 

1019 

1021 

001 

111 

222 

•01 
•02 
•03 

1023 
1047 
1072 

1026 
1050 
1074 

1028 
1052 
1076 

1030 
1054 
1079 

1033 
1057 
1081 

1035 
1059 
1084 

1038 
1062 
1086 

1040 
1064 
1089 

1042 
1067 
1091 

1045 
1069 
1094 

001 
001 
001 

111 
111 
111 

222 
222 
222 

•041 

•06 

1096 
1122 
1148 

1099 
1125 
1151 

1102 
1127 
1153 

1104 
1130 
1156 

1107 
1132 
1159 

1109 
1135 
1161 

1112 
1133 
1164 

1114 
1140 
1167 

1117 
1143 
1169 

1119 
1146 
1172 

Oil 
Oil 
Oil 

1    2  ' 
1    2  j 

1    2| 

222 
222 
222 

•07 
•08 
•09 

1175 
1202 
1230 

1178 
1205 
1233 

1180 
1-208 
1236 

1183 
1211 
1239 

1186 
1213 
1242 

1189 
1216 
1245 

1191 
1219 
1247 

1194 
1222 
1250 

1197 
1225 
1253 

1199 
1227 
1256 

Oil 
Oil 
0  1  1 

1    2 
1    2 
112 

222 
223 
223 

•10 

1259 

1262 

1265 

1268 

1271 

1274 

1276 

1279 

1282 

1285 

Oil 

112 

223 

•11 
•12 
•13 

1288 
1318 
1349 

1291 
1321 
1352 

1294 
1324 
1355 

1297 
1327 
1353 

1300 
1330 
1361 

1303 
1334 
13.5 

1306 
1337 
1363 

1309 
1340 
1371 

1312 
1343 
1374 

1315 
1346 
1377 

Oil 
Oil 
Oil 

122 
122 
122 

223 
223 
233 

•14 
•15 

•1C 

13SO 
1413 
1445 

13S4 
1416 
1449 

13S7 
141£ 
1452 

1390 
1422 
1455 

1393 
1426 
1459 

1396 
14_>9 
1462 

1400 
1432 
1466 

1403 
1435 
1469 

1406 
1439 
1472 

1409 
1442 
1476 

Oil 
Oil 
0  1  1 

1  2  2 
122 
1  2  2 

233 
233 
233 

•17 

•is 

•19 

1479 
1514 
1549 

1483 
1517 
1552 

14S6 
1521 
1556 

14S9 
1524 
1560 

1493 
1523 
1563 

1496 
1531 
1567 

1500 
1535 
1570 

1503 
1538 
1574 

1507 
1542 
1578 

1510 
1545 
1581 

0  1  1 
Oil 
0  1  1 

122 
122 
122 

233 
233 
333 

•20 

1585 

1589 

1592 

1596 

1600 

1603 

1607 

1611 

1614 

1613 

Oil 

122 

333 

•21 
•22 
•23 

1022 
1660 
1093 

1626 
1603 
1702 

1620 
1667 
1706 

1633 
1671 
1710 

1637 
1675 
1714 

1641 
1679 
1713 

1644 
16^3 
1722 

1643 
1687 
1726 

1652 
1690 
1730 

1656 
1694 
1734 

Oil 
0  1  1 
Oil 

222 
222 
222 

333 
333 
334 

•24 
•25 
•26 

1738 
1778 
1820 

1742 
1782 
1824 

1746 
1786 
1828 

1750 
1791 
1832 

1754 
1795 
1837 

1758 
1799 
1841 

1762, 
1803" 
1845 

1766 
1307 
1849 

1770 
1811 
1854 

1774 
1816 
1858 

Oil 
Oil 
Oil 

222 
222 
223 

334 
334 
334 

•27 
•28 
•29 

1862 
1905 
1950 

1866 
1910 
1954 

1871 
1914 
1959 

1875 
1919 
1963 

1879 
1923 
1968 

1884 
1928 
1972 

1888 
1932 
1977 

1892 
1936 
1982 

1897 
1941 
1986 

1901 
1945 
1991 

Oil 
0  1  1 
Oil 

223 
223 
223 

334 
344 
344 

•30 

1995 

2000 

2004 

2009 

2014 

2018 

2023 

2028 

2032 

2037 

Oil 

223 

344 

•31 
•32 
•33 

2042 
2089 
2138 

*2046 
2094 
2143 

2051 
2099 
2148 

2056 
2104 
2153 

2061 
2109 
2153 

2065 
2113 
2163 

2070 
2118 
2168 

2075 
2123 
2173 

2080 
2123 
2173 

20S4 
2133 
2183 

Oil 
Oil 
Oil 

223 
223 
223 

344 
344 
344 

•34 
•35 
•36 

2188 
2239 
2291 

2193 
2244 
2296 

2198 
2249 
2301 

2203 
2254 
2307 

2203 
2259 
2312 

2213 
2265 
2317 

2218 
2270 
2323 

2223 
2275 
2328 

2228 
2280 
2333 

2234 
2286 
2339 

112 
112 
112 

233 
233 
233 

4  5 
4  5 
4  5 

•37 
•38 
•39 

2344 
2399 
2455 

2350 

2404 
2460 

2355 
2410 
2466 

2360 
2415 
2472 

2366 
2421 
2477 

2371 
2427 
2483 

2377 
2432 
2489 

2382 
2438 
2495 

2333 
2443 
2500 

2393 
2449 
2506 

112 
112 

112 

233 
233 
233 

4  5 
4  5 
5  5 

•40 

2512 

2518 

2523 

2529 

2535 

2541 

2547 

2553 

2-559 

2564 

112 

234 

455 

•41 
•42 
•43 

2570 
2630 
2692 

2576 
2636 
2698 

2582 
2642 
2704 

2588 
2649 
2710 

2594 
2655 
2716 

2600 
2661 

2723 

2606 
2667 
2729 

2612 
2673 
2735 

2618 
2679 
2742 

2624 
26S5 
2743 

112 
112 
112 

2  3 
2  3 
3  3 

455 
456 
456 

•44 
•45 
•46 

2754 
2818 

28S4 

2761 
2825 
2891 

2767 
2831 
2897 

2773 
2833 
2904 

2780 
2S44 
2911 

2786 
2851 
2917 

2793 
2853 
2924 

2799 
2864 
2931 

2805 
2371 
2938 

2312 
2S77 
2944 

112 
112 
112 

3  3 
3  3 
3  3 

456 
556 
556 

•47 
•48 
•49 

2951 
3020 

:3000 

.2953 
3027 
3097 

2965 
3034 
3105 

2972 
3041 
3112 

2979 
3048 
3119 

2985 
3055 
3126 

2992 
3062 
3133 

2999 
3069 
3141 

3006 
3076 
3148 

3013 
30S3 
3155 

112 

112 
112 

3  3 

i  3  4 
3  4 

556 
566 
566 

TABLE    OF   ANTILOGARITHMS. —  continued. 


0 

1 

2 

3 

* 

5 

6 

7 

8 

9 

1  2  3 

456 

789 

•50 

3162 

3170 

3177 

3184 

3192 

3199 

3206 

3214 

3221 

3228 

112 

344 

567 

•51 
•53 

3236 
3311 

3388 

3243 
3319 
3396 

3251 
3327 
3404 

3258 
3334 
3412 

3266 
3342 
3420 

3273 
3350 
3428 

3281 
3357 
3436 

32S9 
3365 
3443 

32% 
3373 
3451 

3304 
3381 
3459 

2  2 
2  2 

2  2 

3  4  5 
3  5 

3  5 

567 
567 
667 

•54 
•55 
•56 

3467 
3548 
3631 

3475 
3556 
3639 

34S3 
3565 
3648 

3491 
3573 
3656 

3499 
3581 
3664 

3508 
35S9 
3673 

3516 
3597 
3  81 

3524 
3606 
3690 

3532 
3614 
3698 

3540 

3622 
3707 

2  2 
2  2 
2  3 

3  5 
3  5 

3  5 

667 
677 
678 

•57 
•58 
•59 

3715 
SS02 
3S90 

3724 
3811 
3899 

3733 

3819 
3908 

3741 
3  28 
39J7 

3750 
3837 
3926 

3758 
3846 
3936 

3767 
3855 
3945 

3776 
3864 
3954 

3784 
3.S73 
3963 

3793 

3882 
3972 

123 
123 
123 

345 
446 
455 

678 

678 
678 

•60 

3981 

3990 

3999 

4009 

4018 

4027 

4036 

4046 

4055 

4064 

123 

456 

678 

•61 
•62 
63 

4074 
4169 
4266 

4083 
4178 
4276 

4093 
4188 
4285 

4102 
4198 
4295 

4111 
4207 
4305 

4121 
4217 
4315 

4130 
4227 
4325 

4140 
4236 
4335 

4150 
4246 
4345 

4159 
42C6 
4355 

123 
123 
123 

456 
456 
456 

789 
789 

789 

•64 
•65 
•66 

4365 
4467 
4571 

4375 

4477 
4581 

4385 
4487 
4592 

4395 

4498 
4603 

4406 
4508 
4613 

4416 
4519 
4624 

4426 
4529 
4634 

4436 
4539 
4645 

4446 
4550 
4656 

4457 
4560 
4567 

123 
123 
123 

450 
456 
456 

789 
789 
7  9  10 

•67 
•68 
•69 

4677 
47S6 
4898 

46S8 
4797 
4909 

4G99 
4£OS 
49iO 

4710 
4819 
4932 

4721 
4S31 
4943 

4732 
4842 
4955 

4742 
4853 
4966 

4753 
4864 
4977 

4764 
4S75 
4989 

4775 

4887 
5000 

123 
123 
123 

457 
467 
567 

8  9  10 
8  9  10 
8  9  10 

•70 

5012 

5023 

5G35 

5047 

50:^ 

5070 

5082 

5093 

5105 

5117 

1  2  4 

567 

8  9  11 

•71 
"72 
•73 

5129 
5248 
5370 

5140 
5260 
5383 

5152 
5272 
5395 

5164 

5284 
5408 

517J 
5297 
5420 

51S8 
5309 
5433 

5200 
6321 
5445 

5212 
63.-J3 
5458 

5224 
5346 
5470 

5236 
5358 
5483 

1  2 
1  2 
1  3 

567 
567 
368 

8  10  11 
9  10  11 
9  10  11 

•74 
•75 
•76 

5495 
5623 
5754 

5508 
5636 
5768 

5521 
5049 
5781 

5534 
5662 
5794 

5546 
5675 
5808 

5559 
56S9 
5821 

5572 
5702 
5S34 

5585 
5715 

5S48 

5598 
5728 
5861 

5610 
5741 
5875 

1  3 
1  3 
134 

568 

578 
578 

9  10  12 
9  10  12 
9  11  12 

•77 
•78 
•79 

5888 
C026 
6166 

5902 
6039 
6180 

£916 
6053 
6194 

5929 
CC67 
6209 

5943 
6Gol 
6223 

5957 
6095 
6237 

6970 
6109 
6252 

5984 
(124 
6266 

5998 
6138 
C281 

G012 
6152 
6295 

134 
134 
134 

578 
678 
679 

10  11  12 
10  11  13 
10  11  18 

•80 

6310 

C324 

6339 

6353 

6368 

6383 

6397 

6412 

6427 

6442 

134 

679 

10  12  13 

•81 

•82 
•83 

6457 
6607 
6761 

6471 
6622 
6776 

G-JS6 
6637 
6792 

6501 
6653 
6i08 

6516 
6G68 
SS23 

6531 
66S3 
6839 

6546 
6699 
6S55 

6561 
6714 

6S71 

6577 
6730 
6887 

6592 
6745 
6902 

235 
2  C  5 
235 

689 
089 
689 

11  12  14 
11  12  14 
11  13  14 

•84 
•35 
•86 

6918 
7079 
7244 

6934 
7096 
7261 

6950 
7112 
7278 

6C66 
7129 
7295 

69£2 

7145 
7311 

6998 
7161 
7328 

7015 
7173 
7345 

7031 
7194 
7362 

7047 
7211 
7379 

7CC3 
7228 
7396 

235 
235 

235 

6  8  10 
7  8  10 
7  8  10 

11  13  15 
12  13  15 
,12  13  15 

•87 
•88 
•89 

7413 
7586 
7762 

7430 
7603 
7780 

7447 
76:>1 
7798 

7464 
7638 
7816 

7482 
7656 
7834 

7499 
7674 
7852 

7516 
7091 
7S70 

7534 
7709 
78t9 

7551 
7727 
7907 

75C8 
7745 
7925 

235 

2  4  5 

245 

7  9  10 
7  9  11 
7  9  11 

12  14  16 
121418- 
13  14  16 

•90 

7943 

7962 

7980 

7998 

8017 

8035 

8054 

8072 

8891 

8110 

246 

7  9  11 

13  15  17 

•91 
•92 
•93 

8128 
8318 
8511 

8147 
8337 
8531 

8166 
8356 
8551 

8185 
8570 

8204 
8395 
8590 

8222 
8414 
8610 

8241 
8433 
8630 

8260 
8453 
8650 

8279 
8472 
8670 

8299 
8492 
8690 

240 
240 
240 

8  9  11 
8  10  12 
8  10  12 

[13  15  17 
14  15  17 

J14  16  18 

•94 
•95 
•96 

8710 
8913 
9120 

8730 
8933 
9141 

8750 
8954 
9162 

8770 
8974 
9183 

8790 
8995 
9204 

8810 
9016 
9226 

8S31 
9036 
9247 

8851 
9057 
9268 

8872 
9078 
9290 

8892 
9099 
9311 

|2  4  6 

240 
240 

8  10  12 
8  10  12 
8  11  13 

14  If  18 
15  17  19 
15  17  19 

•97 
•98 
•99 

9333 
9550 
9772 

9354 
9572 
9795 

9376 

9594 
9817 

9397 
9616 
9S40 

9419 
9638 
98C3 

9441 
9661 
9886 

9462 
9683 
9908 

9484 
9705 
9931 

9506 
9727 
C954 

9528 
9750 
9977 

247 
247 
257 

9  11  13 
9  11  13 
9  11  14 

15  17  20 
16  18  20 
16  18  20 

TABLE  OF  FUNCTIONS  OF  IHOLM. 


419 


1 

Angle. 

; 

Chord. 

fine. 

Tangent. 

Co-  tangent. 

Cosine. 

Degrees 

Radians. 

0° 

0 

000             C 

0 

ao 

1 

1-414 

1-5703 

90» 

1 

•0175 

•017     l    '0175 

•0175 

57-2900 

•9998 

1-402 

1-5533 

89 

2 

•034D 

•035 

•0349 

•0349 

28-6363 

•9994 

1-3S9 

1-5359 

83 

3 

•0524 

•052 

•0523 

•0524 

19-0311 

•99S6 

1-377 

1-51S4 

87 

4 

•0698 

•070 

•0698 

•0699 

14-3007 

•9976 

1-364 

1-5010 

86 

5 

•OS73 

•087 

•0872 

•0875 

11-4301 

•9962 

1-351 

1-4S35 

85 

6 

•1047 

•105 

•1045 

•1051 

9-5144 

•9945 

1-338 

1-4661 

84 

7 

•1222 

•122 

•1219 

•1228 

8-1443 

•9925 

1325 

1-4486 

83 

8 

•1396 

•140 

•1392 

•1405 

7-1154 

•9903 

1-312 

1-4312 

82 

9 

•1571 

•157 

•1564 

•1584 

6-3133 

•9S77 

1-299 

1-4137 

81 

10 

•1745 

•174 

•1736 

•1763 

5-6713 

•9848 

1-2S6 

i-rso 

80 

11 

•1929 

•192 

•1903 

•1944 

5-1446 

•9316 

1-272 

l-37t3 

79 

12 

•2094 

•209 

•2079 

•2126 

4-7046 

•97S1 

1-259 

1-3614 

73 

13 

•2269 

•228 

•2250 

•2309 

4-3315 

•9744 

1-245 

1-3439 

i 

H 

•2443 

•244 

•2419 

•2493 

4-0108 

•9703 

1-231 

1-3265 

Z      ' 

w    a 

•2613 

•261 

•2588 

•8679 

87821 

•9659 

1-213 

13090 

5 

16 

•2793 

•278 

•2756 

•2S67 

3-4S74 

•9613 

1-204 

1-2915 

4 

17 

•2967 

•296 

•2924 

•3057 

32709 

•9563 

1-190 

1-2741 

3      { 

13 

•3142 

•313 

•3090 

•3249 

3-0777 

•9511 

T176 

1-2566 

2      ' 

19 

•3316 

•330 

•3256 

•3443 

2-9042 

•9455 

1-161 

1-2302 

1 

20 

•3491 

•347 

•3420 

•3640 

2-7475 

•9397 

1-147 

1-2217 

0 

21 

•3665 

•364 

•3584 

•3S39 

2*6051 

•9336 

1-133 

"2043 

69 

22 

•3840 

•382 

•3746 

•4040 

2-4751 

•9272 

1-118 

•1S63 

6* 

23 

•4014 

•399 

•3907 

•4245 

2-3559 

•9205 

1-104 

•1694 

67 

24 

•4189 

•416 

•4067 

•4452 

2-2460 

•9135 

1-089 

•1519 

66 

25 

•4363 

•433 

•4226 

•4663 

2-1445 

•9063 

1-075 

•1345 

65 

20 

•4538 

•450 

•4384 

•4877 

2-0503 

•89S8 

1-060 

1-1170 

64 

"7 

•4712 

•467 

•4540 

•5095 

1-9626 

•8910 

1-045 

1-0996 

63 

23 

•4387 

•484 

•4695 

•5317 

1-SS07 

•8829 

1-030 

1-0821 

62 

29 

•5061 

•501 

•4848 

•5543 

1-SC40 

•8746 

1-015 

1-0647 

61 

30 

•5236 

•518 

•5000 

•5774 

1-7321 

•8660 

1-000 

1-0472 

60 

31 

•5411 

•534 

•5150 

•6009 

1-6643 

•8572 

•985 

1-0297 

59 

32 

•5585 

•551 

•5299 

•6249 

1-6003 

•84SO 

•970 

1-0123 

58 

33 

•5760 

•568 

•5446 

•6494 

1-5399 

•8387 

•954 

•9943 

57 

34 

•5934 

•585 

•5592 

•6745 

1-4826 

•8290 

•939 

•9774 

56 

35 

•6109 

•601 

•5736 

•7002 

1-4281 

•8192 

•923 

•9599 

55 

36 

•G283 

•618 

•5S78 

•7265 

1-3764 

•8090 

•908 

•9425 

54 

37 

•6458 

•635 

•6018 

•7536 

1-3270 

•79S6 

•892 

•9250 

53 

33 

•6632 

•651 

•6157 

•7813 

1-2793 

•7880 

•877 

•9076 

52 

39 

•6S07 

•668 

•6293 

•8098 

1-2349 

•7771 

•861 

•8901 

51 

40 

•C9S1 

•684 

•6428 

•8391 

1-1918 

•7660 

•845 

•8727 

50 

41 

•71.  '6 

•700 

•6561 

•8693 

1-1504 

•7547 

•829 

•8552 

49 

42 

•7330 

•717 

•6691 

•9004 

1-1106 

•7431 

•813 

•8373 

43 

43 

•7505 

•733 

•6820 

•9325 

1-0724 

•7314 

•797 

•8203 

47 

44 

•7679 

•749 

•6947 

•9657 

1-0355 

•7193 

•781 

•8029 

46 

45° 

•7S54 

•765 

•7071 

1-0000 

1-0000 

•7071 

•765 

•7854 

45° 

• 

Radians. 

Degrees. 

Cosine. 

Co-  tangent. 

Tangent. 

Sine. 

Chord. 

Angle. 

INDEX 


ACCELERATION,  definition,  260,  421 
,,  due  to  gravity,  260 

,,  unit  of,  260 

Accumulated  work,  280 

,,  work  in  a   fly-wheel, 

281 
,,  work    in    a    rotating 

body, 281 

Accumulator,  hydraulic,  253-2^5 
Action  and  reaction,  3 

,,      of     the     common     suction- 
pump,  227-229 

Activity,  definition  of,  12,  412 
Actual  or  working  advantage,  67 
Admission  of  Students  to  the  Insti- 
tution   of    Civil    Engineers, 
rules  and  syllabus  of  Exam?, 
for,  370 
Air  vessel,  action  of  an,  232 

,,      force  pump  with,  231 
Ampere,  413 

Angle  of  repose,  io5 ;  resistance,  107 
Angular  and  linear  motion,  272 

,,        velocity,  260 
Anti-friction  wheels,  109 
Anti-logarithms,  Appendix  D,  417 
Applied  Mechanics,  definition  of,  I 
Archimedes,  law  of,  217-219 
Atmospheric  pressure,  220 
Atwood's  machine,  262-269 
Axle,  wheel  and,  55-57 

„        ,,    compound,  72-74 


BA.CKLA.SS,    in    whael    and    screw- 
gearings,  158 

B  ick  motion  gear  in  a  lathe,  1 74-1 8  5 
Balance,  Roman,  35 

,,          bent  lever,  43 
Balancing  fast  spsed  machinery,270 
Biles,  screw  for  pissing,  165 
Ball  bearings,  no 
Birometer,  the  mercurial,  221 


Bear,  hydraulic,  or  portable  punch- 
ing machine,  251 
Bsaringa,  ball,  1 10 

,,         roller,  183 

B3d  or  shears  of  a  lathe,  183,  187. 
B3ll  crank  lever,  43 
Belt-gaaring  reversing  motions,  122- 

124 
„     shape  of  pulley  faces 

for,  127 

,,  ,,     velocity  ratio  of  pul- 

leys in,  119-121 

Baits,  brake     horse-power     trans- 
mitted by,  118 

,,       difference  of  tension  in,  116 
,,       open  and  crossed,  121 
,,       tendency  of,  to  run  on  high- 
est parts  of  pulleys,  127 
Bench  vice,  screw,  166-167 
B^nt  lavar  balance,  43 
,,     levers,  42 
,,         ,,      duplex,  43 
Bavel  wheel  and  clutch  reversing 

gsar,  345 
Bioyole  and  railway  curves,  motion 

on,  287 
Block  and  tackle,  65-68 

,,      snatch,  65 
Block,  Weston's  differential  pullev. 

74-77 

BDird  of  Education  Exam.,  instruc- 
tions for,  368 
Baird  of  Trade  unit,  413 
Bjiies,  formulae  for  falling,  261 

pith    of    projected    badies 
which     fall     under     the 
action  of  gravity,  273 
,,       proof  by  Atwood's  michine 
of    formulae    for    falling, 
262-269 
BDiler  plates,  large  hydraulic  press 

for  flanging,  245-247 
»»        .»         single       riveted     lap 

joints,  310 

B)lt,  holding  down,  156 
B>n-accord  centrifugal  pump,  236 
421 


422  INDEX 

Bottle  screw- jack,  162-164 

Brake^  horse-power  transmitted  by 

belts,  118 
Bramah's  hydraulic  press,  241-245 

leather  collar  packing,  243 
Bucket  pump,  combined  plunger 
r  and,  234 

Buttress  screw  thread,  155 


c 

CALCULATIONS,   note  regarding  en- 
gineering, 5 
Cams,  330-334 
Capstan,  ships,  57-59 
Carpenter's  turkus,  44 
Centimetre,  411 
Centre  of  gravity,  28 

,,      of  parallel  forces,  26 
,,      of  pressure,  217 
Centrifugal  force,  275 

,,          force  machine,  277 

pumps,  236-238 

^     ,,          stress  in  fly-wheels,  279 
Centripetal  force,  276 
Chains,  stresses  in,  316 
Change  wheels  in  a  lathe,  176-179, 

183,  184,  187 

,,  ,,       reversing     plate     or 

quadrant,  183,  187 
Chinese  windlass,  72 
Circle,  definition  of  pitch,  131 
Circular  discs,  velocity  ratio  of,  130 
City  and  Guilds   exam.,  instruction 
^        for,  369 
Civil  Engineers,  Students'  Exam., 

rules  and  syllabus  of,  370 
Clarke's    patent    adjustable    curve, 

16,  17 

Click,  reversible,  335 
Co-efficient  of  friction,  105,  in 
Cohesion  of  matter,  224,  298 
Combined  lever,  screw,  and  pulley 

gear,  160 
,,          plunger       and       bucket 

pump,  234 

Comparison  of  dynamical  formulae 
^         for  linear  and  angular  mo- 
tion, 2721 
Components  and  resultant  of  forces, 

4 

,9  of  a  force  at  right  angles 

to  each  other,  84 


Composition  and  resolution  of  forces, 
82,  84  ;  velocities,  260 

Compound,     Weems',     screw     and 
hydraulic  jack,  249-251 

Compound  wheel  and  axle,  72-74 

Compressibility  of  matter,  298 

Compressing  a  bar  within  the  elastic 
limit,  work  done  in,  307 

Compressive  stress  and  strain,  301 

Cones,  stepped  speed,  124 

Constants — appendix  D,  414 

Coulomb,  413 

Couple,  definition  of,  25 

Coupling  joint,  Hooke's,  329 

,,        screw,  for  carriages,  156 

Crab,  double  purchase,  143,  144 
„     single  purchase,  140-142 

Crania,  stresses  in  various  membciB 
and  jib  arrester,  85,  92 

Crank,  bell,  lever,  43 

Curve,  focus  and  directrix  of  a,  272 

Curves,  motion  on  bicycle  and  rail- 
way, 287 

Cylinder,  forming  a  screw  thread 
on  a,  149 


D 


DEAD  load,  definition  of,  500 
Density  of  matter,  298 
Differential  pulley  blocks,  Weston's, 

74-77 

Dilatibility  of  matter,  298 
Directrix  of  a  curve,  272 
Discs,  velocity  ratio  of,  130 
Distinction,    solids,    liquids,    gases, 

fluids,  &c.,  224 

Double  acting  force  pump,  234 
,,       Hooke's  joint,  329 
,,       purchase  winch  or  crab,  143, 

144 

,,       threaded  screws,  157 
Driving  belt,  difference  of  tension 

in  a, 116 

Ductility,  definition  of,  299 
Duplex  bent  levers,  43 
Dynamical  formulae  for  linear  and 
angular  motion,  comparison 
of,  272 

E 

EFFICIENCY,  apparatus  for  deter- 
mining, of  screw 
gear,  160 


INDEX 


423 


Efficiency      of     combined     lever, 
screw,     and     pulley 

fear,  160 
a  machine,  defini- 
tion of,  53 

,,  of  screws,  151 

Elasticity,  definition  of,  300 
,,         limits  of,  303 
,,         modulus  of,  304-306 
,,          safe  loads  and,  302 
,,          table  of  moduli  of,  305 
Ellipse,  273 
Elliptic    wheels   for    quick    return, 

351-353 

Endless  screw  and  worm-wheel,  168 
Energy,  kinetic,  potential,  280,412 
Eqglish  gauges,  360 
Equilibriant  of  parallel  forces,  25 
,   Jlibriurn,  conditions  of,  In  O*M 
of    floating    bodies, 
218 
Equilibrium,  forces  in,  3 

,,  graphic  demonstration 

of  three  forces  in,  80 
Erg,  6,  412 
Exams.  B.  of  E.  C  and  G  ;  Inst.  C.E., 

Appendix  C,  403 

Experimentally     determining     the 
energy  stored  up  in  a  rotating 
fly-wheel,  285 
Extending  a  bar  within  the  elastic 

limit,  work  done  in,  307 
Extension  of  matter,  297 


FACTORS  of  safety,  302 
Falling  bodies,  formulae  for,  261 
,,       proof    of  formulae    for,   by 
Atwood's  machine,  262-269 
Farad,  413 
Feed,  silent,  340-342 
Floating  bodies,  conditions  of  equi- 
librium, 218 

Fluids,  solids,  gases,  224 
Fly- press,  the,  283 
Fly-wheels,  centrifugal  stress  in,  279 
»»  ,,     energy  stored  up  in,  281 

Fly-wheel,  to   find  experimentally 
the  energy  stored  up  in   a 
rotating,  285 
Focus  of  a  curve,  272 
Force^definition  of,  i,  412 
..     [elements  of  a,  2 


Force,  moment  of  a,  21 

,,      pump,  single-acting,  229-231 
,,      pump,  double-acting,  234 
,,      pump  with  air  vessel,  231 
,,      resolution  of  a,  into  two  com- 
ponents at  right  angles,  84 
,,      unit  of,  2,  412 

Forces,  centrifugal,  centripetal,  275 
,,       graphic  representation  of,  3 
,,       in  equilibrium,  3 
„       parallel,  25-28 
,,       parallelogram  of,  82 
,,       resultant  of,  4 
,,       straight  lever  acted  on  by 

inclined,  42 
,,       three  equal,  in  equilibrium, 

83 

„       triangle  of,  82 
„       two,  at  right  angles,  83 
Forging     and    hardening     luri-lng 

tools,  202 

Friction,  angle  of,  106 
,,        anti-,  wheels,  109 
,,        circular     discs,      velocity 

ratio  of,  130 

,,         co-efficient  of,  105,  in 
,,        definition  of,  101 
,,        heat  developed  by,  102 
,,        inclined  plane  with,   110- 

112 

,,        inclined  plane  without,  93 
.,,        laws  of,  103-109 
,,        cone  reversing  gear,  344 
Fulcrum,  position  of,  in  a  lever,  30 
,,         pressure  on,  and  reaction 

from,  25 

Fuscc,  the,  59-61 
Fusibility,  definition  of,  300 


G 


GALILEO'S   and  Kater's   pendulum 

experiments,  269 
Gas  gauge,  222  ;  gases,  224 
Gauge  limits,  micrometer,  &c.,  357 

,,      Stubs'  wire,  358 
Gauges,  low  pressure,  &c.,  221 
Gear,  back  motion  in  lathe,   174- 

192 
,,      change  wheels  in  lathe,  176- 

195 
,,      efficiency  of  combined  lev«r, 

screw  and  pulley,  160 
,,      screw  and  pulley,  160 


424 


INDEX 


Gear,  starting  and  stopping,  124 

,,      worm -wheel  lifting,  170 
Gearing,  backlash    in     wheel    and 

screw,  158 
,,          belt,    reversing    motions, 

122-124 

,,         belt,  shape  of  pulley  faces 
,,  for,  127 

,,          pitch  of  teeth  in  wheel,  132 
,,         principle  of  work  applied 

to  wheel,  135 
,,          velocity   ratio   of   pulleys 

in  belt,  119-121 

,,          velocity  ratio  in  wheel,  123 
,,          wheel,  in  jib-cranes,  144- 

146 

Grain,  screw  for  moving,  148 
Gramme,  41 1 

Graphic  representation  of  forces,  3 
,,  ,,        ,,     velocities, 

260 

Gravity,  acceleration  due  to,  260 
,,        centre  of,  28 
,,        specific,  definition  of,  214 
Gyration,  radius  of,  282 

iH 

HARDENING  the  tools  for  a  lathe, 

202 

Head  or  pressure  of  a  liquid,  205 
Headstock,  fast  or  fixed,  of  a  lathe, 

184, 185 
,,          movable,  for  a  common 

lathe,  179,  180 

,,          movable,  for  a   screw- 
cutting  lathe,  181 
Heart-shaped  cam,  3^1 
Heat  developed  by  friction,  102 

,,     relation  between, and  work,  104 
Helix  of  a  screw  thread,  148 
Henry,  413 
Herbert's  hexagon  turret  lathe,  188- 

195 
Hollow  round  shafts,  strength  of, 

324 

Homogeneous  materials,  299 
Hooke's  coupling  joint,  329 
,,        double      ,,        329 

law,  303 
Horse-power  brake,  transmitted  by 

belts,  118 

»»          »         definition  of,  12, 412 
.»  >,         of  working  agent,  13 


Huyghen's  pendulum  experiments, 
269 

Hydraulic  accumulator,  253-255 
bear,  251 
jacks,  247-251 
machines,  227-255 
press,  Bramah's,  241-245 
press,  large,  245-247 

Hydraulics,  207-258 

Hyperbola,  273 


IDLE  wheel  (note  on),  138 
Immersion  of  solids  in  fluids,  217 
Impenetrability  of  matter,  297 
Improved  Standard  Measuring  Ma- 
chine, 363 
Inclined  forces,  straight  lever  acted 

on  by,  42 

,,        plane,  the  screw  as  an,  149 
,,        planes,  principle  of  work, 

93-99,  270-272 
applied  to,  97,  no,  112 
Indicator,  motion  for  Richard's,  344 
Inertia  definition,  290 
Intermittent  motion  for  cam,  332 
Internal  and  external  limit  gauges, 

355-357 

Isotropic  material,  299 
Instructions  for  Board  of  Education 

Exams.,  368 
,,  ,,     City     and     Guilds 

E<cam.,  369 

,,  ,,     Institution  of  Civil 

Engineers      Stu- 


JACK,  bottle  screw,  162-164 
,,      traversing  screw,  164 
Jacks,  hydraulic,  247-251 
Jib  cranes,  stresses  in,  85-92 
,,         ,,      wheelgearingin,i44-i46 
Joint,  Hooke's  coupling,  329 

,,        double,  329 
Joints,  single -riveted  lap,  310 
Joule's  relation  between  heat  and 
work,  102,  413 

K 

KATER'S    and    Galileo's    pendulum 
experiments,  269 


INDEX 


425 


Kelvin's,  Lord,  wire-testing  ma- 
chine, 210 

Kilowatt,  413 

Kinetic  and  potential  energy,  defi- 
nitions of,  280 

Kinetic  energy  imparted  to  a  falling 
body, 271 

Knuckle-joint,  46-49 


LAP-JOINTS,  single  riveted,  310 
Lathe,  back  motion  gear  of,   174- 

185 

„      bed  of  a,  183,  187 
,,      change    wheels    in    a,    176- 

179,  183,  187 
,,      cutting  forces  and  H.P.  for, 

199-202 
,,      forging   and  hardening   the 

tools,  202 

,,      hexagon  turret,  188-195 
,,  ,,  ,,      feed  motion, 

192 
,,      fixed  headstock  of  a,    184, 

185,  187 

,,      leading  screw  of  a,  175,  1 86 
,,      mechanism  in  a  screw-cut- 
ting, 174-188 

,,      motions  of  saddle  and  slide- 
rest  of  a,  176 
,,      movable    headstock    for    a 

179,  180, 181 
,,      reversing  plate  for  change  of 

wheels  of  a,  184,  187 
,,      saddle  of  a,  176 
Law,  Archimedes',  217-219 
,,     Hooke's,  303 
,,     Pascal's,  208 
Laws  of  friction,  103-109 

,,      ,,  motion,  Newton's,  260 
Leading  screw  of  a  lathe,  175,  186 
Leather  collar  packing,  Bramah's, 

243 

Lever,  bell  crank,  43 
,,       bent,  42 
,,      combined    with    screw    and 

pulley,  1 60 

,,      definition  of  a,  22 
,,      duplex  bent,  43 
,,      machine  for  testing  tensile 

strength  of  materials,  41 
,,      position  of  fulcrum  of  a,  30 


Lever,  practical  applications  of  the, 

35-46 
,,      pressure    on,    and   reaction 

from,  the  fulcrum  of  a,  25 
,,      principle    of    moments    ap- 
plied to  the,  22 
„      principle  of  work  applied  to 

the,  53 

,,      safety  valve,  38-40 
,,      straight,    acted    on    by    in- 
clined forces,  42 
,,      when    its    weight    is    taken 

into  account,  29 
Lifting  gear,  worm-wheel,  170 
Limiting  angle  of  resistance,  107 

„        stress,  301 

Limit  and  micrometer  gauges,  355 
Limits  in  calculations,  5 
,,      of  elasticity,  303 
Linear  motion,       comparison       of 
dynamical     formulae     for 
angular  and,  272 
,,      velocity,  259 
,,  ,,        formulae   for,    with 

uniform  accelera- 
tion, 261 

Liquid,  definition  of  a,  207,  224 
,,       immersion  of  solids  in  a,  217 
, .       pressure  due  to  head  of  a,  209 
,,  ,,        on    any   immersed 

surface,  209, 21 1 
,,       transmission  of  pressure  hv 

a,  208 
Loads,  definition  of  live  and  dead, 

300 

,,       safe,  and  elasticity,  302 
Lockfast  lever  and  safety  valve,  39 
Logarithms — Appendix  D,  41 5 
Low  pressure  gauges,  22 1 
Lubrication,  109 
Lumberer's  tongs,  43 


M 


MACHINE,  efficiency  of  a,  53 

measuring,  363 
,,         modulus  of  a,  53,  136 
,,         testing,  40 
Machinery,  importance  of  balancing 

high  speed,  279 
Malleability,  definition  of,  298 
Mass,  definition  of,  289,  290 
Materials,  machine  for  testing  ten- 
sile  strength  of,  47   $2 


426  INDEX 


Materials,  properties  of,  297-311 
Matter,  definition  of,  I 
Measuring  tools  and  gauges,  35  5-366 
Mechanical  advantage,  66,  68 
Mechanics,  definition  of  applied,  I 
Mercurial  barometer,  the,  221 
Metals,  melting  points  of,  300 
Micrometer  screw  gauges,  357 
Modulus  of  elasticity,  304-306,  321 
,,        of  rigidity  of  a  material, 

319 

,,        of  a  machine,  53,  136 
Moment  of  a  force,  21 

,,        of  momentum,  290 
Moments,  principle  of,  21  ;   applied 
to  the  lever,  22  ;    ap- 
plied to  the  wheel  and 
axle,  55 

Momentum,  definition  of  260.  280 
Uotion  and  velocity,  250 
,,       equations  of,  261 
,,       Newton's  laws  of,  260 
,,       of  saddle  and  slide  rest  of  a 

lathe,  176 
,,       on   a    curved,    inclined,    or 

"  banked  "  track,  287 
Motions,  reversing,  343 

,,        by  belt  gearing,  122-124 

N 

NEWTON'S  laws  of  motion,  260 
Nippers,  example  of,  45 


o 


Ohm,  413 


PACKING,  Bramah's  collar,  243 
Pantograph,  339 
Parabola,  hyperbola,  ellipse,  273 
Parallel  forces,  centre  of,  26 

,,  ,,      equilibriant  and  re- 

sultant of,  25-28 
,,       motion,  339 
,,  ,,       Watt's,  338 

Parallelogram  of  forces,  82 
Pascal's  law,  208 
Passive  resistance,  101 
Path   of   a   projected    body    which 
falls    under    the    action    of 
gravity,  273 


Pawl  and  ratchet  wheel,  334 

,,     reversible,  335 
Pendulum  experiments  by  Galileo, 

Huyghens  and  Kater,  269 
Percentage  efficiency  of  a  machine, 

63,  67 
Pincers,  44 
Pinion,  rack  and,  132 
Pitch  circle,  definition  of,  131 
,,      surface,  definition  of,  131 
,,      of  rivets,  310 
,,      of  teeth  in  wheel  gearing,  132 
Piano,  inclined,  with  friction,   no- 

112 

,,      without  friction,  93,  270-272 
,,      principle  of  work  applied  to 

the  inclined,  97 
?ka&aof  belts  and  palleya  z 

gearing,  125 
Inclined,  93-99 
fianing  machine,  343 
Plates,  hydraulic  press  for  flanging 

boiler,  245 

Porosity  of  matter,  297 
Potential  energy,  definition  of,  280 
Poundal,  the,  2 
Power,  definition  of,  12,  412 
,,       horse,  definition  of,  12 
,,  ,,      transmitted  by  belts, 

118 

, ,       that  steel  shafting  will  trans- 
mit at  various  speeds,  323 
.,       units  of,  12,  412 
Press,  the  fly,  283 

,,      Bramah's  hydraulic,  241-245 

large,  245-247 
,,      screw,  for  compressing  bales, 

165 

Presses,  packing  for  hydraulic,  243 
Pressure,  atmospheric,  220 
,,         centre  of,  5,  217 
. ,         due  to  head  of  a  liquid ,  209 
,,         low    and    vacuum    vater 

gauges,  221 
,,        on  fulcrum  of  a  lever,  19, 

23-25 
„         on    ram    of    a    Bramah's 

press,  244 

,,         on  sluice  gate,  217 
,,         on   surface  in  liquid,  209 

211 
,,         measure  of,  &c.,  5 

transmited  by  liquids,  208 
total  or  thrust,  5 


INDEX 


427 


Principle  of  moments,  2 1  ;  applied 
to  the  lever,  22,  35  ;  to 
the  wheel  and  axle,  55  ; 
to  the  wheel  and  com- 
pound axle,  73 

„  of  work,  52  ;  applied  to 
the  leve'r,  53  ;  to  the 
wheel  and  axle,  56 ;  to 
the  ordinary  block  and 
tackle,  67  ;  to  the  wheel 
and  compound  axle,  73  ; 
to  Weston's  pulley  block, 
76;  to  the  inclined 
plane,  97;  to  wheel 
gearing,  135 
Pulley  blocks  and  tackle,  ordinary, 

65 
„      combined    with    lever    and 

screw,  1 68 
,,      combined  with  worm-wheel 

and  winch  barrel,  168 
,,      faces  for  belts,  shape  of,  127 
,,      Weston's  differential,  74-77 
Pulleys,  63-65 

,,        arrangement  of  driving  and 
following,     in     different 
planes,  125 
,,        combinations   of  fast   and 

loose,  122-124 
,,        tendency  of  belts  to  run  on 

highest  parts  of,  127 
,,        velocity   ratio   of,   in    belt 

gearing,  119-121 
Pump,     combined     plunger      and 

bucket,  234 

,,       common  suction,  227-229 
,,       double  acting  force,  234 
,,      force,  with  air  vessel,  231 
, ,      plunger  force,  229-23 1 
,,      rods,  tension  in,  229 
Pumps,  centrifugal,  236 

,,       continuous   delivery   force, 

without  air  vessels,  232 
Punching  machine,  portable,  or  hy- 
draulic bear,  251 

Q 

QUADRANT  or  reversing  plate  for 

change  wheels,  183-187 
Quantity  (motion  or  momentum), 

26x3 

Quick  return,  cam,  334 
»»          »      common,  350 


Quick  return  motion,  Whitworth's, 

347     , 

,,          ,,      reversing  motion,  346 
„      elliptic  wheels,  351 


R 

RACK  and  pinion,  132 

Radius  of  gyration,  282 

Railway  carriages,   screw   coupling 

for,  156 
,,        curves,  motion  on  bicycle 

and, 287 
Ratchet,  masked,  336 

,,        wheel,  pawl  and,  334 
Ratio,  velocity,  of  change  wheels  in 

a  lathe,  176-179 
,,  ,,        definition  of,  67 

>t  ,,        of   pulleys   in    belt 

gearing,  119-121 

»  ,,        of  two  friction  cir- 

cular discs,  130 
,,  t»        in  wheel  gearing, 

133 
Reaction,  action  and,  3 

,,        from  fulcrum  of  a  lever, 

25>  30 

Relation  between  twisting  moment, 
diameter,   and  horse   power 
transmitted  by  shafting,  224 
Repose,  angle  of,  106 
Resilience,  definitions,  307,  318 
Resistance,  electrical  unit,  413 
,,          limiting  angle  of,  107 
,,          passive,  or  friction,  101 
,,          work  in  overcoming  a 

uniform,  6-9 
,,          work  in  overcoming  a 

variable,  8-1 1 

Resolution  and  composition  of 
forces,  82-84  ;  of  velocities, 
260 

Resolution    of    a    force    into    two 
components  at  right  angles, 
84 
Resultant  and  components,  4 

,,         of  parallel  forces,  25-28 
,,         two   forces  at  angle  and 
any  number  at  a  point, 
84 

,,         pressure  and  thrust,  5 
Reversible  pawl,  335 

,,        pendulum,  269 


428  INDEX 


Reversing    by   friction    cones    and 

bevel  wheels,  344 
,,  gear,  bevel  wheel  and 

clutch,  345 

„  „     friction  cone,  344 

,,     quick  return,  346 
,,     Whitworth,  345 
motions,  343 
for  belt  gearing,  122-124 
plate  for  change  wheels, 
183-187 
Rigidity,  definition  of,  298 

,,          modulus  of,  319,  321 
Rivets,  pitch,  310 

,,       shearing  stress  of,  310 
Rods,  tension  in  pump,  187 

,,     torsion  of,  318 
Roller  or  ball  bearings,  no,  183 
Roman  balance,  35 
Roof  truss,  stresses  in  a,  88,  89 
Rotating  body,  accumulated  work 

in  a,  281 
Rotor,  50 

Rounded  screw  threads,  155 
Rules  and  syllabus  of  Exams,  -for 
admission  of  Students  to  the 
Institution  of  Civil  Engineers, 
370 


SADDLE  and  slide  rest  of  a  lathe, 

176,  185 

Safe  loads  and  elasticity,  302 
Safety,  factors  of,  for  materials,  302 

,,  *    valve,  38-40 
Sawing  machine,  vertical,  337 
Scalars,  50 
Screw  bench  vice,  166,  167 

,,      combined  with  lever  and  pul- 
ley, 1 60 

,,      -coupling    for    railway    car- 
riages, 156 
,,     -cutting  lathe,  description  of 

a,  180-188 

,,     -cutting  lathe,self-acting,  182 
,,     -cutting     mechanism      in     a 

lathe,  174-188 

,,     endless  and  worm-wheel,  168 
,,     -gauges,  micrometer,  357 
,,     gear,   apparatus    for   demon- 
strating efficiency  of,  160 
,,     gearing,  backlash  in,  158 
,,     -jack,  bottle,  162-164 


Screw-jack,     compound     hydraulic 

and, 249 

,,     -jack,  traversing,  164 
,,     leading,  of  a  lathe,  175,  186; 
split  nut  for  engaging,  185 
,,     -press  for  compressing  bales, 

165 

,,     or  spiral,  for  grain,  148 
,,     pressure  or  thrust,  5 
,,     viewed  as  an  inclined  plane, 

149 

Screws,  right  and  left-hand,  156 
,,       single-,  double-,  and  treble- 
threaded,  157 
,,       strength,     durability,     and 

efficiency  of,  151 
Screw  thread,  148 

,,       buttress,  155 
.,  ,,       rounded,  155 

,,       forming  a,  on  a  cylin- 
der, 149 

,,       square,  154 

Sere  \v  threads,  characteristics  of,  151 
,,        different    forms    of, 

151-156 

,,        Seller's,  154 
,,  ,,        Whitworth,  151-154 

Second  of  Time,  411 
Shafts,  strength  of  hollow,  324 

,,      strength  of  solid  round,  322 
Shaping  machine,  350 
Shearing  strength  of  rivets,  310 
Shears  or  bed  of  a  lathe,  183,  186 
Ship's  capstan,  57-59 
Silent  feed,  336 
Single-riveted  lap  joints,  310 
Siphon, the,  222 
Slide  rest  of  a  lathe,  176,  185 
Sliding  angle,  107 
Slotting  machine,  vertical,  341 

,,  ,,         Whitworth 's,  349 

Sluice  gate,  pressure  on,  217 
Snatch  block,  65 
Solid  shafts,  strength  of,  322 
Solids,  immersion,  217  ;   defn.,  224 
Specific  gravity,  214,  298 
Speed  cones,  124 
Split  nut  for  engaging  leading  screw 

o  a  lathe,  185 
Squared  paper,  14-17 
Standard  measuring  machines,  355- 

366 

Starting  and  stopping  gear,  124 
Starrett  micrometer  gauge,  357 
Steel,  high  speed,  195 


INDEX  429 


Steel,  specific  gravity  test,  203 
Steelyard,  35-38 
Stepped  speed  cones,  124 
Straight  levers  acted  on  by  inclined 

forces,  42 

Strain,  compressive  stress  and,  301 
„      definition  of,  300 
,,      shearing  stress  and,  317 
,,       tensile  stress  and,  301 
Strength  of  materials,  machine  for 

testing,  40-42 
of  materials,  ultimate,  301 
of  solid  and  hollow  round 

shafts,  322-324 

Stress,  centrifugal,  on  fly- wheels, 2 79 
,,       definition  of,  300 
,,      intensity  of,  300 
,,      limiting,or  ultimate  strength, 

301 

,,       shearing  and  strain,  317 
,,      total,  300 
Stresses  in  chains,  316 

,,         in  jib  cranes,  85-89 
„        in  simple  roof  truss,  88,  89 
, ,        tensile  and  compressive,3O  I 
Stubs'  wire  gauge,  358 
Students'    Exams.,    Institution    of 
Civil  Engineers,  389-401, 407 
Suction  pump,  common,  227-229 
Sun  and  planet  wheels,  330 
Surface,  definition  of  pitch,  131 
„       immersed  in  liquid,  211 
Swing  radius,  282 


TABLE  of  melting  points  of  metals, 

300 

, ,      of  moduli  of  elasticity,  305 
.,       of  power  that  steel  shafting 
will  transmit   at   various 
speeds,  323 
,,      of    ultimate     strengths     of 

materials,  302 
Tackle,  block  and,  65-68 
Tangentometer,  364 
Tearing  strength  of  plates,  310 
Teeth,  pitch  of,  in  wheel  gearing, 

132 

Tenacity,  definition  of,  298 
Tensile  strength  of  materials, 

machine  for  testing,  40-42 
,,       stress  and  strain,  501 
Tension  in  driving[belts,  116 


Tension  in  pump  rods,  229 

Test  of   steel,    by    specific    gravity 

method,  203 
,,    specimen,  work  done  per  cubic 

inch  in  fracturing  a,  309 
Testing  machine,  40-42 
Theoretical  advantage,  66-68 
Toggle  joint,  46-49 
Tongs,  Lumberer's,  43 
Torsion  of  rods  and  wires,  318-320 
Torque  or  twisting  moment,  defini- 
tion of,  322 
Transmission  of  power  by  belting, 

118;   by  liquids,  208 
Traversing  screw-jack,  164 
Treadle    lathe,    self-acting    screw- 
cutting,  182 
Triangle  of  forces,  82 
Turning  tools,  forging  and  harden- 

ing,  202 

,,        tool  holders,  194 
,,        with  high  speed  steel,  195 
Turkus,  carpenter's,  44 
Turret-lathe,  188-195 
Twisting  moment,  322 

u 

ULTIMATE    strength    of    materials 

301 

Uniform  velocity,  definition  of,  259 
Unit  of  acceleration,  260 

„       force,  2,  412 

,,       horse-power,  12,  412 

,,       power,  12,  412 

,,       velocity,  259,  411 

,,       work,  6,  412 
Units,  C.G.S.,4ii,  Practical,  413 
Universal  joint,  Hooke's,  329 
Useful  constants — Appendix  D,  414 

,,      data    regarding    fresh     and 
salt  water,  214 

,,      work  in  a  machine,  52 
Uses  of  squared  paper,  14-17 


VACUUM  water  gauges,  221  ^ 

Variable  resistance,  work         done 

against  a,  8 

,,        velocity,  definition  of ,24  59 
Valve,  lever  safety,  38-40 
V-screw-thread,  Seller's,  154 

,,         ,,  "Whitworth's,  151 


430 

Vector's,  $a 

Velocities,  composition  and  resolu- 
tion of,  260 
,,          graphic       representation 

of,  260 

Velocity,  and  motion,  259 
,,         angular,  260 
,,         attained  by  a  body  sliding 
down    any    smooth    in- 
clined plane,  270 
,,         definition  of,  260,411 
,,         linear,  with    uniform    ac- 
celeration, 260 
,,         linear,  259 
,,         uniform  and  variable,  259 
,,         unit  of,  259,  411 
,,         ratio  of  change  wheels  in 

a  lathe,  176-179 
,,     definition  of,  67 
, ,     pulleys  in  belt-gear- 
ing, 119-121 
two  friction  circular 

discs,  130 

,,  ,,     in  wheel-gearing, 

Vessel,  action  of  an  air,  232 

,,  ,,     force  pump  with, 

Viscous  fluid,  224 
Volt,  41 3 

w 

WATER   gauges,  low  pressure   and 

vacuum,  221 
,,       useful  data  regarding  fresh 

and  salt,  214 
Watt,  413 

Watt's  parallel  motion,  338 
Weems'   compound  screw  and  hy- 
draulic jack,  249-251 
Weston's  differential  pulley  block, 

74-77 
Wheel  and  axle,  55-57 

,,         ,,    compound  axle,  72-74 
Wheel  gearing,  backlash  in,  158 
,,  ,,        in  jib  cranes,  1 44- 1 46 

,,  ,,        pitch  of  teeth  in,  132 

,,  ,,        principle     of     work 

applied  to. 


INDEX 


Wheel  gsiring,  velocity  ratio  in,  133 
Wheels,  anti-friction,  109 

,,  change,  in  a  lathe,  176-179, 
183,  184,  187  ;  quadrant 
or  reversing  plate  for, 
183,  187 

,,        fly-, centrifugal  stress  in,  279 
,,        pawl  and  ratchet,  334 
Wheels,  sun  and  planet,  330 
Whitworth's   millionth      measuring 

machine,  358 
quick  return  motion, 

347 

,,  reversing  gear,  345 

,,  slotting    machine, 

349 
,,  V-screw-threads,  151- 

154 
Winch  barrel,  57 

,,       double  purchase,  143-144 

,,       single  purchase,  140-142 

Wincli  drum  combined  with  pulley, 

worm,  and  worm  wheel,  168 
Windlass,  Chinese,  72 
Wire  gauge,  Stubs',  358 
,,     -testing  machine,    Lord    Kel- 
vin's, 210 

Wires,  torsion  of,  319-321 
Work,  accumulated,  280-282 
,,      definition  of,  6,  4  2 
,,       diagrams  of,  9 
,,      done  against  variable  resist- 
ances, 8,  17 

,,      done  in  extending  or  com- 
pressing a  bar  within  the 
elastic  limit,  307 
,,      done  on  inclines,  110-112 
,,      done  per  cubic  inch  in  frac- 
turing a  test-bar,  309 
,,      principle  of,  52 
,,      relation  between,  and  heat, 

102 

,,      transmitted  by  belts,  118 
,,       unit  of,  6,  412 
Workshop  measuring  machine,  363 
Worm-wheel,  screw  and  worm,  168 

,,      lifting  ffear,  170 
Worssam's  silent  feed,  338 


Co.  LIMITED 
TavistoclTSTrEef,  Covent  Garden ,  Loudon 


THIS  BOOK  IS  DUE  ON  THE  LAST  DATE 
STAMPED'BELOW 


AN  INITIAL  PINE  OF  25  CENTS 

WILL  BE  ASSESSED   FOR   FAILURE  TO   RETURN 

rms  BOOK  ON  THE  DATE  DUE.  THE  PENALTY 

WILL  INCREASE  TO  5O  CENTS  ON  THE  FOURTH 
DAY  AND  TO  $1.OO  ON  THE  SEVENTH  DAY 
OVERDUE. 


FED    4    1944 


LD  21-100rn-7,'40 (6936s) 


VB   I  1 077 


204114